Sunteți pe pagina 1din 500

Tanada v. Tuvera, G.R. No.

L-63915, 29 December 1986, 146 SCRA 446


[G.R. No. L-63915. December 29, 1986.]

LORENZO M. TAÑADA, ABRAHAM F. SARMIENTO, and MOVEMENT OF ATTORNEYS FOR BROTHERHOOD,


INTEGRITY AND NATIONALISM, INC. (MABINI), petitioners, vs. HON. JUAN C. TUVERA. in his capacity as
Executive Assistant to the President, HON. JOAQUIN VENUS, in his capacity as Deputy Executive
Assistant to the President, MELQUIADES P. DE LA CRUZ, ETC., ET AL., respondents.

SYLLABUS

FERNAN, J., concurring:

1. CIVIL LAW; EFFECT AND APPLICATION OF LAWS; ARTICLE 2, CIVIL CODE; PUBLICATION OF LAWS
MADE TO ENSURE CONSTITUTIONAL RIGHT TO DUE PROCESS AND TO INFORMATION. — The categorical
statement by this Court on the need for publication before any law be made effective seeks to prevent
abuses on the part if the lawmakers and, at the time, ensure to the people their constitutional right to
due process and to information on matter of public concern. cda

RESOLUTION

CRUZ, J p:

Due process was invoked by the petitioners in demanding the disclosure or a number of presidential
decrees which they claimed had not been published as required by law. The government argued that
while publication was necessary as a rule, it was not so when it was "otherwise provided," as when the
decrees themselves declared that they were to become effective immediately upon their approval. In
the decision of this case on April 24, 1985, the Court affirmed the necessity for the publication of some
of these decrees, declaring in the dispositive portion as follows:

"WHEREFORE, the Court hereby orders respondents to publish to the Official Gazette all unpublished
presidential issuances which are of general application, and unless so published, they shall have no
binding force and effect."

The petitioners are now before us again, this time to move for reconsideration/clarification of that
decision. 1 Specifically, they ask the following questions:

1. What is meant by "law of public nature" or "general applicability"?

2. Must a distinction be made between laws of general applicability and laws which are not?

3. What is meant by "publication"?

4. Where is the publication to be made?

5. When is the publication to be made?

1
Resolving their own doubts, the petitioners suggest that there should be no distinction between laws of
general applicability and those which are not; that publication means complete publication; and that the
publication must be made forthwith in the Official Gazette. 2

In the Comment 3 required of the then Solicitor General, he claimed first that the motion was a request
for an advisory opinion and should therefore be dismissed, and, on the merits, that the clause "unless it
is otherwise provided" in Article 2 of the Civil Code meant that the publication required therein was not
always imperative; that publication, when necessary, did not have to be made in the Official Gazette;
and that in any case the subject decision was concurred in only by three justices and consequently not
binding. This elicited a Reply 4 refuting these arguments. Came next the February Revolution and the
Court required the new Solicitor General to file a Rejoinder in view of the supervening events, under
Rule 3, Section 18, of the Rules of Court. Responding, he submitted that issuances intended only for the
interval administration of a government agency or for particular persons did not have to be published;
that publication when necessary must be in full and in the Official Gazette; and that, however, the
decision under reconsideration was not binding because it was not supported by eight members of this
Court. 5

The subject of contention is Article 2 of the Civil Code providing as follows:

"ART. 2.Laws shall take effect after fifteen days following the completion of their publication in the
Official Gazette, unless it is otherwise provided. This Code shall take effect one year after such
publication."

After a careful study of this provision and of the arguments of the parties, both on the original petition
and on the instant motion, we have come to the conclusion, and so hold, that the clause "unless it is
otherwise provided" refers to the date of effectivity and not to the requirement of publication itself,
which cannot in any event be omitted. This clause does not mean that the legislature may make the law
effective immediately upon approval, or on any other date, without its previous publication. cdphil

Publication is indispensable in every case, but the legislature may in its discretion provide that the usual
fifteen-day period shall be shortened or extended. An example, as pointed out by the present Chief
Justice in his separate concurrence in the original decision, 6 is the Civil Code which did not become
effective after fifteen days from its publication in the Official Gazette but "one year after such
publication." The general rule did not apply because it was "otherwise provided."

It is not correct to say that under the disputed clause publication may be dispensed with altogether. The
reason is that such omission would offend due process insofar as it would deny the public knowledge of
the laws that are supposed to govern it. Surely, if the legislature could validly provide that a law shall
become effective immediately upon its approval notwithstanding the lack of publication (or after an
unreasonably short period after publication), it is not unlikely that persons not aware of it would be
prejudiced as a result; and they would be so not because of a failure to comply with it but simply
because they did not know of its existence. Significantly, this is not true only of penal laws as is
commonly supposed. One can think of many non-penal measures, like a law on prescription, which must
also be communicated to the persons they may affect before they can begin to operate. LexLib

2
We note at this point the conclusive presumption that every person knows the law, which of course
presupposes that the law has been published if the presumption is to have any legal justification at all. It
is no less important to remember that Section 6 of the Bill of Rights recognizes "the right of the people
to information on matters of public concern," and this certainly applies to, among others, and indeed
especially, the legislative enactments of the government.

The term "laws" should refer to all laws and not only to those of general application, for strictly speaking
all laws relate to the people in general albeit there are some that do not apply to them directly. An
example is a law granting citizenship to a particular individual, like a relative of President Marcos who
was decreed instant naturalization. It surely cannot be said that such a law does not affect the public
although it unquestionably does not apply directly to all the people. The subject of such law is a matter
of public interest which any member of the body politic may question in the political forums or, if he is a
proper party, even in the courts of justice. In fact, a law without any bearing on the public would be
invalid as an intrusion of privacy or as class legislation or as an ultra vires act of the legislature. To be
valid, the law must invariably affect the public interest even if it might be directly applicable only to one
individual, or some of the people only, and not to the public as a whole.

We hold therefore that all statutes, including those of local application and private laws, shall be
published as a condition for their effectivity, which shall begin fifteen days after publication unless a
different effectivity date is fixed by the legislature. LibLex

Covered by this rule are presidential decrees and executive orders promulgated by the President in the
exercise of legislative powers whenever the same are validly delegated by the legislature or, at present,
directly conferred by the Constitution. Administrative rules and regulations must also be published if
their purpose is to enforce or implement existing law pursuant also to a valid delegation.

Interpretative regulations and those merely internal in nature, that is, regulating only the personnel of
the administrative agency and not the public, need not be published. Neither is publication required of
the so-called letters of instructions issued by administrative superiors concerning the rules or guidelines
to be followed by their subordinates in the performance of their duties. cdasia

Accordingly, even the charter of a city must be published notwithstanding that it applies to only a
portion of the national territory and directly affects only the inhabitants of that place. All presidential
decrees must be published, including even, say, those naming a public place after a favored individual or
exempting him from certain prohibitions or requirements. The circulars issued by the Monetary Board
must be published if they are meant not merely to interpret but to "fill in the details" of the Central
Bank Act which that body is supposed to enforce.

However, no publication is required of the instructions issued by, say, the Minister of Social Welfare on
the case studies to be made in petitions for adoption or the rules laid down by the head of a
government agency on the assignments or workload of his personnel or the wearing of office uniforms.
Parenthetically, municipal ordinances are not covered by this rule but by the Local Government Code.

3
We agree that the publication must be in full or it is no publication at all since its purpose is to inform
the public of the contents of the laws. As correctly pointed out by the petitioners, the mere mention of
the number of the presidential decree, the title of such decree, its whereabouts (e.g., "with Secretary
Tuvera"), the supposed date of effectivity, and in a mere supplement of the Official Gazette cannot
satisfy the publication requirement. This is not even substantial compliance. This was the manner,
incidentally, in which the General Appropriations Act for FY 1975, a presidential decree undeniably of
general applicability and interest, was "published" by the Marcos administration. 7 The evident purpose
was to withhold rather than disclose information on this vital law.

Coming now to the original decision, it is true that only four justices were categorically for publication in
the Official Gazette 8 and that six others felt that publication could be made elsewhere as long as the
people were sufficiently informed. 9 One reserved his vote 10 and another merely acknowledged the
need for due publication without indicating where it should be made, 11 It is therefore necessary for the
present membership of this Court to arrive at a clear consensus on this matter and to lay down a binding
decision supported by the necessary vote.

There is much to be said of the view that the publication need not be made in the Official Gazette,
considering its erratic releases and limited readership. Undoubtedly, newspapers of general circulation
could better perform the function of communicating the laws to the people as such periodicals are more
easily available, have a wider readership, and come out regularly. The trouble, though, is that this kind
of publication is not the one required or authorized by existing law. As far as we know, no amendment
has been made of Article 2 of the Civil Code. The Solicitor General has not pointed to such a law, and we
have no information that it exists. If it does, it obviously has not yet been published.

At any rate, this Court is not called upon to rule upon the wisdom of a law or to repeal or modify it if we
find it impractical. That is not our function. That function belongs to the legislature. Our task is merely to
interpret and apply the law as conceived and approved by the political departments of the government
in accordance with the prescribed procedure. Consequently, we have no choice but to pronounce that
under Article 2 of the Civil Code, the publication of laws must be made in the Official Gazette, and not
elsewhere, as a requirement for their effectivity after fifteen days from such publication or after a
different period provided by the legislature. LLphil

We also hold that the publication must be made forthwith, or at least as soon as possible, to give effect
to the law pursuant to the said Article 2. There is that possibility, of course, although not suggested by
the parties that a law could be rendered unenforceable by a mere refusal of the executive, for whatever
reason, to cause its publication as required. This is a matter, however, that we do not need to examine
at this time.

Finally, the claim of the former Solicitor General that the instant motion is a request for an advisory
opinion is untenable, to say the least, and deserves no further comment.

The days of the secret laws and the unpublished decrees are over. This is once again an open society,
with all the acts of the government subject to public scrutiny and available always to public cognizance.

4
This has to be so if our country is to remain democratic, with sovereignty residing in the people and all
government authority emanating from them.

Although they have delegated the power of legislation, they retain the authority to review the work of
their delegates and to ratify or reject it according to their lights, through their freedom of expression
and their right of suffrage. This they cannot do if the acts of the legislature are concealed.

Laws must come out in the open in the clear light of the sun instead of skulking in the shadows with
their dark, deep secrets. Mysterious pronouncements and rumored rules cannot be recognized as
binding unless their existence and contents are confirmed by a valid publication intended to make full
disclosure and give proper notice to the people. The furtive law is like a scabbarded saber that cannot
feint, parry or cut unless the naked blade is drawn.

WHEREFORE, it is hereby declared that all laws as above defined shall immediately upon their approval,
or as soon thereafter as possible, be published in full in the Official Gazette, to become effective only
after fifteen days from their publication, or on another date specified by the legislature, in accordance
with Article 2 of the Civil Code. dctai

SO ORDERED.

Teehankee, C .J ., Feria, Yap, Narvasa, Melencio-Herrera, Alampay, Gutierrez, Jr ., and Paras, JJ ., concur.

Separate Opinions

FERNAN, J ., concurring:

While concurring in the Court's opinion penned by my distinguished colleague, Mr. Justice Isagani A.
Cruz, I would like to add a few observations. Even as a Member of the defunct Batasang Pambansa, I
took a strong stand against the insidious manner by which the previous dispensation had promulgated
and made effective thousands of decrees, executive orders, letters of instructions, etc. Never has the
law-making power which traditionally belongs to the legislature been used and abused to satisfy the
whims and caprices of a one-man legislative mill as it happened in the past regime. Thus, in those days,
it was not surprising to witness the sad spectacle of two presidential decrees bearing the same number,
although covering two different subject matters. In point is the case of two presidential decrees bearing
number 1686 issued on March 19, 1980, one granting Philippine citizenship to Michael M. Keon, the
then President's nephew and the other imposing a tax on every motor vehicle equipped with air-
conditioner. This was further exacerbated by the issuance of PD No. 1686-A also on March 19, 1980
granting Philippine citizenship to basketball players Jeffrey Moore and Dennis George Still. cdll

The categorical statement by this Court on the need for publication before any law may be made
effective seeks to prevent abuses on the part of the lawmakers and, at the same time, ensures to the
people their constitutional right to due process and to information on matters of public concern.

FELICIANO, J ., concurring:

5
I agree entirely with the opinion of the court so eloquently written by Mr. Justice Isagani A. Cruz. At the
same time, I wish to add a few statements to reflect my understanding of what the Court is saying.
cdlex

A statute which by its terms provides for its coming into effect immediately upon approval thereof, is
properly interpreted as coming into effect immediately upon publication thereof in the Official Gazette
as provided in Article 2 of the Civil Code. Such statute, in other words, should not be regarded as
purporting literally to come into effect immediately upon its approval or enactment and without need of
publication. For so to interpret such statute would be to collide with the constitutional obstacle posed
by the due process clause. The enforcement of prescriptions which are both unknown to and
unknowable by those subjected to the statute, has been throughout history a common tool of tyrannical
governments. Such application and enforcement constitutes at bottom a negation of the fundamental
principle of legality in the relations between a government and its people.

At the same time, it is clear that the requirement of publication of a statute in the Official Gazette, as
distinguished from any other medium such as a newspaper of general circulation, is embodied in a
statutory norm and is not a constitutional command. The statutory norm is set out in Article 2 of the
Civil Code and is supported and reinforced by Section 1 of Commonwealth Act No. 638 and Section 35 of
the Revised Administrative Code. A specification of the Official Gazette as the prescribed medium of
publication may therefore be changed. Article 2 of the Civil Code could, without creating a constitutional
problem, be amended by a subsequent statute providing, for instance, for publication either in the
Official Gazette or in a newspaper of general circulation in the country. Until such an amendatory
statute is in fact enacted, Article 2 of the Civil Code must be obeyed and publication effected in the
Official Gazette and not in any other medium. LLjur

6
LORENZO M. TAÑADA, ABRAHAM F. SARMIENTO, and MOVEMENT OF ATTORNEYS FOR BROTHERHOOD,
INTEGRITY AND NATIONALISM, INC. [MABINI], petitioners, vs. HON. JUAN C. TUVERA, in his capacity as
Executive Assistant to the President, HON. JOAQUIN VENUS, in his capacity as Deputy Executive
Assistant to the President, MELQUIADES P. DE LA CRUZ, in his capacity as Director, Malacañang Records
Office, and FLORENDO S. PABLO, in his capacity as Director, Bureau of Printing, respondents.

Lorenzo M. Tañada, Abraham F. Sarmiento, Mabini Legal Aid Committee for petitioners

Solicitor General for respondents.

SYLLABUS

1. CONSTITUTIONAL LAW; STATUTES; PUBLICATION IN THE OFFICIAL GAZETTE; LEGAL


PERSONALITY OF PETITIONERS TO FILE MANDAMUS TO COMPEL PUBLICATION, RECOGNIZED. — The
subject of the petition is to compel the performance of a public duty and petitioners maintain they need
not show any specific interest for their petition to be given due course. The right sought to be enforced
by petitioners is a public right recognized by no less than the fundamental law of the land. If petitioners
were not allowed to institute this proceeding, it would indeed be difficult to conceive of any other
person to initiate the same, considering that the Solicitor General, the government officer generally
empowered to represent the people, has entered his appearance for respondents in this case.

2. ID.; ID.; ID.; ARTICLE 2 CIVIL CODE DOES NOT PRECLUDE PUBLICATION IN THE OFFICIAL GAZETTE
EVEN IF THE LAW ITSELF PROVIDES FOR DATE OF ITS EFFECTIVITY. — That publication in the Official
Gazette is not a sine qua non requirement for the effectivity of laws where the laws themselves provide
for their own effectivity dates is correct only insofar as it equates the effectivity of laws with the fact of
publication. Considered in the light of other statutes applicable to the issue at hand, the conclusion is
easily reached that said Article 2 does not preclude the requirement of publication in the Official
Gazette, even if the law itself provides for the date of its effectivity.

3. ID.; ID.; ID.; RATIONALE. — The clear object of Article 2 of the Civil Code is to give the general
public adequate notice of the various laws which are to regulate their actions and conduct as citizens.
Without such notice and publication, there would be no basis for the application of the maxim
"ignorantia legis non excusat." It would be the height of injustice to punish or otherwise burden a citizen
for the transgression of a law of which he had no notice whatsoever, not even a constructive one.

4. ID.; ID.; ID.; PUBLICATION OF PRESIDENTIAL ISSUANCES "OF A PUBLIC NATURE" OR "OF
GENERAL APPLICABILITY," A REQUIREMENT OF DUE PROCESS; UNPUBLISHED PRESIDENTIAL ISSUANCES
WITHOUT FORCE AND EFFECT. — The publication of all presidential issuances "of a public nature" or "of
general applicability" is mandated by law. Obviously, presidential decrees that provide for fines,
forfeitures or penalties for their violation or otherwise impose a burden on the people, such as tax and
revenue measures, fall within this category. Other presidential issuances which apply only to particular
persons such as administrative and executive orders need not be published on the assumption that they
have been circularized to all concerned. (People vs. Que Po Lay, 94 Phil. 640; Balbuena, et al. vs.
Secretary of Education, et al., 110 Phil. 150) It is needless to add that the publication of presidential
issuances "of a public nature" or "of general applicability" is a requirement of due process. It is a rule of
law that before a person may be bound by law, he must first be officially and specifically informed of its
contents. The Court therefore declares that presidential issuances of general application, which have not
been published, shall have no force and effect.

7
5. ID.; ID.; ID.; DECLARATION OF INVALIDITY OF UNPUBLISHED PRESIDENTIAL DECREES DOES NOT
AFFECT THOSE WHICH HAVE BEEN ENFORCED OR IMPLEMENTED PRIOR TO THEIR PUBLICATION. — The
implementation/enforcement of presidential decrees prior to their publication in the Official Gazette is
"an operative fact which may have consequences which cannot be justly ignored. The past cannot
always be erased by a new judicial declaration . . .that an all inclusive statement of a principle of
absolute retroactive invalidity cannot be justified."

FERNANDO, C.J., concurring with qualification:

1. CONSTITUTIONAL LAW; STATUTES; PUBLICATION REQUIREMENT NEED NOT BE CONFINED TO


THE OFFICIAL GAZETTE. — It is of course true that without the requisite publication, a due process
question would arise if made to apply adversely to a party who is not even aware of the existence of any
legislative or executive act having the force and effect of law. But such publication required need not be
confined to the Official Gazette. From the pragmatic standpoint, there is an advantage to be gained. It
conduces to certainty. That is to be admitted. It does not follow, however, that failure to do so would in
all cases and under all circumstances result in a statute, presidential decree, or any other executive act
of the same category being bereft of any binding force and effect. To so hold would raise a
constitutional question. Such a pronouncement would lend itself to the interpretation that such a
legislative or presidential act is bereft of the attribute of effectivity unless published in the Official
Gazette. There is no such requirement in the Constitution.

2. ID.; ID.; ID.; ID.; REQUIREMENT IN ART. 2 CIVIL CODE DOES NOT HAVE THE JUDICIAL FORCE OF A
CONSTITUTIONAL COMMAND. — The Chief Justice's qualified concurrence goes no further than to
affirm that publication is essential to the effectivity of a legislative or executive act of a general
application. He is not in agreement with the view that such publication must be in the Official Gazette.
The Civil Code itself in its Article 2 expressly recognizes that the rule as to laws taking effect after fifteen
days following the completion of their publication in the Official Gazette is subject to this exception,
"unless it is otherwise provided." Moreover, the Civil Code is itself only a legislative enactment, Republic
Act No. 386. It does not and cannot have the juridical force of a constitutional command. A later
legislative or executive act which has the force and effect of law can legally provide for a different rule.

3. ID.; ID.; ID.; TO DECLARE UNPUBLISHED PRESIDENTIAL ISSUANCES WITHOUT LEGAL FORCE AND
EFFECT WOULD RESULT IN UNDESIRABLE CONSEQUENCES. — Nor does the Chief Justice agree with the
rather sweeping conclusion in the opinion of Justice Escolin that presidential decrees and executive acts
not thus previously published in the Official Gazette would be devoid of any legal character. That would
be, in his opinion, to go too far. It may be fraught, as earlier noted, with undesirable consequences. He
finds himself therefore unable to yield assent to such a pronouncement.

TEEHANKEE, J., concurring:

1. CONSTITUTIONAL LAW; STATUTES, PUBLICATION IN THE OFFICIAL GAZETTE; NECESSARY


PURSUANT TO THE BASIC CONSTITUTIONAL REQUIREMENTS OF DUE PROCESS. — The Rule of Law
connotes a body of norms and laws published and ascertainable and of equal application to all similarly
circumstanced and not subject to arbitrary change but only under certain set procedure. The Court had
consistently stressed that "it is an elementary rule of fair play and justice that a reasonable opportunity
to be informed must be afforded to the people who are commanded to obey before they can be
punished for its violation," (People vs. de Dios, G.R. No. L-11003, August 31, 1959, per the late Chief
Justice Paras) citing the settled principle based on due process enunciated in earlier cases that "before

8
the public is bound by its contents. especially its penal provisions, a law, regulation or circular must first
be published and the people officially and specially informed of said contents and its penalties." Without
official publication in the Official Gazette as required by Article 2 of the Civil Code and Revised
Administrative Code, there would be no basis nor justification for the corollary rule of Article 3 of the
Civil Code (based on constructive notice that the provisions of the law are ascertainable from the public
and official repository where they are duly published) that "Ignorance of the law excuses no one from
compliance therewith."

2. ID.; ID.; ID.; RESPONDENTS' CONTENTION THAT "ONLY LAWS WHICH ARE SILENT AS TO THEIR
EFFECTIVITY DATE NEED TO BE PUBLISHED IN THE OFFICIAL GAZETTE FOR THEIR EFFECTIVITY,"
UNTENABLE. — The plain text and meaning of the Civil Code is that "laws shall take effect after fifteen
days following the completion of their publication in the Official Gazette, unless it is otherwise
provided," i.e. a different effectivity date is provided by the law itself. This proviso perforce refers to a
law that had been duly published pursuant to the basic constitutional requirements of due process. The
best example of this is the Civil Code itself: the same Article 2 provides otherwise that it "shall take
effect (only) one year (not 15 days) after such publication." To sustain respondents misreading that
"most laws or decrees specify the date of their effectivity and for this reason, publication in the Official
Gazette is not necessary for their effectivity" would be to nullify and render nugatory the Civil Code's
indispensable and essential requirement of prior publication in the Official Gazette by the simple
expedient of providing for immediate effectivity or an earlier effectivity date in the law itself before the
completion of 15 days following its publication which is the period generally fixed by the Civil Code for
its proper dissemination.

MELENCIO-HERRERA, J., concurring:

CONSTITUTIONAL LAW; STATUTES; PUBLICATION IN THE OFFICIAL GAZETTE; RETROACTIVITY IN


EFFECTIVITY DATE NOT ALLOWED WHERE IT WILL RUN COUNTER TO CONSTITUTIONAL RIGHTS OR
DESTROY VESTED RIGHTS. — There cannot be any question but that even if a decree provides for a date
of effectivity, it has to be published. When a date effectivity is mentioned in the decree but the decree
becomes effective only fifteen (15) days after its publication in the Official Gazette, it will not mean that
the decree can have retroactive effect to the date of effectivity mentioned in the decree itself. There
should be no retroactivity if the retroactivity will run counter to constitutional rights or shall destroy
vested rights.

PLANA, J., separate opinion:

1. CONSTITUTIONAL LAW; STATUTES; PUBLICATION IN THE OFFICIAL GAZETTE NOT ESSENTIAL FOR
EFFECTIVITY FOR EFFECTIVITY OF LAWS. — The Philippine Constitution does not require the publication
of laws as a prerequisite for their effectivity, unlike some Constitutions elsewhere. It may be said though
that the guarantee of due process requires notice of laws to affected parties before they can be bound
thereby; but such notice is not necessarily by publication in the Official Gazette. The due process clause
is not that precise. Neither is the publication in the Official Gazette required by any statute as a
prerequisite for their effectivity, if said laws already provide for their effectivity date.

2. ID.; ID.; PUBLICATION MAY BE MADE ELSEWHERE THAN IN THE OFFICIAL GAZETTE. — Article 2
of the Civil Code provides that "laws shall take effect after fifteen days following the completion of their
publication in the Official Gazette, unless it is otherwise provided." Two things may be said of this
provision: Firstly, it obviously does not apply to a law with a built-in provision as to when it will take

9
effect. Secondly, it clearly recognizes that each law may provide not only a different period for reckoning
its effectivity date but also a different mode of notice. Thus, a law may prescribe that it shall be
published elsewhere than in the Official Gazette.

3. ID.; ID.; COMMONWEALTH ACT 638 CANNOT NULLIFY OR RESTRICT OPERATION OF A STATUTE
WITH A PROVISION AS TO ITS EFFECTIVITY. — Not all legislative acts are required to be published in the
Official Gazette but only "important" ones "of a public nature." Moreover, Commonwealth Act No. 638
does not provide that publication in the Official Gazette is essential for the effectivity of laws. This is as it
should be, for all statutes are equal and stand on the same footing. A law, especially an earlier one of
general application such as Commonwealth Act No. 638, cannot nullify or restrict the operation of a
subsequent statute that has a provision of its own as to when and how it will take effect. Only a higher
law, which is the Constitution, can assume the role.

DECISION

ESCOLIN, J p:

Invoking the people's right to be informed on matters of public concern, a right recognized in Section 6,
Article IV of the 1973 Philippine Constitution, 1 as well as the principle that laws to be valid and
enforceable must be published in the Official Gazette or otherwise effectively promulgated, petitioners
seek a writ of mandamus to compel respondent public officials to publish, and or cause the publication
in the Official Gazette of various presidential decrees, letters of instructions, general orders,
proclamations, executive orders, letter of implementation and administrative orders.

Specifically, the publication of the following presidential issuances is sought:

a] Presidential Decrees Nos. 12, 22, 37, 38, 59, 64, 103, 171, 179, 184, 197, 200, 234, 265, 286, 298,
303, 312, 324, 325, 326, 337, 355, 358, 359, 360, 361, 368, 404, 406, 415, 427, 429, 445, 447, 473, 486,
491, 503, 504, 521, 528, 551, 566, 573, 574, 594, 599, 644, 658, 661, 718, 731, 733, 793, 800, 802, 835,
836, 923, 935, 961, 1017-1030, 1050, 1060-1061, 1085, 1143, 1165, 1166, 1242, 1246, 1250, 1278, 1279,
1300, 1644, 1772, 1808, 1810, 1813-1817, 1819-1826, 1829-1840, 1842-1847.

b] Letter of Instructions Nos.: 10, 39, 49, 72, 107, 108, 116, 130, 136, 141, 150, 153, 155, 161, 173,
180, 187, 188, 192, 193, 199, 202, 204, 205, 209, 211-213, 215-224, 226-228, 231-239, 241-245, 248-
251, 253-261, 263-269, 271-273, 275-283, 285-289, 291, 293, 297-299, 301-303, 309, 312-315, 325, 327,
343, 346, 349, 357, 358, 362, 367, 370, 382, 385, 386, 396-397, 405, 438-440, 444-445, 473, 486, 488,
498, 501, 399, 527, 561, 576, 587, 594, 599, 600, 602, 609, 610, 611, 612, 615, 641, 642, 665, 702, 712-
713, 726, 837-839, 878-879, 881, 882, 939-940, 964, 997, 1149-1178, 1180-1278.

c] General Orders Nos.: 14, 52, 58, 59, 60, 62, 63, 64 & 65.

d] Proclamation Nos.: 1126, 1144, 1147, 1151, 1196, 1270, 1281, 1319-1526, 1529, 1532, 1535,
1538, 1540-1547, 1550-1558, 1561-1588, 1590-1595, 1594-1600, 1606-1609, 1612-1628, 1630-1649,
1694-1695, 1697-1701, 1705-1723, 1731-1734, 1737-1742, 1744, 1746-1751, 1752, 1754, 1762, 1764-
1787, 1789-1795, 1797, 1800, 1802-1804, 1806-1807, 1812-1814, 1816, 1825-1826, 1829, 1831-1832,
1835-1836, 1839-1840, 1843-1844, 1846-1847, 1849, 1853-1858, 1860, 1866, 1868, 1870, 1876-1889,
1892, 1900, 1918, 1923, 1933, 1952, 1963, 1965-1966, 1968-1984, 1986-2028, 2030-2044, 2046-2145,
2147-2161, 2163-2244.

10
e] Executive Orders Nos.: 411, 413, 414, 427, 429-454, 457-471, 474-492, 494-507, 509-510, 522,
524-528, 531-532, 536, 538, 543-544, 549, 551-553, 560, 563, 567-568, 570, 574, 593, 594, 598-604,
609, 611-647, 649-677, 679-703, 705-707, 712-786, 788-852, 854-857.

f] Letters of Implementation Nos.: 7, 8, 9, 10, 11-22, 25-27, 39, 50, 51, 59, 76, 80-81, 92, 94, 95,
107, 120, 122, 123.

g] Administrative Orders Nos.: 347, 348, 352-354, 360-378, 380-433, 436-439.

The respondents, through the Solicitor General, would have this case dismissed outright on the ground
that petitioners have no legal personality or standing to bring the instant petition. The view is submitted
that in the absence of any showing that petitioners are personally and directly affected or prejudiced by
the alleged non-publication of the presidential issuances in question 2 said petitioners are without the
requisite legal personality to institute this mandamus proceeding, they are not being "aggrieved parties"
within the meaning of Section 3, Rule 65 of the Rules of Court, which we quote: Cdpr

"SEC. 3. Petition for Mandamus. — When any tribunal, corporation, board or person unlawfully neglects
the performance of an act which the law specifically enjoins as a duty resulting from an office, trust, or
station, or unlawfully excludes another from the use and enjoyment of a right or office to which such
other is entitled, and there is no other plain, speedy and adequate remedy in the ordinary course of law,
the person aggrieved thereby may file a verified petition in the proper court alleging the facts with
certainty and praying that judgment be rendered commanding the defendant, immediately or at some
other specified time, to do the act required to be done to protect the rights of the petitioner, and to pay
the damages sustained by the petitioner by reason of the wrongful acts of the defendant."

Upon the other hand, petitioners maintain that since the subject of the petition concerns a public right
and its object is to compel the performance of a public duty, they need not show any specific interest for
their petition to be given due course.

The issue posed is not one of first impression. As early as the 1910 case of Severino vs. Governor
General, 3 this Court held that while the general rule is that "a writ of mandamus would be granted to a
private individual only in those cases where he has some private or particular interest to be subserved,
or some particular right to be protected, independent of that which he holds with the public at large,"
and "it is for the public officers exclusively to apply for the writ when public rights are to be subserved
[Mitchell vs. Boardmen, 79 M.e., 469]," nevertheless, "when the question is one of public right and the
object of the mandamus is to procure the enforcement of a public duty, the people are regarded as the
real party in interest and the relator at whose instigation the proceedings are instituted need not show
that he has any legal or special interest in the result, it being sufficient to show that he is a citizen and as
such interested in the execution of the laws [High, Extraordinary Legal Remedies, 3rd ed., sec. 431]."

Thus, in said case, this Court recognized the relator Lope Severino, a private individual, as a proper party
to the mandamus proceedings brought to compel the Governor General to call a special election for the
position of municipal president in the town of Silay, Negros Occidental. Speaking for this Court, Mr.
Justice Grant T. Trent said: Cdpr

"We are therefore of the opinion that the weight of authority supports the proposition that the relator
is a proper party to proceedings of this character when a public right is sought to be enforced. If the
general rule in America were otherwise, we think that it would not be applicable to the case at bar for
the reason 'that it is always dangerous to apply a general rule to a particular case without keeping in

11
mind the reason for the rule, because, if under the particular circumstances the reason for the rule does
not exist, the rule itself is not applicable and reliance upon the rule may well lead to error.'

"No reason exists in the case at bar for applying the general rule insisted upon by counsel for the
respondent. The circumstances which surround this case are different from those in the United States,
inasmuch as if the relator is not a proper party to these proceedings no other person could be, as we
have seen that it is not the duty of the law officer of the Government to appear and represent the
people in cases of this character."

The reasons given by the Court in recognizing a private citizen's legal personality in the aforementioned
case apply squarely to the present petition. Clearly, the right sought to be enforced by petitioners herein
is a public right recognized by no less than the fundamental law of the land. If petitioners were not
allowed to institute this proceeding, it would indeed be difficult to conceive of any other person to
initiate the same, considering that the Solicitor General, the government officer generally empowered
to represent the people, has entered his appearance for respondents in this case.

Respondents further contend that publication in the Official Gazette is not a sine qua non requirement
for the effectivity of laws where the laws themselves provide for their own effectivity dates. It is thus
submitted that since the presidential issuances in question contain special provisions as to the date they
are to take effect, publication in the Official Gazette is not indispensable for their effectivity. The point
stressed is anchored on Article 2 of the Civil Code:

"Art. 2. Laws shall take effect after fifteen days following the completion of their publication in the
Official Gazette, unless it is otherwise provided, . . ."

The interpretation given by respondent is in accord with this Court's construction of said article. In a
long line of decisions, 4 this Court has ruled that publication in the Official Gazette is necessary in those
cases where the legislation itself does not provide for its effectivity date — for then the date of
publication is material for determining its date of effectivity, which is the fifteenth day following its
publication — but not when the law itself provides for the date when it goes into effect.

Respondents' argument, however, is logically correct only insofar as it equates the effectivity of laws
with the fact of publication. Considered in the light of other statutes applicable to the issue at hand, the
conclusion is easily reached that said Article 2 does not preclude the requirement of publication in the
Official Gazette, even if the law itself provides for the date of its effectivity. Thus, Section 1 of
Commonwealth Act 638 provides as follows:

"Section 1. There shall be published in the Official Gazette [1] all important legislative acts and
resolutions of a public nature of the Congress of the Philippines; [2] all executive and administrative
orders and proclamations, except such as have no general applicability; [3] decisions or abstracts of
decisions of the Supreme Court and the Court of Appeals as may be deemed by said courts of sufficient
importance to be so published; [4] such documents or classes of documents as may be required so to be
published by law; and [5] such documents or classes of documents as the President of the Philippines
shall determine from time to time to have general applicability and legal effect, or which he may
authorize so to be published. . . ."

The clear object of the above quoted provision is to give the general public adequate notice of the
various laws which are to regulate their actions and conduct as citizens. Without such notice and

12
publication, there would be no basis for the application of the maxim "ignorantia legis non excusat." It
would be the height of injustice to punish or otherwise burden a citizen for the transgression of a law of
which he had no notice whatsoever, not even a constructive one. cdphil

Perhaps at no time since the establishment of the Philippine Republic has the publication of laws taken
so vital significance that at this time when the people have bestowed upon the President a power
heretofore enjoyed solely by the legislature. While the people are kept abreast by the mass media of the
debates and deliberations in the Batasan Pambansa — and for the diligent ones, ready access to the
legislative records — no such publicity accompanies the law-making process of the President. Thus,
without publication, the people have no means of knowing what presidential decrees have actually been
promulgated, much less a definite way of informing themselves of the specific contents and texts of
such decrees. As the Supreme Court of Spain ruled: "Bajo la denominacion genrica de leyes, se
comprenden tambin los reglamentos, Reales decretos, Instrucciones, Circulares y Reales ordines
dictadas de conformidad con las mismas por el Gobierno en uso de su potestad." 5

The very first clause of Section 1 of Commonwealth Act 638 reads: "There shall be published in the
Official Gazette . . ." The word "shall" used therein imposes upon respondent officials an imperative
duty. That duty must be enforced if the Constitutional right of the people to be informed on matters of
public concern is to be given substance and reality. The law itself makes a list of what should be
published in the official Gazette. Such listing, to our mind, leaves respondents with no discretion
whatsoever as to what must be included or excluded from such publication.

The publication of all presidential issuances "of a public nature" or "of general applicability" is mandated
by law. Obviously, presidential decrees that provide for fines, forfeitures or penalties for their violation
or otherwise impose a burden on the people, such as tax and revenue measures, fall within this
category. Other presidential issuances which apply only to particular persons or class of persons such as
administrative and executive orders need not be published on the assumption that they have been
circularized to all concerned. 6

It is needless to add that the publication of presidential issuances "of a public nature" or "of general
applicability" is a requirement of due process. It is a rule of law that before a person may be bound by
law, he must first be officially and specifically informed of its contents. As Justice Claudio Teehankee said
in Peralta vs. COMELEC: 7

"In a time of proliferating decrees, orders and letters of instructions which all form part of the law of the
land, the requirement of due process and the Rule of Law demand that the Official Gazette as the
official government repository promulgate and publish the texts of all such decrees, orders and
instructions so that the people may know where to obtain their official and specific contents."

The Court therefore declares that presidential issuances of general application, which have not been
published, shall have no force and effect. Some members of the Court, quite apprehensive about the
possible unsettling effect this decision might have on acts done in reliance of the validity of those
presidential decrees which were published only during the pendency of this petition, have put the
question as to whether the Court's declaration of invalidity apply to P.D.s which had been enforced or
implemented prior to their publication. The answer is all too familiar. In similar situations in the past this
Court had taken the pragmatic and realistic course set forth in Chicot County Drainage District vs. Baxter
Bank 8 to wit: LLjur

13
"The courts below have proceeded on the theory that the Act of Congress, having been found to be
unconstitutional, was not a law; that it was inoperative, conferring no rights and imposing no duties, and
hence affording no basis for the challenged decree. Norton v. Shelby County, 118 U.S. 425, 442; Chicago,
I. & L. Ry. Co. v. Hackett, 228 U.S. 559, 566. It is quite clear, however, that such broad statements as to
the effect of a determination of unconstitutionality must be taken with qualifications. The actual
existence of a statute, prior to such a determination, is an operative fact and may have consequences
which cannot justly be ignored. The past cannot always be erased by a new judicial declaration. The
effect of the subsequent ruling as to invalidity may have to be considered in various aspects — with
respect to particular conduct, private and official. Questions of rights claimed to have become vested, of
status, of prior determinations deemed to have finality and acted upon accordingly, of public policy in
the light of the nature both of the statute and of its previous application, demand examination. These
questions are among the most difficult of those which have engaged the attention of courts, state and
federal, and it is manifest from numerous decisions that an all-inclusive statement of a principle of
absolute retroactive invalidity cannot be justified."

Consistently with the above principle, this Court in Rutter vs. Esteban 9 sustained the right of a party
under the Moratorium Law, albeit said right had accrued in his favor before said law was declared
unconstitutional by this Court.

Similarly, the implementation/enforcement of presidential decrees prior to their publication in the


Official Gazette is "an operative fact which may have consequences which cannot be justly ignored. The
past cannot always be erased by a new judicial declaration . . . that an all-inclusive statement of a
principle of absolute retroactive invalidity cannot be justified."

From the report submitted to the Court by the Clerk of Court, it appears that of the presidential decrees
sought by petitioners to be published in the Official Gazette, only Presidential Decrees Nos. 1019 to
1030, inclusive, 1278, and 1937 to 1939, inclusive, have not been so published. 10 Neither the subject
matters nor the texts of these PDs can be ascertained since no copies thereof are available. But
whatever their subject matter may be, it is undisputed that none of these unpublished PDs has ever
been implemented or enforced by the government. In Pesigan vs. Angeles, 11 the Court, through
Justice Ramon Aquino, ruled that "publication is necessary to apprise the public of the contents of
[penal] regulations and make the said penalties binding on the persons affected thereby." The cogency
of this holding is apparently recognized by respondent officials considering the manifestation in their
comment that "the government, as a matter of policy, refrains from prosecuting violations of criminal
laws until the same shall have been published in the Official Gazette or in some other publication, even
though some criminal laws provide that they shall take effect immediately."

WHEREFORE, the Court hereby orders respondents to publish in the Official Gazette all unpublished
presidential issuances which are of general application, and unless so published, they shall have no
binding force and effect.

SO ORDERED.

14
De Roy v. Court of Appeals, G.R. No. 80718, 29 January 1988

[G.R. No. 80718. January 29, 1988.]

FELISA P. DE ROY and VIRGILIO RAMOS, petitioners, vs. COURT OF APPEALS and LUIS BERNAL, SR., GLENIA
BERNAL, LUIS BERNAL, JR., HEIRS OF MARISSA BERNAL, namely, GLICERIA DELA CRUZ BERNAL and LUIS
BERNAL, SR., respondents.

SYLLABUS

1. REMEDIAL LAW; CIVIL PROCEDURE; APPEAL; PERIOD FOR APPEALING OR FOR FILING A MOTION FOR
RECONSIDERATION, NON-EXTENDIBLE. — The rule laid down in Habaluyas Enterprises, Inc. v. Japzon, [G.R.
No. 70895, August 5, 1985, 138 SCRA 46], that the fifteen-day period for appealing or for filing a motion for
reconsideration cannot be extended.

2. ID.; ID.; ID.; GRACE PERIOD IN BOCAVA CASE, INAPPLICABLE TO THE CASE AT BAR. — The one-month
grace period from the promulgation on May 30, 1986 of the Court's Resolution in the clarificatory Habaluyas
case, or up to June 30, 1986, within which the rule barring extensions of time to file motions for new trial or
reconsideration may still be allowed cannot be invoked by the petitioners as their motion for extension of
time was filed on September 9, 1987, more than a year after the grace period on June 30, 1986.

RESOLUTION

CORTES, J p:

This special civil action for certiorari seeks to declare null and void two (2) resolutions of the Special Division
of the Court of Appeals in the Luis Bernal, Sr., et al. v. Felisa Perdosa De Roy, et al., CA-G.R. CV No. 07286. The
first resolution promulgated on 30 September 1987 denied petitioner's motion for extension of time to file a
motion for reconsideration and directed entry of judgment since the decision in said case had become final;
and the second Resolution dated 27 October 1987 denied petitioners' motion for reconsideration for having
been filed out of time.

At the outset, this Court could have denied the petition outright for not being verified as required by Rule 65
section 1 of the Rules of Court. However, even if the instant petition did not suffer from this defect, this
Court, on procedural and substantive grounds, would still resolve to deny it.

The facts of the case are undisputed. The firewall of a burnedout building owned by petitioners collapsed and
destroyed the tailoring shop occupied by the family of private respondents, resulting in injuries to private
respondents and the death of Marissa Bernal, a daughter. Private respondents had been warned by
petitioners to vacate their shop in view of its proximity to the weakened wall but the former failed to do so.
On the basis of the foregoing facts, the Regional Trial Court. First Judicial Region, Branch XXXVIII, presided by
the Hon. Antonio M. Belen, rendered judgment finding petitioners guilty of gross negligence and awarding
damages to private respondents. On appeal, the decision of the trial court was affirmed in toto by the Court
of Appeals in a decision promulgated on August 17, 1987, a copy of which was received by petitioners on
August 25, 1987. On September 9, 1987, the last day of the fifteen-day period to file an appeal, petitioners
filed a motion for extension of time to file a motion for reconsideration, which was eventually denied by the

15
appellate court in the Resolution of September 30, 1987. Petitioners filed their motion for reconsideration on
September 24, 1987 but this was denied in the Resolution of October 27, 1987.

This Court finds that the Court of Appeals did not commit a grave abuse of discretion when it denied
petitioners' motion for extension of time to file a motion for reconsideration, directed entry of judgment and
denied their motion for reconsideration. It correctly applied the rule laid down in Habaluyas Enterprises, Inc.
v. Japzon, [G.R. No. 70895, August 5, 1985, 138 SCRA 46], that the fifteen-day period for appealing or for
filing a motion for reconsideration cannot be extended. In its Resolution denying the motion for
reconsideration, promulgated on May 30, 1986 (142 SCRA 208), this Court en banc restated and clarified the
rule, to wit:

Beginning one month after the promulgation of this Resolution, the rule shall be strictly enforced that no
motion for extension of time to file a motion for reconsideration may be filed with the Metropolitan or
Municipal Trial Courts, the Regional Trial Courts, and the Intermediate Appellate Court. Such a motion may
be filed only in cases pending with the Supreme Court as the court of last resort, which may in its sound
discretion either grant or deny the extension requested. (at p. 212)

Lacsamana v. Second Special Cases Division of the Intermediate Appellate Court, [G.R. No. 73146-53, August
26, 1986, 143 SCRA 643], reiterated the rule and went further to restate and clarify the modes and periods of
appeal.

Bacaya v. Intermediate Appellate Court, [G.R. No. 74824, Sept. 16, 1985, 144 SCRA 161], stressed the
prospective application of said rule, and explained the operation of the grace period, to wit: LibLex

In other words, there is one-month grace period from the promulgation on May 30, 1986 of the Court's
Resolution in the clarificatory Habaluyas case, or up to June 30, 1986, within which the rule barring
extensions of time to file motions for new trial or reconsideration is, as yet, not strictly enforceable.

Since petitioners herein filed their motion for extension on February 27, 1986, it is still within the grace
period, which expired on June 30, 1986, and may still be allowed.

This grace period was also applied in Mission v. Intermediate Appellate Court [G.R. No. 73669, October 28,
1986, 145 SCRA 306].

In the instant case, however, petitioners' motion for extension of time was filed on September 9, 1987, more
than a year after the expiration of the grace period on June 30, 1986. Hence, it is no longer within the
coverage of the grace period. Considering the length of time from the expiration of the grace period to the
promulgation of the decision of the Court of Appeals on August 25, 1987, petitioners cannot seek refuge in
the ignorance of their counsel regarding said rule for their failure to file a motion for reconsideration within
the reglementary period. prLL

Petitioners contend that the rule enunciated in the Habaluyas case should not be made to apply to the case
at bar owing to the non-publication of the Habaluyas decision in the Official Gazette as of the time the
subject decision of the Court of Appeals was promulgated. Contrary to petitioners' view, there is no law
requiring the publication of Supreme Court decisions in the Official Gazette before they can be binding and as
a condition to their becoming effective. It is the bounden duty of counsel as lawyer in active law practice to

16
keep abreast of decisions of the Supreme Court particularly where issues have been clarified, consistently
reiterated, and published in the advance reports of Supreme Court decisions (G.R.s) and in such publications
as the Supreme Court Reports Annotated (SCRA) and law journals.

This Court likewise finds that the Court of Appeals committed no grave abuse of discretion in affirming the
trial court's decision holding petitioner liable under Article 2190 of the Civil Code, which provides that "the
proprietor of a building or structure is responsible for the damage resulting from its total or partial collapse, if
it should be due to the lack of necessary repairs."

Nor was there error in rejecting petitioners argument that private respondents had the "last clear chance" to
avoid the accident if only they heeded the warning to vacate the tailoring shop and, therefore, petitioners
prior negligence should be disregarded, since the doctrine of "last clear chance," which has been applied to
vehicular accidents, is inapplicable to this case.

WHEREFORE, in view of the foregoing, the Court Resolved to DENY the instant petition for lack of merit.

Fernan (Chairman), Gutierrez, Jr., Feliciano and Bidin, JJ., concur.

17
G.R. No. L-6791 March 29, 1954

THE PEOPLE OF THE PHILIPPINES, plaintiff-appellee, vs. QUE PO LAY, defendant-appellant.

Prudencio de Guzman for appellant.

First Assistant Solicitor General Ruperto Kapunan, Jr., and Solicitor Lauro G. Marquez for appellee.

MONTEMAYOR, J.:

Que Po Lay is appealing from the decision of the Court of First Instance of Manila, finding him guilty of
violating Central Bank Circular No. 20 in connection with section 34 of Republic Act No. 265, and
sentencing him to suffer six months imprisonment, to pay a fine of P1,000 with subsidiary imprisonment
in case of insolvency, and to pay the costs.

The charge was that the appellant who was in possession of foreign exchange consisting of U.S. dollars,
U.S. checks and U.S. money orders amounting to about $7,000 failed to sell the same to the Central
Bank through its agents within one day following the receipt of such foreign exchange as required by
Circular No. 20. the appeal is based on the claim that said circular No. 20 was not published in the
Official Gazette prior to the act or omission imputed to the appellant, and that consequently, said
circular had no force and effect. It is contended that Commonwealth Act. No., 638 and Act 2930 both
require said circular to be published in the Official Gazette, it being an order or notice of general
applicability. The Solicitor General answering this contention says that Commonwealth Act. No. 638 and
2930 do not require the publication in the Official Gazette of said circular issued for the implementation
of a law in order to have force and effect.

We agree with the Solicitor General that the laws in question do not require the publication of the
circulars, regulations and notices therein mentioned in order to become binding and effective. All that
said two laws provide is that laws, resolutions, decisions of the Supreme Court and Court of Appeals,
notices and documents required by law to be of no force and effect. In other words, said two Acts
merely enumerate and make a list of what should be published in the Official Gazette, presumably, for
the guidance of the different branches of the Government issuing same, and of the Bureau of Printing.

However, section 11 of the Revised Administrative Code provides that statutes passed by Congress shall,
in the absence of special provision, take effect at the beginning of the fifteenth day after the completion
of the publication of the statute in the Official Gazette. Article 2 of the new Civil Code (Republic Act No.
386) equally provides that laws shall take effect after fifteen days following the completion of their
publication in the Official Gazette, unless it is otherwise provided. It is true that Circular No. 20 of the
Central Bank is not a statute or law but being issued for the implementation of the law authorizing its
issuance, it has the force and effect of law according to settled jurisprudence. (See U.S. vs. Tupasi
Molina, 29 Phil., 119 and authorities cited therein.) Moreover, as a rule, circulars and regulations

18
especially like the Circular No. 20 of the Central Bank in question which prescribes a penalty for its
violation should be published before becoming effective, this, on the general principle and theory that
before the public is bound by its contents, especially its penal provisions, a law, regulation or circular
must first be published and the people officially and specifically informed of said contents and its
penalties.

Our Old Civil code, (Spanish Civil Code of 1889) has a similar provision about the effectivity of laws,
(Article 1 thereof), namely, that laws shall be binding twenty days after their promulgation, and that
their promulgation shall be understood as made on the day of the termination of the publication of the
laws in the Gazette. Manresa, commenting on this article is of the opinion that the word "laws" include
regulations and circulars issued in accordance with the same. He says:

El Tribunal Supremo, ha interpretado el articulo 1. del codigo Civil en Sentencia de 22 de Junio de 1910,
en el sentido de que bajo la denominacion generica de leyes, se comprenden tambien los Reglamentos,
Reales decretos, Instrucciones, Circulares y Reales ordenes dictadas de conformidad con las mismas por
el Gobierno en uso de su potestad. Tambien el poder ejecutivo lo ha venido entendiendo asi, como lo
prueba el hecho de que muchas de sus disposiciones contienen la advertencia de que empiezan a regir
el mismo dia de su publicacion en la Gaceta, advertencia que seria perfectamente inutil si no fuera de
aplicacion al caso el articulo 1.o del Codigo Civil. (Manresa, Codigo Civil Español, Vol. I. p. 52).

In the present case, although circular No. 20 of the Central Bank was issued in the year 1949, it was not
published until November 1951, that is, about 3 months after appellant's conviction of its violation. It is
clear that said circular, particularly its penal provision, did not have any legal effect and bound no one
until its publication in the Official Gazzette or after November 1951. In other words, appellant could not
be held liable for its violation, for it was not binding at the time he was found to have failed to sell the
foreign exchange in his possession thereof.

But the Solicitor General also contends that this question of non-publication of the Circular is being
raised for the first time on appeal in this Court, which cannot be done by appellant. Ordinarily, one may
raise on appeal any question of law or fact that has been raised in the court below and which is within
the issues made by the parties in their pleadings. (Section 19, Rule 48 of the Rules of Court). But the
question of non-publication is fundamental and decisive. If as a matter of fact Circular No. 20 had not
been published as required by law before its violation, then in the eyes of the law there was no such
circular to be violated and consequently appellant committed no violation of the circular or committed
any offense, and the trial court may be said to have had no jurisdiction. This question may be raised at
any stage of the proceeding whether or not raised in the court below.

In view of the foregoing, we reverse the decision appealed from and acquit the appellant, with costs de
oficio.

Paras, C.J., Bengzon, Padilla, Reyes, Bautista Angelo, Labrador, Concepcion and Diokno, JJ., concur.

19
G.R. No. 176006 March 26, 2010

NATIONAL POWER CORPORATION, Petitioner,

vs.

PINATUBO COMMERCIAL, represented by ALFREDO A. DY, Respondent.

DECISION

CORONA, J.:

The National Power Corporation (NPC)1 questions the decision dated June 30, 2006 rendered by the
Regional Trial Court (RTC) of Mandaluyong City, Branch 213 declaring items 3 and 3.1 of NPC Circular No.
99-75 unconstitutional. The dispositive portion of the decision provides:

WHEREFORE then, in view of the foregoing, judgment is hereby rendered declaring item[s] 3 and 3.1 of
NAPOCOR Circular No. 99-75, which [allow] only partnerships or corporations that

directly use aluminum as the raw material in producing finished products either purely or partly out of
aluminum, to participate in the bidding for the disposal of ACSR wires as unconstitutional for being
violative of substantial due process and the equal protection clause of the Constitution as well as for
restraining competitive free trade and commerce.

The claim for attorney’s fees is denied for lack of merit.

No costs.

SO ORDERED.2

20
NPC also assails the RTC resolution dated November 20, 2006 denying its motion for reconsideration for
lack of merit.3

In this petition, NPC poses the sole issue for our review:

WHETHER OR NOT THE RTC GRAVELY ERRED WHEN IT DECLARED ITEMS 3 AND 3.1 OF NAPOCOR
CIRCULAR NO. 99-75 AS UNCONSTITUTIONAL FOR BEING VIOLATIVE OF SUBSTANTIAL DUE PROCESS
AND THE EQUAL PROTECTION CLAUSE OF THE CONSTITUTION AS WELL AS FOR RESTRAINING
COMPETITIVE FREE TRADE AND COMMERCE.4

NPC Circular No. 99-755 dated October 8, 1999 set the guidelines in the "disposal of scrap aluminum
conductor steel-reinforced or ACSRs in order to decongest and maintain good housekeeping in NPC
installations and to generate additional income for NPC." Items 3 and 3.1 of the circular provide:

3. QUALIFIED BIDDERS

3.1 Qualified bidders envisioned in this circular are partnerships or corporations that directly use
aluminum as the raw material in producing finished products either purely or partly out of aluminum, or
their duly appointed representatives. These bidders may be based locally or overseas.6

In April 2003, NPC published an invitation for the pre-qualification of bidders for the public sale of its
scrap ACSR7 cables. Respondent Pinatubo Commercial, a trader of scrap materials such as copper,
aluminum, steel and other ferrous and non-ferrous materials, submitted a pre-qualification form to NPC.
Pinatubo, however, was informed in a letter dated April 29, 2003 that its application for pre-qualification
had been denied.8 Petitioner asked for reconsideration but NPC denied it.9

Pinatubo then filed a petition in the RTC for the annulment of NPC Circular No. 99-75, with a prayer for
the issuance of a temporary restraining order and/or writ of preliminary injunction.10 Pinatubo argued
that the circular was unconstitutional as it violated the due process and equal protection clauses of the
Constitution, and ran counter to the government policy of competitive public bidding.11

21
The RTC upheld Pinatubo’s position and declared items 3 and 3.1 of the circular unconstitutional. The
RTC ruled that it was violative of substantive due process because, while it created rights in favor of
third parties, the circular had not been published. It also pronounced that the circular violated the equal
protection clause since it favored manufacturers and processors of aluminum scrap vis-à-vis
dealers/traders in the purchase of aluminum ACSR cables from NPC. Lastly, the RTC found that the
circular denied traders the right to exercise their business and restrained free competition inasmuch as
it allowed only a certain sector to participate in the bidding.12

In this petition, NPC insists that there was no need to publish the circular since it was not of general
application. It was addressed only to particular persons or class of persons, namely the disposal
committees, heads of offices, regional and all other officials involved in the disposition of ACSRs. NPC
also contends that there was a substantial distinction between manufacturers and traders of aluminum
scrap materials specially viewed in the light of RA 7832.13 According to NPC, by limiting the prospective
bidders to manufacturers, it could easily monitor the market of its scrap ACSRs. There was rampant
fencing of stolen NPC wires. NPC likewise maintains that traders were not prohibited from participating
in the pre-qualification as long as they had a tie-up with a manufacturer.14

The questions that need to be resolved in this case are:

(1) whether NPC Circular No. 99-75 must be published; and

(2) whether items 3 and 3.1 of NPC Circular No. 99-75 -

(a) violated the equal protection clause of the Constitution and

(b) restrained free trade and competition.

Tañada v. Tuvera15 stressed the need for publication in order for statutes and administrative rules and
regulations to have binding force and effect, viz.:

22
x x x all statutes, including those of local application and private laws, shall be published as a condition
for their effectivity, which shall begin fifteen days after publication unless a different effectivity is fixed
by the legislature.

Covered by this rule are presidential decrees and executive orders promulgated by the President in the
exercise of legislative power or, at present, directly conferred by the Constitution. Administrative Rules
and Regulations must also be published if their purpose is to enforce or implement existing law pursuant
also to a valid delegation.16

Tañada, however, qualified that:

Interpretative regulations and those merely internal in nature, that is, regulating only the personnel of
the administrative agency and not the public, need not be published. Neither is publication required of
the so-called letters of instructions issued by administrative superiors concerning the rules or guidelines
to be followed by their subordinates in the performance of their duties.17 (emphasis ours)

In this case, NPC Circular No. 99-75 did not have to be published since it was merely an internal rule or
regulation. It did not purport to enforce or implement an existing law but was merely a directive issued
by the NPC President to his subordinates to regulate the proper and efficient disposal of scrap ACSRs to
qualified bidders. Thus, NPC Circular No. 99-75 defined the responsibilities of the different NPC
personnel in the disposal, pre-qualification, bidding and award of scrap ACSRS.18 It also provided for the
deposit of a proposal bond to be submitted by bidders, the approval of the award, mode of payment
and release of awarded scrap ACSRs.19 All these guidelines were addressed to the NPC personnel
involved in the bidding and award of scrap ACSRs. It did not, in any way, affect the rights of the public in
general or of any other person not involved in the bidding process. Assuming it affected individual
rights, it did so only remotely, indirectly and incidentally.

Pinatubo’s argument that items 3 and 3.1 of NPC Circular No. 99-75 deprived it of its "right to bid" or
that these conferred such right in favor of a third person is erroneous. Bidding, in its comprehensive
sense, means making an offer or an invitation to prospective contractors whereby the government
manifests its intention to invite proposals for the purchase of supplies, materials and equipment for
official business or public use, or for public works or repair.20 Bidding rules may specify other conditions
or require that the bidding process be subjected to certain reservations or qualifications.21 Since a bid
partakes of the nature of an offer to contract with the government,22 the government agency involved
may or may not accept it. Moreover, being the owner of the property subject of the bid, the

23
government has the power to determine who shall be its recipient, as well as under what terms it may
be awarded. In this sense, participation in the bidding process is a privilege inasmuch as it can only be
exercised under existing criteria imposed by the government itself. As such, prospective bidders,
including Pinatubo, cannot claim any demandable right to take part in it if they fail to meet these
criteria. Thus, it has been stated that under the traditional form of property ownership, recipients of
privileges or largesse from the government cannot be said to have property rights because they possess
no traditionally recognized proprietary interest therein.23

Also, as the discretion to accept or reject bids and award contracts is of such wide latitude, courts will
not interfere, unless it is apparent that such discretion is exercised arbitrarily, or used as a shield to a
fraudulent award. The exercise of that discretion is a policy decision that necessitates prior inquiry,
investigation, comparison, evaluation, and deliberation. This task can best be discharged by the
concerned government agencies, not by the courts. Courts will not interfere with executive or legislative
discretion exercised within those boundaries. Otherwise, they stray into the realm of policy decision-
making.24

Limiting qualified bidders in this case to partnerships or corporations that directly use aluminum as the
raw material in producing finished products made purely or partly of aluminum was an exercise of
discretion by the NPC. Unless the discretion was exercised arbitrarily or used as a subterfuge for fraud,
the Court will not interfere with the exercise of such discretion.

This brings to the fore the next question: whether items 3 and 3.1 of NPC Circular No. 99-75 violated the
equal protection clause of the Constitution.

The equal protection clause means that "no person or class of persons shall be deprived of the same
protection of laws which is enjoyed by other persons or other classes in the same place and in like
circumstances."25 The guaranty of the equal protection of the laws is not violated by a legislation based
on a reasonable classification.26 The equal protection clause, therefore, does not preclude classification
of individuals who may be accorded different treatment under the law as long as the classification is
reasonable and not arbitrary.271avvphi1

Items 3 and 3.1 met the standards of a valid classification. Indeed, as juxtaposed by the RTC, the
purpose of NPC Circular No. 99-75 was to dispose of the ACSR wires.28 As stated by Pinatubo, it was also
meant to earn income for the government.29 Nevertheless, the disposal and revenue-generating
objective of the circular was not an end in itself and could not bar NPC from imposing conditions for the

24
proper disposition and ultimately, the legitimate use of the scrap ACSR wires. In giving preference to
direct manufacturers and producers, it was the intent of NPC to support RA 7832, which penalizes the
theft of ACSR in excess of 100 MCM.30 The difference in treatment between direct manufacturers and
producers, on one hand, and traders, on the other, was rationalized by NPC as follows:

x x x NAPOCOR can now easily monitor the market of its scrap ACSR wires and verify whether or not a
person’s possession of such materials is legal or not; and consequently, prosecute under R.A. 7832,
those whose possession, control or custody of such material is unexplained. This is based upon the
reasonable presumption that if the buyer were a manufacturer or processor, the scrap ACSRs end with
him as the latter uses it to make finished products; but if the buyer were a trader, there is greater
probability that the purchased materials may pass from one trader to another. Should traders without
tie-up to manufacturers or processors of aluminum be allowed to participate in the bidding, the ACSRs
bidded out to them will likely co-mingle with those already proliferating in the illegal market. Thus, great
difficulty shall be encountered by NAPOCOR and/or those authorities tasked to implement R.A. 7832 in
determining whether or not the ACSRs found in the possession, control and custody of a person
suspected of theft [of] electric power transmission lines and materials are the fruit of the offense
defined in Section 3 of R.A. 7832.31

Items 3 and 3.1 clearly did not infringe on the equal protection clause as these were based on a
reasonable classification intended to protect, not the right of any business or trade but the integrity of
government property, as well as promote the objectives of RA 7832. Traders like Pinatubo could not
claim similar treatment as direct manufacturers/processors especially in the light of their failure to
negate the rationale behind the distinction.

Finally, items 3 and 3.1 of NPC Circular No. 99-75 did not restrain free trade or competition.

Pinatubo contends that the condition imposed by NPC under items 3 and 3.1 violated the principle of
competitiveness advanced by RA 9184 (Government Procurement Reform Act) which states:

SEC. 3. Governing Principles on Government Procurement. – All procurement of the national


government, its departments, bureaus, offices and agencies, including state universities and colleges,
government-owned and/or controlled corporations, government financial institutions and local
government units, shall, in all cases, be governed by these principles:

25
xxx

(b) Competitiveness by extending equal opportunity to enable private contracting parties who are
eligible and qualified to participate in public bidding. (emphasis ours)

The foregoing provision imposed the precondition that the contracting parties should be eligible and
qualified. It should be emphasized that the bidding process was not a "free-for-all" where any and all
interested parties, qualified or not, could take part. Section 5(e) of RA 9184 defines competitive bidding
as a "method of procurement which is open to participation by any interested party and which consists
of the following processes: advertisement, pre-bid conference, eligibility screening of prospective
bidders, receipt and opening of bids, evaluation of bids, post-qualification, and award of contract x x x."
The law categorically mandates that prospective bidders are subject to eligibility screening, and as
earlier stated, bidding rules may specify other conditions or order that the bidding process be subjected
to certain reservations or qualifications.32 Thus, in its pre-qualification guidelines issued for the sale of
scrap ACSRs, the NPC reserved the right to pre-disqualify any applicant who did not meet the
requirements for pre-qualification.33 Clearly, the competitiveness policy of a bidding process
presupposes the eligibility and qualification of a contestant; otherwise, it defeats the principle that only
"responsible" and "qualified" bidders can bid and be awarded government contracts.34 Our free
enterprise system is not based on a market of pure and unadulterated competition where the State
pursues a strict hands-off policy and follows the let-the-devil-devour-the-hindmost rule.35

Moreover, the mere fact that incentives and privileges are granted to certain enterprises to the
exclusion of others does not render the issuance unconstitutional for espousing unfair competition.36
While the Constitution enshrines free enterprise as a policy, it nonetheless reserves to the government
the power to intervene whenever necessary to promote the general welfare.37 In the present case, the
unregulated disposal and sale of scrap ACSR wires will hamper the government’s effort of curtailing the
pernicious practice of trafficking stolen government property. This is an evil sought to be prevented by
RA 7832 and certainly, it was well within the authority of the NPC to prescribe conditions in order to
prevent it.

WHEREFORE, the petition is hereby GRANTED. The decision of the Regional Trial Court of Mandaluyong
City, Branch 213 dated June 30, 2006 and resolution dated November 20, 2006 are REVERSED and SET
ASIDE. Civil Case No. MC-03-2179 for the annulment of NPC Circular No. 99-75 is hereby DISMISSED.

SO ORDERED.

26
G.R. No. 180643 September 4, 2008

ROMULO L. NERI, petitioner,

vs.

SENATE COMMITTEE ON ACCOUNTABILITY OF PUBLIC OFFICERS AND INVESTIGATIONS, SENATE


COMMITTEE ON TRADE AND COMMERCE, AND SENATE COMMITTEE ON NATIONAL DEFENSE AND
SECURITY, respondents.

RESOLUTION

LEONARDO-DE CASTRO, J.:

Executive privilege is not a personal privilege, but one that adheres to the Office of the President. It
exists to protect public interest, not to benefit a particular public official. Its purpose, among others, is
to assure that the nation will receive the benefit of candid, objective and untrammeled communication
and exchange of information between the President and his/her advisers in the process of shaping or
forming policies and arriving at decisions in the exercise of the functions of the Presidency under the
Constitution. The confidentiality of the President’s conversations and correspondence is not unique. It is
akin to the confidentiality of judicial deliberations. It possesses the same value as the right to privacy of
all citizens and more, because it is dictated by public interest and the constitutionally ordained
separation of governmental powers.

In these proceedings, this Court has been called upon to exercise its power of review and arbitrate a
hotly, even acrimoniously, debated dispute between the Court’s co-equal branches of government. In
this task, this Court should neither curb the legitimate powers of any of the co-equal and coordinate
branches of government nor allow any of them to overstep the boundaries set for it by our Constitution.
The competing interests in the case at bar are the claim of executive privilege by the President, on the
one hand, and the respondent Senate Committees’ assertion of their power to conduct legislative
inquiries, on the other. The particular facts and circumstances of the present case, stripped of the
politically and emotionally charged rhetoric from both sides and viewed in the light of settled
constitutional and legal doctrines, plainly lead to the conclusion that the claim of executive privilege
must be upheld.

Assailed in this motion for reconsideration is our Decision dated March 25, 2008 (the "Decision"),
granting the petition for certiorari filed by petitioner Romulo L. Neri against the respondent Senate
Committees on Accountability of Public Officers and Investigations,1 Trade and Commerce,2 and
National Defense and Security (collectively the "respondent Committees").3

A brief review of the facts is imperative.

27
On September 26, 2007, petitioner appeared before respondent Committees and testified for about
eleven (11) hours on matters concerning the National Broadband Project (the "NBN Project"), a project
awarded by the Department of Transportation and Communications ("DOTC") to Zhong Xing
Telecommunications Equipment ("ZTE"). Petitioner disclosed that then Commission on Elections
("COMELEC") Chairman Benjamin Abalos offered him P200 Million in exchange for his approval of the
NBN Project. He further narrated that he informed President Gloria Macapagal Arroyo ("President
Arroyo") of the bribery attempt and that she instructed him not to accept the bribe. However, when
probed further on President Arroyo and petitioner’s discussions relating to the NBN Project, petitioner
refused to answer, invoking "executive privilege." To be specific, petitioner refused to answer questions
on: (a) whether or not President Arroyo followed up the NBN Project,4 (b) whether or not she directed
him to prioritize it,5 and (c) whether or not she directed him to approve it.6

Respondent Committees persisted in knowing petitioner’s answers to these three questions by requiring
him to appear and testify once more on November 20, 2007. On November 15, 2007, Executive
Secretary Eduardo R. Ermita wrote to respondent Committees and requested them to dispense with
petitioner’s testimony on the ground of executive privilege.7 The letter of Executive Secretary Ermita
pertinently stated:

Following the ruling in Senate v. Ermita, the foregoing questions fall under conversations and
correspondence between the President and public officials which are considered executive privilege
(Almonte v. Vasquez, G.R. 95637, 23 May 1995; Chavez v. PEA, G.R. 133250, July 9, 2002). Maintaining
the confidentiality of conversations of the President is necessary in the exercise of her executive and
policy decision making process. The expectation of a President to the confidentiality of her
conversations and correspondences, like the value which we accord deference for the privacy of all
citizens, is the necessity for protection of the public interest in candid, objective, and even blunt or
harsh opinions in Presidential decision-making. Disclosure of conversations of the President will have a
chilling effect on the President, and will hamper her in the effective discharge of her duties and
responsibilities, if she is not protected by the confidentiality of her conversations.

The context in which executive privilege is being invoked is that the information sought to be disclosed
might impair our diplomatic as well as economic relations with the People’s Republic of China. Given the
confidential nature in which these information were conveyed to the President, he cannot provide the
Committee any further details of these conversations, without disclosing the very thing the privilege is
designed to protect.

In light of the above considerations, this Office is constrained to invoke the settled doctrine of executive
privilege as refined in Senate v. Ermita, and has advised Secretary Neri accordingly.

Considering that Sec. Neri has been lengthily interrogated on the subject in an unprecedented 11-hour
hearing, wherein he has answered all questions propounded to him except the foregoing questions
involving executive privilege, we therefore request that his testimony on 20 November 2007 on the ZTE
/ NBN project be dispensed with.

28
On November 20, 2007, petitioner did not appear before respondent Committees upon orders of the
President invoking executive privilege. On November 22, 2007, the respondent Committees issued the
show-cause letter requiring him to explain why he should not be cited in contempt. On November 29,
2007, in petitioner’s reply to respondent Committees, he manifested that it was not his intention to
ignore the Senate hearing and that he thought the only remaining questions were those he claimed to
be covered by executive privilege. He also manifested his willingness to appear and testify should there
be new matters to be taken up. He just requested that he be furnished "in advance as to what else" he
"needs to clarify."

Respondent Committees found petitioner’s explanations unsatisfactory. Without responding to his


request for advance notice of the matters that he should still clarify, they issued the Order dated
January 30, 2008; In Re: P.S. Res. Nos. 127,129,136 & 144; and privilege speeches of Senator Lacson and
Santiago (all on the ZTE-NBN Project), citing petitioner in contempt of respondent Committees and
ordering his arrest and detention at the Office of the Senate Sergeant-at-Arms until such time that he
would appear and give his testimony.

On the same date, petitioner moved for the reconsideration of the above Order.8 He insisted that he
had not shown "any contemptible conduct worthy of contempt and arrest." He emphasized his
willingness to testify on new matters, but respondent Committees did not respond to his request for
advance notice of questions. He also mentioned the petition for certiorari he previously filed with this
Court on December 7, 2007. According to him, this should restrain respondent Committees from
enforcing the order dated January 30, 2008 which declared him in contempt and directed his arrest and
detention.

Petitioner then filed his Supplemental Petition for Certiorari (with Urgent Application for
TRO/Preliminary Injunction) on February 1, 2008. In the Court’s Resolution dated February 4, 2008, the
parties were required to observe the status quo prevailing prior to the Order dated January 30, 2008.

On March 25, 2008, the Court granted his petition for certiorari on two grounds: first, the
communications elicited by the three (3) questions were covered by executive privilege; and second,
respondent Committees committed grave abuse of discretion in issuing the contempt order. Anent the
first ground, we considered the subject communications as falling under the presidential
communications privilege because (a) they related to a quintessential and non-delegable power of the
President, (b) they were received by a close advisor of the President, and (c) respondent Committees
failed to adequately show a compelling need that would justify the limitation of the privilege and the
unavailability of the information elsewhere by an appropriate investigating authority. As to the second
ground, we found that respondent Committees committed grave abuse of discretion in issuing the
contempt order because (a) there was a valid claim of executive privilege, (b) their invitations to
petitioner did not contain the questions relevant to the inquiry, (c) there was a cloud of doubt as to the
regularity of the proceeding that led to their issuance of the contempt order, (d) they violated Section
21, Article VI of the Constitution because their inquiry was not in accordance with the "duly published
rules of procedure," and (e) they issued the contempt order arbitrarily and precipitately.

29
On April 8, 2008, respondent Committees filed the present motion for reconsideration, anchored on the
following grounds:

CONTRARY TO THIS HONORABLE COURT’S DECISION, THERE IS NO DOUBT THAT THE ASSAILED ORDERS
WERE ISSUED BY RESPONDENT COMMITTEES PURSUANT TO THE EXERCISE OF THEIR LEGISLATIVE
POWER, AND NOT MERELY THEIR OVERSIGHT FUNCTIONS.

II

CONTRARY TO THIS HONORABLE COURT’S DECISION, THERE CAN BE NO PRESUMPTION THAT THE
INFORMATION WITHHELD IN THE INSTANT CASE IS PRIVILEGED.

III

CONTRARY TO THIS HONORABLE COURT’S DECISION, THERE IS NO FACTUAL OR LEGAL BASIS TO HOLD
THAT THE COMMUNICATIONS ELICITED BY THE SUBJECT THREE (3) QUESTIONS ARE COVERED BY
EXECUTIVE PRIVILEGE, CONSIDERING THAT:

A. THERE IS NO SHOWING THAT THE MATTERS FOR WHICH EXECUTIVE PRIVILEGE IS CLAIMED
CONSTITUTE STATE SECRETS.

B. EVEN IF THE TESTS ADOPTED BY THIS HONORABLE COURT IN THE DECISION IS APPLIED, THERE IS NO
SHOWING THAT THE ELEMENTS OF PRESIDENTIAL COMMUNICATIONS PRIVILEGE ARE PRESENT.

C. ON THE CONTRARY, THERE IS ADEQUATE SHOWING OF A COMPELLING NEED TO JUSTIFY THE


DISCLOSURE OF THE INFORMATION SOUGHT.

D. TO UPHOLD THE CLAIM OF EXECUTIVE PRIVILEGE IN THE INSTANT CASE WOULD SERIOUSLY IMPAIR
THE RESPONDENTS’ PERFORMANCE OF THEIR PRIMARY FUNCTION TO ENACT LAWS.

E. FINALLY, THE CONSTITUTIONAL RIGHT OF THE PEOPLE TO INFORMATION, AND THE CONSTITUTIONAL
POLICIES ON PUBLIC ACCOUNTABILITY AND TRANSPARENCY OUTWEIGH THE CLAIM OF EXECUTIVE
PRIVILEGE.

30
IV

CONTRARY TO THIS HONORABLE COURT’S DECISION, RESPONDENTS DID NOT COMMIT GRAVE ABUSE
OF DISCRETION IN ISSUING THE ASSAILED CONTEMPT ORDER, CONSIDERING THAT:

A. THERE IS NO LEGITIMATE CLAIM OF EXECUTIVE PRIVILEGE IN THE INSTANT CASE.

B. RESPONDENTS DID NOT VIOLATE THE SUPPOSED REQUIREMENTS LAID DOWN IN SENATE V. ERMITA.

C. RESPONDENTS DULY ISSUED THE CONTEMPT ORDER IN ACCORDANCE WITH THEIR INTERNAL RULES.

D. RESPONDENTS DID NOT VIOLATE THE REQUIREMENTS UNDER ARTICLE VI, SECTION 21 OF THE
CONSTITUTION REQUIRING THAT ITS RULES OF PROCEDURE BE DULY PUBLISHED, AND WERE DENIED
DUE PROCESS WHEN THE COURT CONSIDERED THE OSG’S INTERVENTION ON THIS ISSUE WITHOUT
GIVING RESPONDENTS THE OPPORTUNITY TO COMMENT.

E. RESPONDENTS’ ISSUANCE OF THE CONTEMPT ORDER IS NOT ARBITRARY OR PRECIPITATE.

In his Comment, petitioner charges respondent Committees with exaggerating and distorting the
Decision of this Court. He avers that there is nothing in it that prohibits respondent Committees from
investigating the NBN Project or asking him additional questions. According to petitioner, the Court
merely applied the rule on executive privilege to the facts of the case. He further submits the following
contentions: first, the assailed Decision did not reverse the presumption against executive secrecy laid
down in Senate v. Ermita; second, respondent Committees failed to overcome the presumption of
executive privilege because it appears that they could legislate even without the communications
elicited by the three (3) questions, and they admitted that they could dispense with petitioner’s
testimony if certain NEDA documents would be given to them; third, the requirement of specificity
applies only to the privilege for State, military and diplomatic secrets, not to the necessarily broad and
all-encompassing presidential communications privilege; fourth, there is no right to pry into the
President’s thought processes or exploratory exchanges; fifth, petitioner is not covering up or hiding
anything illegal; sixth, the Court has the power and duty to annul the Senate Rules; seventh, the Senate
is not a continuing body, thus the failure of the present Senate to publish its Rules of Procedure
Governing Inquiries in Aid of Legislation (Rules) has a vitiating effect on them; eighth, the requirement
for a witness to be furnished advance copy of questions comports with due process and the
constitutional mandate that the rights of witnesses be respected; and ninth, neither petitioner nor
respondent has the final say on the matter of executive privilege, only the Court.

For its part, the Office of the Solicitor General maintains that: (1) there is no categorical pronouncement
from the Court that the assailed Orders were issued by respondent Committees pursuant to their
oversight function; hence, there is no reason for them "to make much" of the distinction between
Sections 21 and 22, Article VI of the Constitution; (2) presidential communications enjoy a presumptive
privilege against disclosure as earlier held in Almonte v. Vasquez9 and Chavez v. Public Estates Authority
(PEA)10; (3) the communications elicited by the three (3) questions are covered by executive privilege,

31
because all the elements of the presidential communications privilege are present; (4) the subpoena ad
testificandum issued by respondent Committees to petitioner is fatally defective under existing law and
jurisprudence; (5) the failure of the present Senate to publish its Rules renders the same void; and (6)
respondent Committees arbitrarily issued the contempt order.

Incidentally, respondent Committees’ objection to the Resolution dated March 18, 2008 (granting the
Office of the Solicitor General’s Motion for Leave to Intervene and to Admit Attached Memorandum)
only after the promulgation of the Decision in this case is foreclosed by its untimeliness.

The core issues that arise from the foregoing respective contentions of the opposing parties are as
follows:

(1) whether or not there is a recognized presumptive presidential communications privilege in our legal
system;

(2) whether or not there is factual or legal basis to hold that the communications elicited by the three
(3) questions are covered by executive privilege;

(3) whether or not respondent Committees have shown that the communications elicited by the three
(3) questions are critical to the exercise of their functions; and

(4) whether or not respondent Committees committed grave abuse of discretion in issuing the contempt
order.

We shall discuss these issues seriatim.

There Is a Recognized Presumptive Presidential Communications Privilege

Respondent Committees ardently argue that the Court’s declaration that presidential communications
are presumptively privileged reverses the "presumption" laid down in Senate v. Ermita11 that "inclines
heavily against executive secrecy and in favor of disclosure." Respondent Committees then claim that
the Court erred in relying on the doctrine in Nixon.

Respondent Committees argue as if this were the first time the presumption in favor of the presidential
communications privilege is mentioned and adopted in our legal system. That is far from the truth. The
Court, in the earlier case of Almonte v. Vasquez,12 affirmed that the presidential communications
privilege is fundamental to the operation of government and inextricably rooted in the separation of
powers under the Constitution. Even Senate v. Ermita,13 the case relied upon by respondent
Committees, reiterated this concept. There, the Court enumerated the cases in which the claim of
executive privilege was recognized, among them Almonte v. Chavez, Chavez v. Presidential Commission
on Good Government (PCGG),14 and Chavez v. PEA.15 The Court articulated in these cases that "there
are certain types of information which the government may withhold from the public,16" that there is a

32
"governmental privilege against public disclosure with respect to state secrets regarding military,
diplomatic and other national security matters";17 and that "the right to information does not extend to
matters recognized as ‘privileged information’ under the separation of powers, by which the Court
meant Presidential conversations, correspondences, and discussions in closed-door Cabinet meetings."

Respondent Committees’ observation that this Court’s Decision reversed the "presumption that inclines
heavily against executive secrecy and in favor of disclosure" arises from a piecemeal interpretation of
the said Decision. The Court has repeatedly held that in order to arrive at the true intent and meaning of
a decision, no specific portion thereof should be isolated and resorted to, but the decision must be
considered in its entirety.

Note that the aforesaid presumption is made in the context of the circumstances obtaining in Senate v.
Ermita, which declared void Sections 2(b) and 3 of Executive Order (E.O.) No. 464, Series of 2005. The
pertinent portion of the decision in the said case reads:

From the above discussion on the meaning and scope of executive privilege, both in the United States
and in this jurisprudence, a clear principle emerges. Executive privilege, whether asserted against
Congress, the courts, or the public, is recognized only in relation to certain types of information of a
sensitive character. While executive privilege is a constitutional concept, a claim thereof may be valid or
not depending on the ground invoked to justify it and the context in which it is made. Noticeably absent
is any recognition that executive officials are exempt from the duty to disclose information by the mere
fact of being executive officials. Indeed, the extraordinary character of the exemptions indicates that the
presumption inclines heavily against executive secrecy and in favor of disclosure. (Emphasis and
underscoring supplied)

Obviously, the last sentence of the above-quoted paragraph in Senate v. Ermita refers to the
"exemption" being claimed by the executive officials mentioned in Section 2(b) of E.O. No. 464, solely by
virtue of their positions in the Executive Branch. This means that when an executive official, who is one
of those mentioned in the said Sec. 2(b) of E.O. No. 464, claims to be exempt from disclosure, there can
be no presumption of authorization to invoke executive privilege given by the President to said
executive official, such that the presumption in this situation inclines heavily against executive secrecy
and in favor of disclosure.

Senate v. Ermita 20 expounds on the premise of the foregoing ruling in this wise:

Section 2(b) in relation to Section 3 virtually provides that, once the head of office determines that a
certain information is privileged, such determination is presumed to bear the President’s authority and
has the effect of prohibiting the official from appearing before Congress, subject only to the express
pronouncement of the President that it is allowing the appearance of such official. These provisions thus
allow the President to authorize claims of privilege by mere silence.

33
Such presumptive authorization, however, is contrary to the exceptional nature of the privilege.
Executive privilege, as already discussed, is recognized with respect to information the confidential
nature of which is crucial to the fulfillment of the unique role and responsibilities of the executive
branch, or in those instances where exemption from disclosure is necessary to the discharge of highly
important executive responsibilities. The doctrine of executive privilege is thus premised on the fact that
certain information must, as a matter of necessity, be kept confidential in pursuit of the public interest.
The privilege being, by definition, an exemption from the obligation to disclose information, in this case
to Congress, the necessity must be of such high degree as to outweigh the public interest in enforcing
that obligation in a particular case.

In light of this highly exceptional nature of the privilege, the Court finds it essential to limit to the
President the power to invoke the privilege. She may of course authorize the Executive Secretary to
invoke the privilege on her behalf, in which case the Executive Secretary must state that the authority is
"By order of the President", which means that he personally consulted with her. The privilege being an
extraordinary power, it must be wielded only by the highest official in the executive hierarchy. In other
words, the President may not authorize her subordinates to exercise such power. There is even less
reason to uphold such authorization in the instant case where the authorization is not explicit but by
mere silence. Section 3, in relation to Section 2(b), is further invalid on this score.

The constitutional infirmity found in the blanket authorization to invoke executive privilege granted by
the President to executive officials in Sec. 2(b) of E.O. No. 464 does not obtain in this case.

In this case, it was the President herself, through Executive Secretary Ermita, who invoked executive
privilege on a specific matter involving an executive agreement between the Philippines and China,
which was the subject of the three (3) questions propounded to petitioner Neri in the course of the
Senate Committees’ investigation. Thus, the factual setting of this case markedly differs from that
passed upon in Senate v. Ermita.

Moreover, contrary to the claim of respondents, the Decision in this present case hews closely to the
ruling in Senate v. Ermita, to wit:

Executive privilege

The phrase "executive privilege" is not new in this jurisdiction. It has been used even prior to the
promulgation of the 1986 Constitution. Being of American origin, it is best understood in light of how it
has been defined and used in the legal literature of the United States.

Schwart defines executive privilege as "the power of the Government to withhold information from the
public, the courts, and the Congress. Similarly, Rozell defines it as "the right of the President and high-
level executive branch officers to withhold information from Congress, the courts, and ultimately the
public." x x x In this jurisdiction, the doctrine of executive privilege was recognized by this Court in

34
Almonte v. Vasquez. Almonte used the term in reference to the same privilege subject of Nixon. It
quoted the following portion of the Nixon decision which explains the basis for the privilege:

"The expectation of a President to the confidentiality of his conversations and correspondences, like the
claim of confidentiality of judicial deliberations, for example, he has all the values to which we accord
deference for the privacy of all citizens and, added to those values, is the necessity for protection of the
public interest in candid, objective, and even blunt or harsh opinions in Presidential decision-making. A
President and those who assist him must be free to explore alternatives in the process of shaping
policies and making decisions and to do so in a way many would be unwilling to express except privately.
These are the considerations justifying a presumptive privilege for Presidential communications. The
privilege is fundamental to the operation of government and inextricably rooted in the separation of
powers under the Constitution x x x " (Emphasis and italics supplied)

Clearly, therefore, even Senate v. Ermita adverts to "a presumptive privilege for Presidential
communication," which was recognized early on in Almonte v. Vasquez. To construe the passage in
Senate v. Ermita adverted to in the Motion for Reconsideration of respondent Committees, referring to
the non-existence of a "presumptive authorization" of an executive official, to mean that the
"presumption" in favor of executive privilege "inclines heavily against executive secrecy and in favor of
disclosure" is to distort the ruling in the Senate v. Ermita and make the same engage in self-
contradiction.

Senate v. Ermita expounds on the constitutional underpinning of the relationship between the Executive
Department and the Legislative Department to explain why there should be no implied authorization or
presumptive authorization to invoke executive privilege by the President’s subordinate officials, as
follows:

When Congress exercises its power of inquiry, the only way for department heads to exempt themselves
therefrom is by a valid claim of privilege. They are not exempt by the mere fact that they are
department heads. Only one executive official may be exempted from this power - the President on
whom executive power is vested, hence, beyond the reach of Congress except through the power of
impeachment. It is based on he being the highest official of the executive branch, and the due respect
accorded to a co-equal branch of governments which is sanctioned by a long-standing custom.
(Underscoring supplied)

Thus, if what is involved is the presumptive privilege of presidential communications when invoked by
the President on a matter clearly within the domain of the Executive, the said presumption dictates that
the same be recognized and be given preference or priority, in the absence of proof of a compelling or
critical need for disclosure by the one assailing such presumption. Any construction to the contrary will
render meaningless the presumption accorded by settled jurisprudence in favor of executive privilege. In
fact, Senate v. Ermita reiterates jurisprudence citing "the considerations justifying a presumptive
privilege for Presidential communications."23

35
II

There Are Factual and Legal Bases to Hold that the Communications Elicited by the Three (3) Questions
Are Covered by Executive Privilege

Respondent Committees claim that the communications elicited by the three (3) questions are not
covered by executive privilege because the elements of the presidential communications privilege are
not present.

A. The power to enter into an executive agreement is a "quintessential and non-delegable presidential
power."

First, respondent Committees contend that the power to secure a foreign loan does not relate to a
"quintessential and non-delegable presidential power," because the Constitution does not vest it in the
President alone, but also in the Monetary Board which is required to give its prior concurrence and to
report to Congress.

This argument is unpersuasive.

The fact that a power is subject to the concurrence of another entity does not make such power less
executive. "Quintessential" is defined as the most perfect embodiment of something, the concentrated
essence of substance.24 On the other hand, "non-delegable" means that a power or duty cannot be
delegated to another or, even if delegated, the responsibility remains with the obligor.25 The power to
enter into an executive agreement is in essence an executive power. This authority of the President to
enter into executive agreements without the concurrence of the Legislature has traditionally been
recognized in Philippine jurisprudence.26 Now, the fact that the President has to secure the prior
concurrence of the Monetary Board, which shall submit to Congress a complete report of its decision
before contracting or guaranteeing foreign loans, does not diminish the executive nature of the power.

The inviolate doctrine of separation of powers among the legislative, executive and judicial branches of
government by no means prescribes absolute autonomy in the discharge by each branch of that part of
the governmental power assigned to it by the sovereign people. There is the corollary doctrine of checks
and balances, which has been carefully calibrated by the Constitution to temper the official acts of each
of these three branches. Thus, by analogy, the fact that certain legislative acts require action from the
President for their validity does not render such acts less legislative in nature. A good example is the
power to pass a law. Article VI, Section 27 of the Constitution mandates that every bill passed by
Congress shall, before it becomes a law, be presented to the President who shall approve or veto the
same. The fact that the approval or vetoing of the bill is lodged with the President does not render the
power to pass law executive in nature. This is because the power to pass law is generally a
quintessential and non-delegable power of the Legislature. In the same vein, the executive power to
enter or not to enter into a contract to secure foreign loans does not become less executive in nature

36
because of conditions laid down in the Constitution. The final decision in the exercise of the said
executive power is still lodged in the Office of the President.

B. The "doctrine of operational proximity" was laid down precisely to limit the scope of the presidential
communications privilege but, in any case, it is not conclusive.

Second, respondent Committees also seek reconsideration of the application of the "doctrine of
operational proximity" for the reason that "it maybe misconstrued to expand the scope of the
presidential communications privilege to communications between those who are ‘operationally
proximate’ to the President but who may have "no direct communications with her."

It must be stressed that the doctrine of "operational proximity" was laid down in In re: Sealed Case
precisely to limit the scope of the presidential communications privilege. The U.S. court was aware of
the dangers that a limitless extension of the privilege risks and, therefore, carefully cabined its reach by
explicitly confining it to White House staff, and not to staffs of the agencies, and then only to White
House staff that has "operational proximity" to direct presidential decision-making, thus:

We are aware that such an extension, unless carefully circumscribed to accomplish the purposes of the
privilege, could pose a significant risk of expanding to a large swath of the executive branch a privilege
that is bottomed on a recognition of the unique role of the President. In order to limit this risk, the
presidential communications privilege should be construed as narrowly as is consistent with ensuring
that the confidentiality of the President’s decision-making process is adequately protected. Not every
person who plays a role in the development of presidential advice, no matter how remote and removed
from the President, can qualify for the privilege. In particular, the privilege should not extend to staff
outside the White House in executive branch agencies. Instead, the privilege should apply only to
communications authored or solicited and received by those members of an immediate White House
advisor’s staff who have broad and significant responsibility for investigation and formulating the advice
to be given the President on the particular matter to which the communications relate. Only
communications at that level are close enough to the President to be revelatory of his deliberations or
to pose a risk to the candor of his advisers. See AAPS, 997 F.2d at 910 (it is "operational proximity" to
the President that matters in determining whether "[t]he President’s confidentiality interests" is
implicated). (Emphasis supplied)

In the case at bar, the danger of expanding the privilege "to a large swath of the executive branch" (a
fear apparently entertained by respondents) is absent because the official involved here is a member of
the Cabinet, thus, properly within the term "advisor" of the President; in fact, her alter ego and a
member of her official family. Nevertheless, in circumstances in which the official involved is far too
remote, this Court also mentioned in the Decision the organizational test laid down in Judicial Watch,
Inc. v. Department of Justice.28 This goes to show that the operational proximity test used in the
Decision is not considered conclusive in every case. In determining which test to use, the main
consideration is to limit the availability of executive privilege only to officials who stand proximate to the
President, not only by reason of their function, but also by reason of their positions in the Executive’s

37
organizational structure. Thus, respondent Committees’ fear that the scope of the privilege would be
unnecessarily expanded with the use of the operational proximity test is unfounded.

C. The President’s claim of executive privilege is not merely based on a generalized interest; and in
balancing respondent Committees’ and the President’s clashing interests, the Court did not disregard
the 1987 Constitutional provisions on government transparency, accountability and disclosure of
information.

Third, respondent Committees claim that the Court erred in upholding the President’s invocation,
through the Executive Secretary, of executive privilege because (a) between respondent Committees’
specific and demonstrated need and the President’s generalized interest in confidentiality, there is a
need to strike the balance in favor of the former; and (b) in the balancing of interest, the Court
disregarded the provisions of the 1987 Philippine Constitution on government transparency,
accountability and disclosure of information, specifically, Article III, Section 7;29 Article II, Sections 2430
and 28;31 Article XI, Section 1;32 Article XVI, Section 10;33 Article VII, Section 20;34 and Article XII,
Sections 9,35 21,36 and 22.37

It must be stressed that the President’s claim of executive privilege is not merely founded on her
generalized interest in confidentiality. The Letter dated November 15, 2007 of Executive Secretary
Ermita specified presidential communications privilege in relation to diplomatic and economic relations
with another sovereign nation as the bases for the claim. Thus, the Letter stated:

The context in which executive privilege is being invoked is that the information sought to be disclosed
might impair our diplomatic as well as economic relations with the People’s Republic of China. Given the
confidential nature in which this information were conveyed to the President, he cannot provide the
Committee any further details of these conversations, without disclosing the very thing the privilege is
designed to protect. (emphasis supplied)

Even in Senate v. Ermita, it was held that Congress must not require the Executive to state the reasons
for the claim with such particularity as to compel disclosure of the information which the privilege is
meant to protect. This is a matter of respect for a coordinate and co-equal department.

It is easy to discern the danger that goes with the disclosure of the President’s communication with her
advisor. The NBN Project involves a foreign country as a party to the agreement. It was actually a
product of the meeting of minds between officials of the Philippines and China. Whatever the President
says about the agreement - particularly while official negotiations are ongoing - are matters which China
will surely view with particular interest. There is danger in such kind of exposure. It could adversely
affect our diplomatic as well as economic relations with the People’s Republic of China. We reiterate the
importance of secrecy in matters involving foreign negotiations as stated in United States v. Curtiss-
Wright Export Corp., 38 thus:

38
The nature of foreign negotiations requires caution, and their success must often depend on secrecy,
and even when brought to a conclusion, a full disclosure of all the measures, demands, or eventual
concessions which may have been proposed or contemplated would be extremely impolitic, for this
might have a pernicious influence on future negotiations or produce immediate inconveniences,
perhaps danger and mischief, in relation to other powers. The necessity of such caution and secrecy was
one cogent reason for vesting the power of making treaties in the President, with the advice and
consent of the Senate, the principle on which the body was formed confining it to a small number of
members. To admit, then, a right in the House of Representatives to demand and to have as a matter of
course all the papers respecting a negotiation with a foreign power would be to establish a dangerous
precedent.

US jurisprudence clearly guards against the dangers of allowing Congress access to all papers relating to
a negotiation with a foreign power. In this jurisdiction, the recent case of Akbayan Citizens Action Party,
et al. v. Thomas G. Aquino, et al.39 upheld the privileged character of diplomatic negotiations. In
Akbayan, the Court stated:

Privileged character of diplomatic negotiations

The privileged character of diplomatic negotiations has been recognized in this jurisdiction. In discussing
valid limitations on the right to information, the Court in Chavez v. PCGG held that "information on
inter-government exchanges prior to the conclusion of treaties and executive agreements may be
subject to reasonable safeguards for the sake of national interest." Even earlier, the same privilege was
upheld in People’s Movement for Press Freedom (PMPF) v. Manglapus wherein the Court discussed the
reasons for the privilege in more precise terms.

In PMPF v. Manglapus, the therein petitioners were seeking information from the President’s
representatives on the state of the then on-going negotiations of the RP-US Military Bases Agreement.
The Court denied the petition, stressing that "secrecy of negotiations with foreign countries is not
violative of the constitutional provisions of freedom of speech or of the press nor of the freedom of
access to information." The Resolution went on to state, thus:

The nature of diplomacy requires centralization of authority and expedition of decision which are
inherent in executive action. Another essential characteristic of diplomacy is its confidential nature.
Although much has been said about "open" and "secret" diplomacy, with disparagement of the latter,
Secretaries of State Hughes and Stimson have clearly analyzed and justified the practice. In the words of
Mr. Stimson:

"A complicated negotiation …cannot be carried through without many, many private talks and
discussion, man to man; many tentative suggestions and proposals. Delegates from other countries
come and tell you in confidence of their troubles at home and of their differences with other countries
and with other delegates; they tell you of what they would do under certain circumstances and would
not do under other circumstances… If these reports… should become public… who would ever trust
American Delegations in another conference? (United States Department of State, Press Releases, June
7, 1930, pp. 282-284)

39
xxxx

There is frequent criticism of the secrecy in which negotiation with foreign powers on nearly all subjects
is concerned. This, it is claimed, is incompatible with the substance of democracy. As expressed by one
writer, "It can be said that there is no more rigid system of silence anywhere in the world." (E.J. Young,
Looking Behind the Censorship, J. B. Lipincott Co., 1938) President Wilson in starting his efforts for the
conclusion of the World War declared that we must have "open covenants, openly arrived at." He
quickly abandoned his thought.

No one who has studied the question believes that such a method of publicity is possible. In the
moment that negotiations are started, pressure groups attempt to "muscle in." An ill-timed speech by
one of the parties or a frank declaration of the concession which are exacted or offered on both sides
would quickly lead to a widespread propaganda to block the negotiations. After a treaty has been
drafted and its terms are fully published, there is ample opportunity for discussion before it is approved.
(The New American Government and Its Works, James T. Young, 4th Edition, p. 194) (Emphasis and
underscoring supplied)

Still in PMPF v. Manglapus, the Court adopted the doctrine in U.S. v. Curtiss-Wright Export Corp. that the
President is the sole organ of the nation in its negotiations with foreign countries,viz:

"x x x In this vast external realm, with its important, complicated, delicate and manifold problems, the
President alone has the power to speak or listen as a representative of the nation. He makes treaties
with the advice and consent of the Senate; but he alone negotiates. Into the field of negotiation the
Senate cannot intrude; and Congress itself is powerless to invade it. As Marshall said in his great
arguments of March 7, 1800, in the House of Representatives, "The President is the sole organ of the
nation in its external relations, and its sole representative with foreign nations." Annals, 6th Cong., col.
613… (Emphasis supplied; underscoring in the original)

Considering that the information sought through the three (3) questions subject of this Petition involves
the President’s dealings with a foreign nation, with more reason, this Court is wary of approving the
view that Congress may peremptorily inquire into not only official, documented acts of the President but
even her confidential and informal discussions with her close advisors on the pretext that said questions
serve some vague legislative need. Regardless of who is in office, this Court can easily foresee unwanted
consequences of subjecting a Chief Executive to unrestricted congressional inquiries done with
increased frequency and great publicity. No Executive can effectively discharge constitutional functions
in the face of intense and unchecked legislative incursion into the core of the President’s decision-
making process, which inevitably would involve her conversations with a member of her Cabinet.

40
With respect to respondent Committees’ invocation of constitutional prescriptions regarding the right of
the people to information and public accountability and transparency, the Court finds nothing in these
arguments to support respondent Committees’ case.

There is no debate as to the importance of the constitutional right of the people to information and the
constitutional policies on public accountability and transparency. These are the twin postulates vital to
the effective functioning of a democratic government. The citizenry can become prey to the whims and
caprices of those to whom the power has been delegated if they are denied access to information. And
the policies on public accountability and democratic government would certainly be mere empty words
if access to such information of public concern is denied.

In the case at bar, this Court, in upholding executive privilege with respect to three (3) specific
questions, did not in any way curb the public’s right to information or diminish the importance of public
accountability and transparency.

This Court did not rule that the Senate has no power to investigate the NBN Project in aid of legislation.
There is nothing in the assailed Decision that prohibits respondent Committees from inquiring into the
NBN Project. They could continue the investigation and even call petitioner Neri to testify again. He
himself has repeatedly expressed his willingness to do so. Our Decision merely excludes from the scope
of respondents’ investigation the three (3) questions that elicit answers covered by executive privilege
and rules that petitioner cannot be compelled to appear before respondents to answer the said
questions. We have discussed the reasons why these answers are covered by executive privilege. That
there is a recognized public interest in the confidentiality of such information is a recognized principle in
other democratic States. To put it simply, the right to information is not an absolute right.

Indeed, the constitutional provisions cited by respondent Committees do not espouse an absolute right
to information. By their wording, the intention of the Framers to subject such right to the regulation of
the law is unmistakable. The highlighted portions of the following provisions show the obvious
limitations on the right to information, thus:

Article III, Sec. 7. The right of the people to information on matters of public concern shall be
recognized. Access to official records, and to documents, and papers pertaining to official records, and
to documents, and papers pertaining to official acts, transactions, or decisions, as well as to government
research data used as basis for policy development, shall be afforded the citizen, subject to such
limitations as may be provided by law.

Article II, Sec. 28. Subject to reasonable conditions prescribed by law, the State adopts and implements
a policy of full public disclosure of all its transactions involving public interest. (Emphasis supplied)

41
In Chavez v. Presidential Commission on Good Government,40 it was stated that there are no specific
laws prescribing the exact limitations within which the right may be exercised or the correlative state
duty may be obliged. Nonetheless, it enumerated the recognized restrictions to such rights, among
them: (1) national security matters, (2) trade secrets and banking transactions, (3) criminal matters, and
(4) other confidential information. National security matters include state secrets regarding military and
diplomatic matters, as well as information on inter-government exchanges prior to the conclusion of
treaties and executive agreements. It was further held that even where there is no need to protect such
state secrets, they must be "examined in strict confidence and given scrupulous protection."

Incidentally, the right primarily involved here is the right of respondent Committees to obtain
information allegedly in aid of legislation, not the people’s right to public information. This is the reason
why we stressed in the assailed Decision the distinction between these two rights. As laid down in
Senate v. Ermita, "the demand of a citizen for the production of documents pursuant to his right to
information does not have the same obligatory force as a subpoena duces tecum issued by Congress"
and "neither does the right to information grant a citizen the power to exact testimony from
government officials." As pointed out, these rights belong to Congress, not to the individual citizen. It is
worth mentioning at this juncture that the parties here are respondent Committees and petitioner Neri
and that there was no prior request for information on the part of any individual citizen. This Court will
not be swayed by attempts to blur the distinctions between the Legislature's right to information in a
legitimate legislative inquiry and the public's right to information.

For clarity, it must be emphasized that the assailed Decision did not enjoin respondent Committees from
inquiring into the NBN Project. All that is expected from them is to respect matters that are covered by
executive privilege.

III.

Respondent Committees Failed to Show That

the Communications Elicited by the Three Questions

Are Critical to the Exercise of their Functions

In their Motion for Reconsideration, respondent Committees devote an unusually lengthy discussion on
the purported legislative nature of their entire inquiry, as opposed to an oversight inquiry.

42
At the outset, it must be clarified that the Decision did not pass upon the nature of respondent
Committees’ inquiry into the NBN Project. To reiterate, this Court recognizes respondent Committees’
power to investigate the NBN Project in aid of legislation. However, this Court cannot uphold the view
that when a constitutionally guaranteed privilege or right is validly invoked by a witness in the course of
a legislative investigation, the legislative purpose of respondent Committees’ questions can be
sufficiently supported by the expedient of mentioning statutes and/or pending bills to which their
inquiry as a whole may have relevance. The jurisprudential test laid down by this Court in past decisions
on executive privilege is that the presumption of privilege can only be overturned by a showing of
compelling need for disclosure of the information covered by executive privilege.

In the Decision, the majority held that "there is no adequate showing of a compelling need that would
justify the limitation of the privilege and of the unavailability of the information elsewhere by an
appropriate investigating authority." In the Motion for Reconsideration, respondent Committees argue
that the information elicited by the three (3) questions are necessary in the discharge of their legislative
functions, among them, (a) to consider the three (3) pending Senate Bills, and (b) to curb graft and
corruption.

We remain unpersuaded by respondents’ assertions.

In U.S. v. Nixon, the U.S. Court held that executive privilege is subject to balancing against other
interests and it is necessary to resolve the competing interests in a manner that would preserve the
essential functions of each branch. There, the Court weighed between presidential privilege and the
legitimate claims of the judicial process. In giving more weight to the latter, the Court ruled that the
President's generalized assertion of privilege must yield to the demonstrated, specific need for evidence
in a pending criminal trial.

The Nixon Court ruled that an absolute and unqualified privilege would stand in the way of the primary
constitutional duty of the Judicial Branch to do justice in criminal prosecutions. The said Court further
ratiocinated, through its ruling extensively quoted in the Honorable Chief Justice Puno's dissenting
opinion, as follows:

"... this presumptive privilege must be considered in light of our historic commitment to the rule of law.
This is nowhere more profoundly manifest than in our view that 'the twofold aim (of criminal justice) is

43
that guild shall not escape or innocence suffer.' Berger v. United States, 295 U.S., at 88, 55 S.Ct., at 633.
We have elected to employ an adversary system of criminal justice in which the parties contest all issues
before a court of law. The need to develop all relevant facts in the adversary system is both fundamental
and comprehensive. The ends of criminal justice would be defeated if judgments were to be founded on
a partial or speculative presentation of the facts. The very integrity of the judicial system and public
confidence in the system depend on full disclosure of all the facts, within the framework of the rules of
evidence. To ensure that justice is done, it is imperative to the function of courts that compulsory
process be available for the production of evidence needed either by the prosecution or by the defense.

xxx xxx xxx

The right to the production of all evidence at a criminal trial similarly has constitutional dimensions. The
Sixth Amendment explicitly confers upon every defendant in a criminal trial the right 'to be confronted
with the witness against him' and 'to have compulsory process for obtaining witnesses in his favor.'
Moreover, the Fifth Amendment also guarantees that no person shall be deprived of liberty without due
process of law. It is the manifest duty of the courts to vindicate those guarantees, and to accomplish
that it is essential that all relevant and admissible evidence be produced.

In this case we must weigh the importance of the general privilege of confidentiality of Presidential
communications in performance of the President's responsibilities against the inroads of such a privilege
on the fair administration of criminal justice. (emphasis supplied)

xxx xxx xxx

...the allowance of the privilege to withhold evidence that is demonstrably relevant in a criminal trial
would cut deeply into the guarantee of due process of law and gravely impair the basic function of the
courts. A President's acknowledged need for confidentiality in the communications of his office is
general in nature, whereas the constitutional need for production of relevant evidence in a criminal
proceeding is specific and central to the fair adjudication of a particular criminal case in the
administration of justice. Without access to specific facts a criminal prosecution may be totally
frustrated. The President's broad interest in confidentiality of communication will not be vitiated by
disclosure of a limited number of conversations preliminarily shown to have some bearing on the
pending criminal cases.

44
We conclude that when the ground for asserting privilege as to subpoenaed materials sought for use in
a criminal trial is based only on the generalized interest in confidentiality, it cannot prevail over the
fundamental demands of due process of law in the fair administration of criminal justice. The
generalized assertion of privilege must yield to the demonstrated, specific need for evidence in a
pending criminal trial. (emphasis supplied)

In the case at bar, we are not confronted with a court’s need for facts in order to adjudge liability in a
criminal case but rather with the Senate’s need for information in relation to its legislative functions.
This leads us to consider once again just how critical is the subject information in the discharge of
respondent Committees’ functions. The burden to show this is on the respondent Committees, since
they seek to intrude into the sphere of competence of the President in order to gather information
which, according to said respondents, would "aid" them in crafting legislation.

Senate Select Committee on Presidential Campaign Activities v. Nixon41 expounded on the nature of a
legislative inquiry in aid of legislation in this wise:

The sufficiency of the Committee's showing of need has come to depend, therefore, entirely on whether
the subpoenaed materials are critical to the performance of its legislative functions. There is a clear
difference between Congress' legislative tasks and the responsibility of a grand jury, or any institution
engaged in like functions. While fact-finding by a legislative committee is undeniably a part of its task,
legislative judgments normally depend more on the predicted consequences of proposed legislative
actions and their political acceptability, than on precise reconstruction of past events; Congress
frequently legislates on the basis of conflicting information provided in its hearings. In contrast, the
responsibility of the grand jury turns entirely on its ability to determine whether there is probable cause
to believe that certain named individuals did or did not commit specific crimes. If, for example, as in
Nixon v. Sirica, one of those crimes is perjury concerning the content of certain conversations, the grand
jury's need for the most precise evidence, the exact text of oral statements recorded in their original
form, is undeniable. We see no comparable need in the legislative process, at least not in the
circumstances of this case. Indeed, whatever force there might once have been in the Committee's
argument that the subpoenaed materials are necessary to its legislative judgments has been
substantially undermined by subsequent events. (Emphasis supplied)

Clearly, the need for hard facts in crafting legislation cannot be equated with the compelling or
demonstratively critical and specific need for facts which is so essential to the judicial power to
adjudicate actual controversies. Also, the bare standard of "pertinency" set in Arnault cannot be lightly

45
applied to the instant case, which unlike Arnault involves a conflict between two (2) separate, co-equal
and coordinate Branches of the Government.

Whatever test we may apply, the starting point in resolving the conflicting claims between the Executive
and the Legislative Branches is the recognized existence of the presumptive presidential
communications privilege. This is conceded even in the Dissenting Opinion of the Honorable Chief
Justice Puno, which states:

A hard look at Senate v. Ermita ought to yield the conclusion that it bestowed a qualified presumption in
favor of the Presidential communications privilege. As shown in the previous discussion, U.S. v. Nixon, as
well as the other related Nixon cases Sirica and Senate Select Committee on Presidential Campaign
Activities, et al., v. Nixon in the D.C. Court of Appeals, as well as subsequent cases all recognize that
there is a presumptive privilege in favor of Presidential communications. The Almonte case quoted U.S.
v. Nixon and recognized a presumption in favor of confidentiality of Presidential communications.

The presumption in favor of Presidential communications puts the burden on the respondent Senate
Committees to overturn the presumption by demonstrating their specific need for the information to be
elicited by the answers to the three (3) questions subject of this case, to enable them to craft legislation.
Here, there is simply a generalized assertion that the information is pertinent to the exercise of the
power to legislate and a broad and non-specific reference to pending Senate bills. It is not clear what
matters relating to these bills could not be determined without the said information sought by the three
(3) questions. As correctly pointed out by the Honorable Justice Dante O. Tinga in his Separate
Concurring Opinion:

…If respondents are operating under the premise that the president and/or her executive officials have
committed wrongdoings that need to be corrected or prevented from recurring by remedial legislation,
the answer to those three questions will not necessarily bolster or inhibit respondents from proceeding
with such legislation. They could easily presume the worst of the president in enacting such legislation.

For sure, a factual basis for situations covered by bills is not critically needed before legislatives bodies
can come up with relevant legislation unlike in the adjudication of cases by courts of law. Interestingly,
during the Oral Argument before this Court, the counsel for respondent Committees impliedly admitted
that the Senate could still come up with legislations even without petitioner answering the three (3)
questions. In other words, the information being elicited is not so critical after all. Thus:

46
CHIEF JUSTICE PUNO

So can you tell the Court how critical are these questions to the lawmaking function of the Senate. For
instance, question Number 1 whether the President followed up the NBN project. According to the
other counsel this question has already been asked, is that correct?

ATTY. AGABIN

Well, the question has been asked but it was not answered, Your Honor.

CHIEF JUSTICE PUNO

Yes. But my question is how critical is this to the lawmaking function of the Senate?

ATTY. AGABIN

I believe it is critical, Your Honor.

CHIEF JUSTICE PUNO

Why?

ATTY. AGABIN

47
For instance, with respect to the proposed Bill of Senator Miriam Santiago, she would like to indorse a
Bill to include Executive Agreements had been used as a device to the circumventing the Procurement
Law.

CHIEF JUSTICE PUNO

But the question is just following it up.

ATTY. AGABIN

I believe that may be the initial question, Your Honor, because if we look at this problem in its factual
setting as counsel for petitioner has observed, there are intimations of a bribery scandal involving high
government officials.

CHIEF JUSTICE PUNO

Again, about the second question, were you dictated to prioritize this ZTE, is that critical to the
lawmaking function of the Senate? Will it result to the failure of the Senate to cobble a Bill without this
question?

ATTY. AGABIN

I think it is critical to lay the factual foundations for a proposed amendment to the Procurement Law,
Your Honor, because the petitioner had already testified that he was offered a P200 Million bribe, so if
he was offered a P200 Million bribe it is possible that other government officials who had something to
do with the approval of the contract would be offered the same amount of bribes.

CHIEF JUSTICE PUNO

48
Again, that is speculative.

ATTY. AGABIN

That is why they want to continue with the investigation, Your Honor.

CHIEF JUSTICE PUNO

How about the third question, whether the President said to go ahead and approve the project after
being told about the alleged bribe. How critical is that to the lawmaking function of the Senate? And the
question is may they craft a Bill a remedial law without forcing petitioner Neri to answer this question?

ATTY. AGABIN

Well, they can craft it, Your Honor, based on mere speculation. And sound legislation requires that a
proposed Bill should have some basis in fact.42

The failure of the counsel for respondent Committees to pinpoint the specific need for the information
sought or how the withholding of the information sought will hinder the accomplishment of their
legislative purpose is very evident in the above oral exchanges. Due to the failure of the respondent
Committees to successfully discharge this burden, the presumption in favor of confidentiality of
presidential communication stands. The implication of the said presumption, like any other, is to
dispense with the burden of proof as to whether the disclosure will significantly impair the President’s
performance of her function. Needless to state this is assumed, by virtue of the presumption.

Anent respondent Committees’ bewailing that they would have to "speculate" regarding the questions
covered by the privilege, this does not evince a compelling need for the information sought. Indeed,
Senate Select Committee on Presidential Campaign Activities v. Nixon43 held that while fact-finding by a
legislative committee is undeniably a part of its task, legislative judgments normally depend more on the
predicted consequences of proposed legislative actions and their political acceptability than on a precise
reconstruction of past events. It added that, normally, Congress legislates on the basis of conflicting

49
information provided in its hearings. We cannot subscribe to the respondent Committees’ self-defeating
proposition that without the answers to the three (3) questions objected to as privileged, the
distinguished members of the respondent Committees cannot intelligently craft legislation.

Anent the function to curb graft and corruption, it must be stressed that respondent Committees’ need
for information in the exercise of this function is not as compelling as in instances when the purpose of
the inquiry is legislative in nature. This is because curbing graft and corruption is merely an oversight
function of Congress.44 And if this is the primary objective of respondent Committees in asking the
three (3) questions covered by privilege, it may even contradict their claim that their purpose is
legislative in nature and not oversight. In any event, whether or not investigating graft and corruption is
a legislative or oversight function of Congress, respondent Committees’ investigation cannot transgress
bounds set by the Constitution.

In Bengzon, Jr. v. Senate Blue Ribbon Committee,45 this Court ruled:

The "allocation of constitutional boundaries" is a task that this Court must perform under the
Constitution. Moreover, as held in a recent case, "the political question doctrine neither interposes an
obstacle to judicial determination of the rival claims. The jurisdiction to delimit constitutional
boundaries has been given to this Court. It cannot abdicate that obligation mandated by the 1987
Constitution, although said provision by no means does away with the applicability of the principle in
appropriate cases.46 (Emphasis supplied)

There, the Court further ratiocinated that "the contemplated inquiry by respondent Committee is not
really ‘in aid of legislation’ because it is not related to a purpose within the jurisdiction of Congress,
since the aim of the investigation is to find out whether or not the relatives of the President or Mr.
Ricardo Lopa had violated Section 5 of R.A. No. 3019, the Anti-Graft and Corrupt Practices Act, a matter
that appears more within the province of the courts rather than of the Legislature."47 (Emphasis and
underscoring supplied)

The general thrust and the tenor of the three (3) questions is to trace the alleged bribery to the Office of
the President.48 While it may be a worthy endeavor to investigate the potential culpability of high
government officials, including the President, in a given government transaction, it is simply not a task
for the Senate to perform. The role of the Legislature is to make laws, not to determine anyone’s guilt of
a crime or wrongdoing. Our Constitution has not bestowed upon the Legislature the latter role. Just as
the Judiciary cannot legislate, neither can the Legislature adjudicate or prosecute.

50
Respondent Committees claim that they are conducting an inquiry in aid of legislation and a "search for
truth," which in respondent Committees’ view appears to be equated with the search for persons
responsible for "anomalies" in government contracts.

No matter how noble the intentions of respondent Committees are, they cannot assume the power
reposed upon our prosecutorial bodies and courts. The determination of who is/are liable for a crime or
illegal activity, the investigation of the role played by each official, the determination of who should be
haled to court for prosecution and the task of coming up with conclusions and finding of facts regarding
anomalies, especially the determination of criminal guilt, are not functions of the Senate. Congress is
neither a law enforcement nor a trial agency. Moreover, it bears stressing that no inquiry is an end in
itself; it must be related to, and in furtherance of, a legitimate task of the Congress, i.e. legislation.
Investigations conducted solely to gather incriminatory evidence and "punish" those investigated are
indefensible. There is no Congressional power to expose for the sake of exposure.49 In this regard, the
pronouncement in Barenblatt v. United States50 is instructive, thus:

Broad as it is, the power is not, however, without limitations. Since Congress may only investigate into
the areas in which it may potentially legislate or appropriate, it cannot inquire into matters which are
within the exclusive province of one of the other branches of the government. Lacking the judicial power
given to the Judiciary, it cannot inquire into matters that are exclusively the concern of the Judiciary.
Neither can it supplant the Executive in what exclusively belongs to the Executive. (Emphasis supplied.)

At this juncture, it is important to stress that complaints relating to the NBN Project have already been
filed against President Arroyo and other personalities before the Office of the Ombudsman. Under our
Constitution, it is the Ombudsman who has the duty "to investigate any act or omission of any public
official, employee, office or agency when such act or omission appears to be illegal, unjust, improper, or
inefficient."51 The Office of the Ombudsman is the body properly equipped by the Constitution and our
laws to preliminarily determine whether or not the allegations of anomaly are true and who are liable
therefor. The same holds true for our courts upon which the Constitution reposes the duty to determine
criminal guilt with finality. Indeed, the rules of procedure in the Office of the Ombudsman and the
courts are well-defined and ensure that the constitutionally guaranteed rights of all persons, parties and
witnesses alike, are protected and safeguarded.

Should respondent Committees uncover information related to a possible crime in the course of their
investigation, they have the constitutional duty to refer the matter to the appropriate agency or branch
of government. Thus, the Legislature’s need for information in an investigation of graft and corruption

51
cannot be deemed compelling enough to pierce the confidentiality of information validly covered by
executive privilege. As discussed above, the Legislature can still legislate on graft and corruption even
without the information covered by the three (3) questions subject of the petition.

Corollarily, respondent Committees justify their rejection of petitioner’s claim of executive privilege on
the ground that there is no privilege when the information sought might involve a crime or illegal
activity, despite the absence of an administrative or judicial determination to that effect. Significantly,
however, in Nixon v. Sirica,52 the showing required to overcome the presumption favoring
confidentiality turned, not on the nature of the presidential conduct that the subpoenaed material
might reveal, but, instead, on the nature and appropriateness of the function in the performance of
which the material was sought, and the degree to which the material was necessary to its fulfillment.

Respondent Committees assert that Senate Select Committee on Presidential Campaign Activities v.
Nixon does not apply to the case at bar because, unlike in the said case, no impeachment proceeding
has been initiated at present. The Court is not persuaded. While it is true that no impeachment
proceeding has been initiated, however, complaints relating to the NBN Project have already been filed
against President Arroyo and other personalities before the Office of the Ombudsman. As the Court has
said earlier, the prosecutorial and judicial arms of government are the bodies equipped and mandated
by the Constitution and our laws to determine whether or not the allegations of anomaly in the NBN
Project are true and, if so, who should be prosecuted and penalized for criminal conduct.

Legislative inquiries, unlike court proceedings, are not subject to the exacting standards of evidence
essential to arrive at accurate factual findings to which to apply the law. Hence, Section 10 of the Senate
Rules of Procedure Governing Inquiries in Aid of Legislation provides that "technical rules of evidence
applicable to judicial proceedings which do not affect substantive rights need not be observed by the
Committee." Court rules which prohibit leading, hypothetical, or repetitive questions or questions
calling for a hearsay answer, to name a few, do not apply to a legislative inquiry. Every person, from the
highest public official to the most ordinary citizen, has the right to be presumed innocent until proven
guilty in proper proceedings by a competent court or body.

IV

Respondent Committees Committed Grave

Abuse of Discretion in Issuing the Contempt Order

52
Respondent Committees insist that they did not commit grave abuse of discretion in issuing the
contempt order because (1) there is no legitimate claim of executive privilege; (2) they did not violate
the requirements laid down in Senate v. Ermita; (3) they issued the contempt order in accordance with
their internal Rules; (4) they did not violate the requirement under Article VI, Section 21 of the
Constitution requiring the publication of their Rules; and (5) their issuance of the contempt order is not
arbitrary or precipitate.

We reaffirm our earlier ruling.

The legitimacy of the claim of executive privilege having been fully discussed in the preceding pages, we
see no reason to discuss it once again.

Respondent Committees’ second argument rests on the view that the ruling in Senate v. Ermita,
requiring invitations or subpoenas to contain the "possible needed statute which prompted the need for
the inquiry" along with the "usual indication of the subject of inquiry and the questions relative to and in
furtherance thereof" is not provided for by the Constitution and is merely an obiter dictum.

On the contrary, the Court sees the rationale and necessity of compliance with these requirements.

An unconstrained congressional investigative power, like an unchecked Executive, generates its own
abuses. Consequently, claims that the investigative power of Congress has been abused (or has the
potential for abuse) have been raised many times.53 Constant exposure to congressional subpoena
takes its toll on the ability of the Executive to function effectively. The requirements set forth in Senate
v. Ermita are modest mechanisms that would not unduly limit Congress’ power. The legislative inquiry
must be confined to permissible areas and thus, prevent the "roving commissions" referred to in the
U.S. case, Kilbourn v. Thompson.54 Likewise, witnesses have their constitutional right to due process.
They should be adequately informed what matters are to be covered by the inquiry. It will also allow
them to prepare the pertinent information and documents. To our mind, these requirements concede
too little political costs or burdens on the part of Congress when viewed vis-à-vis the immensity of its
power of inquiry. The logic of these requirements is well articulated in the study conducted by William
P. Marshall,55 to wit:

53
A second concern that might be addressed is that the current system allows committees to continually
investigate the Executive without constraint. One process solution addressing this concern is to require
each investigation be tied to a clearly stated purpose. At present, the charters of some congressional
committees are so broad that virtually any matter involving the Executive can be construed to fall within
their province. Accordingly, investigations can proceed without articulation of specific need or purpose.
A requirement for a more precise charge in order to begin an inquiry should immediately work to limit
the initial scope of the investigation and should also serve to contain the investigation once it is
instituted. Additionally, to the extent clear statements of rules cause legislatures to pause and seriously
consider the constitutional implications of proposed courses of action in other areas, they would serve
that goal in the context of congressional investigations as well.

The key to this reform is in its details. A system that allows a standing committee to simply articulate its
reasons to investigate pro forma does no more than imposes minimal drafting burdens. Rather, the
system must be designed in a manner that imposes actual burdens on the committee to articulate its
need for investigation and allows for meaningful debate about the merits of proceeding with the
investigation. (Emphasis supplied)

Clearly, petitioner’s request to be furnished an advance copy of questions is a reasonable demand that
should have been granted by respondent Committees.

Unfortunately, the Subpoena Ad Testificandum dated November 13, 2007 made no specific reference to
any pending Senate bill. It did not also inform petitioner of the questions to be asked. As it were, the
subpoena merely commanded him to "testify on what he knows relative to the subject matter under
inquiry."

Anent the third argument, respondent Committees contend that their Rules of Procedure Governing
Inquiries in Aid of Legislation (the "Rules") are beyond the reach of this Court. While it is true that this
Court must refrain from reviewing the internal processes of Congress, as a co-equal branch of
government, however, when a constitutional requirement exists, the Court has the duty to look into
Congress’ compliance therewith. We cannot turn a blind eye to possible violations of the Constitution
simply out of courtesy. In this regard, the pronouncement in Arroyo v. De Venecia56 is enlightening,
thus:

"Cases both here and abroad, in varying forms of expression, all deny to the courts the power to inquire
into allegations that, in enacting a law, a House of Congress failed to comply with its own rules, in the

54
absence of showing that there was a violation of a constitutional provision or the rights of private
individuals.

United States v. Ballin, Joseph & Co., the rule was stated thus: ‘The Constitution empowers each House
to determine its rules of proceedings. It may not by its rules ignore constitutional restraints or violate
fundamental rights, and there should be a reasonable relation between the mode or method of
proceeding established by the rule and the result which is sought to be attained."

In the present case, the Court’s exercise of its power of judicial review is warranted because there
appears to be a clear abuse of the power of contempt on the part of respondent Committees. Section 18
of the Rules provides that:

"The Committee, by a vote of majority of all its members, may punish for contempt any witness before it
who disobey any order of the Committee or refuses to be sworn or to testify or to answer proper
questions by the Committee or any of its members." (Emphasis supplied)

In the assailed Decision, we said that there is a cloud of doubt as to the validity of the contempt order
because during the deliberation of the three (3) respondent Committees, only seven (7) Senators were
present. This number could hardly fulfill the majority requirement needed by respondent Committee on
Accountability of Public Officers and Investigations which has a membership of seventeen (17) Senators
and respondent Committee on National Defense and Security which has a membership of eighteen (18)
Senators. With respect to respondent Committee on Trade and Commerce which has a membership of
nine (9) Senators, only three (3) members were present.57 These facts prompted us to quote in the
Decision the exchanges between Senators Alan Peter Cayetano and Aquilino Pimentel, Jr. whereby the
former raised the issue of lack of the required majority to deliberate and vote on the contempt order.

When asked about such voting during the March 4, 2008 hearing before this Court, Senator Francis
Pangilinan stated that any defect in the committee voting had been cured because two-thirds of the
Senators effectively signed for the Senate in plenary session.58

Obviously the deliberation of the respondent Committees that led to the issuance of the contempt order
is flawed. Instead of being submitted to a full debate by all the members of the respondent Committees,
the contempt order was prepared and thereafter presented to the other members for signing. As a
result, the contempt order which was issued on January 30, 2008 was not a faithful representation of

55
the proceedings that took place on said date. Records clearly show that not all of those who signed the
contempt order were present during the January 30, 2008 deliberation when the matter was taken up.

Section 21, Article VI of the Constitution states that:

The Senate or the House of Representatives or any of its respective committees may conduct inquiries in
aid of legislation in accordance with its duly published rules of procedure. The rights of person appearing
in or affected by such inquiries shall be respected. (Emphasis supplied)

All the limitations embodied in the foregoing provision form part of the witness’ settled expectation. If
the limitations are not observed, the witness’ settled expectation is shattered. Here, how could there be
a majority vote when the members in attendance are not enough to arrive at such majority? Petitioner
has the right to expect that he can be cited in contempt only through a majority vote in a proceeding in
which the matter has been fully deliberated upon. There is a greater measure of protection for the
witness when the concerns and objections of the members are fully articulated in such proceeding. We
do not believe that respondent Committees have the discretion to set aside their rules anytime they
wish. This is especially true here where what is involved is the contempt power. It must be stressed that
the Rules are not promulgated for their benefit. More than anybody else, it is the witness who has the
highest stake in the proper observance of the Rules.

Having touched the subject of the Rules, we now proceed to respondent Committees’ fourth argument.
Respondent Committees argue that the Senate does not have to publish its Rules because the same was
published in 1995 and in 2006. Further, they claim that the Senate is a continuing body; thus, it is not
required to republish the Rules, unless the same is repealed or amended.

On the nature of the Senate as a "continuing body," this Court sees fit to issue a clarification. Certainly,
there is no debate that the Senate as an institution is "continuing", as it is not dissolved as an entity with
each national election or change in the composition of its members. However, in the conduct of its day-
to-day business the Senate of each Congress acts separately and independently of the Senate of the
Congress before it. The Rules of the Senate itself confirms this when it states:

RULE XLIV

UNFINISHED BUSINESS

56
SEC. 123. Unfinished business at the end of the session shall be taken up at the next session in the same
status.

All pending matters and proceedings shall terminate upon the expiration of one (1) Congress, but may
be taken by the succeeding Congress as if present for the first time. (emphasis supplied)

Undeniably from the foregoing, all pending matters and proceedings, i.e. unpassed bills and even
legislative investigations, of the Senate of a particular Congress are considered terminated upon the
expiration of that Congress and it is merely optional on the Senate of the succeeding Congress to take
up such unfinished matters, not in the same status, but as if presented for the first time. The logic and
practicality of such a rule is readily apparent considering that the Senate of the succeeding Congress
(which will typically have a different composition as that of the previous Congress) should not be bound
by the acts and deliberations of the Senate of which they had no part. If the Senate is a continuing body
even with respect to the conduct of its business, then pending matters will not be deemed terminated
with the expiration of one Congress but will, as a matter of course, continue into the next Congress with
the same status.

This dichotomy of the continuity of the Senate as an institution and of the opposite nature of the
conduct of its business is reflected in its Rules. The Rules of the Senate (i.e. the Senate’s main rules of
procedure) states:

RULE LI

AMENDMENTS TO, OR REVISIONS OF, THE RULES

SEC. 136. At the start of each session in which the Senators elected in the preceding elections shall begin
their term of office, the President may endorse the Rules to the appropriate committee for amendment
or revision.

The Rules may also be amended by means of a motion which should be presented at least one day
before its consideration, and the vote of the majority of the Senators present in the session shall be
required for its approval. (emphasis supplied)

RULE LII

DATE OF TAKING EFFECT

SEC. 137. These Rules shall take effect on the date of their adoption and shall remain in force until they
are amended or repealed. (emphasis supplied)

57
Section 136 of the Senate Rules quoted above takes into account the new composition of the Senate
after an election and the possibility of the amendment or revision of the Rules at the start of each
session in which the newly elected Senators shall begin their term.

However, it is evident that the Senate has determined that its main rules are intended to be valid from
the date of their adoption until they are amended or repealed. Such language is conspicuously absent
from the Rules. The Rules simply state "(t)hese Rules shall take effect seven (7) days after publication in
two (2) newspapers of general circulation."59 The latter does not explicitly provide for the continued
effectivity of such rules until they are amended or repealed. In view of the difference in the language of
the two sets of Senate rules, it cannot be presumed that the Rules (on legislative inquiries) would
continue into the next Congress. The Senate of the next Congress may easily adopt different rules for its
legislative inquiries which come within the rule on unfinished business.

The language of Section 21, Article VI of the Constitution requiring that the inquiry be conducted in
accordance with the duly published rules of procedure is categorical. It is incumbent upon the Senate to
publish the rules for its legislative inquiries in each Congress or otherwise make the published rules
clearly state that the same shall be effective in subsequent Congresses or until they are amended or
repealed to sufficiently put public on notice.

If it was the intention of the Senate for its present rules on legislative inquiries to be effective even in
the next Congress, it could have easily adopted the same language it had used in its main rules regarding
effectivity.

Lest the Court be misconstrued, it should likewise be stressed that not all orders issued or proceedings
conducted pursuant to the subject Rules are null and void. Only those that result in violation of the
rights of witnesses should be considered null and void, considering that the rationale for the publication
is to protect the rights of witnesses as expressed in Section 21, Article VI of the Constitution. Sans such
violation, orders and proceedings are considered valid and effective.

Respondent Committees’ last argument is that their issuance of the contempt order is not precipitate or
arbitrary. Taking into account the totality of circumstances, we find no merit in their argument.

As we have stressed before, petitioner is not an unwilling witness, and contrary to the assertion of
respondent Committees, petitioner did not assume that they no longer had any other questions for him.
He repeatedly manifested his willingness to attend subsequent hearings and respond to new matters.
His only request was that he be furnished a copy of the new questions in advance to enable him to
adequately prepare as a resource person. He did not attend the November 20, 2007 hearing because
Executive Secretary Ermita requested respondent Committees to dispense with his testimony on the
ground of executive privilege. Note that petitioner is an executive official under the direct control and
supervision of the Chief Executive. Why punish petitioner for contempt when he was merely directed by
his superior? Besides, save for the three (3) questions, he was very cooperative during the September
26, 2007 hearing.

58
On the part of respondent Committees, this Court observes their haste and impatience. Instead of ruling
on Executive Secretary Ermita’s claim of executive privilege, they curtly dismissed it as unsatisfactory
and ordered the arrest of petitioner. They could have informed petitioner of their ruling and given him
time to decide whether to accede or file a motion for reconsideration. After all, he is not just an ordinary
witness; he is a high- ranking official in a co-equal branch of government. He is an alter ego of the
President. The same haste and impatience marked the issuance of the contempt order, despite the
absence of the majority of the members of the respondent Committees, and their subsequent disregard
of petitioner’s motion for reconsideration alleging the pendency of his petition for certiorari before this
Court.

On a concluding note, we are not unmindful of the fact that the Executive and the Legislature are
political branches of government. In a free and democratic society, the interests of these branches
inevitably clash, but each must treat the other with official courtesy and respect. This Court
wholeheartedly concurs with the proposition that it is imperative for the continued health of our
democratic institutions that we preserve the constitutionally mandated checks and balances among the
different branches of government.

In the present case, it is respondent Committees’ contention that their determination on the validity of
executive privilege should be binding on the Executive and the Courts. It is their assertion that their
internal procedures and deliberations cannot be inquired into by this Court supposedly in accordance
with the principle of respect between co-equal branches of government. Interestingly, it is a courtesy
that they appear to be unwilling to extend to the Executive (on the matter of executive privilege) or this
Court (on the matter of judicial review). It moves this Court to wonder: In respondent Committees’
paradigm of checks and balances, what are the checks to the Legislature’s all-encompassing, awesome
power of investigation? It is a power, like any other, that is susceptible to grave abuse.

While this Court finds laudable the respondent Committees’ well-intentioned efforts to ferret out
corruption, even in the highest echelons of government, such lofty intentions do not validate or accord
to Congress powers denied to it by the Constitution and granted instead to the other branches of
government.

There is no question that any story of government malfeasance deserves an inquiry into its veracity. As
respondent Committees contend, this is founded on the constitutional command of transparency and
public accountability. The recent clamor for a "search for truth" by the general public, the religious
community and the academe is an indication of a concerned citizenry, a nation that demands an
accounting of an entrusted power. However, the best venue for this noble undertaking is not in the
political branches of government. The customary partisanship and the absence of generally accepted
rules on evidence are too great an obstacle in arriving at the truth or achieving justice that meets the
test of the constitutional guarantee of due process of law. We believe the people deserve a more
exacting "search for truth" than the process here in question, if that is its objective.

WHEREFORE, respondent Committees’ Motion for Reconsideration dated April 8, 2008 is hereby
DENIED.

59
SO ORDERED.

Valeroso v. People, G.R. No. 164815, 22 February 2008

[G.R. No. 164815. February 22, 2008.]

SR. INSP. JERRY C. VALEROSO, petitioner, vs. THE PEOPLE OF THE PHILIPPINES, respondent.

DECISION

REYES, R.T., J p:

THE law looks forward, never backward. Lex prospicit, non respicit. A new law has a prospective, not
retroactive, effect. 1 However, penal laws that favor a guilty person, who is not a habitual criminal, shall
be given retroactive effect. 1-a These are the rule, the exception and exception to the exception on
effectivity of laws. aEAcHI

Ang batas ay tumitingin sa hinaharap, hindi sa nakaraan. Gayunpaman, ang parusa ng bagong batas ay
iiral kung ito ay pabor sa taong nagkasala na hindi pusakal na kriminal.

We apply the exception rather than the rule in this petition for review on certiorari of the decision of the
Court of Appeals (CA), affirming with modification that of the Regional Trial Court (RTC) in Quezon City,
finding petitioner liable for illegal possession of a firearm. Cdpr

The Facts

On July 10, 1996, at around 9:30 a.m., SPO2 Antonio M. Disuanco of the Criminal Investigation Division,
Central Police District Command, received a dispatch order 2 from the desk officer. 3 The order directed
him and three (3) other policemen to serve a warrant of arrest 4 issued by Judge Ignacio Salvador
against petitioner Sr. Insp. Jerry C. Valeroso in a case for kidnapping with ransom. 5

After a briefing, the team conducted the necessary surveillance on petitioner, checking his hideouts in
Cavite, Caloocan, and Bulacan. 6 Eventually, the team proceeded to the Integrated National Police (INP)
Central Station at Culiat, Quezon City, where they saw petitioner as he was about to board a tricycle. 7
SPO2 Disuanco and his team approached petitioner. 8 They put him under arrest, informed him of his
constitutional rights, and bodily searched him. 9 Found tucked in his waist 10 was a Charter Arms,
bearing Serial Number 52315 11 with five (5) live ammunition. 12 IaAHCE

Petitioner was then brought to the police station for questioning. 13

A verification of the subject firearm at the Firearms and Explosives Division at Camp Crame revealed that
it was not issued to petitioner but to a certain Raul Palencia Salvatierra of Sampaloc, Manila. 14 Epifanio
Deriquito, the records verifier, presented a certification 15 to that effect signed by Edwin C. Roque, chief
records officer of the Firearms and Explosive Division. 16

60
Petitioner was then charged with illegal possession of firearm and ammunition under Presidential
Decree (P.D.) No. 1866, 17 as amended. The Information read:

That on or about the 10th day of July, 1996, in Quezon City, Philippines, the said accused without any
authority of law, did then and there willfully, unlawfully and knowingly have in his/her possession and
under his/her custody and control CaSAcH

One (1) cal. 38 "Charter Arms" revolver bearing Serial No. 52315 with five (5) live ammo.

without first having secured the necessary license/permit issued by the proper authorities.

CONTRARY TO LAW.

Quezon City, Philippines, July 15, 1996.

(Sgd.)

GLORIA VICTORIA C. YAP

Assistant City Prosecutor 18

With the assistance of his counsel de parte, Atty. Oscar Pagulayan, petitioner pleaded not guilty when
arraigned on October 9, 1996. 19 Trial on the merits ensued. EISCaD

SPO2 Disuanco and Deriquito testified for the prosecution in the manner stated above.

Upon the other hand, the defense version was supplied by the combined testimonies of petitioner Sr.
Insp. Jerry C. Valeroso, SPO3 Agustin R. Timbol, Jr. and Adrian Yuson.

Petitioner recounted that on July 10, 1996, he was fast asleep in the boarding house of his children
located at Sagana Homes, Barangay New Era, Quezon City. 20 He was roused from his slumber when
four (4) heavily armed men in civilian clothes bolted the room. 21 They trained their guns at him 22 and
pulled him out of the room. They then tied his hands and placed him near the faucet. 23 The raiding
team went back inside and searched and ransacked the room. 24 SPO2 Disuanco stood guard outside
with him. 25 Moments later, an operative came out of the room and exclaimed, "Hoy, may nakuha
akong baril sa loob!" 26

Petitioner was told by SPO2 Disuanco that "we are authorized to shoot you because there's a shoot to
kill order against you, so if you are planning do so something, do it right now." 27 He was also told that
there was a standing warrant for his arrest. 28 However, he was not shown any proof when he asked for
it. 29 Neither was the raiding group armed with a valid search warrant. 30

According to petitioner, the search done in the boarding house was illegal. The gun seized from him was
duly licensed and covered by necessary permits. He was, however, unable to present the documentation
relative to the firearm because it was confiscated by the police. Petitioner further lamented that when
he was incarcerated, he was not allowed to engage the services of a counsel. Neither was he allowed to
see or talk to his family. 31 AEITDH

61
Petitioner contended that the police had an axe to grind against him. While still with the Narcotics
Command, he turned down a request of Col. Romulo Sales to white-wash a drug-related investigation
involving friends of the said police officer. Col. Sales was likewise subject of a complaint filed with the
Ombudsman by his wife. Col. Sales was later on appointed as the head of the unit that conducted the
search in his boarding house. 32

SPO3 Timbol, Jr. of the Narcotics Command testified that he issued to petitioner a Memorandum
Receipt dated July 1, 1993 33 covering the subject firearm and its ammunition. This was upon the verbal
instruction of Col. Angelito Moreno. SPO3 Timbol identified his signature 34 on the said receipt. 35

Adrian Yuson, an occupant of the room adjacent to where petitioner was arrested, testified that on July
10, 1996, two (2) policemen suddenly entered his room as he was preparing for school. 36 They grabbed
his shoulder and led him out. 37 During all those times, a gun was poked at him. 38 He was asked where
petitioner was staying. Fearing for his life, he pointed to petitioner's room. 39 DIECTc

Four (4) policemen then entered the room. 40 He witnessed how they pointed a gun at petitioner, who
was clad only in his underwear. 41 He also witnessed how they forcibly brought petitioner out of his
room. 42 While a policeman remained near the faucet to guard petitioner, three (3) others went back
inside the room. 43 They began searching the whole place. They forcibly opened his locker, 44 which
yielded the subject firearm. 45

RTC and CA Dispositions

On May 6, 1998, the trial court found petitioner guilty as charged, disposing as follows:

WHEREFORE, the Court hereby finds the accused guilty beyond reasonable doubt of Violation of Section
1 of Presidential Decree No. 1866 as amended by Republic Act No. 8294 and hereby sentences him to
suffer the penalty of prision correccional in its maximum period or from 4 years, 2 months and 1 day as
minimum to 6 years as maximum and to pay the fine in the amount of Fifteen Thousand Pesos
(P15,000.00). ICAcTa

The gun subject of this case is hereby ordered confiscated in favor of the government. Let the same be
put in trust in the hands of the Chief of the PNP.

SO ORDERED. 46

Petitioner moved to reconsider 47 but his motion was denied on August 27, 1998. 48 He appealed to the
CA.

On May 4, 2004, the appellate court affirmed with modification the RTC disposition. The fallo of the CA
decision reads:

Verily, the penalty imposed by the trial court upon the accused-appellant is modified to 4 years and 2
months as minimum up to 6 years as maximum.

62
WHEREFORE, with the foregoing MODIFICATION as to the penalty, the decision appealed from is hereby
AFFIRMED in all other respects. cSTDIC

SO ORDERED. 49

His motion for reconsideration 50 having been denied through a Resolution dated August 3, 2004, 51
petitioner resorted to the present petition under Rule 45.

Issues

Petitioner raises the following issues for Our consideration:

I. THE HONORABLE COURT OF APPEALS COMMITTED SERIOUS ERRORS OF LAW IN AFFIRMING THE
CONVICTION OF PETITIONER DESPITE THE ABSENCE OF PROOF BEYOND REASONABLE DOUBT.

II. THE HONORABLE COURT OF APPEALS COMMITTED SERIOUS ERRORS OF FACT AND LAW IN
SUSTAINING THE LEGALITY OF THE SEARCH AND THE VALIDITY AND ADMISSIBILITY OF THE EVIDENCE
OBTAINED THEREFROM DESPITE THE OVERWHELMING PROOF THAT THE SAME IS THE FRUIT OF THE
POISONOUS TREE. cCAIaD

III. THE HONORABLE COURT OF APPEALS COMMITTED SERIOUS ERRORS OF LAW IN NOT
UPHOLDING THE REGULARITY AND VALIDITY SURROUNDING THE ISSUANCE OF THE MEMORANDUM
RECEIPTS (SIC) IN FAVOR OF PETITIONER WHICH PROVES HIS INNOCENCE OF THE CRIME CHARGE (SIC).
52 (Underscoring supplied)

Our Ruling

In illegal possession of firearm and ammunition, the prosecution has the burden of proving the twin
elements of (1) the existence of the subject firearm and ammunition, and (2) the fact that the accused
who possessed or owned the same does not have the corresponding license for it. 53 DaESIC

The prosecution was able to discharge its burden.

The existence of the subject firearm and its ammunition was established through the testimony of SPO2
Disuanco. 54 Defense witness Yuson also identified the firearm. 55 Its existence was likewise admitted
by no less than petitioner himself. 56

As for petitioner's lack of authority to possess the firearm, Deriquito testified that a verification of the
Charter Arms Caliber .38 bearing Serial No. 52315 with the Firearms and Explosives Division at Camp
Crame revealed that the seized pistol was not issued to petitioner. It was registered in the name of a
certain Raul Palencia Salvatierra of Sampaloc, Manila. 57 As proof, Deriquito presented a certification
signed by Roque, the chief records officer of the same office. 58

The Court on several occasions ruled that either the testimony of a representative of, or a certification
from, the Philippine National Police (PNP) Firearms and Explosive Office attesting that a person is not a

63
licensee of any firearm would suffice to prove beyond reasonable doubt the second element of
possession of illegal firearms. 59 The prosecution more than complied when it presented both. DTEAHI

The certification is outside the scope

of the hearsay rule.

The general rule is that a witness can testify only to those facts which he knows of his personal
knowledge; that is, which are derived from his own perception. 60 Otherwise, the testimony is
objectionable for being hearsay. 61

On this score, the certification from the Firearms and Explosives Division is an exception to the hearsay
rule by virtue of Rule 130, Section 44 of the Rules of Court which provides:

Sec. 44. Entries in official records. — Entries in official records made in the performance of his official
duty by a public officer of the Philippines, or by a person in the performance of a duty specifically
enjoined by law, are prima facie evidence of the facts therein stated. CSDcTH

It may be true that the contents of said certification are only prima facie evidence of the facts stated
there. However, the failure of petitioner to present controverting evidence makes the presumption
unrebutted. Thus, the presumption stands.

Petitioner, however, raises several points which he says entitles him to no less than an acquittal.

The assessment of credibility of

witnesses lies with the trial court.

First, petitioner says that the seizure of the subject firearm was invalid. The search was conducted after
his arrest and after he was taken out of the room he was occupying. 62

This contention deserves scant consideration.

Petitioner's version of the manner and place of his arrest goes into the factual findings made by the trial
court and its calibration of the credibility of witnesses. However, as aptly put by Justice Ynares-Santiago
in People v. Rivera: 63

. . . the manner of assigning values to declarations of witnesses on the witness stand is best and most
competently performed by the trial judge who had the unmatched opportunity to observe the witnesses
and assess their credibility by the various indicia available but not reflected on record. The demeanor of
the person on the stand can draw the line between fact and fancy or evince if the witness is telling the
truth or lying through his teeth. We have consistently ruled that when the question arises as to which of
the conflicting versions of the prosecution and the defense is worthy of belief, the assessment of the
trial courts are generally viewed as correct and entitled to great weight. Furthermore, in an appeal,
where the culpability or innocence of the accused depends on the issue of credibility of witnesses and

64
the veracity of their testimonies, findings of the trial court are given the highest degree of respect if not
finality. 64 (Underscoring supplied) caIACE

The trial court found the prosecution version worthy of credence and belief. We find no compelling
reason not to accept its observation on this score.

Worth noting is the fact that petitioner is a ranking police officer who not only claims to be highly
decorated, 65 but have effected a number of successful arrests 66 as well. Common sense would dictate
that he must necessarily be authorized to carry a gun. We thus agree with the Office of the Solicitor
General that framing up petitioner would have been a very risky proposition. Had the arresting officers
really intended to cause the damnation of petitioner by framing him up, they could have easily "planted"
a more incriminating evidence rather than a gun. That would have made their nefarious scheme easier,
assuming that there indeed was one. SIaHDA

The pieces of evidence show that

petitioner is not legally authorized to

possess the subject firearm and its

five (5) ammunition.

Second, petitioner insists that he is legally authorized to possess the subject firearm and its ammunition
on the basis of the Memorandum Receipt issued to him by the PNP Narcotics Command. 67

Although petitioner is correct in his submission that public officers like policemen are accorded
presumption of regularity in the performance of their official duties, 68 it is only a presumption; it may
be overthrown by evidence to the contrary. The prosecution was able to rebut the presumption when it
proved that the issuance to petitioner of the Memorandum Receipt was anything but regular. IDSETA

SPO3 Timbol, Jr. testified that he issued the Memorandum Receipt to petitioner based on the verbal
instruction of his immediate superior, Col. Moreno. 69 However, a reading of Timbol's testimony on
cross-examination 70 would reveal that there was an unusual facility by which said receipt was issued to
petitioner. Its issuance utterly lacked the usual necessary bureaucratic constraints. Clearly, it was issued
to petitioner under questionable circumstances.

Failure to offer an unlicensed

firearm as evidence is not fatal

provided there is competent

testimony as to its existence.

Third, petitioner claims that the subject firearm and ammunition should have been excluded as evidence
because they were not formally offered by the prosecution 71 in violation of Section 34, Rule 132 of the
Rules of Court. 72

65
We note that petitioner contradicted himself when he argued for the validity of the Memorandum
Receipt and, at the same time, for the exclusion in evidence of the subject firearm and its ammunition.
Petitioner's act may result to an absurd situation where the Memorandum Receipt is declared valid,
while the subject firearm and its ammunition which are supposedly covered by the Memorandum
Receipt are excluded as evidence. That would have made the Memorandum Receipt useless. ITAaCc

In any case, petitioner's contention has no leg to stand on.

Contrary to petitioner's claim, the subject firearm 73 and its five (5) live ammunition 74 were offered in
evidence by the prosecution. 75 Even assuming arguendo that they were not offered, petitioner's stance
must still fail. The existence of an unlicensed firearm may be established by testimony, even without its
presentation at trial. In People v. Orehuela, 76 the non-presentation of the pistol did not prevent the
conviction of the accused. DHcEAa

The doctrine was affirmed in the recent case of People v. Malinao. 77

As previously stated, the existence of the subject firearm and its five (5) live ammunition were
established through the testimony of SPO2 Disuanco. 78 Yuson also identified said firearm. 79 Petitioner
even admitted its existence. 80

We hasten to add that there may also be conviction where an unlicensed firearm is presented during
trial but through inadvertence, negligence, or fortuitous event (for example, if it is lost), it is not offered
in evidence, as long as there is competent testimony as to its existence. EacHSA

Penal and civil liabilities

Petitioner was charged with the crime of illegal possession of firearms and ammunition under the first
paragraph of Section 1 of P.D. No. 1866, as amended. It provides that "[t]he penalty of reclusion
temporal in its maximum period to reclusion perpetua shall be imposed upon any person who shall
unlawfully manufacture, deal in, acquire, dispose, or possess any firearm, part of firearm, ammunition
or machinery, tool or instrument used or intended to be used in the manufacture of any firearm or
ammunition." CSTEHI

P.D. No. 1866, as amended, was the governing law at the time petitioner committed the offense on July
10, 1996. However, R.A. No. 8294 amended P.D. No. 1866 on July 6, 1997, 81 during the pendency of the
case with the trial court. The present law now states:

SECTION 1. Unlawful Manufacture, Sale, Acquisition, Disposition or Possession of Firearms or


Ammunition or Instruments Used or Intended to be Used in the Manufacture of Firearms or
Ammunition. — The penalty of prision correccional in its maximum period and a fine of not less than
Fifteen Thousand Pesos (P15,000) shall be imposed upon any person who shall unlawfully manufacture,
deal in, acquire, dispose, or possess any low-powered firearm, such as rimfire handgun, .380 or .32 and
other firearm of similar firepower, part of firearm, ammunition, or machinery, tool or instrument used
or intended to be used in the manufacture of any firearm or ammunition: Provided, That no other crime
was committed. (Underscoring supplied) AHacIS

66
As a general rule, penal laws should not have retroactive application, lest they acquire the character of
an ex post facto law. 82 An exception to this rule, however, is when the law is advantageous to the
accused. According to Mr. Chief Justice Araullo, this is "not as a right" of the offender, "but founded on
the very principles on which the right of the State to punish and the combination of the penalty are
based, and regards it not as an exception based on political considerations, but as a rule founded on
principles of strict justice." 83

Although an additional fine of P15,000.00 is imposed by R.A. No. 8294, the same is still advantageous to
the accused, considering that the imprisonment is lowered to prision correccional in its maximum period
84 from reclusion temporal in its maximum period to reclusion perpetua 85 under P.D. No. 1866.
EcDATH

Applying the Indeterminate Sentence Law, prision correccional maximum which ranges from four (4)
years, two (2) months and one (1) day to six (6) years, is the prescribed penalty and will form the
maximum term of the indeterminate sentence. The minimum term shall be one degree lower, which is
prision correccional in its medium period (two [2] years, four [4] months and one [1] day to four [4]
years and two [2] months). 86 Hence, the penalty imposed by the CA is correct. The penalty of four (4)
years and two (2) months of prision correccional medium, as minimum term, to six (6) years of prision
correccional maximum, as maximum term, is in consonance with the Court's ruling in Gonzales v. Court
of Appeals 87 and Barredo v. Vinarao. 88 TEIHDa

As to the subject firearm and its five (5) live ammunition, their proper disposition should be made under
Article 45 of the Revised Penal Code 89 which provides, among others, that the proceeds and
instruments or tools of the crime shall be confiscated and forfeited in favor of the government.

WHEREFORE, the Decision of the Court of Appeals dated May 4, 2004 is AFFIRMED in full. CDTSEI

SO ORDERED.

Ynares-Santiago, Austria-Martinez, Chico-Nazario and Nachura, JJ., concur.

67
PNB v. Office of the President, G.R. No. 104528, 18 January 1996

[G.R. No. 104528. January 18, 1996.]

PHILIPPINE NATIONAL BANK, petitioner, vs. OFFICE OF THE PRESIDENT, HOUSING AND LAND USE
REGULATORY BOARD (HLURB), ALFONSO MAGLAYA, ANGELINA MAGLAYA P. REYES, JORGE C.
BERNARDINO, CORAZON DE LEON, VICTORIANO ACAYA, FLORENCIA CULTURA, MARIA CAMPOS,
ERNESTO SARMIENTO, SANTIAGO TAMONAN, APOLONIA TADIAQUE, SIMEON DE LEON, NATIVIDAD J.
CRUZ, NATIVIDAD B. LORESCO, FELICIDAD GARCIA, ANA ANITA TAN, LUCAS SERVILLION, JOSE NARAWAL,
represented by their duly authorized Attorney-in-Fact, CORAZON DE LEON AND SPOUSES LEOPOLDO
AND CARMEN SEBASTIAN, respondents. cdasia

Santiago, Jr. Vidad Corpus & Associates for petitioner.

The Solicitor General for respondents.

SYLLABUS

1. ADMINISTRATIVE LAW; OFFICE OF THE PRESIDENT; APPEAL THEREFROM MAY BE TAKEN TO THE
COURT OF APPEALS AND SUPREME COURT MAY TAKE COGNIZANCE THEREOF IN THE INTEREST OF
SPEEDY JUSTICE. — Under Revised Administrative Circular No. 1-95, "appeals from judgments or final
orders of the . . . Office of the President . . . may be taken to the Court of Appeals . . ." However, in order
to hasten the resolution of this case, which was deemed submitted for decision three years ago, the
Court resolved to make an exception to the said Circular in the interest of speedy justice.

2. CIVIL LAW; EFFECT OF LAWS; GENERALLY, NO RETROACTIVE EFFECT — Pursuant to Article 4 of


the Civil Code, "(l)aws shall have no retroactive effect, unless the contrary is provided." cdasia

3. ID.; THE SUBDIVISION AND CONDOMINIUM BUYERS' PROTECTIVE DECREE (P.D. NO. 957); GIVEN
RETROACTIVE APPLICATION. — It is obvious and indubitable that P.D. 957 was intended to cover even
those real estate mortgages, like the one at issue here, executed prior to its enactment, and such intent
(as succinctly captured in the preamble) must be given effect if the laudable purpose of protecting
innocent purchasers is to be achieved. While P.D. 957 did not expressly provide for retroactivity in its
entirety, yet the same can be plainly inferred from the unmistakable intent of the law to protect
innocent lot buyers from scheming subdivision developers. As between these small lot buyers and the
gigantic financial institutions which the developers deal with, it is obvious that the law — as an
instrument of social justice — must favor the weak. Likewise noteworthy are certain provisions of P.D.
957, which themselves constitute strong arguments in favor of the retroactivity of P.D. 957 as a whole.
These are Sections 20, 21 and 23 thereof, which by their very terms have retroactive effect and will
impact upon even those contracts and transactions entered into prior to P.D. 957's enactment

4. ID.; ID.; PRESIDENTIAL DECREE NO. 957; REAL ESTATE MORTGAGE MADE BY THE SUBDIVISION
OWNER IN FAVOR OF A BANK DECLARED NULL AND VOID WHERE RIGHTS OF BUYERS CLASHED WITH

68
THE MORTGAGEE BANK'S RIGHT TO FORECLOSE. — The decision of the Court of Appeals in Breta and
Hamor vs. Lao, et al., penned by then Court of Appeals Associate Justice Jose A.R. Melo, now a respected
member of this Court, is persuasive, the factual circumstances therein being of great similarity to the
antecedent facts of the case at bench. By the foregoing citation, this Court adopts by reference the
foregoing as part of this Decision. The real estate mortgage in the above cited case, although constituted
in 1975 and outside the beneficial aegis of P.D. 957, was struck down by the Court of Appeals which
found in favor of subdivision lot buyers when the rights of the latter clashed with the mortgagee bank's
right to foreclose the property. The Court of Appeals in that case upheld the decision of the trial court
declaring the real estate mortgage as null and void.

5. ID.; ID.; ID.; MORTGAGEE BANK OBLIGED TO ACCEPT PAYMENT OF REMAINING UNPAID
AMORTIZATIONS OF SUBDIVISION LOT BUYERS. — As to the second issue of non-privity, petitioner
avers that, in view of the provisions of Article 1311 of the Civil Code, PNB, being a "total stranger to the
land purchase agreement," cannot be made to take the developer's place. We disagree. P.D. 957 being
applicable, Section 18 of said law obliges petitioner Bank to accept the payment of the remaining unpaid
amortizations tendered by private respondents. Privity of contracts as a defense does not apply in this
case for the law explicitly grants to the buyer the option to pay the installment payment for his lot or
unit directly to the mortgagee (petitioner), which is required to apply such payments to reduce the
corresponding portion of the mortgage indebtedness secured by the particular lot or unit being paid for.
And, as stated earlier, this is without prejudice to petitioner Bank's seeking relief against the subdivision
developer.

6. STATUTORY CONSTRUCTION; STATUTES; EFFECT MUST BE IN ACCORDANCE WITH THE PURPOSE


AND INTENT OF THE LAWMAKERS. — The intent of a statute is the law. If a statute is valid it is to have
effect according to the purpose and intent of the lawmaker. The intent is the vital part, the essence of
the law, and the primary rule of construction is to ascertain and give effect to the intent. The intention
of the legislature in enacting a law is the law itself, and must be enforced when ascertained; although it
may not be consistent with the strict letter of the statute. Courts will not follow the letter of a statute
when it leads away from the true intent and purpose of the legislature and to conclusions inconsistent
with the general purpose of the act. Intent is the spirit which gives life to a legislative enactment. In
construing statutes the proper course is to start out and follow the true intent of the legislature and to
adopt that sense which harmonizes best with the context and promotes in the fullest manner the
apparent policy and objects of the legislature. (Sutherland, in his well-known treatise on Statutory
Construction [quoted with approval by this Court in an old case of consequence, Ongsiako vs. Gamboa]).

7. CONSTITUTIONAL LAW; BILL OF RIGHTS; NON-IMPAIRMENT CLAUSE; CANNOT PREVAIL OVER


POLICE POWER OF THE STATE. — Despite the impairment clause, a contract valid at the time of its
execution may be legally modified or even completely invalidated by a subsequent law. If the law is a
proper exercise of the police power, it will prevail over the contract. Into each contract are read the
provisions of existing law and, always, a reservation of the police power as long as the agreement deals
with a matter affecting the public welfare. Such a contract, it has been held, suffers a congenital
infirmity, and this is its susceptibility to change by the legislature as a postulate of the legal order.

69
RESOLUTION

PANGANIBAN, J p:

May a buyer of a property at a foreclosure sale dispossess prior purchasers on installment of individual
lots therein, or compel them to pay again for the lots which they previously bought from the defaulting
mortgagor-subdivision developer, on the theory that P.D. 957, "The Subdivision and Condominium
Buyers' Protective Decree", is not applicable to the mortgage contract in question, the same having
been executed prior to the enactment of P.D. 957? This is the question confronting the Court in this
Petition challenging the Decision dated March 10, 1992 of the Office of the President of the Philippines
in O.P. Case No. 4249, signed by the Executive Secretary, Franklin M. Drilon, "by authority of the
President."

Private respondents were buyers on installment of subdivision lots from Marikina Village, Inc.
(represented by spouses Antonio and Susana Astudillo). Notwithstanding the land purchase agreements
it executed over said lots, the subdivision developer mortgaged the lots in favor of the petitioner,
Philippine National Bank. Unaware of this mortgage, private respondents duly complied with their
obligations as lot buyers and constructed their houses on the lots in question.

Subsequently, the subdivision developer defaulted and PNB foreclosed on the mortgage. As highest
bidder at the foreclosure sale, the bank became owner of the lots. cdtai

Acting on suits brought by private respondents (which were later consolidated), the HLURB Office of
Appeals, Adjudication and Legal Affairs (OAALA) in a decision rendered on October 28, 1988 ruled that
PNB — without prejudice to seeking relief against Marikina Village, Inc. — may collect from private
respondents only the "remaining amortizations, in accordance with the land purchase agreements they
had previously entered into with" Marikina Village, Inc., and cannot compel private respondents to pay
all over again for the lots they had already bought from said subdivision developer. On May 2, 1989, the
Housing and Land Use Regulatory Board affirmed this decision. On March 10, 1992, the Office of the
President, invoking P.D. 957, likewise concurred with the HLURB. Hence, the present recourse to this
Court.

Under Revised Administrative Circular No. 1-95, "appeals from judgments or final orders of the . . . .
Office of the President . . . may be taken to the Court of Appeals . . . ." However, in order to hasten the
resolution of this case, which was deemed submitted for decision three years ago, the Court resolved to
make an exception to the said Circular in the interest of speedy justice.

Petitioner bank raised the following issues: cdt

1. The Office of the President erred in applying P.D. 957 because said law was enacted only on July
12, 1976, while the subject mortgage was executed on December 18, 1975; and

2. Petitioner Bank is not privy to the contracts between private respondents and mortgagor-
subdivision developer, hence, the Office of the President erred in ordering petitioner Bank to accept
private respondents' remaining amortizations and issue the corresponding titles after payment thereof.

70
Normally, pursuant to Article 4 of the Civil Code, "(l)aws shall have no retroactive effect, unless the
contrary is provided." However, it is obvious and indubitable that P.D. 957 was intended to cover even
those real estate mortgages, like the one at issue here, executed prior to its enactment, and such intent
(as succinctly captured in the preamble quoted below) must be given effect if the laudable purpose of
protecting innocent purchasers is to be achieved: aisadc

"WHEREAS, it is the policy of the State to afford its inhabitants the requirements of decent human
settlement and to provide them with ample opportunities for improving their quality of life;

"WHEREAS, numerous reports reveal that many real estate subdivision owners, developers, operators,
and/or sellers have reneged on their representations and obligations to provide and maintain properly
subdivision roads, drainage, sewerage, water systems, lighting systems, and other similar basic
requirements, thus endangering the health and safety of home and lot buyers;

"WHEREAS, reports of alarming magnitude also show cases of swindling and fraudulent manipulations
perpetrated by unscrupulous subdivision and condominium sellers and operators, such as failure to
deliver titles to the buyers or titles free from liens and encumbrances, and to pay real estate taxes, and
fraudulent sales of the same subdivision lots to different innocent purchasers for value;" 1 (Emphasis
supplied) cdta

While P.D. 957 did not expressly provide for retroactivity in its entirety, yet the same can be plainly
inferred from the unmistakable intent of the law to protect innocent lot buyers from scheming
subdivision developers. As between these small lot buyers and the gigantic financial institutions which
the developers deal with, it is obvious that the law — as an instrument of social justice — must favor the
weak. Indeed, the petitioner Bank had at its disposal vast resources with which it could adequately
protect its loan activities, and therefore is presumed to have conducted the usual "due diligence"
checking and ascertained (whether thru ocular inspection or other modes of investigation) the actual
status, condition, utilization and occupancy of the property offered as collateral. It could not have been
unaware that the property had been built on by small lot buyers. On the other hand, private
respondents obviously were powerless to discover the attempt of the land developer to hypothecate
the property being sold to them. It was precisely in order to deal with this kind of situation that P.D. 957
was enacted, its very essence and intendment being to provide a protective mantle over helpless
citizens who may fall prey to the razzmatazz of what P.D. 957 termed "unscrupulous subdivision and
condominium sellers. "

The intent of the law, as culled from its preamble and from the situation, circumstances and condition it
sought to remedy, must be enforced. Sutherland, in his well-known treatise on Statutory Construction
(quoted with approval by this Court in an old case of consequence, Ongsiako vs. Gamboa 2), says:

"The intent of a statute is the law. If a statute is valid it is to have effect according to the purpose and
intent of the lawmaker. The intent is the vital part, the essence of the law, and the primary rule of
construction is to ascertain and give effect to the intent. The intention of the legislature in enacting a
law is the law itself, and must be enforced when ascertained; although it may not be consistent with the
strict letter of the statute. Courts will not follow the letter of a statute when it leads away from the true

71
intent and purpose of the legislature and to conclusions inconsistent with the general purpose of the
act. Intent is the spirit which gives life to a legislative enactment. In construing statutes, the proper
course is to start out and follow the true intent of the legislature and to adopt that sense which
harmonizes best with the context and promotes in the fullest manner the apparent policy and objects of
the legislature." 3

Truly, this Court cannot allow the injustice that will be wrought by a strictly prospective application of
the law. Little people who have toiled for years through blood and tears would be deprived of their
homes through no fault of their own. As the Solicitor General, in his comment, argues:

"Verily, if P.D. 957 were to exclude from its coverage the aforecited mortgage contract, the vigorous
regulation which P.D. 957 seeks to impose on unconscientious subdivision sellers will be translated into
a feeble exercise of police power just because the iron hand of the State cannot particularly touch
mortgage contracts badged with the fortunate accident of having been constituted prior to the
enactment of P.D. 957. Indeed, it would be illogical in the extreme if P.D. 957 is to be given full force and
effect and yet, the fraudulent practices and manipulations it seeks to curb in the first instance can
nevertheless be liberally perpetrated precisely because P.D. 957 cannot be applied to existing
antecedent mortgage contracts. The legislative intent could not have conceivably permitted a loophole
which all along works to the prejudice of subdivision lot buyers (private respondents)." 4

Likewise noteworthy are certain provisions of P.D. 957, which themselves constitute strong arguments
in favor of the retroactivity of P.D. 957 as a whole. These are Sections 20, 21 and 23 thereof, which by
their very terms have retroactive effect and will impact upon even those contracts and transactions
entered into prior to P.D. 957's enactment: cdtai

"SEC. 20. Time of Completion. — Every owner or developer shall construct and provide the
facilities, improvements, infrastructures and other forms of development, including water supply and
lighting facilities, which are offered and indicated in the approved subdivision or condominium plans,
brochures, prospectus, printed matters, letters or in any form of advertisement, within one year from
the date of the issuance of the license for the subdivision or condominium project or such other period
of time as may be fixed by the Authority.

"SEC. 21. Sales Prior to Decree. — In cases of subdivision lots or condominium units sold or
disposed of prior to the effectivity of this Decree, it shall be incumbent upon the owner or developer of
the subdivision or condominium project to complete compliance with his or its obligations as provided
in the preceding section within two years from the date of this Decree unless otherwise extended by the
Authority or unless an adequate performance bond is filed in accordance with Section 6 hereof.

"Failure of the owner or developer to comply with the obligations under this and the preceding
provisions shall constitute a violation punishable under Section 38 and 39 of this Decree. cdt

"SEC. 23. Non-Forfeiture of Payments. — No installment payment made by a buyer in a


subdivision or condominium project for the lot or unit he contracted to buy shall be forfeited in favor of
the owner or developer when the buyer, after due notice to the owner or developer, desists from

72
further payment due to the failure of the owner or developer to develop the subdivision or
condominium project according to the approved plans and within the time limit for complying with the
same. Such buyer may, at his option, be reimbursed the total amount paid including amortization
interests but excluding delinquency interests, with interest thereon at the legal rate." (Emphasis
supplied)

As for objections about a possible violation of the impairment clause, we find the following statements
of Justice Isagani Cruz enlightening and pertinent to the case at bench:

"Despite the impairment clause, a contract valid at the time of its execution may be legally modified or
even completely invalidated by a subsequent law. If the law is a proper exercise of the police power, it
will prevail over the contract. aisadc

"Into each contract are read the provisions of existing law and, always, a reservation of the police power
as long as the agreement deals with a matter affecting the public welfare. Such a contract, it has been
held, suffers a congenital infirmity, and this is its susceptibility to change by the legislature as a postulate
of the legal order". 5

This Court ruled along similar lines in Juarez vs. Court of Appeals 6:

"The petitioner complains that the retroactive application of the law would violate the impairment
clause. The argument does not impress. The impairment clause is now no longer inviolate; in fact, there
are many who now believe it is an anachronism in the present-day society. It was quite useful before in
protecting the integrity of private agreements from government meddling, but that was when such
agreements did not affect the community in general. They were indeed purely private agreements then.
Any interference with them at that time was really an unwarranted intrusion that could properly struck
down. cdta

"But things are different now. More and more, the interests of the public have become involved in what
are supposed to be still private agreements, which have as a result been removed from the protection of
the impairment clause. These agreements have come within the embrace of the police power, that
obtrusive protector of the public interest. It is a ubiquitous policeman indeed. As long as the contract
affects the public welfare one way or another so as to require the interference of the State, then must
the police power be asserted, and prevail, over the impairment clause."

The decision of the Court of Appeals in Breta and Hamor vs. Lao, et al. 7, penned by then Court of
Appeals Associate Justice Jose A. R. Melo, now a respected member of this Court, is persuasive, the
factual circumstances therein being of great similarity to the antecedent facts of the case at bench:

"Protection must be afforded small homeowners who toil and save if only to purchase on installment a
tiny home lot they can call their own. The consuming dream of every Filipino is to be able to buy a lot,
no matter how small, so that he may somehow build a house. It has, however, been seen of late that
these honest, hard-living individuals are taken advantage of, with the delivery of titles delayed, the
subdivision facilities, including the most essential such as water installations not completed, or worse

73
yet, as in the instant case, after almost completing the payments for the property and after constructing
a house, the buyer is suddenly confronted by the stark reality, contrived or otherwise, in which another
person would now appear to be owner. cdasia

xxx xxx xxx

"We cannot over emphasize the fact that the BANK cannot barefacedly argue that simply because the
title or titles offered as security were clean of any encumbrance or lien, that it was thereby relieved of
taking any other step to verify the over-reaching implications should the subdivision be auctioned on
foreclosure. The BANK could not have closed its eyes that it was dealing over a subdivision where there
were already houses constructed. Did it not enter the mind of the responsible officers of the BANK that
there may even be subdivision residents who have almost completed their installment payments?" (id.,
pp. 7 & 9)

By the foregoing citation, this Court thus adopts by reference the foregoing as part of this Decision.

The real estate mortgage in the above cited case, although constituted in 1975 and outside the
beneficial aegis of P.D. 957, was struck down by the Court of Appeals which found in favor of subdivision
lot buyers when the rights of the latter clashed with the mortgagee bank's right to foreclose the
property. The Court of Appeals in that case upheld the decision of the trial court declaring the real
estate mortgage as null and void. cdtai

As to the second issue of non-privity, petitioner avers that, in view of the provisions of Article 1311 of
the Civil Code, PNB, being a "total stranger to the land purchase agreement," cannot be made to take
the developer's place.

We disagree. P.D. 957 being applicable, Section 18 of said law obliges petitioner Bank to accept the
payment of the remaining unpaid amortizations tendered by private respondents.

"SEC. 18. Mortgages. — No mortgage on any unit or lot shall be made by the owner or developer
without prior written approval of the Authority. Such approval shall not be granted unless it is shown
that the proceeds of the mortgage loan shall be used for the development of the condominium or
subdivision project and effective measures have been provided to ensure such utilization. The loan value
of each lot or unit covered by the mortgage shall be determined and the buyer thereof, if any, shall be
notified before the release of the loan. The buyer may, at his option, pay his installment for the lot or
unit directly to the mortgagee who shall apply the payments to the corresponding mortgage
indebtedness secured by the particular lot or unit being paid for, with a view to enabling said buyer to
obtain title over the lot or unit promptly after full payment thereof ." (Emphasis supplied) cdt

Privity of contracts as a defense does not apply in this case for the law explicitly grants to the buyer the
option to pay the installment payment for his lot or unit directly to the mortgagee (petitioner), which is
required to apply such payments to reduce the corresponding portion of the mortgage indebtedness

74
secured by the particular lot or unit being paid for. And, as stated earlier, this is without prejudice to
petitioner Bank's seeking relief against the subdivision developer.

Finally, before closing this Resolution, we enjoin petitioner Bank to focus not only on the strictly legal
issues involved in this case but also to take another look at the larger issues including social justice and
the protection of human rights as enshrined in the Constitution; firstly, because legal issues are raised
and decided not in a vacuum but within the context of existing social, economic and political conditions,
law being merely a brick in the up-building of the social edifice; and secondly, petitioner, being THE state
bank, is for all intents and purposes an instrument for the implementation of state policies so cherished
in our fundamental law. These consideration are obviously far more weighty than the winning of any
particular suit or the acquisition of any specific property. Thus, as the country strives to move ahead
towards economic self-sufficiency and to achieve dreams of "NIC-hood" and social well-being for the
majority of our countrymen, we hold that petitioner Bank, the premier bank in the country, which has in
recent years made record earnings and acquired an enviable international stature, with branches and
subsidiaries in key financial centers around the world, should be equally as happy with the disposition of
this case as the private respondents, who were almost deprived and dispossessed of their very homes
purchased through their hard work and with their meager savings.

WHEREFORE, in view of the foregoing considerations, the petition is hereby DENIED, petitioner having
failed to show any REVERSIBLE ERROR or GRAVE ABUSE OF DISCRETION in the assailed decision. No
costs. aisadc

SO ORDERED.

Narvasa, C.J., Davide, Jr., Melo and Francisco, JJ., concur.

75
Commissioner of Internal Revenue v. Philippine Health Care Providers, Inc., G.R. No. 168129, 24 April
2007

[G.R. No. 168129. April 24, 2007.]

COMMISSIONER OF INTERNAL REVENUE, petitioner, vs. PHILIPPINE HEALTH CARE PROVIDERS, INC.,
respondent.

DECISION

SANDOVAL-GUTIERREZ, J p:

For our resolution is the instant Petition for Review on Certiorari under Rule 45 of the 1997 Rules of Civil
Procedure, as amended, seeking to reverse the Decision 1 dated February 18, 2005 and Resolution
dated May 9, 2005 of the Court of Appeals (Fifteenth Division) in CA-G.R. SP No. 76449. DSETcC

The factual antecedents of this case, as culled from the records, are:

The Philippine Health Care Providers, Inc., herein respondent, is a corporation organized and existing
under the laws of the Republic of the Philippines. Pursuant to its Articles of Incorporation, 2 its primary
purpose is "To establish, maintain, conduct and operate a prepaid group practice health care delivery
system or a health maintenance organization to take care of the sick and disabled persons enrolled in
the health care plan and to provide for the administrative, legal, and financial responsibilities of the
organization."

On July 25, 1987, President Corazon C. Aquino issued Executive Order (E.O.) No. 273, amending the
National Internal Revenue Code of 1977 (Presidential Decree No. 1158) by imposing Value-Added Tax
(VAT) on the sale of goods and services. This E.O. took effect on January 1, 1988. DAETcC

Before the effectivity of E.O. No. 273, or on December 10, 1987, respondent wrote the Commissioner of
Internal Revenue (CIR), petitioner, inquiring whether the services it provides to the participants in its
health care program are exempt from the payment of the VAT.

On June 8, 1988, petitioner CIR, through the VAT Review Committee of the Bureau of Internal Revenue
(BIR), issued VAT Ruling No. 231-88 stating that respondent, as a provider of medical services, is exempt
from the VAT coverage. This Ruling was subsequently confirmed by Regional Director Osmundo G. Umali
of Revenue Region No. 8 in a letter dated April 22, 1994.

Meanwhile, on January 1, 1996, Republic Act (R.A.) No. 7716 (Expanded VAT or E-VAT Law) took effect,
amending further the National Internal Revenue Code of 1977. Then on January 1, 1998, R.A. No. 8424
(National Internal Revenue Code of 1997) became effective. This new Tax Code substantially adopted
and reproduced the provisions of E.O. No. 273 on VAT and R.A. No. 7716 on E-VAT. aTEADI

In the interim, on October 1, 1999, the BIR sent respondent a Preliminary Assessment Notice for
deficiency in its payment of the VAT and documentary stamp taxes (DST) for taxable years 1996 and
1997.

76
On October 20, 1999, respondent filed a protest with the BIR.

On January 27, 2000, petitioner CIR sent respondent a letter demanding payment of "deficiency VAT" in
the amount of P100,505,030.26 and DST in the amount of P124,196,610.92, or a total of
P224,702,641.18 for taxable years 1996 and 1997. Attached to the demand letter were four (4)
assessment notices. SIEHcA

On February 23, 2000, respondent filed another protest questioning the assessment notices.

Petitioner CIR did not take any action on respondent's protests. Hence, on September 21, 2000,
respondent filed with the Court of Tax Appeals (CTA) a petition for review, docketed as CTA Case No.
6166.

On April 5, 2002, the CTA rendered its Decision, the dispositive portion of which reads:

WHEREFORE, in view of the foregoing, the instant Petition for Review is PARTIALLY GRANTED. Petitioner
is hereby ORDERED TO PAY the deficiency VAT amounting to P22,054,831.75 inclusive of 25% surcharge
plus 20% interest from January 20, 1997 until fully paid for the 1996 VAT deficiency and P31,094,163.87
inclusive of 25% surcharge plus 20% interest from January 20, 1998 until paid for the 1997 VAT
deficiency. Accordingly, VAT Ruling No. 231-88 is declared void and without force and effect. The 1996
and 1997 deficiency DST assessment against petitioner is hereby CANCELLED AND SET ASIDE.
Respondent is ORDERED to DESIST from collecting the said DST deficiency tax. cAEDTa

SO ORDERED.

Respondent filed a motion for partial reconsideration of the above judgment concerning its liability to
pay the deficiency VAT.

In its Resolution 3 dated March 23, 2003, the CTA granted respondent's motion, thus:

WHEREFORE, in view of the foregoing, the instant Motion for Partial Reconsideration is GRANTED.
Accordingly, the VAT assessment issued by herein respondent against petitioner for the taxable years
1996 and 1997 is hereby WITHDRAWN and SET ASIDE. cDSAEI

SO ORDERED.

The CTA held:

Moreover, this court adheres to its conclusion that petitioner is a service contractor subject to VAT since
it does not actually render medical service but merely acts as a conduit between the members and
petitioner's accredited and recognized hospitals and clinics. CDAHaE

However, after a careful review of the facts of the case as well as the Law and jurisprudence applicable,
this court resolves to grant petitioner's "Motion for Partial Reconsideration." We are in accord with the
view of petitioner that it is entitled to the benefit of non-retroactivity of rulings guaranteed under

77
Section 246 of the Tax Code, in the absence of showing of bad faith on its part. Section 246 of the Tax
Code provides:

Sec. 246. Non-Retroactivity of Rulings. — Any revocation, modification or reversal of any of the
rules and regulations promulgated in accordance with the preceding Sections or any of the rulings or
circulars promulgated by the Commissioner shall not be given retroactive application if the revocation,
modification or reversal will be prejudicial to the taxpayers, . . . .

Clearly, undue prejudice will be caused to petitioner if the revocation of VAT Ruling No. 231-88 will be
retroactively applied to its case. VAT Ruling No. 231-88 issued by no less than the respondent itself has
confirmed petitioner's entitlement to VAT exemption under Section 103 of the Tax Code. In saying so,
respondent has actually broadened the scope of "medical services" to include the case of the petitioner.
This VAT ruling was even confirmed subsequently by Regional Director Ormundo G. Umali in his letter
dated April 22, 1994 (Exhibit M). Exhibit P, which served as basis for the issuance of the said VAT ruling
in favor of the petitioner sufficiently described the business of petitioner and there is no way BIR could
be misled by the said representation as to the real nature of petitioner's business. Such being the case,
this court is convinced that petitioner's reliance on the said ruling is premised on good faith. The facts of
the case do not show that petitioner deliberately committed mistakes or omitted material facts when it
obtained the said ruling from the Bureau of Internal Revenue. Thus, in the absence of such proof, this
court upholds the application of Section 246 of the Tax Code. Consequently, the pronouncement made
by the BIR in VAT Ruling No. 231-88 as to the VAT exemption of petitioner should be upheld. cDSAEI

Petitioner seasonably filed with the Court of Appeals a petition for review, docketed as CA-G.R. SP No.
76449.

In its Decision dated February 18, 2005, the Court of Appeals affirmed the CTA Resolution. ScAaHE

Petitioner CIR filed a motion for reconsideration, but it was denied by the appellate court in its
Resolution 4 dated May 9, 2005.

Hence, the instant petition for review on certiorari raising these two issues: (1) whether respondent's
services are subject to VAT; and (2) whether VAT Ruling No. 231-88 exempting respondent from
payment of VAT has retroactive application.

On the first issue, respondent is contesting petitioner's assessment of its VAT liabilities for taxable years
1996 and 1997.

Section 102 5 of the National Internal Revenue Code of 1977, as amended by E.O. No. 273 (VAT Law)
and R.A. No. 7716 (E-VAT Law), provides:

SEC. 102. Value-added tax on sale of services and use or lease of properties. — (a) Rate and base
of tax. — There shall be levied, assessed and collected, a value-added tax equivalent to 10% of gross
receipts derived from the sale or exchange of services, including the use or lease of properties. AcHEaS

78
The phrase "sale or exchange of service" means the performance of all kinds of services in the
Philippines for a fee, remuneration or consideration, including those performed or rendered by
construction and service contractors. . . .

Section 103 6 of the same Code specifies the exempt transactions from the provision of Section 102,
thus:

SEC. 103. Exempt Transactions. — The following shall be exempt from the value-added tax:

xxx xxx xxx

(l) Medical, dental, hospital and veterinary services except those rendered by professionals

xxx xxx xxx

The import of the above provision is plain. It requires no interpretation. It contemplates the exemption
from VAT of taxpayers engaged in the performance of medical, dental, hospital, and veterinary services.
In Commissioner of Internal Revenue v. Seagate Technology (Philippines), 7 we defined an exempt
transaction as one involving goods or services which, by their nature, are specifically listed in and
expressly exempted from the VAT, under the Tax Code, without regard to the tax status of the party in
the transaction. In Commissioner of Internal Revenue v. Toshiba Information Equipment (Phils.) Inc., 8
we reiterated this definition. cCTESa

In its letter to the BIR requesting confirmation of its VAT-exempt status, respondent described its
services as follows:

Under the prepaid group practice health care delivery system adopted by Health Care, individuals
enrolled in Health Care's health care program are entitled to preventive, diagnostic, and corrective
medical services to be dispensed by Health Care's duly licensed physicians, specialists, and other
professional technical staff participating in said group practice health care delivery system established
and operated by Health Care. Such medical services will be dispensed in a hospital or clinic owned,
operated, or accredited by Health Care. To be entitled to receive such medical services from Health
Care, an individual must enroll in Health Care's health care program and pay an annual fee. Enrollment
in Health Care's health care program is on a year-to-year basis and enrollees are issued identification
cards. HSacEI

From the foregoing, the CTA made the following conclusions:

a) Respondent "is not actually rendering medical service but merely acting as a conduit between
the members and their accredited and recognized hospitals and clinics."

b) It merely "provides and arranges for the provision of pre-need health care services to its
members for a fixed prepaid fee for a specified period of time."

c) It then "contracts the services of physicians, medical and dental practitioners, clinics and
hospitals to perform such services to its enrolled members;" and DcIHSa

79
d) Respondent "also enters into contract with clinics, hospitals, medical professionals and then
negotiates with them regarding payment schemes, financing and other procedures in the delivery of
health services."

We note that these factual findings of the CTA were neither modified nor reversed by the Court of
Appeals. It is a doctrine that findings of fact of the CTA, a special court exercising particular expertise on
the subject of tax, are generally regarded as final, binding, and conclusive upon this Court, more so
where these do not conflict with the findings of the Court of Appeals. 9 Perforce, as respondent does
not actually provide medical and/or hospital services, as provided under Section 103 on exempt
transactions, but merely arranges for the same, its services are not VAT-exempt. THcEaS

Relative to the second issue, Section 246 of the 1997 Tax Code, as amended, provides that rulings,
circulars, rules and regulations promulgated by the Commissioner of Internal Revenue have no
retroactive application if to apply them would prejudice the taxpayer. The exceptions to this rule are: (1)
where the taxpayer deliberately misstates or omits material facts from his return or in any document
required of him by the Bureau of Internal Revenue; (2) where the facts subsequently gathered by the
Bureau of Internal Revenue are materially different from the facts on which the ruling is based, or (3)
where the taxpayer acted in bad faith. IcHEaA

We must now determine whether VAT Ruling No. 231-88 exempting respondent from paying its VAT
liabilities has retroactive application.

In its Resolution dated March 23, 2003, the CTA found that there is no showing that respondent
"deliberately committed mistakes or omitted material facts" when it obtained VAT Ruling No. 231-88
from the BIR. The CTA held that respondent's letter which served as the basis for the VAT ruling
"sufficiently described" its business and "there is no way the BIR could be misled by the said
representation as to the real nature" of said business.

In sustaining the CTA, the Court of Appeals found that "the failure of respondent to refer to itself as a
health maintenance organization is not an indication of bad faith or a deliberate attempt to make false
representations." As "the term health maintenance organization did not as yet have any particular
significance for tax purposes," respondent's failure "to include a term that has yet to acquire its present
definition and significance cannot be equated with bad faith."

We agree with both the Tax Court and the Court of Appeals that respondent acted in good faith. In Civil
Service Commission v. Maala, 10 we described good faith as "that state of mind denoting honesty of
intention and freedom from knowledge of circumstances which ought to put the holder upon inquiry; an
honest intention to abstain from taking any unconscientious advantage of another, even through
technicalities of law, together with absence of all information, notice, or benefit or belief of facts which
render transaction unconscientious." aSTAcH

According to the Court of Appeals, respondent's failure to describe itself as a "health maintenance
organization," which is subject to VAT, is not tantamount to bad faith. We note that the term "health
maintenance organization" was first recorded in the Philippine statute books only upon the passage of

80
"The National Health Insurance Act of 1995" (Republic Act No. 7875). Section 4 (o) (3) thereof defines a
health maintenance organization as "an entity that provides, offers, or arranges for coverage of
designated health services needed by plan members for a fixed prepaid premium." Under this law, a
health maintenance organization is one of the classes of a "health care provider."

It is thus apparent that when VAT Ruling No. 231-88 was issued in respondent's favor, the term "health
maintenance organization" was yet unknown or had no significance for taxation purposes. Respondent,
therefore, believed in good faith that it was VAT exempt for the taxable years 1996 and 1997 on the
basis of VAT Ruling No. 231-88. THDIaC

In ABS-CBN Broadcasting Corp. v. Court of Tax Appeals, 11 this Court held that under Section 246 of the
1997 Tax Code, the Commissioner of Internal Revenue is precluded from adopting a position contrary to
one previously taken where injustice would result to the taxpayer. Hence, where an assessment for
deficiency withholding income taxes was made, three years after a new BIR Circular reversed a previous
one upon which the taxpayer had relied upon, such an assessment was prejudicial to the taxpayer. To
rule otherwise, opined the Court, would be contrary to the tenets of good faith, equity, and fair play.
IaDTES

This Court has consistently reaffirmed its ruling in ABS-CBN Broadcasting Corp. in the later cases of
Commissioner of Internal Revenue v. Borroughs, Ltd., 12 Commissioner of Internal Revenue v. Mega
Gen. Mdsg. Corp. 13 Commissioner of Internal Revenue v. Telefunken Semiconductor (Phils.) Inc., 14 and
Commissioner of Internal Revenue v. Court of Appeals. 15 The rule is that the BIR rulings have no
retroactive effect where a grossly unfair deal would result to the prejudice of the taxpayer, as in this
case.

More recently, in Commissioner of Internal Revenue v. Benguet Corporation, 16 wherein the taxpayer
was entitled to tax refunds or credits based on the BIR's own issuances but later was suddenly saddled
with deficiency taxes due to its subsequent ruling changing the category of the taxpayer's transactions
for the purpose of paying its VAT, this Court ruled that applying such ruling retroactively would be
prejudicial to the taxpayer. AaIDHS

WHEREFORE, we DENY the petition and AFFIRM the assailed Decision and Resolution of the Court of
Appeals in CA-G.R. SP No. 76449. No costs. TSDHCc

SO ORDERED.

Puno, C.J., Corona, Azcuna and Garcia, JJ., concur.

81
D.M. Consunji vs. CA, G.R. No. 137873, April 20, 2001

[G.R. No. 137873. April 20, 2001.]

D.M. CONSUNJI, INC., petitioner, vs. COURT OF APPEALS and MARIA J. JUEGO, respondents.

Castillo Laman Tan Pantaleon and San Jose Law Offices for petitioner.

Manuel Y. Fausto for respondent.

SYNOPSIS

Jose Juego was a construction worker of D.M. Consunji, Inc. when he fell to his death from the 14th floor
of Renaisance Tower. He was performing his work as a carpenter at the elevator core of the building
when suddenly the platform on which he was on board fell down to the basement of the core. His
widow filed a complaint for damages against D.M. Consunji, Inc. Consunji raised the defense that the
widow availed of the benefits from the State Insurance Fund. After trial, the Regional Trial Court (RTC)
rendered a decision in favor of the widow. On appeal by D.M. Consunji, the Court of Appeals (CA)
affirmed the decision of the RTC in toto. D.M. Consunji seeks the reversal of the CA decision questioning
the admissibility of the police report as evidence of the negligence of the petitioner, the applicability of
the doctrine of res ipsa loquitur, the presumption of negligence under Article 2180 of the Civil Code, and
that the respondent was not precluded from recovering damages under the Civil Code. EASCDH

The Supreme Court ruled that the police report in this case was inadmissible for the purpose of proving
the truth of the statements contained therein but was admissible insofar as it constituted part of the
testimony of the police officer involved. However, such inadmissibility loses relevance in the face of the
application of res ipsa loquitur. The effect of the doctrine is to warrant the presumption that the mere
fall of the elevator was a result of the person having charge of the instrumentality was negligent.
Petitioner in this case was unable to present evidence to rebut the inference or presumption of
negligence arising from the application of res ipsa loquitur. The Court also ruled that the respondent
was not precluded from recovering damages under the Civil Code. There was no showing that she knew
of the remedies available to her when the claim before the ECC was filed. The case was remanded to the
Regional Trial Court to determine whether the award decreed in its decision was more than that of the
ECC. Should the award be greater, payments already made to the private respondent pursuant to the
ECC shall be deducted therefrom. The decision of the Court of Appeals was affirmed. cCHITA

SYLLABUS

1. REMEDIAL LAW; EVIDENCE; HEARSAY RULE; CONSTRUED. — The Rules of Court provide that a
witness can testify only to those facts which he knows of his personal knowledge, that is, which are
derived from his perception. A witness, therefore, may not testify as what he merely learned from
others either because he was told or read or heard the same. Such testimony is considered hearsay and
may not be received as proof of the truth of what he has learned. This is known as the hearsay rule.
Hearsay is not limited to oral testimony or statements; the general rule that excludes hearsay as
evidence applies to written, as well as oral statements. The theory of the hearsay rule is that the many

82
possible deficiencies, suppressions, sources of error and untrustworthiness, which lie underneath the
bare untested assertion of a witness, may be best brought to light and exposed by the test of cross-
examination. The hearsay rule, therefore, excludes evidence that cannot be tested by cross-
examination. ADSIaT

2. ID.; ID.; ID.; EXCEPTION; ENTRIES IN OFFICIAL RECORDS; REQUISITES. — The Rules of Court allow
several exceptions to the rule, among which are entries in official records. Section 44, Rule 130 provides:
Entries in official records made in the performance of his duty by a public officer of the Philippines, or by
a person in the performance of a duty specially enjoined by law are prima facie evidence of the facts
therein stated. In Africa, et al. vs. Caltex (Phil.), Inc., et al., this Court, citing the work of Chief Justice
Moran, enumerated the requisites for admissibility under the above rule: (a) that the entry was made by
a public officer or by another person specially enjoined by law to do so; (b) that it was made by the
public officer in the performance of his duties, or by such other person in the performance of a duty
specially enjoined by law; and (c) that the public officer or other person had sufficient knowledge of the
facts by him stated, which must have been acquired by him personally or through official information.

3. ID.; ID.; RES IPSA LOQUITUR; DEFINED AND CONSTRUED. — As a rule of evidence, the doctrine
of res ipsa loquitur is peculiar to the law of negligence which recognizes that prima facie negligence may
be established without direct proof and furnishes a substitute for specific proof of negligence. The
concept of res ipsa loquitur has been explained in this wise: While negligence is not ordinarily inferred
or presumed, and while the mere happening of an accident or injury will not generally give rise to an
inference or presumption that it was due to negligence on defendant's part, under the doctrine of res
ipsa loquitur, which means, literally, the thing or transaction speaks for itself, or in one jurisdiction, that
the thing or instrumentality speaks for itself, the facts or circumstances accompanying an injury may be
such as to raise a presumption, or at least permit an inference of negligence on the part of the
defendant, or some other person who is charged with negligence. . . . where it is shown that the thing or
instrumentality which caused the injury complained of was under the control or management of the
defendant, and that the occurrence resulting in the injury was such as in the ordinary course of things
would not happen if those who had its control or management used proper care, there is sufficient
evidence, or, as sometimes stated, reasonable evidence, in the absence of explanation by the
defendant, that the injury arose from or was caused by the defendant's want of care. One of the
theoretical bases for the doctrine is its necessity, i.e., that necessary evidence is absent or not available.
The res ipsa loquitur doctrine is based in part upon the theory that the defendant in charge of the
instrumentality which causes the injury either knows the cause of the accident or has the best
opportunity of ascertaining it and that the plaintiff has no such knowledge, and therefore is compelled
to allege negligence in general terms and to rely upon the proof of the happening of the accident in
order to establish negligence. The inference which the doctrine permits is grounded upon the fact that
the chief evidence of the true cause, whether culpable or innocent, is practically accessible to the
defendant but inaccessible to the injured person. It has been said that the doctrine of res ipsa loquitur
furnishes a bridge by which a plaintiff, without knowledge of the cause, reaches over to defendant who
knows or should know the cause, for any explanation of care exercised by the defendant in respect of
the matter of which the plaintiff complains. The res ipsa loquitur doctrine, another court has said, is a

83
rule of necessity, in that it proceeds on the theory that under the peculiar circumstances in which the
doctrine is applicable, it is within the power of the defendant to show that there was no negligence on
his part, and direct proof of defendant's negligence is beyond plaintiff's power. Accordingly, some courts
add to the three prerequisites for the application of the res ipsa loquitur doctrine the further
requirement that for the res ipsa loquitur doctrine to apply, it must appear that the injured party had no
knowledge or means of knowledge as to the cause of the accident, or that the party to be charged with
negligence has superior knowledge or opportunity for explanation of the accident. DITEAc

4. ID.; ID.; ID.; PROCEDURAL EFFECT OF THE DOCTRINE; APPLICATION IN CASE AT BAR. —
Petitioner does not dispute the existence of the requisites for the application of res ipsa loquitur, but
argues that the presumption or inference that it was negligent did not arise since it "proved that it
exercised due care to avoid the accident which befell respondent's husband". Petitioner apparently
misapprehends the procedural effect of the doctrine. As stated earlier, the defendant's negligence is
presumed or inferred when the plaintiff establishes the requisites for the application of res ipsa loquitur.
Once the plaintiff makes out a prima facie case of all the elements, the burden then shifts to defendant
to explain. The presumption or inference may be rebutted or overcome by other evidence and, under
appropriate circumstances a disputable presumption, such as that of due care or innocence, may
outweigh the inference. It is not for the defendant to explain or prove its defense to prevent the
presumption or inference from arising. Evidence by the defendant of say, due care, comes into play only
after the circumstances for the application of the doctrine has been established. ADSTCa

5. ID.; ID.; AFFIDAVITS; INADMISSIBLE UNDER THE HEARSAY RULE; RATIONALE; EXCEPTION. —
Affidavits are inadmissible as evidence under the hearsay rule, unless the affiant is placed on the witness
stand to testify thereon. The inadmissibility of this sort of evidence is based not only on the lack of
opportunity on the part of the adverse party to cross-examine the affiant, but also on the commonly
known fact that, generally, an affidavit is not prepared by the affiant himself but by another who uses
his own language in writing the affiant's statements which may either be omitted or misunderstood by
the one writing them. Petitioner, therefore, cannot use said statement as proof of its due care any more
than private respondent can use it to prove the cause of her husband's death. Regrettably, petitioner
does not cite any other evidence to rebut the inference or presumption of negligence arising from the
application of res ipsa loquitur, or to establish any defense relating to the incident. CEcaTH

6. LABOR AND SOCIAL LEGISLATION; LABOR RELATIONS; DEATH BENEFITS; RECOVERY OF


DAMAGES FROM WORKMEN'S COMPENSATION ACT AND TO PROSECUTE AN ORDINARY CIVIL ACTION;
CHOICE OF ONLY ONE REMEDY SHOULD BE AVAILABLE FOR THE HEIRS; EXCEPTION. — Addressing the
issue of whether the heirs had a choice of remedies, majority of the Court En Banc, following the rule in
Pacaña vs. Cebu Autobus Company, held in the affirmative. We now come to the query as to whether or
not the injured employee or his heirs in case of death have a right of selection or choice of action
between availing themselves of the worker's right under the Workmen's Compensation Act and suing in
the regular courts under the Civil Code for higher damages (actual, moral and exemplary) from the
employers by virtue of the negligence or fault of the employers or whether they may avail themselves
cumulatively of both actions, i.e., collect the limited compensation under the Workmen's Compensation
Act and sue in addition for damages in the regular courts. In disposing of a similar issue, this Court in

84
Pacaña vs. Cebu Autobus Company, 32 SCRA 442, ruled that an injured worker has a choice of either to
recover from the employer the fixed amounts set by the Workmen's Compensation Act or to prosecute
an ordinary civil action against the tortfeasor for higher damages but he cannot pursue both courses of
action simultaneously. Nevertheless, the Court allowed some of the petitioners in said case to proceed
with their suit under the Civil Code despite having availed of the benefits provided under the Workmen's
Compensation Act. The Court reasoned: With regard to the other petitioners, it was alleged by Philex in
its motion to dismiss dated May 14, 1968 before the court a quo, that the heirs of the deceased
employees, namely Emerito Obra, Larry Villar, Jr., Aurelio Lanuza, Lorenzo Isla and Saturnino submitted
notices and claims for compensation to the Regional Office No. 1 of the then Department of Labor and
all of them have been paid in full as of August 25, 1967, except Saturnino Martinez whose heirs decided
that they be paid in installments . . . . Such allegation was admitted by herein petitioners in their
opposition to the motion to dismiss dated May 27, 1968 . . . in the lower court, but they set up the
defense that the claims were filed under the Workmen's Compensation Act before they learned of the
official report of the committee created to investigate the accident which established the criminal
negligence and violation of law by Philex, and which report was forwarded by the Director of Mines to
then Executive Secretary Rafael Salas in a letter dated October 19, 1967 only . . . . We hold that although
the other petitioners had received the benefits under the Workmen's Compensation Act, such may not
preclude them from bringing an action before the regular court because they became cognizant of the
fact that Philex has been remiss in its contractual obligations with the deceased miners only after
receiving compensation under the Act. Had petitioners been aware of said violation of government rules
and regulations by Philex, and of its negligence, they would not have sought redress under the
Workmen's Compensation Commission which awarded a lesser amount for compensation. The choice of
the first remedy was based on ignorance or a mistake of fact, which nullifies the choice as it was not an
intelligent choice. The case should therefore be remanded to the lower court for further proceedings.
However, should the petitioners be successful in their bid before the lower court, the payments made
under the Workmen's Compensation Act should be deducted from the damages that may be decreed in
their favor. The ruling in Floresca providing the claimant a choice of remedies was reiterated in Ysmael
Maritime Corporation vs. Avelino, Vda. de Severo vs. Feliciano-Go, and Marcopper Mining Corp. vs.
Abeleda. In the last case, the Court again recognized that a claimant who had been paid under the Act
could still sue under the Civil Code. The Court said: In the Robles case, it was held that claims for
damages sustained by workers in the course of their employment could be filed only under the
Workmen's Compensation Law, to the exclusion of all further claims under other laws. In Floresca, this
doctrine was abrogated in favor of the new rule that the claimants may invoke either the Workmen's
Compensation Act or the provisions of the Civil Code, subject to the consequence that the choice of one
remedy will exclude the other and that the acceptance of compensation under the remedy chosen will
preclude a claim for additional benefits under the other remedy. The exception is where a claimant who
has already been paid under the Workmen's Compensation Act may still sue for damages under the Civil
Code on the basis of supervening facts or developments occurring after he opted for the first remedy.
cda

7. ID.; ID.; ID.; ID.; ID.; WAIVER OF REMEDIES THROUGH ELECTION; EFFECT THEREOF. — When a
party having knowledge of the facts makes an election between inconsistent remedies, the election is

85
final and bars any action, suit, or proceeding inconsistent with the elected remedy, in the absence of
fraud by the other party. The first act of election acts as a bar. Equitable in nature, the doctrine of
election of remedies is designed to mitigate possible unfairness to both parties. It rests on the moral
premise that it is fair to hold people responsible for their choices. The purpose of the doctrine is not to
prevent any recourse to any remedy, but to prevent a double redress for a single wrong. The choice of a
party between inconsistent remedies results in a waiver by election. Hence, the rule in Floresca that a
claimant cannot simultaneously pursue recovery under the Labor Code and prosecute an ordinary
course of action under the Civil Code. The claimant, by his choice of one remedy, is deemed to have
waived the other. Waiver is the intentional relinquishment of a known right. [It] is an act of
understanding that presupposes that a party has knowledge of its rights, but chooses not to assert
them. It must be generally shown by the party claiming a waiver that the person against whom the
waiver is asserted had at the time knowledge, actual or constructive, of the existence of the party's
rights or of all material facts upon which they depended. Where one lacks knowledge of a right, there is
no basis upon which waiver of it can rest. Ignorance of a material fact negates waiver, and waiver
cannot be established by a consent given under a mistake or misapprehension of fact. A person makes a
knowing and intelligent waiver when that person knows that a right exists and has adequate knowledge
upon which to make an intelligent decision. Waiver requires a knowledge of the facts basic to the
exercise of the right waived, with an awareness of its consequences. That a waiver is made knowingly
and intelligently must be illustrated on the record or by the evidence. cCaATD

8. ID.; ID.; ID.; ID.; ID.; ID.; WHEN NULLIFIED; APPLICATION IN CASE AT BAR. — That lack of
knowledge of a fact that nullifies the election of a remedy is the basis for the exception in Floresca. It is
in light of the foregoing principles that we address petitioner's contentions. Waiver is a defense, and it
was not incumbent upon private respondent, as plaintiff, to allege in her complaint that she had availed
of benefits from the ECC. It is, thus, erroneous for petitioner to burden private respondent with raising
waiver as an issue. On the contrary, it is the defendant who ought to plead waiver, as petitioner did in
pages 2-3 of its Answer; otherwise, the defense is waived. It is, therefore, perplexing for petitioner to
now contend that the trial court had no jurisdiction over the issue when petitioner itself pleaded waiver
in the proceedings before the trial court. Does the evidence show that private respondent knew of the
facts that led to her husband's death and the rights pertaining to a choice of remedies? It bears stressing
that what negates waiver is lack of knowledge or a mistake of fact. In this case, the "fact" that served as
a basis for nullifying the waiver is the negligence of petitioner's employees, of which private respondent
purportedly learned only after the prosecutor issued a resolution stating that there may be civil liability.
In Floresca, it was the negligence of the mining corporation and its violation of government rules and
regulations. Negligence, or violation of government rules and regulations, for that matter, however, is
not a fact, but a conclusion of law, over which only the courts have the final say. Such a conclusion binds
no one until the courts have decreed so. It appears, therefore, that the principle that ignorance or
mistake of fact nullifies a waiver has been misapplied in Floresca and in the case at bar. HAECID

9. CIVIL LAW; CIVIL CODE; IGNORANCE OF THE LAW EXCUSES NO ONE FROM COMPLIANCE
THEREWITH (ARTICLE 3); LIMITED TO MANDATORY OR PROHIBITORY LAWS. — The application of Article
3 of the Civil Code is limited to mandatory and prohibitory laws. This may be deduced from the language

86
of the provision, which, notwithstanding a person's ignorance, does not excuse his or her compliance
with the laws. The rule in Floresca allowing private respondent a choice of remedies is neither
mandatory nor prohibitory. Accordingly, her ignorance thereof cannot be held against her. TIaCcD

DECISION

KAPUNAN, J p:

At around 1:30 p.m., November 2, 1990, Jose Juego, a construction worker of D.M. Consunji, Inc., fell 14
floors from the Renaissance Tower, Pasig City to his death. THIECD

PO3 Rogelio Villanueva of the Eastern Police District investigated the tragedy and filed a report dated
November 25, 1990, stating that:

. . . . [The] [v]ictim was rushed to [the] Rizal Medical Center in Pasig, Metro Manila where he was
pronounced dead on arrival (DOA) by the attending physician, Dr. Errol de Yzo[,] at around 2:15 p.m. of
the same date.

Investigation disclosed that at the given time, date and place, while victim Jose A. Juego together with
Jessie Jaluag and Delso Destajo [were] performing their work as carpenter[s] at the elevator core of the
14th floor of the Tower D, Renaissance Tower Building on board a [p]latform made of channel beam
(steel) measuring 4.8 meters by 2 meters wide with pinulid plywood flooring and cable wires attached to
its four corners and hooked at the 5 ton chain block, when suddenly, the bolt or pin which was merely
inserted to connect the chain block with the [p]latform, got loose . . . causing the whole [p]latform
assembly and the victim to fall down to the basement of the elevator core, Tower D of the building
under construction thereby crushing the victim to death, save his two (2) companions who luckily
jumped out for safety.

It is thus manifest that Jose A. Juego was crushed to death when the [p]latform he was then on board
and performing work, fell. And the falling of the [p]latform was due to the removal or getting loose of
the pin which was merely inserted to the connecting points of the chain block and [p]latform but
without a safety lock. 1

On May 9, 1991, Jose Juego's widow, Maria, filed in the Regional Trial Court (RTC) of Pasig a complaint
for damages against the deceased's employer, D.M. Consunji, Inc. The employer raised, among other
defenses, the widow's prior availment of the benefits from the State Insurance Fund.

After trial, the RTC rendered a decision in favor of the widow Maria Juego. The dispositive portion of the
RTC decision reads:

WHEREFORE, judgment is hereby rendered ordering defendant to pay plaintiff, as follows:

1. P50,000.00 for the death of Jose A. Juego.

2. P10,000.00 as actual and compensatory damages.

87
3. P464,000.00 for the loss of Jose A. Juego's earning capacity.

4. P100,000.00 as moral damages.

5. P20,000.00 as attorney's fees, plus the costs of suit.

SO ORDERED. 2

On appeal by D.M. Consunji, the Court of Appeals (CA) affirmed the decision of the RTC in toto.

D.M. Consunji now seeks the reversal of the CA decision on the following grounds:

§ THE APPELLATE COURT ERRED IN HOLDING THAT THE POLICE REPORT WAS ADMISSIBLE
EVIDENCE OF THE ALLEGED NEGLIGENCE OF PETITIONER.

§ THE APPELLATE COURT ERRED IN HOLDING THAT THE DOCTRINE OF RES IPSA LOQUITOR [sic] IS
APPLICABLE TO PROVE NEGLIGENCE ON THE PART OF PETITIONER.

§ THE APPELLATE COURT ERRED IN HOLDING THAT PETITIONER IS PRESUMED NEGLIGENT UNDER
ARTICLE 2180 OF THE CIVIL CODE, AND

§ THE APPELLATE COURT ERRED IN HOLDING THAT RESPONDENT IS NOT PRECLUDED FROM
RECOVERING DAMAGES UNDER THE CIVIL CODE. 3

Petitioner maintains that the police report reproduced above is hearsay and, therefore, inadmissible.
The CA ruled otherwise. It held that said report, being an entry in official records, is an exception to the
hearsay rule.

The Rules of Court provide that a witness can testify only to those facts which he knows of his personal
knowledge, that is, which are derived from his perception. 4 A witness, therefore, may not testify as
what he merely learned from others either because he was told or read or heard the same. Such
testimony is considered hearsay and may not be received as proof of the truth of what he has learned. 5
This is known as the hearsay rule. HAICcD

Hearsay is not limited to oral testimony or statements; the general rule that excludes hearsay as
evidence applies to written, as well as oral statements. 6

The theory of the hearsay rule is that the many possible deficiencies, suppressions, sources of error and
untrustworthiness, which lie underneath the bare untested assertion of a witness, may be best brought
to light and exposed by the test of cross-examination. 7 The hearsay rule, therefore, excludes evidence
that cannot be tested by cross-examination. 8

The Rules of Court allow several exceptions to the rule, 9 among which are entries in official records.
Section 44, Rule 130 provides:

88
Entries in official records made in the performance of his duty made in the performance of his duty by a
public officer of the Philippines, or by a person in the performance of a duty specially enjoined by law
are prima facie evidence of the facts therein stated.

In Africa, et al. vs. Caltex (Phil. ), Inc., et al., 10 this Court, citing the work of Chief Justice Moran,
enumerated the requisites for admissibility under the above rule:

(a) that the entry was made by a public officer or by another person specially enjoined by law to do
so;

(b) that it was made by the public officer in the performance of his duties, or by such other person
in the performance of a duty specially enjoined by law; and

(c) that the public officer or other person had sufficient knowledge of the facts by him stated,
which must have been acquired by him personally or through official information.

The CA held that the police report meets all these requisites. Petitioner contends that the last requisite
is not present.

The Court notes that PO3 Villanueva, who signed the report in question, also testified before the trial
court. In Rodriguez vs. Court of Appeals, 11 which involved a Fire Investigation Report, the officer who
signed the fire report also testified before the trial court. This Court held that the report was
inadmissible for the purpose of proving the truth of the statements contained in the report but
admissible insofar as it constitutes part of the testimony of the officer who executed the report.

. . . . Since Major Enriquez himself took the witness stand and was available for cross-examination, the
portions of the report which were of his personal knowledge or which consisted of his perception and
conclusions were not hearsay. The rest of the report, such as the summary of the statements of the
parties based on their sworn statements (which were annexed to the Report) as well as the latter,
having been included in the first purpose of the offer [as part of the testimony of Major Enriquez], may
then be considered as independently relevant statements which were gathered in the course of the
investigation and may thus be admitted as such, but not necessarily to prove the truth thereof. It has
been said that:

"Where regardless of the truth or falsity of a statement, the fact that it has been made is relevant, the
hearsay rule does not apply, but the statement may be shown. Evidence as to the making of such
statement is not secondary but primary, for the statement itself may constitute a fact in issue, or be
circumstantially relevant as to the existence of such a fact."

When Major Enriquez took the witness stand, testified for petitioners on his Report and made himself
available for cross-examination by the adverse party, the Report, insofar as it proved that certain
utterances were made (but not their truth), was effectively removed from the ambit of the
aforementioned Section 44 of Rule 130. Properly understood, this section does away with the testimony
in open court of the officer who made the official record, considers the matter as an exception to the
hearsay rule and makes the entries in said official record admissible in evidence as prima facie evidence

89
of the facts therein stated. The underlying reasons for this exceptionary rule are necessity and
trustworthiness, as explained in Antillon v. Barcelon.

The litigation is unlimited in which testimony by officials is daily needed; the occasions in which the
officials would be summoned from his ordinary duties to declare as a witness are numberless. The public
officers are few in whose daily work something is not done in which testimony is not needed from
official sources. Were there no exception for official statements, hosts of officials would be found
devoting the greater part of their time to attending as witnesses in court or delivering deposition before
an officer. The work of administration of government and the interest of the public having business with
officials would alike suffer in consequence. For these reasons, and for many others, a certain verity is
accorded such documents, which is not extended to private documents. (3 Wigmore on Evidence, Sec.
1631). AEDCHc

The law reposes a particular confidence in public officers that it presumes they will discharge their
several trusts with accuracy and fidelity; and, therefore, whatever acts they do in discharge of their duty
may be given in evidence and shall be taken to be true under such a degree of caution as to the nature
and circumstances of each case may appear to require.

It would have been an entirely different matter if Major Enriquez was not presented to testify on his
report. In that case the applicability of Section 44 of Rule 143 would have been ripe for determination,
and this Court would have agreed with the Court of Appeals that said report was inadmissible since the
aforementioned third requisite was not satisfied. The statements given by the sources of information of
Major Enriquez failed to qualify as "official information," there being no showing that, at the very least,
they were under a duty to give the statements for record.

Similarly, the police report in this case is inadmissible for the purpose of proving the truth of the
statements contained therein but is admissible insofar as; it constitutes part of the testimony of PO3
Villanueva.

In any case, the Court holds that portions of PO3 Villanueva's testimony which were of his personal
knowledge suffice to prove that Jose Juego indeed died as a result of the elevator crash. PO3 Villanueva
had seen Juego's remains at the morgue, 12 making the latter's death beyond dispute. PO3 Villanueva
also conducted an ocular inspection of the premises of the building the day after the incident 13 and
saw the platform for himself. 14 He observed that the platform was crushed 15 and that it was totally
damaged. 16 PO3 Villanueva also required Garcia and Fabro to bring the chain block to the police
headquarters. Upon inspection, he noticed that the chain was detached from the lifting machine,
without any pin or bolt. 17

What petitioner takes particular exception to is PO3 Villanueva's testimony that the cause of the fall of
the platform was the loosening of the bolt from the chain block. It is claimed that such portion of the
testimony is mere opinion. Subject to certain exceptions, 18 the opinion of a witness is generally not
admissible. 19

90
Petitioner's contention, however, loses relevance in the face of the application of res ipsa loquitur by
the CA. The effect of the doctrine is to warrant a presumption or inference that the mere fall of the
elevator was a result of the person having charge of the instrumentality was negligent. As a rule of
evidence, the doctrine of res ipsa loquitur is peculiar to the law of negligence which recognizes that
prima facie negligence may be established without direct proof and furnishes a substitute for specific
proof of negligence. 20

The concept of res ipsa loquitur has been explained in this wise:

While negligence is not ordinarily inferred or presumed, and while the mere happening of an accident or
injury will not generally give rise to an inference or presumption that it was due to negligence on
defendant's part, under the doctrine of res ipsa loquitur, which means, literally, the thing or transaction
speaks for itself, or in one jurisdiction, that the thing or instrumentality speaks for itself, the facts or
circumstances accompanying an injury may be such as to raise a presumption, or at least permit an
inference of negligence on the part of the defendant, or some other person who is charged with
negligence.

. . . where it is shown that the thing or instrumentality which caused the injury complained of was under
the control or management of the defendant, and that the occurrence resulting in the injury was such as
in the ordinary course of things would not happen if those who had its control or management used
proper care, there is sufficient evidence, or, as sometimes stated, reasonable evidence, in the absence
of explanation by the defendant, that the injury arose from or was caused by the defendant's want of
care. 21

One of the theoretical bases for the doctrine is its necessity, i.e., that necessary evidence is absent or
not available. 22

The res ipsa loquitur doctrine is based in part upon the theory that the defendant in charge of the
instrumentality which causes the injury either knows the cause of the accident or has the best
opportunity of ascertaining it and that the plaintiff has no such knowledge, and therefore is compelled
to allege negligence in general terms and to rely upon the proof of the happening of the accident in
order to establish negligence. The inference which the doctrine permits is grounded upon the fact that
the chief evidence of the true cause, whether culpable or innocent, is practically accessible to the
defendant but inaccessible to the injured person.

It has been said that the doctrine of res ipsa loquitur furnishes a bridge by which a plaintiff, without
knowledge of the cause, reaches over to defendant who knows or should know the cause, for any
explanation of care exercised by the defendant in respect of the matter of which the plaintiff complains.
The res ipsa loquitur doctrine, another court has said, is a rule of necessity, in that it proceeds on the
theory that under the peculiar circumstances in which the doctrine is applicable, it is within the power of
the defendant to show that there was no negligence on his part, and direct proof of defendant's
negligence is beyond plaintiff's power. Accordingly, some courts add to the three prerequisites for the
application of the res ipsa loquitur doctrine the further requirement that for the res ipsa loquitur
doctrine to apply, it must appear that the injured party had no knowledge or means of knowledge as to

91
the cause of the accident, or that the party to be charged with negligence has superior knowledge or
opportunity for explanation of the accident. 23

The CA held that all the requisites of res ipsa loquitur are present in the case at bar: SHIETa

There is no dispute that appellee's husband fell down from the 14th floor of a building to the basement
while he was working with appellant's construction project, resulting to his death. The construction site
is within the exclusive control and management of appellant. It has a safety engineer, a project
superintendent, a carpenter leadman and others who are in complete control of the situation therein.
The circumstances of any accident that would occur therein are peculiarly within the knowledge of the
appellant or its employees. On the other hand, the appellee is not in a position to know what caused the
accident. Res ipsa loquitur is a rule of necessity and it applies where evidence is absent or not readily
available, provided the following requisites are present: (1) the accident was of a kind which does not
ordinarily occur unless someone is negligent; (2) the instrumentality or agency which caused the injury
was under the exclusive control of the person charged with negligence; and (3) the injury suffered must
not have been due to any voluntary action or contribution on the part of the person injured. . . . .

No worker is going to fall from the 14th floor of a building to the basement while performing work in a
construction site unless someone is negligent[;] thus, the first requisite for the application of the rule of
res ipsa loquitur is present. As explained earlier, the construction site with all its paraphernalia and
human resources that likely caused the injury is under the exclusive control and management of
appellant[;] thus[,] the second requisite is also present. No contributory negligence was attributed to the
appellee's deceased husband[;] thus[,] the last requisite is also present. All the requisites for the
application of the rule of res ipsa loquitur are present, thus a reasonable presumption or inference of
appellant's negligence arises. . . . 24

Petitioner does not dispute the existence of the requisites for the application of res ipsa loquitur, but
argues that the presumption or inference that it was negligent did not arise since it "proved that it
exercised due care to avoid the accident which befell respondent's husband."

Petitioner apparently misapprehends the procedural effect of the doctrine. As stated earlier, the
defendant's negligence is presumed or inferred 25 when the plaintiff establishes the requisites for the
application of res ipsa loquitur. Once the plaintiff makes out a prima facie case of all the elements, the
burden then shifts to defendant to explain. 26 The presumption or inference may be rebutted or
overcome by other evidence and, under appropriate circumstances a disputable presumption, such as
that of due care or innocence, may outweigh the inference. 27 It is not for the defendant to explain or
prove its defense to prevent the presumption or inference from arising. Evidence by the defendant of
say, due care, comes into play only after the circumstances for the application of the doctrine has been
established.

In any case, petitioner cites the sworn statement of its leadman Ferdinand Fabro executed before the
police investigator as evidence of its due care. According to Fabro's sworn statement, the company
enacted rules and regulations for the safety and security of its workers. Moreover, the leadman and the
bodegero inspect the chain block before allowing its use.

92
It is ironic that petitioner relies on Fabro's sworn statement as proof of its due care but, in arguing that
private respondent failed to prove negligence on the part of petitioner's employees, also assails the
same statement for being hearsay.

Petitioner is correct. Fabro's sworn statement is hearsay and inadmissible. Affidavits are inadmissible as
evidence under the hearsay rule, unless the affiant is placed on the witness stand to testify thereon. 28
The inadmissibility of this sort of evidence is based not only on the lack of opportunity on the part of the
adverse party to cross-examine the affiant, but also on the commonly known fact that, generally, an
affidavit is not prepared by the affiant himself but by another who uses his own language in writing the
affiant's statements which may either be omitted or misunderstood by the one writing them. 29
Petitioner, therefore, cannot use said statement as proof of its due care any more than private
respondent can use it to prove the cause of her husband's death. Regrettably, petitioner does not cite
any other evidence to rebut the inference or presumption of negligence arising from the application of
res ipsa loquitur, or to establish any defense relating to the incident.

Next, petitioner argues that private respondent had previously availed of the death benefits provided
under the Labor Code and is, therefore, precluded from claiming from the deceased's employer
damages under the Civil Code.

Article 173 of the Labor Code states:

ARTICLE 173. Extent of liability. — Unless otherwise provided, the liability of the State Insurance Fund
under this Title shall be exclusive and in place of all other liabilities of the employer to the employee, his
dependents or anyone otherwise entitled to receive damages on behalf of the employee or his
dependents. The payment of compensation under this Title shall not bar the recovery of benefits as
provided for in Section 699 of the Revised Administrative Code, Republic Act Numbered Eleven hundred
sixty-one, as amended, Republic Act Numbered Six hundred ten, as amended, Republic Act Numbered
Forty-eight hundred sixty-four as amended, and other laws whose benefits are administered by the
System or by other agencies of the government.

The precursor of Article 173 of the Labor Code, Section 5 of the Workmen's Compensation Act, provided
that: DcCEHI

SECTION 5. Exclusive right to compensation. — The rights -and remedies granted by this Act to an
employee by reason of a personal injury entitling him to compensation shall exclude all other rights and
remedies accruing to the employee, his personal representatives, dependents or nearest of kin against
the employer under the Civil Code and other laws because of said injury . . . .

Whether Section 5 of the Workmen's Compensation Act allowed recovery under said Act as well as
under the Civil Code used to be the subject of conflicting decisions. The Court finally settled the matter
in Floresca vs. Philex Mining Corporation, 30 which involved a cave-in resulting in the death of the
employees of the Philex Mining Corporation. Alleging that the mining corporation, in violation of
government rules and regulations, failed to take the required precautions for the protection of the
employees, the heirs of the deceased employees filed a complaint against Philex Mining in the Court of

93
First Instance (CFI). Upon motion of Philex Mining, the CFI dismissed the complaint for lack of
jurisdiction. The heirs sought relief from this Court.

Addressing the issue of whether the heirs had a choice of remedies, majority of the Court En Banc, 31
following the rule in Pacaña vs. Cebu Autobus Company, held in the affirmative.

WE now come to the query as to whether or not the injured employee or his heirs in case of death have
a right of selection or choice of action between availing themselves of the worker's right under the
Workmen's Compensation Act and suing in the regular courts under the Civil Code for higher damages
(actual, moral and exemplary) from the employers by virtue of the negligence or fault of the employers
or whether they may avail themselves cumulatively of both actions, i.e., collect the limited
compensation under the Workmen's Compensation Act and sue in addition for damages in the regular
courts.

In disposing of a similar issue, this Court in Pacaña vs. Cebu Autobus Company, 32 SCRA 442, ruled that
an injured worker has a choice of either to recover from the employer the fixed amounts set by the
Workmen's Compensation Act or to prosecute an ordinary civil action against the tort-feasor for higher
damages but he cannot pursue both courses of action simultaneously. [Emphasis supplied.]

Nevertheless, the Court allowed some of the petitioners in said case to proceed with their suit under the
Civil Code despite having availed of the benefits provided under the Workmen's Compensation Act. The
Court reasoned:

With regard to the other petitioners, it was alleged by Philex in its motion to dismiss dated May 14, 1968
before the court a quo, that the heirs of the deceased employees, namely Emerito Obra, Larry Villar, Jr.,
Aurelio Lanuza, Lorenzo Isla and Saturnino submitted notices and claims for compensation to the
Regional Office No. 1 of the then Department of Labor and all of them have been paid in full as of
August 25, 1967, except Saturnino Martinez whose heirs decided that they be paid in installments . . . .
Such allegation was admitted by herein petitioners in their opposition to the motion to dismiss dated
May 27, 1968 . . . in the lower court, but they set up the defense that the claims were filed under the
Workmen's Compensation Act before they learned of the official report of the committee created to
investigate the accident which established the criminal negligence and violation of law by Philex, and
which report was forwarded by the Director of Mines to then Executive Secretary Rafael Salas in a letter
dated October 19, 1967 only . . . .

WE hold that although the other petitioners had received the benefits under the Workmen's
Compensation Act, such may not preclude them from bringing an action before the regular court
because they became cognizant of the fact that Philex has been remiss in its contractual obligations with
the deceased miners only after receiving compensation under the Act. Had petitioners been aware of
said violation of government rules and regulations by Philex, and of its negligence, they would not have
sought redress under the Workmen's Compensation Commission which awarded a lesser amount for
compensation. The choice of the first remedy was based on ignorance or a mistake of fact, which
nullifies the choice as it was not an intelligent choice. The case should therefore be remanded to the
lower court for further proceedings. However, should the petitioners be successful in their bid before

94
the lower court, the payments made under the Workmen's Compensation Act should be deducted from
the damages that may be decreed in their favor. [Emphasis supplied.]

The ruling in Floresca providing the claimant a choice of remedies was reiterated in Ysmael Maritime
Corporation vs. Avelino, 32 Vda. de Severo vs. Feliciano-Go, 33 and Marcopper Mining Corp. vs. Abeleda.
34 In the last case, the Court again recognized that a claimant who had been paid under the Act could
still sue under the Civil Code. The Court said: aIHCSA

In the Robles case, it was held that claims for damages sustained by workers in the course of their
employment could be filed only under the Workmen's Compensation Law, to the exclusion of all further
claims under other laws. In Floresca, this doctrine was abrogated in favor of the new rule that the
claimants may invoke either the Workmen's Compensation Act or the provisions of the Civil Code,
subject to the consequence that the choice of one remedy will exclude the other and that the
acceptance of compensation under the remedy chosen will preclude a claim for additional benefits
under the other remedy. The exception is where a claimant who has already been paid under the
Workmen's Compensation Act may still sue for damages under the Civil Code on the basis of
supervening facts or developments occurring after he opted for the first remedy. (Emphasis supplied.)

Here, the CA held that private respondent's case came under the exception because private respondent
was unaware of petitioner's negligence when she filed her claim for death benefits from the State
Insurance Fund. Private respondent filed the civil complaint for damages after she received a copy of the
police investigation report and the Prosecutor's Memorandum dismissing the criminal complaint against
petitioner's personnel. While stating that there was no negligence attributable to the respondents in the
complaint, the prosecutor nevertheless noted in the Memorandum that, "if at all," the "case is civil in
nature." The CA thus applied the exception in Floresca:

. . . We do not agree that appellee has knowledge of the alleged negligence of appellant as early as
November 25, 1990, the date of the police investigator's report. The appellee merely executed her
sworn statement before the police investigator concerning her personal circumstances, her relation to
the victim, and her knowledge of the accident. She did not file the complaint for "Simple Negligence
Resulting to Homicide" against appellant's employees. It was the investigator who recommended the
filing of said case and his supervisor referred the same to the prosecutor's office. This. is a standard
operating procedure for police investigators which appellee may not have even known. This may explain
why no complainant is mentioned in the preliminary statement of the public prosecutor in her
memorandum dated February 6, 1991, to wit: "Respondent Ferdinand Fabro . . . are being charged by
complaint of "Simple Negligence Resulting to Homicide." It is also possible that the appellee did not
have a chance to appear before the public prosecutor as can be inferred from the following statement in
said memorandum: "Respondents who were notified pursuant to Law waived their rights to present
controverting evidence," thus there was no reason for the public prosecutor to summon the appellee.
Hence, notice of appellant's negligence cannot be imputed on appellee before she applied for death
benefits under ECC or before she received the first payment therefrom. Her using the police
investigation report to support her complaint filed on May 9, 1991 may just be an afterthought after
receiving a copy of the February 6, 1991 Memorandum of the Prosecutor's Office dismissing the criminal

95
complaint for insufficiency of evidence, stating therein that: "The death of the victim is not attributable
to any negligence on the part of the respondents. If at all and as shown by the records this case is civil in
nature." (Emphasis supplied.) Considering the foregoing, We are more inclined to believe appellee's
allegation that she learned about appellant's negligence only after she applied for and received the
benefits under ECC. This is a mistake of fact that will make this case fall under the exception held in the
Floresca ruling. 35

The CA further held that not only was private respondent ignorant of the facts, but of her rights as well:

. . . . Appellee [Maria Juego] testified that she has reached only elementary school for her educational
attainment; that she did not know what damages could be recovered from the death of her husband;
and that she did not know that she may also recover more from the Civil Code than from the ECC. . . . .
36

Petitioner impugns the foregoing rulings. It contends that private respondent "failed to allege in her
complaint that her application and receipt of benefits from the ECC were attended by ignorance or
mistake of fact. Not being an issue submitted during the trial, the trial court had no authority to hear or
adjudicate that issue."

Petitioner also claims that private respondent could not have been ignorant of the facts because as early
as November 28, 1990, private respondent was the complainant in a criminal complaint for "Simple
Negligence Resulting to Homicide" against petitioner's employees. On February 6, 1991, two months
before the filing of the action in the lower court, Prosecutor Lorna Lee issued a resolution finding that,
although there was insufficient evidence against petitioner's employees, the case was "civil in nature."
These purportedly show that prior to her receipt of death benefits from the ECC on January 2, 1991 and
every month thereafter, private respondent also knew of the two choices of remedies available to her
and yet she chose to claim and receive the benefits from the ECC. IcTEaC

When a party having knowledge of the facts makes an election between inconsistent remedies, the
election is final and bars any action, suit, or proceeding inconsistent with the elected remedy, in the
absence of fraud by the other party. The first act of election acts as a bar. 37 Equitable in nature, the
doctrine of election of remedies is designed to mitigate possible unfairness to both parties. It rests on
the moral premise that it is fair to hold people responsible for their choices. The purpose of the doctrine
is not to prevent any recourse to any remedy, but to prevent a double redress for a single wrong. 38

The choice of a party between inconsistent remedies results in a waiver by election. Hence, the rule in
Floresca that a claimant cannot simultaneously pursue recovery under the Labor Code and prosecute an
ordinary course of action under the Civil Code. The claimant, by his choice of one remedy, is deemed to
have waived the other.

Waiver is the intentional relinquishment of a known right. 39

[It] is an act of understanding that presupposes that a party has knowledge of its rights, but chooses not
to assert them. It must be generally shown by the party claiming a waiver that the person against whom

96
the waiver is asserted had at the time knowledge, actual or constructive, of the existence of the party's
rights or of all material facts upon which they depended. Where one lacks knowledge of a right, there is
no basis upon which waiver of it can rest. Ignorance of a material fact negates waiver, and waiver
cannot be established by a consent given under a mistake or misapprehension of fact.

A person makes a knowing and intelligent waiver when that person knows that a right exists and has
adequate knowledge upon which to make an intelligent decision. Waiver requires a knowledge of the
facts basic to the exercise of the right waived, with an awareness of its consequences. That a waiver is
made knowingly and intelligently must be illustrated on the record or by the evidence. 40

That lack of knowledge of a fact that nullifies the election of a remedy is the basis for the exception in
Floresca.

It is in light of the foregoing principles that we address petitioner's contentions.

Waiver is a defense, and it was not incumbent upon private respondent, as plaintiff, to allege in her
complaint that she had availed of benefits from the ECC. It is, thus, erroneous for petitioner to burden
private respondent with raising waiver as an issue. On the contrary, it is the defendant who ought to
plead waiver, as petitioner did in pages 2-3 of its Answer; 41 otherwise, the defense is waived. It is,
therefore, perplexing for petitioner to now contend that the trial court had no jurisdiction over the issue
when petitioner itself pleaded waiver in the proceedings before the trial court.

Does the evidence show that private respondent knew of the facts that led to her husband's death and
the rights pertaining to a choice of remedies?

It bears stressing that what negates waiver is lack of knowledge or a mistake of fact. In this case, the
"fact" that served as a basis for nullifying the waiver is the negligence of petitioner's employees, of
which private respondent purportedly learned only after the prosecutor issued a resolution stating that
there may be civil liability. In Floresca, it was the negligence of the mining corporation and its violation
of government rules and regulations. Negligence, or violation of government rules and regulations, for
that matter, however, is not a fact, but a conclusion of law, over which only the courts have the final
say. Such a conclusion binds no one until the courts have decreed so. It appears, therefore, that the
principle that ignorance or mistake of fact nullifies a waiver has been misapplied in Floresca and in the
case at bar.

In any event, there is no proof that private respondent knew that her husband died in the elevator crash
when on November 15, 1990 she accomplished her application for benefits from the ECC. The police
investigation report is dated November 25, 1990, 10 days after the accomplishment of the form.
Petitioner filed the application in her behalf on November 27, 1990.

There is also no showing that private respondent knew of the remedies available to her when the claim
before the ECC was filed. On the contrary, private respondent testified that she was not aware of her
rights.

97
Petitioner, though, argues that under Article 3 of the Civil Code, ignorance of the law excuses no one
from compliance therewith. As judicial decisions applying or interpreting the laws or the Constitution
form part of the Philippine legal system (Article 8, Civil Code), private respondent cannot claim
ignorance of this Court's ruling in Floresca allowing a choice of remedies. AaSHED

The argument has no merit. The application of Article 3 is limited to mandatory and prohibitory laws. 42
This may be deduced from the language of the provision, which, notwithstanding a person's ignorance,
does not excuse his or her compliance with the laws. The rule in Floresca allowing private respondent a
choice of remedies is neither mandatory nor prohibitory. Accordingly, her ignorance thereof cannot be
held against her.

Finally, the Court modifies the affirmance of the award of damages. The records do not indicate the
total amount private respondent ought to receive from the ECC, although it appears from Exhibit "K" 43
that she received P3,581.85 as initial payment representing the accrued pension from November 1990
to March 1991. Her initial monthly pension, according to the same Exhibit "K," was P596.97 and present
total monthly pension was P716.40. Whether the total amount she will eventually receive from the ECC
is less than the sum of P644,000.00 in total damages awarded by the trial court is subject to speculation,
and the case is remanded to the trial court for such determination. Should the trial court find that its
award is greater than that of the ECC, payments already received by private respondent under the Labor
Code shall be deducted from the trial court's award of damages. Consistent with our ruling in Floresca,
this adjudication aims to prevent double compensation.

WHEREFORE, the case is REMANDED to the Regional Trial Court of Pasig City to determine whether the
award decreed in its decision is more than that of the ECC. Should the award decreed by the trial court
be greater than that awarded by the ECC, payments already made to private respondent pursuant to the
Labor Code shall be deducted therefrom. In all other respects, the Decision of the Court of Appeals is
AFFIRMED.

SO ORDERED.

Davide, Jr., C.J., Puno, Pardo and Ynares-Santiago, JJ., concur.

98
Cui vs. Arellano University 2 SCRA 205

[G.R. No. L-15127. May 30, 1961.]

EMETERIO CUI, plaintiff-appellant, vs. ARELLANO UNIVERSITY, defendant-appellee.

G.A.S. Sipin, Jr. for plaintiff-appellant.

E. Voltaire Garcia, for defendant-appellee.

SYLLABUS

1. CONTRACTS; STUDENTS AND EDUCATIONAL INSTITUTIONS; SCHOLARSHIPS; STIPULATION


WHEREBY STUDENT CANNOT TRANSFER TO ANOTHER SCHOOL WITHOUT REFUNDING SCHOLARSHIP
CASH NULL AND VOID. — The stipulation in a contract, between a student and the school, that the
student's scholarship is good only if he continues in the same school, and that he waives his right to
transfer to another school without refunding the equivalent of his scholarship in cash, is contrary to
public policy and, hence, null and void, because scholarships are awarded in recognition of merit and to
help gifted students in whom society has an established interest or a first lien, and not to keep
outstanding students in school to bolster its prestige and increase its business potential.

DECISION

CONCEPCION, J p:

Appeal by plaintiff Emeterio Cui from a decision of the Court of First Instance of Manila, absolving
defendant Arellano University from plaintiff's complaint, with costs against the plaintiff, and dismissing
defendant's counterclaim, for insufficiency of proof thereon.

In the language of the decision appealed from:

"The essential facts of this case are short and undisputed. As established by the agreement of facts
Exhibit X and by the respective oral and documentary evidence introduced by the parties, it appears
conclusive that plaintiff, before the school year 1948-1949 took up preparatory law course in the
defendant University. After finishing his preparatory law course plaintiff enrolled in the College of Law of
the defendant from the school year 1948-1949. Plaintiff finished his law studies in the defendant
university up to and including the first semester of the fourth year. During all the school years in which
plaintiff was studying law in defendant law college, Francisco R. Capistrano, brother of the mother of
plaintiff, was the dean of the College of Law and legal counsel of the defendant university. Plaintiff
enrolled for the last semester of his law studies in the defendant university but failed to pay his tuition
fees, because his uncle Dean Francisco R. Capistrano having severed his connection with defendant and
having accepted the deanship and chancellorship of the College of Law of Abad Santos University,
plaintiff left the defendant's law college and enrolled for the last semester of his fourth year law in the
college of law of the Abad Santos University graduating from the college of law of the latter university.
Plaintiff, during all the time he was studying law in defendant university was awarded scholarship

99
grants, for scholastic merit, so that his semestral tuition fees were returned to him after the ends of
semesters and when his scholarship grants were awarded to him. The whole amount of tuition fees paid
by plaintiff to defendant and refunded to him by the latter from the first semester up to and including
the first semester of his last year in the college of law or the fourth year, is in total P1,033.87. After
graduating in law from Abad Santos University he applied to take the bar examination. To secure
permission to take the bar he needed the transcripts of his records in defendant Arellano University.
Plaintiff petitioned the latter to issue to him the needed transcripts. The defendant refused until after
he had paid back the P1,033.87 which defendant refunded to him as above stated. As he could not take
the bar examination without those transcripts, plaintiff paid to defendant the said sum under protest.
This is the sum which plaintiff seeks to recover from defendant in this case.

"Before defendant awarded to plaintiff the scholarship grants as above stated, he was made to sign the
following contract, covenant and agreement:

'In consideration of the scholarship granted to me by the University, I hereby waive my right to transfer
to another school without having refunded to the University (defendant) the equivalent of my
scholarship cash.

(Sgd.) Emeterio Cui'."

It is admitted that, on August 16, 1949, the Director of Private Schools issued Memorandum No. 38,
series of 1949, on the subject of "Scholarships", addressed to "All heads of private schools, colleges and
universities", reading:

"1. School catalogs and prospectuses submitted to this Bureau show that some schools offer full or
partial scholarships to deserving students — for excellence in scholarship or for leadership in
extracurricular activities. Such inducements to poor but gifted students should be encouraged. But to
stipulate the condition that such scholarships are good only if the students concerned continue in the
same school nullifies the principle of merit in the award of these scholarships.

"2. When students are given full or partial scholarships, it is understood that such scholarships are
merited and earned. The amount in tuition and other fees corresponding to these scholarships should
not be subsequently charged to the recipient students when they decide to quit school or to transfer to
another institution. Scholarships should not be offered merely to attract and keep students in a school.

"3. Several complaints have actually been received from students who have enjoyed scholarships,
full or partial, to the effect that they could not transfer to other schools since their credentials would not
be released unless they would pay the fees corresponding to the period of the scholarships. Where the
Bureau believes that the right of the student to transfer is being denied on this ground, it reserves the
right to authorize such transfer."

that defendant herein received a copy of this memorandum; that plaintiff asked the Bureau of Private
Schools to pass upon the issue on his right to secure the transcript of his record in defendant University,
without being required to refund the sum of P1,033.87; that the Bureau of Private Schools upheld the

100
position taken by the plaintiff and so advised the defendant; and that, this notwithstanding, the latter
refused to issue said transcript of record, unless said refund were made, and even recommended to said
Bureau that it issue a written order directing the defendant to release said transcript of record, "so that
the case may be presented to the court for judicial action". As above stated, plaintiff was, accordingly,
constrained to pay, and did pay under protest, said sum of P1,033.87, in order that he could take the bar
examinations in 1953. Subsequently, he brought this action for the recovery of said amount, aside from
P2,000 as moral damages, P500 as exemplary damages, P2,000 as attorney's fees, and P500 as expenses
of litigation.

In its answer, defendant reiterated the stand it took vis-a-vis the Bureau of Private Schools, namely, that
the provisions of its contract with plaintiff are valid and binding, and that the memorandum above-
referred to is null and void. It, likewise, set up a counterclaim for P10,000.00 as damages, and P3,000 as
attorney's fees.

The issue in this case is whether the above quoted provision of the contract between plaintiff and the
defendant whereby the former waived his right to transfer to another school without refunding to the
latter the equivalent of his scholarships in cash, is valid or not. The lower court resolved this question in
the affirmative, upon the ground that the aforementioned memorandum of the Director of Private
Schools is not a law; that the provisions thereof are advisory, not mandatory in nature; and that,
although the contractual provision "may be unethical, yet it was more unethical for plaintiff to quit
studying with the defendant without good reasons and simply because he wanted to follow the example
of his uncle". Moreover, defendant maintains in its brief that the aforementioned memorandum of the
Director of Private Schools is null and void because said officer had no authority to issue it, and because
it had been neither approved by the corresponding department head nor published in the official
gazette.

We do not deem it necessary or advisable to consider, as the lower court did, the question whether
plaintiff had sufficient reasons or not to transfer from defendant University to the Abad Santos
University. The nature of the issue before us, and its far reaching effects, transcend personal equations
and demand a determination of the case from a high impersonal plane. Neither do we deem it essential
to pass upon the validity of said Memorandum No. 38, for, regardless of the same, we are of the opinion
that the stipulation in question is contrary to public policy and hence, null and void. The aforesaid
memorandum merely incorporates a sound principle of public policy. As the Director of Private Schools
correctly pointed out in his letter, Exhibit B, to the defendant,

"There is one more point that merits refutation and that is whether or not the contract entered into
between Cui and Arellano University on September 10, 1951 was void as against public policy. In the
case of Zeigler vs. Illinois Trust and Savings Bank, 245 Ill. 180, 19 Ann. Case 127, the court said: 'In
determining a public policy of the state, courts are limited to a consideration of the Constitution, the
judicial decisions, the statutes, and the practice of government officers.' It might take more than a
government bureau or office to lay down or establish a public policy, as alleged in your communication,
but courts consider the practices of government officials as one of the four factors in determining a
public policy of the state. It has been consistently held in America that under the principles relating to

101
the doctrine of public policy, as applied to the law of contracts, courts of justice will not recognize or
uphold a transaction which in its object, operation, or tendency, is calculated to be prejudicial to the
public welfare, to sound morality, or to civic honesty (Ritter vs. Mutual Life Ins. Co., 169 U. S. 139;
Heding vs. Gallagher, 64 L.R.A. 811; Veazy vs. Allen, 173 N.Y. 359). If Arellano University understood
clearly the real essence of scholarships and the motives which prompted this office to issue
Memorandum No. 38, s. 1949, it should have not entered into a contract of waiver with Cui on
September 10, 1951, which is a direct violation of our Memorandum and an open challenge to the
authority of the Director of Private Schools because the contract was repugnant to sound morality and
civic honesty. And finally, in Gabriel vs. Monte de Piedad, Off. Gazette Supp. Dec. 6, 1941, p. 67 we read:
'In order to declare a contract void as against public policy, a court must find that the contract as to
consideration or the thing to be done, contravenes some established interest of society, or is
inconsistent with sound policy and good morals, or tends clearly to undermine the security of individual
rights.' The policy enunciated in Memorandum No. 33, s. 1949 is sound policy. Scholarships are awarded
in recognition of merit not to keep outstanding students in school to bolster its prestige. In the
understanding of that university scholarships award is a business scheme designed to increase the
business potential of an educational institution. Thus conceived it is not only inconsistent with sound
policy but also good morals. But what is morals? Manresa has this definition. It is good customs; those
generally accepted principles of morality which have received some kind of social and practical
confirmation. The practice of awarding scholarships to attract students and keep them in school is not
good customs nor has it received some kind of social and practical confirmation except in some private
institutions as in Arellano University. The University of the Philippines which implements Section 5 of
Article XIV of the Constitution with reference to the giving of free scholarships to gifted children, does
not require scholars to reimburse the corresponding value of the scholarships if they transfer to other
schools. So also with the leading colleges and universities of the United States after which our
educational practices or policies are patterned. In these institutions scholarships are granted not to
attract and to keep brilliant students in school for their propaganda value but to reward merit or help
gifted students in whom society has an established interest or a first lien." (Emphasis supplied.)

WHEREFORE, the decision appealed from is hereby reversed, and another one shall be entered
sentencing the defendant to pay to the plaintiff the sum of P1,033.87, with interest thereon at the legal
rate from September 1, 1954, date of the institution of this case, as well as the costs, and dismissing
defendant's counterclaim. It is so ordered.

Bengzon, C.J., Padilla, Labrador, Reyes, J.B.L., Barrera, Paredes, Dizon, De Leon and Natividad, JJ., concur.

Bautista Angelo, J., reverses his vote.

C o p y r i g h t 1 9 9 4 - 1 9 9 9 C D T e c h n o l o g i e s A s i a, I n c.

G.R. No. L-14860 May 30, 1961

102
In re Tan v. Republic

EN BANC

[G.R. No. L-14860. May 30, 1961.]

IN THE MATTER OF THE PETITION FOR ADMISSION TO PHILIPPINE CITIZENSHIP, ZACARIAS G. TAN,
petitioner-appellee, vs. REPUBLIC OF THE PHILIPPINES, oppositor-appellant.

J. Gonzales Chung Jr. for petitioner-appellee.

Solicitor General for oppositor-appellant.

SYLLABUS

1. CITIZENSHIP; NATURALIZATION; AMENDED PETITIONS; WHEN ANOTHER PUBLICATION IS


NECESSARY. — The amendment to the petition to the effect that applicant completed his elementary
and secondary education in schools recognised by the Government, is a material fact which entitles
petitioner to exemption from filing a declaration of intention. For this reason, the fact alleged in the
amendment should be published to enable the public to furnish the material evidence and information
to the Government regarding such qualification of the petitioner. Where the amended petition was not
published according to law, the court loses jurisdiction to hear and grant the petition, because Section 1
of Republic Act No. 530 provides that no petition for naturalization should be heard until after six
months from the date of the last publication. (Kiat Chun Tan vs. Republic, 92 Phil., 987.)

2. ID.; ID.; REQUIREMENT AS TO LUCRATIVE INCOME; P120.00 MONTHLY INCOME NOT


SUFFICIENT. — Where petitioner has an occupation and a monthly income of P120.00 therefrom, his
occupation cannot be considered as sufficiently lucrative because of the high cost of living now
prevailing. (Swee Din Tan vs. Republic, 109 Phil., 287; Republic vs. Lim, L-3030, Jan. 31, 1951; Pang Kok
Hua vs. Republic, 91, Phil., 254.)

3. ID.; ID.; ID.; WHEN PARENT'S CERTIFICATION IS DOUBTFUL. — The fact that his own father is
petitioner's employer, and that petitioner is still living with him, makes doubtful the truth of the father's
certification as to petitioner's employment, and gives rise to the suspicion that he was employed by his
father, if it were true that he was, only for the purpose of the petition.

4. ID.; ID.; TWO-YEAR PERIOD AFTER PROMULGATION OF DECISION; WHEN DECISION IS


OBJECTIONABLE. — Where the dispositive part of the decision gives the impression that the petitioner
can take his oath without waiting for the two-year period provided by law, because it directs the clerk of
court to forward the papers to the different government offices as soon as possible, the said decision is
objectionable, because it is contrary to the provisions of Republic Act No. 530, in that it allows the
petitioner to be naturalized 30 days after the promulgation of the decision.

DECISION

LABRADOR, J p:

103
Appeal from a decision of the Court of First Instance of Leyte, Hon. Gaudencio Cloribel, presiding, finding
Zacarias G. Tan, a Chinese citizen, qualified for admission as citizen of the Philippines.

The facts necessary for the resolution of the appeal are briefly as follows: On May 31, 1956, Zacarias G.
Tan filed a petition for naturalization, alleging, among other things, that he is an employee in a business
concern at Maasin, Leyte, since May 15, 1956, with a monthly salary of P120.00; that he was born on
November 5, 1933 in Maasin and as a citizen of Nationalist China; that he is not suffering from any
incurable contagious disease; that it is his intention in good faith to become a citizen of the Philippines
and to renounce absolutely and forever all allegiance and fidelity to any foreign prince, potentate, state,
or sovereignty, and particularly to the Republic of China; that he has all the qualifications and none of
the disqualifications which bar him from becoming a citizen of the Philippines; that Miguel Orito, Sr. and
Pedro M. Flores, both of Maasin, Leyte and Filipino citizens, are his witnesses. He prays that he be
admitted a citizen of the Philippines. No declaration of intention was submitted by the petitioner with
his application.

The Republic filed a motion to dismiss the petition for failure to allege therein such of petitioner's
educational qualifications as would exempt him from filing a declaration of intention. Acting upon the
motion the lower court on May 8, 1957, ordered the amendment of the petition to include the
necessary allegations. In compliance with the order, petitioner on May 20, 1957 filed his amended
petition, incorporating in paragraph 4 thereof the phrase "and I have completed my elementary and
secondary education in schools recognized by the Philippine Government". This amended petition was
admitted by the lower court. A motion for reconsideration of the order, on the ground that a
republication of the amended petition is necessary before the Court proceed with the trial, was denied,
and so the hearing on the case was continued.

On November 18, 1957, the lower court, rendered the decision appealed from. In said decision the court
found that petitioner has completed his elementary education in the public schools duly recognized by
the Government; that he is not afflicted with any contagious and incurable disease; that he will
renounce allegiance and loyalty to the Republic of Nationalist China of which he is a citizen; that he is
employed by Tan Yu Chin at the rate of P120.00 a month; that he possesses all the qualifications to
become a citizen of the Philippines and none of the disqualifications. The court granted the petition and
directed the Clerk of Court to forward "as soon as possible" copies of the decision and all pertinent
papers in connection with the case to the Solicitor General, the National Bureau of Investigation,
Philippine Constabulary, the Commissioner of Immigration and the Local Civil Registrar of Maasin, Leyte.

Against the above decision, the Solicitor General has prosecuted this appeal before this Court.

The first ground for assailing the correctness and validity of the decision is that the lower court had no
jurisdiction to try the naturalization case because the amended petition was not republished. An
examination of the record discloses that although the original petition for naturalization was published
in the Official Gazette and in a newspaper of general circulation, the amended petition was not. The
amendment, i. e., that applicant completed his elementary and secondary education in schools
recognized by the Government, is a material fact which entitles petitioner to exemption from filing a

104
declaration of intention. The fact alleged in the amendment should be made known to the public to
enable the latter to furnish the material evidence and information to the Government regarding such
qualification of the petitioner. Inasmuch as the amended petition was not published in accordance with
law and Section 1 of Republic Act No. 530 provides that no petition for naturalization should be heard
until after six months from the date of the last publication, the court below did not have jurisdiction to
hear the amended petition and grant the same. (Kiat Chun Tan vs. Republic, G.R. No. L-4802, April 29,
1953.)

As to the merits of the petition, upon consideration of the evidence at the trial, we find no credible
proof to sustain the lower court's finding that the petitioner has a lucrative profession or occupation.
Except for the unworn certification of Tan Yu Chin, petitioner's father, and petitioner's own testimony,
no other evidence was presented to prove petitioner's alleged income of P120.00 a month. This Court is
not bound by a mere statement of the petitioner. The certification issued by petitioner's own father is
incompetent, not only because it is hearsay but also because it is not sworn to. Moreover, the fact that
his own father is his employer and that he is still living with him makes doubtful the truth of petitioner's
employment and gives rise to the suspicion that he was employed by his father, if it were true that he
was, only for the purpose of this petition.

But even if we concede that petitioner has an occupation because of his supposed monthly income of
P120.00, still his occupation cannot be considered as sufficiently lucrative because of the high cost of
living now prevailing (Swee Din Tan vs. Republic, G.R. No. L-13177, August 31, 1960; Republic vs. Lim, L-
3030, January 31, 1951; Pang Kok Hua vs. Republic, L-5047, May 8, 1952.) The second ground of the
Republic in assailing the decision of the lower court is therefore meritorious.

The third ground of the Solicitor General's appeal is that the decision is contrary to the provisions of
Republic Act No. 530, in that it allows the petitioner to be naturalized 30 days after the promulgation of
the decision. This argument is also well founded. The dispositive part of the decision gives the
impression that the petitioner can take his oath without waiting for the two-year period provided for by
law because it directs the clerk of court to forward the papers to the different government offices as
soon as possible. The decision is apparently objectionable on this ground.

In view of the foregoing considerations, we find that the court below exceeded its jurisdiction in hearing
the petition as amended; and that petitioner is not qualified for admission for naturalization because of
lack of a lucrative occupation. The decision is hereby reversed and the petition dismissed, with costs
against the petitioner.

Bengzon, C.J., Padilla, Bautista Angelo, Concepcion, Reyes, J.B.L., Paredes, Dizon, De Leon and Natividad,
JJ., concur.

Barrera, J., took no part.

105
Mecano v. COA, G.R. No. 103982, 11 December 1992

[G.R. No. 103982. December 11, 1992.]

ANTONIO A. MECANO, petitioner, vs. COMMISSION ON AUDIT, respondent.

SYLLABUS

1. STATUTORY CONSTRUCTION; STATUTES; KINDS OF REPEALS. — The question of whether a


particular law has been repealed or not by a subsequent law is a matter of legislative intent. The
lawmakers may expressly repeal a law by incorporating therein a repealing provision which expressly
and specifically cites the particular law or laws, and portions thereof, that are intended to be repealed. A
declaration in a statute, usually in its repealing clause, that a particular and specific law, identified by its
number or title, is repealed is an express repeal; all others are implied repeals.

2. ID.; ID.; REPEALS BY IMPLICATION; NECESSITY OF A CLEAR INDICATION OF LEGISLATIVE PURPOSE


TO REPEAL. — The fact that a later enactment may relate to the same subject matter as that of an
earlier statute is not of itself sufficient to cause an implied repeal of the prior act, since the new statute
may merely be cumulative or a continuation of the old one. What is necessary is a manifest indication of
legislative purpose to repeal. Repeal by implication proceeds on the premise that where a statute of
later date clearly reveals an intention on the part of the legislature to abrogate a prior act on the
subject, that intention must be given effect. Hence, before there can be a repeal, there must be a clear
showing on the part of the lawmaker that the intent in enacting the new law was to abrogate the old
one. The intention to repeal must be clear and manifest; otherwise, at least, as a general rule, the later
act is to be construed as a continuation of, and not a substitute for, the first act and will continue so far
as the two acts are the same from the time of the first enactment.

3. ID.; ID.; ID.; CATEGORIES THEREOF. — There are two categories of repeal by implication. The
first is where provisions in the two acts on the same subject matter are in an irreconcilable conflict, The
later act to the extent of the conflict constitutes an implied repeal of the earlier one. The second is if the
later act covers the whole subject of the earlier one and is clearly intended as a substitute, it will
operate to repeal the earlier law. Implied repeal by irreconcilable inconsistency takes place when the
two statutes cover the same subject matter; they are so clearly inconsistent and incompatible with each
other that they cannot be reconciled or harmonized; and both cannot be given effect, that is, that one
law cannot be enforced without nullifying the other. The second category of repeal — the enactment of
a statute revising or codifying the former laws on the whole subject matter. This is only possible if this
revised statute or code was intended to cover the whole subject to be a complete and perfect system in
itself. It is the rule that a subsequent statute is deemed to repeal a prior law if the former revises the
whole subject matter of the former statute. When both intent and scope clearly evince the idea of a
repeal, then all parts and provisions of the prior act that are omitted from the revised act are deemed
repealed. Furthermore, before there can be an implied repeal under this category, it must be the clear
intent of the legislature that the later act be the substitute to the prior act.

106
4. ID.; ID.; ID.; ID.; NOT IMPLIED REPEAL OF SECTION 699 OF THE REVISED ADMINISTRATIVE CODE
BY ADMINISTRATIVE CODE OF 1987; CASE AT BAR. — Comparing the two Codes, it is apparent that the
new Code does not cover nor attempt to cover the entire subject matter of the old Code. There are
several matters treated in the old Code which are not found in the new Code, such as the provisions on
notaries public, the leave law, the public bonding law, military reservations, claims for sickness benefits
under Section 699, and still others. Moreover, the COA failed to demonstrate that the provisions of the
two Codes on the matter of the subject claim are in an irreconcilable conflict. In fact, there can be no
such conflict because the provision on sickness benefits of the nature being claimed by petitioner has
not been restated in the Administrative Code of 1987.

5. ADMINISTRATIVE LAW; ADMINISTRATIVE CONSTRUCTION AND INTERPRETATION OF LAWS;


WEIGHT OF OPINIONS OF THE SECRETARY OF JUSTICE ON STATUTES IN PARI MATERIA; CASE AT BAR. —
According to Opinion No. 73, S. 1991 of the Secretary of Justice, what appears clear is the intent to cover
only those aspects of government that pertain to administration, organization and procedure,
understandably because of the many changes that transpired in the government structure since the
enactment of the RAC decades of years ago. The COA challenges the weight that this opinion carries in
the determination of this controversy inasmuch as the body which had been entrusted with the
implementation of this particular provision has already rendered its decision. The COA relied on the rule
in administrative law enunciated in the case of Sison vs. Pangramuyen that in the absence of palpable
error or grave abuse of discretion, the Court would be loathe to substitute its own judgment for that of
the administrative agency entrusted with the enforcement and implementation of the law. This will not
hold water. This principle is subject to limitations. Administrative decisions may be reviewed by the
courts upon a showing that the decision is vitiated by fraud, imposition or mistake. It has been held that
Opinions of the Secretary and Undersecretary of Justice are material in the construction of statutes in
pari materia.

6. STATUTORY CONSTRUCTION; REPEALS BY IMPLICATION NOT FAVORED. — Lastly, it is a well-


settled rule of statutory construction that repeals of statutes by implication are not favored. The
presumption is against inconsistency and repugnancy for the legislature is presumed to know the
existing laws on the subject and not to have enacted inconsistent or conflicting statutes. This Court, in a
case, explains the principle in detail as follows: "Repeals by implication are not favored, and will not be
decreed unless it is manifest that the legislature so intended. As laws are presumed to be passed with
deliberation with full knowledge of all existing ones on the subject, it is but reasonable to conclude that
in passing a statute it was not intended to interfere with or abrogate any former law relating to some
matter, unless the repugnancy between the two is not only irreconcilable, but also clear and convincing,
and flowing necessarily from the language used, unless the later act fully embraces the subject matter of
the earlier, or unless the reason for the earlier act is beyond peradventure renewed. Hence, every effort
must be used to make all acts stand and if, by any reasonable construction, they can be reconciled, the
later act will not operate as a repeal of the earlier."

7. LABOR CODE; ARTICLE 173 THEREOF; EMPLOYEES COMPENSATION; PAYMENT OF


COMPENSATION THEREUNDER NOT A BAR TO RECOVERY OF BENEFITS UNDER SEC. 699 OF THE REVISED
ADMINISTRATIVE CODE. — Regarding respondent's contention that recovery under this subject section

107
shall bar the recovery of benefits under the Employees' Compensation Program, the same cannot be
upheld. The second sentence of Article 173, Chapter II, Title II (dealing on Employees' Compensation and
State Insurance Fund), Book IV of the Labor Code, as amended by P.D. 1921, expressly provides that "the
payment of compensation under this Title shall not bar the recovery of benefits as provided for in
Section 669 of the Revised Administrative Code . . . whose benefits are administered by the system
(meaning SSS or GSIS) or by other agencies of the government."

DECISION

CAMPOS, JR., J p:

Antonio A. Mecano, through a petition for certiorari, seeks to nullify the decision of the Commission on
Audit (COA, for brevity) embodied in its 7th Indorsement, dated January 16, 1992, denying his claim for
reimbursement under Section 699 of the Revised Administrative Code (RAC), as amended, in the total
amount of P40,831.00.

Petitioner is a Director II of the National Bureau of Investigation (NBI). He was hospitalized for
cholecystitis from March 26, 1990 to April 7, 1990, on account of which he incurred medical and
hospitalization expenses, the total amount of which he is claiming from the COA.

On May 11, 1990, in a memorandum to the NBI Director, Alfredo S. Lim (Director Lim, for brevity), he
requested reimbursement for his expenses on the ground that he is entitled to the benefits under
Section 699 1 of the RAC, the pertinent provisions of which read:

"SECTION 699. Allowances in case of injury, death, or sickness incurred in performance of duty. —
When a person in the service of the national government or in the service of the government of a
province, city, municipality or municipal district is so injured in the performance of duty as thereby to
receive some actual physical hurt or wound, the proper Head of Department may direct that absence
during any period of disability thereby occasioned shall be on full pay, though not more than six months,
and in such case he may in his discretion also authorize the payment of the medical attendance,
necessary transportation, subsistence and hospital fees of the injured person. Absence in the case
contemplated shall be charged first against vacation leave, if any there be.

xxx xxx xxx

"In case of sickness caused by or connected directly with the performance of some act in the line of
duty, the Department head may in his discretion authorize the payment of the necessary hospital fees."
LLjur

Director Lim then forwarded petitioner's claim, in a 1st Indorsement dated June 22, 1990, to the
Secretary of Justice, along with the comment, bearing the same date, of Gerarda Galang, Chief, LED of
the NBI, "recommending favorable action thereof". Finding petitioner's illness to be service -connected,
the Committee on Physical Examination of the Department of Justice favorably recommended the
payment of petitioner's claim.

108
However, then Undersecretary of Justice Silvestre H. Bello III, in a 4th Indorsement dated November 21,
1990, returned petitioner's claim to Director Lim, having considered the statements of the Chairman of
the COA in its 5th Indorsement dated 19 September 1990, to the effect that the RAC being relied upon
was repealed by the Administrative Code of 1987.

Petitioner then re-submitted his claim to Director Lim, with a copy of Opinion No. 73, S. 1991 2 dated
April 26, 1991 of then Secretary of Justice Franklin M. Drilon (Secretary Drilon, for brevity) stating that
"the issuance of the Administrative Code did not operate to repeal or abrogate in its entirety the
Revised Administrative Code, including the particular Section 699 of the latter."

On May 10, 1991, Director Lim, under a 5th Indorsement transmitted anew Mecano's claim to then
Undersecretary Bello for favorable consideration. Under a 6th Indorsement, dated July 2, 1991,
Secretary Drilon forwarded petitioner's claim to the COA Chairman, recommending payment of the
same. COA Chairman Eufemio C. Domingo, in his 7th Indorsement of January 16, 1992, however denied
petitioner's claim on the ground that Section 699 of the RAC has been repealed by the Administrative
Code of 1987, solely for the reason that the same section was not restated nor re-enacted in the
Administrative Code of 1987. He commented, however, that the claim may be filed with the Employees'
Compensation Commission, considering that the illness of Director Mecano occurred after the effectivity
of the Administrative Code of 1987.

Eventually, petitioner's claim was returned by Undersecretary of Justice Eduardo Montenegro to


Director Lim under a 9th Indorsement dated February 7, 1992, with the advice that petitioner "elevate
the matter to the Supreme Court if he so desires."

On the sole issue of whether or not the Administrative Code of 1987 repealed or abrogated Section 699
of the RAC, this petition was brought for the consideration of this Court. cdphil

Petitioner anchors his claim on Section 699 of the RAC, as amended, and on the aforementioned
Opinion No. 73, S. 1991 of Secretary Drilon. He further maintains that in the event that a claim is filed
with the Employees' Compensation Commission, as suggested by respondent, he would still not be
barred from filing a claim under the subject section. Thus, the resolution of whether or not there was a
repeal of the Revised Administrative Code of 1917 would decide the fate of petitioner's claim for
reimbursement.

The COA, on the other hand, strongly maintains that the enactment of the Administrative Code of 1987
(Exec. Order No. 292) operated to revoke or supplant in its entirety the Revised Administrative Code of
1917. The COA claims that from the "whereas" clauses of the new Administrative Code, it can be
gleaned that it was the intent of the legislature to repeal the old Code. Moreover, the COA questions the
applicability of the aforesaid opinion of the Secretary of Justice in deciding the matter. Lastly, the COA
contends that employment-related sickness, injury or death is adequately covered by the Employees'
Compensation Program under P.D. 626, such that to allow simultaneous recovery of benefits under both
laws on account of the same contingency would be unfair and unjust to the government.

109
The question of whether a particular law has been repealed or not by a subsequent law is a matter of
legislative intent. The lawmakers may expressly repeal a law by incorporating therein a repealing
provision which expressly and specifically cites the particular law or laws, and portions thereof, that are
intended to be repealed. 3 A declaration in a statute, usually in its repealing clause, that a particular and
specific law, identified by its number or title, is repealed is an express repeal; all others are implied
repeals. 4

In the case of the two Administrative Codes in question, the ascertainment of whether or not it was the
intent of the legislature to supplant the old Code with the new Code partly depends on the scrutiny of
the repealing clause of the new Code. This provision is found in Section 27, Book VII (Final Provisions) of
the Administrative Code of 1987 which reads:

"SECTION 27. Repealing Clause. — All laws, decrees, orders, rules and regulations, or portions thereof,
inconsistent with this Code are hereby repealed or modified accordingly."

The question that should be asked is: What is the nature of this repealing clause? It is certainly not an
express repealing clause because it fails to identify or designate the act or acts that are intended to be
repealed. 5 Rather, it is an example of a general repealing provision, as stated in Opinion No. 73, S. 1991.
It is a clause which predicates the intended repeal under the condition that a substantial conflict must
be found in existing and prior acts. The failure to add a specific repealing clause indicates that the intent
was not to repeal any existing law, unless an irreconcilable inconsistency and repugnancy exist in the
terms of the new and old laws. 6 This latter situation falls under the category of an implied repeal.

Repeal by implication proceeds on the premise that where a statute of later date clearly reveals an
intention on the part of the legislature to abrogate a prior act on the subject, that intention must be
given effect. 7 Hence, before there can be a repeal, there must be a clear showing on the part of the
lawmaker that the intent in enacting the new law was to abrogate the old one. The intention to repeal
must be clear and manifest; 8 otherwise, at least, as a general rule, the later act is to be construed as a
continuation of, and not a substitute for, the first act and will continue so far as the two acts are the
same from the time of the first enactment. 9

There are two categories of repeal by implication. The first is where provisions in the two acts on the
same subject matter are in an irreconcilable conflict, The later act to the extent of the conflict
constitutes an implied repeal of the earlier one. The second is if the later act covers the whole subject of
the earlier one and is clearly intended as a substitute, it will operate to repeal the earlier law. 10

Implied repeal by irreconcilable inconsistency takes place when the two statutes cover the same subject
matter; they are so clearly inconsistent and incompatible with each other that they cannot be reconciled
or harmonized; and both cannot be given effect, that is, that one law cannot he enforced without
nullifying the other. 11 LexLib

Comparing the two Codes, it is apparent that the new Code does not cover nor attempt to cover the
entire subject matter of the old Code. There are several matters treated in the old Code which are not

110
found in the new Code, such as the provisions on notaries public, the leave law, the public bonding law,
military reservations, claims for sickness benefits under Section 699, and still others.

Moreover, the COA failed to demonstrate that the provisions of the two Codes on the matter of the
subject claim are in an irreconcilable conflict. In fact, there can be no such conflict because the provision
on sickness benefits of the nature being claimed by petitioner has not been restated in the
Administrative Code of 1987. However, the COA would have Us consider that the fact that Section 699
was not restated in the Administrative Code of 1987 meant that the same section had been repealed. It
further maintained that to allow the particular provisions not restated in the new Code to continue in
force argues against the Code itself. The COA anchored this argument on the whereas clause of the 1987
Code, which states:

"WHEREAS, the effectiveness of the Government will be enhanced by a new Administrative Code which
incorporates in a unified document the major structural, functional and procedural principles and rules
of governance; and

xxx xxx xxx"

It argues, in effect, that what is contemplated is only one Code — the Administrative Code of 1987. This
contention is untenable.

The fact that a later enactment may relate to the same subject matter as that of an earlier statute is not
of itself sufficient to cause an implied repeal of the prior act, since the new statute may merely be
cumulative or a continuation of the old one. 12 What is necessary is a manifest indication of legislative
purpose to repeal. 13

We come now to the second category of repeal — the enactment of a statute revising or codifying the
former laws on the whole subject matter. This is only possible if this revised statute or code was
intended to cover the whole subject to be a complete and perfect system in itself. It is the rule that a
subsequent statute is deemed to repeal a prior law if the former revises the whole subject matter of the
former statute. 14 When both intent and scope clearly evince the idea of a repeal, then all parts and
provisions of the prior act that are omitted from the revised act are deemed repealed. 15 Furthermore,
before there can be an implied repeal under this category, it must be the clear intent of the legislature
that the later act be the substitute to the prior act. 16

According to Opinion No. 73, S. 1991 of the Secretary of Justice, what appears clear is the intent to cover
only those aspects of government that pertain to administration, organization and procedure,
understandably because of the many changes that transpired in the government structure since the
enactment of the RAC decades of years ago. The COA challenges the weight that this opinion carries in
the determination of this controversy inasmuch as the body which had been entrusted with the
implementation of this particular provision has already rendered its decision. The COA relied on the rule
in administrative law enunciated in the case of Sison vs. Pangramuyen 17 that in the absence of palpable
error or grave abuse of discretion, the Court would be loathe to substitute its own judgment for that of
the administrative agency entrusted with the enforcement and implementation of the law. This will not

111
hold water. This principle is subject to limitations. Administrative decisions may be reviewed by the
courts upon a showing that the decision is vitiated by fraud, imposition or mistake. 18 It has been held
that Opinions of the Secretary and Undersecretary of Justice are material in the construction of statutes
in pari materia. 19

Lastly, it is a well-settled rule of statutory construction that repeals of statutes by implication are not
favored. 20 The presumption is against inconsistency and repugnancy for the legislature is presumed to
know the existing laws on the subject and not to have enacted inconsistent or conflicting statutes. 21

This Court, in a case, explains the principle in detail as follows: "Repeals by implication are not favored,
and will not be decreed unless it is manifest that the legislature so intended. As laws are presumed to be
passed with deliberation with full knowledge of all existing ones on the subject, it is but reasonable to
conclude that in passing a statute it was not intended to interfere with or abrogate any former law
relating to some matter, unless the repugnancy between the two is not only irreconcilable, but also
clear and convincing, and flowing necessarily from the language used, unless the later act fully embraces
the subject matter of the earlier, or unless the reason for the earlier act is beyond peradventure
renewed. Hence, every effort must be used to make all acts stand and if, by any reasonable
construction, they can be reconciled, the later act will not operate as a repeal of the earlier. 22

Regarding respondent's contention that recovery under this subject section shall bar the recovery of
benefits under the Employees' Compensation Program, the same cannot be upheld. The second
sentence of Article 173, of the Labor Code, as amended by P.D. 1921, expressly provides that "the
payment of compensation under this Title shall not bar the recovery of benefits as provided for in
Section 669 of the Revised Administrative Code xxx whose benefits are administered by the system
(meaning SSS or GSIS) or by other agencies of the government."

WHEREFORE, premises considered, the Court resolves to GRANT the petition; respondent is hereby
ordered to give due course to petitioner's claim for benefits. No costs.

SO ORDERED.

Narvasa, C .J ., Cruz, Feliciano, Padilla, Bidin, Griño-Aquino, Regalado, Davide, Jr., Romero, Nocon,
Bellosillo and Melo, JJ ., concur.

Gutierrez, Jr., J ., concurs in the result.

112
People v. Licera G.R. No. L-39990, July 2, 1975

[G.R. No. L-39990. July 22, 1975.]

THE PEOPLE OF THE PHILIPPINES, plaintiff-appellee, vs. RAFAEL LICERA, defendant-appellant.

Solicitor General Felix Q. Antonio, Assistant Solicitor General Crispin V. Bautista and Solicitor Pedro A.
Ramirez for plaintiff-appellee.

Romeo Mercado (Counsel de Oficio) for defendant-appellant.

SYNOPSIS

In the municipal court, defendant was charged with the offenses of illegal possession of firearm and
assault upon an agent of a person in authority. Found guilty of the first charge, he appealed to the Court
of First Instance of the province. The second case against him was forwarded to the same court where
the parties agreed to a joint trial of the two cases but was only convicted of illegal possession of firearm.
Plaintiff brought the case to the Court of Appeals invoking as his legal jurisdiction for his possession of
firearm his appointment as a secret agent by the Governor of Batangas. He claimed that as secret agent
he was a "peace officer" and, thus, pursuant to People vs. Macarandang (L-12081, Dec. 23, 1959), he
was exempt from the requirements relating to issuance of license to possess. He alleged that the lower
court erred in relying on the later case of People vs. Mapa (L-22301, Aug. 30, 1967), which held that
Section 879 of the Revised Administrative Code provides no exemption for persons appointed as secret
agents by provincial governors for the requirements relating to firearms licenses. The case was certified
to this Court on the ground that a question of law was involved.

The Supreme Court held that pursuant to the Macarandang rule obtaining not only at the time of
defendant's appointment as secret agent, which appointment included a grant of authority to possess
the firearm, but as well as at the time of his apprehension, defendants incurred no criminal liability for
possession of the said rifle, notwithstanding his non-compliance with the legal requirements relating to
firearm licenses.

SYLLABUS

1. COURT DECISIONS; PRINCIPLE OF STARE DECISIS. — Article 8 of the Civil Code of the Philippines
decrees that judicial decisions applying or interpreting the laws of the Constitution form part of this
jurisdiction's legal system. These decisions, although in themselves are not laws, constitute evidence of
what the laws mean. The application or interpretation merely establishes the contemporaneous
legislative intent that the construed law purports to carry into effect.

2. ILLEGAL POSSESSION OF FIREARMS; DOCTRINE EXEMPTING SECRET AGENTS FROM THE


FIREARM LICENSE REQUIREMENT, ABANDONED. — The rule enunciated in Macarandang (106 Phil. 713)
to the effect that the appointment of a civilian as a "secret agent to assist in the maintenance of peace
and order campaigns and detection of crimes sufficiently puts him within the category of a 'peace
officer' equivalent to a member of the municipal police" whom Section 879 of the Revised

113
Administrative Code exempts from the requirements relating to firearms licenses, had been revoked by
the rule in Mapa (L-22301, August 30, 1967) which held that said section provides no exemption for
persons appointed as secret agents by provincial governors from the firearm license requirement.

3. CONSTITUTIONAL LAW; EX POST FACTO LAW; CONSTITUTIONAL GUARANTEE AGAINST EX POST


FACTO LAW APPLIED TO JUDICIAL DOCTRINES. — Where a new doctrine abrogates an old rule, the new
doctrine should operate prospectively only and should not adversely affect those favored by the old
rule, especially those who relied thereon and acted on the faith thereof. This holds more especially true
in the application or interpretation of statutes in the field of penal law, for, in this area, more than in any
other it is imperative that the punishability of an act be reasonably foreseen for the guidance of society.

4. ID.; ID.; ILLEGAL POSSESSION OF FIREARMS; DOCTRINE OBTAINING AT THE TIME OF POSSESSION
OF FIREARM BY SECRET AGENT APPLIES. — Where the rule obtaining not only at the time of his
appointment as secret agent, but as well as at the time of his apprehension, accused as such secret
agent was exempt from the firearm license requirements under Section 879 of the Revised
Administrative Code and therefore incurred no criminal liability for possession of the firearm, a
subsequent rule holding that said law does not exempt a secret agent from the firearm license
requirement shall not adversely affect said accused who was favored by the abandoned doctrine.

DECISION

CASTRO, J p:

This is an appeal, on a question of law, by Rafael Licera from the judgment dated August 14, 1968 of the
Court of First Instance of Occidental Mindoro convicting him of the crime of illegal possession of firearm
and sentencing him to imprisonment of five (5) years. We reverse the judgment of conviction, for the
reasons hereunder stated.

On December 3, 1965 the Chief of Police of Abra de Ilog, Occidental Mindoro, filed a complaint,
subscribed and sworn to by him, with the municipal court of the said municipality, charging Rafael Licera
with illegal possession of a Winchester rifle, Model 55, Caliber .30. On August 13, 1966 the municipal
court rendered judgment finding Licera guilty of the crime charged, sentencing him to suffer an
indeterminate penalty ranging five years and one day to six years and eight months of imprisonment.
Licera appealed to the Court of First Instance of Occidental Mindoro.

In the Court of First Instance, the parties agreed to the joint trial of the case for illegal possession of
firearm and another case, likewise filed against Licera with the municipal court but already forwarded to
the said Court of First Instance, for assault upon an agent of a person in authority, the two offenses
having arisen from the same occasion: apprehension of Licera by the Chief of Police and a patrolman of
Abra de Ilog on December 2, 1965 for possession of the Winchester rifle without the requisite license or
permit therefor.

On August 14, 1968 the court a quo rendered judgment acquitting Licera of the charge of assault upon
an agent of a person in authority, but convicting him of illegal possession of firearm, sentencing him to

114
suffer five years of imprisonment, and ordering the forfeiture of the Winchester rifle in favor of the
Government.

Licera's appeal to the Court of Appeals was certified on October 16, 1974 to this Court as involving only
one question of law.

Licera invokes as his legal justification for his possession of the Winschester rifle his appointment as
secret agent on December 11, 1961 by Governor Feliciano Leviste of Batangas. He claims that as secret
agent, he was a "peace officer" and, thus, pursuant to People vs. Macarandang, 1 was exempt from the
requirements relating to the issuance of license to possess firearms. He alleges that the court a quo
erred in relying on the later case of People vs. Mapa 2 which held that section 879 of the Revised
Administrative Code provides no exemption for persons appointed as secret agents by provincial
governors from the requirements relating to firearm licenses.

The principal question thus posed calls for a determination of the rule that should be applied to the case
at bar — that enunciated in Macarandang or that in Mapa.

The appointment given to Licera by Governor Leviste which bears the date "December 11, 1961"
includes a grant of authority to Licera to possess the Winchester rifle in these terms: "In accordance
with the decision of the Supreme Court in G.R. No. L-12088 dated December 23, 1959, you will have the
right to bear a firearm . . . for use in connection with the performance of your duties." Under the rule
then prevailing enunciated in Macarandang, 3 the appointment of a civilian as a "secret agent to assist in
the maintenance of peace and order campaigns and detection of crimes sufficiently put[s] him within
the category of a 'peace officer' equivalent even to a member of the municipal police" whom section
879 of the Revised Administrative Code exempts from the requirement relating to firearm licenses.

Article 8 of the Civil Code of the Philippines decrees that judicial decisions applying or interpreting the
laws or the Constitution form part of this jurisdiction's legal system. These decisions, although in
themselves not laws, constitute evidence of what the laws mean. The application or interpretation
placed by the Court upon a law is part of the law as of the date of the enactment of the said law since
the Court's application or interpretation merely establishes the contemporaneous legislative intent that
the construed law purports to carry into effect. 4

At the time of Licera's designation as secret agent in 1961 and at the time of his apprehension for
possession of the Winchester rifle without the requisite license or permit therefor in 1965, the
Macarandang rule — the Courts interpretation of section 879 of the Revised Administrative Code —
formed part of our jurisprudence and, hence, of this jurisdiction's legal system. Mapa revoked the
Macarandang precedent only in 1967. Certainly, where a new doctrine abrogates an old rule, the new
doctrine should operate prospectively only and should not adversely affect those favored by the old
rule, especially those who relied thereon and acted on the faith thereof. This holds more especially true
in the application or interpretation of statutes in the field of penal law, for, in this area, more than in any
other, it is imperative that the punishability of an act be reasonably foreseen for the guidance of society.
5

115
Pursuant to the Macarandang rule obtaining not only at the time of Licera's appointment as secret
agent, which appointment included a grant of authority to possess the Winchester rifle, but as well at
the time as of his apprehension, Licera incurred no criminal liability for possession of the said rifle,
notwithstanding his non-compliance with the legal requirements relating to firearm licenses.

ACCORDINGLY, the judgment a quo is reversed, and Rafael Licera is hereby acquitted. Costs de oficio.

Makasiar, Esguerra, Muñoz, Palma and Martin, JJ., concur.

Teehankee, J., is on leave.

116
Chu Jan vs. Bernas, 34 Phil 631 (1916)

[G.R. No. 10010. August 1, 1916.]

CHU JAN, plaintiff-appellee, vs. LUCIO BERNAS, defendant-appellant.

Sulpicio V. Cea for appellant.

No appearance for appellee.

SYLLABUS

JUDGMENT; DISMISSAL OF ACTION. — Ignorance of the special law applicable to a case does not justify
the court in terminating the proceeding by dismissing it without a decision.

DECISION

ARAULLO, J p:

On the afternoon of June 26, 1913, a match was held in the cockpit of the municipality of Tabaco, Albay,
between two cocks belonging to the plaintiff and to the defendant respectively. Each of said persons
had put up a wager of P160; and as the referee of the cockpit had declared the defendant's cock the
winner in the bout, the plaintiff brought suit against the defendant in the justice of the peace court of
the said pueblo, asking that his own rooster be declared the winner. The justice of the peace court
decided that the bout was a draw. From this judgment the defendant appealed to the Court of First
Instance of the province. For the purpose of the appeal, the plaintiff files his complaint and prayed this
court to render judgment ordering the defendant to abide by and comply with the rules and regulations
governing cockfights, to pay the stipulated wager of P160; to return the other like amount (both sums or
wager being held for safe-keeping by the cockpit owner, Tomas Almonte) and to assess the costs of both
instances against the defendant.

The defendant denied each and all of the allegations of the complaint and moved to dismiss with the
costs against the plaintiff. On September 11, 1913, the said Court of first Instance rendered judgment
dismissing the appeal without special finding as to costs. The defendant excepted to this judgment as
well as to an order dictated by the same court on November 8th of the same year, on the plaintiff's
motion, ordering the provincial treasurer of Albay and, if necessary, the municipal treasurer of Tabaco of
the same province, to release the deposit of P160 and return it to its owner, the plaintiff Chinaman, Chu
Jan. These proceedings have come before us on appeal by means of the proper bill of exceptions.

The grounds for the dismissal pronounced by the lower court in the judgment appealed from were that
court has always dismissed cases of this nature, that he is not familiar with the rules governing
cockfights and the duties of referees thereof; that he does not know where to find the law on the
subject and, finally, that he knows of no law whatever that governs the rights of the plaintiff and the
defendant in questions concerning cockfights.

117
The ignorance of the court or his lack of knowledge regarding the law applicable to a case submitted to
him for decision, the fact that the court does not know the rules applicable to a certain matter that is
the subject of an appeal which must be decided by him and his not knowing where to find the law
relative to the case, are not reasons that can serve to excuse the court for terminating the proceedings
by dismissing them without deciding the issues. Such an excuse is the less acceptable because,
foreseeing that a case might arise to which no law would be exactly applicable, the Civil Code, in the
second paragraph of article 6, provides that the customs of the place shall be observed, and, in the
absence thereof, the general principles of law.

Therefore the judgment and the order appealed from, hereinbefore mentioned, are reversed and the
record of the proceedings shall be remanded to the court from whence they came for due trial and
judgment as provided by law. No special finding is made with regard to costs. So ordered.

Arellano, C. J., Torres, Johnson, and Trent, JJ., concur.

Moreland, J., did not take part.

118
People vs. Purisima, G.R. Nos. L-42050-66, L-46229-32, L-46313-16, L-46997, 20 November 1978

[G.R. Nos. L-42050-66. November 20, 1978.]

THE PEOPLE OF THE PHILIPPINES, petitioner, vs. HONORABLE JUDGE AMANTE P. PURISIMA, COURT OF
FIRST INSTANCE OF MANILA, BRANCH VII, and PORFIRIO CANDELOSAS, NESTOR BAES, ELIAS L. GARCIA,
SIMEON BUNDALIAN, JR., JOSEPH C. MAISO, EDUARDO A. LIBORDO, ROMEO L. SUGAY, FEDERICO T.
DIZON, GEORGE M. ALBINO, MARIANO COTIA, JR., ARMANDO L. DIZON, ROGELIO B. PAREÑO, RODRIGO
V. ESTRADA, ALFREDO A. REYES, JOSE A. BACARRA, REYNALDO BOGTONG, and EDGARDO M. MENDOZA,
respondents.

[G.R. No. L-46229-32. November 20, 1978.]

THE PEOPLE OF THE PHILIPPINES, petitioner, vs. JUDGE MAXIMO A. MACEREN, COURT OF FIRST
INSTANCE OF MANILA, BRANCH XVIII, and REYNALDO LAQUI Y AQUINO, ELPIDIO ARPON, VICTOR
EUGENIO Y ROQUE and ALFREDO VERSOZA, respondents.

[G.R. No. L-46313-16. November 20, 1978.]

THE PEOPLE OF THE PHILIPPINES, petitioner, vs. JUDGE MAXIMO A. MACEREN, COURT OF FIRST
INSTANCE OF MANILA, BRANCH XVIII, and JUANITO DE LA CRUZ Y NUÑEZ, SABINO BUENO Y CACAL,
TIRSO ISAGAN Y FRANCISCO and BEN CASTILLO Y UBALDO, respondents.

[G.R. No. L-46997. November 20, 1978.]

THE PEOPLE OF THE PHILIPPINES, petitioner, vs. THE HONORABLE WENCESLAO M. POLO, Judge of the
Court of First Instance of Samar, and PANCHITO REFUNCION, respondents.

Jose L. Gamboa, Fermin Martin, Jr. & Jose D. Cajucom, Office of the City of Fiscal of Manila and the
Office of Provincial Fiscal of Samar for petitioners. Norberto Parto for respondents Candelosas, Baes and
Garcia.

Amado C . de la Marced for respondents Simeon Bundalian Jr ., et al.

Manuel F . de Jesus for all the respondents in L-46229-32 and L-46313-16.

Norberto L. Apostol for respondent Panchito Refuncion.

Hon. Amante P. Purisima for and in his own behalf.

DECISION

MUÑOZ PALMA, J p:

These twenty-six (26) Petitions for Review filed by the People of the Philippines represented,
respectively, by the Office of the City Fiscal of Manila, the Office of the Provincial Fiscal of Samar, and

119
joined by the Solicitor General, are consolidated in this one Decision as they involve one basic question
of law.

These Petitions or appeals involve three Courts of First Instance, namely: the Court of First Instance of
Manila, Branch VII, presided by Hon. Amante P. Purisima (17 Petitions), the Court of First Instance of
Manila, Branch XVIII, presided by Hon. Maximo A. Maceren (8 Petitions) and, the Court of First Instance
of Samar, with Hon. Wenceslao M. Polo, presiding, (1 Petition).

Before those courts, Informations were filed charging the respective accused with "illegal possession of
deadly weapon" in violation of Presidential Decree No. 9. On a motion to quash filed by the accused, the
three Judges mentioned above issued in the respective cases filed before them — the details of which
will be recounted below — an Order quashing or dismissing the Informations, on a common ground, viz,
that the Information did not allege facts which constitute the offense penalized by Presidential Decree
No. 9 because it failed to state one essential element of the crime.

Thus, are the Informations filed by the People sufficient in form and substance to constitute the offense
of "illegal possession of deadly weapon" penalized under Presidential Decree (PD for short) No. 9? This is
the central issue which we shall resolve and dispose of, all other corollary matters not being
indispensable for the moment.

A — The Information filed by the People —

1. In L-42050-66, one typical Information filed with the Court presided by Judge Purisima follows:

"THE PEOPLE OF THE PHILIPPINES, plaintiff, versus PORFIRIO CANDELOSAS Y DURAN, accused.

Crim. Case No. 19639

VIOLATION OF PAR. 3, PRES. DECREE No. 9 OF PROCLAMATION 1081.

"INFORMATION

"The undersigned accuses PORFIRIO CANDELOSAS Y DURAN of a violation of paragraph 3, Presidential


Decree No. 9 of Proclamation 1081, committed as follows:

"That on or about the 14th day of December, 1974, in the City of Manila, Philippines, the said accused
did then and there wilfully, unlawfully, feloniously and knowingly have in his possession and under his
custody and control one (1) carving knife with a blade of 6-1/2 inches and a wooden handle of 5-1/4
inches, or an overall length of 11-3/4 inches, which the said accused carried outside of his residence, the
said weapon not being used as a tool or implement necessary to earn his livelihood nor being used in
connection therewith.

"Contrary to law." (p. 32, rollo of L-42050-66)

The other Informations are similarly worded except for the name of the accused, the date and place of
the commission of the crime, and the kind of weapon involved. cdphil

120
2. In L-46229-32 and L-46313-16, the Information filed with the Court presided by Judge Maceren
follows:

"THE PEOPLE OF THE PHILIPPINES, plaintiff, versus REYNALDO LAQUI Y AQUINO, accused.

CRIM. CASE NO. 29677

VIOL. OF PAR. 3,

PD 9 IN REL. TO LOI

No. 266 of the Chief

Executive dated April 1, 1975

"INFORMATION

"The undersigned accuses REYNALDO LAQUI Y AQUINO of a VIOLATION OF PARAGRAPH 3,


PRESIDENTIAL DECREE NO. 9 in relation to Letter of Instruction No. 266 of the Chief Executive dated
April 1, 1975, committed as follows:

"That on or about the 28th day of January, 1977, in the City of Manila, Philippines, the said accused did
then and there wilfully, unlawfully and knowingly carry outside of his residence a bladed and pointed
weapon, to wit: an ice pick with an overall length of about 8 1/2 inches, the same not being used as a
necessary tool or implement to earn his livelihood nor being used in connection therewith.

"Contrary to law." (p. 14, rollo of L-46229-32).

The other Informations are likewise similarly worded except for the name of the accused, the date and
place of the commission of the crime, and the kind of weapon involved.

3. In L-46997, the Information before the Court of First Instance of Samar is quoted hereunder:

"PEOPLE OF THE PHILIPPINES, complainant, versus PANCHITO REFUNCION, accused.

CRIM. CASE NO. 933

For:

ILLEGAL POSSESSION OF

DEADLY WEAPON

(VIOLATION OF PD NO. 9)

"INFORMATION

"The undersigned First Assistant Provincial Fiscal of Samar, accuses PANCHITO REFUNCION of the crime
of ILLEGAL POSSESSION OF DEADLY WEAPON or VIOLATION OF PD NO. 9 issued by the President of the

121
Philippines on Oct. 2, 1972, pursuant to Proclamation No. 1081 dated Sept. 21 and 23, 1972, committed
as follows:

"That on or about the 6th day of October, 1976, in the evening at Barangay Barruz, Municipality of
Matuginao, Province of Samar Philippines, and within the jurisdiction of this Honorable Court, the
abovenamed accused, knowingly, wilfully, unlawfully and feloniously carried with him outside of his
residence a deadly weapon called socyatan, an instrument which from its very nature is no such as could
be used as a necessary tool or instrument to earn a livelihood, which act committed by the accused is a
Violation of Presidential Decree No. 9.

"CONTRARY TO LAW. " (p. 8, rollo of L-46997)

B — The Orders of dismissal —

In dismissing or quashing the Informations the trial courts concurred with the submittal of the defense
that one essential element of the offense charged is missing from the Information, viz: that the carrying
outside of the accused's residence of a bladed, pointed or blunt weapon is in furtherance or on the
occasion of, connected with or related to subversion, insurrection, or rebellion, organized lawlessness or
public disorder.

1. Judge Purisima reasoned out, inter alia, in this manner:

". . . the Court is of the opinion that in order that possession of bladed weapon or the like outside
residence may be prosecuted and tried under P.D. No. 9, the information must specifically allege that
the possession of bladed weapon charged was for the purpose of abetting, or in furtherance of the
conditions of rampant criminality, organized lawlessness, public disorder, etc. as are contemplated and
recited in Proclamation No. 1081, as justification therefor. Devoid of this specific allegation, not
necessarily in the same words, the information is not complete, as it does not allege sufficient facts to
constitute the offense contemplated in P.D. No. 9. The information in these cases under consideration
suffer from this defect.

"xxx xxx xxx

"And while there is no proof of it before the Court, it is not difficult to believe the murmurings of
detained persons brought to Court upon a charge of possession of bladed weapons under P.D. No. 9,
that more than ever before, policemen — of course not all can be so heartless — now have in their
hands P.D. No. 9 as a most convenient tool for extortion, what with the terrifying risk of being sentenced
to imprisonment of five to ten years for a rusted kitchen knife or a pair of scissors, which only God
knows where it came from. Whereas before martial law an extortion-minded peace officer had to have a
stock of the cheapest paltik, and even that could only convey the coercive message of one year in jail,
now anything that has the semblance of a sharp edge or pointed object, available even in trash cans,
may already serve the same purpose, and yet five to ten times more incriminating than the infamous
paltik.

122
"For sure, P.D. No. 9 was conceived with the best of intentions and wisely applied, its necessity can
never be assailed. But it seems it is back-firing, because it is too hot in the hands of policemen who are
inclined to backsliding.

"The checkvalves against abuse of P.D. No. 9 are to be found in the heart of the Fiscal and the
conscience of the Court, and hence this resolution, let alone technical legal basis, is prompted by the
desire of this Court to apply said checkvalves." (pp. 55-57, rollo of L-42050-66).

2. Judge Maceren in turn gave his grounds for dismissing the charges as follows:

"xxx xxx xxx

"As earlier noted the 'desired result' sought to be attained by Proclamation No. 1081 is the maintenance
of law and order throughout the Philippines and the prevention and suppression of all forms of lawless
violence as well as any act of insurrection or rebellion. It is therefore reasonable to conclude from the
foregoing premises that the carrying of bladed, pointed or blunt weapons outside of one's residence
which is made unlawful and punishable by said par. 3 of P.D. No. 9 is one that abets subversion,
insurrection or rebellion, lawless violence, criminality, chaos and public disorder or is intended to bring
about these conditions. This conclusion is further strengthened by the fact that all previously existing
laws that also made the carrying of similar weapons punishable have not been repealed, whether
expressly or impliedly. It is noteworthy that Presidential Decree No. 9 does not contain any repealing
clause or provisions.

"xxx xxx xxx

"The mere carrying outside of one's residence of these deadly weapons if not concealed in one's person
and if not carried in any of the aforesaid specified places, would appear to be not unlawful and
punishable by law.

"With the promulgation of Presidential Decree No. 9, however, the prosecution, through Assistant Fiscal
Hilario H. Laqui, contends in his opposition to the motion to quash, that this act is now made unlawful
and punishable, particularly by paragraph 3 thereof, regardless of the intention of the person carrying
such weapon because the law makes it 'mala prohibita'. If the contention of the prosecution is correct,
then if a person happens to be caught while on his way home by law enforcement officers carrying a
kitchen knife that said person had just bought from a store in order that the same may be used by one's
cook for preparing the meals in one's home, such person will be liable for punishment with such a
severe penalty as imprisonment from five to ten years under the decree. Such person cannot claim that
said knife is going to be used by him to earn a livelihood because he intended it merely for use by his
cook in preparing his meals.

"This possibility cannot be discounted if Presidential Decree No. 9 were to be interpreted and applied in
the manner that the prosecution wants it to be done. The good intentions of the President in
promulgating this decree may thus be perverted by some unscrupulous law enforcement officers. It may
be used as a tool of oppression and tyranny or of extortion.

123
"xxx xxx xxx

"It is therefore the considered and humble view of this Court that the act which the President intended
to make unlawful and punishable by Presidential Decree No. 9, particularly by paragraph 3 thereof, is
one that abets or is intended to abet subversion, rebellion, insurrection, lawless violence, criminality,
chaos and public disorder." (pp. 28-30, rollo of L-46229-32)

3. Judge Polo of the Court of First Instance of Samar expounded his order dismissing the
Information filed before him, thus: LLpr

". . . We believe that to constitute an offense under the aforecited Presidential Decree, the same should
be or there should be an allegation that a felony was committed in connection or in furtherance of
subversion, rebellion, insurrection, lawless violence and public disorder. Precisely Proclamation No.
1081 declaring a state of martial law throughout the country was issued because of wanton destruction
to lives and properties widespread lawlessness and anarchy. And in order to restore the tranquility and
stability of the country and to secure the people from violence and loss of lives in the quickest possible
manner and time, carrying firearms, explosives and deadly weapons without a permit unless the same
would fall under the exception is prohibited. This conclusion becomes more compelling when we
consider the penalty imposable, which is from five years to ten years. A strict enforcement of the
provision of the said law would mean the imposition of the Draconian penalty upon the accused.

"xxx xxx xxx

"It is public knowledge that in rural areas, even before and during martial law, as a matter of status
symbol, carrying deadly weapons is very common, not necessarily for committing a crime nor as their
farm implement but for self-preservation or self-defense if necessity would arise specially in going to
and from their farm." (pp. 18-19, rollo of L-46997)

In most if not all of the cases, the orders of dismissal were given before arraignment of the accused. In
the criminal case before the Court of (First Instance of Samar the accused was arraigned but at the same
time moved to quash the Information. In all the cases where the accused were under arrest, the three
Judges ordered their immediate release unless held on other charges.

C — The law under which the Informations in question were filed by the People.

As seen from the Informations quoted above, the accused are charged with illegal possession of deadly
weapon in violation of Presidential Decree No. 9, Paragraph 3.

We quote in full Presidential Decree No. 9, to wit:

"PRESIDENTIAL DECREE NO. 9

"DECLARING VIOLATIONS OF GENERAL ORDERS NO. 6 and NO. 7 DATED SEPTEMBER 22, 1972, AND
SEPTEMBER 23, 1972, RESPECTIVELY, TO BE UNLAWFUL AND PROVIDING PENALTIES THEREFORE.

124
"WHEREAS, pursuant to Proclamation No. 1081 dated September 21, 1972, the Philippines has been
placed under a state of martial law;

"WHEREAS, by virtue of said Proclamation No. 1081, General Order No. 6 dated September 22, 1972 and
General Order No. 7 dated September 23, 1972, have been promulgated by me;

"WHEREAS, subversion, rebellion, insurrection, lawless violence, criminality, chaos and public disorder
mentioned in the aforesaid Proclamation No. 1081 are committed and abetted by the use of firearms,
explosives and other deadly weapons;

"NOW, THEREFORE, I, FERDINAND E. MARCOS, Commander-in-Chief of all the Armed Forces of the
Philippines, in order to attain the desired result of the aforesaid Proclamation No. 1081 and General
Orders Nos. 6 and 7, do hereby order and decree that:

"1. Any violation of the aforesaid General Orders Nos. 6 and 7 is unlawful and the violator shall,
upon conviction suffer:

(a) The mandatory penalty of death by a firing squad or electrocution as a Military


Court/Tribunal/Commission may direct, if the firearm involved in the violation is unlicensed and is
attended by assault upon, or resistance to persons in authority or their agents in the performance of
their official functions resulting in death to said persons in authority or their agent; or if such unlicensed
firearm is used in the commission of crimes against persons, property or chastity causing the death of
the victim, or used in violation of any other General Orders and/or Letters of Instructions promulgated
under said Proclamation No. 1081:

(b) The penalty of imprisonment ranging from twenty years to life imprisonment as a Military
Court/Tribunal/Commission may direct, when the violation is not attended by any of the circumstances
enumerated under the preceding paragraph;

(c) The penalty provided for in the preceding paragraphs shall be imposed upon the owner,
president, manager, members of the board of directors or other responsible officers of any public or
private firms, companies, corporations or entities who shall willfully or knowingly allow any of the
firearms owned by such firm, company, corporation or entity concerned to be used in violation of said
General Orders Nos. 6 and 7.

"2. It is unlawful to possess deadly weapons, including hand grenades, rifle grenades and other
explosives, including, but not limited to, 'pill box bombs,' 'molotov cocktail bombs,' 'fire bombs,' or other
incendiary device consisting of any chemical, chemical compound, or detonating agents containing
combustible units or other ingredients in such proportion, quantity, packing, or bottling that ignites by
fire, by friction, by concussion, by percussion, or by detonation of all or part of the compound or mixture
which may cause such a sudden generation of highly heated gases that the resultant gaseous pressures
are capable of producing destructive effects on contiguous objects or of causing injury or death of a
persons and any person convicted thereof shall be punished by imprisonment ranging from ten to
fifteen years as a Military Court/Tribunal/Commission may direct.

125
"3. It is unlawful to carry outside of residence any bladed, pointed or blunt weapon such as 'fan
knife,' 'spear,' 'dagger,' 'bolo,' 'balisong,' 'barong,' 'kris,' or club, except where such articles are being
used as necessary tools or implements to earn a livelihood and while being used in connection
therewith; and any person found guilty thereof shall suffer the penalty of imprisonment ranging from
five to ten years as a Military Court/Tribunal/Commission may direct.

"4. When the violation penalized in the preceding paragraphs 2 and 3 is committed during the
commission of or for the purpose of committing, any other crime, the penalty shall be imposed upon the
offender in its maximum extent, in addition to the penalty provided for the particular offenses
committed or intended to be committed.

"Done in the City of Manila, this 2nd day of October in the year of Our Lord, nineteen hundred and
seventy-two.

"(SGD) FERDINAND E. MARCOS

President

Republic of the Philippines"

D — The arguments of the People —

In the Comment filed in these cases by the Solicitor General who as stated earlier joins the City Fiscal of
Manila and the Provincial Fiscal of Samar in seeking the setting aside of the questioned orders of
dismissal, the main argument advanced on the issue now under consideration is that a perusal of
paragraph 3 of P.D. 9 shows that the prohibited acts need not be related to subversive activities; that
the act proscribed is essentially a malum prohibitum penalized for reasons of public policy. 1

The City Fiscal of Manila in his brief adds further that in statutory offenses the intention of the accused
who commits the act is immaterial; that it is enough if the prohibited act is voluntarily perpetuated; that
P.D. 9 provides and condemns not only the carrying of said weapon in connection with the commission
of the crime of subversion or the like, but also that of criminality in general, that is, to eradicate lawless
violence which characterized pre-martial law days. It is also argued that the real nature of the criminal
charge is determined not from the caption or preamble of the information nor from the specification of
the provision of law alleged to have been violated but by the actual recital of facts in the complaint or
information. 2

E — Our Ruling on the matter —

1. It is a constitutional right of any person who stands charged in a criminal prosecution to be


informed of the nature and cause of the accusation against him. 3

Pursuant to the above, Section 5, Rule 110 of the Rules of Court, expressly requires that for a complaint
or information to be sufficient it must, inter alia, state the designation of the offense by the statute, and

126
the acts or omissions complained of as constituting the offense. This is essential to avoid surprise on the
accused and to afford him the opportunity to prepare his defense accordingly. 4

To comply with these fundamental requirements of the Constitution and the Rules on Criminal
Procedure, it is imperative for the specific statute violated to be designated or mentioned in the charge.
In fact, another compelling reason exists why a specification of the statute violated is essential in these
cases. As stated in the order of respondent Judge Maceren the carrying of so-called "deadly weapons" is
the subject of another penal statute and a Manila city ordinance. Thus, Section 26 of Act No. 1780
provides: cdrep

"SECTION 26. It should be unlawful for any person to carry concealed about his person any bowie
knife, dirk, dagger, kris, or other deadly weapon: . . . Any person violating the provisions of this section
shall, upon conviction in a court of competent jurisdiction, be punished by a fine not exceeding five
hundred pesos, or by imprisonment for a period not exceeding six months, or both such fine and
imprisonment, in the discretion of the court."

Ordinance No. 3820 of the City of Manila as amended by Ordinance No. 3928 which took effect on
December 4, 1957, in turn penalizes with a fine of not more than P200.00 or imprisonment for not more
than one months, or both, at the discretion of the court, anyone who shall carry concealed in his person
in any manner that would disguise its deadly character any kind of firearm, bowie knife, or other deadly
weapon . . . in any public place. Consequently, it is necessary that the particular law violated be specified
as there exists a substantial difference between the statute and city ordinance on the one hand and P.D.
9 (3) on the other regarding the circumstances of the commission of the crime and the penalty imposed
for the offense.

We do not agree with petitioner that the above-mentioned statute and the city ordinance are deemed
repealed by P.D. 9 (3). 5 P. D. 9(3) does not contain any repealing clause or provision, and repeal by
implication is not favored. 6 This principle holds true with greater force with regards to penal statutes
which as a rule are to be construed strictly against the state and liberally in favor of the accused. 7 In
fact, Article 7 of the New Civil Code provides that laws are repealed only by subsequent ones and their
violation or non-observance shall not be excused by disuse, or custom or practice to the contrary.

Thus we are faced with the situation where a particular act may be made to fall, at the discretion of a
police officer or a prosecuting fiscal, under the statute, or the city ordinance, or the presidential decree.
That being the case, the right becomes more compelling for an accused to be confronted with the facts
constituting the essential elements of the offense charged against him, if he is not to become an easy
pawn of oppression and harassment, or of negligent or misguided official action - a fear understandably
shared by respondent Judges who by the nature of their judicial functions are daily exposed to such
dangers.

2. In all the Informations filed by petitioner the accused are charged in the caption as well as in the
body of the Information with a violation of paragraph 3, P.D. 9. What then are the elements of the
offense treated in the presidential decree in question?

127
We hold that the offense carries two elements: first, the carrying outside one's residence of any bladed,
blunt, or pointed weapon, etc. not used as a necessary tool or implement for a livelihood; and second,
that the act of carrying the weapon was either in furtherance of, or to abet, or in connection with
subversion, rebellion, insurrection, lawless violence, criminality, chaos, or public disorder.

It is the second element which removes the act of carrying a deadly weapon, if concealed, outside of the
scope of the statute or the city ordinance mentioned above. In other words, a simple act of carrying any
of the weapons described in the presidential decree is not a criminal offense in itself. What makes the
act criminal or punishable under the decree is the motivation behind it. Without that motivation, the act
falls within the purview of the city ordinance or some statute when the circumstances so warrant.

Respondent Judges correctly ruled that this can be the only reasonably, logical, and valid construction
given to P.D. 9(3).

3. The position taken by petitioner that P.D. 9(3) covers one and all situations where a person
carries outside his residence any of the weapons mentioned or described in the decree irrespective of
motivation, intent, or purpose, converts these cases into one of "statutory construction." That there is
ambiguity in the presidential decree is manifest from the conflicting views which arise from its
implementation. When ambiguity exists, it becomes a judicial task to construe and interpret the true
meaning and scope of the measure, guided by the basic principle that penal statutes are to be construed
and applied liberally in favor of the accused and strictly against the state. cdphil

4. In the construction or interpretation of a legislative measure — a presidential decree in these


cases — the primary rule is to search for and determine the intent and spirit of the law. Legislative
intent is the controlling factor, for in the words of this Court in Hidalgo v. Hidalgo, per Mr. Justice
Claudio Teehankee, whatever is within the spirit of a statute is within the statute, and this has to be so if
strict adherence to the letter would result in absurdity, injustice and contradictions. 8

There are certain aids available to Us to ascertain the intent or reason for P.D. 9(3).

First, the presence of events which led to or precipitated the enactment of P.D. 9. These events are
clearly spelled out in the "Whereas" clauses of the presidential decree, thus: (1) the state of martial law
in the country pursuant to Proclamation 1081 dated September 21, 1972; (2) the desired result of
Proclamation 1081 as well as General Orders Nos. 6 and 7 which are particularly mentioned in P.D. 9;
and (3) the alleged fact that subversion, rebellion, insurrection, lawless violence, criminality, chaos, and
public disorder mentioned in Proclamation 1081 are committed and abetted by the use of firearms and
explosives and other deadly weapons.

The Solicitor General however contends that a preamble of a statute usually introduced by the word
"whereas", is not an essential part of an act and cannot enlarge or confer powers, or cure inherent
defects in the statute (p. 120, rollo of L-42050-66); that the explanatory note or enacting clause of the
decree, if it indeed limits the violation of the decree, cannot prevail over the text itself inasmuch as such
explanatory note merely states or explains the reason which prompted the issuance of the decree. (pp.
114-115, rollo of 46997)

128
We disagree with these contentions. Because of the problem of determining what acts fall within the
purview of P.D. 9, it becomes necessary to inquire into the intent and spirit of the decree and this can be
found among others in the preamble or "whereas" clauses which enumerate the facts or events which
justify the promulgation of the decree and the stiff sanctions stated therein.

"A 'preamble' is the key of the statute, to open the minds of the makers as to the mischiefs which are to
be remedied, and objects which are to be accomplished, by the provisions of the statute." (West
Norman Timber v. State, 224 P. 2d 635, 639, cited in Words and Phrases, "Preamble"; emphasis
supplied).

"While the preamble of a statute is not strictly a part thereof, it may, when the statute is in itself
ambiguous and difficult of interpretation, be resorted to, but not to create a doubt or uncertainty which
otherwise does not exist." (James v. Du Bois, 16 N.J.L. (1 Har.) 285, 294, cited in Words and Phrases,
"Preamble").

In Aboitiz Shipping Corporation, et al., v. The City of Cebu, et al., this Court had occasion to state that
"(L)egislative intent must be ascertained from a consideration of the statute as a whole, and not of an
isolated part or a particular provision alone. This is a cardinal rule of statutory construction. For taken in
the abstract, a word or phrase might easily convey a meaning quite different from the one actually
intended and evident when the word or phrase is considered with those with which it is associated.
Thus, an apparently general provision may have a limited application if read together with other
provisions." 9

Second, the result or effects of the presidential decree must be within its reason or intent.

In the paragraph immediately following the last "Whereas" clause, the presidential decree states:

"NOW, THEREFORE, I, FERDINAND E. MARCOS, Commander-in-Chief of all the Armed Forces of the
Philippines, in order to attain the desired result of the aforesaid Proclamation No. 1081 and General
Orders Nos. 6 and 7, do hereby order and decree that:

"xxx xxx xxx

From the above it is clear that the acts penalized in P.D. 9 are those related to the desired result of
Proclamation 1081 and General Orders Nos. 6 and 7. General Orders Nos. 6 and 7 refer to firearms and
therefore have no relevance to P.D. 9(3) which refers to blunt or bladed weapons. With respect to
Proclamation 1081 some of the underlying reasons for its issuance are quoted hereunder:

"WHEREAS, these lawless elements having taken up arms against our duly constituted government and
against our people, and having committed and are still committing acts of armed insurrection and
rebellion consisting of armed raids, forays, sorties, ambushes, wanton acts of murders, spoilage,
plunder, looting, arsons, destruction of public and private buildings, and attacks against innocent and
defenseless civilian lives and property, all of which activities have seriously endangered and continue to
endanger public order and safety and the security of the nation, . . . ."

129
"xxx xxx xxx

"WHEREAS, it is evident that there is throughout the land a state of anarchy and lawlessness, chaos and
disorder, turmoil and destruction of a magnitude equivalent to an actual war between the forces of our
duly constituted government and the New People's Army and their satellite organizations because of the
unmitigated forays, raids, ambuscades, assaults, violence, murders, assassinations, acts of terror,
deceits, coercions, threats, intimidations, treachery, machinations, arsons, plunders and depredations
committed and being committed by the aforesaid lawless elements who have pledged to the whole
nation that they will not stop their dastardly effort and scheme until and unless they have fully attained
their primary and ultimate purpose of forcibly seizing political and state power in this country by
overthrowing our present duly constituted government, . . . ." (See Book I, Vital Documents on the
Declaration of Martial Law in the Philippines by the Supreme Court of the Philippines, pp. 13-39).

It follows that it is only that act of carrying a blunt or bladed weapon with a motivation connected with
or related to the afore-quoted desired result of Proclamation 1081 that is within the intent of P.D. 9(3),
and nothing else.

"Statutes are to be construed in the light of purposes to be achieved and the evils sought to be
remedied." (U.S. v. American Tracking Association, 310 U.S. 534, cited in LVN Pictures v. Philippine
Musicians Guild, 110 Phil. 725, 731; emphasis supplied)

"When construing a statute, the reason for its enactment should be kept in mind, and the statute should
be construed with reference to its intended scope and purpose." (Statutory Construction by E.T.
Crawford, pp. 604-605, cited in Commissioner of Internal Revenue v. Filipinas Compañia de Seguros, 107
Phil. 1055, 1060; emphasis supplied)

5. In the construction of P.D. 9(3) it becomes relevant to inquire into the consequences of the
measure if a strict adherence to the letter of the paragraph is followed. LLjur

It is a salutary principle in statutory construction that there exists a valid presumption that undesirable
consequences were never intended by a legislative measure, and that a construction of which the
statute is fairly susceptible is favored, which will avoid all objectionable, mischievous, indefensible,
wrongful, evil, and injurious consequences. 9a

It is to be presumed that when P.D. 9 was promulgated by the President of the Republic there was no
intent to work a hardship or an oppressive result, a possible abuse of authority or act of oppression,
arming one person with a weapon to impose hardship on another, and so on. 10

At this instance We quote from the order of Judge Purisima the following:

"And while there is no proof of it before the Court, it is not difficult to believe the murmurings of
detained persons brought to Court upon a charge of possession of bladed weapons under P.D. No. 9,
that more than ever before, policemen — of course not all can be so heartless — now have in their
hands P.D. No. 9 as a most convenient tool for extortion, what with the terrifying risk of being sentenced
to imprisonment of five to ten years for a rusted kitchen knife or a pair of scissors, which only God

130
knows where it came from. Whereas before martial law an extortion-minded peace officer had to have a
stock of the cheapest paltik, and even that could only convey the coercive message of one year in jail,
now anything that has the semblance of a sharp edge or pointed object, available even in trash cans,
may already serve the same purpose, and yet five to ten times more incriminating than the infamous
paltik." (pp. 72-73, rollo L-42050-66)

And as respondent Judge Maceren points out, the people's interpretation of P.D. 9(3) results in
absurdity at times. To his example We may add a situation where a law-abiding citizen, a lawyer by
profession, after gardening in his house remembers to return the bolo used by him to his neighbor who
lives about 30 meters or so away and while crossing the street meets a policeman. The latter upon
seeing the bolo being carried by that citizen places him under arrest and books him for a violation of
P.D. 9(3). Could the presidential decree have been conceived to produce such absurd, unreasonable,
and insensible results?

6. Penal statutes are to be construed strictly against the state and liberally in favor of an accused.

American jurisprudence sets down the reason for this rule to be "the tenderness of the law of the rights
of individuals; the object is to establish a certain rule by conformity to which mankind would be safe,
and the discretion of the court limited." 11 The purpose is not to enable a guilty person to escape
punishment through a technicality but to provide a precise definition of forbidden acts. 12

Our own decisions have set down the same guidelines in this manner, viz:

"Criminal statutes are to be construed strictly. No person should be brought within their terms who is
not clearly within them, nor should any act be pronounced criminal which is not made clearly so by the
statute." (U.S. v. Abad Santos, 36 Phil. 243, 246).

"The rule that penal statutes are given a strict construction is not the only factor controlling the
interpretation of such laws, instead, the rule merely serves as an additional, single factor to be
considered as an aid in determining the meaning of penal laws." (People v. Manantan, 5 SCRA 684, 692).

F. The Informations filed by petitioner are fatally defective.

The two elements of the offense covered by P.D. 9(3) must be alleged in the Information in order that
the latter may constitute a sufficiently valid charged. The sufficiency of an Information is determined
solely by the facts alleged therein. 13 Where the facts are incomplete and do not convey the elements
of the crime, the quashing of the accusation is in order.

Section 2(a), Rule 117 of the Rules of Court provides that the defendant may move to quash the
complaint or information when the facts charged do not constitute an offense.

In U.S. v. Gacutan, 1914, it was held that where an accused is charged with knowingly rendering an
unjust judgment under Article 204 of the Revised Penal Code, failure to allege in the Information that
the judgment was rendered knowing it to be unjust, is fatal. 14

131
In People v. Yadao, 1954, this Court through then Justice Cesar Bengzon who later became Chief Justice
of the Court affirmed an order of the trial court which quashed an Information wherein the facts recited
did not constitute a public offense as defined in Section 1, Republic Act 145. 15

G — The filing of these Petitions was unnecessary because the People could have availed itself of other
available remedies below.

Pertinent provisions of the Rules of Court follow:

"RULE 117, SECTION 7. Effect of sustaining the motion to quash. — If the motion to quash is sustained
the court may order that another information be filed. If such order is made the defendant, if in custody,
shall remain so unless he shall be admitted to bail. If such order is not made or if having been made
another information is not filed within the time to be specified in the order, or within such further time
as the court may allow for good cause shown, the defendant, if in custody, shall be discharged
therefrom, unless he is in custody on some other charge."

"RULE 110, SECTION 13. Amendment. — The information or complaint may be amended, in substance or
form, without leave of court, at any time before the defendant pleads; and thereafter and during the
trial as to all matters of form, by leave and at the discretion of the court, when the same can be done
without prejudice to the rights of the defendant."

xxx xxx xxx

Two courses of action were open to Petitioner upon the quashing of the Informations in these cases, viz:

First, if the evidence on hand so warranted, the People could have filed an amended Information to
include the second element of the offense as defined in the disputed orders of respondent Judges. We
have ruled that if the facts alleged in the Information do not constitute a punishable offense, the case
should not be dismissed but the prosecution should be given an opportunity to amend the Information.
16

Second, if the facts so justified, the People could have filed a complaint either under Section 26 of Act
No. 1780, quoted earlier, or Manila City Ordinance No. 3820, as amended by Ordinance No. 3928,
especially since in most if not all of the cases, the dismissal was made prior to arraignment of the
accused and on a motion to quash. prcd

Section 8. Rule 117 states that:

"An order sustaining the motion to quash is not a bar to another prosecution for the same offense
unless the motion was based on the grounds specified in section 2, subsections (f) and (h) of this rule."

Under the foregoing, the filing of another complaint or Information is barred only when the criminal
action or liability had been extinguished (Section 2[f]) or when the motion to quash was granted for
reasons of double jeopardy. (ibid., [h]).

132
As to whether or not a plea of double jeopardy may be successfully invoked by the accused in all these
cases should new complaints be filed against them, is a matter We need not resolve for the present.

H — We conclude with high expectations that police authorities and the prosecuting arm of the
government true to the oath of office they have taken will exercise utmost circumspection and good
faith in evaluating the particular circumstances of a case so as to reach a fair and just conclusion if a
situation falls within the purview of P.D. 9(3) and the prosecution under said decree is warranted and
justified. This obligation becomes a sacred duty in the face of the severe penalty imposed for the
offense.

On this point, We commend the Chief State Prosecutor Rodolfo A. Nocon on his letter to the City Fiscal
of Manila on October 15, 1975, written for the Secretary, now Minister of Justice, where he stated the
following:

"In any case, please study well each and every case of this nature so that persons accused of carrying
bladed weapons, specially those whose purpose is not to subvert the duly constituted authorities, may
not be unduly indicted for the serious offenses falling under P.D. No. 9." 17

Yes, while it is not within the power of courts of justice to inquire into the wisdom of a law, it is however
a judicial task and prerogative to determine if official action is within the spirit and letter of the law and
if basic fundamental rights of an individual guaranteed by the Constitution are not violated in the
process of its implementation. We have to face the fact that it is an unwise and unjust application of a
law, necessary and justified under prevailing circumstances, which renders the measure an instrument
of oppression and evil and leads the citizenry to lose their faith in their government.

WHEREFORE, We DENY these 26 Petitions for Review and We AFFIRM the Orders of respondent Judges
dismissing or quashing the Information concerned, subject however to Our observations made in the
preceding pages 23 to 25 of this Decision regarding the right of the State or Petitioner herein to file
either an amended Information under Presidential Decree No. 9, paragraph 3, or a new one under other
existing statute or city ordinance as the facts may warrant.

Without costs.

SO ORDERED.

Fernando, Teehankee, Santos, Fernandez and Guerrero, JJ ., concur.

Castro, C . J ., and Antonio, J ., concur in the result.

Aquino, J ., took no part.

Separate Opinions

BARREDO, J ., concurring:

133
[I] concur(s) with the qualification that under existing jurisprudence conviction is possible, without the
need of amending the information, for violation of other laws or ordinances on concealment of deadly
weapons.

MAKASIAR, J ., concurring:

[I] concur(s) with Justice Barredo in that under the information, the accused can be validly convicted of
violating Sec. 26 of Act No. 1780 or the city or town ordinances on carrying concealed weapons.

CONCEPCION JR., J ., concurring:

[I] concur(s) with the additional observation that accused could properly be convicted of a violation of
Act 1780 of the Philippine Commission or of the ordinance.

134
Martinez vs. Van Buskirk, 18 Phil 79 (1910)

[G.R. No. 5691. December 27, 1910.]

S. D. MARTINEZ and his wife, CARMEN ONG DE MARTINEZ, plaintiffs-appellees, vs. WILLIAM VAN
BUSKIRK, defendant-appellant.

Lionel D. Hargis for appellant.

Sanz and Oppisso for appellee.

SYLLABUS

MASTER AND SERVANT; DRIVERS OF HORSES; CUSTOM AND USAGE; NEGLIGENCE. — A coachman or
driver, who had driven the horses composing his team for a considerable time, during which the animals
has shown no disposition to become unruly, left his team as usual and was assisting in unloading the
wagon when the horses bolted and running into the plaintiffs' carriage caused personal injuries to the
plaintiff and damage to the vehicle. It was further shown that, to leave teams under like circumstances
and to assist in unloading the wagon, is the custom of drivers in the city and that the custom is
sanctioned by employers. Held: That acts, the performance of which has not proven destructive or
injurious and which have been generally acquiesced in by society for so long a time as to have ripened
into a custom, can not be held to be unreasonable or imprudent and that, under the circumstances, the
driver was not guilty of negligence in so leaving his team while assisting in unloading his wagon.

DECISION

MORELAND, J p:

The facts found by the trial court are undisputed by either party in this case. They are —

"That on the 11th day of September, 1908, the plaintiff, Carmen Ong de Martinez, was riding in a
carromata on Calle Real, district of Ermita, city of Manila, P.I., along the left-hand side of the street as
she was going, when a delivery wagon belonging to the defendant used for the purpose of
transportation of fodder by the defendant, and to which was attached a pair of horses, came along the
street in the opposite direction to that the in which said plaintiff was proceeding, and that thereupon
the driver of the said plaintiff's carromata, observing that the delivery wagon of the defendant was
coming at great speed, crowded close to the sidewalk on the left-hand side of the street and stopped, in
order to give defendant's delivery wagon an opportunity to pass by, but that instead of passing by the
defendant's wagon and horses ran into the carromata occupied by said plaintiff with her child and
overturned it, severely wounding said plaintiff by making a serious cut upon her head, and also injuring
the carromata itself and the harness upon the horse which was drawing it.

xxx xxx xxx

"These facts are not dispute, but the defendant presented evidence to the effect that the cochero, who
was driving his delivery wagon at the time the accident occurred, was a good servant and was

135
considered a safe and reliable cochero; that the delivery wagon had sent to deliver some forage at Paco
Livery Stable on Calle Herran, and that for the purpose of delivery thereof the cochero driving the team
as defendant's employee tied the driving lines of the horses to the front end of the delivery wagon and
then went back inside of the wagon for the purpose of unloading the forage to be delivered; that while
unloading the forage and in the act of carrying some of it out, another vehicle drove by, the driver of
which cracked a whip and made some other noises, which frightened the horses attached to the delivery
wagon and they ran away, and the driver was thrown from the inside of the wagon out through the rear
upon the ground and was unable to stop the horses; that the horses then ran up and on which street
they came into collision with the carromata in which the plaintiff, Carmen Ong de Martinez, was riding."

The defendant himself was not with the vehicle on the day in question.

Upon these facts the court below found the defendant guilty of negligence and gave judgment against
him for P442.50, with interest thereon at the rate of 6 per cent per annum from the 17th day of
October, 1908, and for the costs of the action. The case is before us on an appeal from that judgment.

There is no general law of negligence in the Philippine Islands except that embodied in the Civil Code.
The provisions of that code pertinent to this case are —

"Art. 1902. A person who by an act or omission causes damage to another when there is fault or
negligence shall be obliged to repair the damage so done.

"Art. 1903. The obligation imposed by preceding article is demandable, not only for personal acts
and omissions, but also for those of the persons for whom they should be responsible.

"The father, and on his death or incapacity the mother, is liable for the damages caused by the minors
who live with them.

"Guardians are liable for the damages caused by minors or incapacitated persons who are under their
authority and live with them.

"Owners of directors of an establishment or enterprise are equally liable for the damages caused by the
employees in the service of the branches in which the latter may be employed or on account of their
duties.

"The State is liable in this sense when it acts through a special agent, but not when the damages should
have been caused by the official to whom properly it pertained to do the act performed, in which case
the provisions of the preceding article shall be applicable.

"Finally, masters or directors of arts and trades are liable for the damages caused by their pupils or
apprentices while they are under their custody.

"The liability referred to in this article shall cease when the persons mentioned therein prove that they
employed all the diligence of a good father of a family to avoid the damage."

136
Passing the question whether or not an employer who has furnished a gentle and tractable team and a
trusty and capable driver is, under the last paragraph of the above provisions, liable for the negligence
of such driver in handling the team, we are of the opinion that the judgment must be reversed upon the
ground that the evidence does not disclose that the cochero was negligent.

While the law relating to negligence in this jurisdiction may possibly be some what different from that in
Anglo-Saxon countries, a question we do not now discuss, the rules under which the fact of negligence is
determined are, nevertheless, generally the same. That is to say, while the law designating the person
responsible for a negligent act may not be the same here as in many jurisdictions, the law determining
what is a negligent act is the same here, generally speaking, as elsewhere. (Supreme court of Spain, 4
December, 1903; 16 May, 1893; 27 June, 1894; 9 April, 1896; 14 March, 1901; 2 March, 1904; 7
February, 1905; 16 June, 1905; 23 June, 1905; 13 April, 1903; 7 March, 1902; 12 June, 1900; 2 March,
1907; 18 March, 1898; 3 June, 1901.)

It appears from the undisputed evidence that the horses which caused the damage were gentle and
tractable; that the cochero was experienced and capable; that he had driven one of the horses several
years and the other five or six months; that he had been in the habit, during all that time, of leaving
them in the condition in which they were left on the day of the accident; that they had never run away
up to that time and there had been, therefore, no accident due to such practice; that to leave the horses
and assist in unloading the merchandise in the manner described on the day of the accident was the
custom of all cochero who delivered merchandise of the character of that which was being delivered by
the cochero of the defendant on the day in question, which custom was sanctioned by their employers.

In our judgment, the cochero of the defendant was not negligent in leaving the horses in the manner
described by the evidence in this case, either under Spanish or American jurisprudence. (Lynch vs.
Nurdin, 1 Q. B., 422; Rumsey vs. Nelson, 58 Vt., 590; Drake vs. Mount, 33 N. J. L., 442; Hoboken Land and
Improvement Co. vs. Lally, 48 N. J. L., 604; Wasmer vs. D. L. & W. R. R. Co., 80 N. Y., 212.)

In the case of Hayman vs. Hewitt (Peake N. P. Cas., pt. 2, p. 170), Lord Kenyon said:

"He was performing his duty while removing the goods into the house, and, if every person who
suffered a cart to remain in the street while he took goods out of it was obliged to employ another to
look after the horses, it would be impossible for the business of the metropolis to go on.

In the case of Griggs vs. Fleckenstein (14 Minn., 81), the court said:

"The degree of care required of the plaintiff, or those in charge of his horse, at the time of the injury, is
that which would be exercised by a person of ordinary care and prudence under like circumstances. It
can not be said that the fact of leaving the horse unhitched is in itself negligence. Whether it is
negligence to leave a horse unhitched must be depend upon the disposition of the horse; whether he
was under the observation and control of some person all the time, and many other circumstances; and
is a question to be determined by the jury from the facts of each case."

137
In the case of Belles vs. Kellner (67 N. J. L., 255), it was held that it was error on the part of the trial court
to refuse to charge that "it is not negligence for the driver of a quite, gentle horse to leave him
unhitched and otherwise unattended on the side of a public highways while the driver is upon the
sidewalk loading goods on the wagon." The said court closed its opinion with these words:

"There was evidence which could have fully justified the jury in finding that the horse was quite and
gentle, and that the driver was upon the sidewalk loading goods on the wagon, at time of the alleged
injury, and that the horse had been used for years in that way without accident. The refusal of the trial
court to charge as requested left the jury free to find was verdict against the defendant, although the
jury was convinced that these facts were proven.

In the case of Southworth vs. Ry. Co. (105 Mass., 342), it was held:

"That evidence that a servant, whom traders employed to deliver goods, upon stopping with his horse
and wagon to deliver a parcel at a house from fifty to a hundred rods from a railroad crossing, left the
horse unfastened for four or five minutes while he was in the house, knowing that it was not afraid of
cars, and having used it for three or four months without ever hitching it or knowing it to start, is not
conclusive, as a matter of law, of a want of due care on his part."

The duty, a violation of which is claimed to be negligence in the respect in question, is to exercise
reasonable care and prudence. Where reasonable care is employed in doing an act not itself illegal or
inherently likely to produce damage to others, there will be no liability, although damage in fact ensues.
(Milwaukee Ry. Co. vs. Arms, 91 U. S., 489; Parrott vs. Wells, 15 Wall., 524; Brown vs. Kendall, 6 Cushing,
292; Jackson Architectural Iron Works vs. Hurlbut, 158 N. Y., 34 Westerfield vs. Levis, 43 La. An., 63;
Niosi vs. Empire Steam Laundry, 117 Cal., 257.)

The act of defendant's driver in leaving the horses in the manner proved was not unreasonable or
imprudent. Acts the performance of which has not proved destructive or injurious and which have,
therefore, been acquiesced in by society for so long a time that they have ripened into custom, can not
be held to be themselves unreasonable or imprudent. Indeed the very reason why they have been
permitted by society is that they beneficial rather than prejudicial. Accidents sometimes happen and
injuries result from the most ordinary acts of life. But such are not their natural or customary results. To
hold that, because such an act once resulted in accident or injury, the actor is necessarily negligent, is to
go far. The fact that the doctrine of res ipsa loquitur is sometimes successfully invoked in such a case,
does not in any sense militate against the reasoning presented. That maxim at most only creates a prima
facie case, and that only in the absence of proof of the circumstances under which the act complained of
was performed. It is something invoked in favor of the plaintiff before defendant's case showing the
conditions and circumstances under which the injury occurred, the creative reason for the doctrine of
res ipsa loquitur disappears. This is demonstrated by the case of Inland and Seaboard Costing Co. vs.
Tolson (139 U.S., 551), where the court said (p. 554):

". . . The whole effect of the instruction in question, as applied to the case before the jury, was that if
the steamboat, on a calm day and in smooth water, was thrown with such force against a wharf properly
built, as to tear up some of the planks of the flooring, this would be prima facie evidence of negligence

138
on the part of the defendant's agent in making the landing, unless upon the whole evidence in the case
this prima facie evidence was rebutted. As such damage to a wharf is not ordinarily done by a steamboat
under control of her officers and carefully managed by them, evidence that such damage was done in
this case was prima facie, and, if unexplained, sufficient evidence of negligence on their part, and the
jury might properly be so instructed."

There was presented in this case, and by the plaintiffs themselves, not only the fact of the runway and
the accident resulting therefrom, but also the conditions under which the runaway occurred. Those
conditions showing of themselves that the defendant's cochero was not negligent in the management of
the horse, the prima facie case in plaintiffs' favor, if any, was destroyed as soon as made.

It is a matter of common knowledge as well as proof that it is the universal practice of merchants to
deliver merchandise of the kind of that being delivered at the time of the injury, in the manner in which
that was then being delivered; and that it is the universal practice to leave the horses in the manner in
which they were left at the time of the accident. This is the custom in all cities. It has not been
productive of accidents or injuries. The public, finding itself unprejudiced by such practice, has
acquiesced for years without objection. Ought the public now, through the courts, without prior
objection or notice, to be permitted to reverse the practice of decades and thereby make culpable and
guilty one who had every reason and assurance to believe that he was acting under the sanction of the
strongest of all civil forces, the custom of a people? We think not.

The judgment is reversed, without special finding as to costs. So ordered.

Arellano, C.J., Mapa, Johnson, Carson and Trent, JJ., concur.

Separate Opinions

TORRES, J., dissenting:

I am of the opinion that the judgment should be affirmed.

139
Yao Kee vs. Sy-Gonzales, G.R. No. 55960, November 24, 1988

[G.R. No. 55960. November 24, 1988.]

YAO KEE, SZE SOOK WAH, SZE LAI CHO, and SY CHUN YEN, petitioners, vs. AIDA SY-GONZALES, MANUEL
SY, TERESITA SY-BERNABE, RODOLFO SY, and HONORABLE COURT OF APPEALS, respondents.

Montesa, Albon & Associates for petitioner.

De Lapa, Salonga, Fulgencio & De Lunas for respondents.

SYLLABUS

1. CIVIL LAW; CUSTOM, DEFINED. — Custom is defined as "a rule of conduct formed by repetition
of acts, uniformly observed (practiced) as a social rule, legally binding and obligatory" [In the Matter of
the Petition for Authority to Continue Use of the Firm Name "Ozaeta, Romulo, de Leon, Mabanta and
Reyes", July 30, 1979, 92 SCRA 3, 12].

2. ID.; ID.; MUST BE PROVED ACCORDING TO THE RULES ON EVIDENCE. — The law requires that "a
custom must be proved as a fact, according to the rules of evidence" [Article 12, Civil Code.] On this
score the Court had occasion to state that "a local custom as a source of right can not be considered by
a court of justice unless such custom is properly established by competent evidence like any other fact"
[Patriarca v. Orate, 7 Phil. 390, 395 (1907).] The same evidence, if not one of a higher degree, should be
required of a foreign custom.

3. ID.; FOREIGN MARRIAGE; HOW PROVED. — To establish a valid foreign marriage two things
must be proven, namely: (1) the existence of the foreign law as a question of fact; and (2) the alleged
foreign marriage by convincing evidence [Adong v. Cheong Seng Gee, 43 Phil. 43, 49 (1922)].

4. REMEDIAL LAW; EVIDENCE; PROOF OF WRITTEN FOREIGN LAW; TESTIMONY OF COMPETENT


WITNESS, INCLUDED. — In proving a foreign law the procedure is provided in the Rules of Court. Proof
of a written foreign law, on the other hand, is provided for under Rule 132 Section 25. The Court has
interpreted section 25 to include competent evidence like the testimony of a witness to prove the
existence of a written foreign law [Collector of Internal Revenue v. Fisher 110 Phil. 686, 700-701 (1961)
citing Willamette Iron and Steel Works v. Muzzal, 61 Phil. 471 (1935).]

5. ID.; ID.; ID.; MARRIAGE CONTRACTED PURSUANT TO A FOREIGN LAW MUST BE PROVED TO BE
RECOGNIZED; CASE AT BAR. — Accordingly, in the absence of proof of the Chinese law on marriage, it
should be presumed that it is the same as ours . . . [Wong Woo Yiu v. Vivo, G.R. No. L-21076, March 31,
1965, 13 SCRA 552, 555.] Since Yao Kee admitted in her testimony that there was no solemnizing officer
as is known here in the Philippines [See Article 56, Civil Code] when her alleged marriage to Sy Kiat was
celebrated it therefore follows that her marriage to Sy Kiat, even if true, cannot be recognized in this
jurisdiction [Wong Woo Yiu v. Vivo, supra., pp. 555-556.]

140
6. ID.; PHILIPPINE COURTS CANNOT TAKE JUDICIAL NOTICE OF FOREIGN LAWS. — Philippine courts
cannot take judicial notice of foreign laws. They must be alleged and proved as any other fact [Yam Ka
Lim v. Collector of Customs, 30 Phil. 46, 48 (1915); Fluemer v. Hix, 54 Phil. 610 (1930).]

7. ID.; EVIDENCE; PROOF OF WRITTEN FOREIGN LAW; RULING IN MEMORACION CASE, NOT
APPLICABLE TO CASE AT BAR. — The Memoracion case however is not applicable to the case at bar as
said case did not concern a foreign marriage and the issue posed was whether or not the oral testimony
of a spouse is competent evidence to prove the fact of marriage in a complaint for adultery.

8. CIVIL LAW; PERSONS; PATERNITY AND FILIATION; FAILURE TO ESTABLISH CELEBRATION OF


MARRIAGE ACCORDING TO THE LAWS OF CHINA; EFFECT ON STATUS OF CHILDREN. — Failure to
establish the marriage of Yao Kee with Sy Kiat according to the laws of China, they cannot be accorded
the status of legitimate children but only that of acknowledged natural children. Petitioners are natural
children, it appearing that at the time of their conception Yao Kee and Sy Kiat were not disqualified by
any impediment to marry one another [See Art. 269, Civil Code.] And they are acknowledged children of
the deceased because of Sy Kiat's recognition of Sze Sook Wah.

9. ID.; ID.; ID.; RECOGNITION OF NATURAL CHILDREN; ACKNOWLEDGMENT OF ONE NATURAL


CHILD BENEFITS HER SISTERS AND BROTHERS OF THE FULL BLOOD. — The acknowledgment of Sze Sook
Wah extends to Sze Lai Cho and Sy Chun Yen who are her sisters of the full blood [See Art. 271, Civil
Code].

10. ID.; ID.; ID.; COMPROMISE AGREEMENT ACKNOWLEDGING THEIR NATURAL CHILDREN AND
PROVIDING FOR THEIR SUPPORT CONSTITUTES A STATEMENT BEFORE A COURT OF RECORD. —
Compromise agreement entered into by their parents acknowledging their five (5) natural children and
providing for their support approved by the Court of First Instance constitutes a statement before a
court of record by which a child may be voluntarily acknowledged [See Art. 278, Civil Code].

11. REMEDIAL LAW; BATAS PAMBANSA BLG. 129; FUNCTIONS AND JURISDICTION OF THE JUVENILE
AND DOMESTIC RELATIONS COURTS, VESTED NOW WITH THE REGIONAL TRIAL COURT. — With the
enactment of Batas Pambansa Blg. 129, otherwise known as the Judiciary Reorganization Act of 1980,
the Juvenile and Domestic Relations Courts were abolished. Their functions and jurisdiction are now
vested with the Regional Trial Courts [See Section 19 (7), B.P. Blg. 129 and Divinagracia v. Bellosillo, G.R.
No. L-47407, August 12, 1986, 143 SCRA 356, 360]

12. ID.; ACTIONS; ISSUE ON PATERNITY AND FILIATION MAY BE PASSED UPON IN A TESTATE OR
INTESTATE PROCEEDING; REASON. — A case involving paternity and acknowledgment may be ventilated
as an incident in the intestate or testate proceeding (See Baluyot vs. Ines Luciano, L-42215, July 13,
1976). But that legal provision presupposes that such an administration proceeding is pending or
existing and has not been terminated. The reason for this rule is not only "to obviate the rendition of
conflicting rulings on the same issue by the Court of First Instance and the Juvenile and Domestic
Relations Court" [Vda. de Baluyut v. Luciano, G.R. No. L-42215, July 13, 1976, 72 SCRA 52, 63] but more
importantly to prevent multiplicity of suits.

141
DECISION

CORTES, J p:

Sy Kiat, a Chinese national, died on January 17, 1977 in Caloocan City where he was then residing,
leaving behind real and personal properties here in the Philippines worth P300,000.00 more or less.

Thereafter, Aida Sy-Gonzales, Manuel Sy, Teresita Sy-Bernabe and Rodolfo Sy filed a petition for the
grant of letters of administration docketed as Special Proceedings Case No. C-699 of the then Court of
First Instance of Rizal Branch XXXIII, Caloocan City. In said petition they alleged among others that (a)
they are the children of the deceased with Asuncion Gillego; (b) to their knowledge Sy Kiat died
intestate; (c) they do not recognize Sy Kiat's marriage to Yao Kee nor the filiation of her children to him;
and, (d) they nominate Aida Sy-Gonzales for appointment as administratrix of the intestate estate of the
deceased [Record on Appeal, pp. 4-9; Rollo, p. 107.]

The petition was opposed by Yao Kee, Sze Sook Wah, Sze Lai Cho and Sy Yun Chen who alleged that: (a)
Yao Kee is the lawful wife of Sy Kiat whom he married on January 19, 1931 in China; (b) the other
oppositors are the legitimate children of the deceased with Yao Kee; and, (c) Sze Sook Wah is the eldest
among them and is competent, willing and desirous to become the administratrix of the estate of Sy Kiat
[Record on Appeal, pp. 12-13; Rollo, p. 107.]

After hearing, the probate court, finding among others that:

(1) Sy Kiat was legally married to Yao Kee [CFI decision, pp. 12-27; Rollo, pp. 49-64;]

(2) Sze Sook Wah, Sze Lai Cho and Sze Chum Yen are the legitimate children of Yao Kee with Sy Kiat
[CFI decision, pp. 28-31; Rollo. pp. 65-68;] and,

(3) Aida Sy-Gonzales, Manuel Sy, Teresita Sy-Bernabe and Rodolfo Sy are the acknowledged
illegitimate offsprings of Sy Kiat with Asuncion Gillego [CFI decision, pp. 27-28; Rollo, pp. 64-65.]

held in favor of the oppositors (petitioners herein) and appointed Sze Sook Wah as the administratrix of
the intestate estate of the deceased [CFI decision, pp. 68-69; Rollo, pp. 106.]

On appeal the Court of Appeals rendered a decision modifying that of the probate court, the dispositive
portion of which reads:

IN VIEW OF THE FOREGOING, the decision of the lower Court is hereby MODIFIED and SET ASIDE and a
new judgment rendered as follows:

(1) Declaring petitioners Aida Sy-Gonzales, Manuel Sy, Teresita Sy-Bernabe and Rodolfo Sy
acknowledged natural children of the deceased Sy Kiat with Asuncion Gillego, an unmarried woman with
whom he lived as husband and wife without benefit of marriage for many years:

(2) Declaring oppositors Sze Sook Wah, Sze Lai Chu and Sze Chun Yen, the acknowledged natural
children of the deceased Sy Kiat with his Chinese wife Yao Kee, also known as Yui Yip, since the legality

142
of the alleged marriage of Sy Kiat to Yao Kee in China had not been proven to be valid to the laws of the
Chinese People's Republic of China (sic);

(3) Declaring the deed of sale executed by Sy Kiat on December 7, 1976 in favor of Tomas Sy
(Exhibit "G-1", English translation of Exhibit "G") of the Avenue Tractor and Diesel Parts Supply to be
valid and accordingly, said property should be excluded from the estate of the deceased Sy Kiat; and

(4) Affirming the appointment by the lower court of Sze Sook Wah as judicial administratrix of the
estate of the deceased. [CA decision, pp. 11-12; Rollo, pp. 36-37.]

From said decision both parties moved for partial reconsideration, which was however denied by
respondent court. They thus interposed their respective appeals to this Court.

Private respondents filed a petition with this Court docketed as G.R. No. 56045 entitled "Aida Sy-
Gonzales, Manuel Sy, Teresita Sy-Bernabe and Rodolfo Sy v. Court of Appeals, Yao Kee, Sze Sook Wah,
Sze Lai Cho and Sy Chun Yen" questioning paragraphs (3) and (4) of the dispositive portion of the Court
of Appeals' decision. The Supreme Court however resolved to deny the petition and the motion for
reconsideration. Thus on March 8, 1982 entry of judgment was made in G.R. No. 56045. *

The instant petition, on the other hand, questions paragraphs (1) and (2) of the dispositive portion of
the decision of the Court of Appeals. This petition was initially denied by the Supreme Court on June 22,
1981. Upon motion of the petitioners the Court in a resolution dated September 16, 1981 reconsidered
the denial and decided to give due course to this petition.

Herein petitioners assign the following as errors:

I. RESPONDENT COURT OF APPEALS SERIOUSLY ERRED IN DECLARING THE MARRIAGE OF SY KIAT


TO YAO YEE AS NOT HAVE (sic) BEEN PROVEN VALID IN ACCORDANCE WITH LAWS OF THE PEOPLE'S
REPUBLIC OF CHINA.

II. RESPONDENT COURT OF APPEALS GRAVELY ERRED IN DECLARING AIDA SY-GONZALES, MANUEL
SY, TERESITA SY-BERNABE AND RODOLFO SY AS NATURAL CHILDREN OF SY KIAT WITH ASUNCION
GILLEGO. [Petition, p. 2; Rollo, p. 6.]

I. Petitioners argue that the marriage of Sy Kiat to Yao Kee in accordance with Chinese law and
custom was conclusively proven. To buttress this argument they rely on the following testimonial and
documentary evidence.

First, the testimony of Yao Kee summarized by the trial court as follows:

Yao Kee testified that she was married to Sy Kiat on January 19, 1931 in Fookien, China; that she does
not have a marriage certificate because the practice during that time was for elders to agree upon the
betrothal of their children, and in her case, her elder brother was the one who contracted or entered
into [an] agreement with the parents of her husband; that the agreement was that she and Sy Kiat
would be married, the wedding date was set, and invitations were sent out; that the said agreement

143
was complied with; that she has five children with Sy Kiat, but two of them died; that those who are
alive are Sze Sook Wah, Sze Lai Cho, and Sze Chun Yen, the eldest being Sze Sook Wah who is already 38
years old; that Sze Sook Wah was born on November 7, 1939; that she and her husband, Sy Kiat, have
been living in Fookien, China before he went to the Philippines on several occasions; that the practice
during the time of her marriage was a written document [is exchanged] just between the parents of the
bride and the parents of the groom, or any elder for that matter; that in China, the custom is that there
is a go-between, a sort of marriage broker who is known to both parties who would talk to the parents
of the bride-to-be; that if the parents of the bride-to-be agree to have the groom-to-be their son-in-law,
then they agree on a date as an engagement day; that on engagement day, the parents of the groom
would bring some pieces of jewelry to the parents of the bride-to-be, and then one month after that, a
date would be set for the wedding, which in her case, the wedding date to Sy Kiat was set on January 19,
1931; that during the wedding the bridegroom brings with him a couch (sic) where the bride would ride
and on that same day, the parents of the bride would give the dowry for her daughter and then the
document would be signed by the parties but there is no solemnizing officer as is known in the
Philippines; that during the wedding day, the document is signed only by the parents of the bridegroom
as well as by the parents of the bride; that the parties themselves do not sign the document; that the
bride would then be placed in a carriage where she would be brought to the town of the bridegroom
and before departure the bride would be covered with a sort of a veil; that upon reaching the town of
the bridegroom, the bridegroom takes away the veil; that during her wedding to Sy Kiat (according to
said Chinese custom), there were many persons present; that after Sy Kiat opened the door of the
carriage, two old ladies helped her go down the carriage and brought her inside the house of Sy Kiat;
that during her wedding, Sy Chiok, the eldest brother of Sy Kiat, signed the document with her mother;
that as to the whereabouts of that document, she and Sy Kiat were married for 46 years already and the
document was left in China and she doubt if that document can still be found now; that it was left in the
possession of Sy Kiat's family; that right now, she does not know the whereabouts of that document
because of the lapse of many years and because they left it in a certain place and it was already eaten by
the termites; that after her wedding with Sy Kiat, they lived immediately together as husband and wife,
and from then on, they lived together; that Sy Kiat went to the Philippines sometime in March or April in
the same year they were married; that she went to the Philippines in 1970, and then came back to
China; that again she went back to the Philippines and lived with Sy Kiat as husband and wife; that she
begot her children with Sy Kiat during the several trips by Sy Kiat made back to China. [CFI decision, pp.
13-15; Rollo, pp. 50-52.]

Second, the testimony of Gan Ching, a younger brother of Yao Kee who stated that he was among the
many people who attended the wedding of his sister with Sy Kiat and that no marriage certificate is
issued by the Chinese government, a document signed by the parents or elders of the parties being
sufficient [CFI decision, pp. 15-16; Rollo, pp. 52-53.]

Third, the statements made by Asuncion Gillego when she testified before the trial court to the effect
that (a) Sy Kiat was married to Yao Kee according to Chinese custom; and, (b) Sy Kiat's admission to her
that he has a Chinese wife whom he married according to Chinese custom [CFI decision, p. 17; Rollo, p.
54.]

144
Fourth, Sy Kiat's Master Card of Registered Alien issued in Caloocan City on October 3, 1972 where the
following entries are found: "Marital status — Married"; "If married give name of spouse — Yao Kee";
"Address — China"; "Date of marriage — 1931"; and "Place of marriage — China" [Exhibit "SS-1".]

Fifth, Sy Kiat's Alien Certificate of Registration issued in Manila on January 12, 1968 where the following
entries are likewise found: "Civil status — Married"; and, "If married, state name and address of spouse
— Yao Kee Chingkang, China" [Exhibit "4".]

And lastly, the certification issued in Manila on October 28, 1977 by the Embassy of the People's
Republic of China to the effect that "according to the information available at the Embassy Mr. Sy Kiat a
Chinese national and Mrs. Yao Kee alias Yui Yip also Chinese were married on January 19, 1931 in
Fukien, the People's Republic of China" [Exhibit "5".]

These evidence may very well prove the fact of marriage between Yao Kee and Sy Kiat. However, the
same do not suffice to establish the validity of said marriage in accordance with Chinese law or custom.

Custom is defined as "a rule of conduct formed by repetition of acts, uniformly observed (practiced) as a
social rule, legally binding and obligatory" [In the Matter of the Petition for Authority to Continue Use of
the Firm Name "Ozaeta, Romulo, de Leon, Mabanta and Reyes", July 30, 1979, SCRA 3, 12 citing JBL
Reyes & RC Puno, Outline of Phil. Civil Law, Fourth Ed. Vol. 1, p. 7.] The law requires that "a custom must
be proved as a fact, according to the rules of evidence" [Article 12, Civil Code.] On this score the Court
had occasion to state that "a local custom as a source of right can not be considered by a court of justice
unless such custom is properly established by competent evidence like any other fact" [Patriarca v.
Orate, 7 Phil. 390, 395 (1907).] The same evidence, if not one of a higher degree, should be required of a
foreign custom.

The law on foreign marriages is provided by Article 71 of the Civil Code which states that:

Art. 71. All marriages performed outside the Philippines in accordance with the laws in force in the
country where they were performed, and valid there as such, shall also be valid in this country, except
bigamous, polygamous, or incestuous marriages, as determined by Philippine law. (Emphasis supplied.)
**

Construing this provision of law the Court has held that to establish a valid foreign marriage two things
must be proven, namely: (1) the existence of the foreign law as a question of fact; and (2) the alleged
foreign marriage by convincing evidence [Adong v. Cheong Seng Gee, 43 Phil. 43, 49 (1922).]

In proving a foreign law the procedure is provided in the Rules of Court. With respect to an unwritten
foreign law, Rule 130 section 45 states that:

SEC. 45. Unwritten law. — The oral testimony of witnesses, skilled therein, is admissible as evidence of
the unwritten law of a foreign country, as are also printed and published books of reports of decisions of
the courts of the foreign country, if proved to be commonly admitted in such courts.

145
Proof of a written foreign law, on the other hand, is provided for under Rule 132 section 25, thus:

SEC. 25. Proof of public or official record. — An official record or an entry therein, when admissible for
any purpose, may be evidenced by an official publication thereof or by a copy attested by the officer
having the legal custody of the record, or by his deputy, and accompanied, if the record is not kept in
the Philippines, with a certificate that such officer has the custody. If the office in which the record is
kept is in a foreign country, the certificate may be made by a secretary of embassy or legation, consul
general, consul, vice consul, or consular agent or by any officer in the foreign service of the Philippines
stationed in the foreign country in which the record is kept and authenticated by the seal of his office.

The Court has interpreted section 25 to include competent evidence like the testimony of a witness to
prove the existence of a written foreign law [Collector of Internal Revenue v. Fisher, 110 Phil. 686, 700-
701 (1961) citing Willamette Iron and Steel Works v. Muzzal, 61 Phil. 471 (1935).]

In the case at bar petitioners did not present any competent evidence relative to the law and custom of
China on marriage. The testimonies of Yao and Gan Ching cannot be considered as proof of China's law
or custom on marriage not only because they are self-serving evidence, but more importantly, there is
no showing that they are competent to testify on the subject matter. For failure to prove the foreign law
or custom, and consequently, the validity of the marriage in accordance with said law or custom, the
marriage between Yao Kee and Sy Kiat cannot be recognized in this jurisdiction.

Petitioners contend that contrary to the Court of Appeals' ruling they are not duty bound to prove the
Chinese law on marriage as judicial notice thereof had been taken by this Court in the case of Sy Joc
Lieng v. Sy Quia [16 Phil. 137 (1910).]

This contention is erroneous. Well-established in this jurisdiction is the principle that Philippine courts
cannot take judicial notice of foreign laws. They must be alleged and proved as any other fact [Yam Ka
Lim v. Collector of Customs, 30 Phil. 46, 48 (1915); Fluemer v. Hix, 54 Phil. 610 (1930).]

Moreover a reading of said case would show that the party alleging the foreign marriage presented a
witness, one Li Ung Bieng, to prove that matrimonial letters mutually exchanged by the contracting
parties constitute the essential requisite for a marriage to be considered duly solemnized in China.
Based on his testimony, which as found by the Court is uniformly corroborated by authors on the
subject of Chinese marriage, what was left to be decided was the issue of whether or not the fact of
marriage in accordance with Chinese law was duly proven [Sy Joc Lieng v. Sy Quia, supra., at p. 160.]

Further, even assuming for the sake of argument that the Court has indeed taken judicial notice of the
law of China on marriage in the aforecited case, petitioners however have not shown any proof that the
Chinese law or custom obtaining at the time the Sy Joc Lieng marriage was celebrated in 1847 was still
the law when the alleged marriage of Sy Kiat to Yao Kee took place in 1931 or eighty-four (84) years
later.

Petitioners moreover cite the case of U.S. v. Memoracion [34 Phil. 633 (1916)] as being applicable to the
instant case. They aver that the judicial pronouncement in the Memoracion case, that the testimony of

146
one of the contracting parties is competent evidence to show the fact of marriage, holds true in this
case.

The Memoracion case however is not applicable to the case at bar as said case did not concern a foreign
marriage and the issue posed was whether or not the oral testimony of a spouse is competent evidence
to prove the fact of marriage in a complaint for adultery.

Accordingly, in the absence of proof of the Chinese law on marriage, it should be presumed that it is the
same as ours *** [Wong Woo Yiu v. Vivo, G.R. No. L-21076, March 31, 1965, 13 SCRA 552, 555.] Since
Yao Kee admitted in her testimony that there was no solemnizing officer as is known here in the
Philippines [See Article 56, Civil Code] when her alleged marriage to Sy Kiat was celebrated [CFI decision,
p. 14; Rollo, p. 51], it therefore follows that her marriage to Sy Kiat, even if true, cannot be recognized in
this jurisdiction [Wong Woo Yiu v. Vivo, supra., pp. 555-556.]

II. The second issue raised by petitioners concerns the status of private respondents.

Respondent court found the following evidence of petitioners' filiation:

(1) Sy Kiat's Master Card of Registered Alien where the following are entered: "Children if any: give
number of children — Four"; and, "Name — All living in China" [Exhibit "SS-1";]

(2) the testimony of their mother Yao Kee who stated that she had five children with Sy Kiat, only
three of whom are alive namely, Sze Sook Wah Sze Lai Chu and Sze Chin Yan [TSN, December 12, 1977,
pp. 9-11;] and,

(3) an affidavit executed on March 22, 1961 by Sy Kiat for presentation to the Local Civil Registrar of
Manila to support Sze Sook Wah's application for a marriage license, wherein Sy Kiat expressly stated
that she is his daughter [Exhibit "3".]

Likewise on the record is the testimony of Asuncion Gillego that Sy Kiat told her he has three daughters
with his Chinese wife, two of whom — Sook Wah and Sze Kai Cho — she knows, and one adopted son
[TSN, December 6, 1977, pp. 87-88.]

However, as petitioners failed to establish the marriage of Yao Kee with Sy Kiat according to the laws of
China, they cannot be accorded the status of legitimate children but only that of acknowledged natural
children. Petitioners are natural children, it appearing that at the time of their conception Yao Kee and
Sy Kiat were not disqualified by any impediment to marry one another [See Art. 269, Civil Code.] And
they are acknowledged children of the deceased because of Sy Kiat's recognition of Sze Sook Wah
[Exhibit "3"] and its extension to Sze Lai Cho and Sy Chun Yen who are her sisters of the full blood [See
Art. 271, Civil Code.]

Private respondents on the other hand are also the deceased's acknowledged natural children with
Asuncion Gillego, a Filipina with whom he lived for twenty-five (25) years without the benefit of
marriage. They have in their favor their father's acknowledgment, evidenced by a compromise
agreement entered into by and between their parents and approved by the Court of First Instance on

147
February 12, 1974 wherein Sy Kiat not only acknowledged them as his children by Asuncion Gillego but
likewise made provisions for their support and future inheritance, thus:

xxx xxx xxx

2. The parties also acknowledge that they are common-law husband and wife and that out of such
relationship, which they have likewise decided to definitely and finally terminate effective immediately,
they begot five children, namely: Aida Sy, born on May 30, 1950; Manuel Sy, born on July 1, 1953;
Teresita Sy, born on January 28, 1955; Ricardo Sy now deceased, born on December 14, 1956; and
Rodolfo Sy, born on May 7, 1958.

3. With respect to the AVENUE TRACTOR AND DIESEL PARTS SUPPLY . . ., the parties mutually
agree and covenant that —

(a) The stocks and merchandise and the furniture and equipments . . ., shall be divided into two
equal shares between, and distributed to, Sy Kiat who shall own one-half of the total and the other half
to Asuncion Gillego who shall transfer the same to their children, namely, Aida Sy, Manuel Sy, Teresita
Sy, and Rodolfo Sy.

(b) the business name and premises . . . shall be retained by Sy Kiat. However, it shall be his
obligation to give to the aforenamed children an amount of One Thousand Pesos (P1,000;00) monthly
out of the rental of the two doors of the same building now occupied by Everett Construction.

xxx xxx xxx

(5) With respect to the acquisition, during the existence of the common-law husband-and-wife
relationship between the parties, of the real estates and properties registered and/or appearing in the
name of Asuncion Gillego . . ., the parties mutually agree and covenant that the said real estates and
properties shall be transferred in equal shares to their children, namely, Aida Sy, Manuel Sy, Teresita Sy,
and Rodolfo Sy, but to be administered by Asuncion Gillego during her lifetime. . . . [Exhibit "D".]
(Emphasis supplied.)

xxx xxx xxx

This compromise agreement constitutes a statement before a court of record by which a child may be
voluntarily acknowledged [See Art. 278, Civil Code.]

Petitioners further argue that the questions on the validity of Sy Kiat's marriage to Yao Kee and the
paternity and filiation of the parties should have been ventilated in the Juvenile and Domestic Relations
Court.

Specifically, petitioners rely on the following provision of Republic Act No. 5502, entitled "An Act
Revising Rep. Act No. 3278, otherwise known as the Charter of the City of Caloocan"; with regard to the
Juvenile and Domestic Relations Court:

SEC. 91-A. Creation and Jurisdiction of the Court. —

148
xxx xxx xxx

The provisions of the Judiciary Act to the contrary notwithstanding, the court shall have exclusive
original jurisdiction to hear and decide the following cases;

xxx xxx xxx

(2) Cases involving custody, guardianship, adoption, revocation of adoption, paternity and
acknowledgment;

(3) Annulment of marriages, relief from marital obligations legal separation of spouses, and actions
for support;

(4) Proceedings brought under the provisions of title six and title seven, chapters one to three of
the civil code;

xxx xxx xxx

and the ruling in the case of Bartolome v. Bartolome [G.R. No. L-23661, 21 SCRA 1324] reiterated in
Divinagracia v. Rovira [G.R. No. L-42615, 72 SCRA 307.]

With the enactment of Batas Pambansa Blg. 129, otherwise known as the Judiciary Reorganization Act of
1980, the Juvenile and Domestic Relations Courts were abolished. Their functions and jurisdiction are
now vested with the Regional Trial Courts [See Section 19 (7), B.P. Blg. 129 and Divinagracia v. Bellosillo,
G.R. No. L-47407, August 12, 1986, 143 SCRA 356, 360] hence it is no longer necessary to pass upon the
issue of jurisdiction raised by petitioners.

Moreover, even without the enactment of Batas Pambansa Blg. 129 we find in Rep. Act No. 5502 sec.
91-A last paragraph that:

xxx xxx xxx

If any question involving any of the above matters should arise as an incident in any case pending in the
ordinary court, said incident shall be determined in the main case.

xxx xxx xxx

As held in the case of Divinagracia v. Rovira [G.R. No. L-42615. August 10, 1976, 72 SCRA 307]:

xxx xxx xxx

It is true that under the aforequoted section 1 of Republic Act No. 4834 **** a case involving paternity
and acknowledgment may be ventilated as an incident in the intestate or testate proceeding (See
Baluyot vs. Ines Luciano, L-42215, July 13, 1976). But that legal provision presupposes that such an
administration proceeding is pending or existing and has not been terminated. [at pp. 313-314.]
(Emphasis supplied.)

149
xxx xxx xxx

The reason for this rule is not only "to obviate the rendition of conflicting rulings on the same issue by
the Court of First Instance and the Juvenile and Domestic Relations Court" [Vda. de Baluyut v. Luciano,
G.R. No. L-42215, July 13, 1976, 72 SCRA 52, 63] but more importantly to prevent multiplicity of suits.

Accordingly, this Court finds no reversible error committed by respondent court.

WHEREFORE, the decision of the Court of Appeals is hereby AFFIRMED.

SO ORDERED.

Fernan, C .J ., Gutierrez, Jr., Feliciano and Bidin, JJ ., concur.

Footnotes

* The petition for review in G.R. No. 56045 was denied for lack of merit on March 9, 1981. Counsel
for the petitioners then filed a Motion for Consolidation and for Extension of Time to File Motion for
Reconsideration which was granted on July 8, 1981. On February 17, 1982, however, petitioners' motion
for reconsideration of the resolution of March 9, 1981 was denied.

** Other than the exceptions contained in this article, this provision of law is derived from Section
19, Act No. 3613 and Section IV, General Order No. 68.

*** The presumption that, in the absence of proof, the foreign law is the same as the law of the
forum, is known as processual presumption which has been applied by this Court in the cases of Lim v.
The Insular Collector of Customs, 36 Phil. 472 (1917); International Harvester Co. in Russia v. Hamburg-
American Line, 42 Phil. 845 (1918); Miciano v. Brimo, 50 Phil. 867 (1924); and Rayray v. Chae Kyung Lee,
G.R. No. L-18176, October 26, 1966, 18 SCRA 450.

**** Rep. Act 4834 created the Juvenile and Domestic Relations Court of Iloilo. Section 1 of said Act is
the exact copy of section 19-A of Rep. Act 5502.

150
CIR vs. Primetown, G.R. 162155, August 28, 2007

[G.R. No. 162155. August 28, 2007.]

COMMISSIONER OF INTERNAL REVENUE and ARTURO V. PARCERO in his official capacity as Revenue
District Officer of Revenue District No. 049 (Makati), petitioners, vs. PRIMETOWN PROPERTY GROUP,
INC., respondent.

DECISION

CORONA, J p:

This petition for review on certiorari 1 seeks to set aside the August 1, 2003 decision 2 of the Court of
Appeals (CA) in CA-G.R. SP No. 64782 and its February 9, 2004 resolution denying reconsideration. 3
AEDcIH

On March 11, 1999, Gilbert Yap, vice chair of respondent Primetown Property Group, Inc., applied for
the refund or credit of income tax respondent paid in 1997. In Yap's letter to petitioner revenue district
officer Arturo V. Parcero of Revenue District No. 049 (Makati) of the Bureau of Internal Revenue (BIR), 4
he explained that the increase in the cost of labor and materials and difficulty in obtaining financing for
projects and collecting receivables caused the real estate industry to slowdown. 5 As a consequence,
while business was good during the first quarter of 1997, respondent suffered losses amounting to
P71,879,228 that year. 6

According to Yap, because respondent suffered losses, it was not liable for income taxes. 7 Nevertheless,
respondent paid its quarterly corporate income tax and remitted creditable withholding tax from real
estate sales to the BIR in the total amount of P26,318,398.32. 8 Therefore, respondent was entitled to
tax refund or tax credit. 9

On May 13, 1999, revenue officer Elizabeth Y. Santos required respondent to submit additional
documents to support its claim. 10 Respondent complied but its claim was not acted upon. Thus, on
April 14, 2000, it filed a petition for review 11 in the Court of Tax Appeals (CTA). CSIDTc

On December 15, 2000, the CTA dismissed the petition as it was filed beyond the two-year prescriptive
period for filing a judicial claim for tax refund or tax credit. 12 It invoked Section 229 of the National
Internal Revenue Code (NIRC):

Sec. 229. Recovery of Taxes Erroneously or Illegally Collected. — No suit or proceeding shall be
maintained in any court for the recovery of any national internal revenue tax hereafter alleged to have
been erroneously or illegally assessed or collected, or of any penalty claimed to have been collected
without authority, or of any sum alleged to have been excessively or in any manner wrongfully collected,
until a claim for refund or credit has been duly filed with the Commissioner; but such suit or proceeding
may be maintained, whether or not such tax, penalty, or sum has been paid under protest or duress.
ASIETa

151
In any case, no such suit or proceeding shall be filed after the expiration of two (2) years from the date
of payment of the tax or penalty regardless of any supervening cause that may arise after payment:
Provided, however, That the Commissioner may, even without a claim therefor, refund or credit any tax,
where on the face of the return upon which payment was made, such payment appears clearly to have
been erroneously paid. (emphasis supplied)

The CTA found that respondent filed its final adjusted return on April 14, 1998. Thus, its right to claim a
refund or credit commenced on that date. 13

The tax court applied Article 13 of the Civil Code which states:

Art. 13. When the law speaks of years, months, days or nights, it shall be understood that years are of
three hundred sixty-five days each; months, of thirty days; days, of twenty-four hours, and nights from
sunset to sunrise.

If the months are designated by their name, they shall be computed by the number of days which they
respectively have. CSHcDT

In computing a period, the first day shall be excluded, and the last included. (emphasis supplied)

Thus, according to the CTA, the two-year prescriptive period under Section 229 of the NIRC for the filing
of judicial claims was equivalent to 730 days. Because the year 2000 was a leap year, respondent's
petition, which was filed 731 days 14 after respondent filed its final adjusted return, was filed beyond
the reglementary period. 15

Respondent moved for reconsideration but it was denied. 16 Hence, it filed an appeal in the CA. 17

On August 1, 2003, the CA reversed and set aside the decision of the CTA. 18 It ruled that Article 13 of
the Civil Code did not distinguish between a regular year and a leap year. According to the CA:

The rule that a year has 365 days applies, notwithstanding the fact that a particular year is a leap year.
19 SIaHTD

In other words, even if the year 2000 was a leap year, the periods covered by April 15, 1998 to April 14,
1999 and April 15, 1999 to April 14, 2000 should still be counted as 365 days each or a total of 730 days.
A statute which is clear and explicit shall be neither interpreted nor construed. 20

Petitioners moved for reconsideration but it was denied. 21 Thus, this appeal.

Petitioners contend that tax refunds, being in the nature of an exemption, should be strictly construed
against claimants. 22 Section 229 of the NIRC should be strictly applied against respondent inasmuch as
it has been consistently held that the prescriptive period (for the filing of tax refunds and tax credits)
begins to run on the day claimants file their final adjusted returns. 23 Hence, the claim should have been
filed on or before April 13, 2000 or within 730 days, reckoned from the time respondent filed its final
adjusted return. IECAaD

152
The conclusion of the CA that respondent filed its petition for review in the CTA within the two-year
prescriptive period provided in Section 229 of the NIRC is correct. Its basis, however, is not.

The rule is that the two-year prescriptive period is reckoned from the filing of the final adjusted return.
24 But how should the two-year prescriptive period be computed?

As already quoted, Article 13 of the Civil Code provides that when the law speaks of a year, it is
understood to be equivalent to 365 days. In National Marketing Corporation v. Tecson, 25 we ruled that
a year is equivalent to 365 days regardless of whether it is a regular year or a leap year. 26

However, in 1987, EO 27 292 or the Administrative Code of 1987 was enacted. Section 31, Chapter VIII,
Book I thereof provides:

Sec. 31. Legal Periods. — "Year" shall be understood to be twelve calendar months; "month" of thirty
days, unless it refers to a specific calendar month in which case it shall be computed according to the
number of days the specific month contains; "day", to a day of twenty-four hours and; "night" from
sunrise to sunset. (emphasis supplied) CcAESI

A calendar month is "a month designated in the calendar without regard to the number of days it may
contain." 28 It is the "period of time running from the beginning of a certain numbered day up to, but
not including, the corresponding numbered day of the next month, and if there is not a sufficient
number of days in the next month, then up to and including the last day of that month." 29 To illustrate,
one calendar month from December 31, 2007 will be from January 1, 2008 to January 31, 2008; one
calendar month from January 31, 2008 will be from February 1, 2008 until February 29, 2008. 30

A law may be repealed expressly (by a categorical declaration that the law is revoked and abrogated by
another) or impliedly (when the provisions of a more recent law cannot be reasonably reconciled with
the previous one). 31 Section 27, Book VII (Final Provisions) of the Administrative Code of 1987 states:

Sec. 27. Repealing clause. — All laws, decrees, orders, rules and regulation, or portions thereof,
inconsistent with this Code are hereby repealed or modified accordingly. DAaHET

A repealing clause like Sec. 27 above is not an express repealing clause because it fails to identify or
designate the laws to be abolished. 32 Thus, the provision above only impliedly repealed all laws
inconsistent with the Administrative Code of 1987.

Implied repeals, however, are not favored. An implied repeal must have been clearly and unmistakably
intended by the legislature. The test is whether the subsequent law encompasses entirely the subject
matter of the former law and they cannot be logically or reasonably reconciled. 33

Both Article 13 of the Civil Code and Section 31, Chapter VIII, Book I of the Administrative Code of 1987
deal with the same subject matter — the computation of legal periods. Under the Civil Code, a year is
equivalent to 365 days whether it be a regular year or a leap year. Under the Administrative Code of
1987, however, a year is composed of 12 calendar months. Needless to state, under the Administrative
Code of 1987, the number of days is irrelevant.

153
There obviously exists a manifest incompatibility in the manner of computing legal periods under the
Civil Code and the Administrative Code of 1987. For this reason, we hold that Section 31, Chapter VIII,
Book I of the Administrative Code of 1987, being the more recent law, governs the computation of legal
periods. Lex posteriori derogat priori. TIaCAc

Applying Section 31, Chapter VIII, Book I of the Administrative Code of 1987 to this case, the two-year
prescriptive period (reckoned from the time respondent filed its final adjusted return 34 on April 14,
1998) consisted of 24 calendar months, computed as follows:

Year 1 1st calendar month April 15, 1998 to May 14, 1998

2nd calendar month May 15, 1998 to June 14, 1998

3rd calendar month June 15, 1998 to July 14, 1998

4th calendar month July 15, 1998 to August 14, 1998

5th calendar month August 15, 1998 to September 14, 1998

6th calendar month September 15, 1998 to October 14, 1998

7th calendar month October 15, 1998 to November 14, 1998

8th calendar month November 15, 1998 to December 14, 1998

9th calendar month December 15, 1998 to January 14, 1999

10th calendar month January 15, 1999 to February 14, 1999

11th calendar month February 15, 1999 to March 14, 1999

12th calendar month March 15, 1999 to April 14, 1999

Year 2 13th calendar month April 15, 1999 to May 14, 1999

14th calendar month May 15, 1999 to June 14, 1999

15th calendar month June 15, 1999 to July 14, 1999

16th calendar month July 15, 1999 to August 14, 1999

17th calendar month August 15, 1999 to September 14, 1999

18th calendar month September 15, 1999 to October 14, 1999

19th calendar month October 15, 1999 to November 14, 1999

20th calendar month November 15, 1999 to December 14, 1999

154
21st calendar month December 15, 1999 to January 14, 2000

22nd calendar month January 15, 2000 to February 14, 2000

23rd calendar month February 15, 2000 to March 14, 2000

24th calendar month March 15, 2000 to April 14, 2000

We therefore hold that respondent's petition (filed on April 14, 2000) was filed on the last day of the
24th calendar month from the day respondent filed its final adjusted return. Hence, it was filed within
the reglementary period.

Accordingly, the petition is hereby DENIED. The case is REMANDED to the Court of Tax Appeals which is
ordered to expeditiously proceed to hear C.T.A. Case No. 6113 entitled Primetown Property Group, Inc.
v. Commissioner of Internal Revenue and Arturo V. Parcero. DISHEA

No costs.

SO ORDERED.

Puno, C.J., Sandoval-Gutierrez, Azcuna and Garcia, JJ., concur.

155
Van Dorn vs. Ronillo, Jr. et al., 139 SCRA 139

[G.R. No. L-68470. October 8, 1985.]

ALICE REYES VAN DORN, petitioner, vs. HON. MANUEL V. ROMILLO, JR., as Presiding Judge of Branch CX,
Regional Trial Court of the National Capital Region Pasay City, and RICHARD UPTON, respondents.

DECISION

MELENCIO-HERRERA, J p:

In this Petition for Certiorari and Prohibition, petitioner Alice Reyes Van Dorn seeks to set aside the
Orders, dated September 15, 1983 and August 3, 1984, in Civil Case No. 1075-P, issued by respondent
Judge, which denied her Motion to Dismiss said case, and her Motion for Reconsideration of the
Dismissal Order, respectively.

The basic background facts are that petitioner is a citizen of the Philippines while private respondent is a
citizen of the United States; that they were married in Hongkong in 1972; that, after the marriage, they
established their residence in the Philippines; that they begot two children born on April 4, 1973 and
December 18, 1975, respectively; that the parties were divorced in Nevada, United States, in 1982; and
that petitioner has re-married also in Nevada, this time to Theodore Van Dorn.

Dated June 8, 1983, private respondent filed suit against petitioner in Civil Case No. 1075-P of the
Regional Trial Court, Branch CXV, in Pasay City, stating that petitioner's business in Ermita, Manila, (the
Galleon Shop, for short), is conjugal property of the parties, and asking that petitioner be ordered to
render an accounting of that business, and that private respondent be declared with right to manage
the conjugal property. Petitioner moved to dismiss the case on the ground that the cause of action is
barred by previous judgment in the divorce proceedings before the Nevada Court wherein respondent
had acknowledged that he and petitioner had "no community property" as of June 11, 1982. The Court
below denied the Motion to Dismiss in the mentioned case on the ground that the property involved is
located in the Philippines so that the Divorce Decree has no bearing in the case. The denial is now the
subject of this Certiorari proceeding.

Generally, the denial of a Motion to Dismiss in a civil case is interlocutory and is not subject to appeal.
Certiorari and Prohibition are neither the remedies to question the propriety of an interlocutory order of
the trial Court. However, when a grave abuse of discretion was patently committed, or the lower Court
acted capriciously and whimsically, then it devolves upon this Court in a certiorari proceeding to exercise
its supervisory authority and to correct the error committed which, in such a case, is equivalent to lack
of jurisdiction. 1 Prohibition would then lie since it would be useless and a waste of time to go ahead
with the proceedings. 2 We consider the petition filed in this case within the exception, and we have
given it due course.

For resolution is the effect of the foreign divorce on the parties and their alleged conjugal property in
the Philippines.

156
Petitioner contends that respondent is estopped from laying claim on the alleged conjugal property
because of the representation he made in the divorce proceedings before the American Court that they
had no community of property; that the Galleon Shop was not established through conjugal funds; and
that respondent's claim is barred by prior judgment.

For his part, respondent avers that the Divorce Decree issued by the Nevada Court cannot prevail over
the prohibitive laws of the Philippines and its declared national policy; that the acts and declaration of a
foreign Court cannot, especially if the same is contrary to public policy, divest Philippine Courts of
jurisdiction to entertain matters within its jurisdiction.

For the resolution of this case, it is not necessary to determine whether the property relations between
petitioner and private respondent, after their marriage, were upon absolute or relative community
property, upon complete separation of property, or upon any other regime. The pivotal fact in this case
is the Nevada divorce of the parties.

The Nevada District Court, which decreed the divorce, had obtained jurisdiction over petitioner who
appeared in person before the Court during the trial of the case. It also obtained jurisdiction over private
respondent who, giving his address as No. 381 Bush Street, San Francisco, California, authorized his
attorneys in the divorce case, Karp & Gradt, Ltd., to agree to the divorce on the ground of
incompatibility in the understanding that there were neither community property nor community
obligations. 3 As explicitly stated in the Power of Attorney he executed in favor of the law firm of KARP
& GRAD LTD., 336 W. Liberty, Reno, Nevada, to represent him in the divorce proceedings:

xxx xxx xxx

"You are hereby authorized to accept service of Summons, to file an Answer, appear on my behalf and
do all things necessary and proper to represent me, without further contesting, subject to the following:

"1. That my spouse seeks a divorce on the ground of incompatibility.

"2. That there is no community of property to be adjudicated by the Court.

"3. That there are no community obligations to be adjudicated by the court.

xxx xxx xxx" 4

There can be no question as to the validity of that Nevada divorce in any of the States of the United
States. The decree is binding on private respondent as an American citizen. For instance, private
respondent cannot sue petitioner, as her husband, in any State of the Union. What he is contending in
this case is that the divorce is not valid and binding in this jurisdiction, the same being contrary to local
law and public policy.

It is true that owing to the nationality principle embodied in Article 15 of the Civil Code, 5 only Philippine
nationals are covered by the policy against absolute divorces the same being considered contrary to our
concept of public policy and morality. However, aliens may obtain divorces abroad, which may be

157
recognized in the Philippines, provided they are valid according to their national law. 6 In this case, the
divorce in Nevada released private respondent from the marriage from the standards of American law,
under which divorce dissolves the marriage. As stated by the Federal Supreme Court of the United
States in Atherton vs. Atherton, 45 L. Ed. 794, 799:

"The purpose and effect of a decree of divorce from the bond of matrimony by a court of competent
jurisdiction are to change the existing status or domestic relation of husband and wife, and to free them
both from the bond. The marriage tie, when thus severed as to one party, ceases to bind either. A
husband without a wife, or a wife without a husband, is unknown to the law. When the law provides, in
the nature of a penalty, that the guilty party shall not marry again, that party, as well as the other, is still
absolutely freed from the bond of the former marriage."

Thus, pursuant to his national law, private respondent is no longer the husband of petitioner. He would
have no standing to sue in the case below as petitioner's husband entitled to exercise control over
conjugal assets. As he is bound by the Decision of his own country's Court, which validly exercised
jurisdiction over him, and whose decision he does not repudiate, he is estopped by his own
representation before said Court from asserting his right over the alleged conjugal property. cdll

To maintain, as private respondent does, that, under our laws, petitioner has to be considered still
married to private respondent and still subject to a wife's obligations under Article 109, et. seq. of the
Civil Code cannot be just. Petitioner should not be obliged to live together with, observe respect and
fidelity, and render support to private respondent. The latter should not continue to be one of her heirs
with possible rights to conjugal property. She should not be discriminated against in her own country if
the ends of justice are to be served.

WHEREFORE, the Petition is granted, and respondent Judge is hereby ordered to dismiss the Complaint
filed in Civil Case No. 1075-P of his Court.

Without costs.

SO ORDERED.

Teehankee (Chairman), Plana, Relova Gutierrez, Jr., De la Fuente and Patajo, JJ., concur.

158
Pilapil vs. Ibay-Somera 174 SCRA 653

[G.R. No. 80116. June 30, 1989.]

IMELDA MANALAYSAY PILAPIL, petitioner, vs. HON. CORONA IBAY-SOMERA, in her capacity as Presiding
Judge of the Regional Trial Court of Manila, Branch XXVI; HON. LUIS C. VICTOR, in his capacity as the City
Fiscal of Manila; and ERICH EKKEHARD GEILING, respondents.

SYLLABUS

1. REMEDIAL LAW; CRIMINAL PROCEDURE; PROSECUTION OF OFFENSES; ADULTERY AND


CONCUBINAGE; SWORN WRITTEN COMPLAINT OF OFFENDED SPOUSE, JURISDICTIONAL. — Under
Article 344 of the Revised Penal Code, the crime of adultery, as well as four other crimes against
chastity, cannot be prosecuted except upon a sworn written complaint filed by the offended spouse. It
has long since been established, with unwavering consistency, that compliance with this rule is a
jurisdictional, and not merely a formal, requirement. While in point of strict law the jurisdiction of the
court over the offense is vested in it by the Judiciary Law, the requirement for a sworn written complaint
is just as jurisdictional a mandate since it is that complaint which starts the prosecutory proceeding and
without which the court cannot exercise its jurisdiction to try the case.

2. ID.; ID.; ID.; EXCLUSIVE AND SUCCESSIVE RULE IN THE PROSECUTION OF SEDUCTION,
ABDUCTION, RAPE AND ACTS OF LASCIVIOUSNESS, NOT APPLICABLE TO CONCUBINAGE AND ADULTERY.
— Now, the law specifically provides that in prosecutions for adultery and concubinage the person who
can legally file the complaint should be the offended spouse, and nobody else. Unlike the offenses of
seduction, abduction, rape and acts of lasciviousness, no provision is made for the prosecution of the
crimes of adultery and concubinage by the parents, grandparents or guardian of the offended party. The
so-called exclusive and successive rule in the prosecution of the first four offenses above mentioned do
not apply to adultery and concubinage. It is significant that while the State, as parens patriae, was added
and vested by the 1985 Rules of Criminal Procedure with the power to initiate the criminal action for a
deceased or incapacitated victim in the aforesaid offenses of seduction, abduction, rape and acts of
lasciviousness, in default of her parents, grandparents or guardian, such amendment did not include the
crimes of adultery and concubinage. In other words, only the offended spouse, and no other, is
authorized by law to initiate the action therefor.

3. ID.; ID.; ID.; LEGAL CAPACITY TO SUE IN CIVIL CASES, DETERMINED AS OF THE FILING OF THE
COMPLAINT, APPLIED TO PROSECUTION OF CRIMINAL CASES. — Corollary to such exclusive grant of
power to the offended spouse to institute the action, it necessarily follows that such initiator must have
the status, capacity or legal representation to do so at the time of the filing of the criminal action. This is
a familiar and express rule in civil actions; in fact, lack of legal capacity to sue, as a ground for a motion
to dismiss in civil cases, is determined as of the filing of the complaint or petition. The absence of an
equivalent explicit rule in the prosecution of criminal cases does not mean that the same requirement
and rationale would not apply. Understandably, it may not have been found necessary since criminal
actions are generally and fundamentally commenced by the State, through the People of the
Philippines, the offended party being merely the complaining witness therein. However, in the so-called

159
"private crimes", or those which cannot be prosecuted de oficio, and the present prosecution for
adultery is of such genre, the offended spouse assumes a more predominant role since the right to
commence the action, or to refrain therefrom, is a matter exclusively within his power and option.

4. ID.; ID.; ID.; ID.; RATIONALE. — This policy was adopted out of consideration for the aggrieved
party who might prefer to suffer the outrage in silence rather than go through the scandal of a public
trial. Hence, as cogently argued by petitioner, Article 344 of the Revised Penal Code thus presupposes
that the marital relationship is still subsisting at the time of the institution of the criminal action for
adultery. This is a logical consequence since the raison d'etre of said provision of law would be absent
where the supposed offended party had ceased to be the spouse of the alleged offender at the time of
the filing of the criminal case.

5. ID.; ID.; ID.; ADULTERY AND CONCUBINAGE; AFTER A DIVORCE HAS BEEN DECREED, THE
INNOCENT SPOUSE NO LONGER HAS THE RIGHT TO INSTITUTE PROCEEDINGS AGAINST THE OFFENDERS.
— American jurisprudence, on cases involving statutes in that jurisdiction which are in pari materia with
ours, yields the rule that after a divorce has been decreed, the innocent spouse no longer has the right
to institute proceedings against the offenders where the statute provides that the innocent spouse shall
have the exclusive right to institute a prosecution for adultery. Where, however, proceedings have been
properly commenced, a divorce subsequently granted can have no legal effect on the prosecution of the
criminal proceedings to a conclusion.

6. ID.; ID.; ID.; ID.; U.S. RULE APPLIED IN THIS JURISDICTION. — We see no reason why the same
doctrinal rule should not apply in this case and in our jurisdiction, considering our statutory law and jural
policy on the matter. We are convinced that in cases of such nature, the status of the complainant vis-a-
vis the accused must be determined as of the time the complaint was filed. Thus, the person who
initiates the adultery case must be an offended spouse, and by this is meant that he is still married to
the accused spouse, at the time of the filing of the complaint.

7. CIVIL LAW; PERSONS AND FAMILY RELATIONS; MARRIAGE IN THE FEDERAL REPUBLIC OF
GERMANY BETWEEN A FILIPINA AND A GERMAN, RECOGNIZED IN THE PHILIPPINES. — In the present
case, the fact that private respondent obtained a valid divorce in his country, the Federal Republic of
Germany, is admitted. Said divorce and its legal effects may be recognized in the Philippines insofar as
private respondent is concerned in view of the nationality principle in our civil law on the matter of
status of persons.

8. ID.; ID.; ID.; SEVERANCE OF MATERIAL BOND HAD THE EFFECT OF DISSOCIATING THE FORMER
SPOUSES FROM EACH OTHER. — The allegation of private respondent that he could not have brought
this case before the decree of divorce for lack of knowledge, even if true, is of no legal significance or
consequence in this case. When said respondent initiated the divorce proceeding, he obviously knew
that there would no longer be a family nor marriage vows to protect once a dissolution of the marriage
is decreed. Neither would there be a danger of introducing spurious heirs into the family, which is said
to be one of the reasons for the particular formulation of our law on adultery, since there would
thenceforth be no spousal relationship to speak of. The severance of the marital bond had the effect of

160
dissociating the former spouses from each other, hence the actuations of one would not affect or cast
obloquy on the other.

9. REMEDIAL LAW; CRIMINAL PROCEDURE; PROSECUTION OF OFFENSES; RULE IN MATA CASE (18
PHIL. 4 90), NOT APPLICABLE TO CASE AT BAR. — The aforecited case of United States vs. Mata cannot
be successfully relied upon by private respondent. In applying Article 433 of the old Penal Code,
substantially the same as Article 333 of the Revised Penal Code, which punished adultery "although the
marriage be afterwards declared void", the Court merely stated that "the lawmakers intended to declare
adulterous the infidelity of a married woman to her marital vows, even though it should be made to
appear that she is entitled to have her marriage contract declared null and void, until and unless she
actually secures a formal judicial declaration to that effect". Definitely, it cannot be logically inferred
therefrom that the complaint can still be filed after the declaration of nullity because such declaration
that the marriage is void ab initio is equivalent to stating that it never existed. There being no marriage
from the beginning, any complaint for adultery filed after said declaration of nullity would no longer
have a leg to stand on. Moreover, what was consequently contemplated and within the purview of the
decision in said case is the situation where the criminal action for adultery was filed before the
termination of the marriage by a judicial declaration of its nullity ab initio. The same rule and requisite
would necessarily apply where the termination of the marriage was effected, as in this case, by a valid
foreign divorce.

DECISION

REGALADO, J p:

An ill-starred marriage of a Filipina and a foreigner which ended in a foreign absolute divorce, only to be
followed by a criminal infidelity suit of the latter against the former, provides Us the opportunity to lay
down a decisional rule on what hitherto appears to be an unresolved jurisdictional question.

On September 7, 1979, petitioner Imelda Manalaysay Pilapil, a Filipino citizen, and private respondent
Erich Ekkehard Geiling, a German national, were married before the Registrar of Births, Marriages and
Deaths at Friedensweiler in the Federal Republic of Germany. The marriage started auspiciously enough,
and the couple lived together for some time in Malate, Manila where their only child, Isabella Pilapil
Geiling, was born on April 20, 1980. 1

Thereafter, marital discord set in, with mutual recriminations between the spouses, followed by a
separation de facto between them.

After about three and a half years of marriage, such connubial disharmony eventuated in private
respondent initiating a divorce proceeding against petitioner in Germany before the Schoneberg Local
Court in January, 1983. He claimed that there was failure of their marriage and that they had been living
apart since April, 1982. 2

161
Petitioner, on the other hand, filed an action for legal separation, support and separation of property
before the Regional Trial Court of Manila, Branch XXXII, on January 23, 1983 where the same is still
pending as Civil Case No. 83-15866. 3

On January 15, 1986, Division 20 of the Schoneberg Local Court, Federal Republic of Germany,
promulgated a decree of divorce on the ground of failure of marriage of the spouses. The custody of the
child was granted to petitioner. The records show that under German law said court was locally and
internationally competent for the divorce proceeding and that the dissolution of said marriage was
legally founded on and authorized by the applicable law of that foreign jurisdiction. 4

On June 27, 1986, or more than five months after the issuance of the divorce decree, private respondent
filed two complaints for adultery before the City Fiscal of Manila alleging that, while still married to said
respondent, petitioner "had an affair with a certain William Chia as early as 1982 and with yet another
man named Jesus Chua sometime in 1983". Assistant Fiscal Jacinto A. de los Reyes, Jr., after the
corresponding investigation, recommended the dismissal of the cases on the ground of insufficiency of
evidence. 5 However, upon review, the respondent city fiscal approved a resolution, dated January 8,
1986, directing the filing of two complaints for adultery against the petitioner. 6 The complaints were
accordingly filed and were eventually raffled to two branches of the Regional Trial Court of Manila. The
case entitled "People of the Philippines vs. Imelda Pilapil and William Chia", docketed as Criminal Case
No. 87-52435, was assigned to Branch XXVI presided by the respondent judge; while the other case,
"People of the Philippines vs. Imelda Pilapil and James Chua", docketed as Criminal Case No. 87-52434
went to the sala of Judge Leonardo Cruz, Branch XXV, of the same court. 7

On March 14, 1987, petitioner filed a petition with the Secretary of Justice asking that the aforesaid
resolution of respondent fiscal be set aside and the cases against her be dismissed. 8 A similar petition
was filed by James Chua, her co-accused in Criminal Case No. 87-52434. The Secretary of Justice,
through the Chief State Prosecutor, gave due course to both petitions and directed the respondent city
fiscal to inform the Department of Justice "if the accused have already been arraigned and if not yet
arraigned, to move to defer further proceedings" and to elevate the entire records of both cases to his
office for review. 9

Petitioner thereafter filed a motion in both criminal cases to defer her arraignment and to suspend
further proceedings thereon. 10 As a consequence, Judge Leonardo Cruz suspended proceedings in
Criminal Case No. 87-52434. On the other hand, respondent judge merely reset the date of the
arraignment in Criminal Case No. 87-52435 to April 6, 1987. Before such scheduled date, petitioner
moved for the cancellation of the arraignment and for the suspension of proceedings in said Criminal
Case No. 87-52435 until after the resolution of the petition for review then pending before the Secretary
of Justice. 11 A motion to quash was also filed in the same case on the ground of lack of jurisdiction, 12
which motion was denied by the respondent judge in an order dated September 8, 1987. The same
order also directed the arraignment of both accused therein, that is, petitioner and William Chia. The
latter entered a plea of not guilty while the petitioner refused to be arraigned. Such refusal of the
petitioner being considered by respondent judge as direct contempt, she and her counsel were fined

162
and the former was ordered detained until she submitted herself for arraignment. 13 Later, private
respondent entered a plea of not guilty. 14

On October 27, 1987, petitioner filed this special civil action for certiorari and prohibition, with a prayer
for a temporary restraining order, seeking the annulment of the order of the lower court denying her
motion to quash. The petition is anchored on the main ground that the court is without jurisdiction "to
try and decide the charge of adultery, which is a private offense that cannot be prosecuted de officio
(sic), since the purported complainant, a foreigner, does not qualify as an offended spouse having
obtained a final divorce decree under his national law prior to his filing the criminal complaint." 15

On October 21, 1987, this Court issued a temporary restraining order enjoining the respondents from
implementing the aforesaid order of September 8, 1987 and from further proceeding with Criminal Case
No. 87-52435. Subsequently, on March 23, 1988 Secretary of Justice Sedfrey A. Ordoñez acted on the
aforesaid petitions for review and, upholding petitioner's ratiocinations, issued a resolution directing the
respondent city fiscal to move for the dismissal of the complaints against the petitioner. 16

We find this petition meritorious. The writs prayed for shall accordingly issue. LexLib

Under Article 344 of the Revised Penal Code, 17 the crime of adultery, as well as four other crimes
against chastity, cannot be prosecuted except upon a sworn written complaint filed by the offended
spouse. It has long since been established, with unwavering consistency, that compliance with this rule
is a jurisdictional, and not merely a formal, requirement. 18 While in point of strict law the jurisdiction of
the court over the offense is vested in it by the Judiciary Law, the requirement for a sworn written
complaint is just as jurisdictional a mandate since it is that complaint which starts the prosecutory
proceeding 19 and without which the court cannot exercise its jurisdiction to try the case.

Now, the law specifically provides that in prosecutions for adultery and concubinage the person who can
legally file the complaint should be the offended spouse, and nobody else. Unlike the offenses of
seduction, abduction, rape and acts of lasciviousness, no provision is made for the prosecution of the
crimes of adultery and concubinage by the parents, grandparents or guardian of the offended party. The
so-called exclusive and successive rule in the prosecution of the first four offenses above mentioned do
not apply to adultery and concubinage. It is significant that while the State, as parens patriae, was added
and vested by the 1985 Rules of Criminal Procedure with the power to initiate the criminal action for a
deceased or incapacitated victim in the aforesaid offenses of seduction, abduction, rape and acts of
lasciviousness, in default of her parents, grandparents or guardian, such amendment did not include the
crimes of adultery and concubinage. In other words, only the offended spouse, and no other, is
authorized by law to initiate the action therefor.

Corollary to such exclusive grant of power to the offended spouse to institute the action, it necessarily
follows that such initiator must have the status, capacity or legal representation to do so at the time of
the filing of the criminal action. This is a familiar and express rule in civil actions; in fact, lack of legal
capacity to sue, as a ground for a motion to dismiss in civil cases, is determined as of the filing of the
complaint or petition.

163
The absence of an equivalent explicit rule in the prosecution of criminal cases does not mean that the
same requirement and rationale would not apply. Understandably, it may not have been found
necessary since criminal actions are generally and fundamentally commenced by the State, through the
People of the Philippines, the offended party being merely the complaining witness therein. However, in
the so-called "private crimes", or those which cannot be prosecuted de oficio, and the present
prosecution for adultery is of such genre, the offended spouse assumes a more predominant role since
the right to commence the action, or to refrain therefrom, is a matter exclusively within his power and
option.

This policy was adopted out of consideration for the aggrieved party who might prefer to suffer the
outrage in silence rather than go through the scandal of a public trial. 20 Hence, as cogently argued by
petitioner, Article 344 of the Revised Penal Code thus presupposes that the marital relationship is still
subsisting at the time of the institution of the criminal action for adultery. This is a logical consequence
since the raison d'etre of said provision of law would be absent where the supposed offended party had
ceased to be the spouse of the alleged offender at the time of the filing of the criminal case. 21

In these cases, therefore, it is indispensable that the status and capacity of the complainant to
commence the action be definitely established and, as already demonstrated, such status or capacity
must indubitably exist as of the time he initiates the action. It would be absurd if his capacity to bring
the action would be determined by his status before or subsequent to the commencement thereof,
where such capacity or status existed prior to but ceased before, or was acquired subsequent to but did
not exist at the time of, the institution of the case. We would thereby have the anomalous spectacle of a
party bringing suit at the very time when he is without the legal capacity to do so.

To repeat, there does not appear to be any local precedential jurisprudence on the specific issue as to
when precisely the status of a complainant as an offended spouse must exist where a criminal
prosecution can be commenced only by one who in law can be categorized as possessed of such status.
Stated differently and with reference to the present case, the inquiry would be whether it is necessary in
the commencement of a criminal action for adultery that the marital bonds between the complainant
and the accused be unsevered and existing at the time of the institution of the action by the former
against the latter. cdphil

American jurisprudence, on cases involving statutes in that jurisdiction which are in pari materia with
ours, yields the rule that after a divorce has been decreed, the innocent spouse no longer has the right
to institute proceedings against the offenders where the statute provides that the innocent spouse shall
have the exclusive right to institute a prosecution for adultery. Where, however, proceedings have been
properly commenced, a divorce subsequently granted can have no legal effect on the prosecution of the
criminal proceedings to a conclusion. 22

In the cited Loftus case, the Supreme Court of Iowa held that —

" 'No prosecution for adultery can be commenced except on the complaint of the husband or wife.'
Section 4932, Code. Though Loftus was husband of defendant when the offense is said to have been
committed, he had ceased to be such when the prosecution was begun; and appellant insists that his

164
status was not such as to entitle him to make the complaint. We have repeatedly said that the offense is
against the unoffending spouse, as well as the state, in explaining the reason for this provision in the
statute; and we are of the opinion that the unoffending spouse must be such when the prosecution is
commenced." (Emphasis supplied.)

We see no reason why the same doctrinal rule should not apply in this case and in our jurisdiction,
considering our statutory law and jural policy on the matter. We are convinced that in cases of such
nature, the status of the complainant vis-a-vis the accused must be determined as of the time the
complaint was filed. Thus, the person who initiates the adultery case must be an offended spouse, and
by this is meant that he is still married to the accused spouse, at the time of the filing of the complaint.

In the present case, the fact that private respondent obtained a valid divorce in his country, the Federal
Republic of Germany, is admitted. Said divorce and its legal effects may be recognized in the Philippines
insofar as private respondent is concerned 23 in view of the nationality principle in our civil law on the
matter of status of persons.

Thus, in the recent case of Van Dorn vs. Romillo, Jr., et al., 24 after a divorce was granted by a United
States court between Alice Van Dorn, a Filipina, and her American husband, the latter filed a civil case in
a trial court here alleging that her business concern was conjugal property and praying that she be
ordered to render an accounting and that the plaintiff be granted the right to manage the business.
Rejecting his pretensions, this Court perspicuously demonstrated the error of such stance, thus:

"There can be no question as to the validity of that Nevada divorce in any of the States of the United
States. The decree is binding on private respondent as an American citizen. For instance, private
respondent cannot sue petitioner, as her husband, in any State of the Union . . .

"It is true that owing to the nationality principle embodied in Article 15 of the Civil Code, only Philippine
nationals are covered by the policy against absolute divorces the same being considered contrary to our
concept of public policy and morality. However, aliens may obtain divorces abroad, which may be
recognized in the Philippines, provided they are valid according to their national law . . .

"Thus, pursuant to his national law, private respondent is no longer the husband of petitioner. He would
have no standing to sue in the case below as petitioner's husband entitled to exercise control over
conjugal assets . . ." 25

Under the same considerations and rationale, private respondent, being no longer the husband of
petitioner, had no legal standing to commence the adultery case under the imposture that he was the
offended spouse at the time he filed suit.

The allegation of private respondent that he could not have brought this case before the decree of
divorce for lack of knowledge, even if true, is of no legal significance or consequence in this case. When
said respondent initiated the divorce proceeding, he obviously knew that there would no longer be a
family nor marriage vows to protect once a dissolution of the marriage is decreed. Neither would there
be a danger of introducing spurious heirs into the family, which is said to be one of the reasons for the

165
particular formulation of our law on adultery, 26 since there would thenceforth be no spousal
relationship to speak of. The severance of the marital bond had the effect of dissociating the former
spouses from each other, hence the actuations of one would not affect or cast obloquy on the other.

The aforecited case of United States vs. Mata cannot be successfully relied upon by private respondent.
In applying Article 433 of the old Penal Code, substantially the same as Article 333 of the Revised Penal
Code, which punished adultery "although the marriage be afterwards declared void", the Court merely
stated that "the lawmakers intended to declare adulterous the infidelity of a married woman to her
marital vows, even though it should be made to appear that she is entitled to have her marriage
contract declared null and void, until and unless she actually secures a formal judicial declaration to that
effect". Definitely, it cannot be logically inferred therefrom that the complaint can still be filed after the
declaration of nullity because such declaration that the marriage is void ab initio is equivalent to stating
that it never existed. There being no marriage from the beginning, any complaint for adultery filed after
said declaration of nullity would no longer have a leg to stand on. Moreover, what was consequently
contemplated and within the purview of the decision in said case is the situation where the criminal
action for adultery was filed before the termination of the marriage by a judicial declaration of its nullity
ab initio. The same rule and requisite would necessarily apply where the termination of the marriage
was effected, as in this case, by a valid foreign divorce.

Private respondent's invocation of Donio-Teves, et al. vs. Vamenta, herein before cited, 27 must suffer
the same fate of inapplicability. A cursory reading of said case reveals that the offended spouse therein
had duly and seasonably filed a complaint for adultery, although an issue was raised as to its sufficiency
but which was resolved in favor of the complainant. Said case did not involve a factual situation akin to
the one at bar or any issue determinative of the controversy herein.

WHEREFORE, the questioned order denying petitioner's motion to quash is SET ASIDE and another one
entered DISMISSING the complaint in Criminal Case No. 87-52435 for lack of jurisdiction. The temporary
restraining order issued in this case on October 21, 1987 is hereby made permanent.

SO ORDERED.

Melencio-Herrera, Padilla and Sarmiento, JJ., concur.

Separate Opinion

PARAS, J ., concurring:

It is my considered opinion that regardless of whether We consider the German absolute divorce as
valid also in the Philippines, the fact is that the husband in the instant case, by the very act of his
obtaining an absolute divorce in Germany can no longer be considered as the offended party in case his
former wife actually has carnal knowledge with another, because in divorcing her, he already implicitly
authorized the woman to have sexual relations with others. A contrary ruling would be less than fair for
a man, who is free to have sex will be allowed to deprive the woman of the same privilege.

166
In the case of Recto v. Harden (100 Phil. 427 [1956]), the Supreme Court considered the absolute
divorce between the American husband and his American wife as valid and binding in the Philippines on
the theory that their status and capacity are governed by their National law, namely, American law.
There is no decision yet of the Supreme Court regarding the validity of such a divorce if one of the
parties, say an American, is married to a Filipino wife, for then two (2) different nationalities would be
involved.

In the book of Senate President Jovito Salonga entitled Private International Law and precisely because
of the National law doctrine, he considers the absolute divorce as valid insofar as the American husband
is concerned but void insofar as the Filipino wife is involved. This results in what he calls a "socially
grotesque situation," where a Filipino woman is still married to a man who is no longer her husband. It is
the opinion however, of the undersigned that very likely the opposite expresses the correct view. While
under the national law of the husband the absolute divorce will be valid, still one of the exceptions to
the application of the proper foreign law (one of the exceptions to comity) is when the foreign law will
work an injustice or injury to the people or residents of the forum. Consequently since to recognize the
absolute divorce as valid on the part of the husband would be injurious or prejudicial to the Filipino wife
whose marriage would be still valid under her national law, it would seem that under our law existing
before the new Family Code (which took effect on August 3, 1988) the divorce should be considered
void both with respect to the American husband and the Filipino wife.

The recent case of Van Dorn v. Romillo, Jr.(139 SCRA [1985]) cannot apply despite the fact that the
husband was an American with a Filipino wife because in said case the validity of the divorce insofar as
the Filipino wife is concerned was NEVER put in issue.

167
San Luis v. San Luis, G.R. Nos. 133743 & 134029, February 6, 2007

[G.R. No. 133743. February 6, 2007.]

EDGAR SAN LUIS, petitioner, vs. FELICIDAD SAN LUIS, respondent.

[G.R. No. 134029. February 6, 2007]

RODOLFO SAN LUIS, petitioner, vs. FELICIDAD SAGALONGOS alias FELICIDAD SAN LUIS, respondent.

DECISION

YNARES-SANTIAGO, J p:

Before us are consolidated petitions for review assailing the February 4, 1998 Decision 1 of the Court of
Appeals in CA-G.R. CV No. 52647, which reversed and set aside the September 12, 1995 2 and January
31, 1996 3 Resolutions of the Regional Trial Court of Makati City, Branch 134 in SP. Proc. No. M-3708;
and its May 15, 1998 Resolution 4 denying petitioners' motion for reconsideration. HTDAac

The instant case involves the settlement of the estate of Felicisimo T. San Luis (Felicisimo), who was the
former governor of the Province of Laguna. During his lifetime, Felicisimo contracted three marriages.
His first marriage was with Virginia Sulit on March 17, 1942 out of which were born six children, namely:
Rodolfo, Mila, Edgar, Linda, Emilita and Manuel. On August 11, 1963, Virginia predeceased Felicisimo.

Five years later, on May 1, 1968, Felicisimo married Merry Lee Corwin, with whom he had a son, Tobias.
However, on October 15, 1971, Merry Lee, an American citizen, filed a Complaint for Divorce 5 before
the Family Court of the First Circuit, State of Hawaii, United States of America (U.S.A.), which issued a
Decree Granting Absolute Divorce and Awarding Child Custody on December 14, 1973. 6

On June 20, 1974, Felicisimo married respondent Felicidad San Luis, then surnamed Sagalongos, before
Rev. Fr. William Meyer, Minister of the United Presbyterian at Wilshire Boulevard, Los Angeles,
California, U.S.A. 7 He had no children with respondent but lived with her for 18 years from the time of
their marriage up to his death on December 18, 1992.

Thereafter, respondent sought the dissolution of their conjugal partnership assets and the settlement of
Felicisimo's estate. On December 17, 1993, she filed a petition for letters of administration 8 before the
Regional Trial Court of Makati City, docketed as SP. Proc. No. M-3708 which was raffled to Branch 146
thereof.

Respondent alleged that she is the widow of Felicisimo; that, at the time of his death, the decedent was
residing at 100 San Juanico Street, New Alabang Village, Alabang, Metro Manila; that the decedent's
surviving heirs are respondent as legal spouse, his six children by his first marriage, and son by his
second marriage; that the decedent left real properties, both conjugal and exclusive, valued at
P30,304,178.00 more or less; that the decedent does not have any unpaid debts. Respondent prayed
that the conjugal partnership assets be liquidated and that letters of administration be issued to her.
TAaIDH

168
On February 4, 1994, petitioner Rodolfo San Luis, one of the children of Felicisimo by his first marriage,
filed a motion to dismiss 9 on the grounds of improper venue and failure to state a cause of action.
Rodolfo claimed that the petition for letters of administration should have been filed in the Province of
Laguna because this was Felicisimo's place of residence prior to his death. He further claimed that
respondent has no legal personality to file the petition because she was only a mistress of Felicisimo
since the latter, at the time of his death, was still legally married to Merry Lee.

On February 15, 1994, Linda invoked the same grounds and joined her brother Rodolfo in seeking the
dismissal 10 of the petition. On February 28, 1994, the trial court issued an Order 11 denying the two
motions to dismiss.

Unaware of the denial of the motions to dismiss, respondent filed on March 5, 1994 her opposition 12
thereto. She submitted documentary evidence showing that while Felicisimo exercised the powers of his
public office in Laguna, he regularly went home to their house in New Alabang Village, Alabang, Metro
Manila which they bought sometime in 1982. Further, she presented the decree of absolute divorce
issued by the Family Court of the First Circuit, State of Hawaii to prove that the marriage of Felicisimo to
Merry Lee had already been dissolved. Thus, she claimed that Felicisimo had the legal capacity to marry
her by virtue of paragraph 2, 13 Article 26 of the Family Code and the doctrine laid down in Van Dorn v.
Romillo, Jr. 14

Thereafter, Linda, Rodolfo and herein petitioner Edgar San Luis, separately filed motions for
reconsideration from the Order denying their motions to dismiss. 15 They asserted that paragraph 2,
Article 26 of the Family Code cannot be given retroactive effect to validate respondent's bigamous
marriage with Felicisimo because this would impair vested rights in derogation of Article 256 16 of the
Family Code. cSCTID

On April 21, 1994, Mila, another daughter of Felicisimo from his first marriage, filed a motion to
disqualify Acting Presiding Judge Anthony E. Santos from hearing the case.

On October 24, 1994, the trial court issued an Order 17 denying the motions for reconsideration. It ruled
that respondent, as widow of the decedent, possessed the legal standing to file the petition and that
venue was properly laid. Meanwhile, the motion for disqualification was deemed moot and academic 18
because then Acting Presiding Judge Santos was substituted by Judge Salvador S. Tensuan pending the
resolution of said motion.

Mila filed a motion for inhibition 19 against Judge Tensuan on November 16, 1994. On even date, Edgar
also filed a motion for reconsideration 20 from the Order denying their motion for reconsideration
arguing that it does not state the facts and law on which it was based.

On November 25, 1994, Judge Tensuan issued an Order 21 granting the motion for inhibition. The case
was re-raffled to Branch 134 presided by Judge Paul T. Arcangel.

On April 24, 1995, 22 the trial court required the parties to submit their respective position papers on
the twin issues of venue and legal capacity of respondent to file the petition. On May 5, 1995, Edgar

169
manifested 23 that he is adopting the arguments and evidence set forth in his previous motion for
reconsideration as his position paper. Respondent and Rodolfo filed their position papers on June 14, 24
and June 20, 25 1995, respectively.

On September 12, 1995, the trial court dismissed the petition for letters of administration. It held that,
at the time of his death, Felicisimo was the duly elected governor and a resident of the Province of
Laguna. Hence, the petition should have been filed in Sta. Cruz, Laguna and not in Makati City. It also
ruled that respondent was without legal capacity to file the petition for letters of administration
because her marriage with Felicisimo was bigamous, thus, void ab initio. It found that the decree of
absolute divorce dissolving Felicisimo's marriage to Merry Lee was not valid in the Philippines and did
not bind Felicisimo who was a Filipino citizen. It also ruled that paragraph 2, Article 26 of the Family
Code cannot be retroactively applied because it would impair the vested rights of Felicisimo's legitimate
children. CDTHSI

Respondent moved for reconsideration 26 and for the disqualification 27 of Judge Arcangel but said
motions were denied. 28

Respondent appealed to the Court of Appeals which reversed and set aside the orders of the trial court
in its assailed Decision dated February 4, 1998, the dispositive portion of which states:

WHEREFORE, the Orders dated September 12, 1995 and January 31, 1996 are hereby REVERSED and SET
ASIDE; the Orders dated February 28 and October 24, 1994 are REINSTATED; and the records of the case
is REMANDED to the trial court for further proceedings. 29

The appellate court ruled that under Section 1, Rule 73 of the Rules of Court, the term "place of
residence" of the decedent, for purposes of fixing the venue of the settlement of his estate, refers to the
personal, actual or physical habitation, or actual residence or place of abode of a person as distinguished
from legal residence or domicile. It noted that although Felicisimo discharged his functions as governor
in Laguna, he actually resided in Alabang, Muntinlupa. Thus, the petition for letters of administration
was properly filed in Makati City.

The Court of Appeals also held that Felicisimo had legal capacity to marry respondent by virtue of
paragraph 2, Article 26 of the Family Code and the rulings in Van Dorn v. Romillo, Jr. 30 and Pilapil v.
Ibay-Somera. 31 It found that the marriage between Felicisimo and Merry Lee was validly dissolved by
virtue of the decree of absolute divorce issued by the Family Court of the First Circuit, State of Hawaii.
As a result, under paragraph 2, Article 26, Felicisimo was capacitated to contract a subsequent marriage
with respondent. Thus —

With the well-known rule — express mandate of paragraph 2, Article 26, of the Family Code of the
Philippines, the doctrines in Van Dorn, Pilapil, and the reason and philosophy behind the enactment of
E.O. No. 227, — there is no justiciable reason to sustain the individual view — sweeping statement — of
Judge Arc[h]angel, that "Article 26, par. 2 of the Family Code, contravenes the basic policy of our state
against divorce in any form whatsoever." Indeed, courts cannot deny what the law grants. All that the
courts should do is to give force and effect to the express mandate of the law. The foreign divorce

170
having been obtained by the Foreigner on December 14, 1992, 32 the Filipino divorcee, "shall . . . have
capacity to remarry under Philippine laws". For this reason, the marriage between the deceased and
petitioner should not be denominated as "a bigamous marriage. SDATEc

Therefore, under Article 130 of the Family Code, the petitioner as the surviving spouse can institute the
judicial proceeding for the settlement of the estate of the deceased. . . . 33

Edgar, Linda, and Rodolfo filed separate motions for reconsideration 34 which were denied by the Court
of Appeals.

On July 2, 1998, Edgar appealed to this Court via the instant petition for review on certiorari. 35 Rodolfo
later filed a manifestation and motion to adopt the said petition which was granted. 36

In the instant consolidated petitions, Edgar and Rodolfo insist that the venue of the subject petition for
letters of administration was improperly laid because at the time of his death, Felicisimo was a resident
of Sta. Cruz, Laguna. They contend that pursuant to our rulings in Nuval v. Guray 37 and Romualdez v.
RTC, Br. 7, Tacloban City, 38 "residence" is synonymous with "domicile" which denotes a fixed
permanent residence to which when absent, one intends to return. They claim that a person can only
have one domicile at any given time. Since Felicisimo never changed his domicile, the petition for letters
of administration should have been filed in Sta. Cruz, Laguna.

Petitioners also contend that respondent's marriage to Felicisimo was void and bigamous because it was
performed during the subsistence of the latter's marriage to Merry Lee. They argue that paragraph 2,
Article 26 cannot be retroactively applied because it would impair vested rights and ratify the void
bigamous marriage. As such, respondent cannot be considered the surviving wife of Felicisimo; hence,
she has no legal capacity to file the petition for letters of administration.

The issues for resolution: (1) whether venue was properly laid, and (2) whether respondent has legal
capacity to file the subject petition for letters of administration. DScTaC

The petition lacks merit.

Under Section 1, 39 Rule 73 of the Rules of Court, the petition for letters of administration of the estate
of Felicisimo should be filed in the Regional Trial Court of the province "in which he resides at the time
of his death." In the case of Garcia Fule v. Court of Appeals, 40 we laid down the doctrinal rule for
determining the residence — as contradistinguished from domicile — of the decedent for purposes of
fixing the venue of the settlement of his estate:

[T]he term "resides" connotes ex vi termini "actual residence" as distinguished from "legal residence or
domicile." This term "resides," like the terms "residing" and "residence," is elastic and should be
interpreted in the light of the object or purpose of the statute or rule in which it is employed. In the
application of venue statutes and rules — Section 1, Rule 73 of the Revised Rules of Court is of such
nature — residence rather than domicile is the significant factor. Even where the statute uses the word
"domicile" still it is construed as meaning residence and not domicile in the technical sense. Some cases
make a distinction between the terms "residence" and "domicile" but as generally used in statutes fixing

171
venue, the terms are synonymous, and convey the same meaning as the term "inhabitant." In other
words, "resides" should be viewed or understood in its popular sense, meaning, the personal, actual or
physical habitation of a person, actual residence or place of abode. It signifies physical presence in a
place and actual stay thereat. In this popular sense, the term means merely residence, that is, personal
residence, not legal residence or domicile. Residence simply requires bodily presence as an inhabitant in
a given place, while domicile requires bodily presence in that place and also an intention to make it
one's domicile. No particular length of time of residence is required though; however, the residence
must be more than temporary. 41 (Emphasis supplied) STIcEA

It is incorrect for petitioners to argue that "residence," for purposes of fixing the venue of the
settlement of the estate of Felicisimo, is synonymous with "domicile." The rulings in Nuval and
Romualdez are inapplicable to the instant case because they involve election cases. Needless to say,
there is a distinction between "residence" for purposes of election laws and "residence" for purposes of
fixing the venue of actions. In election cases, "residence" and "domicile" are treated as synonymous
terms, that is, the fixed permanent residence to which when absent, one has the intention of returning.
42 However, for purposes of fixing venue under the Rules of Court, the "residence" of a person is his
personal, actual or physical habitation, or actual residence or place of abode, which may not necessarily
be his legal residence or domicile provided he resides therein with continuity and consistency. 43 Hence,
it is possible that a person may have his residence in one place and domicile in another.

In the instant case, while petitioners established that Felicisimo was domiciled in Sta. Cruz, Laguna,
respondent proved that he also maintained a residence in Alabang, Muntinlupa from 1982 up to the
time of his death. Respondent submitted in evidence the Deed of Absolute Sale 44 dated January 5,
1983 showing that the deceased purchased the aforesaid property. She also presented billing
statements 45 from the Philippine Heart Center and Chinese General Hospital for the period August to
December 1992 indicating the address of Felicisimo at "100 San Juanico, Ayala Alabang, Muntinlupa."
Respondent also presented proof of membership of the deceased in the Ayala Alabang Village
Association 46 and Ayala Country Club, Inc., 47 letter-envelopes 48 from 1988 to 1990 sent by the
deceased's children to him at his Alabang address, and the deceased's calling cards 49 stating that his
home/city address is at "100 San Juanico, Ayala Alabang Village, Muntinlupa" while his office/provincial
address is in "Provincial Capitol, Sta. Cruz, Laguna."

From the foregoing, we find that Felicisimo was a resident of Alabang, Muntinlupa for purposes of fixing
the venue of the settlement of his estate. Consequently, the subject petition for letters of
administration was validly filed in the Regional Trial Court 50 which has territorial jurisdiction over
Alabang, Muntinlupa. The subject petition was filed on December 17, 1993. At that time, Muntinlupa
was still a municipality and the branches of the Regional Trial Court of the National Capital Judicial
Region which had territorial jurisdiction over Muntinlupa were then seated in Makati City as per
Supreme Court Administrative Order No. 3. 51 Thus, the subject petition was validly filed before the
Regional Trial Court of Makati City. CaSAcH

Anent the issue of respondent Felicidad's legal personality to file the petition for letters of
administration, we must first resolve the issue of whether a Filipino who is divorced by his alien spouse

172
abroad may validly remarry under the Civil Code, considering that Felicidad's marriage to Felicisimo was
solemnized on June 20, 1974, or before the Family Code took effect on August 3, 1988. In resolving this
issue, we need not retroactively apply the provisions of the Family Code, particularly Art. 26, par. (2)
considering that there is sufficient jurisprudential basis allowing us to rule in the affirmative.

The case of Van Dorn v. Romillo, Jr. 52 involved a marriage between a foreigner and his Filipino wife,
which marriage was subsequently dissolved through a divorce obtained abroad by the latter. Claiming
that the divorce was not valid under Philippine law, the alien spouse alleged that his interest in the
properties from their conjugal partnership should be protected. The Court, however, recognized the
validity of the divorce and held that the alien spouse had no interest in the properties acquired by the
Filipino wife after the divorce. Thus:

In this case, the divorce in Nevada released private respondent from the marriage from the standards of
American law, under which divorce dissolves the marriage. As stated by the Federal Supreme Court of
the United States in Atherton vs. Atherton, 45 L. Ed. 794, 799:

"The purpose and effect of a decree of divorce from the bond of matrimony by a competent jurisdiction
are to change the existing status or domestic relation of husband and wife, and to free them both from
the bond. The marriage tie, when thus severed as to one party, ceases to bind either. A husband without
a wife, or a wife without a husband, is unknown to the law. When the law provides, in the nature of a
penalty, that the guilty party shall not marry again, that party, as well as the other, is still absolutely
freed from the bond of the former marriage."

Thus, pursuant to his national law, private respondent is no longer the husband of petitioner. He would
have no standing to sue in the case below as petitioner's husband entitled to exercise control over
conjugal assets. As he is bound by the Decision of his own country's Court, which validly exercised
jurisdiction over him, and whose decision he does not repudiate, he is estopped by his own
representation before said Court from asserting his right over the alleged conjugal property. 53

As to the effect of the divorce on the Filipino wife, the Court ruled that she should no longer be
considered married to the alien spouse. Further, she should not be required to perform her marital
duties and obligations. It held:

To maintain, as private respondent does, that, under our laws, petitioner has to be considered still
married to private respondent and still subject to a wife's obligations under Article 109, et. seq. of the
Civil Code cannot be just. Petitioner should not be obliged to live together with, observe respect and
fidelity, and render support to private respondent. The latter should not continue to be one of her heirs
with possible rights to conjugal property. She should not be discriminated against in her own country if
the ends of justice are to be served. 54 (Emphasis added) AcaEDC

This principle was thereafter applied in Pilapil v. Ibay-Somera 55 where the Court recognized the validity
of a divorce obtained abroad. In the said case, it was held that the alien spouse is not a proper party in
filing the adultery suit against his Filipino wife. The Court stated that "the severance of the marital bond

173
had the effect of dissociating the former spouses from each other, hence the actuations of one would
not affect or cast obloquy on the other." 56

Likewise, in Quita v. Court of Appeals, 57 the Court stated that where a Filipino is divorced by his
naturalized foreign spouse, the ruling in Van Dorn applies. 58 Although decided on December 22, 1998,
the divorce in the said case was obtained in 1954 when the Civil Code provisions were still in effect.

The significance of the Van Dorn case to the development of limited recognition of divorce in the
Philippines cannot be denied. The ruling has long been interpreted as severing marital ties between
parties in a mixed marriage and capacitating the Filipino spouse to remarry as a necessary consequence
of upholding the validity of a divorce obtained abroad by the alien spouse. In his treatise, Dr. Arturo M.
Tolentino cited Van Dorn stating that "if the foreigner obtains a valid foreign divorce, the Filipino spouse
shall have capacity to remarry under Philippine law." 59 In Garcia v. Recio, 60 the Court likewise cited
the aforementioned case in relation to Article 26. 61

In the recent case of Republic v. Orbecido III, 62 the historical background and legislative intent behind
paragraph 2, Article 26 of the Family Code were discussed, to wit:

Brief Historical Background

On July 6, 1987, then President Corazon Aquino signed into law Executive Order No. 209, otherwise
known as the "Family Code," which took effect on August 3, 1988. Article 26 thereof states:

All marriages solemnized outside the Philippines in accordance with the laws in force in the country
where they were solemnized, and valid there as such, shall also be valid in this country, except those
prohibited under Articles 35, 37, and 38. HSCAIT

On July 17, 1987, shortly after the signing of the original Family Code, Executive Order No. 227 was
likewise signed into law, amending Articles 26, 36, and 39 of the Family Code. A second paragraph was
added to Article 26. As so amended, it now provides:

ART. 26. All marriages solemnized outside the Philippines in accordance with the laws in force in
the country where they were solemnized, and valid there as such, shall also be valid in this country,
except those prohibited under Articles 35(1), (4), (5) and (6), 36, 37 and 38.

Where a marriage between a Filipino citizen and a foreigner is validly celebrated and a divorce is
thereafter validly obtained abroad by the alien spouse capacitating him or her to remarry, the Filipino
spouse shall have capacity to remarry under Philippine law. (Emphasis supplied)

xxx xxx xxx

Legislative Intent

Records of the proceedings of the Family Code deliberations showed that the intent of Paragraph 2 of
Article 26, according to Judge Alicia Sempio-Diy, a member of the Civil Code Revision Committee, is to

174
avoid the absurd situation where the Filipino spouse remains married to the alien spouse who, after
obtaining a divorce, is no longer married to the Filipino spouse.

Interestingly, Paragraph 2 of Article 26 traces its origin to the 1985 case of Van Dorn v. Romillo, Jr. The
Van Dorn case involved a marriage between a Filipino citizen and a foreigner. The Court held therein
that a divorce decree validly obtained by the alien spouse is valid in the Philippines, and consequently,
the Filipino spouse is capacitated to remarry under Philippine law. 63 (Emphasis added)

As such, the Van Dorn case is sufficient basis in resolving a situation where a divorce is validly obtained
abroad by the alien spouse. With the enactment of the Family Code and paragraph 2, Article 26 thereof,
our lawmakers codified the law already established through judicial precedent. HAaECD

Indeed, when the object of a marriage is defeated by rendering its continuance intolerable to one of the
parties and productive of no possible good to the community, relief in some way should be obtainable.
64 Marriage, being a mutual and shared commitment between two parties, cannot possibly be
productive of any good to the society where one is considered released from the marital bond while the
other remains bound to it. Such is the state of affairs where the alien spouse obtains a valid divorce
abroad against the Filipino spouse, as in this case.

Petitioners cite Articles 15 65 and 17 66 of the Civil Code in stating that the divorce is void under
Philippine law insofar as Filipinos are concerned. However, in light of this Court's rulings in the cases
discussed above, the Filipino spouse should not be discriminated against in his own country if the ends
of justice are to be served. 67 In Alonzo v. Intermediate Appellate Court, 68 the Court stated:

But as has also been aptly observed, we test a law by its results; and likewise, we may add, by its
purposes. It is a cardinal rule that, in seeking the meaning of the law, the first concern of the judge
should be to discover in its provisions the intent of the lawmaker. Unquestionably, the law should never
be interpreted in such a way as to cause injustice as this is never within the legislative intent. An
indispensable part of that intent, in fact, for we presume the good motives of the legislature, is to
render justice.

Thus, we interpret and apply the law not independently of but in consonance with justice. Law and
justice are inseparable, and we must keep them so. To be sure, there are some laws that, while
generally valid, may seem arbitrary when applied in a particular case because of its peculiar
circumstances. In such a situation, we are not bound, because only of our nature and functions, to apply
them just the same, in slavish obedience to their language. What we do instead is find a balance
between the word and the will, that justice may be done even as the law is obeyed. TAECaD

As judges, we are not automatons. We do not and must not unfeelingly apply the law as it is worded,
yielding like robots to the literal command without regard to its cause and consequence. "Courts are apt
to err by sticking too closely to the words of a law," so we are warned, by Justice Holmes again, "where
these words import a policy that goes beyond them."

xxx xxx xxx

175
More than twenty centuries ago, Justinian defined justice "as the constant and perpetual wish to render
every one his due." That wish continues to motivate this Court when it assesses the facts and the law in
every case brought to it for decision. Justice is always an essential ingredient of its decisions. Thus when
the facts warrants, we interpret the law in a way that will render justice, presuming that it was the
intention of the lawmaker, to begin with, that the law be dispensed with justice. 69

Applying the above doctrine in the instant case, the divorce decree allegedly obtained by Merry Lee
which absolutely allowed Felicisimo to remarry, would have vested Felicidad with the legal personality
to file the present petition as Felicisimo's surviving spouse. However, the records show that there is
insufficient evidence to prove the validity of the divorce obtained by Merry Lee as well as the marriage
of respondent and Felicisimo under the laws of the U.S.A. In Garcia v. Recio, 70 the Court laid down the
specific guidelines for pleading and proving foreign law and divorce judgments. It held that presentation
solely of the divorce decree is insufficient and that proof of its authenticity and due execution must be
presented. Under Sections 24 and 25 of Rule 132, a writing or document may be proven as a public or
official record of a foreign country by either (1) an official publication or (2) a copy thereof attested by
the officer having legal custody of the document. If the record is not kept in the Philippines, such copy
must be (a) accompanied by a certificate issued by the proper diplomatic or consular officer in the
Philippine foreign service stationed in the foreign country in which the record is kept and (b)
authenticated by the seal of his office. 71

With regard to respondent's marriage to Felicisimo allegedly solemnized in California, U.S.A., she
submitted photocopies of the Marriage Certificate and the annotated text 72 of the Family Law Act of
California which purportedly show that their marriage was done in accordance with the said law. As
stated in Garcia, however, the Court cannot take judicial notice of foreign laws as they must be alleged
and proved. 73

Therefore, this case should be remanded to the trial court for further reception of evidence on the
divorce decree obtained by Merry Lee and the marriage of respondent and Felicisimo.

Even assuming that Felicisimo was not capacitated to marry respondent in 1974, nevertheless, we find
that the latter has the legal personality to file the subject petition for letters of administration, as she
may be considered the co-owner of Felicisimo as regards the properties that were acquired through
their joint efforts during their cohabitation. TIEHDC

Section 6, 74 Rule 78 of the Rules of Court states that letters of administration may be granted to the
surviving spouse of the decedent. However, Section 2, Rule 79 thereof also provides in part:

SEC. 2. Contents of petition for letters of administration. — A petition for letters of administration must
be filed by an interested person and must show, as far as known to the petitioner: . . . .

An "interested person" has been defined as one who would be benefited by the estate, such as an heir,
or one who has a claim against the estate, such as a creditor. The interest must be material and direct,
and not merely indirect or contingent. 75

176
In the instant case, respondent would qualify as an interested person who has a direct interest in the
estate of Felicisimo by virtue of their cohabitation, the existence of which was not denied by petitioners.
If she proves the validity of the divorce and Felicisimo's capacity to remarry, but fails to prove that her
marriage with him was validly performed under the laws of the U.S.A., then she may be considered as a
co-owner under Article 144 76 of the Civil Code. This provision governs the property relations between
parties who live together as husband and wife without the benefit of marriage, or their marriage is void
from the beginning. It provides that the property acquired by either or both of them through their work
or industry or their wages and salaries shall be governed by the rules on co-ownership. In a co-
ownership, it is not necessary that the property be acquired through their joint labor, efforts and
industry. Any property acquired during the union is prima facie presumed to have been obtained
through their joint efforts. Hence, the portions belonging to the co-owners shall be presumed equal,
unless the contrary is proven. 77

Meanwhile, if respondent fails to prove the validity of both the divorce and the marriage, the applicable
provision would be Article 148 of the Family Code which has filled the hiatus in Article 144 of the Civil
Code by expressly regulating the property relations of couples living together as husband and wife but
are incapacitated to marry. 78 In Saguid v. Court of Appeals, 79 we held that even if the cohabitation or
the acquisition of property occurred before the Family Code took effect, Article 148 governs. 80 The
Court described the property regime under this provision as follows:

The regime of limited co-ownership of property governing the union of parties who are not legally
capacitated to marry each other, but who nonetheless live together as husband and wife, applies to
properties acquired during said cohabitation in proportion to their respective contributions. Co-
ownership will only be up to the extent of the proven actual contribution of money, property or
industry. Absent proof of the extent thereof, their contributions and corresponding shares shall be
presumed to be equal. AcTDaH

xxx xxx xxx

In the cases of Agapay v. Palang, and Tumlos v. Fernandez, which involved the issue of co-ownership of
properties acquired by the parties to a bigamous marriage and an adulterous relationship, respectively,
we ruled that proof of actual contribution in the acquisition of the property is essential. . . .

As in other civil cases, the burden of proof rests upon the party who, as determined by the pleadings or
the nature of the case, asserts an affirmative issue. Contentions must be proved by competent evidence
and reliance must be had on the strength of the party's own evidence and not upon the weakness of the
opponent's defense. . . . 81

In view of the foregoing, we find that respondent's legal capacity to file the subject petition for letters of
administration may arise from her status as the surviving wife of Felicisimo or as his co-owner under
Article 144 of the Civil Code or Article 148 of the Family Code.

WHEREFORE, the petition is DENIED. The Decision of the Court of Appeals reinstating and affirming the
February 28, 1994 Order of the Regional Trial Court which denied petitioners' motion to dismiss and its

177
October 24, 1994 Order which dismissed petitioners' motion for reconsideration is AFFIRMED. Let this
case be REMANDED to the trial court for further proceedings.

SO ORDERED.

Austria-Martinez, Callejo, Sr. and Chico-Nazario, JJ., concur.

178
Uypitching vs. Quiamco, G.R. No. 146322, December 6, 2006

[G.R. No. 146322. December 6, 2006.]

ERNESTO RAMAS UYPITCHING and RAMAS UYPITCHING SONS, INC., petitioners, vs. ERNESTO QUIAMCO,
respondent.

DECISION

CORONA, J p:

Honeste vivere, non alterum laedere et jus suum cuique tribuere. To live virtuously, not to injure others
and to give everyone his due. These supreme norms of justice are the underlying principles of law and
order in society. We reaffirm them in this petition for review on certiorari assailing the July 26, 2000
decision 1 and October 18, 2000 resolution of the Court of Appeals (CA) in CA-G.R. CV No. 47571.
AEHTIC

In 1982, respondent Ernesto C. Quiamco was approached by Juan Davalan, 2 Josefino Gabutero and Raul
Generoso to amicably settle the civil aspect of a criminal case for robbery 3 filed by Quiamco against
them. They surrendered to him a red Honda XL-100 motorcycle and a photocopy of its certificate of
registration. Respondent asked for the original certificate of registration but the three accused never
came to see him again. Meanwhile, the motorcycle was parked in an open space inside respondent's
business establishment, Avesco-AVNE Enterprises, where it was visible and accessible to the public.

It turned out that, in October 1981, the motorcycle had been sold on installment basis to Gabutero by
petitioner Ramas Uypitching Sons, Inc., a family-owned corporation managed by petitioner Atty. Ernesto
Ramas Uypitching. To secure its payment, the motorcycle was mortgaged to petitioner corporation. 4

When Gabutero could no longer pay the installments, Davalan assumed the obligation and continued
the payments. In September 1982, however, Davalan stopped paying the remaining installments and
told petitioner corporation's collector, Wilfredo Veraño, that the motorcycle had allegedly been "taken
by respondent's men."

Nine years later, on January 26, 1991, petitioner Uypitching, accompanied by policemen, 5 went to
Avesco-AVNE Enterprises to recover the motorcycle. The leader of the police team, P/Lt. Arturo
Vendiola, talked to the clerk in charge and asked for respondent. While P/Lt. Vendiola and the clerk
were talking, petitioner Uypitching paced back and forth inside the establishment uttering "Quiamco is a
thief of a motorcycle."

On learning that respondent was not in Avesco-AVNE Enterprises, the policemen left to look for
respondent in his residence while petitioner Uypitching stayed in the establishment to take photographs
of the motorcycle. Unable to find respondent, the policemen went back to Avesco-AVNE Enterprises
and, on petitioner Uypitching's instruction and over the clerk's objection, took the motorcycle. EDIHSC

179
On February 18, 1991, petitioner Uypitching filed a criminal complaint for qualified theft and/or
violation of the Anti-Fencing Law 6 against respondent in the Office of the City Prosecutor of Dumaguete
City. 7 Respondent moved for dismissal because the complaint did not charge an offense as he had
neither stolen nor bought the motorcycle. The Office of the City Prosecutor dismissed the complaint 8
and denied petitioner Uypitching's subsequent motion for reconsideration.

Respondent filed an action for damages against petitioners in the RTC of Dumaguete City, Negros
Oriental, Branch 37. 9 He sought to hold the petitioners liable for the following: (1) unlawful taking of
the motorcycle; (2) utterance of a defamatory remark (that respondent was a thief) and (3) precipitate
filing of a baseless and malicious complaint. These acts humiliated and embarrassed the respondent and
injured his reputation and integrity.

On July 30, 1994, the trial court rendered a decision 10 finding that petitioner Uypitching was motivated
with malice and ill will when he called respondent a thief, took the motorcycle in an abusive manner and
filed a baseless complaint for qualified theft and/or violation of the Anti-Fencing Law. Petitioners' acts
were found to be contrary to Articles 19 11 and 20 12 of the Civil Code. Hence, the trial court held
petitioners liable to respondent for P500,000 moral damages, P200,000 exemplary damages and
P50,000 attorney's fees plus costs.

Petitioners appealed the RTC decision but the CA affirmed the trial court's decision with modification,
reducing the award of moral and exemplary damages to P300,000 and P100,000, respectively. 13
Petitioners sought reconsideration but it was denied. Thus, this petition.

In their petition and memorandum, petitioners submit that the sole (allegedly) issue to be resolved here
is whether the filing of a complaint for qualified theft and/or violation of the Anti-Fencing Law in the
Office of the City Prosecutor warranted the award of moral damages, exemplary damages, attorney's
fees and costs in favor of respondent. ESDcIA

Petitioners' suggestion is misleading. They were held liable for damages not only for instituting a
groundless complaint against respondent but also for making a slanderous remark and for taking the
motorcycle from respondent's establishment in an abusive manner.

CORRECTNESS OF THE FINDINGS

OF THE RTC AND CA

As they never questioned the findings of the RTC and CA that malice and ill will attended not only the
public imputation of a crime to respondent 14 but also the taking of the motorcycle, petitioners were
deemed to have accepted the correctness of such findings. This alone was sufficient to hold petitioners
liable for damages to respondent.

Nevertheless, to address petitioners' concern, we also find that the trial and appellate courts correctly
ruled that the filing of the complaint was tainted with malice and bad faith. Petitioners themselves in
fact described their action as a "precipitate act." 15 Petitioners were bent on portraying respondent as a

180
thief. In this connection, we quote with approval the following findings of the RTC, as adopted by the
CA:

. . . There was malice or ill-will [in filing the complaint before the City Prosecutor's Office] because Atty.
Ernesto Ramas Uypitching knew or ought to have known as he is a lawyer, that there was no probable
cause at all for filing a criminal complaint for qualified theft and fencing activity against [respondent].
Atty. Uypitching had no personal knowledge that [respondent] stole the motorcycle in question. He was
merely told by his bill collector ([i.e.] the bill collector of Ramas Uypitching Sons, Inc.)[,] Wilfredo
Veraño[,] that Juan Dabalan will [no longer] pay the remaining installment(s) for the motorcycle because
the motorcycle was taken by the men of [respondent]. It must be noted that the term used by Wilfredo
Veraño in informing Atty. Ernesto Ramas Uypitching of the refusal of Juan Dabalan to pay for the
remaining installment was [']taken['], not [']unlawfully taken['] or 'stolen.' Yet, despite the double
hearsay, Atty. Ernesto Ramas Uypitching not only executed the [complaint-affidavit] wherein he named
[respondent] as 'the suspect' of the stolen motorcycle but also charged [respondent] of 'qualified theft
and fencing activity' before the City [Prosecutor's] Office of Dumaguete. The absence of probable cause
necessarily signifies the presence of malice. What is deplorable in all these is that Juan Dabalan, the
owner of the motorcycle, did not accuse [respondent] or the latter's men of stealing the motorcycle[,]
much less bother[ed] to file a case for qualified theft before the authorities. That Atty. Uypitching's act
in charging [respondent] with qualified theft and fencing activity is tainted with malice is also shown by
his answer to the question of Cupid Gonzaga 16 [during one of their conversations] — "why should you
still file a complaint? You have already recovered the motorcycle. . ."[:] "Aron motagam ang kawatan ug
motor." ("To teach a lesson to the thief of motorcycle.") 17

Moreover, the existence of malice, ill will or bad faith is a factual matter. As a rule, findings of fact of the
trial court, when affirmed by the appellate court, are conclusive on this Court. We see no compelling
reason to reverse the findings of the RTC and the CA. SEIcHa

PETITIONERS ABUSED THEIR

RIGHT OF RECOVERY AS

MORTGAGEE(S)

Petitioners claim that they should not be held liable for petitioner corporation's exercise of its right as
seller-mortgagee to recover the mortgaged vehicle preliminary to the enforcement of its right to
foreclose on the mortgage in case of default. They are clearly mistaken.

True, a mortgagee may take steps to recover the mortgaged property to enable it to enforce or protect
its foreclosure right thereon. There is, however, a well-defined procedure for the recovery of possession
of mortgaged property: if a mortgagee is unable to obtain possession of a mortgaged property for its
sale on foreclosure, he must bring a civil action either to recover such possession as a preliminary step
to the sale, or to obtain judicial foreclosure. 18

181
Petitioner corporation failed to bring the proper civil action necessary to acquire legal possession of the
motorcycle. Instead, petitioner Uypitching descended on respondent's establishment with his policemen
and ordered the seizure of the motorcycle without a search warrant or court order. Worse, in the course
of the illegal seizure of the motorcycle, petitioner Uypitching even mouthed a slanderous statement.

No doubt, petitioner corporation, acting through its co-petitioner Uypitching, blatantly disregarded the
lawful procedure for the enforcement of its right, to the prejudice of respondent. Petitioners' acts
violated the law as well as public morals, and transgressed the proper norms of human relations.

The basic principle of human relations, embodied in Article 19 of the Civil Code, provides:

Art. 19. Every person must in the exercise of his rights and in the performance of his duties, act with
justice, give every one his due, and observe honesty and good faith. EAHcCT

Article 19, also known as the "principle of abuse of right," prescribes that a person should not use his
right unjustly or contrary to honesty and good faith, otherwise he opens himself to liability. 19 It seeks
to preclude the use of, or the tendency to use, a legal right (or duty) as a means to unjust ends.

There is an abuse of right when it is exercised solely to prejudice or injure another. 20 The exercise of a
right must be in accordance with the purpose for which it was established and must not be excessive or
unduly harsh; there must be no intention to harm another. 21 Otherwise, liability for damages to the
injured party will attach.

In this case, the manner by which the motorcycle was taken at petitioners' instance was not only
attended by bad faith but also contrary to the procedure laid down by law. Considered in conjunction
with the defamatory statement, petitioners' exercise of the right to recover the mortgaged vehicle was
utterly prejudicial and injurious to respondent. On the other hand, the precipitate act of filing an
unfounded complaint could not in any way be considered to be in accordance with the purpose for
which the right to prosecute a crime was established. Thus, the totality of petitioners' actions showed a
calculated design to embarrass, humiliate and publicly ridicule respondent. Petitioners acted in an
excessively harsh fashion to the prejudice of respondent. Contrary to law, petitioners willfully caused
damage to respondent. Hence, they should indemnify him. 22

WHEREFORE, the petition is hereby DENIED. The July 26, 2000 decision and October 18, 2000 resolution
of the Court of Appeals in CA-G.R. CV No. 47571 are AFFIRMED.

Triple costs against petitioners, considering that petitioner Ernesto Ramas Uypitching is a lawyer and an
officer of the court, for his improper behavior. SDIaHE

SO ORDERED.

Puno, Sandoval-Gutierrez, Azcuna and Garcia, JJ., concur.

182
Wassmer vs. Velez, G.R. No. L-20089, December 26, 1964

[G.R. No. L-20089. February 26, 1965.]

BEATRIZ P. WASSMER, plaintiff-appellee, vs. FRANCISCO X. VELEZ, defendant-appellant.

RESOLUTION

BENGZON, J.P., J p:

Defendant-appellant has filed a motion for reconsideration of this Court's decision promulgated
December 26, 1964. The only point movant raises is the alleged validity of his affidavit of merits
attached to his petition for relief in the lower court.

The affidavit of merits in question states "that he (defendant) has a good and valid defense, his failure to
marry plaintiff as scheduled having been due to a fortuitous event and/or circumstances beyond his
control." The movant contends that this is not a mere opinion or conclusion but positive and categorical
statement of a valid defense; that it stated there are fortuitous events, i.e., fortuitous facts, which
defendant puts forward as valid defense. The previous rulings of this Court, movant further contends,
held invalid only affidavits of merits that disclosed no defense.

The rulings of this court require affidavits of merits to state not mere conclusions or opinions but facts
(Vaswani vs. Tarachand Bros., L-15800, Dec. 29, 1960). An affidavit is a statement under oath of facts.
Defendant's affidavit of merits stated no facts, but merely an inference that defendant's failure was due
to fortuitous events under circumstances beyond his control. This is a conclusion of fact, no a fact.

An affidavit of merits is required to avoid waste of the court's time if the defense turns out to be
ineffective (Vda. de Yulo vs. Chua Chuco, 48 O.G. 554, 555). Statements too vague or merely general do
not — as movant admits — serve the afore-stated purpose.

Defendant's affidavit of merits provides no means for the court to see the merits of his defense and
determine whether reopening the case would be worth its time. Said affidavit revealed nothing of the
"event" or "circumstances" constituting the defense. It stated, in substance, only defendant's opinion
that the event was "fortuitous" and that the circumstances were "beyond his control"; and his
conclusion that his failure to marry plaintiff on schedule was "due to" them. The court, not the
defendant, should form such opinions and draw such conclusions on the basis of facts provided in the
affidavit. As it is, defendant's affidavit leaves the court guessing as to the facts.

Conformably to previous rulings of this Court, therefore, the affidavit of merits aforementioned is not
valid. To repeat, it states a conclusion of facts, not facts themselves; it leaves the court guessing as to
the facts; it provides no basis for determining the probable merits of the defense as a jurisdiction for
reopening the case.

WHEREFORE, the motion for reconsideration is hereby denied. So ordered.

183
Bengzon, C.J., Bautista Angelo, Concepcion, Reyes, J.B.L., Barrera, Paredes, Dizon, Regala, Makalintal and
Zaldivar, JJ., concur.

C o p y r i g h t 1 9 9 4 - 1 9 9 9 C D T e c h n o l o g i e s A s i a, I n c.

G.R. No. L-20019 February 26, 1965

In re Go v. Republic

EN BANC

[G.R. No. L-20019. February 26, 1965.]

In the matter of the petition for admission to Philippine citizenship. LORENZO GO alias LORENZO CHUA,
petitioner-appellee, vs. REPUBLIC OF THE PHILIPPINES, oppositor-appellant.

Valeriano S. Kaamino for petitioner-appellee.

Solicitor General for oppositor-appellant.

SYLLABUS

1. CITIZENSHIP; NATURALIZATION; LUCRATIVE INCOME; MONTHLY SALARY OF P150 FOR


UNMARRIED APPLICANT NOT LUCRATIVE. — A monthly salary of P150 per month for an unmarried
applicant is not lucrative under existing jurisprudence and the Naturalization Law due to the high cost of
living and the present low purchasing power of the peso.

2. ID.; ID.; ABILITY TO SPEAK AND WRITE ANY OF THE PRINCIPAL DIALECTS OF THE COUNTRY MUST
BE PROVEN. — In the absence of proof thereof, the court cannot simply presume that petitioner can
speak and write any of the principal dialects of the country, a circumstance which fatally affects his right
to Philippine citizenship.

DECISION

PAREDES, J p:

On November 16, 1960, Lorenzo Go alias Lorenzo Chua, petitioner-appellee herein, presented with the
CFI of Misamis Occidental, a petition for Naturalization, containing all the jurisdictional allegations, and
accompanied by a Joint Affidavit of Maximo Lago and Vitaliano Jimenez, who appeared as character
witnesses. Against this petition, an Opposition was registered by the Republic, it appearing that
petitioner stated in his petition that his income was only P150.00 per month. After trial, the lower court
rendered judgment, the pertinent portions of which recite:

184
"The evidence further shows that the petitioner was born in Ozamiz City on November 14, 1939, and has
continuously resided in Ozamiz City up to the present time; that the petitioner has been employed as
salesman since May, 1960, receiving a salary of P1,800.00 annually, being single; . . .

xxx xxx xxx

"Considering, therefore, that the petitioner has all the qualifications and none of the disqualifications to
become Filipino citizen, judgment is hereby rendered declaring that the petitioner is entitled to acquire
Philippine citizenship and as soon as this decision shall have become final and executory in accordance
with the Paredes Law and upon previous hearing that he has complied with all the conditions and
requisites under said law, he shall be allowed to take his oath of allegiance and fidelity to the Philippine
Republic and thereafter the Clerk of Court shall issue in his favor letters or certificate of citizenship."

The above judgment was appealed by the Solicitor General on two (2) counts, to wit: the lower court
erred in granting the petition, because —

(1) petitioner's income does not measure up to the minimum standard of lucrative income as fixed
by the Supreme Court; and

(2) petitioner's ability to speak and write English and one of the principal Philippine languages has
not been duly proven by actual demonstration in Court.

In a number of cases, this Court has consistently held that P150.00 earnings per month for an unmarried
applicant, who was a student and a salesman at the same time, is not lucrative and substantial, within
the meaning and contemplation of the Naturalization Law, considering the high cost of living and the
low purchasing power of the peso at present. In fact, this Court has even ruled that P250.00 per month,
is not a lucrative income (Ong vs. Rep., G.R. No. L-15764, May 19, 1961). On this score alone, the
decision of the lower court cannot be sustained.

Moreover, there is nothing in the records to show, that although petitioner-appellee could speak and
write the English language, he can also speak and write any of the principal dialects of the country. No
question in any of the dialects was propounded to him in order to demonstrate his knowledge. In the
absence of such proof, We cannot just simply presume that petitioner can speak and write the Cebu-
Visayan or Tagalog dialects, a circumstance which fatally affects his right to Philippine citizenship. It is
incumbent upon applicants to prove that they have all the qualifications, and failing to do so, We should
assume contrariwise.

WHEREFORE, the decision appealed from should be, as it is hereby reversed, and another entered,
declaring that petitioner is not qualified, under the circumstances appearing on record, to acquire
Philippine citizenship. Costs against herein petitioner-appellee Lorenzo Go alias Lorenzo Chua.

Bengzon, C.J., Bautista Angelo, Concepcion, Reyes, J.B.L., Barrera, Dizon, Regala, Makalintal, Bengzon,
J.P. and Zaldivar JJ., concur.

185
Nikko Hotel Manila Garden, et all vs. Reyes G.R. No. 154259, February 28, 2005

[G.R. No. 154259. February 28, 2005.]

NIKKO HOTEL MANILA GARDEN and RUBY LIM, petitioners, vs. ROBERTO REYES, a.k.a. "AMAY BISAYA,"
respondent.

Angara Abello Concepcion Regala & Cruz for petitioners.

Vicente D. Millora for respondent.

SYLLABUS

1. CIVIL LAW; DAMAGES; CANNOT BE RECOVERED WHEN THE DOCTRINE OF VOLENTI NON FIT
INJURIA IS APPLICABLE. — The doctrine of volenti non fit injuria ("to which a person assents is not
esteemed in law as injury") refers to self-inflicted injury or to the consent to injury which precludes the
recovery of damages by one who has knowingly and voluntarily exposed himself to danger, even if he is
not negligent in doing so. aETADI

2. REMEDIAL LAW; CIVIL PROCEDURE; APPEALS; PETITION FOR REVIEW ON CERTIORARI; LIMITED
TO REVIEWING ERRORS OF LAW; EXCEPTION; PRESENT IN CASE AT BAR. — As the trial court and the
appellate court reached divergent and irreconcilable conclusions concerning the same facts and
evidence of the case, this Court is left without choice but to use its latent power to review such findings
of facts. Indeed, the general rule is that we are not a trier of facts as our jurisdiction is limited to
reviewing and revising errors of law. One of the exceptions to this general rule, however, obtains herein
as the findings of the Court of Appeals are contrary to those of the trial court. The lower court ruled that
Ms. Lim did not abuse her right to ask Mr. Reyes to leave the party as she talked to him politely and
discreetly. The appellate court, on the other hand, held that Ms. Lim is liable for damages as she
needlessly embarrassed Mr. Reyes by telling him not to finish his food and to leave the place within
hearing distance of the other guests. Both courts, however, were in agreement that it was Dr. Filart's
invitation that brought Mr. Reyes to the party.

3. CIVIL LAW; PRINCIPLE OF ABUSE OF RIGHTS; ELEMENTS. — Article 19, known to contain what is
commonly referred to as the principle of abuse of rights, is not a panacea for all human hurts and social
grievances. . . . Elsewhere, we explained that when "a right is exercised in a manner which does not
conform with the norms enshrined in Article 19 and results in damage to another, a legal wrong is
thereby committed for which the wrongdoer must be responsible". The object of this article, therefore,
is to set certain standards which must be observed not only in the exercise of one's rights but also in the
performance of one's duties. These standards are the following: act with justice, give everyone his due
and observe honesty and good faith. Its antithesis, necessarily, is any act evincing bad faith or intent to
injure. Its elements are the following: (1) There is a legal right or duty; (2) which is exercised in bad faith;
(3) for the sole intent of prejudicing or injuring another. When Article 19 is violated, an action for
damages is proper under Articles 20 or 21 of the Civil Code. Article 20 pertains to damages arising from a

186
violation of law which does not obtain herein as Ms. Lim was perfectly within her right to ask Mr. Reyes
to leave. SECIcT

4. ID.; ACTS CONTRA BONUS MORES; ELEMENTS. — Article 21 refers to acts contra bonus mores
and has the following elements: (1) There is an act which is legal; (2) but which is contrary to morals,
good custom, public order, or public policy; and (3) it is done with intent to injure.

DECISION

CHICO-NAZARIO, J p:

In this petition for review on certiorari, petitioners Nikko Hotel Manila Garden (Hotel Nikko) 1 and Ruby
Lim assail the Decision 2 of the Court of Appeals dated 26 November 2001 reversing the Decision 3 of
the Regional Trial Court (RTC) of Quezon City, Branch 104, as well as the Resolution 4 of the Court of
Appeals dated 09 July 2002 which denied petitioners' motion for reconsideration. SECcIH

The cause of action before the trial court was one for damages brought under the human relations
provisions of the New Civil Code. Plaintiff thereat (respondent herein) Roberto Reyes, more popularly
known by the screen name "Amay Bisaya," alleged that at around 6:00 o'clock in the evening of 13
October 1994, while he was having coffee at the lobby of Hotel Nikko, 5 he was spotted by his friend of
several years, Dr. Violeta Filart, who then approached him. 6 Mrs. Filart invited him to join her in a party
at the hotel's penthouse in celebration of the natal day of the hotel's manager, Mr. Masakazu Tsuruoka.
7 Mr. Reyes asked if she could vouch for him for which she replied: "of course." 8 Mr. Reyes then went
up with the party of Dr. Filart carrying the basket of fruits which was the latter's present for the
celebrant. 9 At the penthouse, they first had their picture taken with the celebrant after which Mr.
Reyes sat with the party of Dr. Filart. 10 After a couple of hours, when the buffet dinner was ready, Mr.
Reyes lined-up at the buffet table but, to his great shock, shame and embarrassment, he was stopped by
petitioner herein, Ruby Lim, who claimed to speak for Hotel Nikko as Executive Secretary thereof. 11 In a
loud voice and within the presence and hearing of the other guests who were making a queue at the
buffet table, Ruby Lim told him to leave the party ("huwag ka nang kumain, hindi ka imbitado, bumaba
ka na lang"). 12 Mr. Reyes tried to explain that he was invited by Dr. Filart. 13 Dr. Filart, who was within
hearing distance, however, completely ignored him thus adding to his shame and humiliation. 14 Not
long after, while he was still recovering from the traumatic experience, a Makati policeman approached
and asked him to step out of the hotel. 15 Like a common criminal, he was escorted out of the party by
the policeman. 16 Claiming damages, Mr. Reyes asked for One Million Pesos actual damages, One
Million Pesos moral and/or exemplary damages and Two Hundred Thousand Pesos attorney's fees. 17

Ruby Lim, for her part, admitted having asked Mr. Reyes to leave the party but not under the
ignominious circumstance painted by the latter. Ms. Lim narrated that she was the Hotel's Executive
Secretary for the past twenty (20) years. 18 One of her functions included organizing the birthday party
of the hotel's former General Manager, Mr. Tsuruoka. 19 The year 1994 was no different. For Mr.
Tsuruoka's party, Ms. Lim generated an exclusive guest list and extended invitations accordingly. 20 The
guest list was limited to approximately sixty (60) of Mr. Tsuruoka's closest friends and some hotel
employees and that Mr. Reyes was not one of those invited. 21 At the party, Ms. Lim first noticed Mr.

187
Reyes at the bar counter ordering a drink. 22 Mindful of Mr. Tsuruoka's wishes to keep the party
intimate, Ms. Lim approached Mr. Boy Miller, the "captain waiter," to inquire as to the presence of Mr.
Reyes who was not invited. 23 Mr. Miller replied that he saw Mr. Reyes with the group of Dr. Filart. 24
As Dr. Filart was engaged in conversation with another guest and as Ms. Lim did not want to interrupt,
she inquired instead from the sister of Dr. Filart, Ms. Zenaida Fruto, who told her that Dr. Filart did not
invite Mr. Reyes. 25 Ms. Lim then requested Ms. Fruto to tell Mr. Reyes to leave the party as he was not
invited. 26 Mr. Reyes, however, lingered prompting Ms. Lim to inquire from Ms. Fruto who said that Mr.
Reyes did not want to leave. 27 When Ms. Lim turned around, she saw Mr. Reyes conversing with a
Captain Batung whom she later approached. 28 Believing that Captain Batung and Mr. Reyes knew each
other, Ms. Lim requested from him the same favor from Ms. Fruto, i.e., for Captain Batung to tell Mr.
Reyes to leave the party as he was not invited. 29 Still, Mr. Reyes lingered. When Ms. Lim spotted Mr.
Reyes by the buffet table, she decided to speak to him herself as there were no other guests in the
immediate vicinity. 30 However, as Mr. Reyes was already helping himself to the food, she decided to
wait. 31 When Mr. Reyes went to a corner and started to eat, Ms. Lim approached him and said: "alam
ninyo, hindi ho kayo dapat nandito. Pero total nakakuha na ho kayo ng pagkain, ubusin na lang ninyo at
pagkatapos kung pwede lang po umalis na kayo." 32 She then turned around trusting that Mr. Reyes
would show enough decency to leave, but to her surprise, he began screaming and making a big scene,
and even threatened to dump food on her. 33

Dr. Violeta Filart, the third defendant in the complaint before the lower court, also gave her version of
the story to the effect that she never invited Mr. Reyes to the party. 34 According to her, it was Mr.
Reyes who volunteered to carry the basket of fruits intended for the celebrant as he was likewise going
to take the elevator, not to the penthouse but to Altitude 49. 35 When they reached the penthouse, she
reminded Mr. Reyes to go down as he was not properly dressed and was not invited. 36 All the while,
she thought that Mr. Reyes already left the place, but she later saw him at the bar talking to Col. Batung.
37 Then there was a commotion and she saw Mr. Reyes shouting. 38 She ignored Mr. Reyes. 39 She was
embarrassed and did not want the celebrant to think that she invited him. 40

After trial on the merits, the court a quo dismissed the complaint, 41 giving more credence to the
testimony of Ms. Lim that she was discreet in asking Mr. Reyes to leave the party. The trial court likewise
ratiocinated that Mr. Reyes assumed the risk of being thrown out of the party as he was uninvited:

Plaintiff had no business being at the party because he was not a guest of Mr. Tsuruoka, the birthday
celebrant. He assumed the risk of being asked to leave for attending a party to which he was not invited
by the host. Damages are pecuniary consequences which the law imposes for the breach of some duty
or the violation of some right. Thus, no recovery can be had against defendants Nikko Hotel and Ruby
Lim because he himself was at fault (Garciano v. Court of Appeals, 212 SCRA 436). He knew that it was
not the party of defendant Violeta Filart even if she allowed him to join her and took responsibility for
his attendance at the party. His action against defendants Nikko Hotel and Ruby Lim must therefore fail.
42

188
On appeal, the Court of Appeals reversed the ruling of the trial court as it found more commanding of
belief the testimony of Mr. Reyes that Ms. Lim ordered him to leave in a loud voice within hearing
distance of several guests:

In putting appellant in a very embarrassing situation, telling him that he should not finish his food and to
leave the place within the hearing distance of other guests is an act which is contrary to morals, good
customs . . ., for which appellees should compensate the appellant for the damage suffered by the latter
as a consequence therefore (Art. 21, New Civil Code). The liability arises from the acts which are in
themselves legal or not prohibited, but contrary to morals or good customs. Conversely, even in the
exercise of a formal right, [one] cannot with impunity intentionally cause damage to another in a
manner contrary to morals or good customs. 43

The Court of Appeals likewise ruled that the actuation of Ms. Lim in approaching several people to
inquire into the presence of Mr. Reyes exposed the latter to ridicule and was uncalled for as she should
have approached Dr. Filart first and both of them should have talked to Mr. Reyes in private:

Said acts of appellee Lim are uncalled for. What should have been done by appellee Lim was to
approach appellee Mrs. Filart and together they should have told appellant Reyes in private that the
latter should leave the party as the celebrant only wanted close friends around. It is necessary that Mrs.
Filart be the one to approach appellant because it was she who invited appellant in that occasion. Were
it not for Mrs. Filart's invitation, appellant could not have suffered such humiliation. For that, appellee
Filart is equally liable. cTDaEH

xxx xxx xxx

The acts of [appellee] Lim are causes of action which are predicated upon mere rudeness or lack of
consideration of one person, which calls not only protection of human dignity but respect of such
dignity. Under Article 20 of the Civil Code, every person who violates this duty becomes liable for
damages, especially if said acts were attended by malice or bad faith. Bad faith does not simply connote
bad judgment or simple negligence. It imports a dishonest purpose or some moral obliquity and
conscious doing of a wrong, a breach of a known duty to some motive or interest or ill-will that partakes
of the nature of fraud (Cojuangco, Jr. v. CA, et al., 309 SCRA 603). 44

Consequently, the Court of Appeals imposed upon Hotel Nikko, Ruby Lim and Dr. Violeta Filart the
solidary obligation to pay Mr. Reyes (1) exemplary damages in the amount of Two Hundred Thousand
Pesos (P200,000); (2) moral damages in the amount of Two Hundred Thousand Pesos (P200,000); and
(3) attorney's fees in the amount of Ten Thousand Pesos (P10,000). 45 On motion for reconsideration,
the Court of Appeals affirmed its earlier decision as the argument raised in the motion had "been amply
discussed and passed upon in the decision sought to be reconsidered." 46

Thus, the instant petition for review. Hotel Nikko and Ruby Lim contend that the Court of Appeals
seriously erred in —

I.

189
. . . NOT APPLYING THE DOCTRINE OF VOLENTI NON FIT INJURIA CONSIDERING THAT BY ITS OWN
FINDINGS, AMAY BISAYA WAS A GATE-CRASHER ASDCaI

II.

. . . HOLDING HOTEL NIKKO AND RUBY LIM JOINTLY AND SEVERALLY LIABLE WITH DR. FILART FOR
DAMAGES SINCE BY ITS OWN RULING, AMAY BISAYA "COULD NOT HAVE SUFFERED SUCH
HUMILIATION," "WERE IT NOT FOR DR. FILART'S INVITATION"

III.

. . . DEPARTING FROM THE FINDINGS OF FACT OF THE TRIAL COURT AS REGARDS THE CIRCUMSTANCES
THAT ALLEGEDLY CAUSED THE HUMILIATION OF AMAY BISAYA

IV.

. . . IN CONCLUDING THAT AMAY BISAYA WAS TREATED UNJUSTLY BECAUSE OF HIS POVERTY,
CONSIDERING THAT THIS WAS NEVER AN ISSUE AND NO EVIDENCE WAS PRESENTED IN THIS REGARD

V.

. . . IN FAILING TO PASS UPON THE ISSUE ON THE DEFECTS OF THE APPELLANT'S BRIEF, THEREBY
DEPARTING FROM THE ACCEPTED AND USUAL COURSE OF JUDICIAL PROCEEDINGS

Petitioners Lim and Hotel Nikko contend that pursuant to the doctrine of volenti non fit injuria, they
cannot be made liable for damages as respondent Reyes assumed the risk of being asked to leave (and
being embarrassed and humiliated in the process) as he was a "gate-crasher." HcDSaT

The doctrine of volenti non fit injuria ("to which a person assents is not esteemed in law as injury" 47 )
refers to self-inflicted injury 48 or to the consent to injury 49 which precludes the recovery of damages
by one who has knowingly and voluntarily exposed himself to danger, even if he is not negligent in doing
so. 50 As formulated by petitioners, however, this doctrine does not find application to the case at bar
because even if respondent Reyes assumed the risk of being asked to leave the party, petitioners, under
Articles 19 and 21 of the New Civil Code, were still under obligation to treat him fairly in order not to
expose him to unnecessary ridicule and shame. iatdcjur

Thus, the threshold issue is whether or not Ruby Lim acted abusively in asking Roberto Reyes, a.k.a.
"Amay Bisaya," to leave the party where he was not invited by the celebrant thereof thereby becoming
liable under Articles 19 and 21 of the Civil Code. Parenthetically, and if Ruby Lim were so liable, whether
or not Hotel Nikko, as her employer, is solidarily liable with her.

As the trial court and the appellate court reached divergent and irreconcilable conclusions concerning
the same facts and evidence of the case, this Court is left without choice but to use its latent power to
review such findings of facts. Indeed, the general rule is that we are not a trier of facts as our jurisdiction
is limited to reviewing and revising errors of law. 51 One of the exceptions to this general rule, however,
obtains herein as the findings of the Court of Appeals are contrary to those of the trial court. 52 The

190
lower court ruled that Ms. Lim did not abuse her right to ask Mr. Reyes to leave the party as she talked
to him politely and discreetly. The appellate court, on the other hand, held that Ms. Lim is liable for
damages as she needlessly embarrassed Mr. Reyes by telling him not to finish his food and to leave the
place within hearing distance of the other guests. Both courts, however, were in agreement that it was
Dr. Filart's invitation that brought Mr. Reyes to the party.

The consequential question then is: Which version is credible?

From an in depth review of the evidence, we find more credible the lower court's findings of fact.
CTHDcE

First, let us put things in the proper perspective.

We are dealing with a formal party in a posh, five-star hotel, 53 for-invitation-only, thrown for the
hotel's former Manager, a Japanese national. Then came a person who was clearly uninvited (by the
celebrant) 54 and who could not just disappear into the crowd as his face is known by many, being an
actor. While he was already spotted by the organizer of the party, Ms. Lim, the very person who
generated the guest list, it did not yet appear that the celebrant was aware of his presence. Ms. Lim,
mindful of the celebrant's instruction to keep the party intimate, would naturally want to get rid of the
"gate-crasher" in the most hush-hush manner in order not to call attention to a glitch in an otherwise
seamless affair and, in the process, risk the displeasure of the celebrant, her former boss. To
unnecessarily call attention to the presence of Mr. Reyes would certainly reflect badly on Ms. Lim's
ability to follow the instructions of the celebrant to invite only his close friends and some of the hotel's
personnel. Mr. Reyes, upon whom the burden rests to prove that indeed Ms. Lim loudly and rudely
ordered him to leave, could not offer any satisfactory explanation why Ms. Lim would do that and risk
ruining a formal and intimate affair. On the contrary, Mr. Reyes, on cross-examination, had unwittingly
sealed his fate by admitting that when Ms. Lim talked to him, she was very close. Close enough for him
to kiss:

Q: And, Mr. Reyes, you testified that Miss Lim approached you while you were at the buffet table?
How close was she when she approached you?

A: Very close because we nearly kissed each other.

Q: And yet, she shouted for you to go down? She was that close and she shouted?

A: Yes. She said, "wag kang kumain, hindi ka imbitado dito, bumaba ka na lang."

Q: So, you are testifying that she did this in a loud voice? HTcADC

xxx xxx xxx

A: Yes. If it is not loud, it will not be heard by many. 55

In the absence of any proof of motive on the part of Ms. Lim to humiliate Mr. Reyes and expose him to
ridicule and shame, it is highly unlikely that she would shout at him from a very close distance. Ms. Lim

191
having been in the hotel business for twenty years wherein being polite and discreet are virtues to be
emulated, the testimony of Mr. Reyes that she acted to the contrary does not inspire belief and is
indeed incredible. Thus, the lower court was correct in observing that —

Considering the closeness of defendant Lim to plaintiff when the request for the latter to leave the party
was made such that they nearly kissed each other, the request was meant to be heard by him only and
there could have been no intention on her part to cause embarrassment to him. It was plaintiff's
reaction to the request that must have made the other guests aware of what transpired between them.
...

Had plaintiff simply left the party as requested, there was no need for the police to take him out. 56

Moreover, another problem with Mr. Reyes's version of the story is that it is unsupported. It is a basic
rule in civil cases that he who alleges proves. Mr. Reyes, however, had not presented any witness to
back his story up. All his witnesses — Danny Rodinas, Pepito Guerrero and Alexander Silva — proved
only that it was Dr. Filart who invited him to the party. 57

Ms. Lim, not having abused her right to ask Mr. Reyes to leave the party to which he was not invited,
cannot be made liable to pay for damages under Articles 19 and 21 of the Civil Code. Necessarily,
neither can her employer, Hotel Nikko, be held liable as its liability springs from that of its employee. 58

Article 19, known to contain what is commonly referred to as the principle of abuse of rights, 59 is not a
panacea for all human hurts and social grievances. Article 19 states:

Art. 19. Every person must, in the exercise of his rights and in the performance of his duties, act with
justice, give everyone his due, and observe honesty and good faith. aIcTCS

Elsewhere, we explained that when "a right is exercised in a manner which does not conform with the
norms enshrined in Article 19 and results in damage to another, a legal wrong is thereby committed for
which the wrongdoer must be responsible." 60 The object of this article, therefore, is to set certain
standards which must be observed not only in the exercise of one's rights but also in the performance of
one's duties. 61 These standards are the following: act with justice, give everyone his due and observe
honesty and good faith. 62 Its antithesis, necessarily, is any act evincing bad faith or intent to injure. Its
elements are the following: (1) There is a legal right or duty; (2) which is exercised in bad faith; (3) for
the sole intent of prejudicing or injuring another. 63 When Article 19 is violated, an action for damages
is proper under Articles 20 or 21 of the Civil Code. Article 20 pertains to damages arising from a violation
of law 64 which does not obtain herein as Ms. Lim was perfectly within her right to ask Mr. Reyes to
leave. Article 21, on the other hand, states:

Art. 21. Any person who willfully causes loss or injury to another in a manner that is contrary to morals,
good customs or public policy shall compensate the latter for the damage.

Article 21 65 refers to acts contra bonus mores and has the following elements: (1) There is an act which
is legal; (2) but which is contrary to morals, good custom, public order, or public policy; and (3) it is done
with intent to injure. 66

192
A common theme runs through Articles 19 and 21, 67 and that is, the act complained of must be
intentional. 68

As applied to herein case and as earlier discussed, Mr. Reyes has not shown that Ms. Lim was driven by
animosity against him. These two people did not know each other personally before the evening of 13
October 1994, thus, Mr. Reyes had nothing to offer for an explanation for Ms. Lim's alleged abusive
conduct except the statement that Ms. Lim, being "single at 44 years old," had a "very strong bias and
prejudice against (Mr. Reyes) possibly influenced by her associates in her work at the hotel with foreign
businessmen." 69 The lameness of this argument need not be belabored. Suffice it to say that a
complaint based on Articles 19 and 21 of the Civil Code must necessarily fail if it has nothing to
recommend it but innuendos and conjectures. HIACEa

Parenthetically, the manner by which Ms. Lim asked Mr. Reyes to leave was likewise acceptable and
humane under the circumstances. In this regard, we cannot put our imprimatur on the appellate court's
declaration that Ms. Lim's act of personally approaching Mr. Reyes (without first verifying from Mrs.
Filart if indeed she invited Mr. Reyes) gave rise to a cause of action "predicated upon mere rudeness or
lack of consideration of one person, which calls not only protection of human dignity but respect of such
dignity." 70 Without proof of any ill-motive on her part, Ms. Lim's act of by-passing Mrs. Filart cannot
amount to abusive conduct especially because she did inquire from Mrs. Filart's companion who told her
that Mrs. Filart did not invite Mr. Reyes. 71 If at all, Ms. Lim is guilty only of bad judgment which, if done
with good intentions, cannot amount to bad faith.

Not being liable for both actual and moral damages, neither can petitioners Lim and Hotel Nikko be
made answerable for exemplary damages 72 especially for the reason stated by the Court of Appeals.
The Court of Appeals held —

Not a few of the rich people treat the poor with contempt because of the latter's lowly station in life.
This has to be limited somewhere. In a democracy, such a limit must be established. Social equality is
not sought by the legal provisions under consideration, but due regard for decency and propriety (Code
Commission, pp. 33-34). And by way of example or correction for public good and to avert further
commission of such acts, exemplary damages should be imposed upon appellees. 73

The fundamental fallacy in the above-quoted findings is that it runs counter with the very facts of the
case and the evidence on hand. It is not disputed that at the time of the incident in question, Mr. Reyes
was "an actor of long standing; a co-host of a radio program over DZRH; a Board Member of the Music
Singer Composer (MUSICO) chaired by popular singer Imelda Papin; a showbiz Coordinator of Citizen
Crime Watch; and 1992 official candidate of the KBL Party for Governor of Bohol; and an awardee of a
number of humanitarian organizations of the Philippines." 74 During his direct examination on rebuttal,
Mr. Reyes stressed that he had income 75 and nowhere did he say otherwise. On the other hand, the
records are bereft of any information as to the social and economic standing of petitioner Ruby Lim.
Consequently, the conclusion reached by the appellate court cannot withstand scrutiny as it is without
basis. CAcDTI

193
All told, and as far as Ms. Lim and Hotel Nikko are concerned, any damage which Mr. Reyes might have
suffered through Ms. Lim's exercise of a legitimate right done within the bounds of propriety and good
faith, must be his to bear alone.

WHEREFORE, premises considered, the petition filed by Ruby Lim and Nikko Hotel Manila Garden is
GRANTED. The Decision of the Court of Appeals dated 26 November 2001 and its Resolution dated 09
July 2002 are hereby REVERSED and SET ASIDE. The Decision of the Regional Trial Court of Quezon City,
Branch 104, dated 26 April 1999 is hereby AFFIRMED. No costs. AcIaST

SO ORDERED.

Puno, Austria-Martinez, Callejo, Sr. and Tinga, JJ., concur.

194
Gashem Shookat Baksh vs. CA, G.R. No. 97336, February 19, 1993

[G.R. No. 97336. February 19, 1993.]

GASHEM SHOOKAT BAKSH, petitioner, vs. HON. COURT OF APPEALS and MARILOU T. GONZALES,
respondents.

Public Attorney's Office for petitioner.

Corleto R. Castro for private respondent.

SYLLABUS

1. REMEDIAL LAW; EVIDENCE; CREDIBILITY OF WITNESSES; FINDINGS OF THE TRIAL COURT; RULE
AND EXCEPTIONS. — It is the rule in this jurisdiction that appellate courts will not disturb the trial court's
findings as to the credibility of witnesses, the latter court having heard the witnesses and having had the
opportunity to observe closely their deportment and manner of testifying, unless the trial court had
plainly overlooked facts of substance or value which, if considered, might affect the result of the case.
(People vs. Garcia, 89 SCRA 440 [1979]; People vs. Bautista, 92 SCRA 465 [1979]; People vs. Abejuela, 92
SCRA 503 [1979]; People vs. Arciaga, 98 SCRA 1 [1980]; People vs. Marzan, 128 SCRA 203 [1984]; People
vs., Alcid, 135 SCRA 280 [1985]; People vs. Sanchez, 199 SCRA 414 [1991]; People vs. Atilano, 204 SCRA
278 [1991]). Equally settled is the rule that only questions of law may be raised in a petition for review
on certiorari under Rule 45 of the Rules of Court. It is not the function of this Court to analyze or weigh
all over again the evidence introduced by the parties before the lower court. There are, however,
recognized exceptions to this rule. Thus, in Medina vs. Asistio, Jr., this Court took the time, again, to
enumerate these exceptions: "(1) When the conclusion is a finding grounded entirely on speculation,
surmises or conjectures (Joaquin v. Navarro, 93 Phil. 257 [1953]); (2) When the inference made is
manifestly mistaken, absurd or impossible (Luna v. Linatok, 74 Phil. 15 [1942]); (3) Where there is a
grave abuse of discretion (Buyco v. People, 95 Phil. 453 [1955]); (4) When the judgment is based on a
misapprehension of facts (Cruz v. Sosing, L-4875, Nov. 27, 1953); (5) When the findings of fact are
conflicting (Casica v. Villaseca, L-9590 Ap. 30, 1957; unrep.) (6) When the Court of Appeals, in making its
findings, went beyond the issues of the case and the same is contrary to the admissions of both
appellant and appellee (Evangelista v. Alto Surety and Insurance Co., 103 Phil. 401 [1958]); (7) The
findings of the Court of Appeals are contrary to those of the trial court (Garcia v. Court of Appeals, 33
SCRA 622 [1970]; Sacay v. Sandiganbayan, 142 SCRA 593 [1986]); (8) When the findings of fact are
conclusions without citation of specific evidence on which they are based (Ibid.,); (9) When the facts set
forth in the petition as well as in the petitioners' main and reply briefs are not disputed by the
respondents (Ibid.,); and (10) The finding of fact of the Court of Appeals is premised on the supposed
absence of evidence and is contradicted by the evidence on record (Salazar v. Gutierrez, 33 SCRA 242
[1970])." Petitioner has not endeavored to point out to Us the existence of any of the above quoted
exceptions in this case. Consequently, the factual findings of the trial and appellate courts must be
respected.

195
2. CIVIL LAW; QUASI-DELICT; TORTS; ART. 21 OF THE CIVIL CODE; CONSTRUED. — Article 2176 of
the Civil Code, which defines a quasi-delict is limited to negligent acts or omissions and excludes the
notion of willfulness or intent. Quasi-delict, known in Spanish legal treatises as culpa aquiliana, is a civil
law concept while torts is an Anglo-American or common law concept. Torts is much broader than culpa
aquiliana because it includes not only negligence, but intentional criminal acts as well such as assault
and battery, false imprisonment and deceit. In the general scheme of the Philippine legal system
envisioned by the Commission responsible for drafting the New Civil Code, intentional and malicious
acts. with certain exceptions, are to. be governed by the Revised Penal Code while negligent acts or
omissions are to be covered by Article 2176 of the Civil Code. In between these opposite spectrums are
injurious acts which, in the absence of Article 21, would have been beyond redress. Thus, Article 21 fills
that vacuum. It is even postulated that together with Articles 19 and 20 of the Civil Code, Article 21 has
greatly broadened the scope of the law on civil wrongs; it has become much more supple and adaptable
than the Anglo-American law on torts. (TOLENTINO, A.M. Commentaries and Jurisprudence on the Civil
Code of the Philippines, vol. 1, 1985 ed., 72).

3. ID.; ID.; ID.; ID.; BREACH OF PROMISE TO MARRY; RULE; RATIONALE. — The existing rule is that a
breach of promise to marry per se is not an actionable wrong (Hermosisima vs. Court of Appeals, 109
Phil. 629 [1960]; Estopa vs. Piansay, 109 Phil. 640 [1960]) Congress deliberately eliminated from the
draft of the New Civil Code the provisions that would have made it so. The reason therefor is set forth in
the report of the Senate Committee on the Proposed Civil Code, from which We quote: "The elimination
of this chapter is proposed. That breach of promise to marry is not actionable has been definitely
decided in the case of De Jesus vs. Syquia (58 Phil. 866 [1933]). The history of breach of promise suits in
the United States and in England has shown that no other action lends itself more readily to abuse by
designing women and unscrupulous men. It is this experience which has led to the abolition of rights of
action in the so-called Heart Balm suits in many of the American states . . ." This notwithstanding, the
said Code contains a provision, Article 21, which is designed to expand the concept of torts or quasi-
delict in this jurisdiction by granting adequate legal remedy for the untold number of moral wrongs
which is impossible for human foresight to specifically enumerate and punish in the statute books
(Philippine National Bank vs. Court of Appeals, 83 SCRA 237 [1978]).

4. ID.; ID.; ID.; ID.; ID.; AWARD OF DAMAGES, JUSTIFIED BECAUSE OF FRAUD AND DECEIT BEHIND
IT; CASE AT BAR. — In the light of the above laudable purpose of Article 21, We are of the opinion, and
so hold, that where a man's promise to marry is in fact the proximate cause of the acceptance of his love
by a woman and his representation to fulfill that promise thereafter becomes the proximate cause of
the giving of herself unto him in a sexual congress, proof that he had, in reality, no intention of marrying
her and that the promise was only a subtle scheme or deceptive device to entice or inveigle her to
accept him and to obtain her consent to the sexual act, could justify the award of damages pursuant to
Article 21 not because of such promise to marry but because of the fraud and deceit behind it and the
willful injury to her honor and reputation which followed thereafter. It is essential, however, that such
injury should have been committed in a manner contrary to morals, good customs or public policy. In
the instant case, respondent Court found that it was the petitioner's "fraudulent and deceptive
protestations of love for and promise to marry plaintiff that made her surrender her virtue and

196
womanhood to him and to live with him on the honest and sincere belief that he would keep said
promise, and it was likewise these fraud and deception on appellant's part that made plaintiff's parents
agree to their daughter's living-in with him preparatory to their supposed marriage." In short, the
private respondent surrendered her virginity, the cherished possession of every single Filipina, not
because of lust but because of moral seduction — the kind illustrated by the Code Commission in its
example earlier adverted to. The petitioner could not be held liable for criminal seduction punished
under either Article 337 or Article 338 of the Revised Penal Code because the private respondent was
above eighteen (18) years of age at the time of the seduction. Prior decisions of this Court clearly
suggest that Article 21 may be applied-in a breach of promise to marry where the woman is a victim of
moral seduction.

5. ID.; PARI DELICTO RULE; DEFINED; NOT APPRECIATED IN CASE AT BAR. — The pari delicto rule
does not apply in this case for while indeed, the private respondent may not have been impelled by the
purest of intentions, she eventually submitted to the petitioner in sexual congress not out of lust, but
because of moral seduction. In fact, it is apparent that she had qualms of conscience about the entire
episode for as soon as she found out that the petitioner was not going to marry her after all, she left
him. She is not, therefore, in pari delicto with the petitioner. Pari delicto means "in equal fault; in a
similar offense or crime; equal in guilt or in legal fault." (Black's Law Dictionary, Fifth ed., 1004). At most,
it could be conceded that she is merely in delicto. "Equity often interferes for the relief of the less guilty
of the parties, where his transgression has been brought about by the imposition or undue influence of
the party on whom the burden of the original wrong principally rests, or where his consent to the
transaction was itself procured by fraud." (37 AM Jur 2d. 401). In Mangayao vs. Lasud, (11 SCRA 158
[1964]) We declared: "Appellants likewise stress that both parties being at fault, there should be no
action by one against the other (Art. 1412, New Civil Code). This rule, however, has been interpreted as
applicable only where the fault on both sides is, more or less, equivalent. It does not apply where one
party is literate or intelligent and the other one is not (c.f. Bough vs. Cantiveros, 40 Phil. 209)."

DECISION

DAVIDE, JR., J p:

This is an appeal by certiorari under Rule 45 of the Rules of Court seeking to review and set aside the
Decision 1 of the respondent Court of Appeals in CA-G.R. CV No. 24256 which affirmed in toto the 16
October 1989 Decision or Branch 38 (Lingayen) of the Regional Trial Court (RTC) of Pangasinan in Civil
Case No. 16503. Presented is the issue of whether or not damages may be recovered for a breach of
promise to marry on the basis of Article 21 of the Civil Code of the Philippines. cdphil

The antecedents of this case are not complicated:

On 27 October 1987, private respondent, without the assistance of counsel, filed with the aforesaid trial
court a complaint 2 for damages against the petitioner for the alleged violation of their agreement to
get married. She alleges in said complaint that: she is twenty-two (22) years old, single, Filipino and a
pretty lass of good moral character and reputation duly respected in her community; petitioner, on the
other hand, is an Iranian citizen residing at the Lozano Apartments, Guilig, Dagupan City, and is an

197
exchange student taking a medical course at the Lyceum Northwestern Colleges in Dagupan City; before
20 August 1987; the latter courted and proposed to marry her; she accepted his love on the condition
that they would get married; they therefore agreed to get married after the end of the school semester,
which was in October of that year; petitioner then visited the private respondent's parents in Bañaga,
Bugallon, Pangasinan to secure their approval to the marriage; sometime in 20 August 1987, the
petitioner forced her to live with him in the Lozano Apartments; she was a virgin before she began living
with him; a week before the filing of the complaint, petitioner's attitude towards her started to change;
he maltreated and threatened to kill her; as a result of such maltreatment, she sustained injuries, during
a confrontation with a representative of the barangay captain of Guilig a day before the filing of the
complaint, petitioner repudiated their marriage agreement and asked her not to live with him anymore
and; the petitioner is already married to someone living in Bacolod City. Private respondent then prayed
for judgment ordering the petitioner to pay her damages in the amount of not less than P45,000.00,
reimbursement for actual expenses amounting to P600.00, attorney's fees and costs, and granting her
such other relief and remedies as may be just and equitable. The complaint was docketed as Civil Case
No 16503.

In his Answer with Counterclaim, 3 petitioner admitted only the personal circumstances of the parties as
averred in the complaint and denied the rest of the allegations either for lack of knowledge or
information sufficient to form a belief as to the truth thereof or because the true facts are those alleged
as his Special and Affirmative Defenses. He thus claimed that he never proposed marriage to or agreed
to be married with the private respondent; he neither sought the consent and approval of her parents
nor forced her to live in his apartment; he did not maltreat her, but only told her to stop coming to his
place because he discovered that she had deceived him by stealing his money and passport; and finally,
no confrontation took place with a representative of the barangay captain. Insisting, in his Counterclaim,
that the complaint is baseless and unfounded and that as a result thereof, he was unnecessarily dragged
into court and compelled to incur expenses, and has suffered mental anxiety and a besmirched
reputation, he prayed for an award of P5,000.00 for miscellaneous expenses and P25,000.00 as moral
damages.

After conducting a pre-trial on 25 January 1988, the trial court issued a Pre-Trial Order 4 embodying the
stipulated facts which the parties had agreed upon, to wit:

"1. That the plaintiff is single and resident (sic) of Bañaga, Bugallon, Pangasinan, while the
defendant is single, Iranian, citizen and resident (sic) of Lozano Apartment, Guilig, Dagupan City since
September 1, 1987 up to the present;

2. That the defendant is presently studying at Lyceum-Northwestern, Dagupan City, College of


Medicine, second year medicine proper.

3. That the plaintiff is (sic) an employee at Mabuhay Luncheonette, Fernandez Avenue, Dagupan
City since July, 1986 up to the present and a (sic) high school graduate;

4. That the parties happened to know each other when the Manager of the Mabuhay
Luncheonette, Johnny Rabino introduced the defendant to the plaintiff on August 3, 1986." cdll

198
After trial on the merits, the lower court, applying Article 21 of the Civil Code, rendered on 16 October
1989 a decision 5 favoring the private respondent. The petitioner was thus ordered to pay the latter
damages and attorney's fees; the dispositive portion of the decision reads:

"IN THE LIGHT of the foregoing consideration, judgment is hereby rendered in favor of the plaintiff and
against the defendant.

1. Condemning (sic) the defendant to pay the plaintiff the sum of twenty thousand (P20,000.00)
pesos as moral damages.

2. Condemning further the defendant to pay the plaintiff the sum of three thousand (P3,000.00)
pesos as atty's fees and two thousand (P2,000.00) pesos at (sic) litigation expenses and to pay the costs.

3. All other claims are denied." 6

The decision is anchored on the trial court's findings and conclusions that (a) petitioner and private
respondent were lovers, (b) private respondent is not a woman of loose morals or questionable virtue
who readily submits to sexual advances, (c) petitioner, through machinations, deceit and false
pretenses, promised to marry private respondent, (d) because of his persuasive promise to marry her,
she allowed herself to be deflowered by him, (e) by reason of that deceitful promise, private respondent
and her parents — in accordance with Filipino customs and traditions — made some preparations for
the wedding that was to be held at the end of October 1987 by looking for pigs and chickens, inviting
friends and relatives and contracting sponsors, (f) petitioner did not fulfill his promise to marry her and
(g) such acts of the petitioner, who is a foreigner and who has abused Philippine hospitality, have
offended our sense of morality, good customs, culture and traditions. The trial court gave full credit to
the private respondent's testimony because, inter alia, she would not have had the temerity and
courage to come to court and expose her honor and reputation to public scrutiny and ridicule if her
claim was false. 7

The above findings and conclusions were culled from the detailed summary of the evidence for the
private respondent in the foregoing decision, digested by the respondent Court as follows:

"According to plaintiff, who claimed that she was a virgin at the time and that she never had a boyfriend
before, defendant started courting her just a few days after they first met. He later proposed marriage
to her several times and she accepted his love as well as his proposal of marriage on August 20, 1987, on
which same day he went with her to her hometown of Banaga, Bugallon, Pangasinan, as he wanted to
meet her parents and inform them of their relationship and their intention to get married. The
photographs Exhs. "A" to "E" (and their submarkings) of defendant with members of plaintiff's family or
with plaintiff, were taken that day. Also on that occasion, defendant told plaintiff's parents and brothers
and sisters that he intended to marry her during the semestral break in October, 1987, and because
plaintiff's parents thought he was good and trusted him, they agreed to his proposal for him to marry
their daughter, and they likewise allowed him to stay in their house and sleep with plaintiff during the
few days that they were in Bugallon. When plaintiff and defendant later returned to Dagupan City, they
continued to live together in defendant's apartment. However, in the early days of October, 1987,

199
defendant would tie plaintiff's hands and feet while he went to school, and he even gave her medicine
at 4 o'clock in the morning that made her sleep the whole day and night until the following day. As a
result of this live-in relationship, plaintiff became pregnant, but defendant gave her some medicine to
abort the foetus. Still plaintiff continued to live with defendant and kept reminding him of his promise to
marry her until he told her that he could not do so because he was already married to a girl in Bacolod
City. That was the time plaintiff left defendant, went home to her parents, and thereafter consulted a
lawyer who accompanied her to the barangay captain in Dagupan City. Plaintiff, her lawyer, her
godmother, and a barangay tanod send by the barangay captain went to talk to defendant to still
convince him to marry plaintiff, but defendant insisted that he could not do so because he was already
married to a girl in Bacolod City, although the truth, as stipulated by the parties at the pre-trial, is that
defendant is still single.

Plaintiff's father, a tricycle driver, also claimed that after defendant had informed them of his desire to
marry Marilou, he already looked for sponsors for the wedding, started preparing for the reception by
looking for pigs and chickens, and even already invited many relatives and friends to the forthcoming
wedding." 8

Petitioner appealed the trial court's decision to the respondent Court of Appeals which docketed the
case as CA-G R. CV No. 24256. In his Brief, 9 he contended that the trial court erred (a) in not dismissing
the case for lack of factual and legal basis and (b) in ordering him to pay moral damages, attorney's fees,
litigation expenses and costs. Cdpr

On 18 February 1991, respondent Court promulgated the challenged decision 10 affirming in toto the
trial court's ruling of 16 October 1989. In sustaining the trial court's findings of fact, respondent Court
made the following analysis:

"First of all, plaintiff, then only 21 years old when she met defendant who was already 23 years old at
the time, does not appear to be a girl of loose morals. It is uncontradicted that she was a virgin prior to
her unfortunate experience with defendant and never had a boyfriend. She is, as described by the lower
court, a barrio lass 'not used and accustomed to the trend of modern urban life', and certainly would
(sic) not have allowed 'herself to be deflowered by the defendant if there was no persuasive promise
made by the defendant to marry her.' In fact, we agree with the lower court that plaintiff and defendant
must have been sweethearts or so the plaintiff must have thought because of the deception of
defendant, for otherwise, she would not have allowed herself to be photographed with defendant in
public in so (sic) loving and tender poses as those depicted in the pictures Exhs. "D" and "E". We cannot
believe, therefore, defendant's pretense that plaintiff was a nobody to him except a waitress at the
restaurant where he usually ate. Defendant in fact admitted that he went to plaintiff's hometown of
Banaga, Bugallon, Pangasinan, at least thrice; at (sic) the town fiesta on February 27, 1987 (p. 54, tsn
May 18, 1988), at (sic) a beach party together with the manager and employees of the Mabuhay
Luncheonette on March 3, 1987 (p. 50, tsn id.), and on April 1, 1987 when he allegedly talked to
plaintiff's mother who told him to marry her daughter (pp. 55-56, tsn id.). Would defendant have left
Dagupan City where he was involved in the serious study of medicine to go to plaintiff's hometown in
Banaga, Bugallon, unless there was (sic) some kind of special relationship between them? And this

200
special relationship must indeed have led to defendant's insincere proposal of marriage to plaintiff,
communicated not only to her but also to her parents, and (sic) Marites Rabino, the owner of the
restaurant where plaintiff was working and where defendant first proposed marriage to her, also knew
of this love affair and defendant's proposal of marriage to plaintiff, which she declared was the reason
why plaintiff resigned from her job at the restaurant after she had accepted defendant's proposal (pp. 6-
7, tsn March 7, 1988).

Upon the other hand, appellant does not appear to be a man of good moral character and must think so
low and have so little respect and regard for Filipino women that he openly admitted that when he
studied in Bacolod City for several years where he finished his B.S. Biology before he came to Dagupan
City to study medicine, he had a common-law wife in Bacolod City. In other words, he also lived with
another woman in Bacolod City but did not marry that woman, just like what he did to plaintiff. It is not
surprising, then, that he felt so little compunction or remorse in pretending to love and promising to
marry plaintiff, a young, innocent, trustful country girl, in order to satisfy his lust on her." 11

and then concluded:

"In sum, we are strongly convinced and so hold that it was defendant-appellant's fraudulent and
deceptive protestations of love for and promise to marry plaintiff that made her surrender her virtue
and womanhood to him and to live with him on the honest and sincere belief that he would keep said
promise, and it was likewise these (sic) fraud and deception on appellant's part that made plaintiff's
parents agree to their daughter's living-in with him preparatory to their supposed marriage. And as
these acts of appellant are palpably and undoubtedly against morals, good customs, and public policy,
and are even gravely and deeply derogatory and insulting to our women, coming as they do from a
foreigner who has been enjoying the hospitality of our people and taking advantage of the opportunity
to study in one of our institutions of learning, defendant-appellant should indeed be made, under Art.
21 of the Civil Code of the Philippines, to compensate for the moral damages and injury that he had
caused plaintiff, as the lower court ordered him to do in its decision in this case." 12

Unfazed by his second defeat, petitioner filed the instant petition on 26 March 1991; he raises therein
the single issue of whether or not Article 21 of the Civil Code applies to the case at bar. 13

It is petitioner's thesis that said Article 21 is not applicable because he had not committed any moral
wrong or injury or violated any good custom or public policy; he has not professed love or proposed
marriage to the private respondent; and he has never maltreated her. He criticizes the trial court for
liberally invoking Filipino customs, traditions and culture, and ignoring the fact that since he is a
foreigner, he is not conversant with such Filipino customs, traditions and culture. As an Iranian Moslem,
he is not familiar with Catholic and Christian ways. He stresses that even if he had made a promise to
marry, the subsequent failure to fulfill the same is excusable or tolerable because of his Moslem
upbringing; he then alludes to the Muslim Code which purportedly allows a Muslim to take four (4)
wives and concludes that on the basis thereof, the trial court erred in ruling that he does not possess
good moral character. Moreover, his controversial "common law wife" is now his legal wife as their
marriage had been solemnized in civil ceremonies in the Iranian Embassy. As to his unlawful

201
cohabitation with the private respondent, petitioner claims that even if responsibility could be pinned
on him for the live-in relationship, the private respondent should also be faulted for consenting to an
illicit arrangement. Finally, petitioner asseverates that even if it was to be assumed arguendo that he
had professed his love to the private respondent and had also promised to marry her, such acts would
not be actionable in view of the special circumstances of the case. The mere breach of promise is not
actionable. 14

On 26 August 1991, after the private respondent had filed her Comment to the petition and the
petitioner had filed his Reply thereto, this Court gave due course to the petition and required the parties
to submit their respective Memoranda, which they subsequently complied with. prLL

As may be gleaned from the foregoing summation of the petitioner's arguments in support of his thesis,
it is clear that questions of fact, which boil down to the issue of the credibility of witnesses, are also
raised. It is the rule in this jurisdiction that appellate courts will not disturb the trial court's findings as to
the credibility of witnesses, the latter court having heard the witnesses and having had the opportunity
to observe closely their deportment and manner of testifying, unless the trial court had plainly
overlooked facts of substance or value which, if considered, might affect the result of the case. 15

Petitioner has miserably failed to convince Us that both the appellate and trial courts had overlooked
any fact of substance or value which could alter the result of the case.

Equally settled is the rule that only questions of law may be raised in a petition for review on certiorari
under Rule 45 of the Rules of Court. It is not the function of this Court to analyze or weigh all over again
the evidence introduced by the parties before the lower court. There are, however, recognized
exceptions to this rule. Thus, in Medina vs. Asistio, Jr., 16 this Court took the time, again, to enumerate
these exceptions:

xxx xxx xxx

"(1) When the conclusion is a finding grounded entirely on speculation, surmises or conjectures
(Joaquin v. Navarro, 93 Phil. 257 [1953]); (2) When the inference made is manifestly mistaken, absurd or
impossible (Luna v. Linatok, 74 Phil. 15 [1942]); (3) Where there is a grave abuse of discretion (Buyco v.
People, 95 Phil. 453 [1955]); (4) When the judgment is based on a misapprehension of facts (Cruz v.
Sosing, L-4875, Nov. 27, 1953); (5) When the findings of fact are conflicting (Casica v. Villaseca, L-9590
Ap. 30, 1957; unrep.) (6) When the Court of Appeals, in making its findings, went beyond the issues of
the case and the same is contrary to the admissions of both appellant and appellee (Evangelista v. Alto
Surety and Insurance Co., 103 Phil. 401 [1958]); (7) The findings of the Court of Appeals are contrary to
those of the trial court (Garcia v. Court of Appeals, 33 SCRA 622 [1970]; Sacay v. Sandiganbayan, 142
SCRA 593 [1986]); (8) When the findings of fact are conclusions without citation of specific evidence on
which they are based (Ibid.,); (9) When the facts set forth in the petition as well as in the petitioners'
main and reply briefs are not disputed by the respondents (Ibid.,); and (10) The finding of fact of the
Court of Appeals is premised on the supposed absence of evidence and is contradicted by the evidence
on record (Salazar v. Gutierrez, 33 SCRA 242 [1970])."

202
Petitioner has not endeavored to point out to Us the existence of any of the above quoted exceptions in
this case. Consequently, the factual findings of the trial and appellate courts must be respected.

And now to the legal issue.

The existing rule is that a breach of promise to marry per se is not an actionable wrong. 17 Congress
deliberately eliminated from the draft of the New Civil Code the provisions that would have made it so.
The reason therefor is set forth in the report of the Senate Committee on the Proposed Civil Code, from
which We quote:

"The elimination of this chapter is proposed. That breach of promise to marry is not actionable has been
definitely decided in the case of De Jesus vs. Syquia. 18 The history of breach of promise suits in the
United States and in England has shown that no other action lends itself more readily to abuse by
designing women and unscrupulous men. It is this experience which has led to the abolition of rights of
action in the so-called Heart Balm suits in many of the American states .." 19

This notwithstanding, the said Code contains a provision, Article 21, which is designed to expand the
concept of torts or quasi-delict in this jurisdiction by granting adequate legal remedy for the untold
number of moral wrongs which is impossible for human foresight to specifically enumerate and punish
in the statute books. 20

As the Code Commission itself stated in its Report:

"'But the Code Commission has gone farther than the sphere of wrongs defined or determined by
positive law. Fully sensible that there are countless gaps in the statutes, which leave so many victims of
moral wrongs helpless, even though they have actually suffered material and moral injury, the
Commission has deemed it necessary, in the interest of justice, to incorporate in the proposed Civil Code
the following rule: LLpr

'ARTICLE 23. Any person who wilfully causes loss or injury to another in a manner that is contrary to
morals, good customs or public policy shall compensate the latter for the damage.'

'An example will illustrate the purview of the foregoing norm: 'A' seduces the nineteen-year old
daughter of 'X.' A promise of marriage either has not been made, or can not be proved. The girl becomes
pregnant. Under the present laws, there is no crime, as the girl is above eighteen years of age. Neither
can any civil action for breach of promise of marriage be filed. Therefore, though the grievous moral
wrong has been committed, and though the girl and her family have suffered incalculable moral
damage, she and her parents cannot bring any action for damages. But under the proposed article, she
and her parents would have such a right of action.

Thus at one stroke, the legislator, if the foregoing rule is approved, would vouchsafe adequate legal
remedy for that untold number of moral wrongs which it is impossible for human foresight to provide
for specifically in the statutes." 21

Article 2176, of the Civil Code, which defines a quasi-delict thus:

203
"Whoever by act or omission causes damage to another, there being fault or negligence, is obliged to
pay for the damage done. Such fault or negligence, if there is no pre-existing contractual relation
between the parties, is called a quasi-delict and is governed by the provisions of this Chapter."

is limited to negligent acts or omissions and excludes the notion of willfulness or intent. Quasi-delict,
known in Spanish legal treatises as culpa aquiliana, is a civil law concept while torts is an Anglo-American
or common law concept. Torts is much broader than culpa aquiliana because it includes not only
negligence, but intentional criminal acts as well such as assault and battery, false imprisonment and
deceit. In the general scheme of the Philippine legal system envisioned by the Commission responsible
for drafting the New Civil Code, intentional and malicious acts. with certain exceptions, are to. be
governed by the Revised Penal Code while negligent acts or omissions are to be covered by Article 2176
of the Civil Code. 22 In between these opposite spectrums are injurious acts which, in the absence of
Article 21, would have been beyond redress. Thus, Article 21 fills that vacuum. It is even postulated that
together with Articles 19 and 20 of the Civil Code, Article 21 has greatly broadened the scope of the law
on civil wrongs; it has become much more supple and adaptable than the Anglo-American law on torts.
23

In the light of the above laudable purpose of Article 21, We are of the opinion, and so hold, that where a
man's promise to marry is in fact the proximate cause of the acceptance of his love by a woman and his
representation to fulfill that promise thereafter becomes the proximate cause of the giving of herself
unto him in a sexual congress, proof that he had, in reality, no intention of marrying her and that the
promise was only a subtle scheme or deceptive device to entice or inveigle her to accept him and to
obtain her consent to the sexual act, could justify the award of damages pursuant to Article 21 not
because of such promise to marry but because of the fraud and deceit behind it and the willful injury to
her honor and reputation which followed thereafter. It is essential, however, that such injury should
have been committed in a manner contrary to morals, good customs or public policy.

In the instant case, respondent Court found that it was the petitioner's "fraudulent and deceptive
protestations of love for and promise to marry plaintiff that made her surrender her virtue and
womanhood to him and to live with him on the honest and sincere belief that he would keep said
promise, and it was likewise these fraud and deception on appellant's part that made plaintiff's parents
agree to their daughter's living-in with him preparatory to their supposed marriage." 24 In short, the
private respondent surrendered her virginity, the cherished possession of every single Filipina, not
because of lust but because of moral seduction — the kind illustrated by the Code Commission in its
example earlier adverted to. The petitioner could not be held liable for criminal seduction punished
under either Article 337 or Article 338 of the Revised Penal Code because the private respondent was
above eighteen (18) years of age at the time of the seduction.

Prior decisions of this Court clearly suggest that Article 21 may be applied-in a breach of promise to
marry where the woman is a victim of moral seduction. Thus, in Hermosisima vs. Court of Appeals, 25
this Court denied recovery of damages to the woman because: LibLex

204
" . . . we find ourselves unable to say that petitioner is morally guilty of seduction, not only because he is
approximately ten (10) years younger than the complainant — who was around thirty-six (36) years of
age, and as highly enlightened as a former high school teacher and a life insurance agent are supposed
to be — when she became intimate with petitioner, then a mere apprentice pilot, but, also, because the
court of first instance found that, complainant 'surrendered herself' to petitioner because,
'overwhelmed by her love' for him, she 'wanted to bind' him 'by having a fruit of their engagement even
before they had the benefit of clergy.'"

In Tanjanco vs. Court of Appeals, 26 while this Court likewise hinted at possible recovery if there had
been moral seduction, recovery was eventually denied because We were not convinced that such
seduction existed. The following enlightening disquisition and conclusion were made in the said case:

"The Court of Appeals seems to have overlooked that the example set forth in the Code Commission's
memorandum refers to a tort upon a minor who had been seduced. The essential feature is seduction,
that in law is more than mere sexual intercourse, or a breach of a promise of marriage; it connotes
essentially the idea of deceit, enticement, superior power or abuse of confidence on the part of the
seducer to which the woman has yielded (U.S. vs. Buenaventura, 27 Phil. 121; U.S. vs. Arlante, 9 Phil.
595).

It has been ruled in the Buenaventura case (supra) that —

'To constitute seduction there must in all cases be some sufficient promise or inducement and the
woman must yield because of the promise or other inducement. If she consents merely from carnal lust
and the intercourse is from mutual desire, there is no seduction (43 Cent Dig. tit. Seduction, par. 56).
She must be induced to depart from the path of virtue by the use of some species of arts, persuasions
and wiles, which are calculated to have and do have that effect, and which result in her ultimately
submitting her person to the sexual embraces of her seducer' (27 Phil. 123).

And in American Jurisprudence we find:

'On the other hand, in an action by the woman, the enticement, persuasion or deception is the essence
of the injury; and a mere proof of intercourse is insufficient to warrant a recovery.

Accordingly it is not seduction where the willingness arises out sexual desire or curiosity of the female,
and the defendant merely affords her the needed opportunity for the commission of the act. It has been
emphasized that to allow a recovery in all such cases would tend to the demoralization of the female
sex, and would be a reward for unchastity by; which a class of adventuresses would be swift to profit.'
(47 Am. Jur. 662).

xxx xxx xxx

Over and above the partisan allegations, the facts stand out that for one whole year, from 1958 to 1959,
the plaintiff-appellee, a woman of adult age, maintained intimate sexual relations with appellant, with
repeated acts of intercourse. Such conduct is incompatible with the idea of seduction. Plainly there is
here voluntariness and mutual passion; for had the appellant been deceived, had she surrendered

205
exclusively because of the deceit, artful persuasions and wiles of the defendant, she would not have
again yielded to his embraces, much less for one year, without exacting early fulfillment of the alleged
promises of marriage, and would have cut short all sexual relations upon finding that defendant did not
intend to fulfill his promise. Hence, we conclude that no case is made under Article 21 of the Civil Code,
and no other cause of action being alleged, no error was committed by the Court of First Instance in
dismissing the complaint." 27

In his annotations on the Civil Code, 28 Associate Justice Edgardo L. Paras, who recently retired from this
Court, opined that in a breach of promise to marry where there had been carnal knowledge, moral
damages may be recovered:

" . . . if there be criminal or moral seduction, but not if the intercourse was due to mutual lust.
(Hermosisima vs. Court of Appeals, L-14628, Sept. 30, 1960; Estopa vs. Piansay, Jr., L-14733, Sept. 30,
1960; Batarra vs. Marcos, 7 Phil. 56 (sic); Beatriz Galang vs. Court of Appeals, et al., L-17248, Jan. 29,
1962). (In other words, if the CAUSE be the promise to marry, and the EFFECT be the carnal knowledge,
there is a chance that there was criminal or moral seduction, hence recovery of moral damages will
prosper. If it be the other way around, there can be no recovery of moral damages, because here mutual
lust has intervened). . . . ."

together with "ACTUAL damages, should there be any, such as the expenses for the wedding
preparations (See Domalagon v. Bolifer, 33 Phil. 471)." LLpr

Senator Arturo M. Tolentino 29 is also of the same persuasion:

"It is submitted that the rule in Batarra vs. Marcos 30 still subsists, notwithstanding the incorporation of
the present article 31 in the Code. The example given by the Code Commission is correct, if there was
seduction, not necessarily in the legal sense, but in the vulgar sense of deception. But when the sexual
act is accomplished without any deceit or qualifying circumstance of abuse of authority or influence, but
the woman, already of age, has knowingly given herself to a man, it cannot be said that there is an injury
which can be the basis for indemnity.

But so long as there is fraud, which is characterized by wilfullness (sic), the action lies. The court,
however, must weigh the degree of fraud, if it is sufficient to deceive the woman under the
circumstances, because an act which would deceive a girl sixteen years of age may not constitute deceit
as to an experienced woman thirty years of age. But so long as there is a wrongful act and a resulting
injury, there should be civil liability, even if the act is not punishable under the criminal law and there
should have been an acquittal or dismissal of the criminal case for that reason."

We are unable to agree with the petitioner's alternative proposition to the effect that granting, for
argument's sake, that he did promise to marry the private respondent, the latter is nevertheless also at
fault. According to him, both parties are in pari delicto; hence, pursuant to Article 1412(1) of the Civil
Code and the doctrine laid down in Batarra vs. Marcos, 32 the private respondent cannot recover
damages from the petitioner. The latter even goes as far as stating that if the private respondent had
"sustained any injury or damage in their relationship, it is primarily because of her own doing," 33 for:

206
" . . . She is also interested in the petitioner as the latter will become a doctor sooner or later. Take
notice that she is a plain high school graduate and a mere employee . . (Annex C ) or a waitress (TSN, p.
51, January 25, 1988) in a luncheonette and without doubt, is in need of a man who can give her
economic security. Her family is in dire need of financial assistance (TSN, pp. 51-53, May 18, 1988). And
this predicament prompted her to accept a proposition that may have been offered by the petitioner."
34

These statements reveal the true character and motive of the petitioner. It is clear that he harbors a
condescending, if not sarcastic, regard for the private respondent on account of the latter's ignoble
birth, inferior educational background, poverty and, as perceived by him, dishonorable employment.
Obviously then, from the very beginning, he was not at all moved by good faith and an honest motive.
Marrying with a woman so circumstanced could not have even remotely occurred to him. Thus, his
profession of love and promise to marry were empty words directly intended to fool, dupe, entice,
beguile and deceive the poor woman into believing that indeed, he loved her and would want her to be
his life partner. His was nothing but pure lust which he wanted satisfied by a Filipina who honestly
believed that by accepting his proffer of love and proposal of marriage, she would be able to enjoy a life
of ease and security. Petitioner clearly violated the Filipino's concept of morality and so brazenly defied
the traditional respect Filipinos have for their women. It can even be said that the petitioner committed
such deplorable acts in blatant disregard of Article 19 of the Civil Code which directs every person to act
with justice, give everyone his due and observe honesty and good faith in the exercise of his rights and
in the performance of his obligations.

No foreigner must be allowed to make a mockery of our laws, customs and traditions.

The pari delicto rule does not apply in this case for while indeed, the private respondent may not have
been impelled by the purest of intentions, she eventually submitted to the petitioner in sexual congress
not out of lust, but because of moral seduction. In fact, it is apparent that she had qualms of conscience
about the entire episode for as soon as she found out that the petitioner was not going to marry her
after all, she left him. She is not, therefore, in pari delicto with the petitioner. Pari delicto means "in
equal fault; in a similar offense or crime; equal in guilt or in legal fault." 35 At most, it could be
conceded that she is merely in delicto. cdphil

"Equity often interferes for the relief of the less guilty of the parties, where his transgression has been
brought about by the imposition or undue influence of the party on whom the burden of the original
wrong principally rests, or where his consent to the transaction was itself procured by fraud." 36

In Mangayao vs. Lasud, 37 We declared:

"Appellants likewise stress that both parties being at fault, there should be no action by one against the
other (Art. 1412, New Civil Code). This rule, however, has been interpreted as applicable only where the
fault on both sides is, more or less, equivalent. It does not apply where one party is literate or intelligent
and the other one is not (c.f. Bough vs. Cantiveros, 40 Phil. 209)."

207
We should stress, however, that while We find for the private respondent, let it not be said that this
Court condones the deplorable behavior of her parents in letting her and the petitioner stay together in
the same room in their house after giving approval to their marriage. It is the solemn duty of parents to
protect the honor of their daughters and infuse upon them the higher values of morality and dignity.

WHEREFORE, finding no reversible error in the challenged decision, the instant petition is hereby
DENIED, with costs against the petitioner. cdll

SO ORDERED.

Feliciano, J ., Bidin, Romero and Melo, JJ ., concur.

Gutierrez, Jr., J ., is on terminal leave.

208
Pe vs. Pe, G.R. No. L-17396, May 30, 1962

[G.R. No. L-17396. May 30, 1962.]

CECILIO PE, ET AL., plaintiffs-appellants, vs. ALFONSO PE, defendant-appellee.

Cecilio L. Pe for and in his own behalf as plaintiff-appellant.

Leodegario L. Mogol for defendant-appellee.

SYLLABUS

1. DAMAGES; ACTS CONTRARY TO MORALS. — Defendant won Lolita's affection thru an ingenious
scheme or trickery and seduced her to the extent of making her fall in love with him. This is shown by
the fact that defendant frequented the house of Lolita on the pretext that he wanted her to teach him
how to pray the rosary. Because of the frequency of his visits to the latter's family who was allowed free
access because he was a collateral relative and was considered as a member of her family, the two
eventually fell in love with each other and conducted clandestine love affairs not only in Gasan but in
Boac where Lolita used to teach in a barrio school. When the rumors about their illicit affair reached the
knowledge of her parents, defendant was forbidden from going to their house and even from seeing
Lolita. Plaintiff even filed deportation proceedings against defendant who is a Chinese national.
Nevertheless, defendant continued his love affairs with Lolita until she disappeared from the parental
home, Held; The wrong defendant has caused Lolita and her family is indeed immeasurable considering
the fact that he is a married man. Verily, he has committed an injury to Lolita's family in a manner
contrary to morals, good customs and public policy as contemplated in Article 21 of the New Civil Code.

DECISION

BAUTISTA ANGELO, J p:

Plaintiffs brought this action before the Court of First Instance of Manila to recover moral,
compensatory, exemplary and corrective damages in the amount of P94,000.00, exclusive of attorney's
fees and expenses of litigation.

Defendant, after denying some allegations contained in the complaint, set up as a defense that the facts
alleged therein, even if true, do not constitute a valid cause of action.

After trial, the lower court, after finding that defendant had carried on a love affair with one Lolita Pe,
an unmarried woman, being a married man himself, declared that defendant cannot be held liable for
moral damages it appearing that plaintiffs failed to prove that defendant, being aware of his marital
status, deliberately and in bad faith tried to win Lolita's affection. So it rendered decision dismissing the
complaint.

Plaintiffs brought this case on appeal before this Court on the ground that the issues involved are purely
of law.

209
The facts as found by the trial court are: Plaintiffs are the parents, brothers and sisters of one Lolita Pe.
At the time of her disappearance on April 14, 1957, Lolita was 24 years old and unmarried. Defendant is
a married man and works as agent of the La Perla Cigar and Cigarette Factory. He used to stay in the
town of Gasan, Marinduque, in connection with his aforesaid occupation. Lolita was staying with her
parents in the same town. Defendant was an adopted son of a Chinaman named Pe Beco, a collateral
relative of Lolita's father. Because of such fact and the similarity in their family name, defendant became
close to the plaintiffs who regarded him as a member of their family. Sometime in 1952, defendant
frequented the house of Lolita on the pretext that he wanted her to teach him how to pray the rosary.
The two eventually fell in love with each other and conducted clandestine trysts not only in the town of
Gasan but also in Boac where Lolita used to teach in a barrio school. They exchanged love notes with
each other the contents of which reveal not only their infatuation for each other but also the extent to
which they had carried their relationship. The rumors about their love affair reached the ears of Lolita's
parents sometime in 1955, and since then defendant was forbidden from going to their house and from
further seeing Lolita. The plaintiffs even filed deportation proceedings against defendant who is a
Chinese national. The affair between defendant and Lolita continued nonetheless.

Sometime in April, 1957, Lolita was staying with her brothers and sisters at their residence at 54-B
España Extension, Quezon City. On April 14, 1957, Lolita disappeared from said house. After she left, her
brothers and sisters checked up her things and found that Lolita's clothes were gone. However, plaintiffs
found a note on a crumpled piece of paper inside Lolita's aparador. Said note, written on a small slip of
paper approximately 4" by 3" in size, was in a handwriting recognized to be that of defendant. In English
it reads:

"Honey, suppose I leave here on Sunday night, and that's 13th of this month and we will have a date on
the 14th, that's Monday morning at 10 a.m.

Reply

Love"

The disappearance of Lolita was reported to the police authorities and the NBI but up to the present
there is no news or trace of her whereabouts.

The present action is based on Article 21 of the new Civil Code which provides:

"Any person who wilfully causes loss or injury to another in a manner which is contrary to morals, good
customs or public policy shall compensate the latter for the damage."

There is no doubt that the claim of plaintiffs for damages is based on the fact that defendant, being a
married man, carried on a love affair with Lolita Pe thereby causing plaintiffs injury in a manner contrary
to morals, good customs and public policy. But in spite of the fact that plaintiffs have clearly established
that an illicit affair was carried on between defendant and Lolita which caused great damage to the
name and reputation of plaintiffs who are her parents, brothers and sisters, the trial court considered
their complaint not actionable for the reason that they failed to prove that defendant deliberately and

210
in bad faith tried to win Lolita's affection. Thus, the trial court said: "In the absence of proof on this
point, the court may not presume that it was the defendant who deliberately induced such relationship.
We cannot be unmindful of the uncertainties and sometimes inexplicable mysteries of the human
emotions. It is a possibility that the defendant and Lolita simply fell in love with each other, not only
without any desire on their part, but also against their better judgment and in full consciousness of the
disastrous consequences that such an affair would naturally bring on both of them. This is specially so
with respect to Lolita, being an unmarried woman, falling in love with defendant who is a married man."

We disagree with this view. The circumstances under which defendant tried to win Lolita's affection
cannot lead to any other conclusion than that it was he who, thru an ingenious scheme or trickery,
seduced the latter to the extent of making her fall in love with him. This is shown by the fact that
defendant frequented the house of Lolita on the pretext that he wanted her to teach him how to pray
the rosary. Because of the frequency of his visits to the latter's family who was allowed free access
because he was a collateral relative and was considered as a member of her family, the two eventually
fell in love with each other and conducted clandestine love affairs not only in Gasan but in Boac where
Lolita used to teach in a barrio school. When the rumors about their illicit affair reached the knowledge
of her parents, defendant was forbidden from going to their house and even from seeing Lolita.
Plaintiffs even filed deportation proceedings against defendant who is a Chinese national. Nevertheless,
defendant continued his love affairs with Lolita until she disappeared from the parental home. Indeed,
no other conclusion can be drawn from this chain of events than that defendant not only deliberately,
but through a clever strategy, succeeded in winning the affection and love of Lolita to the extent of
having illicit relations with her. The wrong he has caused her and her family is indeed immeasurable
considering the fact that he is a married man. Verily, he has committed an injury to Lolita's family in a
manner contrary to morals, good customs and public policy as contemplated in Article 21 of the new
Civil Code.

WHEREFORE, the decision appealed from is reversed. Defendant is hereby sentenced to pay the
plaintiffs the sum of P5,000.00 as damages and P2,500.00 as attorney's fees and expenses of litigation.
Costs against appellee.

Padilla, Labrador, Concepcion, Reyes, J.B.L., Barrera, Paredes and Dizon, JJ. concur.

211
Globe Mackay Cable vs. CA, 176 SCRA[G.R. No. 81262. August 25, 1989.]

GLOBE MACKAY CABLE AND RADIO CORP., and HERBERT C. HENDRY, petitioners, vs. THE HONORABLE
COURT OF APPEALS and RESTITUTO M. TOBIAS, respondents.

Atencia & Arias Law Offices for petitioners.

Romulo C. Felizmena for private respondent.

SYLLABUS

1. CIVIL LAW; HUMAN RELATIONS; PRINCIPLE OF ABUSE OF RIGHTS; STANDARDS. — Article 19 of


the Civil Code known to contain what is commonly referred to as the principle of abuse of rights, sets
certain standards which must be observed not only in the exercise of one's rights but also in the
performance of one's duties. These standards are the following: to act with justice; to give everyone his
due; and to observe honesty and good faith.

2. ID.; ID.; ID.; REMEDY AVAILABLE. — In determining whether or not the principle of abuse of
rights may be invoked, there is no rigid test which can be applied. While the Court has not hesitated to
apply Article 19 whether the legal and factual circumstances called for its application the question of
whether or not the principle of abuse of rights has been violated resulting in damages under Article 20
or Article 21 or other applicable provision of law, depends on the circumstances of each case.

3. LABOR AND SOCIAL LEGISLATION; LABOR CODE; EMPLOYMENT; RIGHT OF EMPLOYER TO


DISMISS AN EMPLOYEE SHOULD NOT BE CONFUSED WITH THE MANNER IN WHICH THE RIGHT IS
EXERCISED. — The right of the employer to dismiss an employee should not be confused with the
manner in which the right is exercised and the effects flowing therefrom. If the dismissal is done
abusively, then the employer is liable for damages to the employee.

4. REMEDIAL LAW; CRIMINAL PROSECUTION; RIGHT TO FILE CRIMINAL CASES; CAN NOT BE
EXERCISED MALICIOUSLY AND IN BAD FAITH. — While sound principles of justice and public policy
dictate that persons shall have free resort to the courts for redress of wrongs and vindication of their
rights, the right to institute criminal prosecutions can not be exercised maliciously and in bad faith.

5. ID.; ID.; ID.; SHOULD NOT BE USED AS A WEAPON TO FORCE A DEBTOR TO PAY HIS OBLIGATION.
— The right to file criminal complaints should not be used as a weapon to force an alleged debtor to pay
an indebtedness. To do so would be a clear perversion of the function of the criminal processes and of
the courts of justice.

6. CIVIL LAW; DAMAGES; MALICIOUS PROSECUTION; PROOF NECESSARY. — To constitute


malicious prosecution, there must be proof that the prosecution was prompted by a design to vex and
humiliate a person and that it was initiated deliberately by the defendant knowing that the charges
were false and groundless.

212
7. ID.; ID.; ID.; MERE FILING OF A SUIT, NOT CONSTITUTE THEREOF. — The filing of a suit, by itself,
does not render a person liable for malicious prosecution.

8. ID.; ID.; ID.; MERE DISMISSAL BY THE FISCAL OF THE COMPLAINT NOT A GROUND FOR THE
AWARD. — The mere dismissal by the fiscal of the criminal complaint is not a ground for an award of
damages for malicious prosecution if there is no competent evidence to show that the complainant had
acted in bad faith.

9. ID.; ID.; PRINCIPLE OF DAMNUM ABSQUE INJURIA. — According to the principle of damnum
absque injuria, damage or loss which does not constitute a violation of a legal right or amount to a legal
wrong is not actionable.

10. ID.; ID.; MORAL DAMAGES; RECOVERABLE IN CASES MENTIONED IN ARTICLE 21 OF THE CIVIL
CODE. —[P]er express provision of Article 2219 (10) of the New Civil Code, moral damages are
recoverable in the cases mentioned in Article 21 of said Code.

11. ID.; ID.; EXEMPLARY DAMAGES; IF GROSS NEGLIGENCE WARRANTS THE AWARD THEREOF WITH
MORE REASON WHEN THE ACT IS DELIBERATE, MALICIOUS AND IN BAD FAITH. — That if gross
negligence warrants the award of exemplary damages, with more reason is its imposition justified when
the act performed is deliberate, malicious and tainted with bad faith.

DECISION

CORTES, J p:

Private respondent Restituto M. Tobias was employed by petitioner Globe Mackay Cable and Radio
Corporation (GLOBE MACKAY) in a dual capacity as a purchasing agent and administrative assistant to
the engineering operations manager. In 1972, GLOBE MACKAY discovered fictitious purchases and other
fraudulent transactions for which it lost several thousands of pesos.

According to private respondent it was he who actually discovered the anomalies and reported them on
November 10, 1972 to his immediate superior Eduardo T. Ferraren and to petitioner Herbert C. Hendry
who was then the Executive Vice-President and General Manager of GLOBE MACKAY.

On November 11, 1972, one day after private respondent Tobias made the report, petitioner Hendry
confronted him by stating that he was the number one suspect, and ordered him to take a one week
forced leave, not to communicate with the office, to leave his table drawers open, and to leave the
office keys.

On November 20, 1972, when private respondent Tobias returned to work after the forced leave,
petitioner Hendry went up to him and called him a "crook" and a "swindler." Tobias was then ordered to
take a lie detector test. He was also instructed to submit specimen of his handwriting, signature, and
initials for examination by the police investigators to determine his complicity in the anomalies.

213
On December 6, 1972, the Manila police investigators submitted a laboratory crime report (Exh. "A")
clearing private respondent of participation in the anomalies.

Not satisfied with the police report, petitioners hired a private investigator, retired Col. Jose G.
Fernandez, who on December 10, 1972, submitted a report (Exh. "2") finding Tobias guilty. This report
however expressly stated that further investigation was still to be conducted.

Nevertheless, on December 12, 1972, petitioner Hendry issued a memorandum suspending Tobias from
work preparatory to the filing of criminal charges against him.

On December 19, 1972, Lt. Dioscoro V. Tagle, Metro Manila Police Chief Document Examiner, after
investigating other documents pertaining to the alleged anomalous transactions, submitted a second
laboratory crime report (Exh. "B") reiterating his previous finding that the handwritings, signatures, and
initials appealing in the checks and other documents involved in the fraudulent transactions were not
those of Tobias. The lie detector tests conducted on Tobias also yielded negative results.

Notwithstanding the two police reports exculpating Tobias from the anomalies and the fact that the
report of the private investigator, was, by its own terms, not yet complete, petitioners filed with the City
Fiscal of Manila a complaint for estafa through falsification of commercial documents, later amended to
just estafa. Subsequently, five other criminal complaints were filed against Tobias, four of which were
for estafa through falsification of commercial document while the fifth was for violation of Article 290 of
the Revised Penal Code (Discovering Secrets Through Seizure of Correspondence). Two of these
complaints were refiled with the Judge Advocate General's Office, which however, remanded them to
the fiscal's office. All of the six criminal complaints were dismissed by the fiscal. Petitioners appealed
four of the fiscal's resolutions dismissing the criminal complaints with the Secretary of Justice, who,
however, affirmed their dismissal.

In the meantime, on January 17, 1973, Tobias received a notice (Exh. "F") from petitioners that his
employment has been terminated effective December 13, 1972. Whereupon, Tobias filed a complaint
for illegal dismissal. The labor arbiter dismissed the complaint. On appeal, the National Labor Relations
Commission (NLRC) reversed the labor arbiter's decision. However, the Secretary of Labor, acting on
petitioners' appeal from the NLRC ruling, reinstated the labor arbiter's decision. Tobias appealed the
Secretary of Labor's order with the Office of the President. During the pendency of the appeal with said
office, petitioners and private respondent Tobias entered into a compromise agreement regarding the
latter's complaint for illegal dismissal.

Unemployed, Tobias sought employment with the Republic Telephone Company (RETELCO). However,
petitioner Hendry, without being asked by RETELCO, wrote a letter to the latter stating that Tobias was
dismissed by GLOBE MACKAY due to dishonesty.

Private respondent Tobias filed a civil case for damages anchored on alleged unlawful, malicious,
oppressive, and abusive acts of petitioners. Petitioner Hendry, claiming illness, did not testify during the
hearings. The Regional Trial Court (RTC) of Manila, Branch IX, through Judge Manuel T. Reyes rendered
judgment in favor of private respondent by ordering petitioners to pay him eighty thousand pesos

214
(P80,000.00) as actual damages, two hundred thousand pesos (P200,000.00) as moral damages, twenty
thousand pesos (P20,000.00) as exemplary damages, thirty thousand pesos (P30,000.00) as attorney's
fees, and costs. Petitioners appealed the RTC decision to the Court of Appeals. On the other hand,
Tobias appealed as to the amount of damages. However, the Court of Appeals, in a decision dated
August 31, 1987, . . . affirmed the RTC decision in toto. Petitioners' motion for reconsideration having
been denied, the instant petition for review on certiorari was filed.

The main issue in this case is whether or not petitioners are liable for damages to private respondent.
LibLex

Petitioners contend that they could not be made liable for damages in the lawful exercise of their right
to dismiss private respondent.

On the other hand, private respondent contends that because of petitioners' abusive manner in
dismissing him as well as for the inhuman treatment he got from them, the petitioners must indemnify
him for the damage that he had suffered.

One of the more notable innovations of the New Civil Code is the codification of "some basic principles
that are to be observed for the rightful relationship between human beings and for the stability of the
social order." [REPORT ON THE CODE COMMISSION ON THE PROPOSED CIVIL CODE OF THE PHILIPPINES,
p. 39]. The framers of the Code, seeking to remedy the defect of the old Code which merely stated the
effects of the law, but failed to draw out its spirit, incorporated certain fundamental precepts which
were "designed to indicate certain norms that spring from the fountain of good conscience" and which
were also meant to serve as "guides for human conduct [that] should run as golden threads through
society, to the end that law may approach its supreme ideal, which is the sway and dominance of
justice" (Id.) Foremost among these principles is that pronounced in Article 19 which provides:

Art. 19. Every person must, in the exercise of his rights and in the performance of his duties, act with
justice, give everyone his due, and observe honesty and good faith.

This article, known to contain what is commonly referred to as the principle of abuse of rights, sets
certain standards which must be observed not only in the exercise of one's rights but also in the
performance of one's duties. These standards are the following: to act with justice; to give everyone his
due; and to observe honesty and good faith. The law, therefore, recognizes a primordial limitation on all
rights; that in their exercise, the norms of human conduct set forth in Article 19 must be observed. A
right, though by itself legal because recognized or granted by law as such, may nevertheless become the
source of some illegality. When a right is exercised in a manner which does not conform with the norms
enshrined in Article 19 and results in damage to another, a legal wrong is thereby committed for which
the wrongdoer must be held responsible. But while Article 19 lays down a rule of conduct for the
government of human relations and for the maintenance of social order, it does not provide a remedy
for its violation. Generally, an action for damages under either Article 20 or Article 21 would be proper.

Article 20, which pertains to damage arising from a violation of law, provides that:

215
Art. 20. Every person who contrary to law, wilfully or negligently causes damage to another, shall
indemnify the latter for the same.

However, in the case at bar, petitioners claim that they did not violate any provision of law since they
were merely exercising their legal right to dismiss private respondent. This does not, however, leave
private respondent with no relief because Article 21 of the Civil Code provides that:

Art. 21. Any person who wilfully causes loss or injury to another in a manner that is contrary to morals,
good customs or public policy shall compensate the latter for the damage.

This article, adopted to remedy the "countless gaps in the statutes, which leave so many victims of
moral wrongs helpless, even though they have actually suffered material and moral injury" [Id.] should
"vouchsafe adequate legal remedy for that untold number of moral wrongs which it is impossible for
human foresight to provide for specifically in the statutes" [Id., at p. 40; See also PNB v. CA, G.R. No. L-
27155, May 18, 1978, 83 SCRA 237, 247].

In determining whether or not the principle of abuse of rights may be invoked, there is no rigid test
which can be applied. While the Court has not hesitated to apply Article 19 whenever the legal and
factual circumstances called for its application [See for e.g., Velayo v. Shell Co. of the Phil., Ltd., 100 Phil.
186 (1956); PNB v. CA, supra; Grand Union Supermarket, Inc. v. Espino, Jr., G.R. No. L-48250, December
28, 1979, 94 SCRA 953; PAL v. CA, G.R. No. L-46558, July 31, 1981, 106 SCRA 391; United General
Industries, Inc. v. Paler, G.R. No. L-30205, March 15, 1982, 112 SCRA 404; Rubio v. CA, G.R. No. 50911,
August 21, 1987, 153 SCRA 183] the question of whether or not the principle of abuse of rights has been
violated resulting in damages under Article 20 or Article 21 or other applicable provision of law, depends
on the circumstances of each case. And in the instant case, the Court, after examining the record and
considering certain significant circumstances, finds that petitioners have indeed abused the right that
they invoke, causing damage to private respondent and for which the latter must now be indemnified.
LLjur

The trial court made a finding that notwithstanding the fact that it was private respondent Tobias who
reported the possible existence of anomalous transactions, petitioner Hendry "showed belligerence and
told plaintiff (private respondent herein) that he was the number one suspect and to take a one week
vacation leave, not to communicate with the office, to leave his table drawers open, and to leave his
keys to said defendant (petitioner Hendry)" [RTC Decision, p. 2; Rollo, p. 232]. This, petitioners do not
dispute. But regardless of whether or not it was private respondent Tobias who reported the anomalies
to petitioners, the latter's reaction towards the former upon uncovering the anomalies was less than
civil. An employer who harbors suspicions that an employee has committed dishonesty might be
justified in taking the appropriate action such as ordering an investigation and directing the employee to
go on a leave. Firmness and the resolve to uncover the truth would also be expected from such
employer. But the high-handed treatment accorded Tobias by petitioners was certainly uncalled for. And
this reprehensible attitude of petitioners was to continue when private respondent returned to work on
November 20, 1972 after his one week forced leave. Upon reporting for work, Tobias was confronted by
Hendry who said. "Tobby, you are the crook and swindler in this company." Considering that the first

216
report made by the police investigators was submitted only on December 10, 1972 [See Exh. "A"] the
statement made by petitioner Hendry was baseless. The imputation of guilt without basis and the
pattern of harassment during the investigations of Tobias transgress the standards of human conduct
set forth in Article 19 of the Civil Code. The Court has already ruled that the right of the employer to
dismiss an employee should not be confused with the manner in which the right is exercised and the
effects flowing therefrom. If the dismissal is done abusively, then the employer is liable for damages to
the employee [Quisaba v. Sta. Ines-Melale Veneer and Plywood Inc., G.R. No. L-38088, August 30, 1974,
58 SCRA 771; See also Philippine Refining Co., Inc. v. Garcia, G.R. No. L-21871, September 27, 1966, 18
SCRA 107]. Under the circumstances of the instant case, the petitioners clearly failed to exercise in a
legitimate manner their right to dismiss Tobias, giving the latter the right to recover damages under
Article 19 in relation to Article 21 of the Civil Code.

But petitioners were not content with just dismissing Tobias. Several other tortious acts were
committed by petitioners against Tobias after the latter's termination from work. Towards the latter
part of January, 1973, after the filing of the first of six criminal complaints against Tobias, the latter
talked to Hendry to protest the actions taken against him. In response, Hendry cut short Tobias'
protestations by telling him to just confess or else the company would file a hundred more cases against
him until he landed in jail. Hendry added that, "You Filipinos cannot be trusted." The threat unmasked
petitioner's bad faith in the various actions taken against Tobias. On the other hand, the scornful remark
about Filipinos as well as Hendry's earlier statements about Tobias being a "crook" and "swindler" are
clear violations of Tobias' personal dignity [See Article 26, Civil Code].

The next tortious act committed by petitioners was the writing of a letter to RETELCO sometime in
October 1974, stating that Tobias had been dismissed by GLOBE MACKAY due to dishonesty. Because of
the letter, Tobias failed to gain employment with RETELCO and as a result of which, Tobias remained
unemployed for a longer period of time. For this further damage suffered by Tobias, petitioners must
likewise be held liable for damages consistent with Article 2176 of the Civil Code. Petitioners, however,
contend that they have a "moral, if not legal, duty to forewarn other employers of the kind of employee
the plaintiff (private respondent herein) was." [Petition, p. 14; Rollo, p. 15]. Petitioners further claim
that "it is the accepted moral and societal obligation of every man to advise or warn his fellowmen of
any threat or danger to the latter's life, honor or property. And this includes warning one's brethren of
the possible dangers involved in dealing with, or accepting into confidence, a man whose honesty and
integrity is suspect" [Id.]. These arguments, rather than justify petitioners' act, reveal a seeming
obsession to prevent Tobias from getting a job, even after almost two years from the time Tobias was
dismissed.

Finally, there is the matter of the filing by petitioners of six criminal complaints against Tobias.
Petitioners contend that there is no case against them for malicious prosecution and that they cannot be
"penalized for exercising their right and prerogative of seeking justice by filing criminal complaints
against an employee who was their principal suspect in the commission of forgeries and in the
perpetration of anomalous transactions which defrauded them of substantial sums of money" [Petition,
p. 10, Rollo, p. 11].

217
While sound principles of justice and public policy dictate that persons shall have free resort to the
courts for redress of wrongs and vindication of their rights [Buenaventura v. Sto. Domingo, 103 Phil. 239
(1958)], the right to institute criminal prosecutions can not be exercised maliciously and in bad faith
[Ventura v. Bernabe, G.R. No. L-26760, April 30, 1971, 38 SCRA 587]. Hence, in Yutuk v. Manila Electric
Co., G.R. No. L-13016, May 31, 1961, 2 SCRA 337, the Court held that the right to file criminal complaints
should not be used as a weapon to force an alleged debtor to pay an indebtedness. To do so would be a
clear perversion of the function of the criminal processes and of the courts of justice. And in Hawpia v.
CA, G.R. No. L-20047, June 30, 1967, 20 SCRA 536, the Court upheld the judgment against the petitioner
for actual and moral damages and attorney's fees after making a finding that petitioner, with
persistence, filed at least six criminal complaints against respondent, all of which were dismissed.

To constitute malicious prosecution, there must be proof that the prosecution was prompted by a
design to vex and humiliate a person and that it was initiated deliberately by the defendant knowing
that the charges were false and groundless [Manila Gas Corporation v. CA, G.R. No. L-44190, October 30,
1980, 100 SCRA 602]. Concededly, the filing of a suit, by itself, does not render a person liable for
malicious prosecution [Inhelder Corporation v. CA, G.R. No. 52358, May 30, 1983, 122 SCRA 576]. The
mere dismissal by the fiscal of the criminal complaint is not a ground for an award of damages for
malicious prosecution if there is no competent evidence to show that the complainant had acted in bad
faith [Sison v. David, G.R. No. L-11268, January 28, 1961, 1 SCRA 60].

In the instant case, however, the trial court made a finding that petitioners acted in bad faith in filing the
criminal complaints against Tobias, observing that: cdll

xxx xxx xxx

Defendants (petitioners herein) filed with the Fiscal's Office of Manila a total of six (6) criminal cases,
five (5) of which were for estafa thru falsification of commercial document and one for violation of Art.
290 of the Revised Penal Code "discovering secrets thru seizure of correspondence," and all were
dismissed for insufficiency or lack of evidence." The dismissal of four (4) of the cases was appealed to
the Ministry of Justice, but said Ministry invariably sustained the dismissal of the cases. As above
adverted to, two of these cases were refiled with the Judge Advocate General's Office of the Armed
Forces of the Philippines to railroad plaintiffs arrest and detention in the military stockade, but this was
frustrated by a presidential decree transferring criminal cases involving civilians to the civil courts.

xxx xxx xxx

To be sure, when despite the two (2) police reports embodying the findings of Lt. Dioscoro Tagle, Chief
Document Examiner of the Manila Police Department, clearing plaintiff of participation or involvement
in the fraudulent transactions complained of, despite the negative results of the lie detector tests which
defendants compelled plaintiff to undergo, and although the police investigation was "still under follow-
up and a supplementary report will be submitted after all the evidence has been gathered," defendants
hastily filed six (6) criminal cases with the city Fiscal's Office of Manila, five (5) for estafa thru falsification
of commercial document and one (1) for violation of Art. 290 of the Revised Penal Code, so much so that
as was to be expected, all six (6) cases were dismissed, with one of the investigating fiscals, Asst. Fiscal

218
de Guia, commenting in one case that, "Indeed, the haphazard way this case was investigated is evident.
Evident likewise is the flurry and haste in the filing of this case against respondent Tobias," there can be
no mistaking that defendants would not but be motivated by malicious and unlawful intent to harass,
oppress, and cause damage to plaintiff.

xxx xxx xxx

[RTC Decision, pp. 5-6; Rollo, pp. 235-236].

In addition to the observations made by the trial court, the Court finds it significant that the criminal
complaints were filed during the pendency of the illegal dismissal case filed by Tobias against
petitioners. This explains the haste in which the complaints were filed, which the trial court earlier
noted. But petitioners, to prove their good faith, point to the fact that only six complaints were filed
against Tobias when they could have allegedly filed one hundred cases, considering the number of
anomalous transactions committed against GLOBE MACKAY. However, petitioners' good faith is belied
by the threat made by Hendry after the filing of the first complaint that one hundred more cases would
be filed against Tobias. In effect, the possible filing of one hundred more cases was made to hang like
the sword of Damocles over the head of Tobias. In fine, considering the haste in which the criminal
complaints were filed, the fact that they were filed during the pendency of the illegal dismissal case
against petitioners, the threat made by Hendry, the fact that the cases were filed notwithstanding the
two police reports exculpating Tobias from involvement in the anomalies committed against GLOBE
MACKAY, coupled by the eventual dismissal of all the cases, the Court is led into no other conclusion
than that petitioners were motivated by malicious intent in filing the six criminal complaints against
Tobias.

Petitioners next contend that the award of damages was excessive. In the complaint filed against
petitioners, Tobias prayed for the following: one hundred thousand pesos (P100,000.00) as actual
damages; fifty thousand pesos (P50,000.00) as exemplary damages; eight hundred thousand pesos
(P800,000.00) as moral damages; fifty thousand pesos (P50,000.00) as attorney's fees; and costs. The
trial court, after making a computation of the damages incurred by Tobias [See RTC Decision, pp. 7-8;
Rollo, pp. 154-155], awarded him the following: eighty thousand pesos (P80,000.00) as actual damages;
two hundred thousand pesos (P200,000.00) as moral damages; twenty thousand pesos (P20,000.00) as
exemplary damages; thirty thousand pesos (P30,000.00) as attorney's fees; and, costs. It must be
underscored that petitioners have been guilty of committing several actionable tortious acts, i.e., the
abusive manner in which they dismissed Tobias from work including the baseless imputation of guilt and
the harassment during the investigations; the defamatory language heaped on Tobias as well as the
scornful remark on Filipinos; the poison letter sent to RETELCO which resulted in Tobias' loss of possible
employment; and, the malicious filing of the criminal complaints. Considering the extent of the damage
wrought on Tobias, the Court finds that, contrary to petitioners' contention, the amount of damages
awarded to Tobias was reasonable under the circumstances.

Yet, petitioners still insist that the award of damages was improper, invoking the principle of damnum
absque injuria. It is argued that "[t]he only probable actual damage that plaintiff (private respondent

219
herein) could have suffered was a direct result of his having been dismissed from his employment, which
was a valid and legal act of the defendants-appellants (petitioners herein)." [Petition, p. 17; Rollo, p. 18].

According to the principle of damnum absque injuria, damage or loss which does not constitute a
violation of a legal right or amount to a legal wrong is not actionable [Escano v. CA, G. R. No. L-47207,
September 25, 1980, 100 SCRA 197; See also Gilchrist v. Cuddy, 29 Phil. 542 (1915); The Board of
Liquidators v. Kalaw, G.R. No. L-18805, August 14, 1967, 20 SCRA 987]. This principle finds no application
in this case. It bears repeating that even granting that petitioners might have had the right to dismiss
Tobias from work, the abusive manner in which that right was exercised amounted to a legal wrong for
which petitioners must now be held liable. Moreover, the damage incurred by Tobias was not only in
connection with the abusive manner in which he was dismissed but was also the result of several other
quasi-delictual acts committed by petitioners. LLphil

Petitioners next question the award of moral damages. However, the Court has already ruled in
Wassmer v. Velez, G.R. No. L-20089, December 26, 1964, 12 SCRA 648, 653, that "[p]er express provision
of Article 2219 (10) of the New Civil Code, moral damages are recoverable in the cases mentioned in
Article 21 of said Code." Hence, the Court of Appeals committed no error in awarding moral damages to
Tobias.

Lastly, the award of exemplary damages is impugned by petitioners. Although Article 2231 of the Civil
Code provides that "[i]n quasi-delicts, exemplary damages may be granted if the defendant acted with
gross negligence," the Court, in Zulueta v. Pan American World Airways, Inc., G.R. No. L-28589, January
8, 1973, 49 SCRA 1, ruled that if gross negligence warrants the award of exemplary damages, with more
reason is its imposition justified when the act performed is deliberate, malicious and tainted with bad
faith. As in the Zulueta case, the nature of the wrongful acts shown to have been committed by
petitioners against Tobias is sufficient basis for the award of exemplary damages to the latter.

WHEREFORE, the Petition is hereby DENIED and the decision of the Court of Appeals in CA-G.R. CV No.
09055 is AFFIRMED.

SO ORDERED.

Fernan, (C.J.), Gutierrez, Jr. and Bidin, JJ., concur.

Feliciano, J., took no part.

220
University of the East vs. Jader, G.R. No. 132344, February 17, 2000

[G.R. No. 132344. February 17, 2000.]

UNIVERSITY OF THE EAST, petitioner, vs. ROMEO A. JADER, respondent.

Puno and Puno for petitioner.

Thelma A. Jader for private respondent.

SYNOPSIS

Jader was a law student of the UE College of Law. For getting an incomplete grade in his Practice Court I,
he took a removal exam for the same. Unknown to him, however, he was given a failing grade. But still,
he was included in the list of graduates and was able to enroll at the pre-bar review class. Later, he
learned of his deficiency and thus, dropped his review class and was not able to take the bar
examinations. Jader filed an action for damages against the university and both the trial court and the
Court of Appeals ruled in his favor. Thus, this appeal with the question: May an educational institution
be held liable for damages for misleading a student into believing that the latter had satisfied all the
requirements for graduation when such is not the case?

Petitioner, in belatedly informing respondent of the result of the removal examination, particularly at a
time when he had already commenced preparing for the bar exams, cannot be said to have acted in
good faith. Petitioner was guilty of negligence and liable for actual damages. However, petitioner is not
liable for moral damages. Respondent should have verified for himself whether he has completed all
necessary requirements to be eligible for the bar examinations. cDACST

SYLLABUS

1. CIVIL LAW; OBLIGATIONS AND CONTRACTS; CONTRACT OF EDUCATION BETWEEN A LEARNING


INSTITUTION AND THE STUDENT; OBLIGATION OF THE SCHOOL TO INFORM STUDENTS OF PROBLEMS IN
GRADES. — When a student is enrolled in any educational or learning institution, a contract of education
is entered into between said institution and the student. The professors, teachers or instructors hired by
the school are considered merely as agents and administrators tasked to perform the school's
commitment under the contract. Since the contracting parties are the school and the student, the latter
is not duty-bound to deal with the former's agents, such as the professors with respect to the status or
result of his grades, although nothing prevents either professors or students from sharing with each
other such information. The Court takes judicial notice of the traditional practice in educational
institutions wherein the professor directly furnishes his/her students their grades. It is the contractual
obligation of the school to timely inform and furnish sufficient notice and information to each and every
student as to whether he or she had already complied with all the requirements for the conferment of a
degree or whether they would be included among those who will graduate. Although commencement
exercises are but a formal ceremony, it nonetheless is not an ordinary occasion, since such ceremony is
the educational institution's way of announcing to the whole world that the students included in the list
of those who will be conferred a degree during the baccalaureate ceremony have satisfied all the

221
requirements for such degree. Prior or subsequent to the ceremony, the school has the obligation to
promptly inform the student of any problem involving the latter's grades and performance and also
most importantly, of the procedures for remedying the same.

2. ID.; HUMAN RELATIONS; GOOD FAITH; WANTING WHEN A STUDENT WAS BELATEDLY
INFORMED OF HIS FAILING GRADE IN CASE AT BAR. — Petitioner, in belatedly informing respondent of
the result of the removal examination, particularly at a time when he had already commenced preparing
for the bar exams, cannot be said to have acted in good faith. Absence of good faith must be sufficiently
established for a successful prosecution by the aggrieved party in a suit for abuse of right under Article
19 of the Civil Code. Good faith connotes an honest intention to abstain from taking undue advantage of
another, even though the forms and technicalities of the law, together with the absence of all
information or belief of facts, would render the transaction unconscientious. It is the school that has
access to those information and it is only the school that can compel its professors to act and comply
with its rules, regulations and policies with respect to the computation and the prompt submission of
grades. Students do not exercise control, much less influence, over the way an educational institution
should run its affairs, particularly in disciplining its professors and teachers and ensuring their
compliance with the school's rules and orders. Being the party that hired them, it is the school that
exercises general supervision and exclusive control over the professors with respect to the submission of
reports involving the students' standing. Exclusive control means that no other person or entity had any
control over the instrumentality which caused the damage or injury.

3. ID.; DAMAGES; SCHOOL LIABLE FOR THE NEGLIGENCE OF ITS PROFESSORS. — The college dean
is the senior officer responsible for the operation of an academic program, enforcement of rules and
regulations, and the supervision of faculty and student services. He must see to it that his own
professors and teachers, regardless of their status or position outside of the university, must comply
with the rules set by the latter. The negligent act of a professor who fails to observe the rules of the
school, for instance by not promptly submitting a student's grade, is not only imputable to the professor
but is an act of the school, being his employer. Considering further, that the institution of learning
involved herein is a university which is engaged in legal education, it should have practiced what it
inculcates in its students, more specifically the principle of good dealings enshrined in Articles 19 and 20
of the Civil Code. Article 19 was intended to expand the concept of torts by granting adequate legal
remedy for the untold number of moral wrongs which is impossible for human foresight to provide
specifically in statutory law. In civilized society, men must be able to assume that others will do them no
intended injury — that others will commit no internal aggressions upon them; that their fellowmen,
when they act affirmatively will do so with due care which the ordinary understanding and moral sense
of the community exacts and that those with whom they deal in the general course of society will act in
good faith. The ultimate thing in the theory of liability is justifiable reliance under conditions of civilized
society. Schools and professors cannot just take students for granted and be indifferent to them, for
without the latter, the former are useless. Educational institutions are duty-bound to inform the
students of their academic status and not wait for the latter to inquire from the former. The conscious
indifference of a person to the rights or welfare of the person/persons who may be affected by his act or
omission can support a claim for damages. Want of care to the conscious disregard of civil obligations

222
coupled with a conscious knowledge of the cause naturally calculated to produce them would make the
erring party liable. Petitioner ought to have known that time was of the essence in the performance of
its obligation to inform respondent of his grade. It cannot feign ignorance that respondent will not
prepare himself for the bar exams since that is precisely the immediate concern after graduation of an
LL.B. graduate. It failed to act seasonably. Petitioner cannot just give out its student's grades at any time
because a student has to comply with certain deadlines set by the Supreme Court on the submission of
requirements for taking the bar. Petitioner's liability arose from its failure to promptly inform
respondent of the result of an examination and in misleading the latter into believing that he had
satisfied all requirements of the course. Petitioner cannot pass on its blame to the professors to justify
its own negligence that led to the delayed relay of information to respondent. When one of two
innocent parties must suffer, he through whose agency the loss occurred must bear it. The modern
tendency is to grant indemnity for damages in cases where there is abuse of right, even when the act is
not illicit. If mere fault or negligence in one's acts can make him liable for damages for injury caused
thereby, with more reason should abuse or bad faith make him liable. A person should be protected
only when he acts in the legitimate exercise of his right, that is, when he acts with prudence and in good
faith, but not when he acts with negligence or abuse.

4. ID.; ID.; ID.; MORAL DAMAGES, NOT PROPER. — While petitioner was guilty of negligence and
thus liable to respondent for the latter's actual damages, we hold that respondent should not have been
awarded moral damages. We do not agree with the Court of Appeals' findings that respondent suffered
shock, trauma and pain when he was informed that he could not graduate and will not be allowed to
take the bar examinations. At the very least, it behooved on respondent to verify for himself whether he
has completed all necessary requirements to be eligible for the bar examinations. As a senior law
student, respondent should have been responsible enough to ensure that all his affairs, specifically
those pertaining to his academic achievement, are in order. Given these considerations, we fail to see
how respondent could have suffered untold embarrassment in attending the graduation rites, enrolling
in the bar review classes and not being able to take the bar exams. If respondent was indeed humiliated
by his failure to take the bar, he brought this upon himself by not verifying if he has satisfied all the
requirements including his school records, before preparing himself for the bar examination. Certainly,
taking the bar examinations does not only entail a mental preparation on the subjects thereof; there are
also prerequisites of documentation and submission of requirements which the prospective examinee
must meet. aATESD

DECISION

YNARES-SANTIAGO, J p:

May an educational institution be held liable for damages for misleading a student into believing that
the latter had satisfied all the requirements for graduation when such is not the case? This is the issue in
the instant petition for review premised on the following undisputed facts as summarized by the trial
court and adopted by the Court of Appeals (CA), 1 to wit:

223
"Plaintiff was enrolled in the defendants' College of Law from 1984 up to 1988. In the first semester of
his last year (School year 1987-1988), he failed to take the regular final examination in Practice Court I
for which he was given an incomplete grade (Exhibits '2', also Exhibit 'H'). He enrolled for the second
semester as fourth year law student (Exhibit 'A') and on February 1, 1988 he filed an application for the
removal of the incomplete grade given him by Professor Carlos Ortega (Exhibits 'H-2', also Exhibit '2')
which was approved by Dean Celedonio Tiongson after payment of the required fee. He took the
examination on March 28, 1988. On May 30, 1988, Professor Carlos Ortega submitted his grade. It was a
grade of five (5). (Exhibits 'H-4', also Exhibits '2-L', '2-N'). prLL

"In the meantime, the Dean and the Faculty Members of the College of Law met to deliberate on who
among the fourth year students should be allowed to graduate. The plaintiff's name appeared in the
Tentative List of Candidates for graduation for the Degree of Bachelor of Laws (LL.B) as of Second
Semester (1987-1988) with the following annotation:

"JADER ROMEO A.

Def. Conflict of Laws - x-1-87-88, Practice Court I - Inc., 1-87-88. C-1 to submit transcript with S.O.
(Exhibits '3', '3-C-1', '3-C-2')."

"The 35th Investitures & Commencement Ceremonies for the candidates of Bachelor of Laws was
scheduled on the 16th of April 1988 at 3:00 o'clock in the afternoon, and in the invitation for that
occasion the name of the plaintiff appeared as one of the candidates. (Exhibits 'B', 'B-6', 'B-6-A'). At the
foot of the list of the names of the candidates there appeared however the following annotation:

'This is a tentative list. Degrees will be conferred upon these candidates who satisfactorily complete
requirements as stated in the University Bulletin and as approved of the Department of Education,
Culture and Sports (Exhibit 'B-7-A').

"The plaintiff attended the investiture ceremonies at F. dela Cruz Quadrangle, U.E., Recto Campus,
during the program of which he went up the stage when his name was called, escorted by her (sic)
mother and his eldest brother who assisted in placing the Hood, and his Tassel was turned from left to
right, and he was thereafter handed by Dean Celedonio a rolled white sheet of paper symbolical of the
Law Diploma. His relatives took pictures of the occasion (Exhibits 'C' to 'C-6', 'D-3' to 'D-11').

"He tendered a blow-out that evening which was attended by neighbors, friends and relatives who
wished him good luck in the forthcoming bar examination. There were pictures taken too during the
blow-out (Exhibits 'D' to 'D-1').

"He thereafter prepared himself for the bar examination. He took a leave of absence without pay from
his job from April 20, 1988 to September 30, 1988 (Exhibit 'G') and enrolled at the pre-bar review class in
Far Eastern University (Exhibits 'F' to 'F-2'). Having learned of the deficiency he dropped his review class
and was not able to take the bar examination." 2

Consequently, respondent sued petitioner for damages alleging that he suffered moral shock, mental
anguish, serious anxiety, besmirched reputation, wounded feelings and sleepless nights when he was

224
not able to take the 1988 bar examinations arising from the latter's negligence. He prayed for an award
of moral and exemplary damages, unrealized income, attorney's fees, and costs of suit. LexLib

In its answer with counterclaim, petitioner denied liability arguing mainly that it never led respondent to
believe that he completed the requirements for a Bachelor of Laws degree when his name was included
in the tentative list of graduating students. After trial, the lower court rendered judgment as follows:

WHEREFORE, in view of the foregoing judgment is hereby rendered in favor of the plaintiff and against
the defendant ordering the latter to pay plaintiff the sum of THIRTY FIVE THOUSAND FOUR HUNDRED
SEVENTY PESOS (P35,470.00) with legal rate of interest from the filing of the complaint until fully paid,
the amount of FIVE THOUSAND PESOS (P5,000.00) as attorney's fees and the cost of suit.

Defendant's counterclaim is, for lack of merit, hereby dismissed.

SO ORDERED. 3

which on appeal by both parties was affirmed by the Court of Appeals (CA) with modification. The
dispositive portion of the CA decision reads:

WHEREFORE, in the light of the foregoing, the lower Court's Decision is hereby AFFIRMED with the
MODIFICATION that defendant-appellee, in addition to the sum adjudged by the lower court in favor of
plaintiff-appellant, is also ORDERED to pay plaintiff-appellant the amount of FIFTY THOUSAND
(P50,000.00) PESOS for moral damages. Costs against defendant-appellee.

SO ORDERED. 4

Upon the denial of its motion for reconsideration, petitioner UE elevated the case to this Court on a
petition for review under Rule 45 of the Rules of Court, arguing that it has no liability to respondent
Romeo A. Jader, considering that the proximate and immediate cause of the alleged damages incurred
by the latter arose out of his own negligence in not verifying from the professor concerned the result of
his removal exam. prLL

The petition lacks merit.

When a student is enrolled in any educational or learning institution, a contract of education is entered
into between said institution and the student. The professors, teachers or instructors hired by the school
are considered merely as agents and administrators tasked to perform the school's commitment under
the contract. Since the contracting parties are the school and the student, the latter is not duty-bound
to deal with the former's agents, such as the professors with respect to the status or result of his grades,
although nothing prevents either professors or students from sharing with each other such information.
The Court takes judicial notice of the traditional practice in educational institutions wherein the
professor directly furnishes his/her students their grades. It is the contractual obligation of the school to
timely inform and furnish sufficient notice and information to each and every student as to whether he
or she had already complied with all the requirements for the conferment of a degree or whether they
would be included among those who will graduate. Although commencement exercises are but a formal

225
ceremony, it nonetheless is not an ordinary occasion, since such ceremony is the educational
institution's way of announcing to the whole world that the students included in the list of those who
will be conferred a degree during the baccalaureate ceremony have satisfied all the requirements for
such degree. Prior or subsequent to the ceremony, the school has the obligation to promptly inform the
student of any problem involving the latter's grades and performance and also most importantly, of the
procedures for remedying the same.

Petitioner, in belatedly informing respondent of the result of the removal examination, particularly at a
time when he had already commenced preparing for the bar exams, cannot be said to have acted in
good faith. Absence of good faith must be sufficiently established for a successful prosecution by the
aggrieved party in a suit for abuse of right under Article 19 of the Civil Code. Good faith connotes an
honest intention to abstain from taking undue advantage of another, even though the forms and
technicalities of the law, together with the absence of all information or belief of facts, would render the
transaction unconscientious. 5 It is the school that has access to those information and it is only the
school that can compel its professors to act and comply with its rules, regulations and policies with
respect to the computation and the prompt submission of grades. Students do not exercise control,
much less influence, over the way an educational institution should run its affairs, particularly in
disciplining its professors and teachers and ensuring their compliance with the school's rules and orders.
Being the party that hired them, it is the school that exercises general supervision and exclusive control
over the professors with respect to the submission of reports involving the students' standing. Exclusive
control means that no other person or entity had any control over the instrumentality which caused the
damage or injury. 6

The college dean is the senior officer responsible for the operation of an academic program,
enforcement of rules and regulations, and the supervision of faculty and student services. 7 He must see
to it that his own professors and teachers, regardless of their status or position outside of the university,
must comply with the rules set by the latter. The negligent act of a professor who fails to observe the
rules of the school, for instance by not promptly submitting a student’s grade, is not only imputable to
the professor but is an act of the school, being his employer.

Considering further, that the institution of learning involved herein is a university which is engaged in
legal education, it should have practiced what it inculcates in its students, more specifically the principle
of good dealings enshrined in Articles 19 and 20 of the Civil Code which states:

ARTICLE 19. Every person must, in the exercise of his rights and in the performance of his duties, act
with justice, give everyone his due, and observe honesty and good faith.

ARTICLE 20. Every person who, contrary to law, wilfully or negligently causes damage to another,
shall indemnify the latter for the same. cda

Article 19 was intended to expand the concept of torts by granting adequate legal remedy for the untold
number of moral wrongs which is impossible for human foresight to provide specifically in statutory law.
8 In civilized society, men must be able to assume that others will do them no intended injury – that
others will commit no internal aggressions upon them; that their fellowmen, when they act affirmatively

226
will do so with due care which the ordinary understanding and moral sense of the community exacts and
that those with whom they deal in the general course of society will act in good faith. The ultimate thing
in the theory of liability is justifiable reliance under conditions of civilized society. 9 Schools and
professors cannot just take students for granted and be indifferent to them, for without the latter, the
former are useless.

Educational institutions are duty-bound to inform the students of their academic status and not wait for
the latter to inquire from the former. The conscious indifference of a person to the rights or welfare of
the person/persons who may be affected by his act or omission can support a claim for damages. 10
Want of care to the conscious disregard of civil obligations coupled with a conscious knowledge of the
cause naturally calculated to produce them would make the erring party liable. 11 Petitioner ought to
have known that time was of the essence in the performance of its obligation to inform respondent of
his grade. It cannot feign ignorance that respondent will not prepare himself for the bar exams since
that is precisely the immediate concern after graduation of an LL.B. graduate. It failed to act seasonably.
Petitioner cannot just give out its student's grades at any time because a student has to comply with
certain deadlines set by the Supreme Court on the submission of requirements for taking the bar.
Petitioner's liability arose from its failure to promptly inform respondent of the result of an examination
and in misleading the latter into believing that he had satisfied all requirements for the course. Worth
quoting is the following disquisition of the respondent court:

"It is apparent from the testimony of Dean Tiongson that defendant-appellee University had been
informed during the deliberation that the professor in Practice Court I gave plaintiff-appellant a failing
grade. Yet, defendant-appellee still did not inform plaintiff-appellant of his failure to complete the
requirements for the degree nor did they remove his name from the tentative list of candidates for
graduation. Worse, defendant-appellee university, despite the knowledge that plaintiff-appellant failed
in Practice Court I, again included plaintiff-appellant's name in the "tentative" list of candidates for
graduation which was prepared after the deliberation and which became the basis for the
commencement rites program. Dean Tiongson reasons out that plaintiff-appellant's name was allowed
to remain in the tentative list of candidates for graduation in the hope that the latter would still be able
to remedy the situation in the remaining few days before graduation day. Dean Tiongson, however, did
not explain how plaintiff-appellant Jader could have done something to complete his deficiency if
defendant-appellee university did not exert any effort to inform plaintiff-appellant of his failing grade in
Practice Court I." 12

Petitioner cannot pass on its blame to the professors to justify its own negligence that led to the delayed
relay of information to respondent. When one of two innocent parties must suffer, he through whose
agency the loss occurred must bear it. 13 The modern tendency is to grant indemnity for damages in
cases where there is abuse of right, even when the act is not illicit. 14 If mere fault or negligence in one's
acts can make him liable for damages for injury caused thereby, with more reason should abuse or bad
faith make him liable. A person should be protected only when he acts in the legitimate exercise of his
right, that is, when he acts with prudence and in good faith, but not when he acts with negligence or
abuse. 15

227
However, while petitioner was guilty of negligence and thus liable to respondent for the latter's actual
damages, we hold that respondent should not have been awarded moral damages. We do not agree
with the Court of Appeals' findings that respondent suffered shock, trauma and pain when he was
informed that he could not graduate and will not be allowed to take the bar examinations. At the very
least, it behooved on respondent to verify for himself whether he has completed all necessary
requirements to be eligible for the bar examinations. As a senior law student, respondent should have
been responsible enough to ensure that all his affairs, specifically those pertaining to his academic
achievement, are in order. Given these considerations, we fail to see how respondent could have
suffered untold embarrassment in attending the graduation rites, enrolling in the bar review classes and
not being able to take the bar exams. If respondent was indeed humiliated by his failure to take the bar,
he brought this upon himself by not verifying if he has satisfied all the requirements including his school
records, before preparing himself for the bar examination. Certainly, taking the bar examinations does
not only entail a mental preparation on the subjects thereof; there are also prerequisites of
documentation and submission of requirements which the prospective examinee must meet. LLphil

WHEREFORE, the assailed decision of the Court of Appeals is AFFIRMED with MODIFICATION. Petitioner
is ORDERED to PAY respondent the sum of Thirty-five Thousand Four Hundred Seventy Pesos
(P35,470.00), with legal interest of 6% per annum computed from the date of filing of the complaint
until fully paid; the amount of Five Thousand Pesos (P5,000.00) as attorney's fees; and the costs of the
suit. The award of moral damages is DELETED.

SO ORDERED.

Davide, Jr., C.J., Kapunan and Pardo, JJ., concur.

Puno, J., took no part.

228
Tenchavez vs. Escano, 15 SCRA 355

[G.R. No. L-19671. November 29, 1965.]

PASTOR B. TENCHAVEZ, plaintiff-appellant, vs. VICENTA F. ESCAÑO, ET AL., defendants-appellees.

I. V. Binamira & F. B. Barria for plaintiff-appellant.

Jalandoni & Jamir for defendants-appellees.

SYLLABUS

1. HUSBAND AND WIFE; FOREIGN DIVORCE BETWEEN FILIPINO CITIZENS DECREED AFTER THE
EFFECTIVITY OF THE NEW CIVIL CODE; REMARRIAGE OF DIVORCED CONSORT. — A foreign divorce
between Filipino citizens, sought and decreed after the effectivity of the new Civil Code (Republic Act
No. 386), is not entitled to recognition as valid in the Philippines; and neither is the marriage contracted
with another party by the divorced consort, subsequently to the foreign decree of divorce entitled to
validity in this country.

2. ID.; ID.; ID.; INNOCENT CONSORT ENTITLED TO LEGAL SEPARATION. — The remarriage of the
divorced wife and her cohabitation with a person other than the lawful husband entitles the latter to a
decree of legal separation conformably to Philippine law.

3. ID.; ID,; ID.; INVALID DIVORCE ENTITLES INNOCENT CONSORT TO RECOVER DAMAGES. — The
desertion and securing of an invalid divorce decree by one consort entitles the other to recover
damages.

4. ID.; ACTION FOR ALIENATION OF AFFECTIONS AGAINST PARENTS OF ONE CONSORT; ABSENCE
OF PROOF OF MALICE. — An action for alienation of affection against the parents of one consort does
not lie in the absence of proof of malice or unworthy motives on their part.

DECISION

REYES, J. B. L., J p:

Direct appeal, on factual and legal questions, from the judgment of the Court of First Instance of Cebu,
in its Civil Case No. R-4177, denying the claim of the plaintiff-appellant, Pastor B. Tenchavez, for legal
separation and one million pesos in damages against his wife and parents-in-law, the defendants-
appellees, Vicente, Mamerto and Mena 1 , all surnamed "Escaño" respectively. 2

The facts, supported by the evidence of record, are the following:

Missing her late-afternoon classes on 24 February 1948 in the University of San Carlos, Cebu City, where
she was then enrolled as a second year student of commerce, Vicenta Escaño, 27 years of age (scion of a
well-to-do and socially prominent Filipino family of Spanish ancestry and a "sheltered colegiala"),
exchanged marriage vows with Pastor Tenchavez, 32 years of age, an engineer, ex-army officer and of
undistinguished stock, without the knowledge of her parents, before a Catholic chaplain, Lt. Moises

229
Lavares, in the house of one Juan Alburo in the said city. The marriage was the culmination of previous
love affair and was duly registered with the local civil registrar.

Vicenta's letters to Pastor, and his to her, before the marriage indicate that the couple were deeply in
love. Together with a friend, Pacita Noel, their matchmaker and go-between, they had planned out their
marital future whereby Pacita would be the governess of their first-born; they started saving money in a
piggy bank. A few weeks before their secret marriage, their engagement was broken; Vicenta returned
the engagement ring and accepted another suitor, Joseling Lao. Her love for Pastor beckoned; she
pleaded for his return and they reconciled. This time they planned to get married and then elope. To
facilitate the elopement, Vicenta had brought some of her clothes to the room of Pacita Noel in St.
Mary's Hall, which was their usual trysting place.

Although planned for the midnight following their marriage, the elopement did not, however,
materialize because when Vicenta went back to her classes after the marriage, her mother, who got
wind of the intended nuptials, was already waiting for her at the college. Vicenta was taken home where
she admitted that she had already married Pastor. Mamerto and Mena Escaño were surprised, because
Pastor never asked for the hand of Vicenta, and were disgusted because of the great scandal that the
clandestine marriage would provoke (t.s.n., vol. 111, pp. 1105-06). The following morning, the Escaño
spouses sought priestly advice. Father Reynes suggested a recelebration to validate what he believed to
be an invalid marriage, from the standpoint of the Church, due to the lack of authority from the
Archbishop or the parish priest for the officiating chaplain to celebrate the marriage. The recelebration
did not take place, because on 26 February 1948 Mamerto Escaño was handed by a maid, whose name
he claims he does not remember, a letter purportedly coming from San Carlos College students and
disclosing an amorous relationship between Pastor Tenchavez and Pacita Noel; Vicenta translated the
letter to her father, and thereafter would not agree to a new marriage. Vicenta and Pastor met that day
in the house of Mrs. Pilar Mendezona. Thereafter, Vicenta continued living with her parents while Pastor
returned to his job in Manila. Her letter of 22 March 1948 (Exh. "M"), while still solicitous of her
husband's welfare, was not as endearing as her previous letters when their love was aflame.

Vicenta was bred in Catholic ways but is of a changeable disposition and Pastor knew it. She fondly
accepted her being called a "jellyfish". She was not prevented by her parents from communicating with
Pastor (Exh. 1-Escaño"), but her letters became less frequent as the days passed. As of June, 1948 the
newlyweds were already estranged (Exh. "2-Escaño"). Vicenta had gone to Jimenez, Misamis Occidental,
to escape from the scandal that her marriage stirred in Cebu society. There, a lawyer filed for her a
petition, drafted by then Senator Emmanuel Pelaez, to annul her marriage. She did not sign the petition
(Exh. "B-5"). The case was dismissed without prejudice because of her non-appearance at the hearing
(Exh. "B-4").

On 24 June 1950, without informing her husband, she applied for a passport, indicating in her
application that she was single, that her purpose was to study, that she was domiciled in Cebu City, and
that she intended to return after two years. The application was approved, and she left for the United
States. On 22 August 1950, she filed a verified complaint for divorce against the herein plaintiff in the
Second Judicial District Court of the State of Nevada in and for the County of Washoe, on the ground of

230
"extreme cruelty, entirely mental in character". On 21 October 1950, a decree of divorce, "final and
absolute", was issued in open court by the said tribunal.

In 1951 Mamerto and Mena Escaño filed a petition with the Archbishop of Cebu to annul their
daughter's marriage to Pastor (Exh. "D"). On 10 September 1954, Vicenta sought papal dispensation of
her marriage (Exh. "D-2").

On 13 September 1954, Vicenta married an American, Russell Leo Moran, in Nevada. She now lives with
him in California, and, by him, has begotten children. She acquired American citizenship on 8 August
1958.

But on 30 July 1955, Tenchavez had initiated the proceedings at bar by a complaint in the Court of First
Instance of Cebu, and amended on 31 May 1956, against Vicenta F. Escaño, her parents, Mamerto and
Mena Escaño, whom he charged with having dissuaded and discouraged Vicenta from joining her
husband, and alienating her affections, and against the Roman Catholic Church, for having, through its
Diocesan Tribunal, decreed the annulment of the marriage, and asked for legal separation and one
million pesos in damages. Vicenta claimed a valid divorce from plaintiff and an equally valid marriage to
her present husband, Russell Leo Moran; while her parents denied that they had in anyway influenced
their daughter's acts, and counterclaimed for moral damages.

The appealed judgment did not decree a legal separation, but freed the plaintiff from supporting his
wife and to acquire property to the exclusion of his wife. It allowed the counterclaim of Mamerto
Escaño and Mena Escaño for moral and exemplary damages and attorney's fees against the plaintiff-
appellant, to the extent of P45,000.00, and plaintiff resorted directly to this Court.

The appellant ascribes, as errors of the trial court, the following:

1. In not declaring legal separation; in not holding defendant Vicenta F. Escaño liable for damages
and in dismissing the complaint;

2. In not holding the defendant parents Don Mamerto Escaño and the heirs of Doña Mena Escaño
liable for damages;

3. In holding the plaintiff liable for and requiring him to pay the damages to the defendant parents
on their counterclaim; and

4. In dismissing the complaint and in denying the relief sought by the plaintiff.

That on 24 February 1948 the plaintiff-appellant, Pastor Tenchavez, and the defendant-appellee, Vicenta
Escaño, were validly married to each other, from the standpoint of our civil law, is clearly established by
the record before us. Both parties were then above the age of majority, and otherwise qualified; and
both consented to the marriage, which was performed by a Catholic priest (army chaplain Lavares) in
the presence of competent witnesses. It is nowhere shown that said priest was not duly authorized
under civil law to solemnize marriages.

231
The chaplain's alleged lack of ecclesiastical authorization from the parish priest and the Ordinary, as
required by Canon law, is irrelevant in our civil law, not only because of the separation of the Church
and State but also because Act 3613 of the Philippine Legislature (which was the marriage law in force at
the time) expressly provided that —

"SEC. 1. Ecclesiastical Requisites. — Essential requisites for marriage are the legal capacity of the
contracting parties and their consent." (Italics supplied)

The actual authority of the solemnizing officer was thus only a formal requirement, and, therefore, not
essential to give the marriage civil effects 3 and this is emphasized by section 27 of said marriage act,
which provided the following:

"SEC. 27. Failure to comply with formal requirements. — No marriage shall be declared invalid
because of the absence of one or several of the formal requirements of this Act if, when it was
performed, the spouses or one of them believed in good faith that the person who solemnized the
marriage was actually empowered to do so, and that the marriage was perfectly legal."

The good faith of all the parties to the marriage (and hence the validity of their marriage) will be
presumed until the contrary is positively proved (Lao vs. Dee Tim, 45 Phil. 739, 745; Francisco vs. Jason,
60 Phil. 442, 448). It is well to note here that in the case at bar, doubts as to the authority of the
solemnizing priest arose only after the marriage, when Vicenta's parents consulted Father Reynes and
the archbishop of Cebu. Moreover, the very act of Vicenta in abandoning her original action for
annulment and subsequently suing for divorce implies an admission that her marriage to plaintiff was
valid and binding.

Defendant Vicenta Escaño argues that when she contracted the marriage she was under the undue
influence of Pacita Noel, whom she charges to have been in conspiracy with appellant Tenchavez. Even
granting, for argument's sake, the truth of that contention, and assuming that Vicenta's consent was
vitiated by fraud and undue influence, such vices did not render her marriage ab initio void, but merely
voidable, and the marriage remained valid until annulled by a competent civil court. This was never
done, and admittedly, Vicenta's suit for annulment in the Court of First Instance of Misamis was
dismissed for non-prosecution.

It is equally clear from the record that the valid marriage between Pastor Tenchavez and Vicenta Escaño
remained subsisting and undissolved under Philippine Law, notwithstanding the decree of absolute
divorce that the wife sought and obtained on 21 October 1950 from the Second Judicial District Court of
Washoe County, State of Nevada, on grounds of "extreme cruelty, entirely mental in character". At the
time the divorce decree was issued, Vicenta Escaño, like her husband, was still a Filipino citizen. 4 She
was then subject to Philippine law, and Article 15 of the Civil Code of the Philippines (Republic Act. No.
386), already in force at the time, expressly provided:

"Laws relating to family rights and duties or to the status, condition and legal capacity of person are
binding upon the citizens of the Philippines, even though living abroad."

232
The Civil Code of the Philippines, now in force, does not admit absolute divorce, quo ad vinculo
matrimonii; and in fact it does not even use that term, to further emphasize its restrictive policy on the
matter, in contrast to the preceding legislation that admitted absolute divorce on grounds of adultery of
the wife or concubinage of the husband (Act 2710). Instead of divorce, the present Civil Code only
provides for legal separation (Title IV, Book K, Arts. 97 to 108), and, even in that case, it expressly
prescribes that "the marriage bonds shall not be severed" (Art. 106, subpar. 1).

For the Philippine courts to recognize and give recognition or effect to a foreign decree of absolute
divorce between Filipino citizens would be a patent violation of the declared public policy of the state,
specially in view of the third paragraph of Article 17 of the Civil Code that prescribes the following:

"Prohibitive laws concerning persons, their acts or property and those which have for their object public
order, public policy and good customs, shall not be rendered ineffective by laws or judgments
promulgated, or by determinations or conventions agreed upon in a foreign country."

Even more, the grant of effectivity in this jurisdiction to such foreign divorce decrees would, in effect,
give rise to an irritating and scandalous discrimination in favor of wealthy citizens, to the detriment of
those members of our polity whose means do not permit them to sojourn abroad and obtain absolute
divorces outside the Philippines.

From this point of view, it is irrelevant that appellant Pastor Tenchavez should have appeared in the
Nevada divorce court. Primarily because the policy of our law can not be nullified by acts of private
parties (Civil Code, Art. 17, jam quot.); and additionally, because the mere appearance of a non-resident
consort can not confer jurisdiction where the court originally had none (Arca vs. Javier, 95 Phil. 579).

From the preceding facts and considerations, there flows as a necessary consequence that in this
jurisdiction Vicenta Escaño's divorce and second marriage are not entitled to recognition as valid; for her
previous union to plaintiff Tenchavez must be declared to be existent and undissolved. It follows,
likewise, that her refusal to perform her wifely duties, and her denial of consortium and her desertion of
her husband constitute in law a wrong caused through her fault, for which the husband is entitled to the
corresponding indemnity (Civil Code, Art. 2176). Neither an unsubstantiated charge of deceit, nor an
anonymous letter charging immorality against the husband constitute, contrary to her claim, adequate
excuse. Wherefore, her marriage and cohabitation with Russell Leo Moran is technically "intercourse
with a person not her husband" from the standpoint of Philippine law, and entitles plaintiff-appellant
Tenchavez to a decree of "legal separation under our law, on the basis of adultery (Revised Penal Code,
Art. 333).

The foregoing conclusions as to the untoward effect of a remarriage after an invalid divorce are in
accord with the previous doctrines and rulings of this court on the subject, particularly those that were
rendered under our laws prior to the approval of the absolute divorce act (Act 2710 of the Philippine
Legislature). As a matter of legal history, our statutes did not recognize divorces a vinculo before 1917,
when Act 2710 became effective: and the present Civil Code of the Philippines, in disregarding absolute
divorces, in effect merely reverted to the policies on the subject prevailing before Act 2710. The rulings,

233
therefore, under the Civil Code of 1889, prior to the Act above-mentioned, are now fully applicable. Of
these, the decision in Ramirez vs. Gmur, 42 Phil. 855, is of particular interest. Said this Court in that case:

"As the divorce granted by the French Court must be ignored, it results that the marriage of Dr. Mory
and Leona Castro, celebrated in London in 1905, could not legalize their relations; and the circumstance
that they afterwards passed for husband and wife in Switzerland until her death is wholly without legal
significance. The claims of the Mory children to participate in the estate of Samuel Bishop must
therefore de rejected. The right to inherit is limited to legitimate, legitimated and acknowledged natural
children. The children of adulterous relations are wholly excluded. The word "descendants" as used in
Article 941 of the Civil Code can not be interpreted to include illegitimates born of adulterous relations."
(Italics supplied)

Except for the fact that the successional rights of the children, begotten from Vicenta's marriage to Leo
Moran after the invalid divorce, are not involved in the case at bar, the Gmur case is authority for the
proposition that such union is adulterous in this jurisdiction, and, therefore, justifies an action for legal
separation on the part of the innocent consort of the first marriage, that stands undissolved in Philippine
law. In not so declaring, the trial court committed error.

True it is that our ruling gives rise to anomalous situations where the status of a person (whether
divorced or not) would depend on the territory where the question arises. Anomalies of this kind are not
new in the Philippines, and the answer to them was given in Barretto vs. Gonzales, 58 Phil. 667:

"The hardships of the existing divorce laws in the Philippine Islands are well known to the members of
the Legislature. It is the duty of the Courts to enforce laws of divorce as written by the Legislature if they
are constitutional. Courts have no right to say that such laws are too strict or too liberal." (p.72)

The appellant's first assignment of error is, therefore, sustained.

However, the plaintiff-appellant's charge that his wife's parents, Dr. Mamerto Escaño and his wife, the
late Doña Mena Escaño, alienated the affection of their daughter and influenced her conduct toward
her husband are not supported by credible evidence. The testimony of Pastor Tenchavez about the
Escaño's animosity toward him strikes us to be merely conjecture and exaggeration, and are belied by
Pastor's own letters written before this suit was begun (Exh. "2- Escaño" and "2-Vicenta", Rec. on App.
pp. 270-274). In these letters he expressly apologized to the defendants for "misjudging them" and for
the "great unhappiness" caused by his "impulsive blunders" and "sinful pride" "effrontery and audacity"
(sic). Plaintiff was admitted to the Escaño house to visit and court Vicenta, and the record shows nothing
to prove that he would not have been accepted to marry Vicenta had he openly asked for her hand, as
good manners and breeding demanded. Even after learning of the clandestine marriage, and despite
their shock at such unexpected event, the parents of Vicenta proposed and arranged that the marriage
be recelebrated in strict conformity with the canons of their religion upon advice that the previous one
was canonically defective. If no recelebration of the marriage ceremony was had it was not due to
defendants Mamerto Escaño and his wife, but to the refusal of Vicenta to proceed with it. That the
spouses Escaño did not seek to compel or induce their daughter to assent to the recelebration but
respected her decision, or that they abided by her resolve, does not constitute in law an alienation of

234
affections. Neither does the fact that Vicenta's parents sent her money while she was in the United
States; for it was natural that they should not wish their daughter to live in penury even if they did not
concur in their decision to divorce Tenchavez (27 Am. Jur. pp. 130-132).

There is no evidence that the parents of Vicenta, out of improper motives, aided and abetted her
original suit for annulment, or her subsequent divorce; she appears to have acted independently and
being of age, she was entitled to judge what was best for her and ask that her decisions be respected.
Her parents, in so doing, certainly can not be charged with alienation of affections in the absence of
malice or unworthy motives, which have not been shown, good faith being always presumed until the
contrary is proved.

"Sec. 529. Liability of Parents, Guardians or kin. — The law distinguishes between the right of a
parent to interest himself in the marital affairs of his child and the absence of right in a stranger to
intermeddle in such affairs. However, such distinction between the liability of parents and that of
strangers is only in regard to what will justify interference. A parent is liable for alienation of affections
resulting from his own malicious conduct, as where he wrongfully entices his son or daughter to leave
his or her spouse, but he is not liable unless he acts maliciously, without justification and from unworthy
motives. He is not liable where he acts and advises his child in good faith with respect to his child's
marital relations, in the interest of his child as he sees it, the marriage of his child not terminating his
right and liberty to interest himself in, and be extremely solicitous for, his child's welfare and happiness,
even where his conduct and advice suggest or result in the separation of the spouses or the obtaining of
a divorce or annulment, or where he acts under mistake or misinformation, or where his advice or
interference are indiscreet or unfortunate, although it has been held that the parent is liable for
consequences resulting from recklessness. He may in good faith take his child into his home and afford
him or her protection and support, so long as he has not maliciously enticed his child away, or does not
maliciously entice or cause him or her to stay away, from his or her spouse. This rule has more
frequently been applied in the case of advice given to a married daughter, but it is equally applicable in
the case of advice given to a son."

Plaintiff Tenchavez, in falsely charging Vicenta's aged parents with racial or social discrimination and
with having exerted efforts and pressured her to seek annulment and divorce, unquestionably caused
them unrest and anxiety, entitling them to recover damages. While his suit may not have been impelled
by actual malice, the charges were certainly reckless in the face of the proven facts and circumstances.
Court actions are not established for parties to give vent to their prejudices or spleen.

In the assessment of the moral damages recoverable by appellant Pastor Tenchavez from defendant
Vicenta Escaño, it is proper to take into account, against his patently unreasonable claim for a million
pesos in damages, that (a) the marriage was celebrated in secret, and its failure was not characterized
by publicity or undue humiliation on appellant's part; (b) that the parties never lived together; and (c)
that there is evidence that appellant had originally agreed to the annulment of the marriage, although
such a promise was legally invalid, being against public policy (cf. Art. 88, Civ. Code). While appellant is
unable to remarry under our law, this fact is a consequence of the indissoluble character of the union
that appellant entered into voluntarily and with open eyes rather than of her divorce and her second

235
marriage. All told, we are of the opinion that appellant should recover P25,000 only by way of moral
damages and attorney's fees.

With regard to the P45,000 damages awarded to the defendants, Dr. Mamerto Escaño and Mena
Escaño, by the court below, we opine that the same are excessive. While the filing of this unfounded suit
have wounded said defendant's feelings and caused them anxiety, the same could in no way have
seriously injured their reputation, or otherwise prejudiced them, lawsuits having become a common
occurrence in present society. What is important, and has been correctly established in the decision of
the court below, is that said defendants were not guilty of any improper conduct in the whole
deplorable affair. This Court, therefore, reduces the damages awarded to P5,000 only.

Summing up, the Court rules:

(1) That a foreign divorce between Filipino citizens, sought and decreed after the effectivity of the
present Civil Code (Rep. Act No. 386), is not entitled to recognition as valid in this jurisdiction; and
neither is the marriage contracted with another party by the divorced consort, subsequently to the
foreign decree of divorce, entitled to validity in this country.

(2) That the remarriage of the divorced wife and her cohabitation with a person other than the
lawful husband entitle the latter to a decree of legal separation conformably to Philippine law;

(3) That the desertion and securing of an invalid divorce decree by one consort entitles the other to
recover damages;

(4) That an action for alienation of affections against the parents of one consort does not lie in the
absence of proof of malice or unworthy motives on their part.

WHEREFORE, the decision under appeal is hereby modified, as follows:

(1) Adjudging plaintiff-appellant Pastor Tenchavez entitled to a decree of legal separation from
defendant Vicenta F. Escaño;

(2) Sentencing defendant-appellee Vicenta Escaño to pay plaintiff-appellant Tenchavez the amount
of P25,000 for damages and attorneys' fees;

(3) Sentencing appellant Pastor Tenchavez to pay the appellee, Mamerto Escaño and the estate of
his wife, the deceased Mena Escaño, P5,000 by way of damages and attorney's fee.

Neither party to recover costs.

Bengzon, C.J., Bautista Angelo, Concepcion, Dizon, Regala, Makalintal, Bengzon, J.P., and Zaldivar, JJ.,
concur.

Footnotes

1. The latter was substituted by her heirs when she died during the pendency of the case in the
trial court.

236
2. The original complaint included the Roman Catholic Church as a defendant, sought to be
enjoined from acting on a petition for the ecclesiastical annulment of the marriage between Pastor
Tenchavez and Vicenta Escaño: the case against the defendant Church was dismissed on a joint motion.

3. In the present Civil Code the contrary rule obtains (Art. 53).

4. She was naturalized as an American citizen only on 8 August 1958.

C o p y r i g h t 1 9 9 4 - 1 9 9 9 C D T e c h n o l o g i e s A s i a, I n c.

G.R. No. L-19585 November 29, 1965

People v. Ortiz

EN BANC

[G.R. No. L-19585. November 29, 1965.]

THE PEOPLE OF THE PHILIPPINES, plaintiff-appellee, vs. NAPOLEON C. ORTIZ, alias "LEON", defendant-
appellant.

Solicitor General for appellee.

Teofilo Guadiz, Jr. and Miguel T. Caguioa for defendant-appellant.

SYLLABUS

CRIMINAL PROCEDURE; PLEA OF GUILTY TO A LESSER CRIME AFTER TRIAL HAD BEGUN. — Appellant was
charged with murder and frustrated murder. Upon arraignment, he pleaded not guilty. After two
witnesses for the prosecution had testified, appellant manifested his willingness to plead guilty to the
lesser offenses of homicide and frustrated homicide. Consequently, the Fiscal, with leave of court,
amended the information accordingly, and upon new arraignment, appellant entered a plea of guilty to
the amended information. Held: The amended information was an entirely new information and no
evidence had been presented to prove the charges made therein when appellant entered his plea of
guilty. Therefore, he was entitled to have the mitigating circumstance of plea of guilty considered in his
favor in connection with the imposition of the corresponding penalty.

DECISION

DIZON, J p:

Charged in the Court of First Instance of Pangasinan (Criminal Case No. D-1210) with the murder of
Alfredo Ducusin and the frustrated murder of the latter's wife, Virginia Munar, Napoleon C. Ortiz, alias
Leon, pleaded not guilty. After two witnesses for the prosecution had testified, said defendant, through

237
counsel, manifested his willingness to plead guilty to the lesser offenses of homicide and frustrated
homicide. Consequently the Assistant Provincial Fiscal, with the approval of the court, amended the
information accordingly, and upon new arraignment, the said defendant entered a plea of guilty to the
amended information.

Thereafter, the defendant, with leave of court, presented as witness the Chief of Police of San Fabian,
Pangasinan, to prove the mitigating circumstance of voluntary surrender. Thereafter, the court,
considering the mitigating circumstance of voluntary surrender, rendered judgment sentencing the
accused to an indeterminate penalty of from 6 years and 1 day of prisión mayor to 12 years and 1 day of
reclusión temporal for the crime of homicide, and from 6 months and 1 day of prision correccional to 6
years and 1 day of prision mayor for the crime of frustrated homicide; to indemnify the offended party,
Virginia Munar, in the sum of P6,000 and to pay the costs.

Court refused to give the defendant the benefit of the mitigating circumstance of plea of guilty because
the same was made after the prosecution had already commenced the presentation of its evidence.
Claiming that the trial court erred in this respect, the defendant interposed the present appeal.

The only question to be resolved in this appeal is whether or not the lower court erred in not
considering in favor of appellant the mitigating circumstance of plea of guilty.

On the strength of our decision in People vs. Intal, 101 Phil., 306, We find appellant's contention — that
said mitigating circumstance should have been considered in his favor — to be meritorious.

It is true that, upon the original information for murder and frustrated murder, the trial had already
begun. However, when in view of the willingness of appellant to plead guilty for a lesser offense the
prosecution, with leave of court, amended said information to make it one for homicide and frustrated
homicide, appellant pleaded guilty thereto. That was an entirely new information and no evidence was
presented in connection with the charges made therein before appellant entered his plea of guilty. We
believe therefor that, as in the Intal case, appellant was entitled to have the mitigating circumstance of
plea of guilty considered in his favor in connection with the imposition of the corresponding penalty.

WHEREFORE, the decision appealed from is hereby modified as follows:

(a) In the homicide case, the defendant is sentenced to suffer an indeterminate penalty of not less
than 2 years, 4 months and 1 day of prisión correccional, nor more than 10 years of prision mayor.

(b) In the case for frustrated homicide, the same appellant is hereby sentenced to suffer an
indeterminate penalty of not less than 6 months of arresto mayor, nor more than 2 years, 4 months of
and 1 day of prisión correccional.

(c) The carbine used by appellant in committing the offenses charged, with serial number 3215519,
Cal. 30 (Irwin-Pedersen) is hereby ordered confiscated.

The appealed judgment is affirmed in all other respects. With costs.

238
Bengzon, C.J., Bautista Angelo, Concepcion, Reyes, J.B.L., Regala, Makalintal, Bengzon, J.P., and Zaldivar,
JJ., concur.

239
St Louis Realty vs. CA, 133 SCRA 179

[G.R. No. L-46061. November 14, 1984.]

ST. LOUIS REALTY CORPORATION, petitioner, vs. COURT OF APPEALS and CONRADO J. ARAMIL,
respondents.

Romeo Z. Comia for petitioner.

Roman R. Bersamin for private respondent.

SYLLABUS

CIVIL LAW; CIVIL CODE; DAMAGES; ACTS AND OMISSIONS FALLING UNDER ARTICLE 26; WRONGFUL
ADVERTISEMENT AND FAILURE TO MAKE IMMEDIATE RECTIFICATION; CASE AT BAR. — Judge Jose M.
Leuterio found that as a result of St. Louis Realty's mistake in misrepresenting the house of Doctor J.
Aramil as belonging to Arcadio S. Arcadio, magnified by its utter lack of sincerity, Doctor Aramil suffered
mental anguish and his income was reduced by about P1,000 to P1,500 a month. Moreover there was a
violation of Aramil's right to privacy (Art. 26, Civil Code). The Appellate Court adopted the facts found by
the trial court. Those factual findings are binding on the Supreme Court. The trial court awarded Aramil
P8,000 as actual damages, P20,000 as moral damages and P2,000) as attorney's fees. They are
sanctioned by Articles 2200, 2208 and 2219 of the Civil Code. Article 2219 allows moral damages for acts
and actions mentioned in Article 26. The acts and omissions of the firm fall under Article 26. St. Louis
Realty's employee was grossly negligent in mixing up the Aramil and Arcadio residences in a widely
circulated publication like the Sunday Times. To suit its purpose, it never made any written apology and
explanation of the mixup. It just contented itself with a cavalier "rectification". Persons who know the
residence of Doctor Aramil, were confused by the distorted, lingering impression that he was renting his
residence from Arcadio or that Arcadio had leased it from him. Either way, his private life was
mistakenly and unnecessarily exposed. He suffered diminution of income and mental anguish.

DECISION

AQUINO, J p:

This case is about the recovery of damages for a wrongful advertisement in the Sunday Times where
Saint Louis Realty Corporation misrepresented that the house of Doctor Conrado J. Aramil belonged to
Arcadio S. Arcadio.

St. Louis Realty caused to be published with the permission of Arcadio S. Arcadio (but without
permission of Doctor Aramil) in the issue of the Sunday Times of December 15, 1968 an advertisement
with the heading "WHERE THE HEART IS". Below that heading was the photograph of the residence of
Doctor Aramil and the Arcadio family and then below the photograph was the following write-up:

"Home is where the heart is. And the hearts of MR. AND MRS. ARCADIO S. ARCADIO and their family
have been captured by BROOKSIDE HILLS. They used to rent a small 2-bedroom house in a cramped

240
neighborhood, sadly inadequate and unwholesome for the needs of a large family. They dream(ed) of a
more pleasant place free from the din and dust of city life yet near all facilities. Plans took shape when
they heard of BROOKSIDE HILLS. With thrift and determination, they bought a lot and built their dream
house . . . for P31,000. The Arcadios are now part of the friendly, thriving community of BROOKSIDE
HILLS . . . a beautiful first-class subdivision planned for wholesome family living."

The same advertisement appeared in the Sunday Times dated January 5, 1969. Doctor Aramil, a
neuropsychiatrist and a member of the faculty of the U. E. Ramon Magsaysay Memorial Hospital,
noticed the mistake. On that same date, he wrote St. Louis Realty the following letter of protest: LLjur

"Dear Sirs:

This is anent to your advertisements appearing in the December 15, 1968 and January 5, 1969 issues of
the Sunday Times which boldly depicted my house at the above-mentioned address and implying that it
belonged to another person. I am not aware of any permission or authority on my part for the use of my
house for such publicity.

"This unauthorized use of my house for your promotional gain and much more the apparent distortions
therein are I believe not only transgression to my private property but also damaging to my prestige in
the medical profession. I have had invited in several occasions numerous medical colleagues, medical
students and friends to my house and after reading your December 15 advertisement, some of them
have uttered some remarks purporting doubts as to my professional and personal integrity. Such sly
remarks although in light vein as `it looks like your house,' `how much are you renting from the
Arcadios?', `like your wife portrayed in the papers as belonging to another husband', etc., have resulted
in no little mental anguish on my part.

"I have referred this matter to the Legal Panel of the Philippine Medical Association and their final
advice is pending upon my submission of supporting ownership papers.

"I will therefore be constrained to pursue court action against your corporation unless you could
satisfactorily explain this matter within a week upon receipt of this letter."

The letter was received by Ernesto Magtoto, an officer of St. Louis Realty in charge of advertising. He
stopped publication of the advertisement. He contacted Doctor Aramil and offered his apologies.
However, no rectification or apology was published.

On February 20, 1969, Aramil's counsel demanded from St. Louis Realty actual, moral and exemplary
damages of P110,000 (Exh. D). In its answer dated March 10, St. Louis Realty claimed that there was an
honest mistake and that if Aramil so desired, rectification would be published in the Manila Times (Exh.
3). LexLib

It published in the issue of the Manila Times of March 18, 1969 a new advertisement with the Arcadio
family and their real house. But it did not publish any apology to Doctor Aramil and an explanation of
the error.

241
On March 29, Aramil filed his complaint for damages. St. Louis Realty published in the issue of the
Manila Times of April 15, 1969 the following "NOTICE OF RECTIFICATION" in a space 4 by 3 inches:

"This will serve as a notice that our print ad `Where the Heart is' which appeared in the Manila Times
issue of March 18, 1969 is a rectification of the same ad that appeared in the Manila Times issues of
December 15, 1968 and January 5, 1969 wherein a photo of the house of another Brookside
Homeowner (Dr. Aramil-private respondent) was mistakenly used as a background for the featured
homeowner's the Arcadio family.

"The ad of March 18, 1969 shows the Arcadio family with their real house in the background, as was
intended all along."

Judge Jose M. Leuterio observed that St. Louis Realty should have immediately published a rectification
and apology. He found that as a result of St. Louis Realty's mistake, magnified by its utter lack of
sincerity, Doctor Aramil suffered mental anguish and his income was reduced by about P1,000 to P1,500
a month. Moreover, there was violation of Aramil's right to privacy (Art. 26, Civil Code).

The trial court awarded Aramil P8,000 as actual damages, P20,000 as moral damages and P2,000 as
attorney's fees. St. Louis Realty appealed to the Court of Appeals.

The Appellate Court affirmed that judgment, with Acting Presiding Justice Magno S. Gatmaitan as
ponente, and Justices Sixto A. Domondon and Samuel F. Reyes concurring. cdrep

The Appellate Court reasoned out that St. Louis Realty committed an actionable quasi-delict under
articles 21 and 26 of the Civil Code because the questioned advertisements pictured a beautiful house
which did not belong to Arcadio but to Doctor Aramil who, naturally, was annoyed by that contretemps.

In this appeal, St. Louis Realty contends that the Appellate Court ignored certain facts and resorted to
surmises and conjectures. This contention is unwarranted. The Appellate Court adopted the facts found
by the trial court. Those factual findings are binding on this Court.

St. Louis Realty also contends that the decision is contrary to law and that the case was decided in a way
not in conformity with the rulings of this Court. It argues that the case is not covered by article 26 which
provides that "every person shall respect the dignity, personality, privacy and peace of mind of his
neighbors and other persons". "Prying into the privacy of another's residence" and "meddling with or
disturbing the private life or family relations of another" and "similar acts", "though they may not
constitute a criminal offense, shall produce a cause of action for damages, prevention and other relief".

The damages fixed by Judge Leuterio are sanctioned by articles 2200, 2208 and 2219 of the Civil Code.
Article 2219 allows moral damages for acts and actions mentioned in article 26. As lengthily explained by
Justice Gatmaitan, the acts and omissions of the firm fall under article 26.

St. Louis Realty's employee was grossly negligent in mixing up the Aramil and Arcadio residences in a
widely circulated publication like the Sunday Times. To suit its purpose, it never made any written
apology and explanation of the mixup. It just contented itself with a cavalier "rectification".

242
Persons, who know the residence of Doctor Aramil, were confused by the distorted, lingering impression
that he was renting his residence from Arcadio or that Arcadio had leased it from him. Either way, his
private life was mistakenly and unnecessarily exposed. He suffered diminution of income and mental
anguish. LexLib

WHEREFORE, the judgment of the Appellate Court is affirmed. Costs against the petitioner.

SO ORDERED.

Makasiar, Concepcion, Jr., Abad Santos, Escolin and Cuevas, JJ ., concur.

243
Gregorio v. Court of Appeals, G.R. No. 179799, September 11, 2009

[G.R. No. 179799. September 11, 2009.]

ZENAIDA R. GREGORIO, petitioner, vs. COURT OF APPEALS, SANSIO PHILIPPINES, INC., and EMMA J.
DATUIN, respondents.

DECISION

NACHURA, J p:

This is a petition 1 for certiorari under Rule 45 of the Rules of Court assailing the Decision 2 of the Court
of Appeals (CA) dated January 31, 2007 and its Resolution 3 dated September 12, 2007 in CA-G.R. SP No.
63602, entitled "Sansio Philippines, Inc., et al. v. Hon. Romulo SG. Villanueva, et al."

The case arose from the filing of an Affidavit of Complaint 4 for violation of Batas Pambansa Bilang (B.P.
Blg.) 22 (Bouncing Checks Law) by respondent Emma J. Datuin (Datuin), as Officer-in-Charge of the
Accounts Receivables Department, and upon authority of petitioner Sansio Philippines, Inc. (Sansio),
against petitioner Zenaida R. Gregorio (Gregorio) and one Vito Belarmino, as proprietors of Alvi
Marketing, allegedly for delivering insufficiently funded bank checks as payment for the numerous
appliances bought by Alvi Marketing from Sansio. DIEACH

As the address stated in the complaint was incorrect, Gregorio was unable to controvert the charges
against her. Consequently, she was indicted for three (3) counts of violation of B.P. Blg. 22, docketed as
Criminal Case Nos. 236544, 236545, and 236546, before the Metropolitan Trial Court (MeTC), Branch 3,
Manila.

The MeTC issued a warrant 5 for her arrest, and it was served upon her by the armed operatives of the
Public Assistance and Reaction Against Crime (PARAC) of the Department of Interior and Local
Government (DILG) on October 17, 1997, Friday, at around 9:30 a.m. in Quezon City while she was
visiting her husband and their two (2) daughters at their city residence. Gregorio was brought to the
PARAC-DILG Office where she was subjected to fingerprinting and mug shots, and was detained. She
was released in the afternoon of the same day when her husband posted a bond for her temporary
liberty.

On December 5, 1997, Gregorio filed before the MeTC a Motion 6 for Deferment of Arraignment and
Reinvestigation, alleging that she could not have issued the bounced checks, since she did not even have
a checking account with the bank on which the checks were drawn, as certified by the branch manager
of the Philippine National Bank, Sorsogon Branch. She also alleged that her signature was patently and
radically different from the signatures appearing on the bounced checks.

The MeTC granted the Motion and a reinvestigation was conducted. In the course of the reinvestigation,
Datuin submitted an Affidavit of Desistance 7 dated August 18, 1998, stating, among others, that
Gregorio was not one of the signatories of the bounced checks subject of prosecution.

244
Subsequently, the assistant city prosecutor filed a Motion to Dismiss 8 dated November 12, 1998 with
respect to Criminal Case Nos. 236544-46. The MeTC granted the motion and ordered the B.P. Blg. 22
cases dismissed. 9

On August 18, 2000, Gregorio filed a complaint 10 for damages against Sansio and Datuin before the
Regional Trial Court (RTC), Branch 12, Ligao, Albay. The complaint, in part, reads —

4. That on or about December 15, 1995, defendant Emma J. Datuin filed with the Office of the City
Prosecutor of Manila an "Affidavit of Complaint" wherein, among others, she alleged under oath that as
an Officer In-charge of the Accounts Receivables Department of SANSIO PHILIPPINES, INC., she was duly
authorized and empowered by said company to file cases against debtors, customers and dealers of the
company;

xxx xxx xxx

5. That while acting under authority of her employer namely the defendant SANSIO PHILIPPINES,
INC., defendant EMMA J. DATUIN falsely stated in the "Affidavit of Complaint" (Annex "A"), among
others, that plaintiff Zenaida R. Gregorio issued and delivered to their office the following checks, to wit:
IEaHSD

a. PNB Check No. C-347108 dated November 30, 1992 in the amount of P9,564.00;

b. PNB Check No. C-347109 dated November 30, 1992 in the amount of P19,194.48; and

c. PNB Check No. C-347104 dated December 2, 1992 in the amount of P10,000.00

and that the above-mentioned PNB Checks bounced when deposited upon maturity;

6. That as a result of the filing of the "Affidavit of Complaint" (Annex "A") wherein defendant
Emma J. Datuin falsely charged the plaintiff with offenses of Estafa and/or violation of B.P. Blg. 22 on
three (3) counts, the Office of the City Prosecutor of Manila issued a Resolution dated April 1, 1996
finding the existence of a probable cause against the plaintiff for violation of Batas Pambansa Blg. 22 on
three counts;

xxx xxx xxx

7. That in the "MEMO OF PRELIMINARY INVESTIGATION" attached hereto as Annex "C", signed by
defendant Emma J. Datuin she falsely indicated the address of plaintiff to be at No. 76 Peñaranda Street,
Legaspi City when the truth of the matter is that the latter's correct address is at Barangay Rizal, Oas,
Albay;

8. That as a consequence of the aforegoing false and misleading indication of address, plaintiff was
therefore not duly notified of the charges filed against her by defendant Emma J. Datuin; and more, she
was not able to controvert them before the investigating prosecutor, finally resulting in the filing in
court of three (3) informations accusing her of violating B.P. 22;

245
xxx xxx xxx

9. That as pernicious result of the unwarranted and baseless accusation by the defendants which
culminated in the filing of three (3) informations in the Metropolitan Trial Court of Manila, Branch 3
indicting the plaintiff on three counts of the offense of violating B.P. 22, the said court issued a Warrant
of Arrest on July 22, 1996 ordering the arrest of the plaintiff;

xxx xxx xxx

10. That taking extra effort to expedite the apprehension of plaintiff, defendants' retained private
prosecutor managed to obtain the Warrant for the Arrest of said plaintiff from the Court as evidenced
by the copy of the letter of lawyer Alquin B. Manguerra of Chua and Associates Law Office (Annex "H")
so much so that in the morning of October 17, 1997, while plaintiff was visiting her husband Jose
Gregorio and their two daughters at their city residence at 78 K-2 Street, Kamuning, Quezon City, and
without the slightest premonition that she was wanted by the law, armed operatives of the Public
Assistance and Reaction Against Crime (PARAC) of DILG suddenly swooped down on their residence,
arrested the plaintiff and brought her to the PARAC DILG Office in Quezon City where she was
fingerprinted and detained like an ordinary criminal; CHcETA

xxx xxx xxx

11. That feeling distraught, helpless and hungry (not having eaten for a whole day) the plaintiff
languished in her place of confinement until the late afternoon of October 17, 1997 when her husband
was able to post a bond for her temporary liberty and secure an order of release (Annex "J") from the
court. It was providential that a city judge was available in the late afternoon of October 17, 1997 which
was a Friday, otherwise plaintiff would have remained in confinement for the entire weekend;

12. That because of her desire to prove and establish her innocence of the unjustified charges
lodged against her by the defendants, the plaintiff was thus compelled to retain the services of counsel
resulting in the filing of a Motion for Deferment of Arraignment and Reinvestigation (Annex "K") which
was granted by the court; the filing of a Request for Reinvestigation with the prosecutor's office (Annex
"L"); and the submission of a Counter-Affidavit to the investigating prosecutor. All of these culminated in
the filing by the investigating prosecutor of a Motion to Dismiss (Annex "M") the three criminal cases as
a consequence of which the Court issued an Order dated June 1, 1999 (Annex "N") dismissing Criminal
Cases No. 236544, No. 236545 and No. 236546, copy of which was received by plaintiff only on July 7,
2000;

13. That previous to the filing of the above-mentioned Motion to Dismiss by the prosecutor and
having been faced with the truth and righteousness of plaintiff's avowal of innocence which was
irrefutable, defendants had no recourse but to concede and recognize the verity that they had wrongly
accused an innocent person, in itself a brazen travesty of justice, so much so that defendant Emma J.
Datuin had to execute an Affidavit of Desistance (Annex "O") admitting that plaintiff is not a signatory to
the three bouncing checks in question, rationalizing, albeit lamely, that the filing of the cases against the
plaintiff was by virtue of an honest mistake or inadvertence on her (Datuin's) part;

246
14. Be that as it may, incalculable damage has been inflicted on the plaintiff on account of the
defendants' wanton, callous and reckless disregard of the fundamental legal precept that "every person
shall respect the dignity, personality, privacy and peace of mind of his neighbors and other persons"
(Art. 26, Civil Code of the Philippines);

15. That the plaintiff, being completely innocent of the charges against her as adverted to in the
preceding paragraphs, was socially humiliated, embarrassed, suffered physical discomfort, mental
anguish, fright, and serious anxiety as a proximate result of her unjustified indictment, arrest and
detention at the PARAC headquarters — all of these ordeals having been exacerbated by the fact that
plaintiff is a woman who comes from a respected family in Oas, Albay, being the wife of an executive of
the Philippine National Construction Corporation, the mother of two college students studying in
Manila, a pharmacist by profession, a businesswoman by occupation, and an incumbent Municipal
Councilor (Kagawad) of Oas, Albay, at the time of her arrest and detention; and that she previously held
the following positions: cISAHT

(a). President, Philippine Pharmaceutical Association (Albay Chapter);

(b). Chairman of the Board, Albay Pharmaceutical Marketing Cooperative (ALPHAMAC);

(c). Charter Secretary, Kiwanis Club of Oas;

(d). Chairman, Polangui Ladies Multi-Purpose Cooperative, Polangui, Albay;

(e). Vicarial Regent, Daughters of Mary Immaculate International, District IX;

(f). Chapter President and Municipal Coordinator, Albay Women Volunteers Association, Inc.,
Legaspi City;

(g). Regent, Daughters of Mary Immaculate International Virgo Clemens Circle, Oas, Albay;

(h). Secretary, Girl Scout of the Philippines District Association; and

(i). Director, Albay Electric Cooperative (ALECO),

not to mention the undue aspersion cast upon her social, professional and business reputation because
of defendants' tortious act of accusing her of Estafa and/or issuing bouncing checks — even without a
scintilla of evidence;

16. That to compound the aforegoing travails and sufferings of the plaintiff she had to devote and
spend much of her time, money and efforts trying to clear her tarnished name and reputation, including
traveling to and from Manila to confer with her lawyer, attend the hearings at the prosecutor's office
and at the Metropolitan Trial Court;

17. By and large, defendants' fault or, at the very least, their reckless imprudence or negligence, in
filing the three (3) criminal cases against the plaintiff unequivocally caused damage to the latter and
because of defendants' baseless and unjustified accusations, plaintiff was constrained to retain the

247
services of a lawyer to represent her at the Metropolitan Trial Court and at the Office of the City
Prosecutor at Manila in order to establish her innocence and cause the dismissal of the three (3)
criminal cases filed against her, reason for which she spent P20,000.00; and in order to institute this
instant action for the redress of her grievances, plaintiff have to pay the sum of P50,000.00 as attorney's
fees and incur litigation expenses in the amount of P35,000.00; CEcaTH

18. That by reason of all the aforegoing and pursuant to the provision of law that "whoever by act
or omission causes damage to another, there being fault or negligence, is obliged to pay for the damage
done", (Article 2176, Civil Code of the Philippines), the plaintiff is entitled to and hereby claims the
following items of damages:

a. P3,000,000.00 as moral damages

b. P50,000.00 as actual damages

c. P50,000.00 as nominal damages

d. P70,000.00 as attorney's fees

e. P35,000.00 as litigation expenses

19. That defendants herein are jointly and solidarily liable for the payment of the above items of
damages being co-tortfeasors. Moreover, defendant SANSIO PHILIPPINES, INC. is vicariously liable as the
employer of defendant Emma J. Datuin who patently acted within the scope of her assigned tasks (Vide:
Art. 2180, Civil Code of the Philippines). 11

Sansio and Datuin filed a Motion to Dismiss 12 on the ground that the complaint, being one for damages
arising from malicious prosecution, failed to state a cause of action, as the ultimate facts constituting
the elements thereof were not alleged in the complaint. Gregorio opposed 13 the Motion. Sansio and
Datuin filed their Reply 14 to the Opposition. Gregorio, in turn, filed her Rejoinder. 15

On October 10, 2000, the RTC issued an Order 16 denying the Motion to Dismiss. Sansio and Datuin filed
a Motion for Reconsideration 17 of the October 10, 2000 Order, but the RTC denied the same in its
Order 18 dated January 5, 2001.

Sansio and Datuin went to the CA via a petition 19 for certiorari under Rule 65 of the Rules of Court
alleging grave abuse of discretion on the part of the presiding judge of the RTC in denying their motions
to dismiss and for reconsideration.

Meanwhile, on March 20, 2003, the RTC rendered its Decision in the civil case for damages instituted by
Gregorio, directing Sansio and Datuin, jointly and solidarily, to pay Gregorio P200,000.00 as moral
damages; P10,000.00 as nominal damages; P35,000.00 as litigation expenses; P30,000.00 as attorney's
fees; and costs of the suit. The RTC expressly stated in its Decision that the complaint was one for
damages based on quasi-delict and not on malicious prosecution.

248
Aggrieved by the March 20, 2003 Decision, Sansio and Datuin appealed to the CA, and the same is now
pending resolution.

On January 31, 2007, the CA rendered a Decision on the certiorari case granting the petition and
ordering the dismissal of the damage suit of Gregorio. The latter moved to reconsider the said Decision
but the same was denied in the appellate court's Resolution dated September 12, 2007. HCDaAS

Hence, this petition.

The core issue to be resolved, as culled from the factual circumstances of this case, is whether the
complaint, a civil suit filed by Gregorio, is based on quasi-delict or malicious prosecution.

It is the position of Sansio and Datuin that the complaint for damages filed by Gregorio before the RTC
was for malicious prosecution, but it failed to allege the elements thereof, such that it was aptly
dismissed on appeal by the CA on the ground of lack of cause of action. In their comment, citing
Albenson Enterprise Corporation v. Court of Appeals, 20 they posit that Article 26 of the Civil Code, cited
by Gregorio as one of the bases for her complaint, and Articles 19, 20, and 21 of the same Code,
mentioned by the RTC as bases for sustaining the complaint, are the very same provisions upon which
malicious prosecution is grounded. And in order to further buttress their position that Gregorio's
complaint was indeed one for malicious prosecution, they even pointed out the fact that Gregorio
prayed for moral damages, which may be awarded only in case of malicious prosecution or, if the case is
for quasi-delict, only if physical injury results therefrom.

We disagree.

A perusal of the allegations of Gregorio's complaint for damages readily shows that she filed a civil suit
against Sansio and Datuin for filing against her criminal charges for violation of B.P. Blg. 22; that
respondents did not exercise diligent efforts to ascertain the true identity of the person who delivered
to them insufficiently funded checks as payment for the various appliances purchased; and that
respondents never gave her the opportunity to controvert the charges against her, because they stated
an incorrect address in the criminal complaint. Gregorio claimed damages for the embarrassment and
humiliation she suffered when she was suddenly arrested at her city residence in Quezon City while
visiting her family. She was, at the time of her arrest, a respected Kagawad in Oas, Albay. Gregorio
anchored her civil complaint on Articles 26, 21 2176, 22 and 2180 23 of the Civil Code. Noticeably,
despite alleging either fault or negligence on the part of Sansio and Datuin, Gregorio never imputed to
them any bad faith in her complaint.

Basic is the legal principle that the nature of an action is determined by the material averments in the
complaint and the character of the relief sought. 24 Undeniably, Gregorio's civil complaint, read in its
entirety, is a complaint based on quasi-delict under Article 2176, in relation to Article 26 of the Civil
Code, rather than on malicious prosecution.

In every tort case filed under Article 2176 of the Civil Code, the plaintiff has to prove by a
preponderance of evidence: (1) the damages suffered by him; (2) the fault or negligence of the

249
defendant or some other person to whose act he must respond; (3) the connection of cause and effect
between the fault or negligence and the damages incurred; and (4) that there must be no preexisting
contractual relation between the parties. 25 TEcADS

On the other hand, Article 26 of the Civil Code grants a cause of action for damages, prevention, and
other relief in cases of breach, though not necessarily constituting a criminal offense, of the following
rights: (1) right to personal dignity; (2) right to personal security; (3) right to family relations; (4) right to
social intercourse; (5) right to privacy; and (6) right to peace of mind. 26

A scrutiny of Gregorio's civil complaint reveals that the averments thereof, taken together, fulfill the
elements of Article 2176, in relation to Article 26 of the Civil Code. It appears that Gregorio's rights to
personal dignity, personal security, privacy, and peace of mind were infringed by Sansio and Datuin
when they failed to exercise the requisite diligence in determining the identity of the person they should
rightfully accuse of tendering insufficiently funded checks. This fault was compounded when they failed
to ascertain the correct address of petitioner, thus depriving her of the opportunity to controvert the
charges, because she was not given proper notice. Because she was not able to refute the charges
against her, petitioner was falsely indicted for three (3) counts of violation of B.P. Blg. 22. Although she
was never found at No. 76 Peñaranda St., Legaspi City, the office address of Alvi Marketing as stated in
the criminal complaint, Gregorio was conveniently arrested by armed operatives of the PARAC-DILG at
her city residence at 78 K-2 St., Kamuning, Quezon City, while visiting her family. She suffered
embarrassment and humiliation over her sudden arrest and detention and she had to spend time,
effort, and money to clear her tarnished name and reputation, considering that she had held several
honorable positions in different organizations and offices in the public service, particularly her being a
Kagawad in Oas, Albay at the time of her arrest. There exists no contractual relation between Gregorio
and Sansio. On the other hand, Gregorio is prosecuting Sansio, under Article 2180 of the Civil Code, for
its vicarious liability, as employer, arising from the act or omission of its employee Datuin.

These allegations, assuming them to be true, sufficiently constituted a cause of action against Sansio and
Datuin. Thus, the RTC was correct when it denied respondents' motion to dismiss.

Sansio and Datuin are in error when they insist that Gregorio's complaint is based on malicious
prosecution. In an action to recover damages for malicious prosecution, it must be alleged and
established that Sansio and Datuin were impelled by legal malice or bad faith in deliberately initiating an
action against Gregorio, knowing that the charges were false and groundless, intending to vex and
humiliate her. 27 As previously mentioned, Gregorio did not allege this in her complaint. Moreover, the
fact that she prayed for moral damages did not change the nature of her action based on quasi-delict.
She might have acted on the mistaken notion that she was entitled to moral damages, considering that
she suffered physical suffering, mental anguish, fright, serious anxiety, besmirched reputation, wounded
feelings, moral shock, and social humiliation on account of her indictment and her sudden arrest.

Verily, Gregorio was only acting within her right when she instituted against Sansio and Datuin an action
she perceived to be proper, given the factual antecedents of the case. ADaSET

250
WHEREFORE, the petition is GRANTED. The Decision dated January 31, 2007 and the Resolution dated
September 12, 2007 are REVERSED and SET ASIDE. Costs against respondents.

SO ORDERED. CIDTcH

Ynares-Santiago, Chico-Nazario, Velasco, Jr. and Peralta, JJ., concur.

251
Capili v. People, G.R. No. 183805, July 3, 2013

[G.R. No. 183805. July 3, 2013.]

JAMES WALTER P. CAPILI, petitioner, vs. PEOPLE OF THE PHILIPPINES and SHIRLEY TISMO-CAPILI,
respondents.

DECISION

PERALTA, J p:

Before us is a Petition for Review on Certiorari under Rule 45 of the Rules of Court seeking the reversal
of the Decision 1 dated February 1, 2008 and Resolution 2 dated July 24, 2008 of the Court of Appeals
(CA) in CA-G.R. CR No. 30444.

The factual antecedents are as follows:

On June 28, 2004, petitioner was charged with the crime of bigamy before the Regional Trial Court (RTC)
of Pasig City in an Information which reads:

On or about December 8, 1999, in Pasig City, and within the jurisdiction of this Honorable Court, the
accused being previously united in lawful marriage with Karla Y. Medina-Capili and without said
marriage having been legally dissolved or annulled, did then and there willfully, unlawfully and
feloniously contract a second marriage with Shirley G. Tismo, to the damage and prejudice of the latter.
cSIADa

Contrary to law. 3

Petitioner thereafter filed a Motion to Suspend Proceedings alleging that: (1) there is a pending civil case
for declaration of nullity of the second marriage before the RTC of Antipolo City filed by Karla Y. Medina-
Capili; (2) in the event that the marriage is declared null and void, it would exculpate him from the
charge of bigamy; and (3) the pendency of the civil case for the declaration of nullity of the second
marriage serves as a prejudicial question in the instant criminal case.

Consequently, the arraignment and pre-trial were reset by the RTC of Pasig City, in view of the filing of
the Motion to Suspend Proceedings filed by petitioner.

In the interim, the RTC of Antipolo City rendered a decision declaring the voidness or incipient invalidity
of the second marriage between petitioner and private respondent on the ground that a subsequent
marriage contracted by the husband during the lifetime of the legal wife is void from the beginning.

Thereafter, the petitioner accused filed his Manifestation and Motion (to Dismiss) praying for the
dismissal of the criminal case for bigamy filed against him on the ground that the second marriage
between him and private respondent had already been declared void by the RTC. EHaASD

In an Order 4 dated July 7, 2006, the RTC of Pasig City granted petitioner's Manifestation and Motion to
Dismiss, to wit:

252
The motion is anchored on the allegation that this case should be dismissed as a decision dated
December 1, 2004 had already been rendered by the Regional Trial Court of Antipolo City, Branch 72 in
Civil Case No. 01-6043 (entitled: "Karla Medina-Capili versus James Walter P. Capili and Shirley G.
Tismo," a case for declaration of nullity of marriage) nullifying the second marriage between James
Walter P. Capili and Shirley G. Tismo and said decision is already final.

In the opposition filed by the private prosecutor to the motion, it was stated, among others, that the
issues raised in the civil case are not similar or intimately related to the issue in this above-captioned
case and that the resolution of the issues in said civil case would not determine whether or not the
criminal action may proceed.

WHEREFORE, after a judicious evaluation of the issue and arguments of the parties, this Court is of the
humble opinion that there is merit on the Motion to dismiss filed by the accused as it appears that the
second marriage between James Walter P. Capili and Shirley G. Tismo had already been nullified by the
Regional Trial Court, Branch 72 of Antipolo City which has declared "the voidness, non-existent or
incipient invalidity" of the said second marriage. As such, this Court submits that there is no more
bigamy to speak of. IDSETA

SO ORDERED.

Aggrieved, private respondent filed an appeal before the CA.

Thus, in a Decision 5 dated February 1, 2008, the CA reversed and set aside the RTC's decision. The fallo
reads:

WHEREFORE, premises considered, the Order dated 07 July 2006 of the Regional Trial Court of Pasig
City, Branch 152 in Crim. Case No. 128370 is REVERSED and SET ASIDE. The case is remanded to the trial
court for further proceedings. No costs.

SO ORDERED. 6

Petitioner then filed a Motion for Reconsideration against said decision, but the same was denied in a
Resolution 7 dated July 24, 2008.

Accordingly, petitioner filed the present petition for review on certiorari alleging that:

1. THERE IS NO LEGAL BASIS FOR THE COURT OF APPEALS TO DISREGARD EXISTING


JURISPRUDENCE PRONOUNCED BY THIS HONORABLE SUPREME COURT AND TO REVERSE THE ORDER
DATED JULY 7, 2006 OF THE TRIAL COURT (REGIONAL TRIAL COURT, PASIG CITY, BRANCH 152) ISSUED IN
CRIMINAL CASE NO. 128370 GRANTING THE MOTION TO DISMISS THE CASE OF BIGAMY AGAINST
PETITIONER, INASMUCH AS THE ISSUANCE OF THE SAID ORDER IS BASED ON THE FINDINGS AND/OR
FACTS OF THE CASE IN THE DECISION OF THE REGIONAL TRIAL COURT OF ANTIPOLO CITY, BRANCH 72,
IN CIVIL CASE NO. 01-6043 AND THE CONCLUDING AND DISPOSITIVE PORTION IN THE SAID DECISION
WHICH STATES THAT, AFTER PERUSAL OF THE EVIDENCE ON RECORD AND THE TESTIMONIES OF

253
WITNESSES . . ., THE MARRIAGE BETWEEN PETITIONER JAMES WALTER P. CAPILI AND PRIVATE
RESPONDENT SHIRLEY G. TISMO, IS HEREBY NULL AND VOID. CSaHDT

2. THE COURT OF APPEALS GRAVELY ERRED AND ABUSED ITS DISCRETION AMOUNTING TO LACK
OF JURISDICTION IN HOLDING THAT THE DECLARATION OF NULLITY OF MARRIAGE BETWEEN
PETITIONER JAMES WALTER P. CAPILI AND SHIRLEY G. TISMO BY THE REGIONAL TRIAL COURT OF
ANTIPOLO CITY, BRANCH 72 IN ITS DECISION IN CIVIL CASE NO. 01-6043, IS ON THE GROUND THAT IT IS
BIGAMOUS IN NATURE, DESPITE THE ABSENCE OF ANY SUCH FINDINGS OR FACTS ON WHICH IT IS
BASED IN VIOLATION OF ARTICLE VIII, SECTION 14 OF THE 1987 CONSTITUTION, AND IN CONCLUDING
THAT THE SAID DECLARATION OF NULLITY OF MARRIAGE IS NOT A GROUND FOR DISMISSAL OF THE
BIGAMY CASE AGAINST THE PETITIONER, WHICH RULING IS NOT IN ACCORDANCE WITH THE FACTS OF
THE CASE OF THE SAID DECISION AND WHICH IS CONTRARY TO APPLICABLE LAWS AND ESTABLISHED
JURISPRUDENCE.

3. THE CASE OF TENEBRO V. COURT OF APPEALS SPEAKS FOR ITSELF. IT IS AN EXCEPTION TO


EXISTING JURISPRUDENCE INVOLVING DECLARATION OF NULLITY OF MARRIAGE AND IS APPLICABLE
ONLY TO THE SET OF FACTS IN THE SAID CASE, AND THE GROUND FOR DECLARATION OF NULLITY OF
MARRIAGE IS PSYCHOLOGICAL INCAPACITY, HENCE, THERE IS NO LEGAL BASIS FOR ABANDONING
EXISTING JURISPRUDENCE AS WHERE IN THE INSTANT CASE THE GROUND FOR DECLARATION OF
NULLITY OF MARRIAGE IS VIOLATIVE OF ARTICLE 3 IN RELATION TO ARTICLE 4 OF THE FAMILY CODE.
CTaSEI

4. THE COURT OF APPEALS GRAVELY ERRED IN NOT HOLDING THAT THE USE BY RESPONDENT
SHIRLEY G. TISMO OF THE SURNAME "CAPILI" IS ILLEGAL INASMUCH AS THE DECISION OF THE
REGIONAL TRIAL COURT OF ANTIPOLO CITY, BRANCH 72 IN CIVIL CASE NO. 01-6043 DECLARING NULL
AND VOID THE MARRIAGE BETWEEN JAMES WALTER P. CAPILI AND SHIRLEY G. TISMO HAD LONG
BECOME FINAL AND UNAPPEALABLE AS OF THE DATE OF THE SAID DECISION ON DECEMBER 1, 2004
AND DULY RECORDED IN THE RECORDS OF ENTRIES IN THE CORRESPONDING BOOK IN THE OFFICE OF
THE CIVIL REGISTRAR OF PASIG CITY AND THE NATIONAL STATISTICS OFFICE. 8

In essence, the issue is whether or not the subsequent declaration of nullity of the second marriage is a
ground for dismissal of the criminal case for bigamy.

We rule in the negative.

Article 349 of the Revised Penal Code defines and penalizes the crime of bigamy as follows:

Art. 349. Bigamy. — The penalty of prision mayor shall be imposed upon any person who shall contract a
second or subsequent marriage before the former marriage has been legally dissolved, or before the
absent spouse has been declared presumptively dead by means of a judgment rendered in the proper
proceedings. SaHIEA

The elements of the crime of bigamy, therefore, are: (1) the offender has been legally married; (2) the
marriage has not been legally dissolved or, in case his or her spouse is absent, the absent spouse could

254
not yet be presumed dead according to the Civil Code; (3) that he contracts a second or subsequent
marriage; and (4) that the second or subsequent marriage has all the essential requisites for validity. 9

In the present case, it appears that all the elements of the crime of bigamy were present when the
Information was filed on June 28, 2004.

It is undisputed that a second marriage between petitioner and private respondent was contracted on
December 8, 1999 during the subsistence of a valid first marriage between petitioner and Karla Y.
Medina-Capili contracted on September 3, 1999. Notably, the RTC of Antipolo City itself declared the
bigamous nature of the second marriage between petitioner and private respondent. Thus, the
subsequent judicial declaration of the second marriage for being bigamous in nature does not bar the
prosecution of petitioner for the crime of bigamy.

Jurisprudence is replete with cases holding that the accused may still be charged with the crime of
bigamy, even if there is a subsequent declaration of the nullity of the second marriage, so long as the
first marriage was still subsisting when the second marriage was celebrated. cACEaI

In Jarillo v. People, 10 the Court affirmed the accused's conviction for bigamy ruling that the crime of
bigamy is consummated on the celebration of the subsequent marriage without the previous one having
been judicially declared null and void, viz.:

The subsequent judicial declaration of the nullity of the first marriage was immaterial because prior to
the declaration of nullity, the crime had already been consummated. Moreover, petitioner's assertion
would only delay the prosecution of bigamy cases considering that an accused could simply file a
petition to declare his previous marriage void and invoke the pendency of that action as a prejudicial
question in the criminal case. We cannot allow that.

The outcome of the civil case for annulment of petitioner's marriage to [private complainant] had no
bearing upon the determination of petitioner's innocence or guilt in the criminal case for bigamy,
because all that is required for the charge of bigamy to prosper is that the first marriage be subsisting at
the time the second marriage is contracted.

Thus, under the law, a marriage, even one which is void or voidable, shall be deemed valid until declared
otherwise in a judicial proceeding. In this case, even if petitioner eventually obtained a declaration that
his first marriage was void ab initio, the point is, both the first and the second marriage were subsisting
before the first marriage was annulled. 11 DEICTS

In like manner, the Court recently upheld the ruling in the aforementioned case and ruled that what
makes a person criminally liable for bigamy is when he contracts a second or subsequent marriage
during the subsistence of a valid first marriage. It further held that the parties to the marriage should
not be permitted to judge for themselves its nullity, for the same must be submitted to the judgment of
competent courts and only when the nullity of the marriage is so declared can it be held as void, and so
long as there is no such declaration the presumption is that the marriage exists. Therefore, he who

255
contracts a second marriage before the judicial declaration of the first marriage assumes the risk of
being prosecuted for bigamy. 12

Finally, it is a settled rule that the criminal culpability attaches to the offender upon the commission of
the offense, and from that instant, liability appends to him until extinguished as provided by law. 13 It is
clear then that the crime of bigamy was committed by petitioner from the time he contracted the
second marriage with private respondent. Thus, the finality of the judicial declaration of nullity of
petitioner's second marriage does not impede the filing of a criminal charge for bigamy against him.
CHIaTc

WHEREFORE, premises considered, the petition is DENIED. The Decision dated February 1, 2008 and
Resolution dated July 24, 2008 of the Court of Appeals in CA-G.R. CR No. 30444 are hereby AFFIRMED.

SO ORDERED.

Velasco, Jr., Abad, Mendoza and Leonen, JJ., concur.

256
Tenebro v. Court of Appeals, G.R. No. 150758, 18 February 2004

[G.R. No. 150758. February 18, 2004.]

VERONICO TENEBRO, petitioner, vs. THE HONORABLE COURT OF APPEALS, respondent.

Urbano Palamos & Fabros for petitioner.

Solicitor General for respondent.

SYNOPSIS

The Court of Appeals affirmed the decision of the trial court that herein petitioner Veronico Tenebro
was guilty of the crime of bigamy for contracting second marriage to Leticia Ancajas while his marriage
to Hilda Villareyes was still subsisting and sentence to an indeterminate prison term. Thus, petitioner
interposed this petition for review by denying the existence of his first marriage to Villareyes and by
arguing that since his second marriage with Ancajas has ultimately been declared void ab initio on the
ground of the latter's psychological incapacity, he should be acquitted for the crime of bigamy. cHAIES

In affirming the decision of the Court of Appeals, the Court ruled that the marriage contract presented
by the prosecution served as positive evidence as to the existence of the marriage between Tenebro and
Villareyes, which should be given greater credence than documents testifying merely as to the absence
of any record of the marriage, especially considering that there is absolutely no requirement in the law
that a marriage contract needs to be submitted to the civil registrar as a condition precedent for the
validity of a marriage. The mere fact that no record of a marriage exists does not invalidate the
marriage, provided all requisites for its validity are present. There was no evidence presented by the
defense that would indicate that the marriage between Tenebro and Villareyes lacked any requisite for
validity, apart from the self-serving testimony of the accused himself.

Furthermore, the declaration of the nullity of the second marriage on the ground of psychological
incapacity is not an indicator that petitioner's marriage to Ancajas lacks the essential requisites for
validity. Although the judicial declaration of the nullity of a marriage on the ground of psychological
incapacity retroacts to the date of the celebration of the marriage insofar as the vinculum between the
spouses is concerned, it is significant to note that the said marriage is not without legal effects. There is
a recognition written into the law itself that such a marriage although void ab initio, may still produce
legal consequences. Among these legal consequences is incurring criminal liability for bigamy.

SYLLABUS

1. CRIMINAL LAW; BIGAMY; ELEMENTS. — Under Article 349 of the Revised Penal Code, the
elements of the crime of Bigamy are: (1) that the offender has been legally married; (2) that the first
marriage has not been legally dissolved or, in case his or her spouse is absent, the absent spouse could
not yet be presumed dead according to the Civil Code; (3) that he contracts a second or subsequent
marriage; and (4) that the second or subsequent marriage has all the essential requisites for validity.
TcDAHS

257
2. REMEDIAL LAW; EVIDENCE; ADMISSIBILITY; BEST EVIDENCE RULE; CERTIFIED COPY OF THE
MARRIAGE CONTRACT ISSUED BY A PUBLIC OFFICER IN THE CUSTODY THEREOF IS ADMISSIBLE. — This
being the case, the certified copy of the marriage contract, issued by a public officer in custody thereof,
was admissible as the best evidence of its contents. The marriage contract plainly indicates that a
marriage was celebrated between petitioner and Villareyes on November 10, 1986, and it should be
accorded the full faith and credence given to public documents.

3. ID.; ID.; ID.; ID.; DOCUMENTARY EVIDENCE AS TO THE ABSENCE OF RECORD IS DIFFERENT FROM
DOCUMENTARY EVIDENCE AS TO THE ABSENCE OF MARRIAGE CEREMONY. — [A]n examination of the
wordings of the certification issued by the National Statistics Office on October 7, 1995 and that issued
by the City Civil Registry of Manila on February 3, 1997 would plainly show that neither document
attests as a positive fact that there was no marriage celebrated between Veronico B. Tenebro and Hilda
B. Villareyes on November 10, 1986. Rather, the documents merely attest that the respective issuing
offices have no record of such a marriage. Documentary evidence as to the absence of a record is quite
different from documentary evidence as to the absence of a marriage ceremony, or documentary
evidence as to the invalidity of the marriage between Tenebro and Villareyes. HcSETI

4. CIVIL LAW; PERSONS AND FAMILY RELATIONS; MARRIAGE; THERE IS NO REQUIREMENT IN THE
LAW THAT A MARRIAGE CONTRACT NEED TO BE SUBMITTED TO THE CIVIL REGISTRAR AS A CONDITION
PRECEDENT FOR THE VALIDITY OF A MARRIAGE. — The marriage contract presented by the prosecution
serves as positive evidence as to the existence of the marriage between Tenebro and Villareyes, which
should be given greater credence than documents testifying merely as to absence of any record of the
marriage, especially considering that there is absolutely no requirement in the law that a marriage
contract needs to be submitted to the civil registrar as a condition precedent for the validity of a
marriage. The mere fact that no record of a marriage exists does not invalidate the marriage, provided
all requisites for its validity are present. There is no evidence presented by the defense that would
indicate that the marriage between Tenebro and Villareyes lacked any requisite for validity, apart from
the self-serving testimony of the accused himself. Balanced against this testimony are Villareyes' letter,
Ancajas' testimony that petitioner informed her of the existence of the valid first marriage, and
petitioner's own conduct, which would all tend to indicate that the first marriage had all the requisites
for validity.

5. CRIMINAL LAW; BIGAMY; DECLARATION OF THE NULLITY OF THE SECOND MARRIAGE ON THE
GROUND OF PSYCHOLOGICAL INCAPACITY IS OF ABSOLUTELY NO MOMENT. — Petitioner makes much
of the judicial declaration of the nullity of the second marriage on the ground of psychological
incapacity, invoking Article 36 of the Family Code. What petitioner fails to realize is that a declaration of
the nullity of the second marriage on the ground of psychological incapacity is of absolutely no moment
insofar as the State's penal laws are concerned. aSACED

6. ID.; ID.; PENALIZES THE MERE ACT OF CONTRACTING A SECOND OR SUBSEQUENT MARRIAGE
DURING THE SUBSISTENCE OF A VALID MARRIAGE. — As a second or subsequent marriage contracted
during the subsistence of petitioner's valid marriage to Villareyes, petitioner's marriage to Ancajas
would be null and void ab initio completely regardless of petitioner's psychological capacity or

258
incapacity. Since a marriage contracted during the subsistence of a valid marriage is automatically void,
the nullity of this second marriage is not per se an argument for the avoidance of criminal liability for
bigamy. Pertinently, Article 349 of the Revised Penal Code criminalizes "any person who shall contract a
second or subsequent marriage before the former marriage has been legally dissolved, or before the
absent spouse has been declared presumptively dead by means of a judgment rendered in the proper
proceedings". A plain reading of the law, therefore, would indicate that the provision penalizes the mere
act of contracting a second or a subsequent marriage during the subsistence of a valid marriage.

7. ID.; ID.; THERE IS NO REASON FOR DISTINGUISHING BETWEEN A SUBSEQUENT MARRIAGE THAT
IS NULL AND VOID PURELY BECAUSE IT IS A SECOND OR SUBSEQUENT MARRIAGE, AND A SUBSEQUENT
MARRIAGE THAT IS NULL AND VOID ON THE GROUND OF PSYCHOLOGICAL INCAPACITY. — Thus, as soon
as the second marriage to Ancajas was celebrated on April 10, 1990, during the subsistence of the valid
first marriage, the crime of bigamy had already been consummated. To our mind, there is no cogent
reason for distinguishing between a subsequent marriage that is null and void purely because it is a
second or subsequent marriage, and a subsequent marriage that is null and void on the ground of
psychological incapacity, at least insofar as criminal liability for bigamy is concerned. The State's penal
laws protecting the institution of marriage are in recognition of the sacrosanct character of this special
contract between spouses, and punish an individual's deliberate disregard of the permanent character
of the special bond between spouses, which petitioner has undoubtedly done. AHcaDC

8. CIVIL LAW; FAMILY CODE; VOID MARRIAGE BY REASON OF PSYCHOLOGICAL INCAPACITY; NOT
AN INDICATION THAT THE MARRIAGE LACKS THE ESSENTIAL REQUISITES FOR VALIDITY. — [T]he
declaration of the nullity of the second marriage on the ground of psychological incapacity is not an
indicator that petitioner's marriage to Ancajas lacks the essential requisites for validity. The requisites
for the validity of a marriage are classified by the Family Code into essential (legal capacity of the
contracting parties and their consent freely given in the presence of the solemnizing officer) and formal
(authority of the solemnizing officer, marriage license, and marriage ceremony wherein the parties
personally declare their agreement to marry before the solemnizing officer in the presence of at least
two witnesses). Under Article 5 of the Family Code, any male or female of the age of eighteen years or
upwards not under any of the impediments mentioned in Articles 37 and 38 may contract marriage. In
this case, all the essential and formal requisites for the validity of marriage were satisfied by petitioner
and Ancajas. Both were over eighteen years of age, and they voluntarily contracted the second marriage
with the required license before Judge Alfredo B. Perez, Jr. of the City Trial Court of Lapu-lapu City, in
the presence of at least two witnesses.

9. ID.; ID.; ID.; MAY STILL PRODUCE LEGAL CONSEQUENCES. — Although the judicial declaration of
the nullity of a marriage on the ground of psychological incapacity retroacts to the date of the
celebration of the marriage insofar as the vinculum between the spouses is concerned, it is significant to
note that said marriage is not without legal effects. Among these effects is that children conceived or
born before the judgment of absolute nullity of the marriage shall be considered legitimate. There is
therefore a recognition written into the law itself that such a marriage, although void ab initio, may still
produce legal consequences. Among these legal consequences is incurring criminal liability for bigamy.
To hold otherwise would render the State's penal laws on bigamy completely nugatory, and allow

259
individuals to deliberately ensure that each marital contract be flawed in some manner, and to thus
escape the consequences of contracting multiple marriages, while beguiling throngs of hapless women
with the promise of futurity and commitment. HcSaAD

10. CRIMINAL LAW; BIGAMY; ACCUSED DISPLAYS DELIBERATE DISREGARD FOR THE SANCTITY OF
MARRIAGE BY CONTRACTING MARRIAGE FOR A THIRD TIME. — As a final point, we note that based on
the evidence on record, petitioner contracted marriage a third time, while his marriages to Villareyes
and Ancajas were both still subsisting. Although this is irrelevant in the determination of the accused's
guilt for purposes of this particular case, the act of the accused displays a deliberate disregard for the
sanctity of marriage, and the State does not look kindly on such activities. Marriage is a special contract,
the key characteristic of which is its permanence. When an individual manifests a deliberate pattern of
flouting the foundation of the State's basic social institution, the State's criminal laws on bigamy step in.

11. ID.; ID.; PROPER PENALTY. — Under Article 349 of the Revised Penal Code, as amended, the
penalty for the crime of bigamy is prision mayor, which has a duration of six (6) years and one (1) day to
twelve (12) years. There being neither aggravating nor mitigating circumstance, the same shall be
imposed in its medium period. Applying the Indeterminate Sentence Law, petitioner shall be entitled to
a minimum term, to be taken from the penalty next lower in degree, i.e., prision correccional which has
a duration of six (6) months and one (1) day to six (6) years. Hence, the Court of Appeals correctly
affirmed the decision of the trial court which sentenced petitioner to suffer an indeterminate penalty of
four (4) years and two (2) months of prision correccional, as minimum, to eight (8) years, and one (1)
day of prision mayor, as maximum. CDcaSA

VITUG, J., separate opinion:

1. CRIMINAL LAW; BIGAMY; ELUCIDATED. — The offense of bigamy is committed when one
contracts "a second or subsequent marriage before the former marriage has been legally dissolved, or
before the absent spouse has been declared presumptively dead by means of a judgment rendered in
the proper proceedings". Bigamy presupposes a valid prior marriage and a subsequent marriage,
contracted during the subsistence of the prior union, which would have been binding were it not for its
being bigamous.

2. ID.; ID.; ABSOLUTE NULLITY OF EITHER THE FIRST OR THE SECOND MARRIAGE PRIOR TO ITS
JUDICIAL DECLARATION AS BEING VOID CONSTITUTE A VALID DEFENSE. — Would the absolute nullity of
either the first or the second marriage, prior to its judicial declaration as being void, constitute a valid
defense in a criminal action for bigamy? I believe that, except for a void marriage on account of the
psychological incapacity of a party or both parties to the marriage under Article 36 of the Family Code
(as so hereinafter explained), the answer must be in the affirmative. Void marriages are inexistent from
the very beginning, and no judicial decree is required to establish their nullity. As early as the case of
People vs. Aragon, this Court has underscored the fact that the Revised Penal Code itself does not,
unlike the rule then prevailing in Spain, require the judicial declaration of nullity of a prior void marriage
before it can be raised by way of a defense in a criminal case for bigamy. Had the law contemplated
otherwise, said the Court, "an express provision to that effect would or should have been inserted in the

260
law, (but that in) its absence, (the courts) are bound by (the) rule of strict interpretation" of penal
statutes. In contrast to a voidable marriage which legally exists until judicially annulled (and, therefore,
not a defense in a bigamy charge if the second marriage were contracted prior to the decree of
annulment), the complete nullity, however, of a previously contracted marriage, being void ab initio and
legally inexistent, can outrightly be a defense in an indictment for bigamy. ASIDTa

3. ID.; STATUTORY CONSTRUCTION; THERE IS NO CLEAR INDICATION THAT ARTICLE 40 OF THE


FAMILY CODE HAS AMENDED OR INTENDED TO AMEND THE REVISED PENAL CODE. — It is only "for
purposes of remarriage" that the law has expressed that the absolute nullity of the previous marriage
may be invoked "on the basis solely of a final judgment declaring such previous marriage void." It may
not be amiss to state that under the regime of the Civil Code of 1950, the Supreme Court, in Wiegel vs.
Judge Sempio-Diy, has held that a subsequent marriage of one of the spouses of a prior void marriage is
itself (the subsequent marriage) void if it were contracted before a judicial declaration of nullity of the
previous marriage. Although this pronouncement has been abandoned in a later decision of the court in
Yap vs. Court of Appeals, the Family Code, however, has seen it fit to adopt the Wiegel rule but only for
purposes of remarriage which is just to say that the subsequent marriage shall itself be considered void.
There is no clear indication to conclude that the Family Code has amended or intended to amend the
Revised Penal Code or to abandon the settled and prevailing jurisprudence on the matter.

4. CIVIL LAW; FAMILY CODE; VOID MARRIAGE DUE TO PSYCHOLOGICAL INCAPACITY; TAKEN FROM
CANON LAW PRIMARILY TO RECONCILE THE GROUNDS FOR NULLITY OF MARRIAGE UNDER CIVIL LAW
WITH THOSE OF CHURCH LAWS. — A void marriage under Article 36 of the Family Code is a class by
itself. The provision has been taken from Canon Law primarily to reconcile the grounds for nullity of
marriage under civil law with those of church laws. The "psychological incapacity to comply" with the
essential marital obligations of the spouses is completely distinct from other grounds for nullity which
are confined to the essential or formal requisites of a marriage, such as lack of legal capacity or
disqualification of the contracting parties, want of consent, absence of a marriage license, or the like.
cCAIaD

5. ID.; ID.; ID.; HAVE THE EARMARKS OF A VOIDABLE, MORE THAN A VOID, MARRIAGE. — The
effects of a marriage attended by psychological incapacity of a party or the parties thereto may be said
to have the earmarks of a voidable, more than a void, marriage, remaining to be valid until it is judicially
decreed to be a nullity. Thus, Article 54 of the Family Code considers children conceived or born of such
a void marriage before its judicial declaration of nullity to be legitimate similar to the rule on a voidable
marriage. It is expected, even as I believe it safe to assume, that the spouses' rights and obligations,
property regime and successional rights would continue unaffected, as if it were a voidable marriage,
unless and until the marriage is judicially declared void for basically two reasons: First, psychological
incapacity, a newly-added ground for the nullity of a marriage under the Family Code, breaches neither
the essential nor the formal requisites of a valid marriage; and second, unlike the other grounds for
nullity of marriage (i.e., relationship, minority of the parties, lack of license, mistake in the identity of the
parties) which are capable of relatively easy demonstration, psychological incapacity, however, being a
mental state, may not so readily be as evident. It would have been logical for the Family Code to

261
consider such a marriage explicitly voidable rather than void if it were not for an apparent attempt to
make it closely coincide with the Canon Law rules and nomenclature.

6. ID.; ID.; ID.; NOT CONVALIDATED BY EITHER COHABITATION OR PRESCRIPTION. — Indeed, a void
marriage due to psychological incapacity appears to merely differ from a voidable marriage in that,
unlike the latter, it is not convalidated by either cohabitation or prescription. It might be recalled that
prior to Republic Act No. 8533, further amending the Family Code, an action or defense of absolute
nullity of marriages falling under Article 36, celebrated before the effectivity of the Code, could
prescribe in ten years following the effectivity of the Family Code. The initial provision of the ten-year
period of prescription seems to betray a real consciousness by the framers that marriages falling under
Article 36 are truly meant to be inexistent. IHEDAT

7. ID.; ID.; ID.; REMAINS TO BE BINDING AND EFFICACIOUS UNTIL JUDICIALLY DECLARED
OTHERWISE. — Considerations, both logical and practical, would point to the fact that a "void" marriage
due to psychological incapacity remains, for all intents and purposes, to be binding and efficacious until
judicially declared otherwise. Without such marriage having first been declared a nullity (or otherwise
dissolved), a subsequent marriage could constitute bigamy. Thus, a civil case questioning the validity of
the first marriage would not be a prejudicial issue much in the same way that a civil case assailing a prior
"voidable" marriage (being valid until annulled) would not be a prejudicial question to the prosecution
of a criminal offense for bigamy.

8. CRIMINAL LAW; BIGAMY; NOT COMMITTED IN CASES WHERE THE SECOND MARRIAGE IS VOID
ON GROUNDS OTHER THAN THE EXISTENCE OF THE FIRST MARRIAGE. — In cases where the second
marriage is void on grounds other than the existence of the first marriage, this Court has declared in a
line of cases that no crime of bigamy is committed. The Court has explained that for a person to be held
guilty of bigamy, it must, even as it needs only, be shown that the subsequent marriage has all the
essential elements of a valid marriage, were it not for the subsisting first union. Hence, where it is
established that the second marriage has been contracted without the necessary license and thus void,
or that the accused is merely forced to enter into the second (voidable) marriage, no criminal liability for
the crime of bigamy can attach. In both and like instances, however, the lapse refers to the elements
required for contracting a valid marriage. If, then, all the requisites for the perfection of the contract of
marriage, freely and voluntarily entered into, are shown to be extant, the criminal liability for bigamy
can unassailably arise.

9. ID.; ID.; JUDICIAL DECLARATION OF NULLITY OF A BIGAMOUS MARRIAGE ON THE GROUND OF


PSYCHOLOGICAL INCAPACITY MERELY NULLIFIES THE EFFECTS OF THE MARRIAGE BUT IT DOES NOT
NEGATE THE FACT OF PERFECTION OF THE BIGAMOUS MARRIAGE. — Since psychological incapacity,
upon the other hand, does not relate to an infirmity in the elements, either essential or formal, in
contracting a valid marriage, the declaration of nullity subsequent to the bigamous marriage due to that
ground, without more, would be inconsequential in a criminal charge for bigamy. The judicial
declaration of nullity of a bigamous marriage on the ground of psychological incapacity merely nullifies
the effects of the marriage but it does not negate the fact of perfection of the bigamous marriage. Its
subsequent declaration of nullity dissolves the relationship of the spouses but, being alien to the

262
requisite conditions for the perfection of the marriage, the judgment of the court is no defense on the
part of the offender who has entered into it. HCEISc

CARPIO, J., dissenting opinion:

1. CRIMINAL LAW; BIGAMY; ESSENTIAL ELEMENTS. — Under Article 349 of the Revised Penal Code,
the essential elements of the crime of bigamy are: 1. The offender is legally married; 2. The marriage is
not legally dissolved; 3. The offender contracts a second or subsequent marriage; 4. The second or
subsequent marriage is valid except for the existence of the first marriage.

2. ID.; ID.; VOID MARRIAGE DUE TO PSYCHOLOGICAL INCAPACITY CANNOT CONSTITUTE A SECOND
MARRIAGE TO SUSTAIN A CONVICTION THEREFOR. — If the second marriage is void ab initio on grounds
other than the existence of the first marriage, then legally there exists no second marriage. Article 35 of
the Family Code enumerates the marriages that are "void from the beginning." The succeeding article,
Article 36, declares that a marriage contracted by one psychologically incapacitated "shall likewise be
void." Article 1409 of the Civil Code declares "inexistent and void from the beginning" contracts
"expressly . . . declared void by law." Thus, a marriage contracted by one psychologically incapacitated at
the time of the marriage is legally inexistent and void from the beginning. Such void marriage cannot
constitute a second marriage to sustain a conviction for bigamy under Article 349 of the Revised Penal
Code. If the second marriage is void solely because of the existence of the first marriage, the nullity of
the second marriage proceeds from its illegality or bigamous nature. However, if the second marriage is
void on grounds other than the existence of the first marriage, the nullity does not proceed from its
illegality or bigamous nature. The first situation results in the crime of bigamy while the second does
not. This is clear from Article 1411 of the Civil Code[.] . . . Thus, if the second marriage is void because of
Code [.] . . . Thus, if the second marriage is void because of psychological incapacity, the nullity does not
proceed from an illegal or criminal cause, and no prosecution could ensue. However, if the second
marriage is void solely because of the existence of the first marriage, the nullity proceeds from an illegal
or criminal cause, and thus prosecution should follow. DCIEac

3. ID.; ID.; SECOND MARRIAGE MUST BE VALID WERE IT NOT FOR THE EXISTENCE OF THE FIRST
MARRIAGE. — The plain and ordinary meaning of Article 349 could only be that the second marriage
must be valid were it not for the existence of the first marriage. This has been the consistent
interpretation of the Court for more than seven decades since the enactment of the Revised Penal Code.
Text writers in criminal law have never entertained or advanced any other interpretation. There is no
cogent reason to depart from the well-established jurisprudence on Article 349 of the Revised Penal
Code. . . . Article 349 of the Revised Penal Code does not state that it is immaterial whether the second
marriage is valid or void ab initio. This Article does not also state that the mere act of celebration of the
second marriage, while the first marriage subsists, constitutes the crime of bigamy. Article 349 speaks of
a "second or subsequent marriage" which, as commonly understood and applied consistently by the
Court, means a valid second marriage were it not for the existence of the first marriage.

4. STATUTORY CONSTRUCTION; SUBSTANTIVE DUE PROCESS OF LAW REQUIRES A STRICT


INTERPRETATION AGAINST THE STATE AND A LIBERAL INTERPRETATION IN FAVOR OF THE ACCUSED. —

263
Even assuming, for the sake of argument, there is doubt on the interpretation of Article 349, substantive
due process of law requires a strict interpretation of Article 349 against the State and a liberal
interpretation in favor of the accused. The majority opinion reverses this principle and interprets Article
349 of the Revised Penal Code strictly against the accused and liberally in favor of the State.

5. STATUTORY CONSTRUCTION; PENAL LAWS ARE LIBERALLY CONSTRUED IN FAVOR OF THE


ACCUSED AND STRICTLY AGAINST THE STATE. — The principle of statutory construction that penal laws
are liberally construed in favor of the accused and strictly against the State is deeply rooted in the need
to protect constitutional guarantees. This principle serves notice to the public that only those acts
clearly and plainly prohibited in penal laws are subject to criminal sanctions. To expand penal laws
beyond their clear and plain meaning is no longer fair notice to the public. Thus, the principle insures
observance of due process of law. The principle also prevents discriminatory application of penal laws.
State prosecutors have no power to broaden arbitrarily the application of penal laws beyond the plain
and common understanding of the people who are subject to their penalties. Hence, the principle
insures equal protection of the law. acAESC

6. ID.; ID.; ID.; TO HOLD THAT THE VALIDITY OF THE SECOND MARRIAGE IS IMMATERIAL IS A
VIOLATION THEREOF. — To hold that the validity of the second marriage is immaterial, as the majority
opinion so holds, would interpret Article 349 too liberally in favor of the State and too strictly against
the accused. This violates the well-settled principle of statutory construction that the Court declared in
People v. Garcia: The principle of statutory construction that penal laws are liberally construed in favor
of the accused and strictly against the State is deeply rooted in the need to protect constitutional
guarantees. Criminal and penal statutes must be strictly construed, that is, they cannot be enlarged or
extended by intendment, implication, or by any equitable considerations. In other words, the language
cannot be enlarged beyond the ordinary meaning of its terms in order to carry into effect the general
purpose for which the statute was enacted. Only those persons, offenses, and penalties, clearly
included, beyond any reasonable doubt, will be considered within the statute's operation. They must
come clearly within both the spirit and the letter of the statute, and where there is any reasonable
doubt, it must be resolved in favor of the person accused of violating the statute; that is, all questions in
doubt will be resolved in favor of those from whom the penalty is sought. (Statutory Construction,
Crawford, pp. 460-462.)

7. ID.; PRINCIPLE THAT PENAL LAWS ARE LIBERALLY CONSTRUED IN FAVOR OF THE ACCUSED AND
STRICTLY AGAINST THE STATE, ROOTED IN THE NEED TO MAINTAIN THE SEPARATION OF POWERS. —
The principle is also rooted in the need to maintain the separation of powers by insuring that the
legislature, and not the judiciary, defines crimes and prescribes their penalties. As aptly stated by the
U.S. Supreme Court, speaking through Chief Justice John Marshall, in United States v. Wiltberger: The
rule that penal laws are to be construed strictly, is perhaps not much less old than construction itself. It
is founded on the tenderness of the law for the rights of individuals; and on the plain principle that the
power of punishment is vested in the legislature, not in the judicial department. It is the legislature, not
the Court, which is to define a crime, and ordain its punishment. ESCDHA

264
8. ID.; ID.; AS BETWEEN TWO REASONABLE INTERPRETATIONS, THE MORE LENIENT ONE SHOULD
BE APPLIED TO PENAL STATUTES. — These opposing interpretations of a criminal statute call for the
application of another well-established rule that as between two reasonable interpretations, the more
lenient one should be applied to penal statutes. A leading English decision puts it in this wise: If there is
a reasonable interpretation which will avoid the penalty in any particular case, we must adopt that
construction. If there are two reasonable constructions, we must give the more lenient one. That is the
settled rule for construction of penal sections.

CALLEJO, SR., J., separate dissenting opinion:

CRIMINAL LAW; BIGAMY; DOES NOT EXIST IF THE SECOND MARRIAGE IS NULL AND VOID AB INITIO. —
The prosecution was burdened to prove beyond reasonable doubt the corpus delicti, namely, all the
elements of the crime. In this case, the prosecution adduced evidence that the petitioner contracted
marriage with Hilda and during the subsistence of said marriage, he contracted a second marriage with
the private respondent. However, the petitioner adduced in evidence the decision of the Regional Trial
Court in Civil Case No. AU-885 before the court a quo rendered judgment convicting the petitioner of
bigamy declaring null and void ab initio the petitioner's marriage with the private respondent on the
ground of the latter's psychological incapacity. Since the second marriage is null and void ab initio, such
marriage in contemplation of criminal law never existed and for that reason, one of the essential
elements of bigamy has disappeared. HCaDET

DECISION

YNARES-SANTIAGO, J p:

We are called on to decide the novel issue concerning the effect of the judicial declaration of the nullity
of a second or subsequent marriage, on the ground of psychological incapacity, on an individual's
criminal liability for bigamy. We hold that the subsequent judicial declaration of nullity of marriage on
the ground of psychological incapacity does not retroact to the date of the celebration of the marriage
insofar as the Philippines' penal laws are concerned. As such, an individual who contracts a second or
subsequent marriage during the subsistence of a valid marriage is criminally liable for bigamy,
notwithstanding the subsequent declaration that the second marriage is void ab initio on the ground of
psychological incapacity. EIDTAa

Petitioner in this case, Veronico Tenebro, contracted marriage with private complainant Leticia Ancajas
on April 10, 1990. The two were wed by Judge Alfredo B. Perez, Jr. of the City Trial Court of Lapu-lapu
City. Tenebro and Ancajas lived together continuously and without interruption until the latter part of
1991, when Tenebro informed Ancajas that he had been previously married to a certain Hilda Villareyes
on November 10, 1986. Tenebro showed Ancajas a photocopy of a marriage contract between him and
Villareyes. Invoking this previous marriage, petitioner thereafter left the conjugal dwelling which he
shared with Ancajas, stating that he was going to cohabit with Villareyes. 1

On January 25, 1993, petitioner contracted yet another marriage, this one with a certain Nilda Villegas,
before Judge German Lee, Jr. of the Regional Trial Court of Cebu City, Branch 15. 2 When Ancajas

265
learned of this third marriage, she verified from Villareyes whether the latter was indeed married to
petitioner. In a handwritten letter, 3 Villareyes confirmed that petitioner, Veronico Tenebro, was indeed
her husband.

Ancajas thereafter filed a complaint for bigamy against petitioner. 4 The Information, 5 which was
docketed as Criminal Case No. 013095-L, reads:

That on the 10th day of April 1990, in the City of Lapu-lapu, Philippines, and within the jurisdiction of
this Honorable Court, the aforenamed accused, having been previously united in lawful marriage with
Hilda Villareyes, and without the said marriage having been legally dissolved, did then and there
willfully, unlawfully and feloniously contract a second marriage with LETICIA ANCAJAS, which second or
subsequent marriage of the accused has all the essential requisites for validity were it not for the
subsisting first marriage.

CONTRARY TO LAW.

When arraigned, petitioner entered a plea of "not guilty". 6

During the trial, petitioner admitted having cohabited with Villareyes from 1984-1988, with whom he
sired two children. However, he denied that he and Villareyes were validly married to each other,
claiming that no marriage ceremony took place to solemnize their union. 7 He alleged that he signed a
marriage contract merely to enable her to get the allotment from his office in connection with his work
as a seaman. 8 He further testified that he requested his brother to verify from the Civil Register in
Manila whether there was any marriage at all between him and Villareyes, but there was no record of
said marriage. 9

On November 10, 1997, the Regional Trial Court of Lapu-lapu City, Branch 54, rendered a decision
finding the accused guilty beyond reasonable doubt of the crime of bigamy under Article 349 of the
Revised Penal Code, and sentencing him to four (4) years and two (2) months of prision correccional, as
minimum, to eight (8) years and one (1) day of prision mayor, as maximum. 10 On appeal, the Court of
Appeals affirmed the decision of the trial court. Petitioner's motion for reconsideration was denied for
lack of merit.

Hence, the instant petition for review on the following assignment of errors:

I. THE HONORABLE COURT OF APPEALS GRAVELY ERRED, AND THIS ERROR IS CORRECTIBLE IN THIS
APPEAL — WHEN IT AFFIRMED THE DECISION OF THE HONORABLE COURT A QUO CONVICTING THE
ACCUSED FOR (sic) THE CRIME OF BIGAMY, DESPITE THE NON-EXISTENCE OF THE FIRST MARRIAGE AND
INSUFFICIENCY OF EVIDENCE.

II. THE COURT ERRED IN CONVICTING THE ACCUSED FOR (sic) THE CRIME OF BIGAMY DESPITE
CLEAR PROOF THAT THE MARRIAGE BETWEEN THE ACCUSED AND PRIVATE COMPLAINANT HAD BEEN
DECLARED NULL AND VOID AB INITIO AND WITHOUT LEGAL FORCE AND EFFECT. 11

266
After a careful review of the evidence on record, we find no cogent reason to disturb the assailed
judgment.

Under Article 349 of the Revised Penal Code, the elements of the crime of Bigamy are:

(1) that the offender has been legally married;

(2) that the first marriage has not been legally dissolved or, in case his or her spouse is absent, the
absent spouse could not yet be presumed dead according to the Civil Code;

(3) that he contracts a second or subsequent marriage; and

(4) that the second or subsequent marriage has all the essential requisites for validity. 12

Petitioner's assignment of errors presents a two-tiered defense, in which he (1) denies the existence of
his first marriage to Villareyes, and (2) argues that the declaration of the nullity of the second marriage
on the ground of psychological incapacity, which is an alleged indicator that his marriage to Ancajas
lacks the essential requisites for validity, retroacts to the date on which the second marriage was
celebrated. 13 Hence, petitioner argues that all four of the elements of the crime of bigamy are absent,
and prays for his acquittal. 14

Petitioner's defense must fail on both counts.

First, the prosecution presented sufficient evidence, both documentary and oral, to prove the existence
of the first marriage between petitioner and Villareyes. Documentary evidence presented was in the
form of: (1) a copy of a marriage contract between Tenebro and Villareyes, dated November 10, 1986,
which, as seen on the document, was solemnized at the Manila City Hall before Rev. Julieto Torres, a
Minister of the Gospel, and certified to by the Office of the Civil Registrar of Manila; 15 and (2) a
handwritten letter from Villareyes to Ancajas dated July 12, 1994, informing Ancajas that Villareyes and
Tenebro were legally married. 16

To assail the veracity of the marriage contract, petitioner presented (1) a certification issued by the
National Statistics Office dated October 7, 1995; 17 and (2) a certification issued by the City Civil Registry
of Manila, dated February 3, 1997. 18 Both these documents attest that the respective issuing offices
have no record of a marriage celebrated between Veronico B. Tenebro and Hilda B. Villareyes on
November 10, 1986.

To our mind, the documents presented by the defense cannot adequately assail the marriage contract,
which in itself would already have been sufficient to establish the existence of a marriage between
Tenebro and Villareyes.

All three of these documents fall in the category of public documents, and the Rules of Court provisions
relevant to public documents are applicable to all. Pertinent to the marriage contract, Section 7 of Rule
130 of the Rules of Court reads as follows:

267
Sec. 7. Evidence admissible when original document is a public record. — When the original of a
document is in the custody of a public officer or is recorded in a public office, its contents may be
proved by a certified copy issued by the public officer in custody thereof (Emphasis ours).

This being the case, the certified copy of the marriage contract, issued by a public officer in custody
thereof, was admissible as the best evidence of its contents. The marriage contract plainly indicates that
a marriage was celebrated between petitioner and Villareyes on November 10, 1986, and it should be
accorded the full faith and credence given to public documents.

Moreover, an examination of the wordings of the certification issued by the National Statistics Office on
October 7, 1995 and that issued by the City Civil Registry of Manila on February 3, 1997 would plainly
show that neither document attests as a positive fact that there was no marriage celebrated between
Veronico B. Tenebro and Hilda B. Villareyes on November 10, 1986. Rather, the documents merely
attest that the respective issuing offices have no record of such a marriage. Documentary evidence as to
the absence of a record is quite different from documentary evidence as to the absence of a marriage
ceremony, or documentary evidence as to the invalidity of the marriage between Tenebro and
Villareyes.

The marriage contract presented by the prosecution serves as positive evidence as to the existence of
the marriage between Tenebro and Villareyes, which should be given greater credence than documents
testifying merely as to absence of any record of the marriage, especially considering that there is
absolutely no requirement in the law that a marriage contract needs to be submitted to the civil
registrar as a condition precedent for the validity of a marriage. The mere fact that no record of a
marriage exists does not invalidate the marriage, provided all requisites for its validity are present. 19
There is no evidence presented by the defense that would indicate that the marriage between Tenebro
and Villareyes lacked any requisite for validity, apart from the self-serving testimony of the accused
himself. Balanced against this testimony are Villareyes' letter, Ancajas' testimony that petitioner
informed her of the existence of the valid first marriage, and petitioner's own conduct, which would all
tend to indicate that the first marriage had all the requisites for validity.

Finally, although the accused claims that he took steps to verify the non-existence of the first marriage
to Villareyes by requesting his brother to validate such purported non-existence, it is significant to note
that the certifications issued by the National Statistics Office and the City Civil Registry of Manila are
dated October 7, 1995 and February 3, 1997, respectively. Both documents, therefore, are dated after
the accused's marriage to his second wife, private respondent in this case.

As such, this Court rules that there was sufficient evidence presented by the prosecution to prove the
first and second requisites for the crime of bigamy.

The second tier of petitioner's defense hinges on the effects of the subsequent judicial declaration 20 of
the nullity of the second marriage on the ground of psychological incapacity.

268
Petitioner argues that this subsequent judicial declaration retroacts to the date of the celebration of the
marriage to Ancajas. As such, he argues that, since his marriage to Ancajas was subsequently declared
void ab initio, the crime of bigamy was not committed. 21

This argument is not impressed with merit.

Petitioner makes much of the judicial declaration of the nullity of the second marriage on the ground of
psychological incapacity, invoking Article 36 of the Family Code. What petitioner fails to realize is that a
declaration of the nullity of the second marriage on the ground of psychological incapacity is of
absolutely no moment insofar as the State's penal laws are concerned. HEScID

As a second or subsequent marriage contracted during the subsistence of petitioner's valid marriage to
Villareyes, petitioner's marriage to Ancajas would be null and void ab initio completely regardless of
petitioner's psychological capacity or incapacity. 22 Since a marriage contracted during the subsistence
of a valid marriage is automatically void, the nullity of this second marriage is not per se an argument for
the avoidance of criminal liability for bigamy. Pertinently, Article 349 of the Revised Penal Code
criminalizes "any person who shall contract a second or subsequent marriage before the former
marriage has been legally dissolved, or before the absent spouse has been declared presumptively dead
by means of a judgment rendered in the proper proceedings". A plain reading of the law, therefore,
would indicate that the provision penalizes the mere act of contracting a second or a subsequent
marriage during the subsistence of a valid marriage.

Thus, as soon as the second marriage to Ancajas was celebrated on April 10, 1990, during the
subsistence of the valid first marriage, the crime of bigamy had already been consummated. To our
mind, there is no cogent reason for distinguishing between a subsequent marriage that is null and void
purely because it is a second or subsequent marriage, and a subsequent marriage that is null and void
on the ground of psychological incapacity, at least insofar as criminal liability for bigamy is concerned.
The State's penal laws protecting the institution of marriage are in recognition of the sacrosanct
character of this special contract between spouses, and punish an individual's deliberate disregard of
the permanent character of the special bond between spouses, which petitioner has undoubtedly done.

Moreover, the declaration of the nullity of the second marriage on the ground of psychological
incapacity is not an indicator that petitioner's marriage to Ancajas lacks the essential requisites for
validity. The requisites for the validity of a marriage are classified by the Family Code into essential (legal
capacity of the contracting parties and their consent freely given in the presence of the solemnizing
officer) 23 and formal (authority of the solemnizing officer, marriage license, and marriage ceremony
wherein the parties personally declare their agreement to marry before the solemnizing officer in the
presence of at least two witnesses). 24 Under Article 5 of the Family Code, any male or female of the
age of eighteen years or upwards not under any of the impediments mentioned in Articles 37 25 and 38
26 may contract marriage. 27

In this case, all the essential and formal requisites for the validity of marriage were satisfied by
petitioner and Ancajas. Both were over eighteen years of age, and they voluntarily contracted the

269
second marriage with the required license before Judge Alfredo B. Perez, Jr. of the City Trial Court of
Lapu-lapu City, in the presence of at least two witnesses.

Although the judicial declaration of the nullity of a marriage on the ground of psychological incapacity
retroacts to the date of the celebration of the marriage insofar as the vinculum between the spouses is
concerned, it is significant to note that said marriage is not without legal effects. Among these effects is
that children conceived or born before the judgment of absolute nullity of the marriage shall be
considered legitimate. 28 There is therefore a recognition written into the law itself that such a
marriage, although void ab initio, may still produce legal consequences. Among these legal
consequences is incurring criminal liability for bigamy. To hold otherwise would render the State's penal
laws on bigamy completely nugatory, and allow individuals to deliberately ensure that each marital
contract be flawed in some manner, and to thus escape the consequences of contracting multiple
marriages, while beguiling throngs of hapless women with the promise of futurity and commitment.

As such, we rule that the third and fourth requisites for the crime of bigamy are present in this case, and
affirm the judgment of the Court of Appeals.

As a final point, we note that based on the evidence on record, petitioner contracted marriage a third
time, while his marriages to Villareyes and Ancajas were both still subsisting. Although this is irrelevant
in the determination of the accused's guilt for purposes of this particular case, the act of the accused
displays a deliberate disregard for the sanctity of marriage, and the State does not look kindly on such
activities. Marriage is a special contract, the key characteristic of which is its permanence. When an
individual manifests a deliberate pattern of flouting the foundation of the State's basic social institution,
the State's criminal laws on bigamy step in.

Under Article 349 of the Revised Penal Code, as amended, the penalty for the crime of bigamy is prision
mayor, which has a duration of six (6) years and one (1) day to twelve (12) years. There being neither
aggravating nor mitigating circumstance, the same shall be imposed in its medium period. Applying the
Indeterminate Sentence Law, petitioner shall be entitled to a minimum term, to be taken from the
penalty next lower in degree, i.e., prision correccional which has a duration of six (6) months and one (1)
day to six (6) years. Hence, the Court of Appeals correctly affirmed the decision of the trial court which
sentenced petitioner to suffer an indeterminate penalty of four (4) years and two (2) months of prision
correccional, as minimum, to eight (8) years, and one (1) day of prision mayor, as maximum.

WHEREFORE, in view of all the foregoing, the instant petition for review is DENIED. The assailed decision
of the Court of Appeals in CA-G.R. CR No. 21636, convicting petitioner Veronico Tenebro of the crime of
Bigamy and sentencing him to suffer the indeterminate penalty of four (4) years and two (2) months of
prision correccional, as minimum, to eight (8) years and one (1) day of prision mayor, as maximum, is
AFFIRMED in toto.

SO ORDERED.

Davide, Jr., C.J., Panganiban, Sandoval-Gutierrez, Corona and Azcuna, JJ., concur.

270
Puno, J., joins the opinion of J. Vitug.

Vitug, J., see separate opinion.

Quisumbing, J., joins the dissent in view of void nuptia.

Carpio, J., see dissenting opinion.

Austria-Martinez, Carpio-Morales and Tinga, JJ., join the dissent of J. Carpio.

Callejo, Sr., J., see separate dissent.

Separate Opinions

VITUG, J.:

Veronico Tenebro has been charged with bigamy for contracting, while still being married to Hilda
Villareyes, a second marriage with private complainant Leticia Ancajas. Tenebro argues that since his
second marriage with Ancajas has ultimately been declared void ab initio on the ground of the latter's
psychological incapacity, he should be acquitted for the crime of bigamy.

The offense of bigamy is committed when one contracts "a second or subsequent marriage before the
former marriage has been legally dissolved, or before the absent spouse has been declared
presumptively dead by means of a judgment rendered in the proper proceedings". 1 Bigamy
presupposes a valid prior marriage and a subsequent marriage, contracted during the subsistence of the
prior union, which would have been binding were it not for its being bigamous.

Would the absolute nullity of either the first or the second marriage, prior to its judicial declaration as
being void, constitute a valid defense in a criminal action for bigamy?

I believe that, except for a void marriage on account of the psychological incapacity of a party or both
parties to the marriage under Article 36 of the Family Code (as so hereinafter explained), the answer
must be in the affirmative. Void marriages are inexistent from the very beginning, and no judicial decree
is required to establish their nullity. 2 As early as the case of People vs. Aragon, 3 this Court has
underscored the fact that the Revised Penal Code itself does not, unlike the rule then prevailing in Spain,
require the judicial declaration of nullity of a prior void marriage before it can be raised by way of a
defense in a criminal case for bigamy. Had the law contemplated otherwise, said the Court, "an express
provision to that effect would or should have been inserted in the law, (but that in) its absence, (the
courts) are bound by (the) rule of strict interpretation" of penal statutes. In contrast to a voidable
marriage which legally exists until judicially annulled (and, therefore, not a defense in a bigamy charge if
the second marriage were contracted prior to the decree of annulment), 4 the complete nullity,
however, of a previously contracted marriage, being void ab initio and legally inexistent, can outrightly
be a defense in an indictment for bigamy.

It has been held that, by virtue of Article 40 of the Family Code, a person may be convicted of bigamy
although the first marriage is ultimately adjudged void ab initio if, at the time the second marriage is

271
contracted, there has as yet been no judicial declaration of nullity of the prior marriage. 5 I maintain
strong reservations to this ruling. Article 40 of the Family Code reads:

"Article 40. The absolute nullity of a previous marriage may be invoked for purposes of remarriage
on the basis solely of a final judgment declaring such previous marriage void."

It is only "for purposes of remarriage" that the law has expressed that the absolute nullity of the
previous marriage may be invoked "on the basis solely of a final judgment declaring such previous
marriage void." It may not be amiss to state that under the regime of the Civil Code of 1950, the
Supreme Court, in Wiegel vs. Judge Sempio-Diy, 6 has held that a subsequent marriage of one of the
spouses of a prior void marriage is itself (the subsequent marriage) void if it were contracted before a
judicial declaration of nullity of the previous marriage. Although this pronouncement has been
abandoned in a later decision of the court in Yap vs. Court of Appeals, 7 the Family Code, however, has
seen it fit to adopt the Wiegel rule but only for purposes of remarriage which is just to say that the
subsequent marriage shall itself be considered void. There is no clear indication to conclude that the
Family Code has amended or intended to amend the Revised Penal Code or to abandon the settled and
prevailing jurisprudence on the matter. 8

A void marriage under Article 36 of the Family Code is a class by itself. The provision has been taken
from Canon Law primarily to reconcile the grounds for nullity of marriage under civil law with those of
church laws. 9 The "psychological incapacity to comply" with the essential marital obligations of the
spouses is completely distinct from other grounds for nullity which are confined to the essential or
formal requisites of a marriage, such as lack of legal capacity or disqualification of the contracting
parties, want of consent, absence of a marriage license, or the like.

The effects of a marriage attended by psychological incapacity of a party or the parties thereto may be
said to have the earmarks of a voidable, more than a void, marriage, remaining to be valid until it is
judicially decreed to be a nullity. Thus, Article 54 of the Family Code considers children conceived or
born of such a void marriage before its judicial declaration of nullity to be legitimate similar to the rule
on a voidable marriage. It is expected, even as I believe it safe to assume, that the spouses' rights and
obligations, property regime and successional rights would continue unaffected, as if it were a voidable
marriage, unless and until the marriage is judicially declared void for basically two reasons: First,
psychological incapacity, a newly-added ground for the nullity of a marriage under the Family Code,
breaches neither the essential nor the formal requisites of a valid marriage; 10 and second, unlike the
other grounds for nullity of marriage (i.e., relationship, minority of the parties, lack of license, mistake in
the identity of the parties) which are capable of relatively easy demonstration, psychological incapacity,
however, being a mental state, may not so readily be as evident. 11 It would have been logical for the
Family Code to consider such a marriage explicitly voidable rather than void if it were not for an
apparent attempt to make it closely coincide with the Canon Law rules and nomenclature.

Indeed, a void marriage due to psychological incapacity appears to merely differ from a voidable
marriage in that, unlike the latter, it is not convalidated by either cohabitation or prescription. It might
be recalled that prior to Republic Act No. 8533, further amending the Family Code, an action or defense

272
of absolute nullity of marriages falling under Article 36, celebrated before the effectivity of the Code,
could prescribe in ten years following the effectivity of the Family Code. The initial provision of the ten-
year period of prescription seems to betray a real consciousness by the framers that marriages falling
under Article 36 are truly meant to be inexistent.

Considerations, both logical and practical, would point to the fact that a "void" marriage due to
psychological incapacity remains, for all intents and purposes, to be binding and efficacious until
judicially declared otherwise. Without such marriage having first been declared a nullity (or otherwise
dissolved), a subsequent marriage could constitute bigamy. Thus, a civil case questioning the validity of
the first marriage would not be a prejudicial issue much in the same way that a civil case assailing a prior
"voidable" marriage (being valid until annulled) would not be a prejudicial question to the prosecution
of a criminal offense for bigamy.

In cases where the second marriage is void on grounds other than the existence of the first marriage,
this Court has declared in a line of cases that no crime of bigamy is committed. 12 The Court has
explained that for a person to be held guilty of bigamy, it must, even as it needs only, be shown that the
subsequent marriage has all the essential elements of a valid marriage, were it not for the subsisting
first union. Hence, where it is established that the second marriage has been contracted without the
necessary license and thus void, 13 or that the accused is merely forced to enter into the second
(voidable) marriage, 14 no criminal liability for the crime of bigamy can attach. In both and like
instances, however, the lapse refers to the elements required for contracting a valid marriage. If, then,
all the requisites for the perfection of the contract of marriage, freely and voluntarily entered into, are
shown to be extant, the criminal liability for bigamy can unassailably arise. aHIEcS

Since psychological incapacity, upon the other hand, does not relate to an infirmity in the elements,
either essential or formal, in contracting a valid marriage, the declaration of nullity subsequent to the
bigamous marriage due to that ground, without more, would be inconsequential in a criminal charge for
bigamy. The judicial declaration of nullity of a bigamous marriage on the ground of psychological
incapacity merely nullifies the effects of the marriage but it does not negate the fact of perfection of the
bigamous marriage. Its subsequent declaration of nullity dissolves the relationship of the spouses but,
being alien to the requisite conditions for the perfection of the marriage, the judgment of the court is no
defense on the part of the offender who has entered into it.

Accordingly, I vote to dismiss the petition.

CARPIO, J., dissenting:

I dissent from the decision of the majority, as expressed in the ponencia of Justice Consuelo Ynares-
Santiago. The majority opinion reverses a well-settled doctrine, established in a long line of decisions,
applying Article 349 of the Revised Penal Code. The reversal finds no support in the plain and ordinary
meaning of Article 349. The reversal also violates the constitutional guarantees of the accused and the
separation of powers.

The majority opinion makes the following ruling:

273
We hold that the subsequent judicial declaration of nullity of marriage on the ground of psychological
incapacity does not retroact to the date of celebration of the marriage insofar as the Philippines' penal
laws are concerned. As such, an individual who contracts a second or subsequent marriage during the
subsistence of a valid marriage is criminally liable for bigamy, notwithstanding the subsequent
declaration that the second marriage is void ab initio on the ground of psychological incapacity.

The issue may be stated thus: if the second marriage is void ab initio on grounds other than the
existence of the first marriage, such as psychological incapacity, is there a crime of bigamy?

In the present case, the prosecution filed the information for bigamy against the accused Veronico
Tenebro before the judicial declaration of nullity of his second marriage. However, before his conviction
for bigamy by the trial court, another court judicially declared his second marriage void ab initio because
of psychological incapacity.

The majority opinion is premised on two basic assertions. First, the mere act of entering into a second
marriage contract while the first marriage subsists consummates the crime of bigamy, even if the
second marriage is void ab initio on grounds other than the mere existence of the first marriage. Second,
a marriage declared by law void ab initio, and judicially confirmed void from the beginning, is deemed
valid for the purpose of a criminal prosecution for bigamy. I shall examine the correctness of these
assertions.

The majority opinion holds that the validity of the second marriage is immaterial and the mere act of
entering into a second marriage, even if void ab initio on grounds other than the existence of the first
marriage, consummates the crime of bigamy. Thus, the majority opinion states:

As a second or subsequent marriage contracted during the subsistence of petitioner's valid marriage to
Villareyes, petitioner's marriage to Ancajas would be null and void ab initio completely regardless of
petitioner's psychological capacity or incapacity. Since a marriage contracted during the subsistence of a
valid marriage is automatically void, the nullity of this second marriage is not per se an argument for the
avoidance of criminal liability for bigamy. Pertinently, Article 349 of the Revised Penal Code criminalizes
"any person who shall contract a second or subsequent marriage before the former marriage has been
legally dissolved, or before the absent spouse has been declared presumptively dead by means of a
judgment rendered in the proper proceedings". A plain reading of the law, therefore, would indicate
that the provision penalizes the mere act of contracting a second or a subsequent marriage during the
subsistence of a valid marriage.

Thus, as soon as the second marriage to Ancajas was celebrated on April 10, 1990, during the
subsistence of the valid first marriage, the crime of bigamy had already been consummated. To our
mind, there is no cogent reason for distinguishing between a subsequent marriage that is null and void
purely because it is a second or subsequent marriage, and a subsequent marriage that is null and void
on the ground of psychological incapacity, at least insofar as criminal liability for bigamy is concerned. . .
.. (Underscoring supplied; italics in the original)

274
The majority opinion concedes that the second marriage in the present case is void ab initio, even
without, need of judicial declaration. The majority expressly admits that the second marriage does not
legally exist, and thus in legal contemplation never took place at all. Nevertheless, the majority holds
that the second marriage is a marriage that exists in law sufficient to convict the accused of the crime of
bigamy.

The majority opinion holds that a judicial declaration of nullity of Tenebro's second marriage is
immaterial in a prosecution for the crime of bigamy. Such judicial declaration that the second marriage
is void from the beginning is absolutely of no moment.

Prior to appellant Tenebro's conviction by the trial court of the crime of bigamy, his second marriage
was in fact judicially declared void ab initio on the ground of psychological incapacity. Tenebro could
count in his favor not only an express provision of law declaring his second marriage void ab initio, he
also had a judicial confirmation of such nullity even prior to his conviction of bigamy by the trial court.
The majority opinion, however, simply brushes aside the law and the judicial confirmation. The majority
opinion holds that the fact that the second marriage is void ab initio on the ground of psychological
incapacity, and judicially declared as void from the very beginning, is immaterial in a bigamy charge.

For more than 75 years now, this Court has consistently ruled that if the second marriage is void on
grounds other than the existence of the first marriage, there is no crime of bigamy. The Court first
enunciated this doctrine in the 1935 case of People v. Mora Dumpo, 1 where the Court held:

Moro Hassan and Mora Dumpo have been legally married according to the rites and practices of the
Mohammedan religion. Without this marriage being dissolved, it is alleged that Dumpo contracted
another marriage with Moro Sabdapal after which the two lived together as husband and wife. Dumpo
was prosecuted for and convicted of the crime of bigamy in the Court of First Instance of Zamboanga
and sentenced to an indeterminate penalty with a maximum of eight years and one day of prision mayor
and a minimum of two years, four months and twenty-one days of prision correccional, with costs.
From this judgment the accused interposed an appeal. The records of the case disclose that it has been
established by the defense, without the prosecution having presented any objection nor evidence to the
contrary, that the alleged second marriage of the accused is null and void according to Mohammedan
rites on the ground that her father had not given his consent thereto.

xxx xxx xxx

It is an essential element of the crime of bigamy that the alleged second marriage, having all the
essential requisites, would be valid were it not for the subsistence of the first marriage. It appearing that
the marriage alleged to have been contracted by the accused with Sabdapal, her former marriage with
Hassan being undissolved, cannot be considered as such, there is no justification to hold her guilty of the
crime charged in the information. (Emphasis supplied)

In People v. Mendoza, 2 decided in 1954, the Court acquitted the accused of bigamy on the ground that
the first marriage was void having been contracted during the subsistence of a still earlier marriage. The
Court held:

275
The following facts are undisputed: On August 5, 1936, the appellant and Jovita de Asis were married in
Marikina, Rizal. On May 14, 1941, during the subsistence of the first marriage, the appellant was married
to Olga Lema in the City of Manila. On February 2, 1943, Jovita de Asis died. On August 19, 1949, the
appellant contracted another marriage with Carmencita Panlilio in Calamba, Laguna. This last marriage
gave rise to his prosecution for and conviction of the crime of bigamy.

The appellant contends that his marriage with Olga Lema on May 14, 1941 is null and void and,
therefore, non-existent, having been contracted while his first marriage with Jovita de Asis August 5,
1936 was still in effect, and that his third marriage to Carmencita Panlilio on August 19, 1949 cannot be
the basis of a charge for bigamy because it took place after the death of Jovita de Asis. The Solicitor
General, however, argues that, even assuming that appellant's second marriage to Olga Lema is void, he
is not exempt from criminal liability, in the absence of a previous judicial annulment of said bigamous
marriage; and the case of People vs. Cotas, 40 Off. Gaz., 3134, is cited.

xxx xxx xxx

In the case at bar, it is admitted that appellant's second marriage with Olga Lema was contracted during
the existence of his first marriage with Jovita de Asis. Section 29 of the marriage law (act 3613), in force
at the time the appellant contracted his second marriage in 1941, provides as follows:

Illegal marriages. — Any marriage subsequently contracted by any person during the lifetime of the first
spouse of such person, with any person other than such first spouse shall be illegal and void from its
performance, unless:

(a) The first marriage was annulled or dissolved;

(b) The first spouse had been absent for seven consecutive years at the time of the second marriage
without the spouse present having news of the absentee being alive, or the absentee being generally
considered as dead and believed to be so by the spouse present at the time of contracting such
subsequent marriage, the marriage so contracted being valid in either case until declared null and void
by a competent court.

This statutory provision plainly makes a subsequent marriage contracted by any person during the
lifetime of his first spouse illegal and void from its performance, and no judicial decree is necessary to
establish its invalidity, as distinguished from mere annullable marriages. There is here no pretense that
appellant's second marriage with Olga Lema was contracted in the belief that the first spouse, Jovita de
Asis, has been absent for seven consecutive years or generally considered as dead, so as to render said
marriage valid until declared null and void by a competent court.

Wherefore, the appealed judgment is reversed and the defendant-appellant acquitted, with costs de
oficio so ordered.

In People v. Lara, 3 decided in 1955, the Court acquitted the accused of bigamy on the ground that his
second marriage was void for lack of a marriage license. Declared the Court in Lara:

276
It is not disputed that the [accused] and Anacoreta Dalanida were married on July 1, 1947 . . .. Neither is
it denied that on August 18, 1951, while the marriage just referred to was subsisting, appellant entered
into a second marriage, this time with Josefa A. Rosales . . ..

In connection with the contract [for the second marriage], undisputed documentary evidence show that
. . . it was only on August 19, 1951, that the marriage license . . . was issued . . .. AECDHS

We are . . . of the opinion that the evidence in this case virtually beyond reasonable doubt that the
marriage license . . . was issued . . . on the date appearing thereon . . . namely, August 19, 1951.

xxx xxx xxx

Article 53 of the Civil Code of the Philippines, . . . which "no marriage shall be solemnized," one of them
being a marriage license duly issued at the time of the celebration of the marriage . . .. Related to this
point, Article 80(3) of the new Civil Code makes it clear that a marriage performed without the
corresponding marriage license is void, this being nothing more than the legitimate consequence
flowing from the fact that the license is the essence of the marriage contract.

Under the provisions of the Revised Penal Code there can be possible conviction for bigamy without
proof that the accused had voluntarily contracted a second marriage during the subsistence of his first
marriage with another person. Such was the interpretation given by the Court in People v. Mora Dumpo
that: "It is an essential element of the crime of bigamy that the alleged second marriage, having all the
essential requisites, would be valid were it not for the subsistence of the first marriage."

xxx xxx xxx

As to its validity, the marriage should be examined as of the time it was entered into. On that precise
date all the essential requisites must be present . . .. In the case before us, the evidence discloses that
the marriage preceded the issuance of the marriage license by one day. The subsequent issuance of the
license cannot in law, to our mind, render valid what in the eyes of the law itself was void from the
beginning . . .. (Emphasis supplied)

In the 1960 case of Merced v. Diez, 4 the Court held that a prior case for annulment of the second
marriage on the ground of vitiated consent constitutes a prejudicial question warranting the suspension
of the criminal case for bigamy. 5 The Court declared:

Before this Court the sole question raised is whether an action to annul the second marriage is a
prejudicial question in a prosecution for bigamy.

xxx xxx xxx

In order that a person may be held guilty of the crime of bigamy, the second and subsequent marriage
must have all the essential elements of a valid marriage, were it not for the subsistence of the first
marriage. This was the ruling of this Court in People vs. Dumpo, 62 Phil. 246, . . ..

277
One of the essential elements of a valid marriage is that the consent thereto of the contracting parties
must be freely and voluntarily given. Without the element of consent a marriage would be illegal and
void. (Section 29, Act No. 3613, otherwise known as the Marriage Law.) But the question of invalidity
cannot ordinarily be decided in the criminal action for bigamy but in a civil action for annulment. Since
the validity of the second marriage, subject of the action for bigamy, cannot be determined in the
criminal case and since prosecution for bigamy does not lie unless the elements of the second marriage
appear to exist, it is necessary that a decision in a civil action to the effect that the second marriage
contains all the essentials of a marriage must first be secured.

We have, therefore, in the case at bar, the issue of the validity of the second marriage, which must be
determined before hand in the civil action, before the criminal action can proceed. We have a situation
where the issue of the validity of the second marriage can be determined or must first be determined in
the civil action before the criminal action for bigamy can be prosecuted. The question of the validity of
the second marriage is, therefore, a prejudicial question, because determination of the validity of the
second marriage is determinable in the civil action and must precede the criminal action for bigamy.
(Emphasis supplied)

In Zapanta v. Montesa, 6 decided in 1962, the Court likewise suspended the proceedings in the criminal
case for bigamy because of a subsequent civil action filed by the accused to annul his second marriage
on the ground of vitiated consent. The Court ruled:

We have heretofore defined a prejudicial question as that which arises in a case, the resolution of which
is a logical antecedent of the issue involved therein, and the cognizance of which pertains to another
tribunal (People vs. Aragon, G.R. No. L-5930, February 17, 1954). The prejudicial question — we further
said — must be determinative of the case before the court, and jurisdiction to try the same must be
lodged in another court (People vs. Aragon, supra). These requisites are present in the case at bar.
Should the question for annulment of the second marriage pending in the Court of First Instance of
Pampanga prosper on the ground that, according to the evidence, petitioner's consent thereto was
obtained by means of duress, force and intimidation, it is obvious that his act was involuntary and can
not be the basis of his conviction for the crime of bigamy with which he was charged in the Court of First
Instance of Bulacan. Thus the issue involved in the action for the annulment of the second marriage is
determinative of petitioner's guilt or innocence of the crime of bigamy. On the other hand, there can be
no question that the annulment of petitioner's marriage with respondent Yco on the grounds relied
upon in the complaint filed in the Court of First Instance of Pampanga is within the jurisdiction of said
court.

In De la Cruz v. Ejercito, 7 decided in 1975, the Court, speaking through Justice Ramon C. Aquino,
dismissed a bigamy case against the accused in view of a final judgment the accused obtained annulling
her second marriage on the ground of vitiated consent. The Court, ruling that the annulment of the
second marriage rendered the criminal case "moot and untenable," explained:

The issue is whether the bigamy case became moot or untenable after the second marriage on which
the prosecution for bigamy is based, was annulled.

278
The City Fiscal of Angeles City contends that the lower court acted correctly in denying the motion to
dismiss the bigamy charge. He argues that the decision in the annulment case should be set up as a
defense by Milagros de la Cruz during the trial and that it would not justify the outright dismissal of the
criminal case.

On the other hand, the Solicitor General manifested that the stand of Milagros de la Cruz should be
sustained because one element of bigamy is that the alleged second marriage, having all the requisites,
would be valid were it not for the subsistence of the first marriage (People vs. Mora Dumpo, 62 Phil.
246, 248; Merced vs. Hon. Diez, 109 Phil. 155; Zapanta vs. Montesa, 114 Phil. 1227).

We hold that the finding in the annulment case that the second marriage contracted by Milagros de la
Cruz with Sergeant Gaccino was a nullity is determinative of her innocence and precludes the rendition
of a verdict that she committed bigamy. To try the criminal case in the face of such a finding would be
unwarranted. (Emphasis supplied)

These decisions of the Court declaring there is no crime of bigamy if the second marriage is void on
grounds other than the existence of the first marriage merely apply the clear language and intent of
Article 349 of the Revised Penal Code. This Article provides as follows:

Article 349. Bigamy. — The penalty of prision mayor shall be imposed upon any person who shall
contract a second or subsequent marriage before the former marriage has been legally dissolved, or
before the absent spouse has been declared presumptively dead by means of judgment rendered in the
proper proceedings.

Under Article 349 of the Revised Penal Code, the essential elements of the crime of bigamy are:

1. The offender is legally married;

2. The marriage is not legally dissolved;

3. The offender contracts a second or subsequent marriage;

4. The second or subsequent marriage is valid except for the existence of the first marriage.

The first three elements reiterate the language of the law. The last element, the validity of the second
marriage except for the existence of the first marriage, necessarily follows from the language of the law
that the offender contracts a "second or subsequent marriage."

If the second marriage is void ab initio on grounds other than the existence of the first marriage, then
legally there exists no second marriage. Article 35 of the Family Code enumerates the marriages that are
"void from the beginning." The succeeding article, Article 36, declares that a marriage contracted by one
psychologically incapacitated "shall likewise be void." Article 1409 of the Civil Code declares "inexistent
and void from the beginning" contracts "expressly . . . declared void by law." Thus, a marriage
contracted by one psychologically incapacitated at the time of the marriage is legally inexistent and void

279
from the beginning. Such void marriage cannot constitute a second marriage to sustain a conviction for
bigamy under Article 349 of the Revised Penal Code.

If the second marriage is void solely because of the existence of the first marriage, the nullity of the
second marriage proceeds from its illegality or bigamous nature. However, if the second marriage is void
on grounds other than the existence of the first marriage, the nullity does not proceed from its illegality
or bigamous nature. The first situation results in the crime of bigamy while the second does not. This is
clear from Article 1411 of the Civil Code which provides:

Article 1411. When the nullity proceeds from the illegality of the cause or object of the contract, and
the act constitutes a criminal act, both parties being in pari delicto, they shall have no action against
each other, and both shall be prosecuted. . . .

The rule shall be applicable when only one of the parties is guilty; . . . .

Thus, if the second marriage is void because of psychological incapacity, the nullity does not proceed
from an illegal or criminal cause, and no prosecution could ensue. However, if the second marriage is
void solely because of the existence of the first marriage, the nullity proceeds from an illegal or criminal
cause, and thus prosecution should follow.

The plain and ordinary meaning of Article 349 could only be that the second marriage must be valid
were it not for the existence of the first marriage. This has been the consistent interpretation of the
Court for more than seven decades since the enactment of the Revised Penal Code. Text writers in
criminal law have never entertained or advanced any other interpretation. There is no cogent reason to
depart from the well-established jurisprudence on Article 349 of the Revised Penal Code.

Even assuming, for the sake of argument, there is doubt on the interpretation of Article 349, substantive
due process of law requires a strict interpretation of Article 349 against the State and a liberal
interpretation in favor of the accused. The majority opinion reverses this principle and interprets Article
349 of the Revised Penal Code strictly against the accused and liberally in favor of the State.

Article 349 of the Revised Penal Code does not state that it is immaterial whether the second marriage is
valid or void ab initio. This Article does not also state that the mere act of celebration of the second
marriage, while the first marriage subsists, constitutes the crime of bigamy. Article 349 speaks of a
"second or subsequent marriage" which, as commonly understood and applied consistently by the
Court, means a valid second marriage were it not for the existence of the first marriage. EHTADa

To hold that the validity of the second marriage is immaterial, as the majority opinion so holds, would
interpret Article 349 too liberally in favor of the State and too strictly against the accused. This violates
the well-settled principle of statutory construction that the Court declared in People v. Garcia: 8

Criminal and penal statutes must be strictly construed, that is, they cannot be enlarged or extended by
intendment, implication, or by any equitable considerations. In other words, the language cannot be
enlarged beyond the ordinary meaning of its terms in order to carry into effect the general purpose for
which the statute was enacted. Only those persons, offenses, and penalties, clearly included, beyond

280
any reasonable doubt, will be considered within the statute's operation. They must come clearly within
both the spirit and the letter of the statute, and where there is any reasonable doubt, it must be
resolved in favor of the person accused of violating the statute; that is, all questions in doubt will be
resolved in favor of those from whom the penalty is sought. (Statutory Construction, Crawford, pp. 460-
462.)

The principle of statutory construction that penal laws are liberally construed in favor of the accused
and strictly against the State is deeply rooted in the need to protect constitutional guarantees. 9 This
principle serves notice to the public that only those acts clearly and plainly prohibited in penal laws are
subject to criminal sanctions. To expand penal laws beyond their clear and plain meaning is no longer
fair notice to the public. Thus, the principle insures observance of due process of law. The principle also
prevents discriminatory application of penal laws. State prosecutors have no power to broaden
arbitrarily the application of penal laws beyond the plain and common understanding of the people who
are subject to their penalties. Hence, the principle insures equal protection of the law.

The principle is also rooted in the need to maintain the separation of powers by insuring that the
legislature, and not the judiciary, defines crimes and prescribes their penalties. 10 As aptly stated by the
U.S. Supreme Court, speaking through Chief Justice John Marshall, in United. States v. Wiltberger: 11

The rule that penal laws are to be construed strictly, is perhaps not much less old than construction
itself. It is founded on the tenderness of the law for the rights of individuals; and on the plain principle
that the power of punishment is vested in the legislature, not in the judicial department. It is the
legislature, not the Court, which is to define a crime, and ordain its punishment. (Emphasis supplied)

This Court has specifically applied the rule on strict interpretation of a criminal statute to the crime of
bigamy. In People v. Aragon, 12 decided in 1957,. the Court ruled:

Appellant in this Court relies on the case, of People vs. Mendoza, (95 Phil., 845; 50 Off. Gaz., [10] 4767).
In this case the majority of this Court declared:

"The statutory provision (section 29 of the Marriage Law or Act No. 3613) plainly makes a subsequent
marriage contracted by any person during the lifetime of his first spouse illegal and void from its
performance, and no judicial decree is necessary to establish its invalidity, as distinguished from mere
annullable marriages. There is here no pretense that appellant's second marriage with Olga Lema was
contracted in the belief that the first spouse, Jovita de Asis, had been absent for seven consecutive years
or generally considered as dead, so as to render said marriage valid until declared null and void by a
subsequent court."

We are aware of the very weighty reasons expressed by Justice Alex Reyes in his dissent in the case
above-quoted. But these weighty reasons notwithstanding, the very fundamental principle of strict
construction of penal laws in favor of the accused, which principle we may not ignore, seems to justify
our stand in the above-cited case of People vs. Mendoza. Our Revised Penal Code is of recent enactment
and had the rule enunciated in Spain and in America requiring judicial declaration of nullity of ab initio
void marriages been within the contemplation of the legislature, an express provision to that effect

281
would or should have been inserted in the law. In its absence, we are bound by said rule of strict
interpretation already adverted to. (Emphasis supplied)

The majority opinion interprets Article 349 of the Revised Penal Code to mean that a second marriage,
even if void ab initio on grounds other than the existence of the first marriage, gives rise to the crime of
bigamy. This dissent interprets Article 349 to mean that for the crime of bigamy to exist, the second
marriage must be a valid marriage except for the existence of the first marriage. Otherwise, the
language of the law would mean nothing when it expressly declares certain marriages void ab initio or
void from the very beginning.

These opposing interpretations of a criminal statute call for the application of another well-established
rule that as between two reasonable interpretations, the more lenient one should be applied to penal
statutes. A leading English decision puts it in this wise:

If there is a reasonable interpretation which will avoid the penalty in any particular case, we must adopt
that construction. If there are two reasonable constructions, we must give the more lenient one. That is
the settled rule for construction of penal sections. 13

In summary, the majority opinion reverses the well-settled doctrine that there is no bigamy if the
second marriage is void on grounds other than the existence of the first marriage. The Court has
consistently applied this doctrine in several cases since 1935. The majority opinion reverses this doctrine
by disregarding the plain and ordinary meaning of the clear language of a criminal statute — Article 349
of the Revised Penal Code. The majority opinion then proceeds to interpret the criminal statute strictly
against the accused and liberally in favor of the State. The majority opinion makes this new
interpretation even as Article 349 has remained unchanged since its enactment into law on 1 January
1932. The majority opinion effectively amends the language of Article 349 of the Revised Penal Code in
violation of the separation of powers.

A final word. Even before appellant Tenebro's conviction of the crime of bigamy, he had already secured
a judicial declaration of nullity of his second marriage on the ground of psychological incapacity. This
judicial declaration merely confirmed what the law already explicitly provides — that a marriage
contracted by one psychologically incapacitated to marry is void from the very beginning and thus
legally inexistent. Inexplicably, the majority opinion still holds that the second marriage exists to warrant
Tenebro's conviction of the crime of bigamy.

Accordingly, I dissent from the majority opinion and vote to grant the petition.

CALLEJO, SR., J., dissenting:

I vote to grant pro hac vice the petition.

The prosecution was burdened to prove beyond reasonable doubt the corpus delicti, namely, all the
elements of the crime. 1 In this case, the prosecution adduced evidence that the petitioner contracted
marriage with Hilda and during the subsistence of said marriage, he contracted a second marriage with

282
the private respondent. However, the petitioner adduced in evidence the decision of the Regional Trial
Court in Civil Case No. AU-885 before the court a quo rendered judgment convicting the petitioner of
bigamy declaring null and void ab initio the petitioner's marriage with the private respondent on the
ground of the latter's psychological incapacity. Since the second marriage is null and void ab initio, such
marriage in contemplation of criminal law never existed and for that reason, one of the essential
elements of bigamy has disappeared. To quote Groizard:

. . . El matrimonio entonces, en realidad, no existe, pierde toda fuerza en virtud del vicio intrinseco que
lleva, y, por tanto, uno de los elementos del delito desaparece y la declaracion de inculpabilidad
procede. Esto que es logico y llano en el terreno de los principios, no puede, sin embargo, admitirse sin
ciertas restricciones en la practica. . . . 2

Whether or not the decision of the RTC declaring the second marriage null and void ab initio, is
erroneous is beside the point. Neither the private respondent nor the State, through the Office of the
Solicitor General, appealed the decision of the court. Entry of judgment was made of record before the
court a quo rendered its decision. Hence, both the State and the private respondent are bound by said
decision.

283
Montañez v. Cipriano, G.R. No. 181089, 22 October 2012

[G.R. No. 181089. October 22, 2012.]

MERLINDA CIPRIANO MONTAÑEZ, complainant, vs. LOURDES TAJOLOSA CIPRIANO, respondent.

DECISION

PERALTA, J p:

For our resolution is a petition for review on certiorari which seeks to annul the Order 1 dated
September 24, 2007 of the Regional Trial Court (RTC) of San Pedro, Laguna, Branch 31, issued in Criminal
Case No. 4990-SPL which dismissed the Information for Bigamy filed against respondent Lourdes
Tajolosa Cipriano. Also assailed is the RTC Resolution 2 dated January 2, 2008 denying the motion for
reconsideration. SHADEC

On April 8, 1976, respondent married Socrates Flores (Socrates) in Lezo, Aklan. 3 On January 24, 1983,
during the subsistence of the said marriage, respondent married Silverio V. Cipriano (Silverio) in San
Pedro, Laguna. 4 In 2001, respondent filed with the RTC of Muntinlupa, Branch 256, a Petition for the
Annulment of her marriage with Socrates on the ground of the latter's psychological incapacity as
defined under Article 36 of the Family Code, which was docketed as Civil Case No. 01-204. On July 18,
2003, the RTC of Muntinlupa, Branch 256, rendered an Amended Decision 5 declaring the marriage of
respondent with Socrates null and void. Said decision became final and executory on October 13, 2003.
6

On May 14, 2004, petitioner Merlinda Cipriano Montañez, Silverio's daughter from the first marriage,
filed with the Municipal Trial Court of San Pedro, Laguna, a Complaint 7 for Bigamy against respondent,
which was docketed as Criminal Case No. 41972. Attached to the complaint was an Affidavit 8
(Malayang Sinumpaang Salaysay) dated August 23, 2004, thumb-marked and signed by Silverio, 9 which
alleged, among others, that respondent failed to reveal to Silverio that she was still married to Socrates.
On November 17, 2004, an Information 10 for Bigamy was filed against respondent with the RTC of San
Pedro, Laguna, Branch 31. The case was docketed as Criminal Case No. 4990-SPL. The Information reads:

That on or about January 24, 1983, in the Municipality of San Pedro, Province of Laguna, Philippines, and
within the jurisdiction of this Honorable Court, the said accused did then and there willfully, unlawfully
and feloniously contract a second or subsequent marriage with one SILVERIO CIPRIANO VINALON while
her first marriage with SOCRATES FLORES has not been judicially dissolved by proper judicial authorities.
11

On July 24, 2007 and before her arraignment, respondent, through counsel, filed a Motion to Quash
Information (and Dismissal of the Criminal Complaint) 12 alleging that her marriage with Socrates had
already been declared void ab initio in 2003, thus, there was no more marriage to speak of prior to her
marriage to Silverio on January 24, 1983; that the basic element of the crime of bigamy, i.e., two valid
marriages, is therefore wanting. She also claimed that since the second marriage was held in 1983, the
crime of bigamy had already prescribed. The prosecution filed its Comment 13 arguing that the crime of

284
bigamy had already been consummated when respondent filed her petition for declaration of nullity;
that the law punishes the act of contracting a second marriage which appears to be valid, while the first
marriage is still subsisting and has not yet been annulled or declared void by the court. ITDSAE

In its Order 14 dated August 3, 2007, the RTC denied the motion. It found respondent's argument that
with the declaration of nullity of her first marriage, there was no more first marriage to speak of and
thus the element of two valid marriages in bigamy was absent, to have been laid to rest by our ruling in
Mercado v. Tan 15 where we held:

In the instant case, petitioner contracted a second marriage although there was yet no judicial
declaration of nullity of his first marriage. In fact, he instituted the Petition to have the first marriage
declared void only after complainant had filed a letter-complaint charging him with bigamy. For
contracting a second marriage while the first is still subsisting, he committed the acts punishable under
Article 349, of the Revised Penal Code.

That he subsequently obtained a judicial declaration of the nullity of the first marriage was immaterial.
To repeat, the crime had already been consummated by then. . . . 16

As to respondent's claim that the action had already prescribed, the RTC found that while the second
marriage indeed took place in 1983, or more than the 15-year prescriptive period for the crime of
bigamy, the commission of the crime was only discovered on November 17, 2004, which should be the
reckoning period, hence, prescription has not yet set in.

Respondent filed a Motion for Reconsideration 17 claiming that the Mercado ruling was not applicable,
since respondent contracted her first marriage in 1976, i.e., before the Family Code; that the petition for
annulment was granted and became final before the criminal complaint for bigamy was filed; and, that
Article 40 of the Family Code cannot be given any retroactive effect because this will impair her right to
remarry without need of securing a declaration of nullity of a completely void prior marriage.

On September 24, 2007, the RTC issued its assailed Order, 18 the dispositive portion of which reads:

Wherefore, the Order of August 3, 2007 is reconsidered and set aside. Let a new one be entered
quashing the information. Accordingly, let the instant case be DISMISSED. aDHCcE

SO ORDERED.

In so ruling, the RTC said that at the time the accused had contracted a second marriage on January 24,
1983, i.e., before the effectivity of the Family Code, the existing law did not require a judicial declaration
of absolute nullity as a condition precedent to contracting a subsequent marriage; that jurisprudence
before the Family Code was ambivalent on the issue of the need of prior judicial declaration of absolute
nullity of the first marriage. The RTC found that both marriages of respondent took place before the
effectivity of the Family Code, thus, considering the unsettled state of jurisprudence on the need for a
prior declaration of absolute nullity of marriage before commencing a second marriage and the principle
that laws should be interpreted liberally in favor of the accused, it declared that the absence of a judicial

285
declaration of nullity should not prejudice the accused whose second marriage was declared once and
for all valid with the annulment of her first marriage by the RTC of Muntinlupa City in 2003.

Dissatisfied, a Motion for Reconsideration was filed by the prosecution, but opposed by respondent. In a
Resolution dated January 2, 2008, the RTC denied the same ruling, among others, that the judicial
declaration of nullity of respondent's marriage is tantamount to a mere declaration or confirmation that
said marriage never existed at all, and for this reason, her act in contracting a second marriage cannot
be considered criminal.

Aggrieved, petitioner directly filed the present petition with us raising the following issues:

I. Whether the judicial nullity of a first marriage prior to the enactment of the Family Code and the
pronouncement in Wiegel vs. Sempio-Diy on the ground of psychological incapacity is a valid defense for
a charge of bigamy for entering into a second marriage prior to the enactment of the Family Code and
the pronouncement in Wiegel vs. Sempio-Diy?

II. Whether the trial court erred in stating that the jurisprudence prior to the enactment of the
Family Code and the pronouncement in Wiegel vs. Sempio-Diy regarding the necessity of securing a
declaration of nullity of the first marriage before entering a second marriage ambivalent, such that a
person was allowed to enter a subsequent marriage without the annulment of the first without incurring
criminal liability. 19 DEcTIS

Preliminarily, we note that the instant petition assailing the RTC's dismissal of the Information for
bigamy was filed by private complainant and not by the Office of the Solicitor General (OSG) which
should represent the government in all judicial proceedings filed before us. 20 Notwithstanding, we will
give due course to this petition as we had done in the past. In Antone v. Beronilla, 21 the offended party
(private complainant) questioned before the Court of Appeals (CA) the RTC's dismissal of the
Information for bigamy filed against her husband, and the CA dismissed the petition on the ground,
among others, that the petition should have been filed in behalf of the People of the Philippines by the
OSG, being its statutory counsel in all appealed criminal cases. In a petition filed with us, we said that we
had given due course to a number of actions even when the respective interests of the government
were not properly represented by the OSG and said:

In Labaro v. Panay, this Court dealt with a similar defect in the following manner:

It must, however, be stressed that if the public prosecution is aggrieved by any order ruling of the trial
judge in a criminal case, the OSG, and not the prosecutor, must be the one to question the order or
ruling before us. . . .

Nevertheless, since the challenged order affects the interest of the State or the plaintiff People of the
Philippines, we opted not to dismiss the petition on this technical ground. Instead, we required the OSG
to comment on the petition, as we had done before in some cases. In light of its Comment, we rule that
the OSG has ratified and adopted as its own the instant petition for the People of the Philippines.
(Emphasis supplied) 22

286
Considering that we also required the OSG to file a Comment on the petition, which it did, praying that
the petition be granted in effect, such Comment had ratified the petition filed with us.

As to the merit of the petition, the issue for resolution is whether or not the RTC erred in quashing the
Information for bigamy filed against respondent. AaEcHC

Article 349 of the Revised Penal Code defines and penalizes bigamy as follow:

Art. 349. Bigamy. — The penalty of prision mayor shall be imposed upon any person who shall
contract a second or subsequent marriage before the former marriage has been legally dissolved, or
before the absent spouse has been declared presumptively dead by means of a judgment rendered in
the proper proceedings.

The elements of the crime of bigamy are: (a) the offender has been legally married; (b) the marriage has
not been legally dissolved or, in case his or her spouse is absent, the absent spouse could not yet be
presumed dead according to the Civil Code; (c) that he contracts a second or subsequent marriage; and
(d) the second or subsequent marriage has all the essential requisites for validity. The felony is
consummated on the celebration of the second marriage or subsequent marriage. 23 It is essential in
the prosecution for bigamy that the alleged second marriage, having all the essential requirements,
would be valid were it not for the subsistence of the first marriage. 24

In this case, it appears that when respondent contracted a second marriage with Silverio in 1983, her
first marriage with Socrates celebrated in 1976 was still subsisting as the same had not yet been
annulled or declared void by a competent authority. Thus, all the elements of bigamy were alleged in
the Information. In her Motion to Quash the Information, she alleged, among others, that:

xxx xxx xxx

2. The records of this case would bear out that accused's marriage with said Socrates Flores was
declared void ab initio on 14 April 2003 by Branch 256 of the Regional Trial Court of Muntinlupa City.
The said decision was never appealed, and became final and executory shortly thereafter.

3. In other words, before the filing of the Information in this case, her marriage with Mr. Flores had
already been declared void from the beginning. ScCEIA

4. There was therefore no marriage prior to 24 January 1983 to speak of. In other words, there was
only one marriage.

5. The basic element of the crime of bigamy, that is, two valid marriages, is therefore wanting. 25

Clearly, the annulment of respondent's first marriage on the ground of psychological incapacity was
declared only in 2003. The question now is whether the declaration of nullity of respondent's first
marriage justifies the dismissal of the Information for bigamy filed against her.

We rule in the negative.

287
In Mercado v. Tan, 26 we ruled that the subsequent judicial declaration of the nullity of the first
marriage was immaterial, because prior to the declaration of nullity, the crime of bigamy had already
been consummated. And by contracting a second marriage while the first was still subsisting, the
accused committed the acts punishable under Article 349 of the Revised Penal Code.

In Abunado v. People, 27 we held that what is required for the charge of bigamy to prosper is that the
first marriage be subsisting at the time the second marriage is contracted. 28 Even if the accused
eventually obtained a declaration that his first marriage was void ab initio, the point is, both the first and
the second marriage were subsisting before the first marriage was annulled. 29

In Tenebro v. CA, 30 we declared that although the judicial declaration of the nullity of a marriage on the
ground of psychological incapacity retroacts to the date of the celebration of the marriage insofar as the
vinculum between the spouses is concerned, it is significant to note that said marriage is not without
legal effects. Among these effects is that children conceived or born before the judgment of absolute
nullity of the marriage shall be considered legitimate. There is, therefore, a recognition written into the
law itself that such a marriage, although void ab initio, may still produce legal consequences. Among
these legal consequences is incurring criminal liability for bigamy. To hold otherwise would render the
State's penal laws on bigamy completely nugatory, and allow individuals to deliberately ensure that each
marital contract be flawed in some manner, and to thus escape the consequences of contracting
multiple marriages, while beguiling throngs of hapless women with the promise of futurity and
commitment. 31 TAScID

And in Jarillo v. People, 32 applying the foregoing jurisprudence, we affirmed the accused's conviction
for bigamy, ruling that the moment the accused contracted a second marriage without the previous one
having been judicially declared null and void, the crime of bigamy was already consummated because at
the time of the celebration of the second marriage, the accused's first marriage which had not yet been
declared null and void by a court of competent jurisdiction was deemed valid and subsisting.

Here, at the time respondent contracted the second marriage, the first marriage was still subsisting as it
had not yet been legally dissolved. As ruled in the above-mentioned jurisprudence, the subsequent
judicial declaration of nullity of the first marriage would not change the fact that she contracted the
second marriage during the subsistence of the first marriage. Thus, respondent was properly charged of
the crime of bigamy, since the essential elements of the offense charged were sufficiently alleged.

Respondent claims that Tenebro v. CA 33 is not applicable, since the declaration of nullity of the
previous marriage came after the filing of the Information, unlike in this case where the declaration was
rendered before the information was filed. We do not agree. What makes a person criminally liable for
bigamy is when he contracts a second or subsequent marriage during the subsistence of a valid
marriage.

Parties to the marriage should not be permitted to judge for themselves its nullity, for the same must be
submitted to the judgment of competent courts and only when the nullity of the marriage is so declared
can it be held as void, and so long as there is no such declaration the presumption is that the marriage

288
exists. 34 Therefore, he who contracts a second marriage before the judicial declaration of nullity of the
first marriage assumes the risk of being prosecuted for bigamy. 35

Anent respondent's contention in her Comment that since her two marriages were contracted prior to
the effectivity of the Family Code, Article 40 of the Family Code cannot be given retroactive effect
because this will impair her right to remarry without need of securing a judicial declaration of nullity of a
completely void marriage. SIAEHC

We are not persuaded.


In Jarillo v. People, 36 where the accused, in her motion for reconsideration, argued that since her
marriages were entered into before the effectivity of the Family Code, then the applicable law is Section
29 of the Marriage Law (Act 3613), 37 instead of Article 40 of the Family Code, which requires a final
judgment declaring the previous marriage void before a person may contract a subsequent marriage.
We did not find the argument meritorious and said:

As far back as 1995, in Atienza v. Brillantes, Jr., the Court already made the declaration that Article 40,
which is a rule of procedure, should be applied retroactively because Article 256 of the Family Code
itself provides that said "Code shall have retroactive effect insofar as it does not prejudice or impair
vested or acquired rights." The Court went on to explain, thus:

The fact that procedural statutes may somehow affect the litigants' rights may not preclude their
retroactive application to pending actions. The retroactive application of procedural laws is not violative
of any right of a person who may feel that he is adversely affected. The reason is that as a general rule,
no vested right may attach to, nor arise from, procedural laws.

In Marbella-Bobis v. Bobis, the Court pointed out the danger of not enforcing the provisions of Article 40
of the Family Code, to wit:
In the case at bar, respondent's clear intent is to obtain a judicial declaration of nullity of his first
marriage and thereafter to invoke that very same judgment to prevent his prosecution for bigamy. He
cannot have his cake and eat it too. Otherwise, all that an adventurous bigamist has to do is disregard
Article 40 of the Family Code, contract a subsequent marriage and escape a bigamy charge by simply
claiming that the first marriage is void and that the subsequent marriage is equally void for lack of a
prior judicial declaration of nullity of the first. A party may even enter into a marriage aware of the
absence of a requisite — usually the marriage license — and thereafter contract a subsequent marriage
without obtaining a declaration of nullity of the first on the assumption that the first marriage is void.
Such scenario would render nugatory the provision on bigamy. 38 AHcaDC
WHEREFORE, considering the foregoing, the petition is GRANTED. The Order dated September 24, 2007
and the Resolution dated January 2, 2008 of the Regional Trial Court of San Pedro, Laguna, Branch 31,
issued in Criminal Case No. 4990-SPL, are hereby SET ASIDE. Criminal Case No. 4990-SPL is ordered
REMANDED to the trial court for further proceedings.

SO ORDERED.

Velasco, Jr., Leonardo-de Castro, * Abad and Mendoza, JJ., concur.

289
San Miguel Proeprties, Inc., v. Perez, G.R. No. 166836, 4 September 2013

[G.R. No. 166836. September 4, 2013.]

SAN MIGUEL PROPERTIES, INC., petitioner, vs. SEC. HERNANDO B. PEREZ, ALBERT C. AGUIRRE, TEODORO
B. ARCENAS, JR., MAXY S. ABAD, JAMES G. BARBERS, STEPHEN N. SARINO, ENRIQUE N. ZALAMEA, JR.,
MARIANO M. MARTIN, ORLANDO O. SAMSON, CATHERINE R. AGUIRRE, and ANTONIO V. AGCAOILI,
respondents.

DECISION

BERSAMIN, J p:

The pendency of an administrative case for specific performance brought by the buyer of residential
subdivision lots in the Housing and Land Use Regulatory Board (HLURB) to compel the seller to deliver
the transfer certificates of title (TCTs) of the fully paid lots is properly considered a ground to suspend a
criminal prosecution for violation of Section 25 of Presidential Decree No. 957 1 on the ground of a
prejudicial question. The administrative determination is a logical antecedent of the resolution of the
criminal charges based on non-delivery of the TCTs. THIASE

Antecedents

Petitioner San Miguel Properties, Inc. (San Miguel Properties), a domestic corporation engaged in the
real estate business, purchased in 1992, 1993 and April 1993 from B.F. Homes, Inc. (BF Homes), then
represented by Atty. Florencio B. Orendain (Orendain) as its duly authorized rehabilitation receiver
appointed by the Securities and Exchange Commission (SEC), 2 130 residential lots situated in its
subdivision BF Homes Parañaque, containing a total area of 44,345 square meters for the aggregate
price of P106,248,000.00. The transactions were embodied in three separate deeds of sale. 3 The TCTs
covering the lots bought under the first and second deeds were fully delivered to San Miguel Properties,
but 20 TCTs covering 20 of the 41 parcels of land with a total area of 15,565 square meters purchased
under the third deed of sale, executed in April 1993 and for which San Miguel Properties paid the full
price of P39,122,627.00, were not delivered to San Miguel Properties. EHTIDA

On its part, BF Homes claimed that it withheld the delivery of the 20 TCTs for parcels of land purchased
under the third deed of sale because Atty. Orendain had ceased to be its rehabilitation receiver at the
time of the transactions after being meanwhile replaced as receiver by FBO Network Management, Inc.
on May 17, 1989 pursuant to an order from the SEC. 4

BF Homes refused to deliver the 20 TCTs despite demands. Thus, on August 15, 2000, San Miguel
Properties filed a complaint-affidavit in the Office of the City Prosecutor of Las Piñas City (OCP Las Piñas)
charging respondent directors and officers of BF Homes with non-delivery of titles in violation of Section
25, in relation to Section 39, both of Presidential Decree No. 957 (I.S. No. 00-2256). 5 ESTCDA

At the same time, San Miguel Properties sued BF Homes for specific performance in the HLURB (HLURB
Case No. REM-082400-11183), 6 praying to compel BF Homes to release the 20 TCTs in its favor.

290
In their joint counter-affidavit submitted in I.S. No. 00-2256, 7 respondent directors and officers of BF
Homes refuted San Miguel Properties' assertions by contending that: (a) San Miguel Properties' claim
was not legally demandable because Atty. Orendain did not have the authority to sell the 130 lots in
1992 and 1993 due to his having been replaced as BF Homes' rehabilitation receiver by the SEC on May
17, 1989; (b) the deeds of sale conveying the lots were irregular for being undated and unnotarized; (c)
the claim should have been brought to the SEC because BF Homes was under receivership; (d) in
receivership cases, it was essential to suspend all claims against a distressed corporation in order to
enable the receiver to effectively exercise its powers free from judicial and extra-judicial interference
that could unduly hinder the rescue of the distressed company; and (e) the lots involved were under
custodia legis in view of the pending receivership proceedings, necessarily stripping the OCP Las Piñas of
the jurisdiction to proceed in the action. aHcDEC

On October 10, 2000, San Miguel Properties filed a motion to suspend proceedings in the OCP Las Piñas,
8 citing the pendency of BF Homes' receivership case in the SEC. In its comment/opposition, BF Homes
opposed the motion to suspend. In the meantime, however, the SEC terminated BF Homes' receivership
on September 12, 2000, prompting San Miguel Properties to file on October 27, 2000 a reply to BF
Homes' comment/opposition coupled with a motion to withdraw the sought suspension of proceedings
due to the intervening termination of the receivership. 9

On October 23, 2000, the OCP Las Piñas rendered its resolution, 10 dismissing San Miguel Properties'
criminal complaint for violation of Presidential Decree No. 957 on the ground that no action could be
filed by or against a receiver without leave from the SEC that had appointed him; that the
implementation of the provisions of Presidential Decree No. 957 exclusively pertained under the
jurisdiction of the HLURB; that there existed a prejudicial question necessitating the suspension of the
criminal action until after the issue on the liability of the distressed BF Homes was first determined by
the SEC en banc or by the HLURB; and that no prior resort to administrative jurisdiction had been made;
that there appeared to be no probable cause to indict respondents for not being the actual signatories
in the three deeds of sale. AICHaS

On February 20, 2001, the OCP Las Piñas denied San Miguel Properties' motion for reconsideration filed
on November 28, 2000, holding that BF Homes' directors and officers could not be held liable for the
non-delivery of the TCTs under Presidential Decree No. 957 without a definite ruling on the legality of
Atty. Orendain's actions; and that the criminal liability would attach only after BF Homes did not comply
with a directive of the HLURB directing it to deliver the titles. 11

San Miguel Properties appealed the resolutions of the OCP Las Piñas to the Department of Justice (DOJ),
but the DOJ Secretary denied the appeal on October 15, 2001, holding:

After a careful review of the evidence on record, we find no cogent reason to disturb the ruling of the
City Prosecutor of Las Piñas City. Established jurisprudence supports the position taken by the City
Prosecutor concerned. HTAIcD

There is no dispute that aside from the instant complaint for violation of PD 957, there is still pending
with the Housing and Land Use Regulatory Board (HLURB, for short) a complaint for specific

291
performance where the HLURB is called upon to inquire into, and rule on, the validity of the sales
transactions involving the lots in question and entered into by Atty. Orendain for and in behalf of BF
Homes.

As early as in the case of Solid Homes, Inc. vs. Payawal, 177 SCRA 72, the Supreme Court had ruled that
the HLURB has exclusive jurisdiction over cases involving real estate business and practices under PD
957. This is reiterated in the subsequent cases of Union Bank of the Philippines versus HLURB, G.R. [No.]
953364, June 29, 1992 and C.T. Torres Enterprises vs. Hilionada, 191 SCRA 286. aESTAI

The said ruling simply means that unless and until the HLURB rules on the validity of the transactions
involving the lands in question with specific reference to the capacity of Atty. Orendain to bind BF
Homes in the said transactions, there is as yet no basis to charge criminally respondents for non-delivery
of the subject land titles. In other words, complainant cannot invoke the penal provision of PD 957 until
such time that the HLURB shall have ruled and decided on the validity of the transactions involving the
lots in question.

WHEREFORE, the appeal is hereby DENIED.

SO ORDERED. 12 (Emphasis supplied) AIDTSE

The DOJ eventually denied San Miguel Properties' motion for reconsideration. 13

Ruling of the CA

Undaunted, San Miguel Properties elevated the DOJ's resolutions to the CA on certiorari and mandamus
(C.A.-G.R. SP No. 73008), contending that respondent DOJ Secretary had acted with grave abuse in
denying their appeal and in refusing to charge the directors and officers of BF Homes with the violation
of Presidential Decree No. 957. San Miguel Properties submitted the issue of whether or not HLURB
Case No. REM-082400-11183 presented a prejudicial question that called for the suspension of the
criminal action for violation of Presidential Decree No. 957. ATHCac

In its assailed decision promulgated on February 24, 2004 in C.A.-G.R. SP No. 73008, 14 the CA dismissed
San Miguel Properties' petition, holding and ruling as follows:

From the foregoing, the conclusion that may be drawn is that the rule on prejudicial question generally
applies to civil and criminal actions only.

However, an exception to this rule is provided in Quiambao vs. Osorio cited by the respondents. In this
case, an issue in an administrative case was considered a prejudicial question to the resolution of a civil
case which, consequently, warranted the suspension of the latter until after termination of the
administrative proceedings. ADaSET

Quiambao vs. Osorio is not the only instance when the Supreme Court relaxed the application of the
rule on prejudicial question.

292
In Tamin vs. CA involving two (2) civil actions, the Highest Court similarly applied the rule on prejudicial
question when it directed petitioner therein to put up a bond for just compensation should the
demolition of private respondents' building proved to be illegal as a result of a pending cadastral suit in
another tribunal.

City of Pasig vs. COMELEC is yet another exception where a civil action involving a boundary dispute was
considered a prejudicial question which must be resolved prior to an administrative proceeding for the
holding of a plebiscite on the affected areas. cETDIA

In fact, in Vidad vs. RTC of Negros Oriental, Br. 42, it was ruled that in the interest of good order, courts
can suspend action in one case pending determination of another case closely interrelated or interliked
with it.

It thus appears that public respondent did not act with grave abuse of discretion . . . when he applied
the rule on prejudicial question to the instant proceedings considering that the issue on the validity of
the sale transactions . . . by . . . Orendain in behalf of BF Homes, Inc., is closely intertwined with the
purported criminal culpability of private respondents, as officers/directors of BF Homes, Inc., arising
from their failure to deliver the titles of the parcels of land included in the questioned conveyance.

All told, to sustain the petitioner's theory that the result of the HLURB proceedings is not determinative
of the criminal liability of private respondents under PD 957 would be to espouse an absurdity. If we
were to assume that the HLURB finds BFHI under no obligation to delve the subject titles, it would be
highly irregular and contrary to the ends of justice to pursue a criminal case against private respondents
for the non-delivery of certificates of title which they are not under any legal obligation to turn over in
the first place. (Bold emphasis supplied) TDCAHE

On a final note, absent grave abuse of discretion on the part of the prosecutorial arm of the government
as represented by herein public respondent, courts will not interfere with the discretion of a public
prosecutor in prosecuting or dismissing a complaint filed before him. A public prosecutor, by the nature
of his office, is under no compulsion to file a criminal information where no clear legal justification has
been shown, and no sufficient evidence of guilt nor prima facie case has been established by the
complaining party.

WHEREFORE, premises considered, the instant Petition for Certiorari and Mandamus is hereby DENIED.
The Resolutions dated 15 October 2001 and 12 July 2002 of the Department of Justice are AFFIRMED.
EcIaTA

SO ORDERED. 15

The CA denied San Miguel Properties' motion for reconsideration on January 18, 2005. 16

Issues

Aggrieved, San Miguel Properties is now on appeal, raising the following for consideration and
resolution, to wit:

293
THE COURT OF APPEALS COMMITTED GRAVE, SERIOUS AND REVERSIBLE ERRORS WHEN IT DISMISSED
PETITIONER'S CERTIORARI AND MANDAMUS PETITION TO ORDER AND DIRECT RESPONDENT SECRETARY
TO INDICT RESPONDENTS FOR VIOLATION OF SECTION 25, P.D. 957 IN THAT: SEHTAC

1. THE OBLIGATION OF PRIVATE RESPONDENTS TO DELIVER TO PETITIONER THE TITLES TO 20


FULLY-PAID LOTS IS MANDATED BY SECTION 25, PD 957. IN FACT, THE OFFICE OF THE PRESIDENT HAD
DULY CONFIRMED THE SAME PER ITS DECISION DATED 27 JANUARY 2005 IN O.P. CASE NO. 03-E-203,
ENTITLED "SMPI V. BF HOMES, INC.".

2. A FORTIORI, PRIVATE RESPONDENTS' FAILURE AND/OR REFUSAL TO DELIVER TO PETITIONER


THE SUBJECT TITLES CONSTITUTES CRIMINAL OFFENSE PER SECTIONS 25 AND 39, PD 957 FOR WHICH IT
IS THE MINISTERIAL DUTY OF RESPONDENT SECRETARY TO INDICT PRIVATE RESPONDENTS THEREFOR.
AEIHCS

3. IN ANY EVENT, THE HLURB CASE DOES NOT PRESENT A "PREJUDICIAL QUESTION" TO THE
SUBJECT CRIMINAL CASE SINCE THE FORMER INVOLVES AN ISSUE SEPARATE AND DISTINCT FROM THE
ISSUE INVOLVED IN THE LATTER. CONSEQUENTLY, THE HLURB CASE HAS NO CORRELATION, TIE NOR
LINKAGE TO THE PRESENT CRIMINAL CASE WHICH CAN PROCEED INDEPENDENTLY THEREOF.

4. IN FACT, THE CRIMINAL CULPABILITY OF PRIVATE RESPONDENTS EMANATE FROM THEIR MALA
PROHIBITA NON-DELIVERY OF THE TITLES TO TWENTY (20) FULLY-PAID PARCELS OF LAND TO
PETITIONER, AND NOT FROM THEIR NON-COMPLIANCE WITH THE HLURB'S RULING IN THE
ADMINISTRATIVE CASE. IDCcEa

5. NONETHELESS, BY DECREEING THAT PETITIONER'S CRIMINAL COMPLAINT IS PREMATURE, BOTH


THE COURT OF APPEALS AND RESPONDENT SECRETARY HAD IMPLIEDLY ADMITTED THE EXISTENCE OF
SUFFICIENT PROBABLE CAUSE AGAINST PRIVATE RESPONDENTS FOR THE CRIME CHARGED. 17

It is relevant at this juncture to mention the outcome of the action for specific performance and
damages that San Miguel Properties instituted in the HLURB simultaneously with its filing of the
complaint for violation of Presidential Decree No. 957. On January 25, 2002, the HLURB Arbiter ruled
that the HLURB was inclined to suspend the proceedings until the SEC resolved the issue of Atty.
Orendain's authority to enter into the transactions in BF Homes' behalf, because the final resolution by
the SEC was a logical antecedent to the determination of the issue involved in the complaint before the
HLURB. Upon appeal, the HLURB Board of Commissioners (HLURB Board), citing the doctrine of primary
jurisdiction, affirmed the HLURB Arbiter's decision, holding that although no prejudicial question could
arise, strictly speaking, if one case was civil and the other administrative, it nonetheless opted to
suspend its action on the cases pending the final outcome of the administrative proceeding in the
interest of good order. 18 HCEcaT

Not content with the outcome, San Miguel Properties appealed to the Office of the President (OP),
arguing that the HLURB erred in suspending the proceedings. On January 27, 2004, the OP reversed the
HLURB Board's ruling, holding thusly:

294
The basic complaint in this case is one for specific performance under Section 25 of the Presidential
Decree (PD) 957 — "The Subdivision and Condominium Buyers' Protective."

As early as August 1987, the Supreme Court already recognized the authority of the HLURB, as successor
agency of the National Housing Authority (NHA), to regulate, pursuant to PD 957, in relation to PD 1344,
the real estate trade, with exclusive original jurisdiction to hear and decide cases "involving specific
performance of contractual and statutory obligation filed by buyers of subdivision lots . . . against the
owner, developer, dealer, broker or salesman," the HLURB, in the exercise of its adjudicatory powers
and functions, "must interpret and apply contracts, determine the rights of the parties under these
contracts and award[s] damages whenever appropriate."

Given its clear statutory mandate, the HLURB's decision to await for some forum to decide — if ever one
is forthcoming — the issue on the authority of Orendain to dispose of subject lots before it peremptorily
resolves the basic complaint is unwarranted, the issues thereon having been joined and the respective
position papers and the evidence of the parties having been submitted. To us, it behooved the HLURB to
adjudicate, with the usual dispatch, the right and obligation of the parties in line with its own
appreciation of the obtaining facts and applicable law. To borrow from Mabubha Textile Mills
Corporation vs. Ongpin, it does not have to rely on the finding of others to discharge this adjudicatory
functions. 19 CHEDAc

After its motion for reconsideration was denied, BF Homes appealed to the CA (C.A.-G.R. SP No. 83631),
raising as issues: (a) whether or not the HLURB had the jurisdiction to decide with finality the question of
Atty. Orendain's authority to enter into the transaction with San Miguel Properties in BF Homes' behalf,
and rule on the rights and obligations of the parties to the contract; and (b) whether or not the HLURB
properly suspended the proceedings until the SEC resolved with finality the matter regarding such
authority of Atty. Orendain.

The CA promulgated its decision in C.A.-G.R. SP No. 83631, 20 decreeing that the HLURB, not the SEC,
had jurisdiction over San Miguel Properties' complaint. It affirmed the OP's decision and ordered the
remand of the case to the HLURB for further proceedings on the ground that the case involved matters
within the HLURB's competence and expertise pursuant to the doctrine of primary jurisdiction, viz.:
IDTSaC

[T]he High Court has consistently ruled that the NHA or the HLURB has jurisdiction over complaints
arising from contracts between the subdivision developer and the lot buyer or those aimed at
compelling the subdivision developer to comply with its contractual and statutory obligations.

Hence, the HLURB should take jurisdiction over respondent's complaint because it pertains to matters
within the HLURB's competence and expertise. The proceedings before the HLURB should not be
suspended.

While We sustain the Office of the President, the case must be remanded to the HLURB. This is in
recognition of the doctrine of primary jurisdiction. The fairest and most equitable course to take under

295
the circumstances is to remand the case to the HLURB for the proper presentation of evidence. 21
HIESTA

Did the Secretary of Justice commit grave abuse of discretion in upholding the dismissal of San Miguel
Properties' criminal complaint for violation of Presidential Decree No. 957 for lack of probable cause and
for reason of a prejudicial question?

The question boils down to whether the HLURB administrative case brought to compel the delivery of
the TCTs could be a reason to suspend the proceedings on the criminal complaint for the violation of
Section 25 of Presidential Decree No. 957 on the ground of a prejudicial question.

Ruling of the Court

The petition has no merit. EAcIST

1.

Action for specific performance, even if pending

in the HLURB, an administrative agency,

raises a prejudicial question

BF Homes' posture that the administrative case for specific performance in the HLURB posed a
prejudicial question that must first be determined before the criminal case for violation of Section 25 of
Presidential Decree No. 957 could be resolved is correct.

A prejudicial question is understood in law to be that which arises in a case the resolution of which is a
logical antecedent of the issue involved in the criminal case, and the cognizance of which pertains to
another tribunal. It is determinative of the criminal case, but the jurisdiction to try and resolve it is
lodged in another court or tribunal. It is based on a fact distinct and separate from the crime but is so
intimately connected with the crime that it determines the guilt or innocence of the accused. 22 The
rationale behind the principle of prejudicial question is to avoid conflicting decisions. 23 The essential
elements of a prejudicial question are provided in Section 7, Rule 111 of the Rules of Court, to wit: (a)
the previously instituted civil action involves an issue similar or intimately related to the issue raised in
the subsequent criminal action, and (b) the resolution of such issue determines whether or not the
criminal action may proceed. AHaDSI

The concept of a prejudicial question involves a civil action and a criminal case. Yet, contrary to San
Miguel Properties' submission that there could be no prejudicial question to speak of because no civil
action where the prejudicial question arose was pending, the action for specific performance in the
HLURB raises a prejudicial question that sufficed to suspend the proceedings determining the charge for
the criminal violation of Section 25 24 of Presidential Decree No. 957. This is true simply because the
action for specific performance was an action civil in nature but could not be instituted elsewhere
except in the HLURB, whose jurisdiction over the action was exclusive and original. 25

296
The determination of whether the proceedings ought to be suspended because of a prejudicial question
rested on whether the facts and issues raised in the pleadings in the specific performance case were so
related with the issues raised in the criminal complaint for the violation of Presidential Decree No. 957,
such that the resolution of the issues in the former would be determinative of the question of guilt in
the criminal case. An examination of the nature of the two cases involved is thus necessary. DAESTI

An action for specific performance is the remedy to demand the exact performance of a contract in the
specific form in which it was made, or according to the precise terms agreed upon by a party bound to
fulfill it. 26 Evidently, before the remedy of specific performance is availed of, there must first be a
breach of the contract. 27 The remedy has its roots in Article 1191 of the Civil Code, which reads:

Article 1191. The power to rescind obligations is implied in reciprocal ones, in case one of the obligors
should not comply with what is incumbent upon him.

The injured party may choose between the fulfillment and the rescission of the obligation, with the
payment of damages in either case. He may also seek rescission, even after he has chosen fulfillment, if
the latter should become impossible. . . . (Emphasis supplied) SIcCEA

Accordingly, the injured party may choose between specific performance or rescission with damages. As
presently worded, Article 1191 speaks of the remedy of rescission in reciprocal obligations within the
context of Article 1124 of the former Civil Code which used the term resolution. The remedy of
resolution applied only to reciprocal obligations, such that a party's breach of the contract equated to a
tacit resolutory condition that entitled the injured party to rescission. The present article, as in the
former one, contemplates alternative remedies for the injured party who is granted the option to
pursue, as principal actions, either the rescission or the specific performance of the obligation, with
payment of damages in either case. 28

On the other hand, Presidential Decree No. 957 is a law that regulates the sale of subdivision lots and
condominiums in view of the increasing number of incidents wherein "real estate subdivision owners,
developers, operators, and/or sellers have reneged on their representations and obligations to provide
and maintain properly" the basic requirements and amenities, as well as of reports of alarming
magnitude of swindling and fraudulent manipulations perpetrated by unscrupulous subdivision and
condominium sellers and operators, 29 such as failure to deliver titles to the buyers or titles free from
liens and encumbrances. Presidential Decree No. 957 authorizes the suspension and revocation of the
registration and license of the real estate subdivision owners, developers, operators, and/or sellers in
certain instances, as well as provides the procedure to be observed in such instances; it prescribes
administrative fines and other penalties in case of violation of, or non-compliance with its provisions.
EaIcAS

Conformably with the foregoing, the action for specific performance in the HLURB would determine
whether or not San Miguel Properties was legally entitled to demand the delivery of the remaining 20
TCTs, while the criminal action would decide whether or not BF Homes' directors and officers were
criminally liable for withholding the 20 TCTs. The resolution of the former must obviously precede that
of the latter, for should the HLURB hold San Miguel Properties to be not entitled to the delivery of the

297
20 TCTs because Atty. Orendain did not have the authority to represent BF Homes in the sale due to his
receivership having been terminated by the SEC, the basis for the criminal liability for the violation of
Section 25 of Presidential Decree No. 957 would evaporate, thereby negating the need to proceed with
the criminal case.

Worthy to note at this juncture is that a prejudicial question need not conclusively resolve the guilt or
innocence of the accused. It is enough for the prejudicial question to simply test the sufficiency of the
allegations in the information in order to sustain the further prosecution of the criminal case. A party
who raises a prejudicial question is deemed to have hypothetically admitted that all the essential
elements of the crime have been adequately alleged in the information, considering that the
Prosecution has not yet presented a single piece of evidence on the indictment or may not have rested
its case. A challenge to the allegations in the information on the ground of prejudicial question is in
effect a question on the merits of the criminal charge through a non-criminal suit. 30 HDaACI

2.

Doctrine of primary jurisdiction is applicable

That the action for specific performance was an administrative case pending in the HLURB, instead of in
a court of law, was of no consequence at all. As earlier mentioned, the action for specific performance,
although civil in nature, could be brought only in the HLURB. This situation conforms to the doctrine of
primary jurisdiction. There has been of late a proliferation of administrative agencies, mostly regulatory
in function. It is in favor of these agencies that the doctrine of primary jurisdiction is frequently invoked,
not to defeat the resort to the judicial adjudication of controversies but to rely on the expertise,
specialized skills, and knowledge of such agencies in their resolution. The Court has observed that one
thrust of the proliferation is that the interpretation of contracts and the determination of private rights
under contracts are no longer a uniquely judicial function exercisable only by the regular courts. 31
cHaICD

The doctrine of primary jurisdiction has been increasingly called into play on matters demanding the
special competence of administrative agencies even if such matters are at the same time within the
jurisdiction of the courts. A case that requires for its determination the expertise, specialized skills, and
knowledge of some administrative board or commission because it involves technical matters or
intricate questions of fact, relief must first be obtained in an appropriate administrative proceeding
before a remedy will be supplied by the courts although the matter comes within the jurisdiction of the
courts. The application of the doctrine does not call for the dismissal of the case in the court but only for
its suspension until after the matters within the competence of the administrative body are threshed
out and determined. 32 DaCEIc

To accord with the doctrine of primary jurisdiction, the courts cannot and will not determine a
controversy involving a question within the competence of an administrative tribunal, the controversy
having been so placed within the special competence of the administrative tribunal under a regulatory
scheme. In that instance, the judicial process is suspended pending referral to the administrative body
for its view on the matter in dispute. Consequently, if the courts cannot resolve a question that is within

298
the legal competence of an administrative body prior to the resolution of that question by the latter,
especially where the question demands the exercise of sound administrative discretion requiring the
special knowledge, experience, and services of the administrative agency to ascertain technical and
intricate matters of fact, and a uniformity of ruling is essential to comply with the purposes of the
regulatory statute administered, suspension or dismissal of the action is proper. 33 AECDHS

3.

Other submissions of petitioner are unwarranted

It is not tenable for San Miguel Properties to argue that the character of a violation of Section 25 of
Presidential Decree No. 957 as malum prohibitum, by which criminal liability attached to BF Homes'
directors and officers by the mere failure to deliver the TCTs, already rendered the suspension
unsustainable. 34 The mere fact that an act or omission was malum prohibitum did not do away with
the initiative inherent in every court to avoid an absurd result by means of rendering a reasonable
interpretation and application of the procedural law. Indeed, the procedural law must always be given a
reasonable construction to preclude absurdity in its application. 35 Hence, a literal application of the
principle governing prejudicial questions is to be eschewed if such application would produce unjust and
absurd results or unreasonable consequences. ITDHSE

San Miguel Properties further submits that respondents could not validly raise the prejudicial question
as a reason to suspend the criminal proceedings because respondents had not themselves initiated
either the action for specific performance or the criminal action. It contends that the defense of a
prejudicial question arising from the filing of a related case could only be raised by the party who filed or
initiated said related case.

The submission is unfounded. The rule on prejudicial question makes no distinction as to who is allowed
to raise the defense. Ubi lex non distinguit nec nos distinguere debemos. When the law makes no
distinction, we ought not to distinguish. 36

WHEREFORE, the Court AFFIRMS the decision promulgated on February 24, 2004 by the Court of
Appeals in CA-G.R. SP NO. 73008; and ORDERS petitioner to pay the costs of suit. SacTCA

SO ORDERED.

Sereno, C.J., Villarama, Jr., Reyes and Perlas-Bernabe, * JJ., concur.

299
Quimiguing vs. Icao, G.R. No. L-26795, July 31, 1970

[G.R. No. L-26795. July 31, 1970.]

CARMEN QUIMIGUING, suing through her parents, ANTONIO QUIMIGUING and JACOBA CABILIN,
plaintiffs-appellants, vs. FELIX ICAO, defendant-appellee.

Torcuato L. Galon for plaintiffs-appellants.

Godardo Jacinto for defendant-appellee.

DECISION

REYES, J p:

Appeal on points of law from an order of the Court of First Instance of Zamboanga del Norte (Judge
Onofre Sison Abalos, presiding), in its Civil Case No. 1590, dismissing a complaint for support and
damages, and another order denying amendment of the same pleading.

The events in the court of origin can be summarized as follows:

Appellant, Carmen Quimiguing, assisted by her parents, sued Felix Icao in the court below. In her
complaint it was averred that the parties were neighbors in Dapitan City, and had close and confidential
relations; that defendant Icao, although married, succeeded in having carnal intercourse with plaintiff
several times by force and intimidation, and without her consent; that as a result she became pregnant,
despite efforts and drugs supplied by defendant, and plaintiff had to stop studying. Hence, she claimed
support at P120.00 per month, damages and attorney's fees.

Duly summoned, defendant Icao moved to dismiss for lack of cause of action since the complaint did not
allege that the child had been born; and after hearing arguments, the trial judge sustained defendant's
motion and dismissed the complaint.

Thereafter, plaintiff moved to amend the complaint to allege that as a result of the intercourse, plaintiff
had later given birth to a baby girl; but the court, sustaining defendant's objection, ruled that no
amendment was allowable, since the original complaint averred no cause of action. Wherefore, the
plaintiff appealed directly to this Court.

We find the appealed orders of the court below to be untenable. A conceived child, although as yet
unborn, is given by law a provisional personality of its own for all purposes favorable to it, as explicitly
provided in Article 40 of the Civil Code of the Philippines. The unborn child, therefore, has a right to
support from it progenitors, particularly of the defendant-appellee (whose paternity is deemed
admitted for the purpose of the motion to dismiss), even if the said child is only "en ventre de sa mere;"
just as a conceived child, even if as yet unborn, may receive donations as prescribed by Article 742 of
the same Code, and its being ignored by the parent in his testament may result in preterition of a forced
heir that annuls the institution of the testamentary heir, even if such child should be born after the
death of the testator (Article 854, Civil Code)

300
"ART. 742. Donations made to conceived and unborn children may be accepted by those persons
who would legally represent them if they were already born."

"ART. 854. The preterition or omission of one, some, or all of the compulsory heirs in the direct
line, whether living at the time of the execution of the will or born after the death of the testator, shall
annul the institution of heir; but the devises and legacies shall be valid insofar as they are not inofficious.

"If the omitted compulsory heirs should die before the testator, the institution shall be effectual,
without prejudice to the right of representation."

It is thus clear that the lower court's theory that Article 291 of the Civil Code declaring that support is an
obligation of parents and illegitimate children "does not contemplate support to children as yet
unborn," violates Article 40 aforesaid, besides imposing a condition that nowhere appears in the text of
Article 291.

It is true that Article 40 prescribing that "the conceived child shall be considered born for all purposes
that are favorable to it" adds further "provided it be born later with the conditions specified in the
following article" (i.e., that the foetus be alive at the time it is completely delivered from the mother's
womb). This proviso, however, is not a condition precedent to the right of the conceived child; for if it
were, the first part of Article 40 would become entirely useless and ineffective. Manresa, in his
Commentaries (5th Ed.) to the corresponding Article 29 of the Spanish Civil Code, clearly points this out:

"Los derechos atribuidos al nasciturus no son simples expectativas, ni aun en el sentido tecnico que la
moderna doctrina da a esta figura juridica, sino que constituyen un caso de los propiamente llamados
'derechos en estado de pendencia'; el nacimiento del sujeto en las condiciones previstas por el art. 30,
no determina el nacimiento de aquellos derechos (que ya existian de antemano), sino que se trata de un
hecho que tiene efectos declarativos. (1 Manresa, Op. cit., page 271)

A second reason for reversing the orders appealed from is that for a married man to force a woman not
his wife to yield to his lust (as averred in the original complaint in this case) constitutes a clear violation
of the rights of his victim that entitles her to claim compensation for the damage caused. Says Article 21
of the Civil Code of the Philippines:

"ART. 21. Any person who wilfully causes loss or injury to another in a manner that is contrary to
morals, good customs or public policy shall compensate the latter for the damage.'

The rule of Article 21 is supported by Article 2219 of the same Code:

"ART. 2219. Moral damages may be recovered in the following and analogous cases:

(3) Seduction, abduction, rape or other lascivious acts:

xxx xxx xxx

(10) Acts and actions referred to in Articles 21, 26, 27, 28 . . ."

301
Thus, independently of the right to support of the child she was carrying, plaintiff herself had a cause of
action for damages under the terms of the complaint; and the order dismissing it for failure to state a
cause of action was doubly in error.

WHEREFORE, the orders under appeal are reversed and set aside. Let the case be remanded to the court
of origin for further proceedings conformable to this decision. Costs against appellee Felix Icao. So
ordered.

Concepcion, C.J., Dizon, Makalintal, Zaldivar, Castro, Fernando, Teehankee, Barredo and Villamor, JJ.,
concur.

302
Geluz vs. CA, G.R. No. L-16439, July 20, 1961

[G.R. No. L-16439. July 20, 1961.]

ANTONIO GELUZ, petitioner, vs. THE HON. COURT OF APPEALS and OSCAR LAZO, respondents.

Mariano H. de Joya for petitioner.

A. P. Salvador for respondents.

SYLLABUS

1. CRIMINAL LAW; ABORTION; CONSENT OF WOMAN OR HUSBAND DOES NOT EXCUSE CRIMINAL
ACT. — Abortion, without medical necessity to warrant it, is a criminal act, and neither the consent of
the woman nor that of the husband would excuse it.

2. DAMAGES; UNBORN FOETUS: WITHOUT PERSONALITY; AWARD FOR DEATH OF A PERSON DOES
NOT COVER UNBORN FOETUS. — The minimum award for the death of a person does not cover the case
of an unborn foetus that is not endowed with personality and incapable of having rights and obligations.

3. ID.; ID.; PARENTS OF UNBORN FOETUS CANNOT SUE FOR DAMAGES ON ITS BEHALF. — Since an
action for pecuniary damages on account of personal injury or death pertains primarily to the injured,
no such right of action could derivatively accrue to the parents or heirs of an unborn child.

4. ID.; ID.; NATURE OF DAMAGES RECOVERABLE BY PARENTS OF UNBORN CHILD. — The damages
which the parents of an unborn child can recover are limited to the moral damages for the illegal arrest
of normal development of the foetus, i.e., on account of distress and anguish attendant to its loss, and
the disappointment of their parental expectations, as well as to exemplary damages, if the
circumstances should warrant them (Art. 2230, New Civil Code).

DECISION

REYES, J.B.L., J p:

This petition for certiorari brings up for review the question whether the husband of a woman, who
voluntarily procured her abortion, could recover damages from the physician who caused the same.

The litigation was commenced in the Court of First Instance of Manila by respondent Oscar Lazo, the
husband of Nita Villanueva, against petitioner Antonio Geluz, a physician. Convinced of the merits of the
complaint upon the evidence adduced, the trial court rendered judgment in favor of plaintiff Lazo and
against defendant Geluz ordering the latter to pay P3,000 as damages, P700 as attorney's fees and the
costs of the suit. On appeal, the Court of Appeals, in a special division of five, sustained the award by a
majority vote of three justices as against two, who rendered a separate dissenting opinion.

The facts are set forth in the majority opinion as follows:

303
"Nita Villanueva came to know the defendant (Antonio Geluz) for the first time in 1948 — through her
aunt Paula Yambot. In 1950 she became pregnant by her present husband before they were legally
married. Desiring to conceal her pregnancy from her parent, and acting on the advice of her aunt, she
had herself aborted by the defendant. After her marriage with the plaintiff, she again became pregnant.
As she was then employed in the Commission on Elections and her pregnancy proved to be
inconvenient, she had herself aborted again by the defendant in October 1953. Less than two years
later, she again became pregnant. On February 21, 1955, accompanied by her sister Purificacion and the
latter's daughter Lucida, she again repaired to the defendant's clinic on Carriedo and P. Gomez streets in
Manila, where the three met the defendant and his wife. Nita was again aborted, of a two-month old
foetus, in consideration of the sum of fifty pesos, Philippine currency. The plaintiff was at this time in the
province of Cagayan, campaigning for his election to the provincial board; he did not know of, nor gave
his consent to, the abortion."

It is the third and last abortion that constitutes plaintiffs basis in filing this action and award of damages.
Upon application of the defendant Geluz, we granted certiorari.

The Court of Appeals and the trial court predicated the award of damages in the sum of P3,000.00 upon
the provisions of the initial paragraph of Article 2206 of the Civil Code of the Philippines. This we believe
to be error, for the said article, in fixing a minimum award of P3,000 for the death of a person, does not
cover the case of an unborn foetus that is not endowed with personality. Under the system of our Civil
Code, "la criatura abortiva no alcanza la categoria de persona natural y en consecuencia es un ser no
nacido a la vida del Derecho" (Casso-Cervera, "Diccionario de Derecho Privado" Vol. 1, p. 49). being
incapable of having rights and obligations.

Since an action for pecuniary damages on account of personal injury or death pertains primarily to the
one injured, it is easy to see that if no action for such damages could be instituted on behalf of the
unborn child on account of the injuries it received, no such right of action could derivatively accrue to its
parents or heirs. In fact, even if a cause of action did accrue on behalf of the unborn child, the same was
extinguished by its pre-natal death, since no transmission to anyone can take place from one that lacked
juridical personality (or juridical capacity, as distinguished from capacity to act). It is no answer to invoke
the provisional personality of a conceived child (conceptus pro nato habetur) under Article 40 of the
Civil Code, because that same article expressly limits such provisional personality by imposing the
condition that the child should be subsequently born alive: "provided it be born later with the conditions
specified in the following article". In the present case, there is no dispute that the child was dead when
separated from its mother's womb.

The prevailing American jurisprudence is to the same effect; and is generally held that recovery can not
be had for the death of an unborn child (Stafford vs. Roadway Transit Co., 70 F. Supp. 555; Dietrich vs.
Northhampton, 52 Am. Rep. 242; and numerous cases collated in the editorial note, 10 ALR (2d) 639).

This is not to say that the parents are not entitled to collect any damages at all. But such damages must
be those inflicted directly upon them, as distinguished from the injury or violation of the rights of the
deceased, his right to life and physical integrity. Because the parents can not expect either help, support

304
or services from an unborn child, they would normally be limited to moral damages for the illegal arrest
of the normal development of the spes hominis that was the foetus, i.e. on account of distress and
anguish attendant to its loss, and the disappointment of their parental expectations (Civ. Code, Art.
2217), as well as to exemplary damages, if the circumstances should warrant them (Art. 2230). But in
the case before us, both the trial court and the Court of Appeals have not found any basis for an award
of moral damages, evidently because the appellee's indifference to the previous abortions of his wife,
also caused by the appellant herein, clearly indicates that he was unconcerned with the frustration of his
parental hopes and affections. The lower court expressly found, and the majority opinion of the Court of
Appeals did not contradict it, that the appellee was aware of the second abortion; and the probabilities
are that he was likewise aware of the first. Yet despite the suspicious repetition of the event, he
appeared to have taken no steps to investigate or pinpoint the causes thereof, and secure the
punishment of the responsible practitioner. Even after learning of the third abortion, the appellee does
not seem to have taken interest in the administrative and criminal cases against the appellant. His only
concern appears to have been directed at obtaining from the doctor a large money payment, since he
sued for P50,000 damages and P3,000 attorneys fees, an "indemnity" claim that, under the
circumstances of record, was clearly exaggerated.

The dissenting Justices of the Court of Appeals have aptly remarked that:

"It seems to us that the normal reaction of a husband who righteously feels outraged by the abortion
which his wife has deliberately sought at the hands of a physician would be high-minded rather than
mercenary; and that his primary concern would be to see to it that the medical profession was purged of
an unworthy member rather than turn his wife's indiscretion to personal profit, and with that idea in
mind to press either the administrative or the criminal cases he had filed, or both, instead of abandoning
them in favor of a civil action for damages of which not only he, but also his wife, would be the
beneficiaries."

It is unquestionable that the appellant's act in provoking the abortion of appellee's wife, without
medical necessity to warrant it, was a criminal and morally reprehensible act, that can not be too
severely condemned; and the consent of the woman or that of her husband does not excuse it. But the
immorality or illegality of the act does not justify an award of damages that, under the circumstances on
record, have no factual or legal basis.

The decision appealed from is reversed, and the complaint ordered dismissed. Without costs.

Let a copy of this decision be furnished the Department of Justice and the Board of Medical Examiners
for their information and such investigation and action against the appellee Antonio Geluz as the facts
may warrant.

Bengzon, C.J., Padilla, Labrador, Barrera, Paredes, Dizon and Natividad, JJ., concur.

Concepcion, J., took no part.

De Leon, J., did not take part.

305
Limjoco vs. Estate of Pedro Fragante, G.R. No. L-770, 27 April 1948

[G.R. No. L-770. April 27, 1948.]

ANGEL T. LIMJOCO, petitioner, vs. INTESTATE ESTATE OF PEDRO O. FRAGANTE, deceased, respondent.

Angel Limjoco, jr. and Delfin L. Gonzales, for petitioner.

Bienvenido A. Tan, for respondent.

SYLLABUS

1. PUBLIC SERVICE COMMISSION; CERTIFICATE OF PUBLIC CONVENIENCE; RIGHT OF ESTATE OF


DECEDENT TO PROSECUTE APPLICATION; CASE AT BAR. — If P. O. F. had not died, there can be no
question that he would have had the right to prosecute his application for a certificate of public
convenience to its final conclusion. No one would have denied him that right. As declared by the
commission in its decision, he had invested in the ice plant in question P35,000, and from what the
commission said regarding his other properties and business, he would certainly have been financially
able to maintain and operate said plant had he not died. His transportation business alone was netting
him about P1,440 monthly. He was a Filipino citizen and continued to be such till his demise. The
commission declared in its decision, in view of the evidence before it, that his estate was financially able
to maintain and operate the ice plant. The aforesaid light of P. O. F. to prosecute said application to its
final conclusion was one which by its nature did not lapse through his death. Hence, it constitutes a part
of the assets of his estate, for such a right was property despite the possibility that in the end the
commission might have denied the application, although under the facts of the case, the commission
granted the application in view of the financial ability of the estate to maintain and operate the ice
plant. Petitioner, in his memorandum of March 19, 1947, admits (p. 3) that a certificate of public
convenience once granted "as a rule, should descend to his estate as an asset." Such certificate would
certainly be property, and the right to acquire such a certificate, by complying with the requisites of the
law, belonged to the decedent in his lifetime, and survived to his estate and judicial administrator after
his death.

2. ID.; ID.; ID.; ESTATE OF DECEDENT, A PERSON; CASE AT BAR. — Within the philosophy of the
present legal system and within the framework of the constitution, the estate of P. O. F. should be
considered an artificial or juridical person for the purposes of the settlement and distribution of his
estate which, of course, include the exercise during the judicial administration thereof of those rights
and the fulfillment of those obligations of his which survived after his death. One of those rights was the
one involved in his pending application before the Public Service Commission in the instant case,
consisting in the prosecution of said application to its final conclusion. An injustice would ensue from
the opposite course.

3. ID.; ID.; ID.; ID.; CITIZENSHIP OF DECEDENT EXTENDED TO HIS ESTATE; CASE AT BAR. — If by
legal fiction the personality of P. O. F. is considered extended so that any debts or obligations left by,
and surviving, him may be paid, and any surviving rights may be exercised for the benefit of his creditors

306
and heirs, respectively, there is no sound and cogent reason for denying the application of the same
fiction to his citizenship, and for not considering it as likewise extended for the purposes of the aforesaid
unfinished proceeding before the Public Service Commission. The outcome of said proceeding, if
successful, would in the end inure to the benefit of the same creditors, and the heirs. Even in that event
petitioner could not allege any prejudice in the legal sense, any more than he could have done if F. had
lived longer and obtained the desired certificate. The fiction of such extension of his citizenship is
grounded upon the same principle, and motivated by the same reason, as the fiction of the extension of
his personality. The fiction is made necessary to avoid the injustice of subjecting his estate, creditors and
heirs, solely by reason of his death, to the loss of the investment amounting to P35,000, which he
already made in the ice plant, not counting the other expenses occasioned by the instant proceeding,
from the Public Service Commission to this court.

DECISION

HILADO, J p:

Under date of May 21, 1946, the Public Service Commission, through Deputy Commissioner Fidel Ibañez,
rendered its decision in case No. 4572 of Pedro O. Fragante, as applicant for a certificate of public
convenience to install, maintain and operate an ice plant in San Juan, Rizal, whereby said commission
held that the evidence therein showed that the public interest and convenience will be promoted in a
proper and suitable manner "by authorizing the operation and maintenance of another ice plant of two
and one-half (2-1/2) tons in the municipality of San Juan; that the original applicant Pedro 0. Fragante
was a Filipino citizen at the time of his death; and that his intestate estate is financially capable of
maintaining the proposed service". The commission, therefore, overruled the opposition filed in the
case and ordered "that under the provisions of section 15 of Commonwealth Act No. 146, as amended, a
certificate of public convenience be issued to the Intestate Estate of the deceased Pedro Fragante,
authorizing said Intestate Estate through its Special or Judicial Administrator, appointed by the proper
court of competent jurisdiction, to maintain and operate an ice plant with a daily productive capacity of
two and one half tons (2-1/2) in the Municipality of San Juan and to sell the ice produced from said plant
in the said Municipality of San Juan and in the Municipality of Mandaluyong, Rizal, and in Quezon City",
subject to the conditions therein set forth in detail (petitioner's brief, pp. 33-34).

Petitioner makes four assignments of error in his brief as follows:

"1. The decision of the Public Service Commission is not in accordance with law.

"2. The decision of the Public Service Commission is not reasonably supported by evidence.

"3. The Public Service Commission erred in not giving petitioner and the Ice and Cold Storage
Industries of the Philippines, Inc., as existing operators, a reasonable opportunity to meet the increased
demand.

"4. The decision of the Public Service Commission is an unwarranted departure from its announced
policy with respect to the establishment and operation of ice plant." (Pp. 1-2, petitioner's brief.)

307
In his argument petitioner contends that it was error on the part of the commission to allow the
substitution of the legal representative of the estate of Pedro O. Fragante for the latter as party
applicant in the case then pending before the commission, and in subsequently granting to said estate
the certificate applied for, which is said to be in contravention of law.

If Pedro O. Fragante had not died, there can be no question that he would have had the right to
prosecute his application before the commission to its final conclusion. No one would have denied him
that right. As declared by the commission in its decision, he had invested in the ice plant in question
P35,000, and from what the commission said regarding his other properties and business, he would
certainly have been financially able to maintain and operate said plant had he not died. His
transportation business alone was netting him about P1,440 monthly. He was a Filipino citizen and
continued to be such till his demise. The commission declared in its decision, in view of the evidence
before it, that his estate was financially able to maintain and operate the ice plant. The aforesaid right of
Pedro O. Fragante to prosecute said application to its final conclusion was one which by its nature did
not lapse through his death. Hence, it constitutes a part of the assets of his estate, for such a right was
property despite the possibility that in the end the commission might have denied the application,
although under the facts of the case, the commission granted the application in view of the financial
ability of the estate to maintain and operate the ice plant. Petitioner, in his memorandum of March 19,
1947, admits (page 3) that a certificate of public convenience once granted "as a rule, should descend to
his estate as an asset". Such certificate would certainly be property, and the right to acquire such a
certificate, by complying with the requisites of the law, belonged to the decedent in his lifetime, and
survived to his estate and judicial administrator after his death.

If Pedro O. Fragante had in his lifetime secured an option to buy a piece of land and during the life of the
option he died, if the option had been given him in the ordinary course of business and not out of
special consideration for his person, there would be no doubt that said option and the right to exercise
it would have survived to his estate and legal representatives. In such a case there would also be the
possibility of failure to acquire the property should he or his estate or legal representative fail to comply
with the conditions of the option. In the case at bar Pedro O. Fragante's undoubted right to apply for
and acquire the desired certificate of public convenience — the evidence established that the public
needed the ice plant — was under the law conditioned only upon the requisite citizenship and economic
ability to maintain and operate the service. Of course, such right to acquire or obtain such certificate of
public convenience was subject to failure to secure its objective through nonfulfillment of the legal
conditions, but the situation here is no different from the legal standpoint from that of the option in the
illustration just given.

Rule 88, section 2, provides that the executor or administrator may bring or defend actions, among
other cases, for the protection of the property or rights of the deceased which survive, and it says that
such actions may be brought or defended "in the right of the deceased".

Rule 82, section 1, paragraph (a), mentions among the duties of the executor or administrator, the
making of an inventory of all goods, chattels, rights, credits, and estate of the deceased which shall
come to his possession or knowledge, or to the possession of any other person for him.

308
In his commentaries on the Rules of Court (Volume II, 2nd ed., pages 366, 367), the present Chief Justice
of this Court draws the following conclusion from the decisions cited by him:

"Therefore, unless otherwise expressly provided by law, any action affecting the property or rights
(underscoring supplied) of a deceased person which may be brought by or against him if he were alive,
may likewise be instituted and prosecuted by or against the administrator, unless the action is for
recovery of money, debt or interest thereon, or unless, by its very nature, it cannot survive, because
death extinguishes the right . . .".

It is true that a proceeding upon an application for a certificate of public convenience before the Public
Service Commission is not an "action". But the foregoing provisions and citations go to prove that the
decedent's rights which by their nature are not extinguished by death go to make up a part and parcel of
the assets of his estate which, being placed under the control and management of the executor or
administrator, can not be exercised but by him in representation of the estate for the benefit of the
creditors, devisees, or legatees, if any, and the heirs of the decedent. And if the right involved happens
to consist in the prosecution of an unfinished proceeding upon an application for a certificate of public
convenience of the deceased before the Public Service Commission, it is but logical that the legal
representative be empowered and entitled in behalf of the estate to make the right effective in that
proceeding.

Manresa (Vol. III, 6th ed., p. 11) says that No. 10 of article 334 and article 336 of the Civil Code,
respectively, consider as immovable and movable things rights which are not material. The same
eminent commentator says in the cited volume (p. 45) that article 336 of the Civil Code has been
deficiently drafted in that it is not sufficiently expressive of all incorporeal rights which are also property
for juridical purposes.

Corpus Juris (Vol. 50, p. 737) states that in the broad sense of the term, property includes, among other
things, "an option", and "the certificate of the railroad commission permitting the operation of a bus
line," and on page 748 of the same volume we read:

"However, these terms (real property, as estate or interest) have also been declared to include every
species of title, inchoate or complete, and embrace rights which lie in contract, whether executory or
executed." (Italics supplied.)

Another important question raised by petitioner is whether the estate of Pedro O. Fragante is a
"person" within the meaning of the Public Service Act.

Words and Phrases, First Series, (Vol. 6, p. 5325), states the following doctrine in the jurisdiction of the
State of Indiana:

"As the estate of a decedent is in law regarded as a person, a forgery committed after the death of the
man whose name purports to be signed to the instrument may be prosecuted as with the intent to
defraud the estate. Billings vs. State, 107 Ind., 54, 55, 6 N. E. 914, 7 N. E. 763, 57 Am. Rep. 77."

309
The Supreme Court of Indiana in the decision cited above had before it a case of forgery committed
after the death of one Morgan for the purpose of defrauding his estate. The objection was urged that
the information did not aver that the forgery was committed with the intent to defraud any person. The
Court, per Elliott, J., disposed of this objection as follows:

". . . The reason advanced in support of this proposition is that the law does not regard the estate of a
decedent as a person. This intention (contention) cannot prevail. The estate of a decedent is a person in
legal contemplation. 'The word "person", says Mr. Abbot, 'in its legal signification, is a generic term, and
includes artificial as well as natural persons,' 2 Abb. Dict. 271; Douglas vs. Pacific, etc., Co., 4 Cal. 304;
Planters', etc., Bank vs. Andrews, 8 Port (Ala.) 404. It is said in another work that 'persons are of two
kinds: natural and artificial. A natural person is a human being. Artificial persons include (1) a collection
or succession of natural persons forming a corporation; (2) a collection of property to which the law
attributes the capacity of having rights and duties. The latter class of artificial persons is recognized only
to a limited extent in our law. Examples are the estate of a bankrupt or deceased person.' 2 Rapalje & L.
Law Dict. 954. Our own cases inferentially recognize the correctness of the definition given by the
authors from whom we have quoted, for they declare that it is sufficient, in pleading a claim against a
decedent's estate, to designate the defendant as the estate of the deceased person, naming him. Ginn
vs. Collins, 43 Ind. 271. Unless we accept this definition as correct, there would be a failure of justice in
cases where, as here, the forgery is committed after the death of the person whose name is forged; and
this is a result to be avoided if it can be done consistent with principle. We perceive no difficulty in
avoiding such a result; for, to our minds, it seems reasonable that the estate of a decedent should be
regarded as an artificial person. It is the creation of law for the purpose of enabling a disposition of the
assets to be properly made, and, although natural persons as heirs, devisees, or creditors, have an
interest in the property, the artificial creature is a distinct legal entity. The interest which natural
persons have in it is not complete until there has been a due administration; and one who forges the
name of the decedent to an instrument purporting to be a promissory note must be regarded as having
intended to defraud the estate of the decedent, and not the natural persons having diverse interests in
it, since he cannot be presumed to have known who those persons were, or what was the nature of
their respective interests. The fraudulent intent is against the artificial person, — the estate, — and not
the natural persons who have direct or contingent interests in it." (107 Ind. 54, 55, 6 N. E. 914-915.)

In the instant case there would also be a failure of justice unless the estate of Pedro O. Fragante is
considered a "person", for the quashing of the proceedings for no other reason than his death would
entail prejudicial results to his investment amounting to P35,000.00 as found by the commission, not
counting the expenses and disbursements which the proceeding can be presumed to have occasioned
him during his lifetime, let alone those defrayed by the estate thereafter. In this jurisdiction there are
ample precedents to show that the estate of a deceased person is also considered as having legal
personality independent of the heirs. Among the most recent cases may be mentioned that of "Estate of
Mota vs. Concepcion, 56 Phil., 712, 717, wherein the principal plaintiff was the estate of the deceased
Lazaro Mota, and this Court gave judgment in favor of said estate along with the other plaintiffs in these
words:

310
". . . the judgment appealed from must be affirmed so far as it holds that defendants Concepcion and
Whitaker are indebted to the plaintiffs in the amount of P245,804.69 . . .."

Under the regime of the Civil Code and before the enactment of the Code of Civil Procedure, the heirs of
a deceased person were considered in contemplation of law as the continuation of his personality by
virtue of the provision of article 661 of the first Code that the heirs succeed to all the rights and
obligations of the decedent by the mere fact of his death. It was so held by this Court in Barrios vs.
Dolor, 2 Phil., 44, 46. However, after the enactment of the Code of Civil Procedure, article 661 of the
Civil Code was abrogated, as held in Suiliong & Co. vs. Chio-Taysan, 12 Phil., 13 22. In that case, as well
as in many others decided by this Court after the innovations introduced by the Code of Civil Procedure
in the matter of estates of deceased persons, it has been the constant doctrine that it is the estate or
the mass of property, rights and assets left by the decedent, instead of the heirs directly, that becomes
vested and charged with his rights and obligations which survive after his demise.

The heirs were formerly considered as the continuation of the decedent's personality simply by legal
fiction, for they might not be even of his flesh and blood — the reason was one in the nature of a legal
exigency derived from the principle that the heirs succeeded to the rights and obligations of the
decedent. Under the present legal system, such rights and obligations as survive after death have to be
exercised and fulfilled only by the estate of the deceased. And if the same legal fiction were not
indulged, there would be no juridical basis for the estate, represented by the executor or administrator,
to exercise those rights and to fulfill those obligations of the deceased. The reason and purpose for
indulging the fiction is identical and the same in both cases. This is why according to the Supreme Court
of Indiana in Billings vs. State, supra, citing 2 Rapalje & L. Dictionary, 954, among the artificial persons
recognized by law figures "a collection of property to which the law attributes the capacity of having
rights and duties", as for instance, the estate of a bankrupt or deceased person.

Petitioner raises the decisive question of whether or not the estate of Pedro O. Fragante can be
considered a "citizen of the Philippines" within the meaning of section 16 of the Public Service Act, as
amended, particularly the proviso thereof expressly and categorically limiting the power of the
commission to issue certificates of public convenience or certificates of public convenience and
necessity "only to citizens of the Philippines or of the United States or to corporations, co-partnerships,
associations, or joint-stock companies constituted and organized under the laws of the Philippines", and
the further proviso that sixty per centum of the stock or paid-up capital of such entities must belong
entirely to citizens of the Philippines or of the United States.

Within the philosophy of the present legal system, the underlying reason for the legal fiction by which,
for certain purposes, the estate of a deceased person is considered a "person" is the avoidance of
injustice or prejudice resulting from the impossibility of exercising such legal rights and fulfilling such
legal obligations of the decedent as survived after his death unless the fiction is indulged. Substantially
the same reason is assigned to support the same rule in the jurisdiction of the State of Indiana, as
announced in Billings vs. State, supra, when the Supreme Court of said State said:

311
". . . It seems reasonable that the estate of a decedent should be regarded as an artificial person. It is the
creation of law for the purpose of enabling a disposition of the assets to be properly made . . .."

Within the framework and principles of the constitution itself, to cite just one example, under the bill of
rights it seems clear that while the civil rights guaranteed therein in the majority of cases relate to
natural persons, the term "person" used in section 1 (1) and (2) must be deemed to include artificial or
juridical persons, for otherwise these latter would be without the constitutional guarantee against being
deprived of property without due process of law, or the immunity from unreasonable searches and
seizures. We take it that it was the intendment of the framers to include artificial or juridical, no less
than natural, persons in these constitutional immunities and in others of similar nature. Among these
artificial or juridical persons figure estates of deceased persons. Hence, we hold that within the
framework of the constitution, the estate of Pedro O. Fragante should be considered an artificial or
juridical person for the purposes of the settlement and distribution of his estate which, of course,
include the exercise during the judicial administration thereof of those rights and the fulfillment of those
obligations of his which survived after his death. One of those rights was the one involved in his pending
application before the Public Service Commission in the instant case, consisting in the prosecution of
said application to its final conclusion. As stated above, an injustice would ensue from the opposite
course.

How about the point of citizenship? If by legal fiction his personality is considered extended so that any
debts or obligations left by, and surviving, him may be paid, and any surviving rights may be exercised
for the benefit of his creditors and heirs, respectively, we find no sound and cogent reason for denying
the application of the same fiction to his citizenship, and for not considering it as likewise extended for
the purposes of the aforesaid unfinished proceeding before the Public Service Commission. The
outcome of said proceeding, if successful, would in the end inure to the benefit of the same creditors
and the heirs. Even in that event petitioner could not allege any prejudice in the legal sense, any more
than he could have done if Fragante had lived longer and obtained the desired certificate. The fiction of
such extension of his citizenship is grounded upon the same principle, and motivated by the same
reason, as the fiction of the extension of his personality. The fiction is made necessary to avoid the
injustice of subjecting his estate, creditors and heirs, solely by reason of his death, to the loss of the
investment amounting to P35,000, which he had already made in the ice plant, not counting the other
expenses occasioned by the instant proceeding, from the Public Service Commission to this Court.

We can perceive no valid reason for holding that within the intent of the Constitution (Article IV), its
provisions on Philippine citizenship exclude the legal principle of extension above adverted to. If for
reasons already stated our law indulges the fiction of extension of personality, if for such reasons the
estate of Pedro O. Fragante should be considered an artificial or juridical person herein, we can find no
justification for refusing to declare a like fiction as to the extension of his citizenship for the purposes of
this proceeding.

Pedro O. Fragante was a Filipino citizen, and as such, if he had lived, in view of the evidence of record,
he would have obtained from the commission the certificate for which he was applying. The situation
has suffered but one change, and that is, his death. His estate was that of a Filipino citizen. And its

312
economic ability to appropriately and adequately operate and maintain the service of an ice plant was
the same that it received from the decedent himself. In the absence of a contrary showing, which does
not exist here, his heirs may be assumed to be also Filipino citizens; and if they are not, there is the
simple expedient of revoking the certificate or enjoining them from inheriting it.

Upon the whole, we are of opinion that for the purposes of the prosecution of said case No. 4572 of the
Public Service Commission to its final conclusion, both the personality and citizenship of Pedro O.
Fragante must be deemed extended, within the meaning and intent of the Public Service Act, as
amended, in harmony with the constitution: it is so adjudged and decreed.

Decision affirmed, without costs. So ordered.

Moran, C. J., Paras, Pablo, Bengzon, Briones, Padilla and Tuason, JJ., concur.

Paras, J., I hereby certify that Mr. Justice Feria voted with the majority.

Separate Opinions

PERFECTO, J., dissenting:

Commonwealth Act No. 146 reserves to Filipino citizens the right to obtain a certificate of public
convenience to operate an ice plant in San Juan, Rizal. The limitation is in accordance with section 8 of
Article XIV of the Constitution which provides.

"No franchise, certificate, or any other form of authorization for the operation of a public utility shall be
granted except to citizens of the Philippines or to corporations or other entities organized under the
laws of the Philippines, sixty per centum of the capital of which is owned by citizens of the Philippines,
nor shall such franchise, certificate, or authorization be exclusive in character or for a longer period than
fifty years. No franchise or right shall be granted to any individual, firm, or corporation, except under the
condition that it shall be subject to amendment, alteration, or repeal by the Congress when the public
interest so requires."

The main question in this case is whether the estate of Pedro O. Fragante fulfills the citizenship
requirement. To our mind, the question can be restated by asking whether the heirs of Pedro O.
Fragante fulfill the citizenship requirement of the law.

The estate is an abstract entity. As such, its legal value depends on what it represents. It is a device by
which the law gives a kind of personality and unity to undetermined tangible persons, the heirs. They
inherit and replace the deceased at the very moment of his death. As there are procedural requisites for
their identification and determination that need time for their compliance, a legal fiction has been
devised to represent them. That legal fiction is the estate, a liquid condition in process of solidification.

The estate, therefore, has only a representative value. What the law calls estate is, as a matter of fact,
intended to designate the heirs of the deceased. The question, therefore, in this case, boils down to the
citizenship of the heirs of Fragante.

313
There is nothing in the record to show conclusively the citizenship of the heirs of Fragante. If they are
Filipino citizens, the action taken by the Public Service Commission should be affirmed. If they are not, it
should be reversed.

Petitioner alleges that the estate is just a front or dummy for aliens to go around the citizenship
constitutional provision. It is alleged that Gaw Suy, the special administrator of the estate, is an alien.

We are of opinion that the citizenship of the heirs of Fragante should be determined by the Commission
upon evidence that the party should present. It should also determine the dummy question raised by
petitioner.

We are of opinion and so vote that the decision of the Public Service Commission of May 21, 1946, be
set aside and that the Commission be instructed to receive evidence on the above factual questions and
render a new decision accordingly.

314
Moy Ya Lim Yao vs. CID, 41 SCRA 292

[G.R. No. L-21289. October 4, 1971.]

MOY YA LIM YAO alias EDILBERTO AGUINALDO LIM and LAU YUEN YEUNG, petitioners-appellants, vs.
THE COMMISSIONER OF IMMIGRATION, respondent-appellee.

Aruego, Mamaril & Associates for petitioners-appellants.

Solicitor General Arturo A. Alafriz, Asst. Sol. Gen. Frine C . Zaballero and Solicitor Sumilang V . Bernardo
for respondent-appellee.

SYLLABUS

1. POLITICAL LAW; CITIZENSHIP; IMMIGRATION ACT; SECTION 9 (G) THEREOF, NOT APPLICABLE TO
ALIEN WHO LEGITIMATELY BECOMES FILIPINO. — Section 9 (g) of the Immigration Act does not apply to
aliens who after coming into the Philippines as temporary visitors, legitimately become Filipino citizens
or acquire Filipino citizenship. Such change of nationality naturally bestows upon them the right to stay
in the Philippines permanently or not, as they may choose, and if they elect to reside here, the
immigration authorities may neither deport them nor confiscate their bonds.

2. ID.; ID.; NATURALIZATION; EFFECTS. — The naturalization of an alien visitor as a Philippine


citizen logically produces the effect of conferring upon him ipso facto all the rights of citizenship
including that of being entitled to permanently stay in the Philippines outside the orbit of authority of
the Commissioner of Immigration vis-avis aliens, if only because by its very nature and express
provisions, the Immigration Law is a law only for aliens and is inapplicable to citizens of the Philippines.

3. STATUTORY CONSTRUCTION; WHERE LANGUAGE OF STATUTE IS SUSCEPTIBLE OF TWO


CONSTRUCTIONS, THAT WHICH CARRIES OUT OBJECT PREVAILS. — A statute is to be construed with
reference to its manifest object, and if the language is susceptible of two constructions, one which will
carry out and the other defeat such manifest object, it should receive the former construction. A
construction will cause objectionable results should be avoided and the court will, if possible, place on
the statute a construction which will not result in injustice, and in accordance with the decisions
construing statutes, a construction will not result in oppression, hardship, or inconveniences will also be
avoided, as will a construction which will prejudice public interest, or construction resulting in
unreasonableness, as well as a construction which will result in absurd consequences.

4. ID.; CONSTRUCTION AVOIDED IF INCONSISTENT WITH LEGISLATIVE INTENT. — So a construction


should, if possible, be avoided if the result would be an apparent inconsistency in legislative intent, as
has been determined by the judicial decisions, or which would result in futility, redundancy, or a
conclusion not contemplated by the legislature; and the court should adopt that construction which will
be the least likely to produce mischief. Unless plainly shown to have been the intention of the legislature
an interpretation which would render the requirements of the statute uncertain and vague is to be
avoided, and the court will not ascribe to the legislature an intent to confer an illusory right.

315
5. POLITICAL LAW; CITIZENSHIP; NATURALIZATION; POLICY OF SELECTIVE ADMISSION, EXPLAINED.
— The avowed policy of "selective admission" more particularly refers to a case where a citizenship is
sought to be acquired in a judicial proceeding for naturalization. In such a case, the courts should no
doubt apply the national policy of selecting only those who are worthy to be come citizens. There is here
a choice between accepting or rejecting the application for citizenship. But this policy finds no
application is cases where citizenship is conferred by operation of law. In such cases, the courts have no
choice to accept or reject. If the individual claiming citizenship by operations of law proves in legal
proceedings that he satisfies the statutory requirements, the cannot do otherwise than to declare that
he is a citizens of the Philippines.

6. ID.; ID.; ID.; ALIEN WOMAN MARRYING FILIPINO IPSO FACTO BECOME CITIZEN PROVIDED NOT
DISQUALIFIED BY LAW. — We now hold, all previous decisions of this Court indicating otherwise
notwithstanding, that under Section 15 of Commonwealth Act 473, an alien woman marrying a Filipino,
native-born or naturalized, becomes ipso facto a Filipina provided she is not disqualified to be a citizen
of the Philippines under Section 4 of the same law. Likewise, an alien woman married an alien who is
subsequently naturalized here follows the Philippine citizenship of her husband the moment he takes his
oath as Filipino citizens, provided that she does not suffer from any of the disqualifications under said
Section 4.

7. ID.; ID.; ID.; ID.; NATURALIZATION PROCEEDING, NOT REQUIRED. — Section 16 is a parallel
provision to Section 15. If the widow of an applicant for naturalization a Filipino, who dies during the
proceedings, is not required to go through a naturalization proceeding, in order to be considered as a
Filipino citizen hereof, it should not follow that the wife of a living Filipino cannot be denied that same
privilege. This is plain common sense and there is absolutely no evidence that the Legislature intended
to treat them differently.

8. ID.; ID.; ID.; MODES OF. — The Constitution itself recognizes as Philippine citizens "Those who
are naturalized in accordance with law" (Section 1 [5], Article IV, Philippine Constitution). Citizens by
naturalization, under this provision, include not only those who are naturalized in accordance with legal
proceedings for the acquisition of citizenship, but also those who acquire citizenship by "derivative
naturalization" or by operation of law, as, for example, the "naturalization" of an alien wife through the
naturalization of her husband, or by marriage of an alien woman to a citizen.

9. ID.; ID.; ID.; SECTION 15 OF REVISED NATURALIZATION LAW; PURPOSE. — The leading idea or
purpose of Section 15 was to confer Philippine citizenship by operation of law upon certain classes of
aliens as a legal consequence of their relationship, by blood or by affinity, to persons who are already
citizens of the Philippines. Whenever the fact of relationship of the persons enumerated in the
provisions concurs with the fact of citizenship of the person to who they are related, the effect is for said
person to become ipso facto citizens of the Philippines. "Ipso facto" as here used does not mean that all
alien wives and all minor children of the Philippine citizens, from the mere fact of relationship,
necessarily become such citizens also. Those who do not meet the statutory requirements do not ipso
facto become citizens; they must apply for naturalization in order to acquire such status. What it does
mean, however, is that in respect of those persons enumerated in Section 15, the relationship to a

316
citizen of the Philippines is the operative fact which establishes the acquisition of Philippine citizenship
by them. Necessarily, it also determines the point of time at which such citizenship commences.

10. ID.; ID.; ID.; ID.; ALIEN WIFE DEEMED A CITIZEN IF SHE MIGHT HERSELF BE NATURALIZED. — The
legislature could not have intended that an alien wife should not be deemed a Philippine citizen unless
and until she proves that she might herself be lawfully naturalized. Far from it, the law states in plain
terms that she shall be deemed a citizen of the Philippines if she is one "who might herself be lawfully
naturalized." The proviso that she must be one "who might herself be lawfully naturalized" is not a
condition precedent to the vesting or acquisition of citizenship; it is only a condition or a state of fact
necessary to establish her citizenship as a factum probandum, i.e., as a fact established and proved in
evidence. The word "might," as used in that phrase, precisely implies that at the time of her marriage to
Philippine citizen, the alien woman "had (the) power" to become such a citizen herself under the laws
then in force.

11. ID.; ID.; RES JUDICATA NOT APPLICABLE TO RULINGS THEREON. — Everytime the citizenship of a
person is material or indispensable in a judicial or administrative case, whatever the corresponding
court or administrative authority decides therein as to such citizenship is generally not considered as res
adjudicata, hence it has to be threshed out again and again as the occasion may demand.

12. ID.; ID.; NATURALIZATION; PROCEDURES FOR ALIEN WIFE TO ACQUIRE PHILIPPINE CITIZENSHIP.
— Regarding the steps that should be taken by an alien woman married to a Filipino citizen in order to
acquire Philippine citizenship, the procedure followed in the Bureau of Immigration is as follows: The
alien woman must file a petition for the cancellation of her alien certificate of registration alleging,
among other things, that she is married to a Filipino citizen and that she is not disqualified from
acquiring her husband's citizenship pursuant to Section 4 of Commonwealth Act No. 473, as amended.
Upon the filing of the said petition, which should be accompanied or supported by the joint affidavit of
the petitioner and her Filipino husband to the effect that the petitioner does not belong to any of the
groups disqualified by the cited Section from becoming naturalized Filipino citizen, the Bureau of
Immigration conducts an investigation and thereafter promulgates its order or decision granting or
denying the petition.

REYES, J.B.L., J., dissenting:

POLITICAL LAW; CITIZENSHIP; NATURALIZATION; ALIEN WOMAN MARRIED TO FILIPINO MUST PROVE
QUALIFICATIONS UNDER SECTION 3. — Our naturalization law separates qualifications from
disqualifications; the positive qualifications under Section 3 thereof express a policy of restriction as to
candidates for naturalization as much as the disqualifications under Section 4. And it has been shown in
our decision in the second Ly Giok Ha case (Ly Giok Ha vs. Galang, L-21332 March 18, 1966, 16 SCRA 416)
that those not disqualified under Section 4 would not necessarily qualify under Section 3, even if the
residence qualification were disregarded. In other words, by giving to Section 15 of our Naturalization
Law the effect of excluding only those women suffering from disqualification under Section 3 could
result in admitting to citizenship woman that Section 2 intends to exclude. In these circumstances, I do

317
not see why American interpretation of the words who might herself be lawfully naturalized should be
considered hinding in this jurisdiction.

DECISION

BARREDO, J p:

Appeal from the following decision of the Court of First Instance of Manila in its Civil Case No. 49705
entitled Moy Ya Lim Yao, etc., et al. vs. The Commissioner of Immigration which, brief as it is, sufficiently
depicts the factual setting of and the fundamental issues involved in this case thus:

"In the instant case, petitioners seek the issuance of a writ of injunction against the Commissioner of
Immigration, 'restraining the latter and/or his authorized representative from ordering plaintiff Lau Yuen
Yeung to leave the Philippines and causing her arrest and deportation and the confiscation of her bond,
upon her failure to do so.'

"The prayer for preliminary injunction embodied in the complaint, having been denied, the case was
heard on the merits and the parties submitted their respective evidence.

"The facts of the case, as substantially and correctly stated by the Solicitor General are these:

'On February 8, 1961, Lau Yuen Yeung applied for a passport visa to enter the Philippines as a non-
immigrant. In the interrogation made in connection with her application for a temporary visitor's visa to
enter the Philippines, she stated that she was a Chinese residing at Kowloon, Hongkong, and that she
desired to take a pleasure trip to the Philippines to visit her great (grand) uncle Lau Ching Ping for a
period of one month (Exhibits '1,' '1-a,' and '2'). She was permitted to come into the Philippines on
March 13, 1961, and was permitted to stay for a period of one month which would expire on April 13,
1961. On the date of her arrival, Asher Y, Cheng filed a bond in the amount of P1,000.00 to undertake,
among others, that said Lau Yuen Yeung would actually depart from the Philippines on or before the
expiration of her authorized period of stay in this country or within the period as in his discretion the
Commissioner of Immigration or his authorized representative might properly allow. After repeated
extensions, petitioner Lau Yuen Yeung was allowed to stay in the Philippines up to February 13, 1962
(Exhibit '4'). On January 25, 1962, she contracted marriage with Moy Ya Lim Yao alias Edilberto
Aguinaldo Lim an alleged Filipino citizen. Because of the contemplated action of respondent to
confiscate her bond and order her arrest and immediate deportation, after the expiration of her
authorized stay, she brought this action for injunction with preliminary injunction. At the hearing which
took place one and a half years after her arrival, it was admitted that petitioner Lau Yuen Yeung could
not write either English or Tagalog. Except for a few words, she could not speak either English or
Tagalog. She could not name any Filipino neighbor, with a Filipino name except one, Rosa. She did not
know the names of her brothers-in-law, or sisters-in-law.'

"Under the facts unfolded above, the Court is of the considered opinion, and so holds, that the instant
petition for injunction cannot be sustained for the same reasons set forth in the Order of this Court,
dated March 19, 1962, the pertinent portions of which read:

318
'First, Section 15 of the Revised Naturalization Law provides:

"'Effect of the naturalization on wife and children. — Any woman who is now or may hereafter be
married to a citizen of the Philippines, and who might herself be lawfully naturalized shall be deemed a
citizen of the Philippines."

The above-quoted provision is clear and its import unequivocal and hence it should be held to mean
what it plainly and explicitly expresses in unmistakable terms. The clause 'who might herself be lawfully
naturalized' incontestably implies that an alien woman may be deemed a citizen of the Philippines by
virtue of her marriage to a Filipino citizen only if she possesses all the qualifications and none of the
disqualifications specified in the law, because these are the explicit requisites provided by law for an
alien to be naturalized. (Lee Suan Ay, Alberto Tan and Lee Chiao vs. Emilio Galang, etc., G. R. No. L-
11855). However, from the allegation of paragraph 3 of the complaint, to wit:

"'3. That plaintiff Lau Yuen Yeung, Chinese by birth, who might herself be lawfully naturalized as a
Filipino citizen (not being disqualified to become such by naturalization), is a Filipino citizen by virtue of
her marriage on January 25, 1962 to plaintiff MOY YA LIM YAO alias EDILBERTO AGUINALDO LIM, under
the Naturalization Laws of the Philippines."

it can be deduced beyond debate that petitioner Lau Yuen Yeung while claiming not to be disqualified,
does not and cannot allege that she possesses all the qualifications to be naturalized, naturally because,
having been admitted as a temporary visitor only on March 13, 1961, it is obvious at once that she lacks
at least, the requisite length of residence in the Philippines (Revised Naturalization Law, Sec. 2, Case No.
2, Sec. 3, Case No. 3).

'Were if the intention of the law that the alien woman, to be deemed a citizen of the Philippines by
virtue of marriage to a Filipino citizen, need only be not disqualified under the Naturalization Law, it
would have been worded "and who herself is not disqualified to become a citizen of the Philippines."

'Second, Lau Yuen Yeung, a temporary Chinese woman visitor, whose authorized stay in the Philippines,
after repeated extensions thereof, was to expire last February 28, 1962, having married her co-plaintiff
only on January 25, 1962, or just a little over one month before the expiry date of her stay, it is evident
that said marriage was effected merely for convenience to defeat or avoid her then impending
compulsory departure, not to say deportation. This cannot be permitted.

'Third, as the Solicitor General has well stated:

"'5. That petitioner Lau Yuen Yeung, having been admitted as a temporary alien visitor on the
strength of a deliberate and voluntary representation that she will enter and stay only for a period of
one month and thereby secured a visa, cannot go back on her representation to stay permanently
without first departing from the Philippines as she had promised." (Chung Tiao Bing, et al. vs.
Commissioner of Immigration, G. R. No. L-9966, September 29, 1956; Ong Se Lun vs. Board of
Commissioners, G. R. No. L-6017, September 16, 1954; Sec. 9, last par., Phil. Immigration Law).

319
The aforequoted argument of the Solicitor General is well buttressed, not only by the decided cases of
the Supreme Court on the point mentioned above, but also on the very provisions of Section 9, sub-
paragraph (g) of the Philippine Immigration Act of 1940 which reads:

" 'An alien who is admitted as a non-immigrant cannot remain in the Philippines permanently. To obtain
permanent admission, a non-immigrant alien must depart voluntarily to some foreign country and
procure from the appropriate Philippine Consul the proper visa and thereafter undergo examination by
the Officers of the Bureau of Immigration at a Philippine port of entry for determination of his
admissibility in accordance with the requirements of this Act. (This paragraph is added by Republic Act
503).'" (Sec. 9, subparagraph (g) of the Philippine Immigration Act of 1940).

'And fourth, respondent Commissioner of Immigration is charged with the administration of all laws
relating to immigration (Sec. 3, Com. Act No. 613) and in the performance of his duties in relation to
alien immigrants, the law gives the Commissioner of Immigration a wide discretion, a quasi-judicial
function in determining cases presented to him (Pedro Uy So vs. Commissioner of Immigration CA-G. R.
No. 23336-R, Dec 15, 1960), so that his decision thereon may not be disturbed unless he acted with
abuse of discretion or in excess of his jurisdiction.'

"It may also be not amiss to state that wife Lau Yuen Yeung, while she barely and insufficiently talk in
broken Tagalog and English, she admitted that she cannot write either language."

The only matter of fact not clearly passed upon by His Honor which could have some bearing in the
resolution of this appeal is the allegation in the brief of petitioners-appellants, not denied in the
government's brief, that "in the hearing . . . , it was shown thru the testimony of the plaintiff Lau Yuen
Yeung that she does not possess any of the disqualifications for naturalization." Of course, as an
additional somehow relevant factual matter, it is also emphasized by said appellants that during the
hearing in the lower court, held almost ten months after the alleged marriage of petitioners, "Lau Yuen
Yeung was already carrying in her womb for seven months a child by her husband."

Appellants have assigned six errors allegedly committed by the court a quo, thus:

THE LOWER COURT ERRED IN HOLDING THAT THE CLAUSE 'WHO MIGHT HERSELF BE LAWFULLY
NATURALIZED' (OF SECTION 15, REVISED NATURALIZATION LAW) INCONTESTABLY IMPLIES THAT AN
ALIEN WOMAN MAY BE DEEMED A CITIZEN OF THE PHILIPPINES BY VIRTUE OF HER MARRIAGE TO A
FILIPINO CITIZEN, ONLY IF SHE POSSESSES ALL THE QUALIFICATIONS AND NONE OF THE
DISQUALIFICATIONS SPECIFIED IN THE LAW.

II

THE LOWER COURT ERRED IN HOLDING THAT A WOMAN FOREIGNER WHO DOES NOT POSSESS ANY OF
THE DISQUALIFICATIONS FOR CITIZENSHIP AND WHO MARRIED A FILIPINO CITIZEN IS STILL CONSIDERED
AN ALIEN EVEN AFTER SUCH MARRIAGE AS TO FALL WITHIN THE REQUIREMENT OF SECTION 9, SUB-
PARAGRAPH (9) OF THE PHILIPPINE IMMIGRATION ACT OF 1940.

320
III

THE COURT ERRED IN CONCLUDING THAT LAU YUEN YEUNG'S MARRIAGE TO A FILIPINO CITIZEN WAS
ONLY FOR CONVENIENCE, MERELY BECAUSE THE SAME WAS CELEBRATED JUST OVER A MONTH BEFORE
THE EXPIRY DATE OF HER AUTHORIZED STAY.

IV

THE LOWER COURT ERRED IN FAILING TO FIND THAT THE COMMISSIONER OF IMMIGRATION ACTED
WITH ABUSE OF DISCRETION OR IN EXCESS OF HIS JURISDICTION WHEN SAID OFFICER THREATENED TO
SEND OUT OF THE COUNTRY PLAINTIFF LAU YUEN YEUNG WITH WARNING THAT HER FAILURE TO DO SO
WOULD MEAN CONFISCATION OF HER BOND, ARREST AND IMMEDIATE DEPORTATION, IN SPITE OF THE
FACT THAT LAU YUEN YEUNG IS NOW A FILIPINO CITIZEN.

THE LOWER COURT ERRED IN DISMISSING PLAINTIFFS-APPELLANTS' COMPLAINT AND IN REFUSING TO


PERMANENTLY ENJOIN THE COMMISSIONER FROM ORDERING PLAINTIFF LAU YUEN YEUNG TO LEAVE
THE PHILIPPINES AS A TEMPORARY VISITOR WHICH SHE IS NOT.

VI

THE LOWER COURT ERRED IN REFUSING TO GRANT PLAINTIFFS-APPELLANTS' MOTION FOR


PRELIMINARY INJUNCTION EMBODIED IN THEIR COMPLAINT, IN AN ORDER DATED MARCH 19, 1962.
(PAGES 36-41, RECORD ON APPEAL).

We need not discuss these assigned errors separately. In effect, the above decision upheld the two main
grounds of objection of the Solicitor General to the petition in the court below, viz:

"That petitioner Lau Yuen Yeung, having been admitted as a temporary alien visitor on the strength of a
deliberate and voluntary representation that she will enter and stay only for a period of one month and
thereby secured a visa, cannot go back on her representation to stay permanently without first
departing from the Philippines as she had promised (Chung Tiao Bing, et al. vs. Commissioner of
Immigration, G.R. No. L-9966, September 29, 1956; Ong Se Lun vs. Board of Commissioners, G.R. No. L-
6017, Sept. 16, 1954, Sec. 9, last par. Phil. Immigration Law);

"That the mere marriage of a Filipino citizen to an alien does not automatically confer on the latter
Philippine citizenship. The alien wife must possess all the qualifications required by law to become a
Filipino citizen by naturalization and none of the disqualifications. (Lee Suan Ay, Alberto Tan and Lee
Chiao vs. Galang, etc., G. R. No. L-11855, Dec. 25, 1959)"

It is obvious from the nature of these objections that their proper resolution would necessarily cover all
the points raised in appellants' assignments of error, hence, We will base our discussions, more or less,
on said objections.

I.

321
The first objection of the Solicitor General which covers the matters dealt with in appellants' second and
fourth assignments of error does not require any lengthy discussion. As a matter of fact, it seems
evident that the Solicitor General's pose that an alien who has been admitted into the Philippines as a
non-immigrant cannot remain here permanently unless he voluntarily leaves the country first and goes
to a foreign country to secure thereat from the appropriate Philippine consul the proper visa and
thereafter undergo examination by officers of the Bureau of Immigration at a Philippine port of entry for
determination of his admissibility in accordance with the requirements of the Philippine Immigration Act
of 1940, as amended by Republic Act 503, is premised on the assumption that petitioner Lau Yuen Yeung
is not a Filipino citizen. We note the same line of reasoning in the appealed decision of the court a quo.
Accordingly, it is but safe to assume that were the Solicitor General and His Honor of the view that said
petitioner had become ipso facto a Filipina by virtue of her marriage to her Filipino husband, they would
have held her as entitled to assume the status of a permanent resident without having to depart as
required of aliens by Section 9(g) of the law.

In any event, to set this point at rest, We hereby hold that portion of Section 9(g) of the Immigration Act
providing:

"An alien who is admitted as a non-immigrant cannot remain in the Philippines permanently. To obtain
permanent admission, a non-immigrant alien must depart voluntarily to some foreign country and
procure from the appropriate Philippine consul the proper visa and thereafter undergo examination by
the officers of the Bureau of Immigration at a Philippine port of entry for determination of his
admissibility in accordance with the requirements of this Act."

does not apply to aliens who after coming into the Philippines as temporary visitors, legitimately
become Filipino citizens or acquire Filipino citizenship. Such change of nationality naturally bestows
upon them the right to stay in the Philippines permanently or not, as they may choose, and if they elect
to reside here, the immigration authorities may neither deport them nor confiscate their bonds. True it
is that this Court has vehement]y expressed disapproval of convenient ruses employed by aliens to
convert their status from temporary visitors to permanent residents in circumvention of the procedure
prescribed by the legal provision already mentioned, such as in Chiong Tiao Bing vs. Commissioner of
Immigration, 99 Phil. 1020, wherein, thru Mr. Justice J.B.L. Reyes, the Court, reiterating the ruling in Ong
Se Lun vs. Board of Immigration Commissioners, 95 Phil. 785, said:

". . . It is clear that if an alien gains admission to the Islands on the strength of a deliberate and voluntary
representation that he will enter only for a limited time, and thereby secures the benefit of a temporary
visa, the law will not allow him subsequently to go back on his representation and stay permanently,
without first departing from the Philippines as he had promised. No officer can relieve him of the
departure requirements of section 9 of the Immigration Act, under the guise of 'change' or 'correction',
for the law makes no distinctions, and no officer is above the law. Any other ruling would, as stated in
our previous decision, encourage aliens to enter the Islands on false pretences; every alien so permitted
to enter for a limited time, might then claim a right to permanent admission, however flimsy such claim
should be, and thereby compel our government to spend time, money and effort to examining and
verifying whether or not every such alien really has a right to take up permanent residence here. In the

322
meanwhile, the alien would be able to prolong his stay and evade his return to the port whence he
came, contrary to what he promised to do when he entered. The damages inherent in such ruling are
self-evident."

On the other hand, however, We cannot see any reason why an alien who has been here as a temporary
visitor but who has in the meanwhile become a Filipino should be required to still leave the Philippines
for a foreign country, only to apply thereat for a re-entry here and undergo the process of showing that
he is entitled to come back, when after all, such right has become incontestible as a necessary
concomitant of his assumption of our nationality by whatever legal means this hag been conferred upon
him. Consider, for example, precisely the case of the minor children of an alien who is naturalized. It is
indubitable that they become ipso facto citizens of the Philippines. Could it be the law that before they
can be allowed permanent residence, they still have to be taken abroad so that they may be processed
to determine whether or not they have a right to have permanent residence here? The difficulties and
hardships which such a requirement entails and its seeming unreasonableness argue against such a
rather absurd construction. Indeed, as early as 1957, in Ly Giok Ha vs. Galang, 101 Phil. 459, Mr. Justice
Concepcion, our present Chief Justice, already ruled thus:

". . . (P)etitioners allege that, upon her marriage to a Filipino, Ly Giok Ha became also a citizen of the
Philippines. Indeed, if this conclusion were correct, it would follow that, in consequence of her marriage,
she had been naturalized as such citizen, and, hence the decision appealed from would have to be
affirmed, for section 40(c) of Commonwealth Act 613 provides that 'in the event of the naturalization as
a Philippine citizen . . . of the alien on whose behalf the bond deposit is given, the bond shall be
cancelled or the be deposited shall be returned to the depositor or his legal representative.'" (At. pp.
462-463) In other words, the applicable statute itself more than implies that the naturalization of an
alien visitor as a Philippine citizen logically produces the effect of conferring upon him ipso facto all the
rights of citizenship including that of being entitled to permanently stay in the Philippines outside the
orbit of authority of the Commissioner of Immigration vis-a-vis aliens, if only because by its very nature
and express provisions, the Immigration Law is a law only for aliens and is inapplicable to citizens of the
Philippines. In the sense thus discussed, therefore, appellants' second and fourth assignments of error
are well taken.

II.

Precisely, the second objection of the Solicitor General sustained by the trial judge is that appellant Lau
Yuen Yeung's marriage to appellant Moya Lim Yao alias Edilberto Aguinaldo whose Filipino citizenship is
not denied did not have the effect of making her a Filipino, since it has not been shown that she "might
herself be lawfully naturalized," it appearing clearly in the record that she does not possess all the
qualifications required of applicants for naturalization by the Revised Naturalization Law,
Commonwealth Act 473, even if she has proven that she does not suffer from any of the
disqualifications thereunder. In other words, the Solicitor General implicitly concedes that had it been
established in the proceedings below that appellant Lau Yuen Yeung possesses all the qualifications
required by the law of applicants for naturalization, she would have been recognized by the respondent

323
as a Filipino citizen in the instant case, without requiring her to submit to the usual proceedings for
naturalization.

To be sure, this position of the Solicitor General is in accord with what used to be the view of this Court
since Lee Suan Ay, et al. v. Emilio Galang, etc., et al., G.R. No. L-11855, promulgated December 23, 1959,
106 Phil., 706, 713,1 for it was only in Zita Ngo Burca vs. Republic, G.R. No. L-24252 which was
promulgated on January 30, 1967 (19 SCRA 186), that over the pen of Mr. Justice Conrado Sanchez, this
Court held that for an alien woman who marries a Filipino to be deemed a Filipina, she has to apply for
naturalization in accordance with the procedure prescribed by the Revised Naturalization Law and prove
in said naturalization proceeding not only that she has all the qualifications and none of the
disqualifications provided in the law but also that she has complied with all the formalities required
thereby like any other applicant for naturalization, 2 albeit said decision is not yet part of our
jurisprudence inasmuch as the motion for its reconsideration is still pending resolution. Appellants are in
effect urging Us, however, in their first and second assignments of error, not only to reconsider Burca
but to even reexamine Lee Suan Ay which, as a matter of fact, is the prevailing rule, having been
reiterated in all subsequent decisions up to Go Im Ty. 3

Actually, the first case in which Section 15 of the Naturalization Law, Commonwealth Act 473,
underwent judicial construction was in the first Ly Giok Ha case, 4 one almost identical to the one at bar.
Ly Giok Ha, a woman of Chinese nationality, was a temporary visitor here whose authority to stay was to
expire on March 14, 1956. She filed a bond to guaranty her timely departure. On March 8, 1956, eight
days before the expiration of her authority to stay, she married a Filipino by the name of Restituto
Lacasta. On March 9, 1956, her husband notified the Commissioner of Immigration of said marriage and,
contending that his wife had become a Filipina by reason of said marriage, demanded for the
cancellation of her bond, but instead of acceding to such request, the Commissioner required her to
leave, and upon her failure to do so, on March 16, 1956, the Commissioner confiscated her bond; a suit
was filed for the recovery of the bond; the lower court sustained her contention that she had no
obligation to leave because she had become Filipina by marriage, hence her bond should be returned.
The Commissioner appealed to this Court. In the said appeal, Mr. Justice Roberto Concepcion, our
present Chief Justice, spoke for the Court, thus:

"The next and most important question for determination is whether her marriage to a Filipino justified
or, at least, excused the aforesaid failure of Ly Giok Ha to depart from the Philippines on or before
March 14, 1956. In maintaining the affirmative view, petitioners alleged that, upon her marriage to a
Filipino, Ly Giok Ha became, also, a citizen of the Philippines. Indeed, if this conclusion were correct, it
would follow that, in consequence of her marriage, she had been naturalized as such citizen, and, hence,
the decision appealed from would have to be affirmed, for section 40(c) of Commonwealth Act No. 613
provides that 'in the event of the naturalization as a Philippine citizen . . . of the alien on whose behalf
the bond deposit is given, the bond shall be cancelled or the sum deposited shall be returned to the
depositor or his legal representative." Thus the issue boils down to whether an alien female who marries
a male citizen of the Philippines follows ipso facto his political status.

"The pertinent part of section 15 of Commonwealth Act No. 473, upon which petitioners rely, reads:

324
'Any woman who is now or may hereafter be married to a citizen of the Philippines, and who might
herself be lawfully naturalized shall be deemed a citizen of the Philippines.'

"Pursuant thereto, marriage to a male Filipino does not vest Philippine citizenship to his foreign wife,
unless she 'herself may be lawfully naturalized.' As correctly held in an opinion of the Secretary of
Justice (O.p. No. 52, series of 1950), * this limitation of section 15 excludes, from the benefits of
naturalization by marriage, those disqualified from being naturalized as citizens of the Philippines under
section 4 of said Commonwealth Act No. 473, namely:

'(a) Persons opposed to organized government or affiliated with any association or group of persons
who uphold and teach doctrines opposing all organized governments;

'(b) Persons defending or teaching the necessity or propriety of violence, personal assault, or
assassination for the success and predominance of their ideas;

'(c) Polygamists or believers in the practice of polygamy;

'(d) Persons convicted of crimes involving moral turpitude;

'(e) Persons suffering from mental alienation or incurable contagious diseases;

'(f) Persons who, during the period of their residence in the Philippines, have not mingled socially
with the Filipinos, or who have not evinced a sincere desire to learn and embrace the customs,
traditions, and ideals of the Filipinos;

'(g) Citizens or subjects of nations with whom the . . . Philippines are at war, during the period of
such war;

'(h) Citizens or subjects of a foreign country other than the United States, whose laws does not grant
Filipinos the right to become naturalized citizens or subjects thereof.'

"In the case at bar, there is neither proof nor allegation in the pleadings that Ly Giok Ha does not fall
under any of the classes disqualified by law. Moreover, as the parties who claim that, despite her failure
to depart from the Philippines within the period specified in the bond in question, there has been no
breach thereof, petitioners have the burden of proving her alleged change of political status, from alien
to citizen. Strictly speaking, petitioners have not made out, therefore a case against the respondents-
appellants.

"Considering, however, that neither in the administrative proceedings, nor in the lower court, had the
parties seemingly felt that there was an issue on whether Ly Giok Ha may 'be lawfully naturalized,' and
this being a case of first impression in our courts, we are of the opinion that, in the interest of equity
and justice, the parties herein should be given an opportunity to introduce evidence, if they have any,
on said issue." (At pp. 462-464.).

As may be seen, although not specifically in so many words, no doubt was left in the above decision as
regards the following propositions:

325
1. That under Section 15 of Commonwealth Act 473, the Revised Naturalization Law, the marriage
of an alien woman to a Filipino makes her a Filipina, if she "herself might be lawfully naturalized";

2. That this Court declared as correct the opinion of the Secretary of Justice that the limitation of
Section 15 of the Naturalization Law excludes from the benefits of naturalization by marriage, only those
disqualified from being naturalized under Section 4 of the law quoted in the decision;

3. That evidence to the effect that she is not disqualified may be presented in the action to recover
her bond confiscated by the Commissioner of Immigration;

4. That upon proof of such fact, she may be recognized as Filipina; and

5. That in referring to the disqualifications enumerated in the law, the Court somehow left the
impression that no inquiry need be made as to qualifications, 5 specially considering that the decision
cited and footnoted several opinions of the Secretary of Justice, the immediate superior of the
Commissioner of Immigration, the most important of which are the following:

"Paragraph (a), section 13 of Act No. 2927, as amended, (now section 15, Commonwealth Act No. 473),
provided that 'any woman who is now or may hereafter be married to a citizen of the Philippines, and
who might herself be lawfully naturalized shall be deemed a citizen of the Philippines.' A similar
provision in the naturalization law of the United States has been construed as not requiring the woman
to have the qualifications of residence, good character, etc., as in the case of naturalization by judicial
proceedings, but merely that she is of the race of persons who may be naturalized. (Kelly v. Owen [Dist.
Col. 1868] 7 Wall 496, 5F, 11, 12; ex parte Tryason [D. C. Wash. 1914] 215 F. 449, 27 Op. Atty. Gen. 507).
(Op. No. 168, s. 1940 of Justice Sec. Jose Abad Santos.)

"In a previous opinion rendered for your Office, I stated that the clause 'who might herself be lawfully
naturalized', should be construed as not requiring the woman to have the qualifications of residence,
good character, etc., as in cases of naturalization by judicial proceedings, but merely that she is of the
race of persons who may be naturalized. (Op. No. 79, s. 1940)

"Inasmuch as the race qualification has been removed by the Revised Naturalization Law, it results that
any woman who married a citizen of the Philippines prior to or after June 17, 1939, and the marriage not
having been dissolved, and on the assumption that she possesses none of the disqualifications
mentioned in Section 4 of Commonwealth Act No. 473, follows the citizenship of her husband." (Op. No.
176, v. 1940 of Justice Sec. Jose Abad Santos.)

"From the foregoing narration of facts, it would seem that the only material point of inquiry is as to the
citizenship of Arce Machura. If he shall be found to be a citizen of the Philippines, his wife, Mrs. Lily
James Machura, shall likewise be deemed a citizen of the Philippines pursuant to the provision of
Section 15, Commonwealth Act No. 473, which reads in part as follows:

'Any woman who is now or may hereafter be married to a citizen of the Philippines, and who might
herself be lawfully naturalized shall be deemed a citizen of the Philippines.'

326
"The phrase 'who might herself be lawfully naturalized', as contained in the above provision, means that
the woman who is married to a Filipino citizen must not belong to any of the disqualified classes
enumerated in Section 4 of the Naturalization Law (Ops., Sec. of Jus., No. 28, s. 1950; No. 43, s. 1948,
No. 95, s. 1941: Nos. 79 and 168, s. 1940). Under the facts stated in the within papers, Mrs. Machura
does not appear to be among the disqualified classes mentioned in the law.

"It having been shown that Arce Machura or Arsenio Guevara was born as an illegitimate of a Filipino
mother, he should be considered as a citizen of the Philippines in consonance with the well-settled rule
that an illegitimate child follows the citizenship of his only legally recognized parent, the mother (Op.,
Sec. of Jus., Nos. 58, 98 & 281, s. 1948; No. 96, s. 1949). Her husband being a Filipino, Mrs. Machura
must necessarily be deemed as a citizen of the Philippines by marriage (Sec. 15, Com. Act No. 473.) (Op.
No. 52, s. 1950 of Justice Sec. Ricardo Nepomuceno.)

The logic and authority of these opinions, compelling as they are, must have so appealed to this Court
that five days later, on May 22, 1957, in Ricardo Cua v. The Board of Commissioners, 101 Phil. 521, Mr.
Justice J.B.L. Reyes, reiterated the same ruling on the basis of the following facts:

Tjioe Wu Suan, an Indonesian, arrived in Manila on November 1, 1952, but it turned out that her
passport was forged. On December 10, 1953, a warrant was issued for her arrest for purposes of
deportation. Later, on December 20, 1953, she married Ricardo Cua, a Filipino, and because of said
marriage, the Board of Special Inquiry considered her a Filipina. Upon a review of the case, however, the
Board of Immigration Commissioners insisted on continuing with the deportation proceedings and so,
the husband filed prohibition and mandamus proceedings. The lower court denied the petition.
Although this Court affirmed said decision, it held, on the other hand, that:

"Granting the validity of marriage, this Court has ruled in the recent case of Ly Giok Ha v. Galang, supra,
p. 459, that the bare fact of a valid marriage to a citizen does not suffice to confer his citizenship upon
the wife. Section 15 of the Naturalization Law requires that the alien woman who marries a Filipino must
show, in addition, that she 'might herself be lawfully naturalized' as a Filipino citizen. As construed in the
decision cited, this last condition requires proof that the woman who married a Filipino is herself not
disqualified under section 4 of the Naturalization Law.

"No such evidence appearing on record, the claim of assumption of Filipino citizenship by Tjioe Wu Suan,
upon her marriage to petitioner, is untenable. The lower court, therefore, committed no error in
refusing to interfere with the deportation proceedings, where she can anyway establish the requisites
indispensable for her acquisition of Filipino citizenship, as well as the alleged validity of her Indonesian
passport." (Ricardo Cua v. The Board of Immigration Commissioners, G. R. No. L-9997, May 22, 1957,
101 Phil. 521, 523.) [Emphasis supplied]

For emphasis, it is reiterated that in the above two cases, this Court expressly gave the parties
concerned opportunity to prove the fact that they were not suffering from any of the disqualifications of
the law without the need of undergoing any judicial naturalization proceeding. It may be stated,
therefore, that according to the above decisions, the law in this country, on the matter of the effect of
marriage of an alien woman to a Filipino is that she thereby becomes a Filipina, if it can be proven that

327
at the time of such marriage, she does not possess any of the disqualifications enumerated in Section 4
of the Naturalization Law, without the need of submitting to any naturalization proceedings under said
law.

It is to be admitted that both of the above decisions made no reference to qualifications, that is, as to
whether or not they need also to be proved, but, in any event, it is a fact that the Secretary of Justice
understood them to mean that such qualifications need not be possessed nor proven. Then Secretary of
Justice Jesus Barrera, who later became a distinguished member of this Court, 6 so ruled in opinions
rendered by him subsequent to Ly Giok Ha, the most illustrative of which held:

"At the outset it is important to note that an alien woman married to a Filipino citizen needs only to
show that she 'might herself be lawfully naturalized' in order to acquire Philippine citizenship.
Compliance with other conditions of the statute, such as those relating to the qualifications of an
applicant for naturalization through judicial proceedings, is not necessary (See: Leonard v. Grant, 5 Fed.
11; 27 Ops. Atty. Gen [U.S.] 507; Ops Sec. of Justice, No. 776, s. 1940, and No. 111, s. 1953.

"This view finds support in the case of Ly Giok Ha et al. v. Galang et al., G.R. No. L-10760, promulgated
May 17, 1957, where the Supreme Court, construing the abovequoted section of the Naturalization Law,
held that 'marriage to a male Filipino does not vest Philippine citizenship to his foreign wife, unless she
'herself may he lawfully naturalized,' and that 'this limitation of Section 15 excludes, from the benefits of
naturalization by marriage, those disqualified from being naturalized as citizens of the Philippines under
Section 4 of said Commonwealth Act No. 473.' In other words, disqualification for any of the causes
enumerated in Section 4 of the Act is the decisive factor that defeats the right of the foreign wife of a
Philippine citizen to acquire Philippine citizenship.

xxx xxx xxx

"Does petitioner, Lim King Bian, belong to any of these groups ? The Commissioner of Immigration does
not say so but merely predicates his negative action on the ground that a warrant of deportation for
'overstaying' is pending against the petitioner.

"We do not believe the position is well taken. Since the grounds for disqualification for naturalization
are expressly enumerated in the law, a warrant of deportation not based on a finding of unfitness to
become naturalized for any of those specified causes may not be invoked to negate acquisition of
Philippine citizenship by a foreign wife of a Philippine citizen under Section 15 of the Naturalization Law.
(Inclusio unius est exclusio alterius)" (Op. No. 12, s. 1958 of Justice Undersec. Jesus G. Barrera.)

"Regarding the steps that should be taken by an alien woman married to a Filipino citizen in order to
acquire Philippine citizenship, the procedure followed in the Bureau of Immigration is as follows: The
alien woman must file a petition for the cancellation of her alien certificate of registration alleging,
among other things, that she is married to a Filipino citizen and that she is not disqualified from
acquiring her husband's citizenship pursuant to section 4 of Commonwealth Act No. 473, as amended.
Upon the filing of said petition, which should be accompanied or supported by the joint affidavit of the
petitioner and her Filipino husband to the effect that the petitioner does not belong to any of the groups

328
disqualified by the cited section from becoming naturalized Filipino citizen (please see attached CEB
Form 1), the Bureau of Immigration conducts an investigation and thereafter promulgates its order or
decision granting or denying the petition." (Op. No. 38, B. 1958 of Justice Sec. Jesus G. Barrera.)

"This view finds support in the case of Ly Giok Ha et al., v. Galang et al. (G.R. No. L-10760, promulgated
May 17, 1957), where the Supreme Court, construing the above-quoted section in the Revised
Naturalization Law, held that 'marriage to a male Filipino does not vest Philippine citizenship to his
foreign wife, unless she 'herself may be lawfully naturalized,' and that 'this limitation of Section 15
excludes from the benefits of naturalization by marriage those disqualified from being naturalized as
citizens of the Philippines under Section 4 of said Commonwealth Act No. 473.' In other words,
disqualification for any of the causes enumerated in section 4 of the Act is the decisive factor that
defeats the right of an alien woman married to a Filipino citizen to acquire Philippine citizenship." (Op.
57, s. 1958 of Justice Sec. Jesus G. Barrera.)

"The contention is untenable. The doctrine enunciated in the Ly Giok Ha case is not a new one. In that
case, the Supreme Court held that under paragraph 1 of Section 15 of Commonwealth Act No. 473,
'marriage to a male Filipino does not vest Philippine citizenship to his foreign wife unless she "herself
may be lawfully naturalized"', and, quoting several earlier opinions of the Secretary of Justice, namely:
No. 52, s. 1950; No. 168, s. 1940; No. 95, s. 1941; No. 63, s. 1948; No. 28, s. 1950, 'this limitation of
section 15 excludes from the benefits of naturalization by marriage, those disqualified from being
naturalized as citizens of the Philippines under section 4 of said Commonwealth Act No. 473." (Op. 134,
B. 1962 of Justice Undersec. Magno S. Gatmaitan.)

It was not until more than two years later that, in one respect, the above construction of the law was
importantly modified by this Court in Lee Suan Ay, supra, in which the facts were as follows:

"Upon expiration of the appellant Lee Suan Ay's authorized period of temporary stay in the Philippines
(25 March 1955), on 26 March 1955 the Commissioner of Immigration asked the bondsman to present
her to the Bureau of Immigration within 24 hours from receipt of notice, otherwise the bond will be
confiscated (Annex 1). For failure of the bondsman to comply with the foregoing order, on 1 April 1955
the Commissioner of Immigration ordered the cash bond confiscated (Annex E). Therefore, there was an
order issued by the Commissioner of Immigration confiscating or forfeiting the cash bond. Unlike in
forfeiture of bail bonds in criminal proceedings, where the Court must enter an order forfeiting the bail
bond and the bondsman must be given an opportunity to present his principal or give a satisfactory
reason for his inability to do so, before final judgment may be entered against the bondsman, (section
15, Rule 110; U.S. v. Bonoan, 22 Phil. 1.) in forfeiture of bonds posted for the temporary stay of an alien
in the Philippines, no court proceeding is necessary. Once a breach of the terms and conditions of the
undertaking in the bond is committed, the Commissioner of Immigration may, under the terms and
conditions thereof, declare it forfeited in favor of the Government." (In the meanwhile, on April 1, 1955,
Lee Suan Ay and Alberto Tan, a Filipino, were joined in marriage by the Justice of the Peace of Las Piñas,
Rizal.)

329
Mr. Justice Sabino Padilla speaking for a unanimous court which included Justices Concepcion and Reyes
who had penned Ly Giok Ha and Ricardo Cua, ruled thus:

"The fact that Lee Suan Ay (a Chinese) was married to a Filipino citizen does not relieve the bondsman
from his liability on the bond. The marriage took place on 1 April 1955, and the violation of the terms
and conditions of; the undertaking in the bond — failure to depart from the Philippines upon expiration
of her authorized period of temporary stay in the Philippines (25 March 1955) and failure to report to
the Commissioner of Immigration within 24 hours from receipt of notice — were committed before the
marriage. Moreover, the marriage of a Filipino citizen to an alien does not automatically confer
Philippine citizenship upon the latter. She must possesses the qualifications required by law to become a
Filipino citizen by naturalization. ** There is no showing that the appellant Lee Suan Ay possesses all the
qualifications and none of the disqualifications provided for by law to become a Filipino citizen by
naturalization."

Pertinently to be noted at once in this ruling, which, to be sure, is the one relied upon in the appealed
decision now before Us, is the fact that the footnote of the statement therein that the alien wife "must
possess the qualifications required by law to become a Filipino citizen by naturalization" makes
reference to Section 15, Commonwealth Act 473 and precisely, also to Ly Giok Ha v. Galang, supra. As
will be recalled, on the other hand, in the opinions of the Secretary of Justice explicitly adopted by the
Court in Ly Giok Ha, among them, Opinion No. 176, Series of 1940, above-quoted, it was clearly held
that "(I)n a previous opinion rendered for your Office, I stated that the clause 'who might herself be
lawfully naturalized', should be construed as not requiring the woman to have the qualifications of
residence, good character, etc., as in cases of naturalization by judicial proceedings, but merely that she
is of the race by persons who may be naturalized." (Op. Na. 79, s. 1940)

Since Justice Padilla gave no reason at all for the obviously significant modification of the construction of
the law, it could be said that there was need for clarification of the seemingly new posture of the Court.
The occasion for such clarification should have been in Kua Suy, etc., et al. vs. The Commissioner of
Immigration, G.R. No. L-13790, October 31, 1963, penned by Mr. Justice J.B.L. Reyes, who had rendered
the opinion in Ricardo Cua, supra, which followed that in Ly Giok Ha, supra, but apparently seeing no
immediate relevancy in the case on hand then of the particular point in issue now, since it was not
squarely raised therein similarly as in Lee Suan Ay, hence, anything said on the said matter would at best
be no more than obiter dictum, Justice Reyes limited himself to holding that "Under Section 15 of the
Naturalization Act, the wife is deemed a citizen of the Philippines only if she 'might herself be lawfully
naturalized,' so that the fact of marriage to a citizen, by itself alone, does not suffice to confer
citizenship, as this Court has previously ruled in Ly Giok Ha v. Galang, 54 O.G. 356, and in Cua v. Board of
Immigration Commissioners, 53 O.G. 8567; and there is here no evidence of record as to the
qualifications or absence of disqualifications of appellee Kua Suy", without explaining the apparent
departure already pointed out from Ly Giok Ha and Ricardo Cua. Even Justice Makalintal, who wrote a
separate concurring and dissenting opinion merely lumped together Ly Giok Ha, Ricardo Cua and Lee
Suan Ay and opined that both qualifications and non-disqualifications have to be shown without
elucidating on what seemed to be departure from the said first two decisions.

330
It was only on November 30, 1963 that to Mr. Justice Roberto Regala fell the task of rationalizing the
Court's position. In La San Tuang v. Galang, G.R. No. L-18775, November 30, 1963, 9 SCRA 638, the facts
were simply these: 10 San Tuang, a Chinese woman, arrived in the Philippines on July 1, 1960 as a
temporary visitor with authority to stay up to June 30, 1961. She married a Filipino on January 7, 1961,
almost six months before the expiry date at her permit, and when she was refused to leave after her
authority to stay had expired, she refused to do so, claiming she had become a Filipina by marriage, and
to bolster her position, she submitted an affidavit stating explicitly that she does not possess any of the
disqualifications enumerated in the Naturalization Law, Commonwealth Act 473. When the case
reached the court, the trial judge held for the government that in addition to not having any of the
disqualifications referred to, there was need that Lo San Tuang should have also possessed all the
qualifications of residence, moral character, knowledge of a native principal dialect, etc., provided by
the law. Recognizing that the issue squarely to be passed upon was whether or not the possession of all
the qualifications were indeed needed to be shown apart from non-disqualification, Justice Regala held
affirmatively for the Court, reasoning out thus:

"It is to be noted that the petitioner has anchored her claim for citizenship on the basis of the decision
laid down in the case of Leonard v. Grant, 5 Swy. 603, 5 F 11, where the Circuit Court of Oregon held
that it was only necessary that the woman 'should be a person of the class or race permitted to be
naturalized by existing laws, and that in respect of the qualifications arising out of her conduct or
opinions, being the wife of a citizen, she is to be regarded as qualified for citizenship, and therefore
considered a citizen.' (In explanation of its conclusion, the Court said: 'If, whenever during the life of the
woman or afterwards, the question of her citizenship arises in a legal proceeding, the party asserting her
citizenship by reason of her marriage with a citizen must not only prove such marriage, but also that the
woman then possessed all the further qualifications necessary to her becoming naturalized under
existing laws, the statute will be practically nugatory, if not a delusion and a snare. The proof of the facts
may have existed at the time of the marriage, but years after, when a controversy arises upon the
subject, it may be lost or difficult to find.')

"In other words, all that she was required to prove was that she was a free white woman or a woman of
African descent or nativity, in order to be deemed an American citizen, because, with respect to the rest
of the qualifications on residence, moral character, etc., she was presumed to be qualified.

"Like the law in the United States, our former Naturalization Law (Act No. 2927, as amended by Act No.
3448) specified the classes of persons who alone might become citizens of the Philippines, even as it
provided who were disqualified. Thus, the pertinent provisions of that law provided:

'Section 1. Who may become Philippine citizens. — Philippine citizenship may be acquired by (a)
natives of the Philippines who are not citizens thereof under the Jones Law; (b) natives of the Insular
possessions of the United States; (c) citizens of the United States, or foreigners who under the laws of
the United States may become citizens of said country if residing therein.

'Section 2. Who are disqualified. — The following cannot be naturalized as Philippine citizens: (a)
Persons opposed to organized government or affiliated with any association or group of persons who

331
uphold and teach doctrines opposing all organized government; (b) persons defending or teaching the
necessity or propriety of violence, personal assault or assassination for the success and predominance of
their ideas; (c) polygamists or believers in the practice of polygamy; (d) persons convicted of crimes
involving moral turpitude; (e) persons suffering from mental alienation or incurable contagious diseases;
(f) citizens or subjects of nations with whom the United States and the Philippines are at war, during the
period of such war.

'Section 3. Qualifications. — The persons comprised in subsection (a) of section one of this Act, in
order to be able to acquire Philippine citizenship, must be not less than twenty-one years of age on the
day of the hearing of their petition.

'The persons comprised in subsections (b) and (c) of said section one shall, in addition to being not less
than twenty-one years of age on the day of the hearing of the petition, have all and each of the
following qualifications:

'First. Residence in the Philippine Islands for a continuous period of not less than five years, except as
provided in the next following section;

'Second. To have conducted themselves in a proper and irreproachable manner during the entire Period
of their residence in the Philippine Islands, in their relation with the constituted government as well as
with the community in which they are living;

'Third. To hold in the Philippine Islands real estate worth not less than one thousand pesos, Philippine
currency, or have some known trade or profession; and

'Fourth. To speak and write English, Spanish, or some native tongue.

'In case the petitioner is a foreign subject, he shall, besides, declare in writing and under oath his
intention of renouncing absolutely and perpetually all faith and allegiance to the foreign authority, state
or sovereignty of which he was a native, citizen or subject.'

"Applying the interpretation given by Leonard v. Grant, supra, to our law as it then stood, alien women
married to citizens of the Philippines must, in order to be deemed citizens of the Philippines, be either
(1) natives of the Philippines who were not citizens thereof under the Jones Law, or (2) natives of other
Insular possessions of the United States, or (3) citizens of the United States or foreigners who under the
laws of the United States might become citizens of that country if residing therein. With respect to the
qualifications set forth in Section 3 of the former law, they were deemed to have the same for all intents
and purposes.

"But, with the approval of the Revised Naturalization Law (Commonwealth Act No. 473) on June 17,
1939, Congress has since discarded class or racial consideration from the qualifications of applicants for
naturalization (according to its proponent, the purpose in eliminating this consideration was, first, to
remove the features of the existing naturalization act which discriminated in favor of the Caucasian} and
against Asiatics who are our neighbors, and are related to us by racial affinity and, second, to foster
amity with all nations [Sinco, Phil. Political Law 502 — 11 ed.]), even as it retained in Section 15 the

332
phrase in question. The result is that the phrase 'who might herself be lawfully naturalized' must be
understood in the context in which it is now found, in a setting so different from that in which it was
found by the Court in Leonard v. Grant.

"The only logical deduction from the elimination of class or racial consideration is that, as the Solicitor
General points out, the phrase 'who might herself be lawfully naturalized' must now be understood as
referring to those who under Section 2 of the law are qualified to become citizens of the Philippines.

"There is simply no support for the view that the phrase 'who might herself be lawfully naturalized' must
now be understood as requiring merely that the alien woman must not belong to the class of
disqualified persons under Section 4 of the Revised Naturalization Law. Such a proposition misreads the
ruling laid down in Leonard v. Grant. A person who is not disqualified is not necessarily qualified to
become a citizen of the Philippines, because the law treats 'qualifications' and 'disqualifications' in
separate sections. And then it must not be lost sight of that even under the interpretation given to the
former law, it was to be understood that the alien woman was not disqualified under Section 2 of that
law. Leonard v. Grant did not rule that it was enough if the alien woman does not belong to the class of
disqualified persons in order that she may be deemed to follow the citizenship of her husband: What
that case held was that the phrase 'who might herself be lawfully naturalized, merely means that she
belongs to the class or race of persons qualified to become citizens by naturalization — the assumption
being always that she is not otherwise disqualified.

"We therefore hold that under the first paragraph of Section 15 of the Naturalization Law, an alien
woman, who is married to a citizen of the Philippines, acquires the citizenship of her husband only if she
has all the qualifications and none of the disqualifications provided by law. Since there is no proof in this
case that petitioner has all the qualifications and is not in any way disqualified, her marriage to a Filipino
citizen does not automatically make her a Filipino citizen. Her affidavit to the effect that she is not in any
way disqualified to become a citizen of this country was correctly disregarded by the trial court, the
same being self-serving."

Naturally, almost a month later in Sun Peck Yong V. Commissioner of Immigration, G.R. No L-20784,
December 27, 1963, 9 SCRA 875, wherein the Secretary of Foreign Affairs reversed a previous resolution
of the preceding administration to allow Sun Peck Yong and her minor son to await the taking of the
oath of Filipino citizenship of her husband two years after the decision granting him nationalization and
required her to leave and this order was contested in court, Justice Barrera held:

"In the case of Lo San Tuang v. Commissioner of Immigration (G.R. No. L-18775, promulgated November
30, 1963; Kua Suy vs. Commissioner of Immigration, L-13790, promulgated October 31, 1963), we held
that the fact that the husband became a naturalized citizen does not automatically make the wife a
citizen of the Philippines. It must also be shown that she herself possesses all the qualifications, and
none of the disqualifications, to become a citizen. In this case, there is no allegation, much less showing,
that petitioner-wife is qualified to become a Filipino citizen herself. Furthermore, the fact that a decision
was favorably made on the naturalization petition of her husband is no assurance that he (the husband)
would become a citizen, as to make a basis for the extension of her temporary stay."

333
On the same day, in Tong Siok Sy v. Vivo, G.R. No. L-21136, December 27, 1963, 9 SCRA 876, Justice
Barrera reiterated the same ruling and citing particularly Lo San Tuang and Kua Suy, held that the
marriage of Tong Siok Sy to a Filipino on November 12, 1960 at Taichung, Taiwan and her taking oath of
Filipino citizenship before the Philippine Vice Consul at Taipeh, Taiwan on January 6, 1961 did not make
her a Filipino citizen, since she came here only in 1961 and obviously, she had not had the necessary
ten-year residence in the Philippines required by the law.

Such then was the status of the jurisprudential law on the matter under discussion when Justice
Makalintal sought a reexamination thereof in Choy King Tee v. Galang, G.R. No. L-18351, March 26,
1965, 13 SCRA 402. Choy King Tee's husband was granted Philippine citizenship on January 13, 1959 and
took the oath on January 31 of the same year, Choy King Tee first came to the Philippines in 1955 and
kept commuting between Manila and Hongkong since then, her last visa before the case being due to
expire on February 14, 1961. On January 27, 1961, her husband asked the Commissioner of Immigration
to cancel her alien certificate of registration, as well as their child's, for the reason that they were
Filipinos, and when the request was denied as to the wife, a mandamus was sought, which the trial
court granted. Discussing anew the issue of the need for qualifications, Justice Makalintal not on]y
reiterated the arguments of Justice Regala in Lo San Tuang but added further that the ruling is believed
to be in line with the national policy of selective admission to Philippine citizenship. 7

No wonder, upon this authority, in Austria v. Conchu, G.R. No. L-20716, June 22, 1965, 14 SCRA 336,
Justice J.P. Bengzon readily reversed the decision of the lower court granting the writs of mandamus and
prohibition against the Commissioner of Immigration, considering that Austria's wife, while admitting
she did not possess all the qualifications for naturalization, had submitted only an affidavit that she had
none of the disqualifications therefor. So also did Justice Dizon similarly hold eight days later in Brito v.
Commissioner, G.R. No. L-16829, June 30, 1965, 14 SCRA 539.

Then came the second Ly Giok Ha case 8 wherein Justice J. B. L. Reyes took occasion to expand on the
reasoning of Choy King Tee by illustrating with examples "the danger of relying exclusively on the
absence of disqualifications, without taking into account the other affirmative requirements of the law."
9

Lastly, in Go Im Ty v. Republic, G.R. No. L-17919, decided on July 30, 1966, 10 Justice Zaldivar held for
the Court that an alien woman who is widowed during the pendency of the naturalization proceedings
of her husband, in order that she may be allowed to take the oath as Filipino, must, aside from proving
compliance with the requirements of Republic Act 530, show that she possesses all the qualifications
and does not suffer from any of the disqualifications under the Naturalization Law, citing in the process
the decision to such effect discussed above, 11 even as he impliedly reversed pro tanto the ruling in Tan
Lin v. Republic, G.R. No. L-13786, May 31, 1961, 2 SCRA 383.

Accordingly, in Burca, Justice Sanchez premised his opinion on the assumption that the point now under
discussion is settled law.

In the case now at bar, the Court is again called upon to rule on the same issue. Under Section 15 of the
Naturalization Law, Commonwealth Act 473, providing that:

334
"SEC. 15. Effect of the naturalization on wife and children. — Any woman, who is now or may
hereafter be married to a citizen of the Philippines, and who might herself be lawfully naturalized shall
be deemed a citizen of the Philippines.

"Minor children of persons naturalized under this law who have been born in the Philippines shall be
considered citizens thereof.

"A foreign-born minor child, if dwelling in the Philippines at the time of the naturalization of the parent,
shall automatically become a Philippine citizen, and a foreign-born child, who is not in the Philippines at
the time the parent is naturalized, shall be deemed a Philippine citizen only during his minority, unless
he begins to reside permanently in the Philippines when still a minor, in which case, he will continue to
be a Philippine citizen even after becoming of age.

"A child born outside of the Philippines after the naturalization of his parent, shall be considered a
Philippine citizen, unless within one year after reaching the age of majority he fails to register himself as
a Philippine citizen at the American Consulate of the country where he resides, and to take the
necessary oath of allegiance.

is it necessary, in order that an alien woman who marries a Filipino or who is married to a man who
subsequently becomes a Filipino, may become a Filipino citizen herself, that, aside from not suffering
from any of the disqualifications enumerated in the law, she must also possess all the qualifications
required by said law? If nothing but the unbroken line from Lee Suan Ay to Go Im Ty, as recounted
above, were to be considered, it is obvious that an affirmative answer to the question would be
inevitable, specially, if it is noted that the present case was actually submitted for decision on January
21, 1964 yet, shortly after Lo San Tuang, Tong Siok Sy and Sun Peck Yong, all supra, and even before
Choy King Tee, supra, were decided. There are other circumstances, however, which make it desirable, if
not necessary, that the Court take up the matter anew. There has been a substantial change in the
membership of the Court since Go Im Ty, and of those who were in the Court already when Burca was
decided, two members, Justice Makalintal and Castro concurred only in the result, precisely, according
to them, because they wanted to leave the point now under discussion open in so far as they are
concerned. 12 Truth to tell, the views and arguments discussed at length with copious relevant
authorities, in the motion for reconsideration as well as in the memorandum of the amici curiae 13 in
the Burca case cannot just be taken lightly and summarily ignored, since they project in the most
forceful manner, not only the legal and logical angles of the issue, but also the imperative practical
aspects thereof in the light of the actual situation of the thousands of alien wives of Filipinos who have
so long, even decades, considered themselves as Filipinas and have always lived and acted as such,
officially or otherwise, relying on the long standing continuous recognition of their status as such by the
administrative authorities in charge of the matter, as well as by the courts. Under these circumstances,
and if only to afford the Court an opportunity to consider the views of the five justices who took no part
in Ga Im Ty (including the writer of this opinion), the Court decided to further reexamine the matter.
After all, the ruling first laid in Lee Suan Ay, and later in Lo San Tuang, Choy King Tee and the second
(1966) Ly Giok Ha, did not categorically repudiate the opinions of the Secretary of Justice relied upon by

335
the first (1959) Ly Giok Ha. Besides, some points brought to light during the deliberations in this case
would seem to indicate that the premises of the later cases can still bear further consideration.

Whether We like it or not, it is undeniably factual that the legal provision We are construing, Section 15,
aforequoted, of the Naturalization Law has been taken directly, copied and adopted from its American
counterpart. To be more accurate, said provision is nothing less than a reenactment of the American
provision. A brief review of its history proves this beyond per adventure of doubt.

The first Naturalization Law of the Philippines approved by the Philippine Legislature under American
sovereignty was that of March 26, 1920, Act No. 2927. Before then, as a consequence of the Treaty of
Paris, our citizenship laws were found only in the Organic Laws, the Philippine Bill of 1902, the Act of the
United States Congress of March 23, 1912 and later the Jones Law of 1916. In fact, Act No. 2927 was
enacted pursuant to express authority granted by the Jones Law. For obvious reasons, the Philippines
gained autonomy on the subjects of citizenship and immigration only after the effectivity of the
Philippine Independence Act. This made it practically impossible for our laws on said subject to have any
perspective or orientation of our own; everything was American.

The Philippine Bill of 1902 provided pertinently:

"SECTION 4. That all inhabitants of the Philippine Islands continuing to reside therein who were
Spanish subjects on the eleventh day of April, eighteen-hundred and ninety-nine, and then resided in
said Islands, and their children born subsequent thereto, shall be deemed and held to be citizens of the
Philippine Islands and as such entitled to the protection of the United States, except such as shall have
elected to preserve their allegiance to the Crown of Spain in accordance with the provisions of the treaty
of peace between the United States and Spain signed at Paris December tenth, eighteen hundred and
ninety-eight."

This Section 4 of the Philippine Bill of 1902 was amended by Act of Congress of March 23, 1912, by
adding a provision as follows:

"Provided, That the Philippine Legislature is hereby authorized to provide by law for the acquisition of
Philippine citizenship by those natives of the Philippine Islands who do not come within the foregoing
provisions, the natives of other insular possessions of the United States, and such other persons residing
in the Philippine Islands who would become citizens of the United States, under the laws of the United
States, if residing therein."

The Jones Law reenacted these provisions substantially:

"SECTION 2. That all inhabitants of the Philippine Islands who were Spanish subjects on the eleventh
day of April, eighteen hundred and ninety-nine, and then resided in said islands, and their children born
subsequent thereto, shall be deemed and held to be citizens of the Philippine Islands, except such as
shall have elected to preserve their allegiance to the Crown of Spain in accordance with the provisions of
the treaty of peace between the United States and Spain, signed at Paris December tenth, eighteen
hundred and ninety-eight and except such others as have since become citizens of some other country:

336
Provided, That the Philippine Legislature, herein provided for, is hereby authorized to provide by law for
the acquisition of Philippine citizenship by those natives of the Philippine Islands who do not come
within the foregoing provisions, the natives of the insular possessions of the United States, and such
other persons residing in the Philippine Islands who are citizens of the United States under the laws of
the United States if residing therein."

For aught that appears, there was nothing in any of the said organic laws regarding the effect of
marriage to a Filipino upon the nationality of an alien woman, albeit under the Spanish Civil Code
provisions on citizenship, Articles 17 to 27, which were, however, abrogated upon the change of
sovereignty, it was unquestionable that the citizenship of the wife always followed that of the husband.
Not even Act 2927 contained any provision regarding the effect of naturalization of an alien upon the
citizenship of his alien wife, nor of the marriage of such alien woman with a native born Filipino or one
who had become a Filipino before the marriage, although Section 13 thereof provided thus:

"SEC. 13. Right of widow and children of petitioners who have died. — In case a petitioner should
die before the final decision has been rendered, his widow and minor children may continue the
proceedings. The decision rendered in the case shall, so far as the widow and minor children are
concerned, produce the same legal effect as if it had been rendered during the life of the petitioner."

It was not until November 30, 1928, upon the approval of Act 3448, amending Act 2977, that the
following provisions were added to the above Section 13:

"SECTION 1. The following new sections are hereby inserted between sections thirteen and fourteen
of Act Numbered Twenty-nine hundred and Twenty-seven:

'SEC. 13 (a). Any woman who is now or may hereafter be married to a citizen of the Philippine
Islands and who might herself be lawfully naturalized, shall be deemed a citizen of the Philippine Islands.

'SEC. 13 (b). Children of persons who have been duly naturalized under this law, being under the age
of twenty-one years at the time of the naturalization of their parents, shall, if dwelling in the Philippine
Islands, be considered citizens thereof.

'SEC. 13 (c). Children of persons naturalized under this law who have been born in the Philippine
Islands after the naturalization of their parents shall be considered citizens thereof.' "

When Commonwealth Act 473, the current naturalization law, was enacted on June 17, 1939, the above
Section 13 became its Section 15 which has already been quoted earlier in this decision. As can be seen,
Section 13(a) abovequoted was re-enacted practically word for word in the first paragraph of this
Section 15 except for the change of Philippine Islands to Philippines. And it could not have been on any
other basis than this legislative history of our naturalization law that each and everyone of the decisions
of this Court from the first Ly Giok Ha to Go Im Ty, discussed above, were rendered.

As stated earlier, in the opinion of Chief Justice Concepcion in the first Ly Giok Ha, it was quite clear that
for an alien woman who marries a Filipino to become herself a Filipino citizen, there is no need for any
naturalization proceeding because she becomes a Filipina ipso facto from the time of such marriage,

337
provided she does not suffer any of the disqualifications enumerated in Section 4 of Commonwealth Act
473, with no mention being made of whether or not the qualifications enumerated in Section 2 thereof
need be shown. It was only in Lee Suan Ay in 1959 that the possession of qualifications were specifically
required, but it was not until 1963, in Lo San Tuang, that Justice Regala reasoned out why the possession
of the qualifications provided by the law should also be shown to be possessed by the alien wife of a
Filipino, for her to become a Filipina by marriage.

As may be recalled, the basic argument advanced by Justice Regala was briefly as follows: That "like the
law in the United States, our Naturalization Law specified the classes of persons who alone might
become citizens, even as it provided who were disqualified," and inasmuch as Commonwealth Act 473,
our Naturalization Law since 1939 did not reenact the section providing who might become citizens,
allegedly in order to remove racial discrimination in favor of Caucasians and against Asiatics, "the only
logical deduction . . . is that the phrase 'who might herself be lawfully naturalized' must now be
understood as referring to those who under Section 2 of the law are qualified to become citizens of the
Philippines" and "there is simply no support for the view that the phrase 'who might herself be lawfully
naturalized' must now be understood as requiring merely that the alien woman must not belong to the
class of disqualified persons under Section 4 of the Revised Naturalization Law." 14

A similar line of reasoning was followed in Choy King Tee, which for ready reference may be quoted:

"The question has been settled by the uniform ruling of this Court in a number of cases. The alien wife of
a Filipino citizen must first prove that she has all the qualifications required by Section 2 and none of the
disqualifications enumerated in Section 4 of the Naturalization Law before she may he deemed a
Philippine citizen (Lao Chay v. Galang, L-19977, Oct. 30, 1964, citing Lo San Tuang v. Galang, L-18775,
Nov. 30, 1963; Sun Peck Yong v. Commissioner of Immigration, L-20784, December 27, 1963; Tong Siok
Sy v. Vivo, L-21136, December 27, 1963). The writer of this opinion has submitted the question anew to
the court for a possible reexamination of the said ruling in the light of the interpretation of a similar law
in the United States after which Section 15 of our Naturalization Law was patterned. That law was
section 2 of the Act of February 10, 1855 (Section 1994 of the Revised Statutes of the U.S.). The local
law, Act No. 3448, was passed on November 30, 1928 as an amendment to the former Philippine
Naturalization Law, Act No. 2927, which was approved on March 26, 1920. Under this Naturalization
Law, acquisition of Philippine citizenship was limited to three classes of persons, (a) Natives of the
Philippines who were not citizens thereof; (b) natives of the other insular possessions of the United
States; and (c) citizens of the United States, or foreigners who, under the laws of the United States, may
become citizens of the latter country if residing therein. The reference in subdivision (c) to foreigners
who may become American Citizens is restrictive in character, for only persons of certain specified races
were qualified thereunder. In other words, in so far as racial restrictions were concerned there was at
the time a similarity between the naturalization laws of the two countries, and hence there was reason
to accord here persuasive force to the interpretation given in the United States to the statutory
provision concerning the citizenship of alien women marrying American citizens.

"This Court, however, believes that such reason has ceased to exist since the enactment of the Revised
Naturalization Law (Commonwealth Act No. 473) on June 17, 1939. The racial restrictions have been

338
eliminated in this Act, but the provision found in Act No. 3448 has been maintained. It is logical to
presume that when Congress chose to retain the said provision — that to be deemed a Philippine citizen
upon marriage the alien wife must be one 'who might herself be lawfully naturalized,' the reference is
no longer to the class or race to which the woman belongs, for class or race has become immaterial, but
to the qualifications and disqualifications for naturalization as enumerated in Sections 2 and 4 of the
statute. Otherwise the requirement that the woman 'might herself be lawfully naturalized' would be
meaningless surplusage, contrary to settled norms of statutory construction.

"The rule laid down by this Court in this and in other cases heretofore decided is believed to be in line
with the national policy of selective admission to Philippine citizenship, which after all is a privilege
granted only to those who are found worthy thereof, and not indiscriminately to anybody at all on the
basis alone of marriage to a man who is a citizen of the Philippines, irrespective of moral character,
ideological beliefs, and identification with Filipino ideals, customs and traditions.

"Appellee here having failed to prove that she has all the qualifications for naturalization, even, indeed,
that she has none of the disqualifications, she is not entitled to recognition as a Philippine citizen."

In the second Ly Giok Ha, the Court further fortified the arguments in favor of the same conclusion thus:

"On cross-examination, she (Ly Giok Ha) failed to establish that: (1) she has been residing in the
Philippines for a continuous period of at least (10) years (p. 27, t.s.n., id.); (2) she has a lucrative trade,
profession, or lawful occupation (p. 13. t.s.n., id.); and (3) she can speak and write English, or any of the
principal Philippine languages (pp. 12, 13, t.s.n., id.)

"While the appellant Immigration Commissioner contends that the words emphasized indicate that the
present Naturalization Law requires that an alien woman who marries a Filipino husband must possess
the qualifications prescribed by section 2 in addition to not being disqualified under any of the eight ('a'
to 'h') subheadings of section 4 of Commonwealth Act No. 473, in order to claim our citizenship by
marriage, both the appellee and the court below (in its second decision) sustain the view that all that
the law demands is that the woman be not disqualified under section 4.

"At the time the present case was remanded to the court of origin (1960) the question at issue could be
regarded as not conclusively settled, there being only the concise pronouncement in Lee Suan Ay, et al.
v. Galang, G. R. No. L-11855, Dec. 23, 1959, to the effect that:

'The marriage of a Filipino citizen to an alien does not automatically confer Philippine citizenship upon
the latter. She must possess the qualifications required by law to become a Filipino citizen by
naturalization.'

"Since that time, however, a long line of decisions of this Court has firmly established the rule that the
requirement of section 15 of Commonwealth Act 473 (the Naturalization Act), that an alien woman
married to a citizen should be one who 'might herself be lawfully naturalized," means not only woman
free from the disqualifications enumerated in section 4 of the Act but also one who possesses the
qualifications prescribed by section 2 of Commonwealth Act 473 (San Tuan v. Galang, L-18775, Nov. 30,

339
1963; Sun Peck Yong v. Com. of Immigration, L-20784, Dec. 27, 1963; Tong Siok Sy v. Vivo, L-21136, Dec.
27, 1963; Austria v. Conchu, L-20716, June 22, 1965; Choy King Tee v. Galang, L-18351, March 26, 1965;
Brito v. Com. of Immigration, L-16829, June 30, 1965).

"Reflection will reveal why this must be so. The qualifications prescribed under section 2 of the
Naturalization Act, and the disqualifications enumerated in its section 4 are not mutually exclusive; and
if all that were to be required is that the wife of a Filipino be not disqualified under section 4, the result
might well be that citizenship would be conferred upon persons in violation of the policy of the statute.
For example, section 4 disqualifies only —

'(c) Polygamists or believers in the practice of polygamy; and

'(d) Persons convicted of crimes involving moral turpitude,'

so that a blackmailer, or a maintainer of gambling or bawdy houses, not previously convicted by a


competent court would not be thereby disqualified; still, it is certain that the law did not intend such
person to be admitted as a citizen in view of the requirement of section 2 that an applicant for
citizenship 'must be of good moral character.'

"Similarly, the citizen's wife might be a convinced believer in racial supremacy, in government by certain
selected classes, in the right to vote exclusively by certain 'herrenvolk', and thus disbelieve in the
principles underlying the Philippine Constitution; yet she would not be disqualified under section 4, as
long as she is not 'opposed to organized government,' nor affiliated to groups 'upholding or teaching
doctrines opposing all organized governments', nor 'defending or teaching the necessity or propriety of
violence, personal assault or assassination for the success or predominance of their ideas.' Et sic de
caeteris.

"The foregoing instances should suffice to illustrate the danger of relying exclusively on the absence of
disqualifications, without taking into account the other affirmative requirements of the law, which, in
the case at bar, the appellee Ly Giok Ha admittedly does not possess.

"As to the argument that the phrase 'might herself be lawfully naturalized' was derived from the U.S.
Revised Statutes (section 1994) and should be given the same territorial and racial significance given to
it by American courts, this Court has rejected the same in Lon San Tuang v. Galang, L-18775, November
30, 1963; and in Choy King Tee v. Galang, L-18351, March 26, 1965."

It is difficult to minimize the persuasive force of the foregoing rationalizations, but a closer study thereof
cannot but reveal certain relevant considerations which adversely affect the premises on which they are
predicated, thus rendering the conclusions arrived thereby not entirely unassailable.

1. The main proposition, for instance, that in eliminating Section 1 of Act 2927 providing who are
eligible for Philippine citizenship, the purpose of Commonwealth Act 473, the Revised Naturalization
Law, was to remove the racial requirements for naturalization, thereby opening the door of Filipino
nationality to Asiatics instead of allowing the admission thereto of Caucasians only, suffers from lack of

340
exact accuracy. It is important to note, to start with, that Commonwealth Act 473 did away with the
whole Section 1 of Act 2927 which reads thus:

"SECTION 1. Who may become Philippines citizens. — Philippine citizenship may be acquired by: (a)
natives of the Philippines who are not citizens thereof under the Jones Law; (b) natives of the other
Insular possessions of the United States; (c) citizens of the United States, or foreigners who under the
laws of the United States may become citizens of said country if residing therein."

and not only subdivision (c) thereof. Nowhere in this whole provision was there any mention of race or
color of the persons who were then eligible for Philippine citizenship. What is more evident from said
provision is that it reflected the inevitable subordination of our legislation during the pre-
Commonwealth American regime to the understandable limitations flowing from our status as a
territory of the United States by virtue of the Treaty of Paris. In fact, Section 1 of Act 2927 was precisely
approved pursuant to express authority, without which it could not have been done, granted by an
amendment to Section 4 of the Philippine Bill of 1902 introduced by the Act of the United States
Congress of March 23, 1912 and which was reenacted as part of the Jones Law of 1916, the pertinent
provisions of which have already been quoted earlier. In truth, therefore, it was because of the
establishment of the Philippine Commonwealth and in the exercise of our legislative autonomy on
citizenship matters under the Philippine Independence Act that Section 1 of Act 2927 was eliminated, 15
and not purposely to eliminate any racial discrimination contained in our Naturalization Law. The
Philippine Legislature naturally wished to free our Naturalization Law from the impositions of American
legislation. In other words, the fact that such discrimination was removed was one of the effects rather
than the intended purpose of the amendment.

2. Again, the statement in Choy King Tee to the effect that "the reference in subdivision (c) (of
Section 1 of Act 2927) to foreigners who may become American citizens is restrictive in character, for
only persons of certain specified races were qualified thereunder" fails to consider the exact import of
the said subdivision. Explicitly, the thrust of the said subdivision was to confine the grant under it of
Philippine citizenship only to the three classes of persons therein mentioned, the third of which were
citizens of the United States and, corollarily, persons who could be American citizens under her laws.
The words used in the provision do not convey any idea of favoring aliens of any particular race or color
and of excluding others, but more accurately, they refer to all the disqualifications of foreigners for
American citizenship under the laws of the United States. The fact is that even as of 1906, or long before
1920, when our Act 2927 became a law, the naturalization laws of the United States already provided
for the following disqualifications in the Act of the Congress of June 29, 1906:

"SEC. 7. That no person who disbelieves in or who is opposed to organized government, or who is a
member of or affiliated with any organization entertaining and teaching such disbelief in or opposition
to organized government, or who advocates or teaches the duty, necessity, or propriety of the unlawful
assaulting or killing of any officer or officers, either of specific individuals or of officers generally, of the
Government of the United States, or of any other organized government, because of his or their official
character, or who is a polygamist, shall be naturalized or be made a citizen of the United States."

341
and all these disqualified persons were, therefore, ineligible for Philippine citizenship under Section 1 of
Act 2927 even if they happened to be Caucasians. More importantly, as a matter of fact, said American
law, which was the first "Act to Establish a Bureau of Immigration and Naturalization and to Provide for
a Uniform Rule for Naturalization of Aliens throughout the United States" contained no racial
disqualification requirement, except as to Chinese, the Act of May 6, 1882 not being among those
expressly repealed by this law, hence it is clear that when Act 2927 was enacted, subdivision (c) of its
Section 1 could not have had any connotation of racial exclusion necessarily, even if it were traced back
to its origin in the Act of the United States Congress of 1912 already mentioned above. 16 Thus, it would
seem that the nationalization in the quoted decisions predicated on the theory that the elimination of
Section 1 of Act 2927 by Commonwealth Act 473 was purposely for no other end than the abolition of
racial discrimination in our naturalization law has no clear factual basis. 17

3. In view of these considerations, there appears to be no cogent reason, why the construction
adopted in the opinions of the Secretary of Justice referred to in the first Ly Giok Ha decision of the Chief
Justice should not prevail. It is beyond dispute that the first paragraph of Section 15 of Commonwealth
Act 473 is a reenactment of Section 13(a) of Act 2927, as amended by Act 3448, and that the latter is
nothing but an exact copy, deliberately made, of Section 1994 of the Revised Statutes of the United
States as it stood before it repeal in 1922. 18 Before such repeal, the phrase "who might herself be
lawfully naturalized" found in said Section 15 had a definite unmistakable construction uniformly
followed in all courts of the United States that had occasion to apply the same and which, therefore,
must be considered as if it were written in the statute itself. It is almost trite to say that when our
legislators enacted said section, they knew of its unvarying construction in the United States and that,
therefore, in adopting verbatim the American statute, they have in effect incorporated into the
provision, as thus enacted, the construction given to it by the American courts as well as the Attorney
General of the United States and all administrative authorities charged with the implementation of the
naturalization and immigration laws of that country. (Lo Cham v. Ocampo, 77 Phil., 635 [1946];
Laxamana v. Baltazar, 92 Phil., 32 [1952]; Hartley v. Commissioner, 295 U.S. 216, 79 L. ed. 1399, 55 S Ct.
756 [1935]; Helvering v. Windmill, 305 U.S. 79, 83 L ed. 52, 59 S Ct. 45 [1938]; Helvering v. R. J. Reynolds
Tobacco Co., 306 U.S. 110, 83 L ed. 536, 59 S Ct. 423 [1939]. [p. 32, Memo of Amicus Curiae]).

A fairly comprehensive summary of the said construction by the American courts and administrative
authorities is contained in United Stats of America ex rel. Dora Sejnensky v. Robert E. Tod, Commissioner
of Immigration, Appt., 285 Fed. 523, decided November 14, 1922, 26 A. L. R. 1316 as follows:

"Section 1994 of the Revised Statutes (Comp. Stat. § 3948, 2 Fed. Sta. Anno. 2d ed. p. 117) provides as
follows: 'Any woman who is now or may hereafter be married to a citizen of the United States, and who
might herself be lawfully naturalized, shall be deemed a citizen.'

"Section 1944 of the Revised Stat. is said to originate in the Act of Congress of February 10, 1855 (10
Stat. at L. 604, chap. 71), which in its second section provided 'that any woman, who might lawfully be
naturalized under the existing laws, married, or who shall be married to a citizen of the United States,
shall be deemed and taken to be a citizen.'

342
"And the American Statute of 1855 is substantially a copy of the earlier British Statute 7 & 8 Vict. chap.
66, . . . 16, 1844, which provided that 'any woman married, or who shall be married, to a natural-born
subject or person naturalized, shall be deemed and taken to be herself naturalized, and have all the
rights and privileges of a natural born subject.'

"The Act of Congress of September 22, 1922 (42 Stat. at L. 1021, chap. 411, Comp. Stat. § 4358b, Fed.
Stat. Anno. Supp. 1922, p. 255), being 'An Act Relative to the Naturalization and Citizenship of Married
Women,' in § 2, provides 'that any woman who marries a citizen of the United States after the passage
of this Act, . . . shall not become a citizen of the United States by reason of such marriage . . .'

"Section 6 of the act also provides 'that . . . 1994 of the Revised Statutes . . . are repealed.'

"Section 6 also provides that 'such repeal shall not terminate citizenship acquired or retained under
either of such sections, . . .' meaning § § 2 and 6. So that this Act of September 22, 1922, has no
application to the facts of the present case, as the marriage of the relator took place prior to its passage.
This case, therefore, depends upon the meaning to be attached to § 1994 of the Revised Statutes.

"In 1868 the Supreme Court, in Kelly v. Owen, 7 Wall. 496, 498, 19 L. ed. 283, 284, construed this
provision as found in the Act of 1855 as follows: 'The term, "who might lawfully be naturalized under the
existing laws," only limits the application of the law to free white women. The previous Naturalization
Act, existing at the time, only required that the person applying for its benefits should be "a free white
person," and not an alien enemy.'

"This construction limited the effect of the statute to those aliens who belonged to the class or race
which might be lawfully naturalized, and did not refer to any of the other provisions of the
naturalization laws as to residence or moral character, or to any of the provisions of the immigration
laws relating to the exclusion or deportation of aliens.

"In 1880, in Leonard v. Grant (C. C.) 5 Fed. 11, District Judge Deady also construed the Act of 1855,
declaring that 'any woman who is now or may hereafter be married to a citizen of the United States, and
might herself be lawfully naturalized, shall be deemed a citizen.' He held that 'upon the authorities, and
the reason, if not the necessity, of the case,' the statute must be construed as in effect declaring that an
alien woman, who is of the class or race that may be lawfully naturalized under the existing laws, and
who marries a citizen of the United States, is such a citizen also, and it was not necessary that it should
appear affirmatively that she possessed the other qualifications at the time of her marriage to entitle
her to naturalization.

"In 1882, the Act of 1855 came before Mr. Justice Harlan, sitting in the circuit court, in United States v.
Kellar, 13 Fed. 82. An alien woman, a subject of Prussia came to the United States and married here a
naturalized citizen. Mr. Justice Harlan, with the concurrence of Judge Treat, held that upon her marriage
she became ipso facto a citizen of the United States as fully as if she had complied with all of the
provisions of the statutes upon the subject of naturalization. He added: 'There can be no doubt of this,
in view of the decision of the Supreme Court of the United States in Kelly v. Owen, 7 Wall. 496, 19 L. ed.
283.' The alien 'belonged to the class of persons' who might be lawfully naturalized.

343
"In 1904, in Hopkins v. Fachant, 65 C. C. A. 1, 130 Fed. 839, an alien woman came to the United States
from France and entered the country contrary to the immigration laws. The immigration authorities
took her into custody at the port of New York, with the view of deporting her. She applied for her
release under a writ of habeas corpus, and pending the disposition of the matter she married a
naturalized American citizen. The circuit court of appeals for the ninth circuit held, affirming the court
below, that she was entitled to be discharged from custody. The court declared: 'The rule is well settled
that her marriage to a naturalized citizen of the United States entitled her to be discharged. The status
of the wife follows that of her husband, . . . and by virtue of her marriage her husband's domicil became
her domicil.'

"In 1908, the circuit court for the district of Rhode Island in Re Rustigian, 165 Fed. 980, had before it the
application of a husband for his final decree of naturalization. It appeared that at that time his wife was
held by the immigration authorities at New York on the ground that she was afflicted with a dangerous
and contagious disease. Counsel on both sides agreed that the effect of the husband's naturalization
would be to confer citizenship upon the wife. In view of that contingency District Judge Brown declined
to pass upon the husband's application for naturalization, and thought it best to wait until it was
determined whether the wife's disease was curable. He placed his failure to act on the express ground
that the effect of naturalizing the husband might naturalize her. At the same time he expressed his
opinion that the husband's naturalization would not effect her naturalization, as she was not one who
could become lawfully naturalized. 'Her own capacity (to become naturalized),' the court stated, 'is a
prerequisite to her attaining citizenship. If herself lacking in that capacity, the married status cannot
confer it upon her.' Nothing, however, was actually decided in that case, and the views expressed
therein are really nothing more than mere dicta. But, if they can be regarded as something more than
that, we find ourselves, with all due respect for the learned judge, unable to accept them.

"In 1909, in United States ex rel. Nicola v. Williams, 173 Fed, 626, District Judge Learned Hand held that
an alien woman, a subject of the Turkish Empire, who married an American citizen while visiting Turkey,
and then came to the United States, could not be excluded, although she had, at the time of her entry, a
disease which under the immigration laws would have been sufficient ground for her exclusion, if she
had not had the status of a citizen. The case was brought into this court on appeal, and in 1911 was
affirmed, in 106 C. C. A. 464, 184 Fed. 322. In that case, however at the time the relators married, they
might have been lawfully naturalized, and we said: 'Even if we assume the contention of the district
attorney to be correct that marriage will not make a citizen of a woman who would be excluded under
our immigration laws, it does not affect these relators.'

"We held that, being citizens, they could not be excluded as aliens; and it was also said to be
inconsistent with the policy of our law that the husband should be a citizen and the wife an alien. The
distinction between that case and the one now before the court is that, in the former case, the marriage
took place before any order of exclusion had been made, while in this the marriage was celebrated after
such an order was made. But such an order is a mere administrative provision, and has not the force of a
judgment of a court, and works no estoppel. The administrative order is based on the circumstances
that existed at the time the order of exclusion was made. If the circumstances change prior to the order
being carried into effect, it cannot be executed. For example, if an order of exclusion should be based on

344
the ground that the alien was at the time afflicted with a contagious disease, and it should be made
satisfactorily to appear, prior to actual deportation, that the alien had entirely recovered from the
disease, we think it plain that the order could not be carried into effect. So, in this case, if, after the
making of the order of exclusion and while she is permitted temporarily to remain, she in good faith
marries an American citizen, we cannot doubt the validity of her marriage, and that she thereby
acquired, under international law and under § 1994 of the Revised Statutes, American citizenship, and
ceased to be an alien. There upon, the immigration authorities lost their jurisdiction over her, as that
jurisdiction applies only to aliens, and not to citizens.

"In 1910, District Judge Dodge, in Ex parte Kaprielian, 188 Fed. 694, sustained the right of the officials to
deport a woman under the following circumstances: She entered this country in July, 1910, being an
alien and having been born in Turkey. She was taken into custody by the immigration authorities in the
following September, and in October a warrant for her deportation was issued. Pending hearings as to
the validity of that order, she was paroled in the custody of her counsel. The ground alleged for her
deportation was that she was afflicted with a dangerous and contagious disease at the time of her entry.
One of the reasons assigned to defeat deportation was that the woman had married a citizen of the
United States pending the proceedings for her deportation. Judge Dodge declared himself unable to
believe that a marriage under such circumstances 'is capable of having the effect claimed, in view of the
facts shown.' He held that it was no part of the intended policy of § 1994 to annul or override the
immigration laws, so as to authorize the admission into the country of the wife of a naturalized alien not
otherwise entitled to enter, and that an alien woman, who is of a class of persons excluded by law from
admission to the United States does not come within the provisions of that section. The court relied
wholly upon the dicta contained in the Rustigian Case. No other authorities were cited.

"In 1914, District Judge Neterer, in Ex parte Grayson, 215 Fed. 449, construed § 1994 and held that
where, pending proceedings to deport an alien native of France as an alien prostitute, she was married
to a citizen of the United States, she thereby became a citizen, and was not subject to deportation until
her citizenship was revoked by due process of law. It was his opinion that if, as was contended, her
marriage was conceived in fraud, and was entered into for the purpose of evading the immigration laws
and preventing her deportation, such fact should be established in a court of competent jurisdiction in
an action commenced for the purpose. The case was appealed and the appeal was dismissed. 134 C. C.
A. 666, 219 Fed. 1022.

"It is interesting also to observe the construction placed upon the language of the statute by the
Department of Justice. In 1874, Attorney General Williams, 14 Ops. Atty. Gen. 402, passing upon the Act
of February 10, 1855, held that residence within the United States for the period required by the
naturalization laws was not necessary in order to constitute an alien woman a citizen, she having
married a citizen of the United States abroad, although she never resided in the United States, she and
her husband having continued to reside abroad after the marriage.

"In 1909, a similar construction was given to the Immigration Act of May 5, 1907, in an opinion rendered
by Attorney General Wickersham. It appeared an unmarried woman, twenty-eight years of age and a
native of Belgium, arrived in New York and went at once to a town in Nebraska, where she continued to

345
reside. About fifteen months after her arrival she was taken before a United States commissioner by
way of instituting proceedings under the Immigration Act (34 Stat. at L. 898, chap. 1134, Comp. Stat. §
4242, 3 Fed. Stat. Anno. 2d ed. p. 637) for her deportation, on the ground that she had entered this
country for the purpose of prostitution, and had been found an inmate of a house of prostitution and
practicing the same within three years after landing. It appeared, however, that after she was taken
before the United States commissioner, but prior to her arrest under a warrant by the Department of
Justice, she was lawfully married to a native-born citizen of the United States. The woman professed at
the time of her marriage an intention to abandon her previous mode of life and to remove with her
husband to his home in Pennsylvania. He knew what her mode of life had been, but professed to believe
in her good intentions. The question was raised as to the right to deport her, the claim being advance
that by her marriage she had become an American citizen and therefore could not be deported. The
Attorney General ruled against the right to deport her as she had become an American citizen. He held
that the words, 'who might herself be lawfully naturalized,' refer to a class or race who might be lawfully
naturalized, and that compliance with the other conditions of the naturalization laws was not required.
27 Ops. Atty. Gen. 507.

"Before concluding this opinion, we may add that it has not escaped our observation that Congress, in
enacting the Immigration Act of 1917, co as to provide, in § 19, 'that the marriage to an American citizen
of a female of the sexually immoral classes . . . shall not invest such female with United States citizenship
if the marriage of such alien female shall be solemnized after her arrest or after the commission of acts
which make her liable to deportation under this act.'

"Two conclusions seem irresistibly to follow from the above change in the law:

"(1) Congress deemed legislation essential to prevent women of the immoral class avoiding
deportation through the device of marrying an American citizen.

"(2) If Congress intended that the marriage of an American citizen with an alien woman of any other
of the excluded classes, either before or after her detention should not confer upon her American
citizenship, thereby entitling her to enter the country, its intention would have been expressed, and § 19
would not have been confined solely to women of the immoral class."

Indeed, We have examined all the leading American decisions on the, subject and We have found no
warrant for the proposition that the phrase "who might herself be lawfully naturalized" in Section 1994
of the Revised Status was meant solely as a racial bar, even if loose statements in some decisions and
other treaties and other writings on the subject would seem to give such impression. The case of Kelly v.
Owen, supra, which appears to be the most cited among the first of these decisions 19 simply held:

"As we construe this Act, it confers the privileges of citizenship upon women married to citizens of the
United States, if they are of the class of persons for whose naturalization the previous Acts of Congress
provide. The terms 'married' or 'who shall be married,' do not refer, in our judgment, to the time when
the ceremony of marriage is celebrated, but to a state of marriage. They mean that, whenever a woman,
who under previous Acts might be naturalized, is in a state of marriage to a citizen, whether his
citizenship existed at the passage of the Act or subsequently, or before or after the marriage, she

346
becomes, by that fact, a citizen also. His citizenship, whenever it exists, confers, under the Act,
citizenship upon her. The construction which would restrict the Act to women whose husbands, at the
time of marriage, are citizens, would exclude far the greater number, for whose benefit, as we think, the
Act was intended. Its object, in our opinion, was to allow her citizenship to follow that of her husband,
without the necessity of any application for naturalization on her part; and, if this was the object, there
is no reason for the restriction suggested.

"The terms, 'who might lawfully be naturalized under the existing laws,' only limit the application of the
law to free white women. The previous Naturalization Act, existing at the time only required that the
person applying for its benefits should be 'a free white person,' and not an alien enemy. Act of April
14th, 1802, 2 Stat. at L. 153.

"A similar construction was given to the Act by the Court of Appeals of New York, in Burton v. Burton, 40
N. Y. 373; and is the one which gives the widest extension to its provisions"

Note that while the court did say that "the terms, 'who might lawfully be naturalized under existing
laws' only limit the application to free white women" 20 it hastened to add that "the previous
Naturalization Act, existing at the time, . . . required that the person applying for its benefits should be
(not only) a 'free white person' (but also) . . . not an alien enemy." This is simply because under the
Naturalization Law of the United States at the time the case was decided, the disqualification of enemy
aliens had already been removed by the Act of July 30, 1813, as may be seen in the corresponding
footnote hereof anon. In other words, if in the case of Kelly v. Owen only the race requirement was
mentioned, the reason was that there was no other non-racial requirement or no more alien-enemy
disqualification at the time; and this is demonstrated by the fact that the court took care to make it clear
that under the previous naturalization law, there was also such requirement in addition to race. This is
important, since as stated in re Rustigian, 165 Fed. Rep. 980, "The expression used by Mr. Justice Field,
(in Kelly v. Owen) the terms 'who might lawfully be naturalized under existing laws' only limit the
application of the law to free white women, must be interpreted in the application to the special facts
and to the incapacities under the then existing laws," (at p. 982) meaning that whether or not an alien
wife marrying a citizen would be a citizen was dependent, not only on her race and nothing more
necessarily, but on whether or not there were other disqualifications under the law in force at the time
of her marriage or the naturalization of her husband.

4. As already stated, in Lo San Tuang, Choy King Tee and the second Ly Giok Ha, the Court drew the
inference that because Section 1 of Act 2927 was eliminated by Commonwealth Act 473, it. follows that
in place of the said eliminated section, particularly its subdivision (c), being the criterion of whether or
not an alien wife "may be lawfully naturalized," what should be required is not only that she must not be
disqualified under Section 4 but that she must also possess the qualifications enumerated in Section 2,
such as those of age, residence, good moral character, adherence to the underlying principles of the
Philippine Constitution, irreproachable conduct, lucrative employment or ownership of real estate,
capacity to speak and write English or Spanish and one of the principal local languages, education of
children in certain schools, etc., thereby implying that, in effect, said Section 2 has been purposely

347
intended to take the place of Section 1 of Act 2927. Upon further consideration of the proper premises,
We have come to the conclusion that such inference is not sufficiently justified.

To begin with, nothing extant in the legislative history, which We have already examined above of the
mentioned provisions has been shown or can be shown to indicate that such was the clear intent of the
legislature. Rather, what is definite is that Section 15 an exact copy of Section 1994 of the Revised
Statutes of the United States, which, at the time of the approval of Commonwealth Act 473 had already
a settled construction by American courts and administrative authorities.

Secondly, as may be gleaned from the summary of pertinent American decisions quoted above, there
can be no doubt that in the construction of the identically worded provision in the Revised Statutes of
the United States, (Section 1994, which was taken from the Act of February 10, 1855) all authorities in
the United States are unanimously agreed that the qualifications of residence, good moral character,
adherence to the Constitution, etc. are not supposed to be considered, and that the only eligibility to be
taken into account is that of the race or class to which the subject belongs, the conceptual scope of
which, We have just discussed. 21 In the very case of Leonard v. Grant, supra, discussed by Justice
Regala in Lo San Tuang, the explanation for such posture of the American authorities was made thus:

"The phrase, 'shall be deemed a citizen,' in section 1994 Rev. St., or as it was in the Act of 1855, supra,
'shall be deemed and taken to be a citizen,' while it may imply that the person to whom it relates has
not actually become a citizen by ordinary means or in the usual way, as by the judgment of a competent
court, upon a proper application and proof, yet it does not follow that such person is on that account
practically any the less a citizen. The word 'deemed' is the equivalent of 'considered' or 'judged'; and,
therefore, whatever an act of Congress requires to be 'deemed' or 'taken' as true of any person or thing,
must, in law, he considered as having been duly adjudged or established concerning such person or
thing, and have force and effect accordingly. When, therefore, Congress declares that an alien woman
shall, under certain circumstances, be 'deemed' an American citizen, the effect when the contingency
occurs, is equivalent to her being naturalized directly by an act of Congress, or in the usual mode
thereby prescribed."

Unless We disregard now the long settled familiar rule of statutory construction that in a situation like
this wherein our legislature has copied an American statute word for word, it is understood that the
construction already given to such statute before its being copied constitute part of our own law, there
seems to be no reason how We can give a different connotation or meaning to the provision in question.
At least, We have already seen that the views sustaining the contrary conclusion appear to be based on
inaccurate factual premises related to the real legislative background of the framing of our
naturalization law in its present form.

Thirdly, the idea of equating the qualifications enumerated in Section 2 of Commonwealth Act 473 with
the eligibility requirements of Section 1 of Act 2927 cannot bear close scrutiny from any point of view.
There is no question that Section 2 of Commonwealth Act 473 is more or less substantially the same as
Section 3 of Act 2927. In other words, Section 1 of Act 2927 co-existed already with practically the same
provision as Section 2 of Commonwealth Act 473. If it were true that the phrase "who may be lawfully

348
naturalized" in Section 13(a) of Act 2927, as amended by Act 3448, referred to the so called racial
requirement in Section 1 of the same Act, without regard to the provisions of Section 3 thereof, how
could the elimination of Section 1 have the effect of shifting the reference to Section 3, when precisely,
according to the American jurisprudence, which was prevailing at the time Commonwealth Act 473 was
approved, such qualifications as were embodied in said Section 3, which had their counterpart in the
corresponding American statutes, are not supposed to be taken into account and that what should be
considered only are the requirements similar to those provided for in said Section 1 together with the
disqualifications enumerated in Section 4?

Fourthly, it is difficult to conceive that the phrase "who might be lawfully naturalized" in Section 15
could have been intended to convey a meaning different than that given to it by the American courts
and administrative authorities. As already stated, Act 3448 which contained said phrase and from which
it was taken by Commonwealth Act 473, was enacted in 1928. By that time, Section 1994 of the Revised
Statutes of the United States was no longer in force because it had been repealed expressly the Act of
September 22, 1922 which did away with the automatic naturalization of alien wives of American
citizens and required, instead, that they submit to regular naturalization proceedings, albeit under more
liberal terms than those of other applicants. In other words, when our legislature adopted the phrase in
question, which, as already demonstrated, had a definite construction in American law, the Americans
had already abandoned said phraseology in favor of a categorical compulsion for alien wives to be
naturalized judicially. Simple logic would seem to dictate that, since our lawmakers, at the time of the
approval of Act 3448, had two choices, one to adopt the phraseology of Section 1994 with its settled
construction and the other to follow the new posture of the Americans of requiring judicial
naturalization, and it appears that they have opted for the first, We have no alternative but to conclude
that our law still follows the old or previous American law on the subject. Indeed, when Commonwealth
Act 473 was approved in 1939, the Philippine Legislature, already autonomous then from the American
Congress, had a clearer chance to disregard the old American law and make one of our own, or, at least,
follow the trend of the Act of the U.S. Congress of 1922, but still, our legislators chose to maintain the
language of the old law. What then is significantly important is not that the legislature maintained said
phraseology after Section 1 of Act 2927 was eliminated, but that it continued insisting on using it even
after the Americans had amended their law in order to provide for what is now contended to be the
construction that should be given to the phrase in question. Stated differently, had our legislature
adopted a phrase from an American statute before the American courts had given it a construction
which was acquiesced to by those given upon to apply the same, it would be possible for Us to adopt a
construction here different from that of the Americans, but as things stand, the fact is that our
legislature borrowed the phrase when there was already a settled construction thereof, and what is
more, it appears that our legislators even ignored the modification of the American law and persisted in
maintaining the old phraseology. Under these circumstances, it would be in defiance of reason and the
principles of Statutory construction to say that Section 15 has a nationalistic and selective orientation
and that it should be construed independently of the previous American posture because of the
difference of circumstances here and in the United States. It is always safe to say that in the
construction of a statute, We cannot fall on possible judicial fiat or perspective when the demonstrated
legislative point of view seems to indicate otherwise.

349
5. Viewing the matter from another angle, there is need to emphasize that in reality and in effect,
the so-called racial requirements, whether under the American laws or the Philippine laws, have hardly
been considered as qualifications in the same sense as those enumerated in Section 3 of Act 2927 and
later in Section 2 of Commonwealth Act 473. More accurately, they have always been considered as
disqualifications, in the sense that those who did not possess them were the ones who could not "be
lawfully naturalized," just as if they were suffering from any of the disqualifications under Section 2 of
Act 2927 and later those under Section 4 of Commonwealth Act 473, which, incidentally, are practically
identical to those in the former law, except those in paragraphs (f) and (h) of the latter. 22 Indeed, such
is the clear impression anyone will surely get after going over all the American decisions and opinions
quoted and/or cited in the latest USCA (1970), Title 8, section 1430, pp. 598-602, and the first decisions
of this Court on the matter, Ly Giok Ha (1959) and Ricardo Cua, citing with approval the opinions of the
Secretary of Justice. 23 Such being the case, that is, that the so-called racial requirements were always
treated as disqualifications in the same light as the other disqualifications under the law, why should
their elimination not be viewed or understood as a subtraction from or a lessening of the
disqualifications? Why should such elimination have instead the meaning that what were previously
considered as irrelevant qualifications have become disqualifications, as seems to be the import of the
holding in Choy King Tee to the effect that the retention in Section 15 of Commonwealth Act 473 of the
same language of what used to be Section 13 (a) of Act 2927 (as amended by Act 3448), notwithstanding
the elimination of Section 1 of the latter, necessarily indicates that the legislature had in mind making
the phrase in question "who may be lawfully naturalized" refer no longer to any racial disqualification
but to the qualification under Section 2 of Commonwealth Act 473? Otherwise stated, under Act 2927,
there were two groups of persons that could not be naturalized, namely, those falling under Section 1
and those falling under Section 2, and surely, the elimination of one group, i.e. those belonging to
Section 1, could not have had, by any process of reasoning, the effect of increasing, rather than
decreasing the disqualifications that used to be before such elimination. We cannot see by what
alchemy of logic such elimination could have converted qualifications into disqualifications, specially in
the light of the fact that, after all, these are disqualifications clearly set out as such in the law distinctly
and separately from qualifications and, as already demonstrated, in American jurisprudence,
qualifications had never been considered to be of any relevance in determining "who might be lawfully
naturalized," as such phrase is used in the statute governing the status of alien wives of American
citizens, and our law on the matter was merely copied verbatim from the American statutes.

6. In addition to these arguments based on the applicable legal provisions and judicial opinions,
whether here or in the United States, there are practical considerations that militate towards the same
conclusions. As aptly stated in the motion for reconsideration of counsel for petitioner-appellee dated
February 23, 1967, filed in the case of Zita Ngo Burca v. Republic, supra:

"Unreasonableness of requiring alien wife to prove 'qualifications —

"There is one practical consideration that strongly militates against a construction that Section 15 of the
law requires that an alien wife of a Filipino must affirmatively prove that she possesses the qualifications
prescribed under Section 2, before she may be deemed a citizen. Such condition, if imposed upon an

350
alien wife, becomes unreasonably onerous and compliance therewith manifestly difficult. The
unreasonableness of such requirement is shown by the following:

"1. One of the qualifications required of an applicant for naturalization under Section 2 of the law is
that the applicant 'must have resided in the Philippines for a continuous period of not less than ten
years.' If this requirement is applied to an alien wife married to a Filipino citizen, this means that for a
period of ten years at least, she cannot hope to acquire the citizenship of her husband. If the wife
happens to be a citizen of a country whose law declares that upon her marriage to a foreigner she
automatically loses her citizenship and acquires the citizenship of her husband, this could mean that for
a period of ten years at least, she would be stateless. And even after having acquired continuous
residence in the Philippines for ten years, there is no guarantee that her petition for naturalization will
be granted, in which case she would remain stateless for an indefinite period of time.

"2. Section 2 of the law likewise requires of the applicant for naturalization that he 'must own real
estate in the Philippines worth not less than five thousand pesos, Philippine currency, or must have
some known lucrative trade, profession, or lawful occupation.' Considering the constitutional
prohibition against acquisition by an alien of real estate except in cases of hereditary succession (Art.
XIII, Sec. 5, Constitution), an alien wife desiring to acquire the citizenship of her husband must have to
prove that she has a lucrative income derived from a lawful trade, profession or occupation. The income
requirement has been interpreted to mean that the petitioner herself must be the one to possess the
said income. (Uy v. Republic, L-19578, Oct. 27, 1964; Tanpa Ong vs. Republic, L-20605, June 30, 1965; Li
Tong Pek v. Republic, L-20912, November 29, 1965). In other words, the wife must prove that she has a
lucrative income derived from sources other than her husband's trade, profession or calling. It is of
common knowledge, and judicial notice may be taken of the fact that most wives in the Philippines do
not have gainful occupations of their own. Indeed, Philippine law, recognizing the dependence of the
wife upon the husband, imposes upon the latter the duty of supporting the former. (Art. 291, Civil
Code). It should be borne in mind that universally, it is an accepted concept that when a woman marries,
her primary duty is to be a wife, mother and housekeeper. If an alien wife is not to be remiss in this
duty, how can she hope to acquire a lucrative income of her own to qualify her for citizenship?

"3. Under Section 2 of the law, the applicant for naturalization 'must have enrolled his minor
children of school age, in any of the public schools or private schools recognized by the Office of the
Private Education of the Philippines, where Philippine history, government and civics are taught or
prescribed as part of the school curriculum during the entire period of residence in the Philippines
required of him prior to the hearing of his petition for naturalization as Philippine citizen.' If an alien
woman has minor children by a previous marriage to another alien before she marries a Filipino, and
such minor children had not been enrolled in Philippine schools during her period of residence in the
country, she cannot qualify for naturalization under the interpretation of this Court. The reason behind
the requirement that children should be enrolled in recognized educational institutions is that they
follow the citizenship of their father. (Chan Ho Lay v. Republic, L-5666, March 30, 1954; Tan Hi v.
Republic, 88 Phil. 117 [1951]; Hao Lian Chu v. Republic, 87 Phil. 668 [950]; Yap Chin v. Republic, L-4177,
May 29, 1953; Lim Lian Hong v. Republic, L-3575, Dec. 26, 1950). Considering that said minor children by

351
her first husband generally follow the citizenship of their alien father, the basis for such requirement as
applied to her does not exist. Cessante ratione legis cessat ipsa lex.

"4. Under Section 3 of the law, the 10-year continuous residence prescribed by Section 2 'shall be
understood as reduced to five years for any petitioner (who is) married to a Filipino woman.' It is absurd
that an alien male married to a Filipino wife should be required to reside only for five years in the
Philippines to qualify for citizenship, whereas an alien woman married to a Filipino husband must reside
for ten years.

"Thus under the interpretation given by this Court, it is more difficult for an alien wife related by
marriage to a Filipino citizen to become such citizen, than for a foreigner who is not so related. And yet,
it seems more than clear that the general purpose of the first paragraph of Section 15 was obviously to
accord to an alien woman, by reason of her marriage to a Filipino, a privilege not similarly granted to
other aliens. It will be recalled that prior to the enactment of Act No. 3448 in 1928, amending Act No.
2927 (the old Naturalization Law), there was no law granting any special privilege to alien wives of
Filipinos. They were treated as any other foreigner. It was precisely to remedy this situation that the
Philippine legislature enacted Act No. 3448. On this point, the observation made by the Secretary of
Justice in 1941 is enlightening:

'It is true that under Article 22 of the (Spanish) Civil Code, the wife follows the nationality of the
husband; but the Department of State of the United States on October 31, 1921, ruled that the alien
wife of a Filipino citizen is not a Filipino citizen, pointing out that our Supreme Court in the leading case
of Roa v. Collector of Customs (23 Phil. 315) held that Articles 17 to 27 of the Civil Code being political
have been abrogated upon the cession of the Philippine Islands to the United States. Accordingly, the
stand taken by the Attorney-General prior to the enactment of Act No. 3448, was that marriage of alien
women to Philippine citizens did not make the former citizens of this country.' (Op. Atty. Gen., March
16, 1928).

'To remedy this anomalous condition, Act No. 5448 was enacted in 1928 adding section 13(a) to Act No.
2997 which provides that "any woman who is now or may hereafter be married to a citizen of the
Philippine Islands, and who might herself be lawfully naturalized, shall be deemed a citizen of the
Philippine Islands.' (Op. No. 22, s. 1941; emphasis ours)

"If Section 15 of the Revised Naturalization Law were to be interpreted, as this Court did, in such a way
as to require that the alien wife must prove the qualifications prescribed in Section 2, the privilege
granted to alien wives would become illusory. It is submitted that such a construction, being contrary to
the manifested object of the statute, must be rejected.

'A statute is to be construed with reference to its manifest object, and if the language is susceptible of
two constructions, one which will carry out and the other defeat such manifest object, it should receive
the former construction.' (In re National Guard, 71 Vt. 493, 45 A. 1051; Singer v. United States, 323 U.S.
333, 89 L. ed. 285. See also, U.S. v. Navarro, 19 Phil. 134 [1911]; U. S. v. Toribio, 15 Phil. 85 [1910]).

352
'. . . A construction which will cause objectionable results should be avoided and the court will, if
possible, place on the statute a construction which will not result in injustice, and in accordance with the
decisions construing statutes, a construction which will result in oppression, hardship, or inconveniences
will also be avoided, as will a construction which will prejudice public interest, or construction resulting
in unreasonableness, as well as a construction which will result in absurd consequences.'

'So a construction should, if possible, be avoided if the result would be an apparent inconsistency in
legislative intent, as has been determined by the judicial decisions, or which would result in futility,
redundancy, or a conclusion not contemplated by the legislature; and the court should adopt that
construction which will be the least likely to produce mischief. Unless plainly shown to have been the
intention of the legislature, an interpretation which would render the requirements of the statute
uncertain and vague is to be avoided, and the court will not ascribe to the legislature an intent to confer
an illusory right . . .' (82 C.J.S., Statutes, sec. 326, pp. 623-632)."

7. In Choy King Tee and the second Ly Giok Ha, emphasis was laid on the need for aligning the
construction of Section 15 with "the national policy of selective admission to Philippine citizenship." But
the question may be asked, is it reasonable to suppose that in the pursuit of such policy, the legislature
contemplated to make it more difficult, if not practically impossible in some instances, for an alien
woman marrying a Filipino to become a Filipina than any ordinary applicant for naturalization, as has
just been demonstrated above? It seems but natural and logical to assume that Section 15 was intended
to extend special treatment to alien women who by marrying a Filipino irrevocably deliver themselves,
their possessions, their fate and fortunes and all that marriage implies to a citizen of this country, "for
better or for worse." Perhaps there can and will be cases wherein the personal conveniences and
benefits arising from Philippine citizenship may motivate such marriage, but must the minority, as such
cases are bound to be, serve as the criterion for the construction of law? Moreover, it is not farfetched
to believe that in joining a Filipino family, the alien woman is somehow disposed to assimilate the
customs, beliefs and ideals of Filipinos among whom, after all, she has to live and associate, but surely,
no one should expect her to do so even before marriage. Besides, it may be considered that in reality
the extension of citizenship to her is made by the law not so much for her sake as for the husband.
Indeed, We find the following observations anent the national policy rationalization in Choy King Tee
and Ly Giok Ha (the second) to be quite persuasive:

"We respectfully suggest that this articulation of the national policy begs the question. The avowed
policy of 'selective admission' more particularly refers to a case where citizenship is sought to be
acquired in a judicial proceeding for naturalization. In such a case, the courts should no doubt apply the
national policy of selecting only those who are worthy to become citizens. There is here a choice
between accepting or rejecting the application for citizenship. But this policy finds no application in
cases where citizenship is conferred by operation of law. In such cases, the courts have no choice to
accept or reject. If the individual claiming citizenship by operation of law proves in legal proceedings
that he satisfies the statutory requirements, the courts cannot do otherwise than to declare that he is a
citizen of the Philippines. Thus, an individual who is able to prove that his father is a Philippine citizen, is
a citizen of the Philippines, 'irrespective of his moral character, ideological beliefs, and identification
with Filipino ideals, customs, and traditions.' A minor child of a person naturalized under the law, who is

353
able to prove the fact of his birth in the Philippines, is likewise a citizen, regardless of whether he has
lucrative income, or he adheres to the principles of the Constitution. So it is with an alien wife of a
Philippine citizen. She is required to prove only that she may herself be lawfully naturalized, i.e., that she
is not one of the disqualified persons enumerated in Section 4 of the law, in order to establish her
citizenship status as a fact.

"A paramount policy consideration of graver import should not be overlooked in this regard, for it
explains and justifies the obviously deliberate choice of words. It is universally accepted that a State, in
extending the privilege of citizenship to an alien wife of one of its citizens could have had no other
objective than to maintain a unity of allegiance among the members of the family. (Nelson v. Nelson,
113 Neb. 453, 203 N. W. 640 [1925]; see also 'Convention on the Nationality of Married Women:
Historical Background and Commentary.' UNITED NATIONS, Department of Economic and Social Affairs
E/CN, 6/399, pp. 8 et seq.). Such objective can only be satisfactorily achieved by allowing the wife to
acquire citizenship derivatively through the husband. This is particularly true in the Philippines where
tradition and law has placed the husband as head of the family, whose personal status and decisions
govern the life of the family group. Corollary to this, our laws look with favor on the unity and solidarity
of the family (Art. 220, Civil Code), in whose preservation of State as a vital and enduring interest. (See
Art. 216, Civil Code). Thus, it has been said that by tradition in our country, there is a theoretic identity
of person and interest between husband and wife, and from the nature of the relation, the home of one
is that of the other. (See De la Viña v. Villareal, 41 Phil. 13). It should likewise be said that because of the
theoretic identity of husband and wife, and the primacy of the husband, the nationality of husband
should be the nationality of the wife, and the laws upon one should be the law upon the other. For as
the court, in Hopkins v. Fachant (9th Cir., 1904) 65 C.C.A., 1, 130 Fed. 839, held: 'The status of the wife
follows that of the husband, . . . and by virtue of her marriage her husband's domicile became her
domicile.' And the presumption under Philippine law being that the property relations of husband and
wife are under the regime of conjugal partnership (Art. 119, Civil Code), the income of one is also that of
the other.

"It is, therefore, not congruent with our cherished traditions of family unity and identity that a husband
should be a citizen and the wife an alien, and that the national treatment of one should be different
from that of the other. Thus, it cannot be that the husband's interests in property and business activities
reserved by law to citizens should not form part of the conjugal partnership and be denied to the wife,
nor that she herself cannot, through her own efforts but for the benefit of the partnership, acquire such
interests. Only in rare instances should the identity of husband and wife be refused recognition, and we
submit that in respect of our citizenship laws, it should only be in the instances where the wife suffers
from the disqualifications stated in Section 4 of the Revised Naturalization Law." (Motion for
Reconsideration, Burca vs. Republic, supra.)

With all these considerations in mind, We are persuaded that it is in the best interest of all concerned
that Section 15 of the Naturalization Law be given effect in the same way as it was understood and
construed when the phrase "who may be lawfully naturalized," found in the American statute from
which it was borrowed and copied verbatim, was applied by the American courts and administrative
authorities. There is merit, of course, in the view that Philippine statutes should be construed in the light

354
of Philippine circumstances, and with particular reference to our naturalization laws. We should realize
the disparity in the circumstances between the United States, as the so-called "melting pot" of peoples
from all over the world, and the Philippines as a developing country whose Constitution is nationalistic
almost in the extreme. Certainly, the writer of this opinion cannot be the last in rather passionately
insisting that our jurisprudence should speak our own concepts and resort to American authorities, to
be sure, entitled to admiration and respect, should not be regarded as source of pride and indisputable
authority. Still, We cannot close our eyes to the undeniable fact that the provision of law now under
scrutiny has no local origin and orientation; it is purely American, factually taken bodily from American
law when the Philippines was under the dominating influence of statutes of the United States Congress.
It is indeed a sad commentary on the work of our own legislature of the late 1920's and 1930's that
given the opportunity to break away from the old American pattern, it took no step in that direction.
Indeed, even after America made it patently clear in the Act of Congress of September 22, 1922 that
alien women marrying Americans cannot be citizens of the United States without undergoing
naturalization proceedings, our legislators still chose to adopt the previous American law of August 10,
1855 as embodied later in Section 1994 of the Revised Statutes of 1874, which, it is worth reiterating,
was consistently and uniformly understood as conferring American citizenship to alien women marrying
Americans ipso facto, without having to submit to any naturalization proceeding and without having to
prove that they possess the special qualifications of residence, moral character, adherence to American
ideals and American constitution, provided they show they did not suffer from any of the
disqualifications enumerated in the American Naturalization Law. Accordingly, We now hold, all
previous decisions of this Court indicating otherwise notwithstanding, that under Section 15 of
Commonwealth Act 473, an alien woman marrying a Filipino, native born or naturalized, becomes ipso
facto a Filipina provided she is not disqualified to be a citizen of the Philippines under Section 4 of the
same law. Likewise, an alien woman married to an alien who is subsequently naturalized here follows
the Philippine citizenship of her husband the moment he takes his oath as Filipino citizen, provided that
she does not suffer from any of the disqualifications under said Section 4.

As under any other law rich in benefits for those coming under it, doubtless there will be instances
where unscrupulous persons will attempt to take advantage of this provision of law by entering into fake
and fictitious marriages or mala fide matrimonies. We cannot as a matter of law hold that just because
of these possibilities, the construction of the provision should be otherwise than as dictated inexorably
by more ponderous relevant considerations, legal, juridical and practical. There can always be means of
discovering such undesirable practices and every case can be dealt with accordingly as it arises.

III.

The third aspect of this case requires necessarily a re-examination of the ruling of this Court in Burca,
supra, regarding the need of judicial naturalization proceedings before the alien wife of a Filipino may
herself be considered or deemed a Filipino. If this case which, as already noted, was submitted for
decision in 1964 yet, had only been decided earlier, before Go Im Ty, the foregoing discussions would
have been sufficient to dispose of it. The Court could have held that despite her apparent lack of
qualifications, her marriage to her co-petitioner made her a Filipina, without her undergoing any
naturalization proceedings, provided she could sustain her claim that she is not disqualified under

355
Section 4 of the law. But as things stand now, with the Burca ruling, the question We have still to decide
is, may she be deemed a Filipina without submitting to a naturalization proceeding?

Naturally, if Burca is to be followed, it is clear that the answer to this question must necessarily be in the
affirmative. As already stated, however, the decision in Burca has not yet become final because there is
still pending with Us a motion for its reconsideration which vigorously submits grounds worthy of
serious consideration by this Court. On this account, and for the reasons expounded earlier in this
opinion, this case is as good an occasion as any other to re-examine the issue.

In the said decision, Justice Sanchez held for the Court:

"We accordingly rule that: (1) An alien woman married to a Filipino who desires to be a citizen of this
country must apply therefore by filing a petition for citizenship reciting that she possesses all the
qualifications set forth in Section 2 and none of the disqualifications under Section 4, both of the
Revised Naturalization Law; (2) Said petition must be filed in the Court of First Instance where petitioner
has resided at least one year immediately preceding the filing of the petition; and (3) Any action by any
other office, agency, board or official, administrative or otherwise — other than the judgment of a
competent court of justice — certifying or declaring that an alien wife of the Filipino citizen is also a
Filipino citizen, is hereby declared null and void.

"3. We treat the present petition as one for naturalization. Or, in the words of law, a 'petition for
citizenship'. This is as it should be. Because a reading of the petition will reveal at once that efforts were
made to act forth therein. and to prove afterwards, compliance with Sections 2 and 4 of the Revised
Naturalization law. The trial court itself apparently considered the petition as one for naturalization,
and, in fact, declared petitioner 'a citizen of the Philippines.'"

In other words, under this holding, in order for an alien woman marrying a Filipino to be vested with
Filipino citizenship, it is not enough that she possesses the qualifications prescribed by Section 2 of the
law and none of the disqualifications enumerated in its Section 4. Over and above all these, she has to
pass thru the whole process of judicial naturalization, apparently from declaration of intention to oath-
taking, before she can become a Filipina. In plain words, her marriage to a Filipino is absolutely of no
consequence to her nationality vis-a-vis that of her Filipino husband; she remains to be the national of
the country to which she owed allegiance before her marriage, and if she desires to be of one nationality
with her husband, she has to wait for the same time that any other applicant for naturalization needs to
complete, the required period of ten year residence, gain the knowledge of English or Spanish and one
of the principal local languages, make her children study in Filipino schools, acquire real property or
engage in some lawful occupation of her own independently of her husband, file her declaration of
intention and after one year her application for naturalization, with the affidavits of two credible
witnesses of her good moral character and other qualifications, etc., etc., until a decision is rendered in
her favor, after which, she has to undergo the two years of probation, and only then, but not before she
takes her oath as citizen, will she begin to be considered and deemed to be a citizen of the Philippines.
Briefly; she can become a Filipino citizen only by judicial declaration.

356
Such being the import of, the Court's ruling, and it being quite obvious, on the other hand, upon a
cursory reading of the provision, in question, that the law intends by it to spell out what is the "effect of
naturalization on (the) wife and children" of an alien, as plainly indicated by its title, and inasmuch as the
language of the provision itself clearly conveys the thought that some effect beneficial to the wife is
intended by it, rather than that she is not in any manner to be benefited thereby, it behooves Us to take
a second hard look at the ruling, if only to see whether or not the Court might have overlooked any
relevant consideration warranting a conclusion different from that contained therein. It is undeniable
that the issue before Us is of grave importance, considering its consequences upon tens of thousands of
persons affected by the ruling therein made by the Court, and surely, it is for Us to avoid, whenever
possible, that Our decision in any case should produce any adverse effect upon them not contemplated
either by the law or by the national policy it seeks to enforce.

AMICI CURIAE in the Burca case, respectable and impressive by their number and standing in the Bar
and well known for their reputation for intellectual integrity, legal acumen and incisive and
comprehensive resourcefulness in research, truly evident in the quality of the memorandum they have
submitted in said case, invite Our attention to the impact of the decision therein thus:

"The doctrine announced by this Honorable Court for the first time in the present case — that an alien
woman who marries a Philippine citizen not only does not ipso facto herself become a citizen but can
acquire such citizenship only through ordinary naturalization proceedings under the Revised
Naturalization Law, and that all administrative actions 'certifying or declaring' such woman to be a
Philippine citizen are 'null and void' — has consequences that reach far beyond the confines of the
present case. Considerably more people are affected, and affected deeply, than simply Mrs. Zita N.
Burca. The newspapers report that as many as 15 thousand women married to Philippine citizens are
affected by this decision of the Court. These are women of many and diverse nationalities, including
Chinese, Spanish, British, American, Columbian, Finnish, Japanese, Chilean, and so on. These members
of the community, some of whom have been married to citizens for two or three decades, have all
exercised rights and privileges reserved by law to Philippine citizens. They will have acquired, separately
or in conjugal partnership with their citizen husbands, real property, and they will have sold and
transferred such property. Many of these women may be in professions membership in which is limited
to citizens. Others are doubtless stockholders or officers or employees in companies engaged in
business activities for which a certain percentage of Filipino equity content is prescribed by law. All
these married women are now faced with possible divestment of personal status and of rights acquired
and privileges exercised in reliance, in complete good faith, upon a reading of the law that has been
accepted as correct for more than two decades by the very agencies of government charged with the
administration of that law. We must respectfully suggest that judicial doctrines which would visit such
comprehensive and far-reaching injury upon the wives and mothers of Philippine citizens deserve
intensive scrutiny and re-examination."

To be sure, this appeal can be no less than what this Court attended to in Gan Tsitung vs. Republic, G.R.
No L-20819, Feb. 21, 1967, 19 SCRA 401—when Chief Justice Concepcion observed:

357
"The Court realizes, however, that the rulings in the Barretto and Delgado cases — although referring to
situations the equities of which are not identical to those obtaining in the case at bar — may have
contributed materially to the irregularities committed therein and in other analogous cases, and induced
the parties concerned to believe, although erroneously, that the procedure followed was valid under the
law.

"Accordingly, and in view of the implications of the issue under consideration, the Solicitor General was
required, not only, to comment thereon, but, also, to state 'how many cases there are, like the one at
bar, in which certificates of naturalization have been issued after notice of the filing of the petition for
naturalization had been published in the Official Gazette only once, within the periods (a) from January
28, 1950' (when the decision in Delgado v. Republic was promulgated) 'to May 29, 1957' (when the Ong
Son Cui was decided) 'and (b) from May 29, 1957 to November 29, 1965' (when the decision in the
present case was rendered).

"After mature deliberation, and in the light of the reasons adduced in appellant's motion for
reconsideration and in the reply thereto of the Government, as well as of the data contained in the
latter, the Court holds that the doctrine laid down in the Ong Son Cui case shall apply and affect the
validity of certificates of naturalization issued after, not on or before May 29, 1957."

Here We are met again by the same problem. In Gan Tsitung, the Court had to expressly enjoin the
prospective application of its construction of the law made in a previous decision 24 which had already
become final, to serve the ends of justice and equity. In the case at bar, We do not have to go that far.
As already observed, the decision in Burca is still under reconsideration, while the ruling in Lee Suan Ay,
Lo San Tuang, Choy King Tee and other that followed them have at the most become the law of the case
only for the parties thereto. If there are good grounds therefor, all We have to do now is to re-examine
the said rulings and clarify them.

For ready reference, We requote Section 15:

"Sec. 15. Effect of the naturalization on wife and children .— Any woman who is now or may
hereafter be married to a citizen of the Philippines, and who might herself be lawfully naturalized shall
be deemed a citizen of the Philippines.

"Minor children of persons naturalized under this law who have been born in the Philippines shall be
consider citizens thereof.

"A foreign-born minor child, if dwelling in the Philippines at the time of naturalization of the parents,
shall automatically become a Philippine citizen, and a foreign-born minor child, who is not in the
Philippines at the time the parent is naturalized, shall be deemed a Philippines citizen only during his
minority, unless he begins to reside permanently in the Philippines when still a minor, in which case, he
will continue to be a Philippine citizen even after becoming of age.

"A child born outside of the Philippines after the naturalization of his parent, shall be considered a
Philippine citizen, unless within one year after reaching the age of minority, he fails to register himself as

358
a Philippine citizen at the American Consulate of the country where he resides, and to take the
necessary oath of allegiance."

It is obvious that the main subject-matter and purpose of the statute, the Revised Naturalization Law or
Commonwealth Act 473, as a whole is to establish a complete procedure for the judicial conferment of
the status of citizenship upon qualified aliens. After having out such a procedure, remarkable for its
elaborate and careful inclusion of all safeguards against the possibility of any undesirable persons
becoming a part of our citizenry, it carefully but categorically states the consequence of the
naturalization of an alien undergoing such procedure it prescribes upon members of his immediate
family, his wife and children, 25 and, to that end, in no uncertain terms it ordains that: (a) all his minor
children who have been born in the Philippines shall be "considered citizens" also; (b) all such minor
children, if born outside the Philippines but dwelling here at the time of such naturalization "shall
automatically become" Filipinos also, but those not born in the Philippines and not in the Philippines at
the time of such naturalization, are also "deemed citizens" of this country provided that they shall lose
said status if they transfer their permanent residence to a foreign country before becoming of age; (c) all
such minor children, if born outside of the Philippines after such naturalization, shall also be
"considered" Filipino citizens, unless they expatriate themselves by failing to register as Filipinos at the
Philippine (American) Consulate of the country where they reside and take the necessary oath of
allegiance; and (d) as to the wife, she "shall be deemed a citizen of the Philippines" if she is one "who
might herself be lawfully naturalized". 26

No doubt whatever is entertained, so Burca holds very correctly, as to the point that the minor children,
failing within the conditions of place and time of birth and residence prescribed in the provision, are
vested with Philippines citizenship directly by legislative fiat or by force of the law itself and without the
need for any judicial proceeding or declaration. (At p. 192 SCRA). Indeed, the language of the provision
is not susceptible of any other interpretation. But it is claimed that the same expression "shall be
deemed a citizen of the Philippines" in reference to the wife, does not necessarily connote the vesting of
citizenship status upon her by legislative fiat because the antecedent phrase requiring that she must be
one "who might herself be lawfully naturalized" implies that such status is intended to attach only after
she has undergone the whole process of judicial naturalization required of any person desiring to
become a Filipino. Stated otherwise, the ruling in Burca is that while Section 15 envisages and intends
legislative naturalization as to the minor children, the same section deliberately treats the wife
differently and leaves her out for ordinary judicial naturalization.

Of course, it goes without saying that it is perfectly within the constitutional authority of the Congress of
the Philippines to confer or vest citizenship status by legislative fiat. (U.S. v. Wong Kim Ark, 169 U.S. 649,
42 L ed. 890 [1898]; See, 1 Tañada & Carreon, Political Law of the Philippines 152 [1961 ed.] ) In fact, it
has done so for particular individuals, like two foreign religious prelates, 27 hence there is no reason it
cannot do it for classes or groups of persons under general conditions applicable to all of the members
of such class or group, like women who marry Filipinos, whether native-born or naturalized. The issue
before Us in this case is whether or not the legislature has done so in the disputed provisions of Section
15 of the Naturalization Law. And Dr. Vicente G. Sinco, one of the most respected authorities on political
law in the Philippines 28 observes in this connection thus: " A special form of naturalization is often

359
observed by some states with respect to women. Thus in the Philippines a foreign woman married to a
Filipino citizen becomes ipso facto naturalized, if she belongs to any of the classes who may apply for
naturalization under the Philippine Laws" (Sinco, Phil. Political Law 498-499 [10th ed. 1954]; emphasis
ours; this comment is substantially reiterated in the 1962 edition, citing Ly Giok Ha and Ricardo Cua ,
supra.)

More importantly, it may be stated at this juncture, that in construing the provision of the United States
statutes from which our law has been copies, 28a the American citizenship by choice but by operation of
law. "In the Revised Statutes the words 'and taken' are omitted. The effect of this statute is that every
alien woman who marries a citizen of the United States becomes perforce a citizen herself, without the
formality of naturalization, and regardless of her wish in that respect." (USCA 8, p. 601 [1970 ed.], citing
Mackenzie v. Hare, 1913, 134 P. 713, 165 Cal. 766, affirmed 36 S. Ct. 106, 239 U.S. 299, 60 L ed. 297.)

We need not recount here again how this provision in question was first enacted as paragraph (a) of
Section 13, by way of an insertion into Act 2927 by Act 3448 of November 30, 1928, and that , in turn,
and paragraph was copied verbatim from Section 1994 of the Revised Statutes of the United States,
which by that time already had a long accepted construction among the courts and administrative
authorities in that country holding that under such provision an alien woman who married a citizen
became, upon such marriage, likewise a citizen by force of law and as a consequence of the marriage
itself without having to undergo any naturalization proceedings, provided that it could be shown that at
the time of such marriage, she was not disqualified to be naturalized under the laws then in force. To
repeat the discussion We already made of these undeniable facts would unnecessarily make this
decision doubly extensive. The only point which might be reiterated for emphasis at this juncture is that
whereas in the United States, the American Congress, recognizing the uniform construction of Section
1994 of the Revised Statutes to be as stated above, and finding it desirable to avoid the effects of such
construction, approved the Act of September 22, 1922 explicitly requiring all such alien wives to submit
to judicial naturalization, albeit under more liberal terms than those for other applicants for citizenship,
on the other hand, the Philippines Legislature, instead of following suit and adopting a requirement,
enacted Act 3448 on November 30, 1928 which copied verbatim the aforementioned Section 1994 of
the Revised Statutes, thereby indicating its preferences to adopts the latter law and its settled
constitution rather than the reform introduced by the Act of 1992.

Obviously, these considerations leave Us no choice. Much as this Court may feel that as the United
States herself has evidently found it to be an improvement of her national policy vis-a-vis the alien wives
of her citizens to discontinue their automatic incorporation into the body of her citizenry without
passing through the judicial scrutiny of a naturalization proceeding, as it used to be before 1922, it
seems but proper, without evidencing any bit of colonial mentality, that as a developing country, the
Philippines adopt a similar policy, unfortunately, the manner in which our own legislature has enacted
our laws on the subject, as recounted above, provides no basis for Us to construe said law along the line
of the 1922 modification of the American Law. For Us to do so would be to indulge in judicial legislation
which it is not constitutionally permissible for this Court to do. Worse, this Court would be going
precisely against the grain of the implicit Legislative intent.

360
There is at least one decision of this Court before Burca wherein it seems it is quite clearly implied that
this Court is of the view that under Section 16 of the Naturalization Law, the widow and children of an
applicant for naturalization who dies during the proceeding do not have to submit themselves to
another naturalization proceeding in order to avail of the benefits of the proceedings involving the
husband. Section 16 provides:

"SEC. 16. Right of widow and children of petitioners who have died. — In case a petitioner should
die before the final decision has been rendered, his widow and minor children may continue the
proceedings. The decision rendered in the case shall, so far same legal effect as if it had been rendered
during the life of the petitioner."

In Tan Lin v. Republic, G.R. No. L-13706, May 31, 1961, 2 SCRA 383 this Court held:

"Invoking the above provisions in their favor, petitioners-appellants argue (1) that under said Sec. 16,
the widow and minor children are allowed to continue the same proceedings and are not substituted for
the original petitioner; (2) that the qualifications of the original petitioner remain to be in issue and not
those of the widow and minor children, and (3) that said Section 16 applies whether the petitioner dies
before or after final decision is rendered, but before the judgment becomes executory.

"There is force in the first and second arguments. Even the second sentence of said Section 16
contemplates the fact that the qualifications of the original petitioner remains the subject of inquiry, for
the simple reason that it states that "The decision rendered in the case shall, so far as the widow and
minor children are concerned, produce the same legal effect as if it had been rendered during the life of
the petitioner.' This phraseology emphasizes the intent of the law to continue the proceedings with the
deceased as the theoretical petitioner, for if it were otherwise, it would have been unnecessary to
consider the decision rendered, as far as it effected the widow and the minor children.

xxx xxx xxx

"The Chua Chian case (supra), cited by the appellee, declared that a dead person can not be bound to do
things stipulated in the oath of allegiance, because an oath is a personal matter. Therein, the widow
prayed that she be allowed to take the oath of allegiance for the deceased. IN the case at bar, petitioner
Tan Lin merely asked that she be allowed to take the oath of allegiance and the proper certificate of
naturalization, once the naturalization proceedings of her deceased husband, shall have been
completed, not on behalf of the deceased, but on her own behalf and of her children, as recipients of
the benefits of his naturalization. In other words, the herein petitioner proposed to take the oath of
allegiance, as a citizen of the Philippines, by virtue of the legal provision that 'any woman who is now or
may hereafter be married to a citizen of the Philippines and who might be lawfully naturalized shall be
deemed a citizen of the Philippines. Minor children of persons naturalized under this law who have been
born in the Philippines shall be considered citizens thereof.' (Section 15, Commonwealth Act No. 473).
The decision granting citizenship to Lee Pa and the record of the case at bar, do not show that the
petitioning widow could not have been lawfully naturalized, at the time Lee Pa filed his petition, apart
from the fact that his 9 minor children were all born in the Philippines. (Decision, In the Matter of the

361
P)etition of Lee Pa to be admitted a citizen of the Philippines, Civil Case No. 16287, CFI, Manila, Annex A;
Record on Appeal, pp. 8-11). The reference for Chua case is, therefore, premature."

Section 16, as may be seen, is a parallel provision to Section 15. If the widow of an applicant for
naturalization as Filipino, who dies during the proceedings, is not required to go through a naturalization
proceedings, in order to be considered as a Filipino citizen hereof, it should follow that the wife of a
living Filipino cannot be denied the same privilege. This is plain common sense and there is absolutely
no evidence that the Legislature intended to treat them differently.

Additionally, We have carefully considered the arguments advanced in the motion for reconsideration in
Burca, and We see no reason to disagree with the following views of counsel:

"It is obvious that the provision itself is a legislative declaration of who may be considered citizens of the
Philippines. It is a proposition too plain to be disputed that Congress has the power not only to prescribe
the mode or manner under which foreigners may acquire citizenship, but also the very power of
conferring citizenship by legislative fiat. (U.S. v. Wong Kim Ark, 169 U.S. 649, 42 L. Ed. 890 [1898]; see 1
Tañada and Carreon, Political Law of the Philippine citizens ed.]). The constitutional itself recognizes as
Philippines citizens 'Those who are naturalized in accordance with law' (Section 1[5], Article IV,
Philippine Constitution). Citizens by naturalization, under this provision, include not only those who are
naturalized in accordance with legal proceedings for the acquisition of citizenship, but also those who
acquire citizenship by 'derivative naturalization' or by operation of law, as. for example, the
'naturalization' of an alien wife through the naturalization of her husband, or by marriage of an alien
woman to a citizen. (See Tañada & Carreon, op. cit supra, at 152 172; Velayo, Philippine Citizenship and
Naturalization 2 [1965 ed.]: 1 Paras, Civil code 186 [1967 ed.]; see also 3 Hackworth, Digest of
International Law 3).

"The phrase 'shall be deemed a citizen of the Philippines found in Section 14 of the Revised
Naturalization Law clearly manifests an intent to confer citizenship. Construing a similar phrase found in
the old U.S. naturalization law (Revised Statutes, 1994) , American courts have uniformly taken it to
mean that upon her marriage, the alien woman becomes by operation of law a citizen of the United
States as fully as if she had complied with all the provisions of the statutes upon the subject of
naturalization. (U.S. v. Keller, 13 F. 82; U.S. Opinions of the US Attorney General dated June 4, 1874 [14
Op. 402], July 20, 1909 [27 Op. 507], December 1, 1910 [28 Op. 508], Jan. 15, 1920 [32 Op. 209] and Jan.
12, 1923 [23 398] ).

'The phrase "shall be deemed a citizen, " in Section 1994 Revised Statute (U.S. Comp. Stat. 1091 1268) or
as it was in the Act of 1855 910 Stat. at L. 604, Chapt. 71, Sec. 2), "shall be deemed and taken to be a
citizen", while it may imply that the person to whom it relates has not actually become a citizen by the
ordinary means or in the usual way, as by the judgment of a competent court, upon a proper application
and proof, yet it does not follow that such person is on that account practically any the less a citizen. The
word "deemed" is the equivalent of "considered" or "judged," and therefore, whatever an Act of
Congress requires to be "deemed" or "taken" as true of any person or thing must, in law, be considered
as having been duly adjudged or established concerning such person or thing, and have force and effect

362
accordingly. When, therefore, Congress declares that an alien woman shall, under certain
circumstances, be "deemed" an American to her being naturalized directly by an Act of Congress or in
the usual mode thereby prescribed.' (Van Dyne, Citizenship of the United States 239, cited in Velayo,
Philippine Citizenship and Naturalization 146-147 [1965 ed.] ; italics ours).

"That this was likewise the intent of the Philippine legislature when it enacted the first paragraph of
Section 15 of the Revised Naturalization provision. In its entirety, Section 15 reads:

(See supra)

The phrases 'shall be deemed,' shall be considered,' and 'shall automatically become,' as used in the
above provision , are undoubtedly synonymous. The leading idea or purpose of the provision was to
confer Philippine citizenship by operation of law upon certain classes of aliens as a legal consequence of
their relationship, by blood affinity, to persons who are already citizens of the Philippines. Whenever the
fact of relationship of the persons enumerated in the provision concurs related, the effect is for said
persons to become ipso facto citizens of the Philippines. 'Ipso facto' as here used does not mean that all
alien wives and all minor children of Philippine citizens, from the mere fact of relationship, necessarily
become such citizens also. Those who do not meet the statutory requirements do not ipso facto become
citizens; they must apply for naturalization in order to acquire such status. What it does mean, however,
is that in respect of those persons numerated in Section 15, the relationship to a citizen of the
Philippines is the operative fact which establishes the acquisition of Philippine citizenship by them.
Necessarily, it also determines the points of time at which such citizenship commences. Thus, under the
second paragraph of Section 15, a minor child of a Filipino naturalized under the law, who was born in
the Philippines, becomes ipso facto a citizen of the Philippines from the time the fact of relationship
concurs with the fact of a citizenship of his parent, and the time when child became a citizen does not
depend upon the time that he is able to prove that he was born in the Philippines. The child may prove
some 25 years after the naturalization of his father that he was born in the Philippines and should,
therefore, be 'considered' a citizen thereof. It does not mean that he became a Philippine citizen only at
that later time. Similarly, an alien woman who married a Philippine citizen may be able to prove only
some 25 years after her marriage (perhaps, because it was only 25 years after the marriage that her
citizenship status became in question), that she is one who 'might herself be lawfully naturalized.' It is
not reasonable to conclude that she acquired Philippine citizenship only after she had proven that she
'might herself be lawfully naturalized.'

"The point that bears emphasis in this regard is that in adopting the very phraseology of the law , the
legislature could not have intended that an alien wife should not be deemed a Philippine citizen unless
and until she proves that she might herself be lawfully naturalized' is not a condition precedent to the
vesting or acquisition of citizenship; it is only a condition or a state of fact necessary to establish her
citizenship as a factum probandum i.e., as a fact established and proved in evidence. The word 'might,'
as used in that phrase, precisely implies that at the time of her marriage to a Philippine citizen, the alien
woman 'had (the) power' to become such a citizen herself under the laws then in force. (Owen v. Kelly, 6
DC 191 [1867], aff'd Kelly v. Owen, power long after her marriage does not alter the fact that at her
marriage, she became a citizen.

363
"(This Court has held) that 'an alien wife of a Filipino citizen may not acquire the status of a citizen of the
Philippines unless there is proof that she herself may be lawfully naturalized' (Decision, pp. 3-4). Under
this view, the acquisition' of citizenship by the alien wife depends on her having proven her
qualifications for citizenship, that is, she is not a lawfully naturalized. It is clear from the words of the law
that the proviso does not mean that she must first prove that deemed (by Congress, not by the courts) a
citizen. Even the 'uniform' decisions cited by this Court (at fn. 2) to support its holding did not rule that
the alien wife becomes a citizen only after she has proven her qualifications for citizenship. What those
decisions ruled was that the alien wives in those cases failed to prove their qualifications and therefore
they failed to establish their claim to citizenship. Thus in Ly Giok Ha v. Galang, 101 Phil. 459 [1957], the
case was remanded to the lower court for determination of whether petitioner, whose claim to
citizenship by marriage to a Filipino was disputed by the Government, 'might herself be lawfully
naturalized,' for the purpose of 'proving her alleged change of political status from alien to citizen' (at
464). In Cua v. Board, 101 Phil. 521 [1957], the alien wife who was being deported, claimed she was a
Philippine citizen by marriage to a Filipino. This Court finding that there was no proof that she was not
disqualified under Section 4 of the Revised Naturalization Law, ruled that: 'No such evidence appearing
on record, the claim of assumption of Philippine citizenship by Tijoe Wu Suan, upon her marriage to
petitioner, is untenable.' (at 523) It will be observed that in these decisions cited by this Court, the lack
of proof that the alien wives 'might (themselves) be lawfully naturalized' did not necessarily imply that
they did not become, in truth and in fact, citizens upon their marriage to Filipinos. What the decisions
merely held was that these wives failed to establish their claim to that status as a proven fact.

"In all instances where citizenship is conferred by operation of law, the time when citizenship is
conferred should not be confused with the time when citizenship status is established as a proven fact.
Thus, even a natural-born citizen of the Philippines, whose citizenship status is put in issue in any
proceeding would be required to prove, for instance, that his father is a citizen of the Philippines in
order to factually establish his claim to citizenship. *** His citizenship status commences from the time
of birth, although his claim thereto is established as a fact only at a subsequent time. Likewise, an alien
woman who might herself be a lawfully naturalized becomes a Philippine citizen at the time of her
marriage to a Filipino husband, not at the time she is able to establish that status as a proven fact by
showing that she might herself be lawfully naturalized. Indeed, there is no difference between a
statutory declaration that a person is deemed a citizen of the Philippines provided his father is such
citizen from a declaration that an alien woman married to a Filipino citizen of the Philippines provided
his father is such citizen from a declaration that an alien woman married to a Filipino citizen of the
Philippines provided she might herself be lawfully naturalized. Both become citizens by operation of law;
the former becomes a citizen ipso facto upon birth; the later ipso facto upon marriage.

"It is true that unless and until the alien wife proves that she might herself be lawfully naturalized, it
cannot be said that she has established her status as a proven fact. But neither can it be said that on
account, she did not become a citizen of the Philippines. If her citizenship status is not questioned in any
legal proceeding, she obviously has no obligation to establish her status as a fact. In such a case, the
presumption of law should be what she claims to be. (U.S. v. Roxas, 5 Phil. 375 [1905] : Hilado v. Assad,

364
51 O.G. 4527 [1955] ). There is a presumption that a representation shown to have been made is true.
(Aetna Indemnity Co. v. George A. Fuller, Co., 73 A. 738 A. 369, 111 ME. 321)."

The question that keeps bouncing back as a consequence of the foregoing views is, what substitute is
there for naturalization proceeding to enable the alien wife of a Philippine citizen to have the matter of
her own citizenship settled and established so that she may not have to be called upon to prove it
everytime she has to perform an act or enter into a transaction or business or exercise a right reserved
only to Filipinos? The ready answer to such question is that as the laws of our country, both substantive
and procedural, stand today, there is no such procedure, but such is no proof that the citizenship under
discussion is not vested as of the date of marriage or the husband's acquisition of citizenship, as the case
may be, for the truth is that the situation obtains even as to native-born Filipinos. Everytime the
citizenship of a person is material or indispensable in a judicial or administrative case, Whatever the
corresponding court or administrative authority decides therein as to such citizenship is generally not
considered as res adjudicata, hence it has to be threshed out again and again as the occasion may
demand. This, as We view it, is the sense in which Justice Dizon referred to "appropriate proceeding" in
Brito v. Commissioner, supra. Indeed, only the good sense and judgment of those subsequently inquiring
into the matter may make the effort easier or simpler for the persons concerned by relying somehow on
the antecedent official findings, even if these are not really binding.

It may not be amiss to suggest, however, that in order to have a good starting point and so that the
most immediate relevant public records may be kept in order the following observations in Opinion No.
38, series of 1958, of then Acting Secretary of Justice Jesus G. Barrera, may be considered as the most
appropriate initial step by the interested parties.

"Regarding the steps that should be taken by an alien woman married to a Filipino citizen in order to
acquire Philippine citizenship, the procedure followed in the Bureau of Immigration is as follows: The
alien woman must file a petition for the cancellation of her alien certificate of registration alleging,
among other things, that she is married to a Filipino citizen and that she is not disqualified from
acquiring her husband's citizenship pursuant to section 4 of Commonwealth Act No. 473, as amended.
Upon the filing of said petition, which should be accompanied or supported by the joint affidavit of the
petitioner and her Filipino husband to the effect that the petitioner does not belong to any of the groups
disqualified by the cited section from becoming naturalized Filipino citizen (please see attached CEB
Form 1), the Bureau of Immigration conducts an investigation and thereafter promulgates its order or
decision granting or denying the petition."

Once the Commissioner of Immigration cancels the subject's registration as an alien, there will probably
be less difficulty in establishing her Filipino citizenship in any other proceeding, depending naturally on
the substance and vigor of the opposition.

Before closing, it is perhaps best to clarify that this third issue We have passed upon was not touched by
the trial court, but as the point is decisive in this case, the Court prefers that the matter be settled once
and for all now.

365
IN VIEW OF ALL THE FOREGOING, the judgment of the Court a quo dismissing appellants' petition for
injunction is hereby reversed and the Commissioner of Immigration and/or his authorized
representative is permanently enjoined from causing the arrest and deportation and the confiscation of
the bond of appellant Lau Yuen Yeung, who is hereby declared to have become a Filipino citizen from
and by virtue of her marriage to her co-appellant Moy Ya Lim Yao al as Edilberto Aguinaldo Lim, a
Filipino citizen of January 25, 1962. No costs.

Dizon, Castro, Teehankee and Villamor, JJ ., concur.

Makalintal J ., reserves his separate concurring opinion.

Fernando, J ., concurs except as the interpretation accorded some American decisions as to which he is
not fully persuaded.

APPENDIX

The following review of all naturalization statutes of the United States from 1790 to 1970 ravel: (1) that
aside from race, various other disqualifications have also been provided for in the said statutes from
time to time, although it was only in 1906 that the familiar and usual grounds of disqualification, like not
being anarchists, polygamists, etc. were incorporated therein, and (2) that qualifications of applicants
for naturalization also varied from time to time.

A — DISQUALIFICATIONS

1. In the first naturalization statute of March 26, 1790, only a "free white person" could be
naturalized, provided he was not "proscribed" by any state, unless it be with the consent of such state.
(Chap. V. 1 Stat. 103)

2. In the Act of January 29, 1795, to the same provisions was added the disqualification of those
"legally convicted of having joined the army of Great Britain, during the late war." (Chap. XX, 1 Stat.
414).

3. In the Act of June 18, 1798, Section 1 thereto provided:

"SECTION 1. Be it enacted by the Senate and House of Representatives of the United States of
America in Congress assembled, That no alien shall be admitted to become a citizen of the United
States, or of any state, unless in the manner prescribed by the act, entitled 'An Act to establish an
uniform rule of naturalization; and to repeal the act heretofore passed on that subject, 'he shall have
declared his intention to become a citizen of the United States, five years, at least, before his admission,
and shall, at the time of his application to be admitted, declare and prove, to the satisfaction of the
court having jurisdiction in the case, that he has resided within the United States fourteen years, at
least, and within the state or territory where, or for which such court is at the time held five years, at
least, besides conforming to the other declarations, renunciations and proofs, by the said act required,
any thing therein to the contrary hereof notwithstanding: Provided, that any alien, who was residing
within the limelights, and under the jurisdiction of the United States, before the twenty-ninth day of

366
January, one thousand seven hundred and ninety-five, may, within one year after the passing of this
act—and any alien who shall have made the declaration of his intention to become a citizen of the
United States, in conformity to the provisions of the act, entitled 'An act to establish an uniform rule of
naturalization, and to repeal the act heretofore passed on that subject,' may, within four years after
having made the declaration aforesaid, be admitted to become a citizen, in the manner prescribed by
the said act, upon his making proof that he has resided five years, at least, within the limits, and under
the jurisdiction of the United States: And provided also, that no alien, who shall be a native, citizen,
denizen or subject of any nation or state with whom the United States shall be at war, at the time of his
application, shall be then admitted to become a citizen of the United States."

There is here no mention of "white persons." (Chap. LIV, 1 Stat. 566).

4. In the Act of April 14, 1802, mentioned in Kelly v. Owen, supra, reference was made again to
"free white persons," and the same enemy alien and "state-proscribed" disqualifications in the former
statutes were carried over. (Chap. XXVIII, 2 Stat. 153.)

5. The Act of March 26, 1804 provided in its Section 1 thus:

"Be it enacted by the Senate and House of Representatives of the United States of America in Congress
assembled, That any alien, being a free white person, who was residing within the limits and under the
jurisdiction of the United States, at any time between the eighteenth day of June, one thousand seven
hundred and ninety-eight, and the fourteenth day of April one thousand eight hundred and two, and
who has continued to reside within the same, may be admitted to become a citizen of the United States,
without a compliance with the first condition specified in the first section of the act, entitled 'An act to
establish an uniform rule of naturalization, and to repeal the acts heretofore passed on that subject.' "

In its Section 2, this Act already provided that:

"SEC. 2. And be it further enacted, That when any alien who shall have complied with the first condition
specified in ,the first section of the said original act, and who shall have pursued the directions
prescribed in the second section of the said act, may die, before he is actually naturalized, the widow
and the children of such alien shall be considered as citizens of the United States, and shall be entitled to
rights and privileges as such, upon taking the oaths prescribed by law." (CHAP. XLVII, 2 Stat. 292)

6. In the Act of July 30, 1813, the disqualification of enemy aliens was removed as follows:

"CHAP. XXXVI. — An Act supplementary to the acts heretofore passed on the subject of an uniform rule
of naturalization. (a)

"Be it enacted by the Senate and House of Representatives of the United States of America in Congress
assembled, That persons resident within the United States, or the territories thereof, on the eighteenth
day of June, in the year one thousand eight hundred and twelve, who had before that day made
declaration according to law, of their intention to become citizens of the United States, or who by the
existing laws of the United States, were on that day entitled to become citizens, without making such
declaration, may be admitted to become citizens thereof, notwithstanding they shall be alien enemies at

367
the times and in the manner prescribed by the laws heretofore passed on that subject: Provided, That
nothing herein contained shall be taken or construed to interfere with or prevent the apprehension and
removal, agreeably to law, of any alien enemy at any time previous to the actual naturalization of such
alien." (Chap. XXXVI, 3 Stat. 53)

7. Neither the Act of March 22, 1816 nor those of May 26, 1824 and May 24, 1828 made any
change in the above requirements. (Chap. XXXII, 3 Stat. 258; Chap. CLXX-XVI, 4 Stat. 69; and Chap. CXVI,
4 Stat. 310).

8. Then the Act of February 10, 1855, important because it gave alien wives of citizens ,the status
of citizens, was enacted providing:

"CHAP. LXXI. — An Act to secure the Right of Citizenship to Children of Citizens of the United States born
out of the Limits thereof.

"Be it enacted by the Senate and House of Representatives of the United States of America in Congress
assembled, That persons heretofore born, or hereafter to be born, out of the limits and jurisdiction of
the United States, whose fathers were or shall be at the time of their birth citizens of the United States,
shall be deemed and considered and are hereby declared to be citizens of the United States: Provided,
however, That the rights of citizenship shall not descend to persons whose fathers never resided in the
United States.

"SEC. 2. And be it further enacted, That any woman who might lawfully be naturalized under the existing
laws, married, or who shall be married to a citizen of the United States, shall loyal be deemed and taken
to be a citizen." (Chap. LXXI, 10 Stat. 604.)

9. The Act of July 14, 1870 mainly provided only for penalties for certain acts related to
naturalization, as punished thereby, but added in its Section 7 "that the naturalization laws are hereby
extended to aliens of African nativity and to African descent." (Chap. CCLIV, 16 Stat. 254.)

10. The Act of February 1, 1876 contained no relevant amendment. (Chap. 5, 19 Stat. 2.)

11. When the statutes of the United States were revised on June 22, 1874, the naturalization law of
the country was embodied in Sections 2165-2174 of saddle Revised Statutes. This contained no racial
disqualification. In fact, it reenacted ;Section 2 of the Act of February 10, 1855 as its Section 1994
thereof, thus:

"SEC. 1994. Any person who is now or may hereafter be married to a citizen of the United States,
and who might herself be lawfully naturalized, shall be deemed a citizen." (18 Stat. 351.)

12. The Act of May 6, 1882 provided expressly that no State court or court of the United States shall
admit Chinese to citizenship. (Chap. 126, Sec. 14, 22 Stat. , 61.)

368
13. The Act of August 9, 1888 extended the benefits of American citizenship to Indian woman
married to Americans thus:

"CHAP. 818. — An Act in relation to marriage between white men and Indian women.

"Be it enacted, That no white man, not otherwise a member of any tribe of Indians, who may hereafter
marry, an Indian woman, member of any Indian tribe in the United States, or any of its Territories except
the five civilized tribes in the Indian Territory, shall by such marriage hereafter acquire any right to any
tribal property, privilege, or interest whatever to which any member of such tribe is entitled.

"SEC. 2. That every Indian woman, member of any such tribe of Indians, who may hereafter be married
to any citizen of the United States, is hereby declared to become by such marriage a citizen of the
United States, with all the right, privileges, and immunities of any such citizen, being a married woman:

"Provided, That nothing in this act contained shall impair or in any way affect the right or title of such
married woman to any tribal property or any interest therein.

"SEC. 2. That whenever the marriage of any white man with any Indian woman, a member of any such
tribe of Indians, is required or offered to be proved in any judicial ,proceeding, evidence of the
admission of such fact by the party against whom the proceeding is had, or evidence of general repute,
or of cohabitation as married persons, or any other circumstantial or presumptive evidence from which
the fact may be inferred, shall be competent. (Aug. 9, 1888) " [25 Stat. 392, Suppl. 1.]

14. The Act of April 19, 1900 extended American citizenship to all citizens of the Republic of Hawaii
on August 12, 1898 as well as the laws of the United States to said Republic, including, of course, those
on naturalization. (Chap. 339, Sec. 4, 31 Stat. 141.)

15. On June 29, 1906. "An Act to establish a Bureau of Immigration and Naturalization, and to
provide a uniform rule for the naturalization of aliens throughout the United States" was approved. No
reference was made therein to "free white persons''; it merely provided in its Section 7 that:

"SEC. 7. That no person who disbelieve in or who is opposed to organized government, or who is a
member of or affiliated with any organization entertaining and teaching such disbelief in or opposition
to organized government, or who advocates or teaches the duty, necessity, or propriety of the unlawful
assaulting or killing of any officer or officers, either of specific individuals or of officers generally of the
Government of the United States, or of any other organized government, because of his or their official
character, or who is a polygamist, shall be naturalized or be made a citizen of the United States." (36
Stat. 598)

Incidentally, the 6th paragraph of its Section 4 provided:

"Sixth. When any alien who has declared his intention to become a citizen of the United States dies
before he is actually naturalized the widow and minor children of such alien may, by complying with the
other provisions of this Act, be naturalized without making any declaration of intention." (36 Stat. 598)

369
16. By the Act of March 2, 1907, alien women who acquired American citizenship by marriage
retained said citizenship, if she continued to reside in the United States and did not renounce it, or, if
she resided outside of the United States by registering with the U.S. Consul of her place of residence.
(CHAP. 2534, Sec. 4, 34 Stat. 1229.)

17. Since United States legislation treats naturalization and citizenship per se separately, Section
1994 of the Revised Statutes remained untouched. In the Act of February 24, 1911 it was provided:

"Be it enacted by the Senate and House of Representatives of the United States of America in Congress
assembled, That when any alien, who has declared his intention to become a citizen of the United
States, becomes insane before he is actually naturalized, and his wife shall thereafter make a homestead
entry under the land laws of the United States, she and their minor children may, by complying with the
other provisions of the naturalization laws be naturalized without making any declaration of intention."
(36 Stat. 929.)

18. The Act of August 11, 1916 merely validated entries filed in certain countries. (CHAP. 316, 39
Stat. 926.)

19. In the Act of May 9, 1918, the U.S. Congress amended the naturalization laws to make possible
the admission of Filipino navy servicemen, and understandably, because of the war then, it provided:

"Seventh. Any native-born Filipino of the age of twenty-one years and upward who has declared his
intention to become a citizen of the United States and who has enlisted or may hereafter enlist in the
United States Navy or Marine Corps or the Naval Auxiliary Service, and who, after service of not less
than three years, may be honorably discharged therefrom, or who may receive an ordinary discharge
with recommendation for reenlistment; or any alien, or any Porto Rican not a citizen of the United
States, . . . ." (40 Stat. 542.)

20. On September 22, 1922, "An Act Relative to the Naturalization and citizenship of married
women" was appareled repeating Section 1994 of the Revised Statutes and otherwise adopting a
different attitude as regards the citizenship and naturalization of married women thus:

"Be it enacted by the Senate cleaned House of Representatives of the United States of America in
Congress assembled, That the right of any woman to become a naturalized citizen of the United States
shall not be denied or abridged because of her sex or because she is a married woman.

"Sec. 2. That any woman who marries a citizen of the United States after the passage of this Act, or any
woman whose husband is naturalized after the passage of this Act, shall not become a citizen of the
United States by reason of such marriage or naturalization; but, if eligible to citizenship, she may be
naturalized upon full and complete compliance with all the requirements of the naturalization laws, with
the following exceptions:

(a) No declaration of intention shall be required;

370
(b) In lieu of the five-year period of residence within the United States and the one-year period of
residence within the State or Territory where the naturalization court is held, she shall have resided
continuously in the United States Hawaii, Alaska, or Porto Rico for at least one year immediately
preceding the filing of the petition.

"Sec. 3. That a woman citizen of the United States shall not cease to be a citizen of the United States by
reason of her marriage after the passage of this Act, unless she makes a formal renunciation of her
citizenship before a court having jurisdiction over naturalization of aliens; Provided, That any woman
citizen who marries an alien ineligible to citizenship shall cease to be a citizen of the United States. If at
the termination of the marital status she is a citizen of the United States she shall retain her citizenship
regardless of her residence. If during the continuance of the marital status she resides continuously for
two years in a foreign State of which her husband is a citizen or subject, or for five years continuously
outside the United States, she shall thereafter be subject to the same presumption as is a naturalized
citizen of the United States under the second paragraph of section 2 of the Act entitled "An Act in
reference to the expatriation of citizens and their protection abroad," approved March 2 1907. Nothing
herein shall be construed to repeal or amend the provisions of Revised Statutes 1999 or of section 2 of
the Expatriation Act of 1907 with reference to expatriation.

"Sec. 4. That a woman who, before the passage of this Act, has lost her United States citizenship by
reason of her marriage to an alien eligible for citizenship, may be naturalized as provided by section 2 of
this Act: Provided, That no certificate of arrival shall be required to be filed with her petition if during
the continuance of the marital status she; shall have resided within the United States. After her
naturalization she shall have the same citizenship status as if her marriage had taken place after the
passage of this Act.

"Sec. 5. That no woman whose husband is not eligible to citizenship shall be naturalized during the
continuance of the marital status.

"Sec. 6. That section 1994 of the Revised Statutes and section 4 of the Expatriation Act of 1907 are
repealed. Such repeal shall not terminate citizenship acquired or retained under either of such sections
nor restore citizenship lost under section 4 of the Expatriation Act of 1907.

"Sec. 7. That section 3 of the Expatriation Act of 1901 is repealed. Such repeal shall not restore
citizenship lost under such section nor terminate citizenship resumed under such section. A woman who
has resumed under such section citizenship lost by marriage shall, upon the passage of this Act, have for
all purposes the same citizenship status as immediately preceding her marriage." (Chap. 411, 42 Stat.
10211022.)

21. When "The Code of the Laws of to United States of America of a General and Permanent
Character in Force on December 7, 1925" was approved, the provisions, corresponding to the
disqualifications for naturalization and the citizenship and naturalization of women embodied therein
were:

371
"367. Naturalization of woman; sex or marriage not a bar. — The right of any woman to become a
naturalized citizen of the United States shall not be denied or abridged because of her sex or because
she is a married woman. ( Sept. 22, 1922, c.411, 1, 42 Stat. 1021.)

"368. Same; women marrying citizens or persons becoming naturalized; procedure. — Any woman
who marries a citizen of the United States after September 22, 1922, or any woman whose husband is
naturalized after that date, shall not become a citizen of the United States by reason of such marriage or
naturalization; but, if eligible to citizenship, she may be naturalized upon full and complete compliance
with the following exceptions:

(a) No declaration of intention shall be required;

(b) In lieu of the five-year period of residence within the United States and the one-year period of
residence within the State or Territory where the naturalization court is held, she shall have resided
continuously in the United States, Hawaii, Alaska, or Porto Rico for at least one year immediately
preceding the filing of the petition. (Sept. 22, 1922, c. 411, § 2, 42 Stat. 1022.)

"369. Same; women who have lost citizenship by Marrying aliens eligible to citizenship; procedure. —
A woman, who, before September 22, 1922, has lost her United States citizenship by reason for her
marriage to an alien eligible for citizenship, may be naturalized as provided in the preceding section. No
certificate of arrival shall be required to be filed with her petition if during the continuance of the
marital status she shall have resided within the United States. After her naturalization she shall have the
same citizenship status as if her marriage had taken place after September 22, 1922. (Swept. 22, 1922, c.
411, § 4, 42 Stat. 1022.)

"370. Same; Women married to persons ineligible to citizenship. — No woman whose husband is not
eligible to citizenship shall be naturalized during the continuance of the marital status. (Swept. 22, 1922,
c. 411, 5, 42 Stat. 1022.)

"371. Same, wife of alien declaring becoming insane before naturalization; minor children. — When
any alien, who has declared his intention to become a citizen of the United States, becomes insane
before he is actually naturalized, and his wife shall thereafter make a homestead entry under the land
laws of the United States, she and their minor children may, by complying with the other provisions of
the naturalization laws be naturalized without making any declaration of intention. (Feb. 24, 1911, c.
151, 36 Stat. 929.)" (Chap. 9, 44 Stat. 156, 158.)

which, of course, must be read together with the provisions on inadmissibility of Chinese, anarchists,
polygamists, non-English speaking persons, etc. in Sections 363-365 of the same Code.

22. The Act of May 26, 1926 extended naturalization privileges to alien veterans of World War I,
thus:

"Be it enacted by the Senate and House of Representatives of the United States of America in Congress
assembled, That (a) as used in this Act, the term "alien veteran" means an individual, a member of the
military or naval forces of the United States at any time after April 5, 1917, and before November 12,

372
1918, who is now an alien not ineligible to citizenship; but does not include (1) any individual at anytime
during such period or thereafter separated from such forces under other than honorable conditions, (2)
any conscientious objector who performed no military duty whatever or refused to wear the uniform, or
(3) any alien at any time during such period or thereafter discharged from the military or naval forces on
account of his alienage.

(b) Terms defined in the Immigration Act of 1924 shall, when used in this Act, have the meaning
assigned to such terms in that Act.

"Sec. 2. An alien veteran shall for the purposes of the Immigration Act of 1924 be considered as a non-
quota immigrant, but shall be subject to all the other provisions of that Act and of the immigration laws,
except that —

(a) He shall not be subject to the head tax imposed by section 2 of the Immigration Act of 1917;

(b) He shall not be required to pay any fee under section 2 or section 7 of the Immigration Act of
1924;

(c) If otherwise admissible, he shall not be excluded under section 3 of the Immigration Act of 1917,
unless excluded under the provisions of that section relating to —

(1) Persons afflicted with a loathsome or dangerous contagious disease, except tuberculosis in any
form;

(2) Polygamy;

(3) Prostitutes, procurers, or other like immoral persons;

(4) Contract laborers;

(5) Persons previously deported;

(6) Persons convicted of crime.

"Sec. 3. The unmarried child under eighteen years of age, the wife, or the husband, of an alien veteran
shall, for the purposes of the Immigration Act of 1924, be considered as a non quota immigrant when
accompanying or following within six months to join him, but shall be subject to all other provisions of
that Act and of the immigration laws.

'Sec. 4. The foregoing provisions of this Act shall not apply to any alien unless the immigration visa is
issued to him before the expiration of one year after the enactment of this Act." (Chap. 398, 44 Stat.
654-655.)

23. The Act of June 21, 1930 authorized repatriation of certain veterans of World War I. (Chap. 559,
46 Stat. 791.)

24. On March 3, 1931, the Act of September 22, 1922 as amended as follows:

373
"Sec. 4.(a) Section 3 of the Act entitled "An Act relative to the naturalization and citizenship of
married women," approved September 22, 1922, as amended, is amended to read as follows:

'Sec. 3.(a) A woman citizen of the United States shall not cease to be a citizen of the United States
by reason of her marriage after this section, as amended, takes effect, unless she makes a formal
renunciation of her citizenship before a court having jurisdiction over naturalization of aliens.

'(b) Any woman who before this section, as amended takes effect, has lost her United States
citizenship by residence abroad after marriage to an alien or by marriage to an alien ineligible to
citizenship may, if she has not acquired any other nationality by affirmative act, be naturalized in the
manner prescribed in section 4 of this Act, as amended. Any woman who was a citizen of the United
States at birth shall not be denied naturalization under section 4 on account of her race.

'(c) No woman shall be entitled to naturalization under section 4 of this Act, as amended, if her
United States citizenship originated solely by a reason of her marriage to a citizen of the United States or
by reason of the acquisition of United States citizenship by her husband.'

"(b) Section 5 of such Act of September 22, 1922, is repealed." (Chap. 442, 46 Stat. 1511-1512.)

25. The Act of May 25, 1932 contained the following somewhat pertinent provisions:

"Be it enacted by the Senate and House of Representatives of the United States of America in Congress
assembled, That

(a) an alien veteran, as defined in section 1 of the Act of May 26, 1926 (ch. 398, 44 Stat. 654; title 8,
sec. 241, U.S. C. Supp. 1), if residing in the United States, be entitled at any time within two years after
the enactment of this Act to naturalization upon the same terms, conditions, and exemptions which
would have been accorded to such alien if he had petitioned before the armistice of the World War,
except that (1) such alien shall be required to prove that immediately preceding the date of this petition
he has resided continuously within the United States for at least two years, in pursuance of a legal
admission for permanent residence, and that during all such period he has behaved as a person of good
moral character; (2) if such admission was subsequent to March 3, 1924, such alien shall file with his
petition a certificate of arrival issued by the Commissioner of Naturalization; (3) final action shall not be
had upon the petition until at least ninety days have elapsed after filing of such petition; and (4) such
alien shall be required to appear and file his petition in person, and to take the prescribed oath of
allegiance in open court. Such residence and good moral character shall be proved either by the
affidavits of two credible witnesses who are citizens of the United States, or by depositions by two such
witnesses made before a naturalization examiner, for each place of residence.

"(b) All petitions for citizenship made outside the United States in accordance with the seventh
subdivision of section 4 of the Naturalization Act of June 29, 1906, as amended, upon which
naturalization has not been heretofore granted, are hereby declared to be invalid for all purposes.

374
"Sec. 2. (a) The seventh subdivision of section 4 of the Naturalization Act of June 29, 1906, as
amended, is amended by striking out 'the National Guard or Naval Militia of any State, Territory, or the
District of Columbia, or the State Militia in Federal Service.'

"(b) This section shall not be applied in the case of any individual whose petition for naturalization
has been filed before the enactment of this Act.

"Sec. 3. The last proviso in the first paragraph of the seventh subdivision of section 4 of such Act of June
29, 1906, as amended, is amended by striking out the period at the end thereof and inserting in lieu
thereof a semicolon and the following: 'except that this proviso shall not apply in the case of service on
American-owned vessels by an alien who has been lawfully admitted to the United States for permanent
residence.'

"Sec. 4. Section 32 of such Act of June 29, 1906, as amended, is amended by adding at the end thereof
the following new subdivisions:

'(c) If the name of any naturalized citizen has, subsequent to naturalization, been changed by order
of a court of competent jurisdiction, or by marriage, the citizen may, upon the payment to the
commissioner of a fee of $10, make application (accompanied by two photographs of the applicant) for
a new certificate of citizenship in the new name of such citizen. If the commissioner finds the name of
the applicant to have been changed as claimed he shall issue to the applicant a new certificate with one
of such photographs of the applicant affixed thereto.

'(d) The Commissioner of Naturalization is authorized to make and issue, without fee, certifications
of any part of the naturalization records of any court, or of any certificate of citizenship, for use in
complying with any statute, State or Federal, or in any judicial proceeding. Any such certification shall be
admitted in evidence equally with the original from which such certification was made in any case in
which the original thereof might be admissible as evidence. No such certification shall be made by any
clerk of court except upon order of the court.'

"Sec. 5. So much of subdivision (a) of section 33 of such Act of June 29, 1906, as amended, as read 'Upon
obtaining a certificate from the Secretary of Labor showing the date, place, and manner of arrival in the
United States,' is hereby repealed.

"Sec. 6. Section 4 of the Act entitled 'An Act to supplement the naturalization laws, and for other
purposes,' approved March 2, 1929, is amended by striking out the period at the end thereof and
inserting in lieu thereof a semicolon and the following: 'except that no such certificate shall be required
if the entry was on or before June 29, 1906.'

"Sec. 7. Despite the provisions of subdivision (a) of section 1 of the Act entitled 'An Act making it a
felony with penalty for certain aliens to enter the United States of America under certain conditions in
violation of law,' approved March 4, 1929, as amended, an alien, if otherwise admissible, shall not be
excluded from admission to the United States under the provisions of such subdivision after the
expiration of one year after the date of deportation if, prior to his reembarkation at a place outside of

375
the United States, or prior to his application in foreign contiguous territory for admission to the United
States, the Secretary of Labor, in his discretion, shall have granted such alien permission to reapply for
admission.

"Sec. 8. The compilation of the statistics to show races nationalities, and other information, authorized
and directed to be prepared by the Commissioner of Naturalization, shall be completed and published at
the same time, as near as practicable, as the Publication of the statistics of the 1930 census except that
reports covering the census of 1910 shall be completed and submitted not later than January 31, 1933,
and reports covering the census of 1920 not later than December 31, 1938. Such statistics shall show the
records of registry made under the provisions of the Act entitled 'An Act to supplement the
naturalization laws, and for other purposes,' approved March 2, 1929. Payment for the equipment used
in preparing such compilation shall be made from appropriations for miscellaneous expenses of the
Bureau of Naturalization.

"Sec. 9. The Secretary of the Treasury, upon the recommendation of the Secretary of Labor, is
authorized to provide quarters without payment of rent, in the building occupied by the Naturalization
Service in New York City, for a photographic studio operated by welfare organizations without profit and
solely for the benefit of aliens seeking naturalization. Such studio shall be under the supervision of the
Commissioner of Naturalization.

"Sec. 10. The tenth subdivision of section 4 of the Act of June 29, 1906 (ch. 3592, 34 Stat. 598), as
amended by the Act of May 9, 1918 (ch. 69, 40, 40 Stat. 545; U.S.C., title 8 sec. 377), is hereby amended
to read as follows:

'Tenth. That any person not an alien enemy, who resided uninterruptedly within the United States
during the period of five years next preceding July 1, 1920, and was on that date otherwise qualified to
become a citizen of the United States, except that he had not made a declaration of intention required
by law and who during or prior to that time, because of misinformation regarding his citizenship status
erroneously exercised the rights and performed the duties of a citizen of the United States in good faith,
may file the petition for naturalization prescribed by law without making the preliminary declaration of
intention required of other aliens, and upon satisfactory proof to the court that he has so acted may be
admitted as a citizen of the United States upon complying in all respects with the other requirements of
the naturalization law.' (Chap. 203, 47 Stat. 165-167.)

26. By June 27, 1952, the right of a person to be naturalized could no longer be denied by reason of
race or sex or because such person was married, although various disqualifications were still
maintained, such as lack understanding, capacity to read and write English, or of the principles of the
constitution and form of government of the United States, being opposed to organized government of
law, favoring totalitarian forms of government, deserters from the armed forces, etc. (Secs. 1422 to
1426, USCA 8-9, 1953; See also Secs. 1421 et seq., USCA 8, 1970.)

B — QUALIFICATIONS

376
Apart from the above disqualifications, the statutes referred tea contained express requirements as to
qualifications as follows:

(1) The Act of 1790 required residence, good moral character and adherence to the principles of the
United States Constitution.

(2) That of 1795 required a declaration of intention. residence, adherence to the U.S. Constitution,
good moral character and no title of nobility.

(3) That of 1798 referred only declaration of intention and residence.

(4) That of 1802 required residence, renunciation of allegiance to former government, adherence to
U.S. Constitution, good moral character and declaration of intention.

(5) That of 1804 was practically I the same as that of 1802.

(6) So also were those of 1813, 1816 and 1824.

(7) That of 1828 mentioned only residence and declaration of intention.

(8) Those of 1855, 1870 and 1888 amended the law in other respects.

(9) That of 1906 contained the following provisions:

"SEC. 4. That an alien may be admitted to become a citizen of the United States in the following manner
and not otherwise:

"First. He A hall declare on oath before the clerk of any court authorized by this Act to naturalize aliens,
or his authorized deputy, in the district in which such alien resides, two years at least prior to his
admission, and after he has reached the age of eighteen years, that it is bona fide his intention to
become a citizen of the united States, and to renounce forever all allegiance and fidelity to any foreign
prince, potentate, state, or sovereignty, and particularly, by name, to the prince, potentate, state, or
sovereignty of which the alien may be at the time a citizen or subject. And such declaration shall set
forth, the name, age, occupation, personal description, place of birth, last foreign residence and
allegiance, the date of arrival, the, name of the vessel, if any, in which he came to the United states, and
the present place of residence in the United States of said alien: Provided, however, That no alien who,
in conformity with the law in force at the date of his declaration, has declared his intention to become a
citizen of the United States shall be required to renew such declaration.

"Second. Not less than two years nor more than seven years after he has made such declaration of
intention he shall make and file, in duplicate, a petition in writing, signed by the applicant in his own
handwriting and duly verified, in which petition such applicant shall state his full name, his place of
residence (by street and number, if possible), his occupation, and, if possible, the date and place of his
birth; the place from which he emigrated, and the date and place of his arrival in the United States, and,
if he entered through a port, the name of the vessel on which he arrived; the time when and the place
and name of the court where he declared his intention to become a citizen of the United States; if he is

377
married he shall state the name of his wife and, if possible, the country of her nativity and her place of
residence at the time of filing his petition; and if he has children, the name, date, and place of birth and
place of residence of each child living at the time of his petition: Provided, That if he has filed his
declaration before the passage of this Act he shall not be required to sign the petition in his own
handwriting.

"The petition shall set forth that he is not a disbeliever in or opposed to organized government, or a
member of or affiliated with any organization or body of persons teaching disbelief in or opposed to
organized government, a polygamist or believer in the practice of polygamy, and that it is his intention
to become a citizen of the United States and to renounce absolutely and forever all allegiance and
fidelity to any foreign prince, potentate, state, or sovereignty, and particularly by name to the prince,
potentate, state, or sovereignty of which he at the time of filing of his petition may be a citizen or
subject, and that it is his intention to reside permanently within the United States, and whether or not
he has been denied admission a, a citizen of the United States, and, if denied, the ground or grounds of
such denial, the court or courts in which such decision was rendered, and that the cause for such denial
has since been cured or removed, and every fact material to his naturalization filed required to be
proved upon the final hearing of his application.

"The petition shall also be verified by the affidavits of at least two credible witnesses, who are citizens of
the United States, and who shall state in their affidavits that they have personally known the applicant
to be a resident of the United States for a period of at least five years continuously, and of the State,
Territory, or district in which the application is made for a period of at least one year immediately
preceding the date of the filing of his petition, and that they each have personal knowledge that the
petitioner is a person of good moral character, and that he is in every way qualified, in their opinion, to
be admitted as a citizen of the United States.

"At the time of filing of his petition there shall be filed with the clerk of the court a certificate from the
Department of Commerce and Labor, if the petitioner arrives in the United States after the passage of
this Act, stating the date, place and manner of his arrival in the United States, and the declaration of
intention of such petitioner, which certificate and declaration shall be attached to and made a part of
said petition.

"Third. He shall, before he is admitted to citizenship, declare on oath in open court that he will support
the Constitution of the United States, and he absolutely and entirely renounces and abjures all
allegiance and fidelity to any foreign prince potentate, state, or sovereignty, and particularly by name to
the prince, potentate, state, or sovereignty of which he was before a citizen or subject; that he will
support and defend the Constitution and laws of the United States against all enemies, foreign and
domestic, and bear true faith and allegiance to the same.

"Fourth. It shall be made to appear to the satisfaction of the court admitting any alien to citizenship
that immediately preceding the date of his application he has resided continuously within the United
States five years at least, and within the State or Territory where such court is at the time held one year
at least, and that during that time he has behaved as a man of good moral character, attached to the

378
principles of the Constitution of the United States, and well disposed to the good order and happiness of
the same. In addition to the oath of the applicant, the testimony of at least two witnesses, citizens of
the United States, as to the facts of residence, moral character, and attachment to the principles of the
Constitution shall be required, and the name, ,place of residence, and occupation of each witness shall
be set forth in the record.

"Fifth. In case the alien applying to be admitted to citizenship has borne any hereditary title, or has
been of any of the orders of nobility in the kingdom or state from which he came, he shall, in addition to
the above requisites make an express renunciation of his title or order of nobility in the court to which
his application is made, and his renunciation shall be recorded in the court.

"Sixth. When any alien who has declared his intention to become a citizen of the United States dies
before he is actually naturalized the widow and minor children of such alien may, by complying with the
other provisions of this Act, be naturalized without making any declaration of intention." (34 stat. 596-
98.)

10. Those of 1911 and 1916 contained amendments as to other matters.

11. That of 1918 provided for different qualifications for Filipinos, Porto Ricans, etc. for
naturalization in addition to service in the U.S. Navy or Philippine Constabulary.

12. Those of years after 1922 when Section 1994 was repealed would have no material bearing in
this case.

Amen.

Separate Opinions

REYES, J.B..L., dissenting:

I regret not being able to assent to the opinion of Mr. Justice Barredo. Without prejudice to a more
extended opinion and I in order not to delay, the release of the decision, I am expressing here the basic
reasons for my disconformity.

The pivotal problem is whether the provision of section, 15 of our Naturalization Law (Commonwealth
Act No. 473) requires that an alien woman, married to a Filipino citizen, must prove that she possesses
all the qualifications and none of the disqualifications prescribed by said law, in order to be deemed a
Filipino citizen. The affirmative has been the constant doctrine of this Court since 1957, in the first Ly
Giok Ha case (101 Phil. 459) or at the very least since 1959, in Lee Suan Sy vs. Galling, 106 Phil. 713.

This established doctrine would now be set aside primarily on the basis that section 15 of our
Naturalization Law is a verbatim reproduction or exact copy of section 1994 of the Revised Statutes of
the United States (Act of Congress of February 10, 1855); that because said section of the Revised
Statutes had been uniformly construed by American courts as requiring merely that the woman

379
marrying a citizen should not be disqualified herself from becoming a citizen, that a similar
interpretation must be given to the aforesaid section 15 our own Naturalization Act.

This view might be tenable if the Philippine statute had been in its entirety a reproduction of the
American model. But where the coincidence is limited to a section of the Philippine statute, which taken
as a whole is different in requirements and spirit, I submit that the rule advocated by the main opinion
does not apply, and that our section 15 should be construed conformably to the context and
intendment of , the statute of which it is a part, and in harmony wealth the whole.

It is worth nabbing that the American law of naturalization stresses primarily the disqualifications for
citizenship (see USCA, Title 8, secs. 363 to 366 and 378). The only positive qualifications are "bona fide
intention to become a citizen of the Unlighted States and to renounce forever all allegiance and fidelity"
to a previous sovereign (Do., sec. 372) and residence for the specified period. This is particularly true of
the American law at the time the first law was enacted concerning acquisition of citizenship by alien
women married to U.S. citizens (the Act of February 10, 1855). Under such conditions, it is
understandable that the interpretation of the words "who might herself lawfully be naturalized" Should
be that the marrying alien woman should not be disqualified from becoming a citizen.

But our naturalization law separates qualifications from disqualifications; the positive qualifications
under section 3 thereof express a policy of restriction, as to candidates for naturalization as much as the
disqualifications under section 4. An a !it has been shown in our decision in the second Lo Giok Ha case
(Ly Giok Ha vs. Galling, L-21332, March 18, 1966, 16 SCRA 416) that those not disqualified under section
4 would not necessarily qualify under section 3, even if the residence qualification were disregarded. In
other words, by giving to section 15 of our Naturalization Law the effect of excluding only those women
suffering from disqualification under section 3 could result in admitting to citizenship women that
section 2 intends to exclude. In these circumstances, I do not see why the American interpretation of
the words "who might herself be lawfully naturalized" should be considered binding in this jurisdiction.

The spirit of the American law, decidedly favorable to the absorption of immigrants, is not embodied in
our Constitution and laws, because of the nationalistic spirit of the latter.

In effect, the main decision introduces marriage to a citizen as a means of acquiring citizenship, a way
not to contemplated by Article IV of the Constitution.

I am not unaware of the fact that the decisions of this Court have made very difficult the acquisition of
citizenship by alien woman marrying Filipinos. But the remedy lies in a change of the statute. And it is
not amiss to observe here that since 1959, when the present doctrine on the matter was adopted, the
Legislature has not expressed any dissent therefrom, when it could have easily altered or clarified the
legal provisions affected if Congress were convinced that this Court had misinterpreted its intent.

Concepcion, C . J ., Zaldivar and Makasiar, JJ ., concur.

380
Republic v. Batuigas, G.R. No. 183110, October 7, 2013

[G.R. No. 183110. October 7, 2013.]

REPUBLIC OF THE PHILIPPINES, petitioner, vs. AZUCENA SAAVEDRA BATUIGAS, respondent.

DECISION

DEL CASTILLO, J p:

"It is universally accepted that a State, in extending the privilege of citizenship to an alien wife of one of
its citizens could have had no other objective than to maintain a unity of allegiance among the members
of the family." 1

This Petition for Review on Certiorari 2 assails the May 23, 2008 Decision 3 of the Court of Appeals (CA)
in CA G.R. CV No. 00523, which affirmed the January 31, 2005 Decision 4 of the Regional Trial Court
(RTC), Branch 29, Zamboanga del Sur that granted the Petition for Naturalization 5 of respondent
Azucena Saavedra Batuigas (Azucena). ECcaDT

Factual Antecedents

On December 2, 2002, Azucena filed a Petition for Naturalization before the RTC of Zamboanga del Sur.
The case was docketed as Naturalization Case No. 03-001 and raffled to Branch 29 of said court.

Azucena alleged in her Petition that she believes in the principles underlying the Philippine Constitution;
that she has conducted herself in a proper and irreproachable manner during the period of her stay in
the Philippines, as well as in her relations with the constituted Government and with the community in
which she is living; that she has mingled socially with the Filipinos and has evinced a sincere desire to
learn and embrace their customs, traditions, and ideals; that she has all the qualifications required
under Section 2 and none of the disqualifications enumerated in Section 4 of Commonwealth Act No.
473 (CA 473); 6 that she is not opposed to organized government nor is affiliated with any association or
group of persons that uphold and teach doctrines opposing all organized governments; that she is not
defending or teaching the necessity or propriety of violence, personal assault, or assassination for the
success and predominance of men's ideas; that she is neither a polygamist nor believes in polygamy;
that the nation of which she is a subject is not at war with the Philippines; that she intends in good faith
to become a citizen of the Philippines and to renounce absolutely and forever all allegiance and fidelity
to any foreign prince, potentate, state or sovereignty, and particularly to China; and that she will reside
continuously in the Philippines from the time of the filing of her Petition up to the time of her
naturalization. ATcaEH

After all the jurisdictional requirements mandated by Section 9 7 of CA 473 had been complied with, the
Office of the Solicitor General (OSG) filed its Motion to Dismiss 8 on the ground that Azucena failed to
allege that she is engaged in a lawful occupation or in some known lucrative trade. Finding the grounds
relied upon by the OSG to be evidentiary in nature, the RTC denied said Motion. 9 Thereafter, the
hearing for the reception of Azucena's evidence was then set on May 18, 2004. 10

381
Neither the OSG nor the Office of the Provincial Prosecutor appeared on the day of the hearing. Hence,
Azucena's counsel moved that the evidence be presented ex-parte, which the RTC granted. Accordingly,
the RTC designated its Clerk of Court as Commissioner to receive Azucena's evidence. 11 During the
November 5, 2004 ex-parte hearing, no representative from the OSG appeared despite due notice. 12
DAETcC

Born in Malangas, Zamboanga del Sur on September 28, 1941 to Chinese parents, 13 Azucena has never
departed the Philippines since birth. She has resided in Malangas, Zamboanga del Sur from 1941-1942;
in Margosatubig, Zamboanga del Sur from 1942-1968; in Bogo City for nine months; in Ipil, Zamboanga
del Sur from 1969-1972; in Talisayan, Misamis Oriental from 1972-1976; and, in Margosatubig,
Zamboanga del Sur, thereafter, up to the filing of her Petition.

Azucena can speak English, Tagalog, Visayan, and Chavacano. Her primary, secondary, and tertiary
education were taken in Philippine schools, i.e., Margosatubig Central Elementary School in 1955, 14
Margosatubig Academy in 1959, 15 and the Ateneo de Zamboanga in 1963, 16 graduating with a degree
in Bachelor of Science in Education. She then practiced her teaching profession at the Pax High School
for five years, in the Marian Academy in Ipil for two years, and in Talisayan High School in Misamis
Oriental for another two years. 17 SIaHTD

In 1968, at the age of 26, Azucena married Santiago Batuigas 18 (Santiago), a natural-born Filipino
citizen. 19 They have five children, namely Cynthia, Brenda, Aileen, Dennis Emmanuel, and Edsel James.
20 All of them studied in Philippine public and private schools and are all professionals, three of whom
are now working abroad. 21

After her stint in Talisayan High School, Azucena and her husband, as conjugal partners, engaged in the
retail business of and later on in milling/distributing rice, corn, and copra. As proof of their income,
Azucena submitted their joint annual tax returns and balance sheets from 2000-2002 22 and from 2004-
2005. 23 The business name and the business permits issued to the spouses' store, 'Azucena's General
Merchandising,' are registered in Santiago's name, 24 and he is also the National Food Authority
licensee for their rice and corn business. 25 During their marital union, the Batuigas spouses bought
parcels of land in Barrio Lombog, Margosatubig. 26

To prove that she has no criminal record, Azucena submitted clearances issued by the Philippine
National Police of Zamboanga del Sur Provincial Office and by the National Bureau of Investigation. 27
She also presented her Health Examination Record 28 declaring her as physically and mentally fit.

To further support Azucena's Petition, Santiago and witnesses Eufemio Miniao and Irineo Alfaro
testified. HSATIC

Ruling of the Regional Trial Court

On January 31, 2005, the RTC found that Azucena has amply supported the allegations in her Petition.
Among these are her lack of a derogatory record, her support for an organized government, that she is
in perfect health, that she has mingled with Filipinos since birth and can speak their language, that she

382
has never had any transgressions and has been a law abiding citizen, that she has complied with her
obligations to the government involving her business operations, and that the business and real
properties she and Santiago own provide sufficient income for her and her family. Thus, the RTC ruled:

. . . In sum, the petitioner has all the qualifications and none of the disqualifications to be admitted as
citizen of the Philippines in accordance with the provisions of the Naturalization Law.

WHEREFORE, premises considered, the petition is hereby granted.

SO ORDERED. 29 AIHDcC

In its Omnibus Motion, 30 the OSG argued that the ex-parte presentation of evidence before the Branch
Clerk of Court violates Section 10 of CA 473, 31 as the law mandates public hearing in naturalization
cases.

Rejecting this argument in its March 21, 2005 Order, 32 the RTC held that the public has been fully
apprised of the naturalization proceedings and was free to intervene. The OSG and its delegate, the
Provincial Prosecutor, are the only officers authorized by law to appear on behalf of the State, which
represents the public. Thus, when the OSG was furnished with a copy of the notice of hearing for the
reception of evidence ex-parte, there was already a sufficient compliance with the requirement of a
public hearing. aTcIAS

The OSG then appealed the RTC judgment to the CA, 33 contending that Azucena failed to comply with
the income requirement under CA 473. The OSG maintained that Azucena is not allowed under the
Retail Trade Law (Republic Act No. 1180) to engage directly or indirectly in the retail trade. Hence, she
cannot possibly meet the income requirement. And even if she is allowed, her business is not a
"lucrative trade" within the contemplation of the law or that which has an appreciable margin of income
over expenses in order to provide for adequate support in the event of unemployment, sickness, or
disability to work. The OSG likewise disputed Azucena's claim that she owns real property because aliens
are precluded from owning lands in the country.

The OSG further asserted that the ex-parte proceeding before the commissioner is not a "public
hearing" as ex-parte hearings are usually done in chambers, without the public in attendance. It claimed
that the State was denied its day in court because the RTC, during the May 18, 2004 initial hearing,
immediately allowed the proceeding to be conducted ex-parte without even giving the State ample
opportunity to be present. IcEACH

Azucena countered that although she is a teacher by profession, she had to quit to help in the retail
business of her husband, and they were able to send all their children to school. 34 It is highly unlikely
that she will become a public charge as she and her spouse have enough savings and could even be
given sufficient support by their children. She contended that the definition of "lucrative trade/income"
should not be strictly applied to her. Being the wife and following Filipino tradition, she should not be
treated like male applicants for naturalization who are required to have their own "lucrative trade."

383
Azucena denied that the hearing for her Petition was not made public, as the hearing before the Clerk of
Court was conducted in the court's session hall. Besides, the OSG cannot claim that it was denied its day
in court as notices have always been sent to it. Hence, its failure to attend is not the fault of the RTC.

Ruling of the Court of Appeals

In dismissing the OSG's appeal, 35 the CA found that Azucena's financial condition permits her and her
family to live with reasonable comfort in accordance with the prevailing standard of living and
consistent with the demands of human dignity. It said: caADSE

Considering the present high cost of living, which cost of living tends to increase rather than decrease,
and the low purchasing power of the Philippine currency, petitioner-appellee, together with her Filipino
husband, nonetheless, was able to send all her children to college, pursue a lucrative business and
maintain a decent existence. The Supreme Court, in recent decisions, adopted a higher standard in
determining whether a petitioner for Philippine citizenship has a lucrative trade or profession that
would qualify him/her for admission to Philippine citizenship and to which petitioner has successfully
convinced this Court of her ability to provide for herself and avoid becoming a public charge or a
financial burden to her community. . . . 36

As for the other issue the OSG raised, the CA held that the RTC had complied with the mandate of the
law requiring notice to the OSG and the Provincial Prosecutor of its scheduled hearing for the Petition.
IESAac

Thus, the instant Petition wherein the OSG recapitulates the same arguments it raised before the CA,
i.e., the alleged failure of Azucena to meet the income and public hearing requirements of CA 473.

Our Ruling

The Petition lacks merit.

Under existing laws, an alien may acquire Philippine citizenship through either judicial naturalization
under CA 473 or administrative naturalization under Republic Act No. 9139 (the "Administrative
Naturalization Law of 2000"). A third option, called derivative naturalization, which is available to alien
women married to Filipino husbands is found under Section 15 of CA 473, which provides that:

"[a]ny woman who is now or may hereafter be married to a citizen of the Philippines and who might
herself be lawfully naturalized shall be deemed a citizen of the Philippines."

Under this provision, foreign women who are married to Philippine citizens may be deemed ipso facto
Philippine citizens and it is neither necessary for them to prove that they possess other qualifications for
naturalization at the time of their marriage nor do they have to submit themselves to judicial
naturalization. Copying from similar laws in the United States which has since been amended, the
Philippine legislature retained Section 15 of CA 473, which then reflects its intent to confer Filipino
citizenship to the alien wife thru derivative naturalization. 37 ECaHSI

384
Thus, the Court categorically declared in Moy Ya Lim Yao v. Commissioner of Immigration: 38

Accordingly, We now hold, all previous decisions of this Court indicating otherwise notwithstanding, that
under Section 15 of Commonwealth Act 473, an alien woman marrying a Filipino, native born or
naturalized, becomes ipso facto a Filipina provided she is not disqualified to be a citizen of the
Philippines under Section 4 of the same law. Likewise, an alien woman married to an alien who is
subsequently naturalized here follows the Philippine citizenship of her husband the moment he takes his
oath as Filipino citizen, provided that she does not suffer from any of the disqualifications under said
Section 4. 39

As stated in Moy Ya Lim Yao, the procedure for an alien wife to formalize the conferment of Filipino
citizenship is as follows: aESICD

Regarding the steps that should be taken by an alien woman married to a Filipino citizen in order to
acquire Philippine citizenship, the procedure followed in the Bureau of Immigration is as follows: The
alien woman must file a petition for the cancellation of her alien certificate of registration alleging,
among other things, that she is married to a Filipino citizen and that she is not disqualified from
acquiring her husband's citizenship pursuant to Section 4 of Commonwealth Act No. 473, as amended.
Upon the filing of said petition, which should be accompanied or supported by the joint affidavit of the
petitioner and her Filipino husband to the effect that the petitioner does not belong to any of the groups
disqualified by the cited section from becoming naturalized Filipino citizen . . ., the Bureau of
Immigration conducts an investigation and thereafter promulgates its order or decision granting or
denying the petition. 40

Records however show that in February 1980, Azucena applied before the then Commission on
Immigration and Deportation (CID) for the cancellation of her Alien Certificate of Registration (ACR) No.
030705 41 by reason of her marriage to a Filipino citizen. The CID granted her application. However, the
Ministry of Justice set aside the ruling of the CID as it found no sufficient evidence that Azucena's
husband is a Filipino citizen 42 as only their marriage certificate was presented to establish his
citizenship. TcHCDE

Having been denied of the process in the CID, Azucena was constrained to file a Petition for judicial
naturalization based on CA 473. While this would have been unnecessary if the process at the CID was
granted in her favor, there is nothing that prevents her from seeking acquisition of Philippine citizenship
through regular naturalization proceedings available to all qualified foreign nationals. The choice of
what option to take in order to acquire Philippine citizenship rests with the applicant. In this case,
Azucena has chosen to file a Petition for judicial naturalization under CA 473. The fact that her
application for derivative naturalization under Section 15 of CA 473 was denied should not prevent her
from seeking judicial naturalization under the same law. It is to be remembered that her application at
the CID was denied not because she was found to be disqualified, but because her husband's citizenship
was not proven. Even if the denial was based on other grounds, it is proper, in a judicial naturalization
proceeding, for the courts to determine whether there are in fact grounds to deny her of Philippine
citizenship based on regular judicial naturalization proceedings. AaHcIT

385
As the records before this Court show, Santiago's Filipino citizenship has been adequately proven. Under
judicial proceeding, Santiago submitted his birth certificate indicating therein that he and his parents are
Filipinos. He also submitted voter's registration, land titles, and business registrations/licenses, all of
which are public records. He has always comported himself as a Filipino citizen, an operative fact that
should have enabled Azucena to avail of Section 15 of CA 473. On the submitted evidence, nothing
would show that Azucena suffers from any of the disqualifications under Section 4 of the same Act.

However, the case before us is a Petition for judicial naturalization and is not based on Section 15 of CA
473 which was denied by the then Ministry of Justice. The lower court which heard the petition and
received evidence of her qualifications and absence of disqualifications to acquire Philippine citizenship,
has granted the Petition, which was affirmed by the CA. We will not disturb the findings of the lower
court which had the opportunity to hear and scrutinize the evidence presented during the hearings on
the Petition, as well as determine, based on Azucena's testimony and deportment during the hearings,
that she indeed possesses all the qualifications and none of the disqualifications for acquisition of
Philippine citizenship. TcHDIA

The OSG has filed this instant Petition on the ground that Azucena does not have the qualification
required in no. 4 of Section 2 of CA 473 as she does not have any lucrative income, and that the
proceeding in the lower court was not in the nature of a public hearing. The OSG had the opportunity to
contest the qualifications of Azucena during the initial hearing scheduled on May 18, 2004. However,
the OSG or the Office of the Provincial Prosecutor failed to appear in said hearing, prompting the lower
court to order ex parte presentation of evidence before the Clerk of Court on November 5, 2004. The
OSG was also notified of the ex parte proceeding, but despite notice, again failed to appear. The OSG
had raised this same issue at the CA and was denied for the reasons stated in its Decision. We find no
reason to disturb the findings of the CA on this issue. Neither should this issue further delay the grant of
Philippine citizenship to a woman who was born and lived all her life, in the Philippines, and devoted all
her life to the care of her Filipino family. She has more than demonstrated, under judicial scrutiny, her
being a qualified Philippine citizen. On the second issue, we also affirm the findings of the CA that since
the government who has an interest in, and the only one who can contest, the citizenship of a person,
was duly notified through the OSG and the Provincial Prosecutor's office, the proceedings have complied
with the public hearing requirement under CA 473. ASIDTa

No. 4, Section 2 of CA 473 provides as qualification to become a Philippine citizen:

4. He must own real estate in the Philippines worth not less than five thousand pesos, Philippine
currency, or must have known lucrative trade, profession, or lawful occupation.

Azucena is a teacher by profession and has actually exercised her profession before she had to quit her
teaching job to assume her family duties and take on her role as joint provider, together with her
husband, in order to support her family. Together, husband and wife were able to raise all their five
children, provided them with education, and have all become professionals and responsible citizens of
this country. Certainly, this is proof enough of both husband and wife's lucrative trade. Azucena herself
is a professional and can resume teaching at any time. Her profession never leaves her, and this is more

386
than sufficient guarantee that she will not be a charge to the only country she has known since birth.
SaETCI

Moreover, the Court acknowledged that the main objective of extending the citizenship privilege to an
alien wife is to maintain a unity of allegiance among family members, thus:

It is, therefore, not congruent with our cherished traditions of family unity and identity that a husband
should be a citizen and the wife an alien, and that the national treatment of one should be different
from that of the other. Thus, it cannot be that the husband's interests in property and business activities
reserved by law to citizens should not form part of the conjugal partnership and be denied to the wife,
nor that she herself cannot, through her own efforts but for the benefit of the partnership, acquire such
interests. Only in rare instances should the identity of husband and wife be refused recognition, and we
submit that in respect of our citizenship laws, it should only be in the instances where the wife suffers
from the disqualifications stated in Section 4 of the Revised Naturalization Law. 43

We are not unmindful of precedents to the effect that there is no proceeding authorized by the law or
by the Rules of Court, for the judicial declaration of the citizenship of an individual. 44 "Such judicial
declaration of citizenship cannot even be decreed pursuant to an alternative prayer therefor in a
naturalization proceeding." 45 cTSHaE

This case however is not a Petition for judicial declaration of Philippine citizenship but rather a Petition
for judicial naturalization under CA 473. In the first, the petitioner believes he is a Filipino citizen and
asks a court to declare or confirm his status as a Philippine citizen. In the second, the petitioner
acknowledges he is an alien, and seeks judicial approval to acquire the privilege of becoming a Philippine
citizen based on requirements required under CA 473. Azucena has clearly proven, under strict judicial
scrutiny, that she is qualified for the grant of that privilege, and this Court will not stand in the way of
making her a part of a truly Filipino family.

WHEREFORE, the Petition is DENIED. The May 23, 2008 Decision of the Court of Appeals in CA-G.R. CV
No. 00523 which affirmed the January 31, 2005 Decision of the Regional Trial Court, Branch 29,
Zamboanga del Sur that granted the Petition for Naturalization, is hereby AFFIRMED. Subject to
compliance with the period and the requirements under Republic Act No. 530 which supplements the
Revised Naturalization Law, let a Certificate of Naturalization be issued to AZUCENA SAAVEDRA
BATUIGAS after taking an oath of allegiance to the Republic of the Philippines. Thereafter, her Alien
Certificate of Registration should be cancelled.

SO ORDERED. CcAIDa

Carpio, Brion, Perez and Perlas-Bernabe, JJ., concur.

387
Frivaldo vs. COMELEC, G.R. No. 120295, June 28, 1996

[G.R. No. 120295. June 28, 1996.]

JUAN G. FRIVALDO, petitioner, vs. COMMISSION ON ELECTIONS, and RAUL R. LEE, respondents.

[G.R. No. 123755. June 28, 1996.]

RAUL R. LEE, petitioner, vs. COMMISSION ON ELECTIONS and JUAN G. FRIVALDO, respondents.

Sixto S. Brillantes, Jr., Juanito G. Arcilla and Teodoro M. Jumamil for Juan G. Frivaldo.

Felix Carao, Jr., Ferdinand Laguna, Gavinoo Barlin and Bernardo P. Fernandez for Raul Lee.

SYLLABUS

1. POLITICAL LAW; CITIZENSHIP; QUALIFICATION REQUIRED FOR ALL ELECTIVE LOCAL OFFICIALS. —
The Local Government Code of 1991 [Republic Act No. 7160] expressly requires Philippine citizenship as
a qualification for elective local officials, including that of provincial governor. Philippine citizenship is an
indispensable requirement for holding an elective public office, and the purpose of the citizenship
qualification is none other than to ensure that no alien, i.e., no person owing allegiance to another
nation, shall govern our people and our country or a unit of territory thereof. Now, an official begins to
govern or to discharge his functions only upon his proclamation and on the day the law mandates his
term of office to begin. The law intended CITIZENSHIP to be a qualification distinct from being a VOTER,
even if being a voter presumes being a citizen first. It also stands to reason that the voter requirement
was included as another qualification (aside from "citizenship"), not to reiterate the need for nationality
but to require that the official be registered as a voter IN THE AREA OR TERRITORY he seeks to govern.
i.e., the law states: "a registered voter in the barangay, municipality, city, or province . . . where he
intends to be elected." It should be emphasized that the Local Government Code requires an elective
official to be a registered voter. It does not require him to vote actually. Hence, registration — not the
actual voting — is the core of this "qualification". In other words, the law's purpose in this second
requirement is to ensure that the prospective official is actually registered in the area he seeks to govern
— and not anywhere else.

2. ID.; ID.; MANNERS OF REACQUISITION UNDER PHILIPPINE LAWS. — Under Philippine law,
citizenship may be reacquired by direct act of Congress, by naturalization or by repatriation.

3. ID.; ID.; IMMIGRATION LAW; P.D. 725 CREATED A RIGHT AND REMEDY TO REACQUIRE
PHILIPPINE CITIZENSHIP NOT ONLY FOR THE BENEFIT OF FILIPINO WOMEN WHO MARRIED ALIENS
BEFORE THE 1973 CONSTITUTION TOOK EFFECT BUT ALSO OF OTHER NATURAL BORN FILIPINO WHO
LOST THEIR PHILIPPINE CITIZENSHIP. — A reading of P.D. 725 immediately shows that it creates a new
right, and also provides for a new remedy, thereby filling certain voids in our laws. Thus, in its preamble,
P.D. 725 expressly recognizes the plight of "many Filipino women (who) had lost their Philippine
citizenship by marriage to aliens" and who could not, under the existing law (C.A. No. 63, as amended)
avail of repatriation until "after the death of their husbands or the termination of their marital status"

388
and who could neither be benefited by the 1973 Constitution's new provision allowing "a Filipino
woman who marries an alien to retain her Philippine citizenship . . ." because "such provision of the new
Constitution does not apply to Filipino women who had married aliens before said constitution took
effect." Thus, P.D. 725 granted a new right to these women — the right to re-acquire Filipino citizenship
even during their marital coverture, which right did not exist prior to P.D. 725. On the other hand, said
statute also provide a new remedy and a new right in favor of other "natural born Filipinos who (had)
lost their Philippine citizenship but now desire to re-acquire Philippine citizenship," because prior to the
promulgation of P.D. 725 such former Filipinos would have had to undergo the tedious and cumbersome
process of naturalization, but with the advent of P.D. 725 they could now re-acquire their Philippine
citizenship under the simplified procedure of repatriation.

4. ID.; ID.; REPATRIATION; REQUIREMENTS THEREOF ARE LESS TEDIOUS AND CUMBERSOME
COMPARED TO NATURALIZATION. — The requirements of repatriation under P.D. No. 725 are not
difficult to comply with, nor are they tedious and cumbersome. In fact, P.D. 725 itself requires very little
of an applicant, and even the rules and regulations to implement the said decree were left to the Special
Committee to promulgate. This is not unusual since, unlike in naturalization where an alien covets a
first-time entry into Philippine political life, in repatriation the applicant is a former natural-born Filipino
who is merely seeking to reacquire his previous citizenship.

5. ID.; ID.; ID.; WHEN MAY BE GIVEN A RETROACTIVE EFFECT; CASE AT BAR. — While it is true that
the law was already in effect at the time that Frivaldo became an American citizen, nevertheless, it is not
only the law itself (P.D. 725) which is to be given retroactive effect, but even the repatriation granted
under said law to Frivaldo on June 30, 1995 is to be deemed to have retroactive to the date of his
application therefor, August 17, 1994. The reason for this is simply that if, as in this case, it was the
intent of the legislative authority that the law should apply to past events — i.e., situations and
transactions existing even before the law came into being — in order to benefit the greatest number of
former Filipinos possible thereby enabling them to enjoy and exercise the constitutionally guaranteed
right of citizenship, and such legislative intention is to be given the fullest effect and expression, then
there is all the more reason to have the law apply in a retroactive or retrospective manner to situations,
events and transactions subsequent to the passage of such law. That is, the repatriation granted to
Frivaldo on June 30, 1995 can and should be made to take effect as of date of his application. As earlier
mentioned, there is nothing in the law that would bar this or would show a contrary intention on the
part of the legislative authority; and there is no showing that damage or prejudice to anyone, or
anything unjust or injurious would result from giving retroactivity to his repatriation.

6. ID.; ID.; THE LOCAL GOVERNMENT CODE DOES NOT SPECIFY WHEN SUCH QUALIFICATION SHALL
BE POSSESSED. — Under Sec. 39 of the Local Government Code, it will be noted that the law does not
specify any particular date or time when the candidate must possess citizenship, unlike that for
residence (which must consist of at least one year's residency immediately preceding the day of
election) and age (at least twenty three years of age on election day). Even from a literal (as
distinguished from liberal) construction, it should be noted that Section 39 of the Local Government
Code speaks of "Qualifications" of 'ELECTIVE OFFICIALS', not of candidates. Literally, such qualifications
unless otherwise expressly conditioned, as in the case of age and residence — should thus be possessed

389
when the "elective [or elected] official" begins to govern, i.e., at the time he is proclaimed and at the
start of his term. Section 39, par. (a) (thereof) speaks of "elective local official" while par. (b) to (f) refer
to "candidates." The citizenship requirement in the Local Government Code is to be possessed by an
elective official at the latest as of the time he is proclaimed and at the start of the term of office to
which he has been elected.

7. STATUTORY CONSTRUCTION; REPEAL OF LAW; BY SUBSEQUENT ONES. — Laws are repealed


only by subsequent ones [Art. 7, Civil Code of the Philippines] and a repeal may be express or implied. It
is obvious that no express repeal was made because then President Aquino in her memorandum —
based on the copy furnished us by Lee — did not categorically and/or impliedly state that PD 725 was
being repealed or was being rendered without any legal effect. In fact, she did not even mention it
specifically by its number or text.

8. ID.; ID.; BY IMPLICATION IS NOT FAVORED; EXCEPTION. — It is a basic rule of statutory


construction that repeals by implication are not favored. An implied repeal will not be allowed "unless it
is convincingly and unambiguously demonstrated that the two laws are clearly repugnant and patently
inconsistent that they cannot co-exist". The memorandum of then President Aquino cannot even be
regarded as a legislative enactment, for not every pronouncement of the Chief Executive even under the
Transitory Provisions of the 1987 Constitution can nor should be regarded as an exercise of her law-
making powers. At best, it could be treated as an executive policy addressed to the Special Committee
to hall the acceptance and processing of applications for repatriation pending whatever "judgment the
first Congress under the 1987 Constitution" might make. In other words, the former President did not
repeal P.D. 725 but left it to the first Congress — once created — to deal with the matter. If she had
intended to repeal such law, she should have unequivocally said so instead of referring the matter to
Congress. The fact is she carefully couched her presidential issuance in terms that clearly indicated the
intention of "the present government, in the exercise of prudence and sound discretion" to leave the
matter of repeal to the new Congress. Any other interpretation of the said Presidential Memorandum,
such as is now being proffered to the Court by Lee, would visit unmitigated violence not only upon
statutory construction but on common sense as well.

9. ID.; THE LAW PRESUMES THAT THE LAW MAKING-BODY INTENDED RIGHT AND JUSTICE TO
PREVAIL. — In case of doubt in the interpretation or application of laws, it is to be presumed that the
lawmaking body intended right and justice to prevail. [Art. 10, Civil Code of the Philippines)

10. CIVIL LAW; RETROSPECTIVE OPERATIONS OF STATUTES; WHEN EFFECTIVE. — It is true that
under Art. 4 of the Civil Code of the Philippines, "(l)aws shall have no retroactive effect, unless the
contrary is provided." But there are settled exceptions to this general rule, such as when the statute is
CURATIVE or REMEDIAL in nature or when it CREATES NEW RIGHTS. According to TOLENTINO, curative
statutes are those which undertake to cure errors and irregularities, thereby validating judicial or
administrative proceedings, acts of public officers, or private deeds and contracts which otherwise
would not produce their intended consequences by reason of some statutory disability or failure to
comply with some technical requirement. They operate on conditions already existing, and are
necessarily retroactive in operation. Agpalo, [Agpalo, Statutory Construction, 1990 ed., pp. 270-271] on

390
the other hand, says that curative statutes are "healing acts . . . curing defects and adding to the means
of enforcing existing obligations . . . (and) are intended to supply defects, abridge superfluities in existing
laws, and curb certain evils. . . . By their very nature, curative statutes are retroactive . . . (and) reach
back to past events to correct errors or irregularities and to render valid and effective attempted acts
which would be otherwise ineffective for the purpose the parties intended." On the other hand,
remedial or procedural laws, i.e., those statutes relating to remedies or modes of procedure, which do
not create new or take away vested rights, but only operate in furtherance of the remedy or
confirmation of such rights, ordinarily do not come within the legal meaning of a retrospective law, nor
within the general rule against the retrospective operation of statutes.

11. POLITICAL LAW; OMNIBUS ELECTION CODE; PROVIDED REMEDY TO QUESTION INELIGIBILITY OF
A CANDIDATE. — Section 253 of the Omnibus Election Code gives any voter, presumably including the
defeated candidate, the opportunity to question the ELIGIBILITY (or the disloyalty) of a candidate. This is
the only provision of the Code that authorizes a remedy on how to contest before the COMELEC an
incumbent's ineligibility arising from failure to meet the qualifications enumerated under Sec. 39 of the
Local Government Code. Such remedy of Quo Warranto can be availed of "within ten days after
proclamation" of the winning candidate. Hence, it is only at such time that the issue of ineligibility may
be taken cognizance of by the Commission.

12. ID.; ID.; PETITION TO DENY DUE COURSE OR TO CANCEL A CERTIFICATE OF CANDIDACY UNDER
SECTION 78 IS MERELY DIRECTORY. — Section 78 of the Omnibus Election Code is merely directory as
Section 6 of R.A. No. 6646 authorizes the Commission to try and decide petitions for disqualifications
even after the elections.

13. ID.; ELECTIONS; THE RULE IS THE INELIGIBILITY OF A CANDIDATE RECEIVING MAJORITY VOTES
DOES NOT ENTITLE THE ELIGIBLE CANDIDATE RECEIVING THE NEXT HIGHEST NUMBER OF VOTES TO BE
DECLARED ELECTED. — "The rule, therefore, is: the ineligibility of a candidate receiving majority votes
does not entitle the eligible candidate receiving the next highest number of votes to be declared
elected. A minority or defeated candidate cannot be deemed elected to the office."

14. ID.; ID.; ELECTORAL LAWS SHOULD BE LIBERALLY AND EQUITABLY CONSTRUED TO GIVE FULLEST
EFFECT TO THE MANIFEST WILL OF OUR PEOPLE. — This Court has time and again liberally and equitably
construed the electoral laws of our country to give fullest effect to the manifest will of our people, for in
case of doubt, political laws must be interpreted to give life and spirit to the popular mandate freely
expressed through the ballot. Otherwise stated, legal niceties and technicalities cannot stand in the way
of the sovereign will. Consistently, we have held: ". . . (L)aws governing election contests must be
liberally construed to the end that the will of the people in the choice of public officials may not be
defeated by mere technical objections." In any action involving the possibility of a reversal of the
popular electoral choice, this Court must exert utmost effort to resolve the issues in a manner that
would give effect to the will of the majority, for it is merely sound public policy to cause elective offices
to be filled by those who are the choice of the majority. To successfully challenge a winning candidate's
qualifications, the petitioner must clearly demonstrate that the ineligibility is so patently antagonistic to

391
constitutional and legal principles that overriding such ineligibility and thereby giving effect to the
apparent will of the people, would ultimately create greater prejudice to the very democratic
institutions and juristic traditions that our Constitution and laws so zealously protect and promote. The
real essence of justice does not emanate from quibbling over patchwork legal technicality. It proceeds
from the spirit's gut consciousness of the dynamic role of law as a brick in the ultimate development of
the social edifice. Thus, the Court struggled against and eschewed the easy, legalistic, technical and
sometimes harsh anachronisms of the law in order to evoke substantial justice in the larger social
context consistent with Frivaldo's unique situation approximating venerability in Philippine political life.

15. ID.; INTERNATIONAL LAW; A STATE DETERMINES ONLY THOSE WHO ARE ITS OWN CITIZENS —
NOT WHO ARE THE CITIZENS OF OTHER COUNTRIES. — Since our courts are charged only with the duty
of the determining who are Philippine nationals, we cannot rule on the legal question of who are or who
are not Americans. It is basic in international law that a State determines ONLY those who are its own
citizens — not who are the citizens of other countries.

PUNO, J., Concurring Opinion:

1. POLITICAL LAW; DECLARATION OF PRINCIPLES AND STATE POLICIES; SOVEREIGNTY OF THE


PEOPLE, CONSTRUED. — The sovereignty of our people is the primary postulate of the 1987
Constitution. For this reason, it appears as the first in our declaration of principles and state policies.
Thus, Section 1 of Article II of our fundamental law proclaims that "[t]he Philippines is a democratic and
republican State. Sovereignty resides in the people and all government authority emanates from them."
The same principle served as the bedrock of our 1973 and 1935 Constitutions. [The 1987 Constitution
added the word "democratic" in the statement of the principle.] It is one of the few principles whose
truth has been cherished by the Americans as self-evident. Section 4, Article IV of the U.S. Constitution
makes it a duty of the Federal government to guarantee to every state a "republican form of
government." With understandable fervor, the American authorities imposed republicanism as the
cornerstone of our 1935 Constitution then being crafted by its Filipino farmers. Borne out of the 1986
people power EDSA revolution, our 1987 Constitution is more people-oriented. Thus, Section 4 of Article
II provides as a state policy that the prime duty of the Government is "to serve and protect the people."
Section 1, Article XI also provides that ". . . public officers . . . must at all times be accountable to the
people . . . " Sections 15 and 16 of Article XIII define the role and rights of people's organizations. Section
5(2) of Article XVI mandates that "[t]he state shall strengthen the patriotic spirit and nationalist
consciousness of the military, and respect for people's rights in the performance of their duty." And
Section 2 of Article XVII provides that "amendments to this Constitution may likewise be directly
proposed by the people through initiative . . ." All these provisions and more are intended to breathe
more life to the sovereignty of our people. To be sure, the sovereignty of our people is not a kabalistic
principle whose dimensions are buried in mysticism. Its metes and bounds are familiar to the framers of
our Constitutions. They knew that in its broadest sense, sovereignty is meant to be supreme, the just
summi imperu, the absolute right to govern. Former Dean Vicente Sinco states that an essential quality
of sovereignty is legal omnipotence, viz: "Legal theory establishes certain essential qualities inherent in
the nature of sovereignty. The first is legal omnipotence. This means that the sovereign is legally
omnipotent and absolute in relation to other legal institutions. It has the power to determine exclusively

392
its legal competence. Its powers are original, not derivative. It is the sole judge of what it should do at
any given time." Citing Barker, Principles of Social and Political Theory, p. 59 (1952 ed.), he adds that a
more amplified definition of sovereignty is that of "a final power of final legal adjustment of all legal
issues." The U.S. Supreme Court expressed the same thought in the landmark case of Yick Wo v.
Hopkins, 118 U.S. 356, where it held that ". . . sovereignty itself is, of course, not subject to law, for it is
the author and source of law; but in our system, while sovereign powers are delegated to the agencies
of government, sovereignty itself remains with the people, by whom and for whom all government
exists and acts."

2. ID.; ID.; REPRESENTATIVE DEMOCRACY DISTINGUISHED FROM PURE DEMOCRACY. — In our


Constitution, the people established a representative democracy as distinguished from a pure
democracy. Justice Isagani Cruz explains ". . . A republic is a representative government, a government
run by and for the people. It is not a pure democracy where the people govern themselves directly. The
essence of republicanism is representation and renovation, the selection by the citizenry of a corps of
public functionaries who derive their mandate for the people and act on their behalf, serving for a
limited period only, after which they are replaced or retained, at the option of their principal. Obviously,
a republican government is a responsible government whose officials hold and discharge their position
as a public trust and shall, according to the Constitution, 'at all times be accountable to the people' they
are sworn to serve. The purpose of a republican government it is almost needless to state, is the
promotion of the common welfare according to the will of the people themselves."

3. ID.; ELECTION CASES; THE COURT SHOULD STRIVE TO ALIGN THE WILL OF THE LEGISLATURE
WITH THE WILL OF THE SOVEREIGN PEOPLE. — In election cases, we should strive to align the will of the
legislature as expressed in its law with the will of the sovereign people as expressed in their ballots. For
law to reign, it must respect the will of the people. For in the eloquent prose of Mr. Justice Laurel, ". . .
an enfranchised citizen is a particle of popular sovereignty and is the ultimate source of established
authority." (Moya v. del Fierro, 69 Phil. 199) The choice of the governed on who shall be their governor
merits the highest consideration by all agencies of government. In cases where the sovereignty of the
people is at stake, we must not only be legally right but also politically correct. We cannot fail by making
the people succeed.

DAVIDE, JR., J., Dissenting Opinion"

1. POLITICAL LAW; LOCAL GOVERNMENT CODE OF 1991; ELECTION; WHEN THE PRESCRIBED
QUALIFICATION SHALL BE POSSESSED. — Justice Davide departs from the view in the ponencia is that
Section 39 of the Local Government Code of 1991 does not specify the time when the citizenship
requirement must be met, and that being the case, then it suffices that citizenship be possessed upon
commencement of the term of the office involved. Section 39 actually prescribes the qualifications of
elective local officials and not those of an elected local official. These adjectives are not synonymous, as
the ponencia seems to suggest, the first refers to the nature of the office, which requires the process of
voting by the electorate involved; while the second refers to a victorious candidate for an elective office.
The section unquestionably refers to elective — not elected — local officials. It falls under Title Two
entitled ELECTIVE OFFICIALS; under Chapter 1 entitled Qualifications and Election; and paragraph (a)

393
thereof begins with the phrases "An elective local official," while paragraphs (b) to (f) thereof speak of
candidates. It is thus obvious that Section 39 refers to no other than the qualifications of candidates for
elective local offices and their election. Hence, in no way may the section be construed to mean that
possession of qualifications should be reckoned from the commencement of the term of office of the
elected candidate. It is not at all true that Section 39 does not specify the time when the citizenship
requirement must be possessed. I submit that the requirement must be satisfied, or that Philippine
citizenship must be possessed, not merely at the commencement of the term, but at an earlier time, the
latest being election day itself. Section 39 is not at all ambiguous nor uncertain that it meant this to be,
as one basic qualification of an elective local official is that he be "A REGISTERED VOTER IN THE
BARANGAY, MUNICIPALITY, CITY OR PROVINCE . . . WHERE HE INTENDS TO VOTE." This simply means
that he possesses all the qualifications to exercise the right of suffrage. The fundamental qualification
for the exercise of this sovereign right is the possession of Philippine citizenship. No less than the
Constitution makes it the first qualification.

2. ID.; ID.; ID.; AVAILABLE REMEDIES ON QUESTIONS OF INELIGIBILITY FOR PUBLIC OFFICE. — Quo
warranto is not the sole remedy available to question a candidate's ineligibility for public office. Section
78 of the Omnibus Election Code allows the filing of a petition to deny due course to or cancel the
certificate of candidacy on the ground that any material representation contained therein as required by
Section 74, is false. Section 74, in turn, requires that the person filing the certificate of candidacy must
state inter alia, that he is eligible for the office, which means that he has all the qualifications (including,
of course, fulfilling the citizenship requirement) and none of the disqualifications as provided by law.
The petitioner under Section 78 may be filed at any time not later than 25 days from the filing of the
certificate of candidacy. Rule 25 of the Revised COMELEC rules of procedure allows the filing of a
petition for disqualification on the ground of failure to possess all the qualifications of a candidate as
provided by the Constitution or by existing laws, "any day after the last day for filing of certificates of
candidacy but not later than the date of proclamation."

3. ID.; CITIZENSHIP; STEPS FOR THE REACQUISITION OF CITIZENSHIP BY REPATRIATION. — The


steps to reacquire Philippine Citizenship by repatriation under P.D. No. 725 are: (1) filing the application;
(2) action by the committee; and (3) taking of the oath of allegiance if the application is approved. It is
only UPON TAKING THE OATH OF ALLEGIANCE that the applicant is deemed ipso jure to have reacquired
Philippine citizenship. If the decree had intended the oath taking to retroact to the date of the filing of
the application, then it should not have explicitly provided otherwise.

4. ID.; INTERNATIONAL LAW; STATELESSNESS, CONSTRUED. — Statelessness may be either de jure,


which is the status of individuals stripped of their nationality by their former government without having
all opportunity to acquired another; or de facto, which is the status of individuals possessed of a
nationality whose country does not give them protection outside their own country, and who are
commonly, albeit imprecisely, referred to as refugees (JORGE R. COQUIA, et al., Conflict of Laws Cases,
Materials and Comments, 1995, ed., 290). Under Chapter I, Article 1 of the United Nations Convention
Regarding the Status of Stateless Persons (Philippine Treaty Series, Compiled and Annotated by Haydee
B. Yorac, vol. III, 363), a stateless person is defined as "a person who is not considered as a national by
any State under the operation of its law.

394
5. ID.; DECLARATION OF PRINCIPLES AND STATE POLICIES; DOCTRINE OF PEOPLE'S SOVEREIGNTY,
CONSTRUED. — The doctrine of people's sovereignty is founded on the principles of democracy and
republicanism and refers exclusively to the sovereignty of the people of the Philippines. Section 1 of
Article II of the 1987 Constitution is quite clear on this. And the Preamble makes it clear when it
solemnly opens it with a clause "We, the sovereign Filipino people. . . " Thus, the sovereignty is an
attribute of the Filipino people as one people, one body. That sovereign power of the Filipino people
cannot be fragmentized by looking at it as the supreme authority of the people of any of the political
subdivisions to determine their own destiny; neither can we convert and treat every fragment as the
whole. In such a case, this Court would provide the formula for the division and destruction of the State
and render the Government ineffective and inutile.

DECISION

PANGANIBAN, J p:

The ultimate question posed before this Court in these twin cases is: Who should be declared the
rightful governor of Sorsogon —

(i) Juan G. Frivaldo, who unquestionably obtained the highest number of votes in three successive
elections but who was twice declared by this Court to be disqualified to hold such office due to his alien
citizenship, and who now claims to have re-assumed his lost Philippine citizenship thru repatriation;

(ii) Raul R. Lee, who was the second placer in the canvass, but who claims that the votes cast in
favor of Frivaldo should be considered void; that the electorate should be deemed to have intentionally
thrown away their ballots; and that legally, he secured the most number of valid votes; or

(iii) The incumbent Vice-Governor, Oscar G. Deri, who obviously was not voted directly to the
position of governor, but who according to prevailing jurisprudence should take over the said post
inasmuch as, by the ineligibility of Frivaldo, a "permanent vacancy in the contested office has occurred"?

In ruling for Frivaldo, the Court lays down new doctrines on repatriation, clarifies/reiterates/amplifies
existing jurisprudence on citizenship and elections, and upholds the superiority of substantial justice
over pure legalisms.

G.R. No. 123755

This is a special civil action under Rules 65 and 58 of the Rules of Court for certiorari and preliminary
injunction to review and annul a Resolution of the respondent Commission on Elections (Comelec), First
Division, 1 promulgated on December 19, 1995 2 and another Resolution of the Comelec en banc
promulgated February 23, 1996 3 denying petitioner's motion for reconsideration.

The Facts

On March 20, 1995, private respondent Juan G. Frivaldo filed his Certificate of Candidacy for the office
of Governor of Sorsogon in the May 8, 1995 elections. On March 23, 1995, petitioner Raul R. Lee,

395
another candidate, filed a petition 4 with the Comelec docketed as SPA No. 95-028 praying that Frivaldo
"be disqualified from seeking or holding any public office or position by reason of not yet being a citizen
of the Philippines", and that his Certificate of Candidacy be cancelled. On May 1, 1995, the Second
Division of the Comelec promulgated a Resolution 5 granting the petition with the following disposition:
6

"WHEREFORE, this Division resolves to GRANT the petition and declares that respondent is
DISQUALIFIED to run for the Office of Governor of Sorsogon on the ground that he is NOT a citizen of the
Philippines. Accordingly, respondent's certificate of candidacy is cancelled."

The Motion for Reconsideration filed by Frivaldo remained unacted upon until after the May 8, 1995
elections. So, his candidacy continued and he was voted for during the elections held on said date. On
May 11, 1995, the Comelec en banc 7 affirmed the aforementioned Resolution of the Second Division.

The Provincial Board of Canvassers completed the canvass of the election returns and a Certificate of
Votes 8 dated May 27, 1995 was issued showing the following votes obtained by the candidates for the
position of Governor of Sorsogon:

Antonio H. Escudero, Jr. 51,060

Juan G. Frivaldo 73,440

Raul R. Lee 53,304

Isagani P. Ocampo 1,925

On June 9, 1995, Lee filed in said SPA No. 95-028, a (supplemental) petition 9 praying for his
proclamation as the duly-elected Governor of Sorsogon.

In an order 10 dated June 21, 1995, but promulgated according to the petition "only on June 29, 1995,"
the Comelec en banc directed "the Provincial Board of Canvassers of Sorsogon to reconvene for the
purpose of proclaiming candidate Raul Lee as the winning gubernatorial candidate in the province of
Sorsogon on June 29, 1995 . . ." Accordingly, at 8:30 in the evening of June 30, 1995, Lee was proclaimed
governor of Sorsogon.

On July 6, 1995, Frivaldo filed with the Comelec a new petition, 11 docketed as SPC No. 95-317, praying
for the annulment of the June 30, 1995 proclamation of Lee and for his own proclamation. He alleged
that on June 30, 1995, at 2:00 in the afternoon, he took his oath of allegiance as a citizen of the
Philippines after "his petition for repatriation under P.D. 725 which he filed with the Special Committee
on Naturalization in September 1994 had been granted". As such, when "the said order (dated June 21,
1995) (of the Comelec) . . . was released and received by Frivaldo on June 30, 1995 at 5:30 o'clock in the
evening, there was no more legal impediment to the proclamation (of Frivaldo) as governor . . ." In the
alternative, he averred that pursuant to the two cases of Labo vs. Comelec, 12 the Vice-Governor — not
Lee — should occupy said position of governor.

396
On December 19, 1995, the Comelec First Division promulgated the herein assailed Resolution 13
holding that Lee, "not having garnered the highest number of votes," was not legally entitled to be
proclaimed as duly-elected governor; and that Frivaldo, "having garnered the highest number of votes,
and . . . having reacquired his Filipino citizenship by repatriation on June 30, 1995 under the provisions
of Presidential Decree No. 725 . . . (is) qualified to hold the office of governor of Sorsogon"; thus:

"PREMISES CONSIDERED, the Commission (First Division), therefore RESOLVES to GRANT the Petition.

Consistent with the decisions of the Supreme Court, the proclamation of Raul R. Lee as Governor of
Sorsogon is hereby ordered annulled, being contrary to law, he not having garnered the highest number
of votes to warrant his proclamation.

Upon the finality of the annulment of the proclamation of Raul R. Lee, the Provincial Board of
Canvassers is directed to immediately reconvene and, on the basis of the completed canvass, proclaim
petitioner Juan G. Frivaldo as the duly elected Governor of Sorsogon having garnered the highest
number of votes, and he having reacquired his Filipino citizenship by repatriation on June 30, 1995
under the provisions of Presidential Decree No. 725 and, thus, qualified to hold the office of Governor of
Sorsogon.

Conformably with Section 260 of the Omnibus Election Code (B.P. Blg. 881), the Clerk of the Commission
is directed to notify His Excellency the President of the Philippines, and the Secretary of the Sangguniang
Panlalawigan of the Province of Sorsogon of this resolution immediately upon the due implementation
thereof."

On December 26, 1995, Lee filed a motion for reconsideration which was denied by the Comelec en
banc in its Resolution 14 promulgated on February 23, 1996. On February 26, 1996, the present petition
was filed. Acting on the prayer for a temporary restraining order, this Court issued on February 27, 1996
a Resolution which inter alia directed the parties "to maintain the status quo prevailing prior to the filing
of this petition."

The Issues in G.R. No. 123755

Petitioner Lee's "position on the matter at hand briefly be capsulized in the following propositions" 15 :

"First — The initiatory petition below was so far insufficient in form and substance to warrant the
exercise by the COMELEC of its jurisdiction with the result that, in effect, the COMELEC acted without
jurisdiction in taking cognizance of and deciding said petition;

Second — The judicially declared disqualification of respondent was a continuing condition and
rendered him ineligible to run for, to be elected to and to hold the Office of Governor;

Third — The alleged repatriation of respondent was neither valid nor is the effect thereof retroactive as
to cure his ineligibility and qualify him to hold the Office of Governor; and

397
Fourth — Correctly read and applied, the Labo Doctrine fully supports the validity of petitioner's
proclamation as duly elected Governor of Sorsogon."

G.R. No. 120295

This is a petition to annul three Resolutions of the respondent Comelec, the first two of which are also at
issue in G.R. No. 123755, as follows:

1. Resolution 16 of the Second Division, promulgated on May 1, 1995, disqualifying Frivaldo from
running for governor of Sorsogon in the May 8, 1995 elections "on the ground that he is not a citizen of
the Philippines";

2. Resolution 17 of the Comelec en banc, promulgated on May 11, 1995; and

3. Resolution 18 of the Comelec en banc, promulgated also on May 11, 1995 suspending the
proclamation of, among others, Frivaldo.

The Facts and the Issue

The facts of this case are essentially the same as those in G.R. No. 123755. However, Frivaldo assails the
above-mentioned resolutions on a different ground: that under Section 78 of the Omnibus Election
Code, which is reproduced hereinunder:

"Section 78. Petition to deny due course or to cancel a certificate of candidacy. — A verified petition
seeking to deny due course or to cancel a certificate of candidacy may be filed by any person exclusively
on the ground that any material representation contained therein as required under Section 74 hereof is
false. The petition may be filed at any time not later than twenty-five days from the time of the filing of
the certificate of candidacy and shall be decided, after notice and hearing, not later than fifteen days
before the election." (Emphasis supplied.)

the Comelec had no jurisdiction to issue said Resolutions because they were not rendered "within the
period allowed by law", i.e., "not later than fifteen days before the election."

Otherwise stated, Frivaldo contends that the failure of the Comelec to act on the petition for
disqualification within the period of fifteen days prior to the election as provided by law is a
jurisdictional defect which renders the said Resolutions null and void.

By Resolution on March 12, 1996, the Court consolidated G.R. Nos. 120295 and 123755 since they are
intimately related in their factual environment and are identical in the ultimate question raised, viz.,
who should occupy the position of governor of the province of Sorsogon.

On March 19, 1995, the Court heard oral argument from the parties and required them thereafter to file
simultaneously their respective memoranda.

The Consolidated Issues

From the foregoing submissions, the consolidated issues may be restated as follows:

398
1. Was the repatriation of Frivaldo valid and legal? If so, did it seasonably cure his lack of
citizenship as to qualify him to be proclaimed and to hold the Office of Governor? If not, may it be given
retroactive effect? If so, from when?

2. Is Frivaldo's "judicially declared" disqualification for lack of Filipino citizenship a continuing bar
to his eligibility to run for, be elected to or hold the governorship of Sorsogon?

3. Did the respondent Comelec have jurisdiction over the initiatory petition in SPC No. 95-317
considering that said petition is not "a pre-proclamation case, an election protest or a quo warranto
case"?

4. Was the proclamation of Lee, a runner-up in the election, valid and legal in light of existing
jurisprudence?

5. Did the respondent Commission on Elections exceed its jurisdiction in promulgating the assailed
Resolutions, all of which prevented Frivaldo from assuming the governorship of Sorsogon, considering
that they were not rendered within the period referred to in Section 78 of the Omnibus Election Code,
viz., "not later than fifteen days before the elections"?

The First Issue: Frivaldo's Repatriation

The validity and effectivity of Frivaldo's repatriation is the lis mota, the threshold legal issue in this case.
All the other matters raised are secondary to this.

The Local Government Code of 1991 19 expressly requires Philippine citizenship as a qualification for
elective local officials, including that of provincial governor, thus:

"Sec. 39. Qualifications. — (a) An elective local official must be a citizen of the Philippines; a
registered voter in the barangay, municipality, city, or province or, in the case of a member of the
sangguniang panlalawigan, sangguniang panlungsod, or sangguniang bayan, the district where he
intends to be elected; a resident therein for at least one (1) year immediately preceding the day of the
election; and able to read and write Filipino or any other local language or dialect.

(b) Candidates for the position of governor, vice governor or member of the sangguniang
panlalawigan, or mayor, vice mayor or member of the sangguniang panlungsod of highly urbanized cities
must be at least twenty-three (23) years of age on election day.

xxx xxx xxx

Inasmuch as Frivaldo had been declared by this Court 20 as a non-citizen, it is therefore incumbent upon
him to show that he has reacquired citizenship; in fine, that he possesses the qualifications prescribed
under the said statute (R.A. 7160).

Under Philippine law, 21 citizenship may be reacquired by direct act of Congress, by naturalization or by
repatriation. Frivaldo told this Court in G.R. No. 104654 22 and during the oral argument in this case that
he tried to resume his citizenship by direct act of Congress, but that the bill allowing him to do so "failed

399
to materialize, notwithstanding the endorsement of several members of the House of Representatives"
due, according to him, to the "maneuvers of his political rivals." In the same case, his attempt at
naturalization was rejected by this Court because of jurisdictional, substantial and procedural defects.

Despite his lack of Philippine citizenship, Frivaldo was overwhelmingly elected governor by the
electorate of Sorsogon, with a margin of 27,000 votes in the 1988 elections, 57,000 in 1992, and 20,000
in 1995 over the same opponent Raul Lee. Twice, he was judicially declared a non-Filipino and thus twice
disqualified from holding and discharging his popular mandate. Now, he comes to us a third time, with a
fresh vote from the people of Sorsogon and a favorable decision from the Commission on Elections to
boot. Moreover, he now boasts of having successfully passed through the third and last mode of
reacquiring citizenship: by repatriation under P.D. No. 725, with no less than the Solicitor General
himself, who was the prime opposing counsel in the previous cases he lost, this time, as counsel for co-
respondent Comelec, arguing the validity of his cause (in addition to his able private counsel Sixto S.
Brillantes, Jr.). That he took his oath of allegiance under the provisions of said Decree at 2:00 p.m. on
June 30, 1995 is not disputed. Hence, he insists that he — not Lee — should have been proclaimed as
the duly-elected governor of Sorsogon when the Provincial Board of Canvassers met at 8:30 p.m. on the
said date since, clearly and unquestionably, he garnered the highest number of votes in the elections
and since at that time, he already reacquired his citizenship.

En contrario, Lee argues that Frivaldo's repatriation is tainted with serious defects, which we shall now
discuss in seriatim.

First, Lee tells us that P.D. No. 725 had "been effectively repealed", asserting that "then President
Corazon Aquino exercising legislative powers under the Transitory Provisions of the 1987 Constitution,
forbade the grant of citizenship by Presidential Decree or Executive Issuances as the same poses a
serious and contentious issue of policy which the present government, in the exercise of prudence and
sound discretion, should best leave to the judgment of the first Congress under the 1987 Constitution",
adding that in her memorandum dated March 27, 1987 to the members of the Special Committee on
Naturalization constituted for purposes of Presidential Decree No. 725, President Aquino directed them
"to cease and desist from undertaking any and all proceedings within your functional area of
responsibility as defined under Letter of Instructions (LOI) No. 270 dated April 11, 1975, as amended."
23

This memorandum dated March 27, 1987 24 cannot by any stretch of legal hermeneutics be construed
as a law sanctioning or authorizing a repeal of P.D. No. 725. Laws are repealed only by subsequent ones
25 and a repeal may be express or implied. It is obvious that no express repeal was made because then
President Aquino in her memorandum — based on the copy furnished us by Lee — did not categorically
and/or impliedly state that P.D. 725 was being repealed or was being rendered without any legal effect.
In fact, she did not even mention it specifically by its number or text. On the other hand, it is a basic rule
of statutory construction that repeals by implication are not favored. An implied repeal will not be
allowed "unless it is convincingly and unambiguously demonstrated that the two laws are clearly
repugnant and patently inconsistent that they cannot co-exist". 26

400
The memorandum of then President Aquino cannot even be regarded as a legislative enactment, for not
every pronouncement of the Chief Executive even under the Transitory Provisions of the 1987
Constitution can nor should be regarded as an exercise of her law-making powers. At best, it could be
treated as an executive policy addressed to the Special Committee to halt the acceptance and
processing of applications for repatriation pending whatever "judgment the first Congress under the
1987 Constitution" might make. In other words, the former President did not repeal P.D. 725 but left it
to the first Congress — once created — to deal with the matter. If she had intended to repeal such law,
she should have unequivocally said so instead of referring the matter to Congress. The fact is she
carefully couched her presidential issuance in terms that clearly indicated the intention of "the present
government, in the exercise of prudence and sound discretion" to leave the matter of repeal to the new
Congress. Any other interpretation of the said Presidential Memorandum, such as is now being
proffered to the Court by Lee, would visit unmitigated violence not only upon statutory construction but
on common sense as well.

Second. Lee also argues that "serious congenital irregularities flawed the repatriation proceedings,"
asserting that Frivaldo's application therefor was "filed on June 29, 1995 . . . (and) was approved in just
one day or on June 30, 1995 . . .", which "prevented a judicious review and evaluation of the merits
thereof." Frivaldo counters that he filed his application for repatriation with the Office of the President
in Malacañang Palace on August 17, 1994. This is confirmed by the Solicitor General. However, the
Special Committee was reactivated only on June 8, 1995, when presumably the said Committee started
processing his application. On June 29, 1995, he filled up and re-submitted the FORM that the
Committee required. Under these circumstances, it could not be said that there was "indecent haste" in
the processing of his application.

Anent Lee's charge that the "sudden reconstitution of the Special Committee on Naturalization was
intended solely for the personal interest of respondent," 27 the Solicitor General explained during the
oral argument on March 19, 1996 that such allegation is simply baseless as there were many others who
applied and were considered for repatriation, a list of whom was submitted by him to this Court,
through a Manifestation 28 filed on April 3, 1996.

On the basis of the parties' submissions, we are convinced that the presumption of regularity in the
performance of official duty and the presumption of legality in the repatriation of Frivaldo have not
been successfully rebutted by Lee. The mere fact that the proceedings were speeded up is by itself not a
ground to conclude that such proceedings were necessarily tainted. After all, the requirements of
repatriation under P.D. No. 725 are not difficult to comply with, nor are they tedious and cumbersome.
In fact, P.D. 725 29 itself requires very little of an applicant, and even the rules and regulations to
implement the said decree were left to the Special Committee to promulgate. This is not unusual since,
unlike in naturalization where an alien covets a first-time entry into Philippine political life, in
repatriation the applicant is a former natural-born Filipino who is merely seeking to reacquire his
previous citizenship. In the case of Frivaldo, he was undoubtedly a natural-born citizen who openly and
faithfully served his country and his province prior to his naturalization in the United States — a
naturalization he insists was made necessary only to escape the iron clutches of a dictatorship he
abhorred and could not in conscience embrace — and who, after the fall of the dictator and the re-

401
establishment of democratic space, wasted no time in returning to his country of birth to offer once
more his talent and services to his people.

So too, the fact that ten other persons, as certified to by the Solicitor General, were granted repatriation
argues convincingly and conclusively against the existence of favoritism vehemently posited by Raul Lee.
At any rate, any contest on the legality of Frivaldo's repatriation should have been pursued before the
Committee itself, and, failing there, in the Office of the President, pursuant to the doctrine of exhaustion
of administrative remedies.

Third. Lee further contends that assuming the assailed repatriation to be valid, nevertheless it could only
be effective as at 2:00 p.m. of June 30, 1995 whereas the citizenship qualification prescribed by the
Local Government Code "must exist on the date of his election, if not when the certificate of candidacy
is filed," citing our decision in G.R. 104654 30 which held that "both the Local Government Code and the
Constitution require that only Philippine citizens can run and be elected to Public office." Obviously,
however, this was a mere obiter as the only issue in said case was whether Frivaldo's naturalization was
valid or not — and NOT the effective date thereof. Since the Court held his naturalization to be invalid,
then the issue of when an aspirant for public office should be a citizen was NOT resolved at all by the
Court. Which question we shall now directly rule on.

Under Sec. 39 of the Local Government Code, "(a)n elective local official must be:

* a citizen of the Philippines;

* a registered voter in the barangay, municipality, city, or province . . . where he intends to be


elected;

* a resident therein for at least one (1) year immediately preceding the day of the election;

* able to read and write Filipino or any other local language or dialect."

* In addition, "candidates for the position of governor . . . must be at least twenty-three (23) years
of age on election day."

From the above, it will be noted that the law does not specify any particular date or time when the
candidate must possess citizenship, unlike that for residence (which must consist of at least one year's
residency immediately preceding the day of election) and age (at least twenty three years of age on
election day).

Philippine citizenship is an indispensable requirement for holding an elective public office, 31 and the
purpose of the citizenship qualification is none other than to ensure that no alien, i.e., no person owing
allegiance to another nation, shall govern our people and our country or a unit of territory thereof. Now,
an official begins to govern or to discharge his functions only upon his proclamation and on the day the
law mandates his term of office to begin. Since Frivaldo re-assumed his citizenship on June 30, 1995 —
the very day 32 the term of office of governor (and other elective officials) began — he was therefore
already qualified to be proclaimed, to hold such office and to discharge the functions and

402
responsibilities thereof as of said date. In short, at that time, he was already qualified to govern his
native Sorsogon. This is the liberal interpretation that should give spirit, life and meaning to our law on
qualifications consistent with the purpose for which such law was enacted. So too, even from a literal
(as distinguished from liberal) construction, it should be noted that Section 39 of the Local Government
Code speaks of "Qualifications" of "ELECTIVE OFFICIALS", not of candidates. Why then should such
qualification be required at the time of election or at the time of the filing of the certificates of
candidacies, as Lee insists? Literally, such qualifications — unless otherwise expressly conditioned, as in
the case of age and residence — should thus be possessed when the "elective [or elected] official"
begins to govern, i.e., at the time he is proclaimed and at the start of his term — in this case, on June 30,
1995. Paraphrasing this Court's ruling in Vasquez vs. Giap and Li Seng Giap & Sons, 33 if the purpose of
the citizenship requirement is to ensure that our people and country do not end up being governed by
aliens, i.e., persons owing allegiance to another nation, that aim or purpose would not be thwarted but
instead achieved by construing the citizenship qualification as applying to the time of proclamation of
the elected official and at the start of his term.

But perhaps the more difficult objection was the one raised during the oral argument 34 to the effect
that the citizenship qualification should be possessed at the time the candidate (or for that matter the
elected official) registered as a voter. After all, Section 39, apart from requiring the official to be a
citizen, also specifies as another item of qualification, that he be a "registered voter". And, under the
law 35 a "voter" must be a citizen of the Philippines. So therefore, Frivaldo could not have been a voter
— much less a validly registered one — if he was not a citizen at the time of such registration.

The answer to this problem again lies in discerning the purpose of the requirement. If the law intended
the citizenship qualification to be possessed prior to election consistent with the requirement of being a
registered voter, then it would not have made citizenship a SEPARATE qualification. The law abhors a
redundancy. It therefore stands to reason that the law intended CITIZENSHIP to be a qualification
distinct from being a VOTER, even if being a voter presumes being a citizen first. It also stands to reason
that the voter requirement was included as another qualification (aside from "citizenship"), not to
reiterate the need for nationality but to require that the official be registered as a voter IN THE AREA OR
TERRITORY he seeks to govern, i.e., the law states: "a registered voter in the barangay, municipality, city,
or province . . . where he intends to be elected." It should be emphasized that the Local Government
Code requires an elective official to be a registered voter. It does not require him to vote actually.
Hence, registration — not the actual voting — is the core of this "qualification". In other words, the law's
purpose in this second requirement is to ensure that the prospective official is actually registered in the
area he seeks to govern — and not anywhere else.

Before this Court, Frivaldo has repeatedly emphasized — and Lee has not disputed — that he "was and
is a registered voter of Sorsogon, and his registration as a voter has been sustained as valid by judicial
declaration . . . In fact, he cast his vote in his precinct on May 8, 1995." 36

So too, during the oral argument, his counsel stead- fastly maintained that "Mr. Frivaldo has always
been a registered voter of Sorsogon. He has voted in 1987, 1988, 1992, then he voted again in 1995. In

403
fact, his eligibility as a voter was questioned, but the court dismissed (sic) his eligibility as a voter and he
was allowed to vote as in fact, he voted in all the previous elections including on May 8, 1995. 37

It is thus clear that Frivaldo is a registered voter in the province where he intended to be elected.

There is yet another reason why the prime issue of citizenship should be reckoned from the date of
proclamation, not necessarily the date of election or date of filing of the certificate of candidacy. Section
253 of the Omnibus Election Code 38 gives any voter, presumably including the defeated candidate, the
opportunity to question the ELIGIBILITY (or the disloyalty) of a candidate. This is the only provision of the
Code that authorizes a remedy on how to contest before the Comelec an incumbent's ineligibility arising
from failure to meet the qualifications enumerated under Sec. 39 of the Local Government Code. Such
remedy of Quo Warranto can be availed of "within ten days after proclamation" of the winning
candidate. Hence, it is only at such time that the issue of ineligibility may be taken cognizance of by the
Commission. And since, at the very moment of Lee's proclamation (8:30 p.m., June 30, 1995), Juan G.
Frivaldo was already and indubitably a citizen, having taken his oath of allegiance earlier in the
afternoon of the same day, then he should have been the candidate proclaimed as he unquestionably
garnered the highest number of votes in the immediately preceding elections and such oath had already
cured his previous "judicially-declared" alienage. Hence, at such time, he was no longer ineligible.

But to remove all doubts on this important issue, we also hold that the repatriation of Frivaldo
RETROACTED to the date of the filing of his application on August 17, 1994.

It is true that under the Civil Code of the Philippines, 39 "(l)aws shall have no retroactive effect, unless
the contrary is provided." But there are settled exceptions 40 to this general rule, such as when the
statute is CURATIVE or REMEDIAL in nature or when it CREATES NEW RIGHTS.

According to Tolentino, 41 curative statutes are those which undertake to cure errors and irregularities,
thereby validating judicial or administrative proceedings, acts of public officers, or private deeds and
contracts which otherwise would not produce their intended consequences by reason of some statutory
disability or failure to comply with some technical requirement. They operate on conditions already
existing, and are necessarily retroactive in operation. Agpalo, 42 on the other hand, says that curative
statutes are "healing acts . . . curing defects and adding to the means of enforcing existing obligations . .
. (and ) are intended to supply defects, abridge superfluities in existing laws, and curb certain evils . . . By
their very nature, curative statutes are retroactive . . . (and) reach back to past events to correct errors
or irregularities and to render valid and effective attempted acts which would be otherwise ineffective
for the purpose the parties intended."

On the other hand, remedial or procedural laws, i.e., those statutes relating to remedies or modes of
procedure, which do not create new or take away vested rights, but only operate in furtherance of the
remedy or confirmation of such rights, ordinarily do not come within the legal meaning of a
retrospective law, nor within the general rule against the retrospective operation of statutes. 43

A reading of P.D. 725 immediately shows that it creates a new right, and also provides for a new
remedy, thereby filling certain voids in our laws. Thus, in its preamble, P.D. 725 expressly recognizes the

404
plight of "many Filipino women (who) had lost their Philippine citizenship by marriage to aliens" and
who could not, under the existing law (C.A. No. 63, as amended) avail of repatriation until "after the
death of their husbands or the termination of their marital status" and who could neither be benefited
by the 1973 Constitution's new provision allowing "a Filipino woman who marries an alien to retain her
Philippine citizenship . . ." because "such provision of the new Constitution does not apply to Filipino
women who had married aliens before said constitution took effect." Thus, P.D. 725 granted a new right
to these women — the right to re-acquire Filipino citizenship even during their marital coverture, which
right did not exist prior to P.D. 725. On the other hand, said statute also provided a new remedy and a
new right in favor of other "natural born Filipinos who (had) lost their Philippine citizenship but now
desire to re-acquire Philippine citizenship", because prior to the promulgation of P.D. 725 such former
Filipinos would have had to undergo the tedious and cumbersome process of naturalization, but with
the advent of P.D. 725 they could now re-acquire their Philippine citizenship under the simplified
procedure of repatriation.

The Solicitor General 44 argues:

"By their very nature, curative statutes are retroactive, (DBP vs. CA, 96 SCRA 342), since they are
intended to supply defects, abridge superfluities in existing laws (Del Castillo vs. Securities and Exchange
Commission, 96 Phil. 119) and curb certain evils (Santos vs. Duata, 14 SCRA 1041).

In this case, P.D. No. 725 was enacted to cure the defect in the existing naturalization law, specifically
C.A. No. 63 wherein married Filipino women are allowed to repatriate only upon the death of their
husbands, and natural-born Filipinos who lost their citizenship by naturalization and other causes faced
the difficulty of undergoing the rigid procedures of C.A. 63 for reacquisition of Filipino citizenship by
naturalization.

Presidential Decree No. 725 provided a remedy for the aforementioned legal aberrations and thus its
provisions are considered essentially remedial and curative."

In light of the foregoing, and prescinding from the wording of the preamble, it is unarguable that the
legislative intent was precisely to give the statute retroactive operation. "(A) retrospective operation is
given to a statute or amendment where the intent that it should so operate clearly appears from a
consideration of the act as a whole, or from the terms thereof." 45 It is obvious to the Court that the
statute was meant to "reach back" to those persons, events and transactions not otherwise covered by
prevailing law and jurisprudence. And inasmuch as it has been held that citizenship is a political and civil
right equally as important as the freedom of speech, liberty of abode, the right against unreasonable
searches and seizures and other guarantees enshrined in the Bill of Rights, therefore the legislative
intent to give retrospective operation to P.D. 725 must be given the fullest effect possible. "(I)t has been
said that a remedial statute must be so construed as to make it effect the evident purpose for which it
was enacted, so that if the reason of the statute extends to past transactions, as well as to those in the
future, then it will be so applied although the statute does not in terms so direct, unless to do so would
impair some vested right or violate some constitutional guaranty." 46 This is all the more true of P.D.

405
725, which did not specify any restrictions on or delimit or qualify the right of repatriation granted
therein.

At this point, a valid question may be raised: How can the retroactivity of P.D. 725 benefit Frivaldo
considering that said law was enacted on June 5, 1975, while Frivaldo lost his Filipino citizenship much
later, on January 20, 1983, and applied for repatriation even later, on August 17, 1994?

While it is true that the law was already in effect at the time that Frivaldo became an American citizen,
nevertheless, it is not only the law itself (P.D. 725) which is to be given retroactive effect, but even the
repatriation granted under said law to Frivaldo on June 30, 1995 is to be deemed to have retroacted to
the date of his application therefor, August 17, 1994. The reason for this is simply that if, as in this case,
it was the intent of the legislative authority that the law should apply to past events — i.e., situations
and transactions existing even before the law came into being — in order to benefit the greatest
number of former Filipinos possible thereby enabling them to enjoy and exercise the constitutionally
guaranteed right of citizenship, and such legislative intention is to be given the fullest effect and
expression, then there is all the more reason to have the law apply in a retroactive or retrospective
manner to situations, events and transactions subsequent to the passage of such law. That is, the
repatriation granted to Frivaldo on June 30, 1995 can and should be made to take effect as of date of his
application. As earlier mentioned, there is nothing in the law that would bar this or would show a
contrary intention on the part of the legislative authority; and there is no showing that damage or
prejudice to anyone, or anything unjust or injurious would result from giving; retroactivity to his
repatriation. Neither has Lee shown that there will result the impairment of any contractual obligation,
disturbance of any vested right or breach of some constitutional guaranty.

Being a former Filipino who has served the people repeatedly, Frivaldo deserves a liberal interpretation
of Philippine laws and whatever defects there were in his nationality should now be deemed mooted by
his repatriation.

Another argument for retroactivity to the date of filing is that it would prevent prejudice to applicants. If
P.D. 725 were not to be given retroactive effect, and the Special Committee decides not to act, i.e., to
delay the processing of applications for any substantial length of time, then the former Filipinos who
may be stateless, as Frivaldo — having already renounced his American citizenship — was, may be
prejudiced for causes outside their control. This should not be. In case of doubt in the interpretation or
application of laws, it is to be presumed that the law-making body intended right and justice to prevail.
47

And as experience will show, the Special Committee was able to process, act upon and grant
applications for repatriation within relatively short spans of time after the same were filed. 48 The fact
that such interregna were relatively insignificant minimizes the likelihood of prejudice to the
government as a result of giving retroactivity to repatriation. Besides, to the mind of the Court, direct
prejudice to the government is possible only where a person's repatriation has the effect of wiping out a
liability of his to the government arising in connection with or as a result of his being an alien, and

406
accruing only during the interregnum between application and approval, a situation that is not present
in the instant case.

And it is but right and just that the mandate of the people, already twice frustrated, should now prevail.
Under the circumstances, there is nothing unjust or iniquitous in treating Frivaldo's repatriation as
having become effective as of the date of his application, i.e., on August 17, 1994. This being so, all
questions about his possession of the nationality qualification — whether at the date of proclamation
(June 30, 1995) or the date of election (May 8, 1995) or date of filing his certificate of candidacy (March
20, 1995) would become moot.

Based on the foregoing, any question regarding Frivaldo's status as a registered voter would also be
deemed settled. Inasmuch as he is considered as having been repatriated — i.e., his Filipino citizenship
restored — as of August 17, 1994, his previous registration as a voter is likewise deemed validated as of
said date.

It is not disputed that on January 20, 1983 Frivaldo became an American. Would the retroactivity of his
repatriation not effectively give him dual citizenship, which under Sec. 40 of the Local Government Code
would disqualify him "from running for any elective local position?" 49 We answer this question in the
negative, as there is cogent reason to hold that Frivaldo was really STATELESS at the time he took said
oath of allegiance and even before that, when he ran for governor in 1988. In his Comment, Frivaldo
wrote that he "had long renounced and had long abandoned his American citizenship — long before
May 8, 1995. At best, Frivaldo was stateless in the interim — when he abandoned and renounced his US
citizenship but before he was repatriated to his Filipino citizenship." 50

On this point, we quote from the assailed Resolution dated December 19, 1995: 51

"By the laws of the United States, petitioner Frivaldo lost his American citizenship when he took his oath
of allegiance to the Philippine Government when he ran for Governor in 1988, in 1992, and in 1995.
Every certificate of candidacy contains an oath of allegiance to the Philippine Government."

These factual findings that Frivaldo has lost his foreign nationality long before the elections of 1995 have
not been effectively rebutted by Lee. Furthermore, it is basic that such findings of the Commission are
conclusive upon this Court, absent any showing of capriciousness or arbitrariness or abuse. 52

The Second Issue: Is Lack of Citizenship a Continuing Disqualification?

Lee contends that the May 1, 1995 Resolution 53 of the Comelec Second Division in SPA No. 95-028 as
affirmed in toto by Comelec En Banc in its Resolution of May 11, 1995 "became final and executory after
five (5) days or on May 17, 1995, no restraining order having been issued by this Honorable Court." 54
Hence, before Lee "was proclaimed as the elected governor on June 30, 1995, there was already a final
and executory judgment disqualifying" Frivaldo. Lee adds that this Court's two rulings (which Frivaldo
now concedes were legally "correct") declaring Frivaldo an alien have also become final and executory
way before the 1995 elections, and these "judicial pronouncements of his political status as an American

407
citizen absolutely and for all time disqualified (him) from running for, and holding any public office in the
Philippines."

We do not agree.

It should be noted that our first ruling in G.R. No. 87193 disqualifying Frivaldo was rendered in
connection with the 1988 elections while that in G.R. No. 104654 was in connection with the 1992
elections. That he was disqualified for such elections is final and can no longer be changed. In the words
of the respondent Commission (Second Division) in its assailed Resolution: 55

"The records show that the Honorable Supreme Court had decided that Frivaldo was not a Filipino
citizen and thus disqualified for the purpose of the 1988 and 1992 elections. However, there is no record
of any 'final judgment' of the disqualification of Frivaldo as a candidate for the May 8, 1995 elections.
What the Commission said in its Order of June 21, 1995 (implemented on June 30, 1995), directing the
proclamation of Raul R. Lee, was that Frivaldo was not a Filipino citizen 'having been declared by the
Supreme Court in its Order dated March 25, 1995, not a citizen of the Philippines.' This declaration of
the Supreme Court, however, was in connection with the 1992 elections."

Indeed, decisions declaring the acquisition or denial of citizenship cannot govern a person's future status
with finality. This is because a person may subsequently reacquire, or for that matter lose, his citizenship
under any of the modes recognized by law for the purpose. Hence, in Lee vs. Commissioner of
Immigration, 56 we held:

"Everytime the citizenship of a person is material or indispensable in a judicial or administrative case,


whatever the corresponding court or administrative authority decides therein as to such citizenship is
generally not considered res judicata, hence it has to be threshed out again and again, as the occasion
demands."

The Third Issue: Comelec's Jurisdiction Over The Petition in SPC No. 95-317

Lee also avers that respondent Comelec had no jurisdiction to entertain the petition in SPC No. 95-317
because the only "possible types of proceedings that may be entertained by the Comelec are a pre-
proclamation case, an election protest or a quo warranto case". Again, Lee reminds us that he was
proclaimed on June 30, 1995 but that Frivaldo filed SPC No. 95-317 questioning his (Lee's) proclamation
only on July 6, 1995 — "beyond the 5-day reglementary period." Hence, according to him, Frivaldo's
"recourse was to file either an election protest or a quo warranto action."

This argument is not meritorious. The Constitution 57 has given the Comelec ample power to "exercise
exclusive original jurisdiction over all contests relating to the elections, returns and qualifications of all
elective. . . provincial . . . officials." Instead of dwelling at length on the various petitions that Comelec, in
the exercise of its constitutional prerogatives, may entertain, suffice it to say that this Court has
invariably recognized the Commission's authority to hear and decide petitions for annulment of
proclamations — of which SPC No. 95-317 obviously is one. 58 Thus, in Mentang vs. COMELEC, 59 we
ruled:

408
"The petitioner argues that after proclamation and assumption of office, a pre-proclamation controversy
is no longer viable. Indeed, we are aware of cases holding that pre-proclamation controversies may no
longer be entertained by the COMELEC after the winning candidate has been proclaimed. (citing
Gallardo vs. Rimando, 187 SCRA 463; Salvacion vs. COMELEC, 170 SCRA 513; Casimiro vs. COMELEC, 171
SCRA 468.) This rule, however, is premised on an assumption that the proclamation is no proclamation
at all and the proclaimed candidate's assumption of office cannot deprive the COMELEC of the power to
make such declaration of nullity. (citing Aguam vs. COMELEC, 23 SCRA 883; Agbayani vs. COMELEC, 186
SCRA 484.)"

The Court however cautioned that such power to annul a proclamation must "be done within ten (10)
days following the proclamation." Inasmuch as Frivaldo's petition was filed only six (6) days after Lee's
proclamation, there is no question that the Comelec correctly acquired jurisdiction over the same.

The Fourth Issue: Was Lee's Proclamation Valid?

Frivaldo assails the validity of the Lee proclamation. We uphold him for the following reasons:

First. To paraphrase this Court in Labo vs. COMELEC, 60 "the fact remains that he (Lee) was not the
choice of the sovereign will," and in Aquino vs. COMELEC, 61 Lee is "a second placer, . . . just that, a
second placer."

In spite of this, Lee anchors his claim to the governorship on the pronouncement of this Court in the
aforesaid Labo 62 case, as follows:

"The rule would have been different if the electorate fully aware in fact and in law of a candidate's
disqualification so as to bring such awareness within the realm of notoriety, would nonetheless cast
their votes in favor of the ineligible candidate. In such case, the electorate may be said to have waived
the validity and efficacy of their votes by notoriously misapplying their franchise or throwing away their
votes, in which case, the eligible candidate obtaining the next higher number of votes may be deemed
elected."

But such holding is qualified by the next paragraph, thus:

"But this is not the situation obtaining in the instant dispute. It has not been shown, and none was
alleged, that petitioner Labo was notoriously known as an ineligible candidate, much less the electorate
as having known of such fact. On the contrary, petitioner Labo was even allowed by no less than the
Comelec itself in its resolution dated May 10, 1992 to be voted for the office of the city mayor as its
resolution dated May 9, 1992 denying due course to petitioner Labo's certificate of candidacy had not
yet become final and subject to the final outcome of this case."

The last-quoted paragraph in Labo, unfortunately for Lee, is the ruling appropriate in this case because
Frivaldo was in 1995 in an identical situation as Labo was in 1992 when the Comelec's cancellation of his
certificate of candidacy was not yet final on election day as there was in both cases a pending motion for
reconsideration, for which reason Comelec issued an (omnibus) resolution declaring that Frivaldo (like
Labo in 1992) and several others can still be voted for in the May 8, 1995 election, as in fact, he was.

409
Furthermore, there has been no sufficient evidence presented to show that the electorate of Sorsogon
was "fully aware in fact and in law" of Frivaldo's alleged disqualification as to "bring such awareness
within the realm of notoriety;" in other words, that the voters intentionally wasted their ballots knowing
that, in spite of their voting for him, he was ineligible. If Labo has any relevance at all, it is that the vice-
governor — and not Lee — should be proclaimed, since in losing the election, Lee was, to paraphrase
Labo again, "obviously not the choice of the people" of Sorsogon. This is the emphatic teaching of Labo:

"The rule, therefore, is: the ineligibility of a candidate receiving majority votes does not entitle the
eligible candidate receiving the next highest number of votes to be declared elected. A minority or
defeated candidate cannot be deemed elected to the office."

Second. As we have earlier declared Frivaldo to have seasonably re-acquired his citizenship and
inasmuch as he obtained the highest number of votes in the 1995 elections, he — not Lee — should be
proclaimed. Hence, Lee's proclamation was patently erroneous and should now be corrected.

The Fifth Issue: Is Section 78 of the Election Code Mandatory?

In G.R. No. 120295. Frivaldo claims that the assailed Resolution of the Comelec (Second Division) dated
May 1, 1995 and the confirmatory en banc Resolution of May 11, 1995 disqualifying him for want of
citizenship should be annulled because they were rendered beyond the fifteen (15) day period
prescribed by Section 78 of the Omnibus Election Code which reads as follows:

"Section 78. Petition to deny due course or to cancel a certificate of candidacy. — A verified petition
seeking to deny due course or to cancel a certificate of candidacy may be filed by any person exclusively
on the ground that any material representation contained therein as required under Section 74 hereof is
false. The petition may be filed at any time not later than twenty-five days from the time of the filing of
the certificate of candidacy and shall be decided after notice and hearing, not later than fifteen days
before the election." (emphasis supplied.)

This claim is now moot and academic inasmuch as these resolutions are deemed superseded by the
subsequent ones issued by the Commission (First Division) on December 19, 1995, affirmed en banc 63
on February 23, 1996, which both upheld his election. At any rate, it is obvious that Section 78 is merely
directory as Section 6 of R.A. No. 6646 authorizes the Commission to try and decide petitions for
disqualifications even after the elections, thus:

"SEC. 6. Effect of Disqualification Case. — Any candidate who has been declared by final judgment to be
disqualified shall not be voted for, and the votes cast for him shall not be counted. If for any reason a
candidate is not declared by final judgment before an election to be disqualified and he is voted for and
receives the winning number of votes in such election, the Court or Commission shall continue with the
trial and hearing of the action, inquiry or protest and, upon motion of the complainant or any
intervenor, may during the pendency thereof order the suspension of the proclamation of such
candidate whenever the evidence of his guilt is strong." (emphasis supplied)

Refutation of

410
Mr. Justice Davide's Dissent

In his dissenting opinion, the esteemed Mr. Justice Hilario G. Davide, Jr. argues that President Aquino's
memorandum dated March 27, 1987 should be viewed as a suspension (not a repeal, as urged by Lee) of
P.D. 725. But whether it decrees a suspension or a repeal is a purely academic distinction because the
said issuance is not a statute that can amend or abrogate an existing law. The existence and subsistence
of P.D. 725 were recognized in the first Frivaldo case; 64 viz, "(u)nder CA No. 63 as amended by CA No.
473 and P.D. No. 725, Philippine citizenship may be reacquired by . . . repatriation". He also contends
that by allowing Frivaldo to register and to remain as a registered voter, the Comelec and in effect this
Court abetted a "mockery" of our two previous judgments declaring him a non-citizen. We do not see
such abetting or mockery. The retroactivity of his repatriation, as discussed earlier, legally cured
whatever defects there may have been in his registration as a voter for the purpose of the 1995
elections. Such retroactivity did not change his disqualifications in 1988 and 1992, which were the
subjects of such previous rulings.

Mr. Justice Davide also believes that Quo Warranto is not the sole remedy to question the ineligibility of
a candidate, citing the Comelec's authority under Section 78 of the Omnibus Election Code allowing the
denial of a certificate of candidacy on the ground of a false material representation therein as required
by Section 74. Citing Loong, he then states his disagreement with our holding that Section 78 is merely
directory. We really have no quarrel. Our point is that Frivaldo was in error in his claim in G.R. No.
120295 that the Comelec Resolutions promulgated on May 1, 1995 and May 11, 1995 were invalid
because they were issued "not later than fifteen days before the election" as prescribed by Section 78.
In dismissing the petition in G.R. No. 120295, we hold that the Comelec did not commit grave abuse of
discretion because "Section 6 of R.A. 6646 authorizes the Comelec to try and decide disqualifications
even after the elections." In spite of his disagreement with us on this point, i.e., that Section 78 "is
merely directory", we note that just like us, Mr. Justice Davide nonetheless votes to "DISMISS G.R. No.
120295". One other point. Loong, as quoted in the dissent, teaches that a petition to deny due course
under Section 78 must be filed within the 25-day period prescribed therein. The present case however
deals with the period during which the Comelec may decide such petition. And we hold that it may be
decided even after the fifteen day period mentioned in Section 78. Here, we rule that a decision
promulgated by the Comelec even after the elections is valid but Loong held that a petition filed beyond
the 25-day period is out of time. There is no inconsistency nor conflict.

Mr. Justice Davide also disagrees with the Court's holding that, given the unique factual circumstances
of Frivaldo, repatriation may be given retroactive effect. He argues that such retroactivity "dilutes" our
holding in the first Frivaldo case. But the first (and even the second Frivaldo) decision did not directly
involve repatriation as a mode of acquiring citizenship. If we may repeat, there is no question that
Frivaldo was not a Filipino for purposes of determining his qualifications in the 1988 and 1992 elections.
That is settled. But his supervening repatriation has changed his political status — not in 1988 or 1992,
but only in the 1995 elections.

Our learned colleague also disputes our holding that Frivaldo was stateless prior to his repatriation,
saying that "informal renunciation or abandonment is not a ground to lose American citizenship." Since

411
our courts are charged only with the duty of the determining who are Philippine nationals, we cannot
rule on the legal question of who are or who are not Americans. It is basic in international law that a
State determines ONLY those who are its own citizens — not who are the citizens of other countries. 65
The issue here is: the Comelec made a finding of fact that Frivaldo was stateless and such finding has not
been shown by Lee to be arbitrary or whimsical. Thus, following settled case law, such finding is binding
and final.

The dissenting opinion also submits that Lee who lost by chasmic margins to Frivaldo in all three
previous elections, should be declared winner because "Frivaldo's ineligibility for being an American was
publicly known." First, there is absolutely no empirical evidence for such "public" knowledge. Second,
even if there is, such knowledge can be true post facto only of the last two previous elections. Third,
even the Comelec and now this Court were/are still deliberating on his nationality before, during and
after the 1995 elections. How then can there be such "public" knowledge?

Mr. Justice Davide submits that Section 39 of the Local Government Code refers to the qualifications of
elective local officials, i.e., candidates, and not elected officials, and that the citizenship qualification
[under par. (a) of that section] must be possessed by candidates, not merely at the commencement of
the term, but by election day at the latest. We see it differently. Section 39, par. (a) thereof speaks of
"elective local official" while par. (b) to (f) refer to "candidates". If the qualifications under par. (a) were
intended to apply to "candidates" and not elected officials, the legislature would have said so, instead of
differentiating par. (a) from the rest of the paragraphs. Secondly, if Congress had meant that the
citizenship qualification should be possessed at election day or prior thereto, it would have specifically
stated such detail, the same way it did in pars. (b) to (f) for other qualifications of candidates for
governor, mayor, etc.

Mr. Justice Davide also questions the giving of retroactive effect to Frivaldo's repatriation on the ground,
among others, that the law specifically provides that it is only after taking the oath of allegiance that
applicants shall be deemed to have reacquired Philippine citizenship. We do not question what the
provision states. We hold however that the provision should be understood thus: that after taking the
oath of allegiance the applicant is deemed to have reacquired Philippine citizenship, which reacquisition
(or repatriation) is deemed for all purposes and intents to have retroacted to the date of his application
therefor.

In any event, our "so too" argument regarding the literal meaning of the word "elective" in reference to
Section 39 of the Local Government Code, as well as regarding Mr. Justice Davide's thesis that the very
wordings of P.D. 725 suggest non-retroactivity, were already taken up rather extensively earlier in this
Decision.

Mr. Justice Davide caps his paper with a clarion call: "This Court must be the first to uphold the Rule of
Law." We agree — we must all follow the rule of law. But that is NOT the issue here. The issue is how
should the law be interpreted and applied in this case so it can be followed, so it can rule!

At balance, the question really boils down to a choice of philosophy and perception of how to interpret
and apply laws relating to elections: literal or liberal; the letter or the spirit; the naked provision or its

412
ultimate purpose; legal syllogism or substantial justice; in isolation or in the context of social conditions;
harshly against or gently in favor of the voters' obvious choice. In applying election laws, it would be far
better to err in favor of popular sovereignty than to be right in complex but little understood legalisms.
Indeed, to inflict a thrice rejected candidate upon the electorate of Sorsogon would constitute
unmitigated judicial tyranny and an unacceptable assault upon this Court's conscience.

EPILOGUE

In sum, we rule that the citizenship requirement in the Local Government Code is to be possessed by an
elective official at the latest as of the time he is proclaimed and at the start of the term of office to
which he has been elected. We further hold P.D. No. 725 to be in full force and effect up to the present,
not having been suspended or repealed expressly nor impliedly at any time, and Frivaldo's repatriation
by virtue thereof to have been properly granted and thus valid and effective. Moreover, by reason of the
remedial or curative nature of the law granting him a new right to resume his political status and the
legislative intent behind it, as well as his unique situation of having been forced to give up his citizenship
and political aspiration as his means of escaping a regime he abhorred, his repatriation is to be given
retroactive effect as of the date of his application therefor, during the pendency of which he was
stateless, he having given up his U.S. nationality. Thus, in contemplation of law, he possessed the vital
requirement of Filipino citizenship as of the start of the term of office of governor, and should have
been proclaimed instead of Lee. Furthermore, since his reacquisition of citizenship retroacted to August
17, 1994, his registration as a voter of Sorsogon is deemed to have been validated as of said date as
well. The foregoing, of course, are precisely consistent with our holding that lack of the citizenship
requirement is not a continuing disability or disqualification to run for and hold public office. And once
again, we emphasize herein our previous rulings recognizing the Comelec's authority and jurisdiction to
hear and decide petitions for annulment of proclamations.

This Court has time and again liberally and equitably construed the electoral laws of our country to give
fullest effect to the manifest will of our people, 66 for in case of doubt, political laws must be
interpreted to give life and spirit to the popular mandate freely expressed through the ballot. Otherwise
stated, legal niceties and technicalities cannot stand in the way of the sovereign will. Consistently, we
have held:

". . . (L)aws governing election contests must be liberally construed to the end that the will of the people
in the choice of public officials may not be defeated by mere technical objections (citations omitted)."
67

The law and the courts must accord Frivaldo every possible protection, defense and refuge, in deference
to the popular will. Indeed, this Court has repeatedly stressed the importance of giving effect to the
sovereign will in order to ensure the survival of our democracy. In any action involving the possibility of
a reversal of the popular electoral choice, this Court must exert utmost effort to resolve the issues in a
manner that would give effect to the will of the majority, for it is merely sound public policy to cause
elective offices to be filled by those who are the choice of the majority. To successfully challenge a
winning candidate's qualifications, the petitioner must clearly demonstrate that the ineligibility is so

413
patently antagonistic 68 to constitutional and legal principles that overriding such ineligibility and
thereby giving effect to the apparent will of the people, would ultimately create greater prejudice to the
very democratic institutions and juristic traditions that our Constitution and laws so zealously protect
and promote. In this undertaking, Lee has miserably failed.

In Frivaldo's case, it would have been technically easy to find fault with his cause. The Court could have
refused to grant retroactivity to the effects of his repatriation and hold him still ineligible due to his
failure to show his citizenship at the time he registered as a voter before the 1995 elections. Or, it could
have disputed the factual findings of the Comelec that he was stateless at the time of repatriation and
thus hold his consequent dual citizenship as a disqualification "from running for any elective local
position." But the real essence of justice does not emanate from quibblings over patchwork legal
technicality. It proceeds from the spirit's gut consciousness of the dynamic role of law as a brick in the
ultimate development of the social edifice. Thus, the Court struggled against and eschewed the easy,
legalistic, technical and sometimes harsh anachronisms of the law in order to evoke substantial justice in
the larger social context consistent with Frivaldo's unique situation approximating venerability in
Philippine political life. Concededly, he sought American citizenship only to escape the clutches of the
dictatorship. At this stage, we cannot seriously entertain any doubt about his loyalty and dedication to
this country. At the first opportunity, he returned to this land, and sought to serve his people once
more. The people of Sorsogon overwhelmingly voted for him three times. He took an oath of allegiance
to this Republic every time he filed his certificate of candidacy and during his failed naturalization bid.
And let it not be overlooked, his demonstrated tenacity and sheer determination to re-assume his
nationality of birth despite several legal set-backs speak more loudly, in spirit, in fact and in truth than
any legal technicality, of his consuming intention and burning desire to re-embrace his native Philippines
even now at the ripe old age of 81 years. Such loyalty to and love of country as well as nobility of
purpose cannot be lost on this Court of justice and equity. Mortals of lesser mettle would have given up.
After all, Frivaldo was assured of a life of ease and plenty as a citizen of the most powerful country in the
world. But he opted, nay, single-mindedly insisted on returning to and serving once more his struggling
but beloved land of birth. He therefore deserves every liberal interpretation of the law which can be
applied in his favor. And in the final analysis, over and above Frivaldo himself, the indomitable people of
Sorsogon most certainly deserve to be governed by a leader of their overwhelming choice.

WHEREFORE, in consideration of the foregoing:

(1) The petition in G.R. No. 123755 is hereby DISMISSED. The assailed Resolutions of the
respondent Commission are AFFIRMED.

(2) The petition in G.R. No. 120295 is also DISMISSED for being moot and academic. In any event, it
has no merit.

No costs.

SO ORDERED.

Francisco, Hermosisima, Jr., Melo, Vitug, Kapunan and Torres, JJ., concur.

414
Padilla, Regalado, Romero and Bellosillo, JJ., pro hac vice.

Narvasa, C.J. and Mendoza, J., took no part.

Separate Opinions

PUNO, J., concurring:

I concur with the path-breaking ponencia of Mr. Justice Panganiban which is pro-people and pierces the
myopia of legalism. Upholding the sovereign will of the people which is the be-all and the end-all of
republicanism, it rests on a foundation that will endure time and its tempest.

The sovereignty of our people is the primary postulate of the 1987 Constitution. For this reason, it
appears as the first in our declaration of principles and state policies. Thus, section 1 of Article II of our
fundamental law proclaims that "[t]he Philippines is a democratic and republican State. Sovereignty
resides in the people and all government authority emanates from them." The same principle served as
the bedrock of our 1973 and 1935 Constitutions. 1 It is one of the few principles whose truth has been
cherished by the Americans as self-evident. Section 4, Article IV of the U.S. Constitution makes it a duty
of the Federal government to guarantee to every state a "republican form of government." With
understandable fervor, the American authorities imposed republicanism as the cornerstone of our 1935
Constitution then being crafted by its Filipino framers. 2

Borne out of the 1986 people power EDSA revolution, our 1987 Constitution is more people-oriented.
Thus, section 4 of Article II provides as a state policy that the prime duty of the Government is "to serve
and protect the people." Section 1, Article XI also provides that ". . . public officers . . . must at all times
be accountable to the people . . ." Sections 15 and 16 of Article XIII define the role and rights of people's
organizations. Section 5(2) of Article XVI mandates that "[t]he state shall strengthen the patriotic spirit
and nationalist consciousness of the military, and respect for people's rights in the performance of their
duty." And Section 2 of Article XVII provides that "amendments to this Constitution may likewise be
directly proposed by the people through initiative . . ." All these provisions and more are intended to
breathe more life to the sovereignty of our people.

To be sure, the sovereignty of our people is not a kabalistic principle whose dimensions are buried in
mysticism. Its metes and bounds are familiar to the framers of our Constitutions. They knew that in its
broadest sense, sovereignty is meant to be supreme, the jus summi imperu, the absolute right to
govern. 3 Former Dean Vicente Sinco 4 states that an essential quality of sovereignty is legal
omnipotence, viz: "Legal theory establishes certain essential qualities inherent in the nature of
sovereignty. The first is legal omnipotence. This means that the sovereign is legally omnipotent and
absolute in relation to other legal institutions. It has the power to determine exclusively its legal
competence. Its powers are original, not derivative. It is the sole judge of what it should do at any given
time." 5 Citing Barker, 6 he adds that a more amplified definition of sovereignty is that of "a final power
of final legal adjustment of all legal issues." The U.S. Supreme Court expressed the same thought in the
landmark case of Yick Wo v. Hopkins, 7 where it held that ". . . sovereignty itself is, of course, not subject
to law, for it is the author and source of law; but in our system, while sovereign powers are delegated to

415
the agencies of government, sovereignty itself remains with the people, by whom and for whom all
government exists and acts."

In our Constitution, the people established a representative democracy as distinguished from a pure
democracy. Justice Isagani Cruz explains: 8

"xxx xxx xxx

A republic is a representative government, a government run by and for the people. It is not a pure
democracy where the people govern themselves directly. The essence of republicanism is
representation and renovation, the selection by the citizenry of a corps of public functionaries who
derive their mandate from the people and act on their behalf, serving for a limited period only, after
which they are replaced or retained, at the option of their principal. Obviously, a republican government
is a responsible government whose officials hold and discharge their position as a public trust and shall,
according to the Constitution, 'at all times be accountable to the people' they are sworn to serve. The
purpose of a republican government it is almost needless to state, is the promotion of the common
welfare according to the will of the people themselves. "

I appreciate the vigorous dissent of Mr. Justice Davide. I agree that sovereignty is indivisible but it need
not always be exercised by the people together, all the time. 9 For this reason, the Constitution and our
laws provide when the entire electorate or only some of them can elect those who make our laws and
those who execute our laws. Thus, the entire electorate votes for our senators but only our district
electorates vote for our congressmen, only our provincial electorates vote for the members of our
provincial boards, only our city electorates vote for our city councilors, and only our municipal
electorates vote for our councilors. Also, the entire electorate votes for our President and Vice-President
but only our provincial electorates vote for our governors, only our city electorates vote for our mayors,
and only our municipal electorates vote for our mayors. By defining and delimiting the classes of voters
who can exercise the sovereignty of the people in a given election, it cannot be claimed that said
sovereignty has been fragmented.

It is my respectful submission that the issue in the case at bar is not whether the people of Sorsogon
should be given the right to defy the law by allowing Frivaldo to sit as their governor. Rather, the issue
is: whether the will of the voters of Sorsogon clearly choosing Frivaldo as governor ought to be given a
decisive value considering the uncertainty of the law on when a candidate ought to satisfy the
qualification of citizenship. The uncertainty of law and jurisprudence, both here and abroad, on this legal
issue cannot be denied. In the United States, 10 there are two (2) principal schools of thought on the
matter. One espouses the view that a candidate must possess the qualifications for office at the time of
his election. The other ventures the view that the candidate should satisfy the qualifications at the time
he assumes the powers of the office. I am unaware of any Philippine decision that has squarely resolved
this difficult question of law. The ponencia of Mr. Justice Panganiban adhered to the second school of
thought while Mr. Justice Davide dissents.

I emphasize the honest-to-goodness difference in interpreting our law on the matter for this is vital to
dispel the fear of Mr. Justice Davide that my opinion can bring about ill effects to the State. Mr. Justice

416
Davide's fear is based on the assumption that Frivaldo continues to be disqualified and we cannot allow
him to sit as governor without transgressing the law. I do not concede this assumption for as stressed
above, courts have been sharply divided by this mind boggling issue. Given this schism, I do not see how
we can derogate on the sovereignty of the people by according more weight to the votes of the people
of Sorsogon.

Mr. Justice Davide warns that should the people of Batanes stage a rebellion, we cannot prosecute them
"because of the doctrine of people's sovereignty." With due respect, the analogy is not appropriate. In
his hypothetical case, rebellion is concededly a crime, a violation of Article 134 of the Revised Penal
Code, an offense against the sovereignty of our people. In the case at bar, it cannot be held with
certitude that the people of Sorsogon violated the law by voting for Frivaldo as governor. Frivaldo's
name was in the list of candidates allowed by COMELEC to run for governor. At that time too, Frivaldo
was taking all steps to establish his Filipino citizenship. And even our jurisprudence has not settled the
issue when a candidate should possess the qualification of citizenship. Since the meaning of the law is
arguable then and now, I cannot imagine how it will be disastrous for the State if we tilt the balance in
the case at bar in favor of the people of Sorsogon.

In sum, I respectfully submit that the sovereign will of our people should be resolutory of the case at bar
which is one of its kind, unprecedented in our political history. For three (3) times, Frivaldo ran of the
province of Sorsogon. For two (2) times, he was disqualified on the ground of citizenship. The people of
Sorsogon voted for him as their governor despite his disqualification. The people never waffled in their
support for Frivaldo. In 1988, they gave him a winning margin of 27,000; in 1992, they gave him a
winning spread of 57,000; in 1995, he posted a margin of 20,000. Clearly then, Frivaldo is the
overwhelming choice of the people of Sorsogon. In election cases, we should strive to align the will of
the legislature as expressed in its law with the will of the sovereign people as expressed in their ballots.
For law to reign, it must respect the will of the people. For in the eloquent prose of Mr. Justice Laurel, ".
. . an enfranchised citizen is a particle of popular sovereignty and is the ultimate source of established
authority.'' 11 The choice of the governed on who shall be their governor merits the highest
consideration by all agencies of government. In cases where the sovereignty of the people is at stake,
we must not only be legally right but also politically correct. We cannot fail by making the people
succeed.

DAVIDE, JR., J., dissenting:

After deliberating on the re-formulated issues and the conclusions reached by my distinguished
colleague, Mr. Justice Artemio V. Panganiban, I find myself unable to join him.

I agree with petitioner Lee that Frivaldo's repatriation was void, but not on the ground that President
Corazon C. Aquino's 27 March 1987 memorandum "effectively repealed" P.D. No. 725. In my view, the
said memorandum only suspended the implementation of the latter decree by divesting the Special
Committee on Naturalization of its authority to further act on grants of citizenship under LOI No. 270, as
amended; P.D. No. 836, as amended; P.D. No. 1379; and "any other related laws, orders, issuances and

417
rules and regulations." A reading of the last paragraph of the memorandum can lead to no other
conclusion, thus:

In view of the foregoing, you as Chairman and members of the Special Committee on Naturalization, are
hereby directed to cease and desist from undertaking any and all proceedings within your functional
area of responsibility, as defined in Letter of Instruction No. 270 dated April 11, 1975 as amended,
Presidential Decree No. 836 dated December 3, 1975, as amended, and Presidential Decree No. 1379
dated May 17, 1978, relative to the grant of citizenship under the said laws, and any other related laws,
orders, issuances and rules and regulations. (emphasis supplied)

It is self-evident that the underscored clause can only refer to those related to LOI No. 270, P.D. No. 836,
and P.D. No. 1379. There is no doubt in my mind that P.D. No. 725 is one such "related law" as it
involves the reacquisition of Philippine citizenship by repatriation and designates the Special Committee
on Naturalization created under LOI No. 270 to receive and act on (i.e., approve or disapprove)
applications under the said decree. The power of President Aquino to suspend these issuances by virtue
of the 27 March 1987 memorandum is beyond question considering that under Section 6, Article XVIII of
the 1987 Constitution, she exercised legislative power until the Congress established therein convened
on the fourth Monday of July 1987.

I disagree with the view expressed in the ponencia that the memorandum of 27 March 1987 was merely
a declaration of "executive policy," and not an exercise of legislative power. LOI No. 270, P.D. No. 836,
P.D. No. 1379 and "any other related laws," such as P.D. No. 725, were issued by President Ferdinand E.
Marcos in the exercise of his legislative powers — not executive power. These laws relate to the
acquisition (by naturalization) and reacquisition (by repatriation) of Philippine citizenship, and in light of
Sections 1(4) and 3, Article IV of the 1987 Constitution (naturalization and reacquisition of Philippine
citizenship shall be in accordance with law), it is indubitable that these subjects are a matter of
legislative prerogative. In the same vein, the creation of the Special Committee on Naturalization by LOI
No. 270 and the conferment of the power to accept and act on applications under P.D. No. 725 are
clearly legislative acts.

Accordingly, the revocation of the cease and desist order and the reactivation or revival of the
Committee can be done only by legislative fiat, i.e., by Congress, since the President had long lost his
authority to exercise "legislative power." Considering that Congress has not seen it fit to do so, the
President cannot, in the exercise of executive power, lift the cease and desist order nor
reactivate/reconstitute/revive the Committee. A multo fortiori, the Committee cannot validly accept
Frivaldo's application for repatriation and approve it.

II

Even assuming arguendo that Frivaldo's repatriation is valid, it did not cure his lack of citizenship." I
depart from the view in the ponencia that Section 39 of the Local Government Code of 1991 does not
specify the time when the citizenship requirement must be met, and that being the case, then it suffices
that citizenship be possessed upon commencement of the term of the office involved; therefore, since
Frivaldo "re-assumed" his Philippine citizenship at 2:00 p.m. on 30 June 1995 and the term of office of

418
Governor commenced at 12:00 noon of that day, he had, therefore, complied with the citizenship
requirement.

In the first place, Section 39 actually prescribes the qualifications of elective local officials and not those
of an elected local official. These adjectives are not synonymous, as the ponencia seems to suggest. The
first refers to the nature of the office, which requires the process of voting by the electorate involved;
while the second refers to a victorious candidate for an elective office. The section unquestionably
refers to elective — not elected — local officials. It falls under Title Two entitled ELECTIVE OFFICIALS;
under Chapter 1 entitled Qualifications and Election; and paragraph (a) thereof begins with the phrase
"An elective local official," while paragraphs (b) to (f) thereof speak of candidates. It reads as follows:

SEC. 39. Qualifications — (a) An elective local official must be a citizen of the Philippines; a registered
voter in the barangay, municipality, city, or province or, in the case of a member of the sangguniang
panlalawigan, sangguniang panlungsod, or sangguniang bayan, the district where he intends to be
elected; a resident therein for at least one (1) year immediately preceding the day of the election; and
able to read and write Filipino or any other local language or dialect.

(b) Candidates for the position of governor, vice governor or member of the sangguniang
panlalawigan, or mayor, vice mayor or member of the sangguniang panlungsod of highly urbanized cities
must be at least twenty-three (23) years of age on election day.

(c) Candidates for the position of mayor or vice mayor of independent component cities
component cities, or municipalities must be at least twenty-one (21) years of age on election day.

(d) Candidates for the position of member of the sangguniang panlungsod or sangguniang bayan
must be at least eighteen (18) years of age on election day.

(e) Candidates for the position of punong barangay or member of the sangguniang barangay must
be at least eighteen (18) years of age on election day.

(f) Candidates for the sangguniang kabataan must be at least fifteen (15) years of age but not more
than twenty-one (21) years of age on election day (emphasis supplied)

It is thus obvious that Section 39 refers to no other than the qualifications of candidates for elective
local offices and their election. Hence, in no way may the section be construed to mean that possession
of qualifications should be reckoned from the commencement of the term of office of the elected
candidate.

For another, it is not at all true that Section 39 does not specify the time when the citizenship
requirement must be possessed. I submit that the requirement must be satisfied, or that Philippine
citizenship must be possessed, not merely at the commencement of the term, but at an earlier time, the
latest being election day itself. Section 39 is not at all ambiguous nor uncertain that it meant this to be,
as one basic qualification of an elective local official is that he be "A REGISTERED VOTER IN THE
BARANGAY, MUNICIPALITY, CITY OR PROVINCE . . . WHERE HE INTENDS TO VOTE." This simply means
that he possesses all the qualifications to exercise the right of suffrage. The fundamental qualification

419
for the exercise of this sovereign right is the possession of Philippine citizenship. No less than the
Constitution makes it the first qualification, as Section 1, Article V thereof provides:

Section 1. Suffrage may be exercised by all citizens of the Philippines not otherwise disqualified by
law, who are at least eighteen years of age, and who shall have resided in the Philippines for at least one
year and in the place wherein they propose to vote for at least six months immediately preceding the
election . . . (emphasis supplied)

And Section 117 of the Omnibus Election Code of the Philippines (B.P. Blg. 881) expressly provides for
the qualifications of a voter. Thus:

SEC. 117. Qualifications of a voter. — Every citizen of the Philippines, not otherwise disqualified
by law, eighteen years of age or over, who shall have resided in the Philippines for one year and in the
city or municipality wherein he proposes to vote for at least six months immediately preceding the
election, may be a registered voter. (emphasis supplied)

It is undisputed that this Court twice voided Frivaldo's election as Governor in the 1988 and 1992
elections on the ground that for lack of Philippine citizenship — he being a naturalized citizen of the
United States of America — he was DISQUALIFIED to be elected as such and to serve the position
(Frivaldo vs. Commission on Elections, 174 SCRA 245 [1989], Republic of the Philippines vs. De la Rosa,
232 SCRA 785 [1994]). This disqualification inexorably nullified Frivaldo's registration as a voter and
declared it void ab initio. Our judgments therein were self-executory and no further act, e.g., a COMELEC
order to cancel his registration as a voter or the physical destruction of his voter's certificate, was
necessary for the ineffectivity. Thus, he was never considered a registered voter for the elections of May
1992, and May 1995, as there is no showing that Frivaldo registered anew as a voter for the latter
elections. Even if he did — in obvious defiance of his decreed disqualification — this did not make him a
Filipino citizen, hence it was equally void ab initio. That he filed his certificate of candidacy for the 1995
elections and was even allowed to vote therein were of no moment. Neither act made him a Filipino
citizen nor nullified the judgments of this Court. On the contrary, said acts made a mockery of our
judgments. For the Court now to validate Frivaldo's registration as a voter despite the judgments of
disqualification is to modify the said judgments by making their effectivity and enforceability dependent
on a COMELEC order cancelling his registration as a voter, or on the physical destruction of his certificate
of registration as a voter which, of course, was never our intention. Moreover, to sanction Frivaldo's
registration as a voter would be to sacrifice substance in favor of form (the piece of paper that is the
book of voters or list of voters or voter's ID), and abet the COMELEC's incompetence in failing to cancel
Frivaldo's registration and allowing him to vote.

The second reason in the ponencia as to why the citizenship disqualification should be reckoned not
from the date of the election nor the filing of the certificate of candidacy, but from the date of
proclamation, is that the only available remedy to question the ineligibility (or disloyalty) of a candidate
is a petition for quo warranto which, under Section 253 of the Omnibus Election Code, may be filed only
within ten days from proclamation and not earlier.

I beg to differ.

420
Clearly, quo warranto is not the sole remedy available to question a candidate's ineligibility for public
office. Section 78 of the Omnibus Election Code allows the filing of a petition to deny due course to or
cancel the certificate of candidacy on the ground that any material representation contained therein, as
required by Section 74, is false. Section 74, in turn, requires that the person filing the certificate of
candidacy must state, inter alia, that he is eligible for the office, which means that he has all the
qualifications (including, of course, fulfilling the citizenship requirement) and none of the
disqualifications as provided by law. The petition under Section 78 may be filed at any time not later
than 25 days from the filing of the certificate of candidacy. The section reads in full as follows:

SEC. 78. Petition to deny due course to or cancel a certificate of candidacy. — A verified petition seeking
to deny due course or to cancel a certificate of candidacy may be filed by any person exclusively on the
ground that any material representation contained therein as required under Section 74 hereof is false
The petition may be filed at any time not later than twenty-five days from the time of the filing of the
certificate of candidacy and shall be decided, after due notice and hearing, not later than fifteen days
before the election.

This remedy was recognized in Loong vs. Commission on Elections (216 SCRA 760, 768 [1992]), where
this Court held:

Thus, if a person qualified to file a petition to disqualify a certain candidate fails to file the petition
within the 25-day period prescribed by Section 78 of the Code for whatever reasons, the election laws
do not leave him completely helpless as he has another chance to raise the disqualification of the
candidate by filing a petition for quo warranto within ten (10) days from the proclamation of the results
of the election, as provided under Section 253 of the Code. Section 1, Rule 21 of the Comelec Rules of
Procedure similarly provides that any voter contesting the election of any regional, provincial or city
official on the ground of ineligibility or of disloyalty to the Republic of the Philippines may file a petition
for quo warranto with the Electoral Contest Adjudication Department The petition may be filed within
ten (10) days from the date the respondent is proclaimed (Section 2).

Likewise, Rule 25 of the Revised COMELEC Rules of Procedure allows the filing of a petition for
disqualification on the ground of failure to possess all the qualifications of a candidate as provided by
the Constitution or by existing laws, "any day after the last day for filing of certificates of candidacy but
not later than the date of proclamation." Sections 1 and 3 thereof provide:

Rule 25 — Disqualification of Candidates.

SECTION 1. Grounds for Disqualification. — Any candidate who does not possess all the
qualifications of a candidate as provided for by the Constitution or by existing law or who commits any
act declared by law to be grounds for disqualification may be disqualified from continuing as a
candidate.

xxx xxx xxx

421
SECTION 3. Period to File Petition. — The petition shall be filed any day after the last day for
filing of certificates of candidacy but not later than the date of proclamation.

While the validity of this rule insofar as it concerns petitions for disqualification on the ground of lack of
all qualifications may be doubtful, its invalidity is not in issue here.

In this connection, it would seem appropriate to take up the last issue grappled within the ponencia,
viz., is Section 78 of the Omnibus Election Code mandatory? The answer is provided in Loong.

We also do not find merit in the contention of respondent Commission that in the light of the provisions
of Sections 6 and 7 of Rep. Act No. 6646, a petition to deny due course to or cancel a certificate of
candidacy may be filed even beyond the 25-day period prescribed by Section 78 of the Code, as long as
it is filed within a reasonable time from the discovery of the ineligibility.

Sections 6 and 7 of Rep. Act No. 6646 are here re-quoted:

SEC. 6. Effect of Disqualification case. — Any candidate who has been declared by final judgment to be
disqualified shall not be voted for, and the votes cast for him shall not be counted. If for any reason a
candidate is not declared by final judgment before an election to be disqualified and he is voted for and
receives the winning number of votes in such election, the Court or Commission shall continue with the
trial and hearing of the action, inquiry or protest and, upon motion of the complainant or any
intervenor, may during the pendency thereof order the suspension of the proclamation of such
candidate whenever the evidence of his guilt is strong.

SEC. 7. Petition to Deny Due Course To or Cancel a Certificate of Candidacy. — The procedure
hereinabove provided shall apply to petitions to deny due course to or cancel a certificate of candidacy
as provided in Section 78 of Batas Pambansa Blg. 881.

It will be noted that nothing in Sections 6 or 7 modifies or alters the 25-day period prescribed by Section
78 of the Code for filing the appropriate action to cancel a certificate of candidacy on account of any
false representation made therein. On the contrary, said Section 7 affirms and reiterates Section 78 of
the Code.

We note that Section 6 refers only to the effects of a disqualification case which may be based on
grounds other than that provided under Section 78 of the Code. But Section 7 of Rep. Act No. 6646 also
makes the effects referred to in Section 6 applicable to disqualification cases filed under Section 78 of
the Code. Nowhere in Sections 6 and 7 of Rep. Act No. 6646 is mention made of the period within which
these disqualification cases may be filed. This is because there are provisions in the Code which supply
the periods within which a petition relating to disqualification of candidates must be filed, such as
Section 78, already discussed, and Section 253 on petitions for quo warranto.

I then disagree with the asseveration in the ponencia that Section 78 is merely directory because Section
6 of R.A. No. 6646 authorizes the COMELEC to try and decide petitions for disqualification even after
elections. I submit that Section 6 refers to disqualifications under Sections 12 and 68 of the Omnibus
Election Code and consequently modifies Section 72 thereof. As such, the proper court or the COMELEC

422
are granted the authority to continue hearing the case after the election, and during the pendency of
the case, suspend the proclamation of the victorious candidate, if the evidence against him is strong.
Sections 12, 68, and 72 of the Code provide:

SEC. 12. Disqualifications. — Any person who has been declared by competent authority insane or
incompetent, or has been sentenced by final judgment for subversion, insurrection, rebellion or for any
offense for which he has been sentenced to a penalty of more than eighteen months or for a crime
involving moral turpitude, shall be disqualified to be a candidate and to hold any office, unless he has
been given plenary pardon or granted amnesty.

The disqualifications to be a candidate herein provided shall be deemed removed upon declaration by
competent authority that said insanity or incompetence had been removed or after the expiration of a
period of five years from his service of sentence, unless within the same period he again becomes
disqualified.

xxx xxx xxx

SEC. 68. Disqualifications. — Any candidate who, in an action or protest in which he is a party is declared
by final decision of a competent court guilty of, or found by the Commission of having (a) given money
or other material consideration to influence, induce or corrupt the voters or public officials performing
electoral functions; (b) committed acts of terrorism to enhance his candidacy; (c) spent in his election
campaign an amount in excess of that allowed by this Code; (d) solicited, received or made any
contribution prohibited under Sections 89, 95, 96, 97 and 104; or (e) violated any of Sections 80, 83, 85,
86 and 261, paragraphs d, e, k, v, and cc, sub-paragraph 6, shall be disqualified from continuing as a
candidate, or if he has been elected, from holding the office. Any person who is a permanent resident of
or an immigrant to a foreign country shall not be qualified to run for any elective office under this Code,
unless said person has waived his status as permanent resident or immigrant of a foreign country in
accordance with the residence requirement provided for in the election laws. (Sec. 25, 1971 EC)

SEC. 72. Effects of disqualification cases and priority. — The Commission and the courts shall give priority
to cases of disqualification by reason of violation of this Act to the end that a final decision shall be
rendered not later than seven days before the election in which the disqualification is sought.

Any candidate who has been declared by final judgment to be disqualified shall not be voted for, and the
votes cast for him shall not be counted. Nevertheless, if for any reason, a candidate is not declared by
final judgment before an election to be disqualified and he is voted for and receives the winning number
of votes in such election, his violation of the provisions of the preceding sections shall not prevent his
proclamation and assumption to office.

III

Still assuming that the repatriation is valid, I am not persuaded by the arguments in support of the thesis
that Frivaldo's repatriation may be given retroactive effect, as such goes against the spirit and letter of
P.D. No. 725. The spirit adheres to the principle that acquisition or re-acquisition of Philippine citizenship

423
is not a right, but a mere privilege. Before the advent of P.D. No. 725, only the following could apply for
repatriation: (a) Army, Navy, or Air Corps deserters; and (b) a woman who lost her citizenship by reason
of her marriage to an alien after the death of her spouse (Section 2[2], C.A. No. 63). P.D. No. 725
expanded this to include Filipino women who lost their Philippine citizenship by marriage to aliens even
before the death of their alien husbands, or the termination of their marital status and to natural-born
Filipino citizens who lost their Philippine citizenship but subsequently desired to reacquire the latter.

Turning now to the letter of the law, P.D. No. 725 expressly provides that repatriation takes effect only
after taking the oath of allegiance to the Republic of the Philippines, thus:

. . . may reacquire Philippine citizenship . . . by applying with the Special Committee on Naturalization
created by Letter of Instruction No. 270, and, if their applications are approved, taking the necessary
oath of allegiance to the Republic of the Philippines, AFTER WHICH THEY SHALL BE DEEMED TO HAVE
REACQUIRED PHILIPPINE CITIZENSHIP. (italicization and capitalization supplied for emphasis)

Clearly then, the steps to reacquire Philippine citizenship by repatriation under the decree are: (1) filing
the application; (2) action by the committee; and (3) taking of the oath of allegiance if the application is
approved. It is only UPON TAKING THE OATH OF ALLEGIANCE that the applicant is deemed ipso jure to
have reacquired Philippine citizenship. If the decree had intended the oath taking to retroact to the date
of the filing of the application, then it should not have explicitly provided otherwise.

This theory in the ponencia likewise dilutes this Court's pronouncement in the first Frivaldo case that
what reacquisition of Filipino citizenship requires is an act "formally rejecting [the] adopted state and
reaffirming . . . allegiance to the Philippines." That act meant nothing less than taking of the oath of
allegiance to the Republic of the Philippines. If we now take this revision of doctrine to its logical end,
then it would also mean that if Frivaldo had chosen and reacquired Philippine citizenship by
naturalization or through Congressional action, such would retroact to the filing of the petition for
naturalization or the bill granting him Philippine citizenship. This is a proposition which both the first and
second Frivaldo cases soundly rejected.

The other reason adduced in the ponencia in support of the proposition that P.D. No. 725 can be given
retroactive effect is its alleged curative or remedial nature.

Again, I disagree. In the first place, by no stretch of legal hermeneutics may P.D. No. 725 be
characterized as a curative or remedial statute:

Curative or remedial statutes are healing acts. They are remedial by curing defects and adding to the
means of enforcing existing obligations. The rule in regard to curative statutes is that if the thing
omitted or failed to be done, and which constitutes the defect sought to be removed or made harmless,
is something the legislature might have dispensed with by a previous statute, it may do so by a
subsequent one.

424
Curative statutes are intended to supply defects, abridge superfluities in existing laws, and curb certain
evils. They are intended to enable a person to carry into effect that which they have designed and
intended, but has failed of expected legal consequence by reason of some statutory disability or
irregularity in their own action. They make valid that which, before the enactment of the statute, was
invalid. (RUBEN E. AGPALO, Statutory Construction, Second ed. [1990], 270-271, citations omitted).

P.D. No. 725 provides for the reacquisition of Philippine citizenship lost through the marriage of a
Filipina to an alien and through naturalization in a foreign country of natural-born Filipino citizens. It
involves then the substantive, nay primordial, right of citizenship. To those for whom it is intended, it
means, in reality, the acquisition of "a new right," as the ponencia cannot but concede. Therefore, it
may not be said to merely remedy or cure a defect considering that one who has lost Philippine
citizenship does not have the right to reacquire it. As earlier stated, the Constitution provides that
citizenship, once lost, may only be reacquired in the manner provided by law. Moreover, it has also been
observed that:

The idea is implicit from many of the cases that remedial statutes are statutes relating to procedure and
not substantive rights. (Sutherland, Statutory Construction, Vol. 3, Third ed. [1943], § 5704 at 74,
citations omitted).

If we grant for the sake of argument, however, that P.D. No. 725 is a curative or remedial statute, it
would be an inexcusable error to give it a retroactive effect since it explicitly provides the date of its
effectivity. Thus:

This Decree shall take effect immediately.

Done in the city of Manila, this 5th day of June, in the year of Our Lord, nineteen hundred and seventy
five.

Nevertheless, if the retroactivity is to relate only to the reacquisition of Philippine citizenship, then
nothing therein supports such theory, for as the decree itself unequivocally provides, it is only after
taking the oath of allegiance to the Republic of the Philippines that the applicant is DEEMED TO HAVE
REACQUIRED PHILIPPINE CITIZENSHIP.

IV

Assuming yet again, for the sake of argument, that taking the oath of allegiance retroacted to the date
of Frivaldo's application for repatriation, the same could not be said insofar as it concerned the United
States of America, of which he was a citizen. For under the laws of the United States of America, Frivaldo
remained an American national until he renounced his citizenship and allegiance thereto at 2:00 p.m. on
30 June 1995, when he took his oath of allegiance to the Republic of the Philippines. Section 401 of the
Nationality Act of 1940 of the United States of America provides that a person who is a national of the
United States of America, whether by birth or naturalization, loses his nationality by, inter alia, "(b)
Taking an oath or making an affirmation or other formal declaration of allegiance to a foreign state
(SIDNEY KANSAS, U.S. Immigration Exclusion and Deportation and Citizenship of the United States of

425
America. Third ed., [1948] 341-342). It follows then that on election day and until the hour of the
commencement of the term for which he was elected — noon of 30 June 1995 as per Section 43 of the
Local Government Code — Frivaldo possessed dual citizenship, viz., (a) as an American citizen, and (b) as
a Filipino citizen through the adoption of the theory that the effects of his taking the oath of allegiance
were retrospective. Hence, he was disqualified to run for Governor for yet another reason: possession of
dual citizenship, in accordance with Section 40(d) of the Local Government Code.

The assertion in the ponencia that Frivaldo may be considered STATELESS on the basis of his claim that
he "had long renounced and had long abandoned his American citizenship — long before May 8, 1985"
— is untenable, for the following reasons: first, it is based on Frivaldo's unproven, self-serving allegation;
second, informal renunciation or abandonment is not a ground to lose American citizenship; and third,
simply put, never did the status of a STATELESS person attach to Frivaldo.

Statelessness may be either de jure, which is the status of individuals stripped of their nationality by
their former government without having an opportunity to acquire another; or de facto, which is the
status of individuals possessed of a nationality whose country does not give them protection outside
their own country, and who are commonly, albeit imprecisely, referred to as refugees (JORGE R.
COQUIA, et al., Conflict of Laws Cases, Materials and Comments. 1995 ed., 290)

Specifically, under Chapter 1, Article 1 of the United Nations Convention Regarding the Status of
Stateless Persons (Philippine Treaty; Series, Compiled and Annotated by Haydee B. Yorac, vol. III, 363), a
stateless person is defined as "a person who is not considered as a national by any State under the
operation of its law." However, it has not been shown that the United States of America ever ceased to
consider Frivaldo its national at any time before he took his oath of allegiance to the Republic of the
Philippines on 30 June 1995.

VI

Finally, I find it in order to also express my view on the concurring opinion of Mr. Justice Reynato S.
Puno. I am absolutely happy to join him in his statement that "[t]he sovereignty of our people is the
primary postulate of the 1987 Constitution" and that the said Constitution is "more people- oriented,"
"borne [as it is] out of the 1986 people power EDSA revolution." I would even go further by saying that
this Constitution is pro-God (Preamble), pro-people (Article II, Sections 1, 3, 4, 5, 9, 15, 16; Article XI,
Section I; Article XII, Sections 1, 6; Article XIII, Sections 1, 11, 15, 16, 18; Article XVI, Sections 5(2), 6), pro-
Filipino (Article XII, Sections 1, 2, 10, 11, 12, 14. Article XIV, Sections 1, 4(2), 13; Article XVI, Section 11),
pro-poor (Article II, Sections 9, 10, 18, 21; Article XII, Sections 1, 2[3]; Article XIII, Sections 1, 3, 4, 5, 6, 7,
9, 10, 11, 13), pro-life (Article II, Section 12), and pro-family (Article II, Section 12; Article XV).

Nevertheless, I cannot be with him in carrying out the principle of sovereignty beyond what I perceive to
be the reasonable constitutional parameters. The doctrine of people's sovereignty is founded on the
principles of democracy and republicanism and refers exclusively to the sovereignty of the people of the
Philippines. Section 1 of Article II is quite clear on this, thus:

426
SECTION 1. The Philippines is a democratic and republican State. Sovereignty resides in the people
and all government authority emanates from them.

And the Preamble makes it clear when it solemnly opens it with a clause "We, the sovereign Filipino
people . . ." Thus, this sovereignty is an attribute of the Filipino people as one people, one body.

That sovereign power of the Filipino people cannot be fragmentized by looking at it as the supreme
authority of the people of any of the political subdivisions to determine their own destiny; neither can
we convert and treat every fragment as the whole. In such a case, this Court would provide the formula
for the division and destruction of the State and render the Government ineffective and inutile. To
illustrate the evil, we may consider the enforcement of laws or the pursuit of a national policy by the
executive branch of the government, or the execution of a judgment by the courts. If these are opposed
by the overwhelming majority of the people of a certain province, or even a municipality, it would
necessarily follow that the law, national policy, or judgment must not be enforced, implemented, or
executed in the said province or municipality. More concretely, if, for instance, the vast majority of the
people of Batanes rise publicly and take up arms against the Government for the purpose of removing
from the allegiance to the said Government or its laws, the territory of the Republic of the Philippines or
any part thereof, or any body of land, naval, or other armed forces, or depriving the Chief Executive or
the Legislature, wholly or partially, of any of their powers or prerogatives, then those who did so — and
which are composed of the vast majority of the people of Batanes — a political subdivision — cannot be
prosecuted for or be held guilty of rebellion in violation of Article 134 of the Revised Penal Code because
of the doctrine of peoples' sovereignty. Indeed, the expansion of the doctrine of sovereignty by
investing upon the people of a mere political subdivision that which the Constitution places in the entire
Filipino people, may be disastrous to the Nation.

So it is in this case if we follow the thesis in the concurring opinion. Thus, simply because Frivaldo had
obtained a margin of 20,000 votes over his closest rival, Lee, i.e., a vast majority of the voters of
Sorsogon had expressed their sovereign will for the former, then this Court must yield to that will and
must, therefore, allow to be set aside, for Frivaldo, not just the laws on qualifications of candidates and
elective officials and naturalization and reacquisition of Philippine citizenship, but even the final and
binding decisions of this Court affecting him.

This Court must be the first to uphold the Rule of Law. I vote then to DISMISS G.R. No. 120295 and
GRANT G.R. No. 123755.

427
Romualdez-Marcos vs. COMELEC, 248 SCRA 300

[G.R. No. 119976. September 18, 1995.]

IMELDA ROMUALDEZ-MARCOS, petitioner, vs. COMMISSION ON ELECTIONS and CIRILO ROY MONTEJO,
respondents.

Estelito P. Mendoza for petitioner.

The Solicitor General for public respondent.

Paquito N . Ochoa, Jr. and Gracelda N . Andres for private respondent.

SYLLABUS

1. CIVIL LAW; DOMICILE; CONSTRUED. — Article 50 of the Civil Code decrees that "[f]or the
exercise of civil rights and the fulfillment of civil obligations, the domicile of natural persons is their
place of habitual residence." In Ong vs. Republic this court took the concept of domicile to mean an
individual's "permanent home," "a place to which, whenever absent for business or for pleasure, one
intends to return, and depends on facts and circumstances in the sense that they disclose intent." Based
on the foregoing, domicile includes the twin elements of "the fact of residing or physical presence in a
fixed place" and animus manendi, or the intention of returning there permanently.

2. ID.; ID.; RESIDENCE, CONSTRUED. — Residence, in its ordinary conception, implies the factual
relationship of an individual to a certain place. It is the physical presence of a person in a given area,
community or country.

3. ID.; ID.; DIFFERENTIATED FROM RESIDENCE. — The essential distinction between residence and
domicile in law is that residence involves the intent to leave when the purpose for which the resident
has taken up his abode ends. One may seek a place for purposes such as pleasure, business, or health. If
a person's intent be to remain, it becomes his domicile; if his intent is to leave as soon as his purpose is
established it is residence. It is thus, quite perfectly normal for an individual to have different residences
in various places. However, a person can only have a single domicile, unless, for various reasons, he
successfully abandons his domicile in favor of another domicile of choice.

4. POLITICAL LAW; ELECTIONS; RESIDENCE USED SYNONYMOUSLY WITH DOMICILE. — For political
purposes the concepts of residence and domicile are dictated by the peculiar criteria of political laws. As
these concepts have evolved in our election law, what has clearly and unequivocally emerged is the fact
that residence for election purposes is used synonymously with domicile.

5. ID.; ID.; ID.; ABSENCE FROM PERMANENT RESIDENCE WITHOUT INTENTION TO ABANDON IT
DOES NOT RESULT IN LOSS OR CHANGE OF DOMICILE. — So settled is the concept (of domicile) in our
election law that in these and other election law cases, this Court has stated that the mere absence of
an individual from his permanent residence without the intention to abandon it does not result in a loss
or change of domicile. The deliberations of the 1987 Constitution on the residence qualification for

428
certain elective positions have placed beyond doubt the principle that when the Constitution speaks of
"residence" in election law, it actually means only "domicile."

6. ID.; ID.; ID.; FACT OF RESIDENCE, NOT STATEMENT IN CERTIFICATE OF CANDIDACY, DECISIVE
FACTOR IN DETERMINING RESIDENCY QUALIFICATION REQUIREMENT. — It is the fact of residence, not a
statement in a certificate of candidacy which ought to be decisive in determining whether or not an
individual has satisfied the constitution's residency qualification requirement. The said statement
becomes material only when there is or appears to be a deliberate attempt to mislead, misinform, or
hide a fact which would otherwise render a candidate ineligible. It would be plainly ridiculous for a
candidate to deliberately and knowingly make a statement in a certificate of candidacy which would
lead to his or her disqualification. cdlex

7. ID.; ID.; ID.; ID.; CASE AT BAR. — It stands to reason therefore, that petitioner merely committed
an honest mistake in jotting down the word "seven" in the space provided for the residency qualification
requirement. The circumstances leading to her filing the questioned entry obviously resulted in the
subsequent confusion which prompted petitioner to write down the period of her actual stay in Tolosa,
Leyte instead of her period of residence in the First district, which was "since childhood" in the space
provided. These circumstances and events are amply detailed in the COMELEC's Second Division's
questioned resolution, albeit with a different interpretation. For instance, when herein petitioner
announced that she would be registering in Tacloban City to make her eligible to run in the First District,
private respondent Montejo opposed the same, claiming that petitioner was a resident of Tolosa, not
Tacloban City. Petitioner then registered in her place of actual residence in the First District, which was
Tolosa, Leyte, a fact which she subsequently noted down in her Certificate of Candidacy. A close look at
said certificate would reveal the possible source of the confusion: the entry for residence (Item No. 7) is
followed immediately by the entry for residence in the constituency where a candidate seeks election.
Having been forced by private respondent to register in her place of actual residence in Leyte instead of
petitioner's claimed domicile, it appears that petitioner had jotted down her period of stay in her legal
residence or domicile. The juxtaposition of entries in Item 7 and Item 8 — the first requiring actual
residence and the second requiring domicile — coupled with the circumstances surrounding petitioner's
registration as a voter in Tolosa obviously led to her writing down an unintended entry for which she
could be disqualified. This honest mistake should not, however, be allowed to negate the fact of
residence in the First District if such fact were established by means more convincing than a mere entry
on a piece of paper. cdll

8. ID.; ID.; ID.; ABSENCE FROM LEGAL RESIDENCE OR DOMICILE OF A TEMPORARY OR SEMI-
PERMANENT NATURE DOES NOT CONSTITUTE LOSS OF RESIDENCE. — We have stated, many times in
the past, that an individual does not lose his domicile even if he has lived and maintained residences in
different places. Residence, it bears repeating, implies a factual relationship to a given place for various
purposes. The absence from legal residence or domicile to pursue a profession, to study or to do other
things of a temporary or semi-permanent nature does not constitute loss of residence. Thus, the
assertion by the COMELEC that "she could not have been a resident of Tacloban City since childhood up
to the time she filed her certificate of candidacy because she became a resident of many places" flies in

429
the face of settled jurisprudence in which this Court carefully made distinctions between (actual)
residence and domicile for election law purposes.

9. CIVIL LAW; DOMICILE; A MINOR FOLLOWS THE DOMICILE OF HIS PARENTS; CASE AT BENCH. — A
minor follows the domicile of his parents. As domicile, once acquired is retained until a new one is
gained, it follows that in spite of the fact of petitioner's being born in Manila, Tacloban, Leyte was her
domicile of origin by operation of law. This domicile was not established only when her father brought
his family back to Leyte contrary to private respondent's averments.

10. ID.; ID.; REQUISITES TO EFFECT CHANGE OF DOMICILE. — Domicile of origin is not easily lost. To
successfully effect a change of domicile, one must demonstrate: 1. An actual removal or an actual
change of domicile; 2. A bona fide intention of abandoning the former place of residence and
establishing a new one; and 3. Acts which correspond with the purpose.

11. ID.; ID.; ID.; CASE AT BENCH. — In the absence of clear and positive proof based on these
criteria, the residence of origin should be deemed to continue. Only with evidence showing concurrence
of all three requirements can the presumption of continuity or residence be rebutted, for a change of
residence requires an actual and deliberate abandonment, and one cannot have two legal residences at
the same time. In the case at bench, the evidence adduced by private respondent plainly lacks the
degree of persuasiveness required to convince this court that an abandonment of domicile of origin in
favor of a domicile of choice indeed occurred. To effect an abandonment requires the voluntary act of
relinquishing petitioner's former domicile with an intent to supplant the former domicile with one of her
own choosing (domicilium voluntarium).

12. ID.; ID.; ID.; MARRIAGE, NOT A CAUSE FOR LOSS OF DOMICILE. — In this connection, it cannot be
correctly argued that petitioner lost her domicile of origin by operation of law as a result of her marriage
to the late President Ferdinand E. Marcos in 1954. For there is a clearly established distinction between
the Civil Code concepts of "domicile" and "residence." The presumption that the wife automatically
gains the husband's domicile by operation of law upon marriage cannot be inferred from the use of the
term "residence" in Article 110 of the Civil Code because the Civil Code is one area where the two
concepts are well delineated. A survey of jurisprudence relating to Article 110 or to the concepts of
domicile or residence as they affect the female spouse upon marriage yields nothing which would
suggest that the female spouse automatically loses her domicile of origin in favor of the husband's
choice of residence upon marriage. Article 110 is a virtual restatement of Article 58 of the Spanish Civil
Code of 1889: La mujer esta obligada a seguir a su marido donde quiera que fije su residencia. Los
Tribunales, sin embargo, podran con justa causa eximirla de esta obligacion cuando el marido transende
su residencia a ultramar o' a pais extranjero. Note the use of the phrase "donde quiera su fije de
residencia" in the aforequoted article, which means wherever (the husband) wishes to establish
residence. This part of the article clearly contemplates only actual residence because it refers to a
positive act of fixing a family home or residence. Moreover, this interpretation is further strengthened
by the phrase "cuando el marido translade su residencia" in the same provision which means, "when the
husband shall transfer his residence," referring to another positive act of relocating the family to
another home or place of actual residence. The article obviously cannot be understood to refer to

430
domicile which is a fixed, fairly-permanent concept when it plainly connotes the possibility of
transferring from one place to another not only once, but as often as the husband may deem fit to move
his family, a circumstance more consistent with the concept of actual residence. Very significantly,
Article 110 of the Civil Code is found under Title V under the heading: RIGHTS AND OBLIGATIONS
BETWEEN HUSBAND AND WIFE. Immediately preceding Article 110 is Article 109 which obliges the
husband and wife to live together. The duty to live together can only be fulfilled if the husband and wife
are physically together. This takes into account the situations where the couple has many residences (as
in the case of petitioner). If the husband has to stay in or transfer to any one of their residences, the
wife should necessarily be with him in order that they may "live together." Hence, it is illogical to
conclude that Art. 110 refers to "domicile" and not to "residence." Otherwise, we shall be faced with a
situation where the wife is left in the domicile while the husband, for professional or other reasons,
stays in one of their (various) residences.

13. ID.; ID.; TERM RESIDENCE REFERS TO "ACTUAL RESIDENCE." — The term residence may mean
one thing in civil law (or under the Civil Code) and quite another thing in political law. What stands clear
is that insofar as the Civil Code is concerned — affecting the rights and obligations of husband and wife
— the term residence should only be interpreted to mean "actual residence." The inescapable
conclusion derived from this unambiguous civil law delineation therefore, is that when petitioner
married the former President in 1954, she kept her domicile of origin and merely gained a new home,
not a domicilium necessarium.

14. STATUTORY CONSTRUCTION; STATUTE REQUIRING RENDITION OF JUDGMENT WITHIN


SPECIFIED TIME, MERELY DIRECTORY. — It is a settled doctrine that a statute requiring rendition of
judgment within a specified time is generally construed to be merely directory, "so that non-compliance
with them does not invalidate the judgment on the theory that if the statute had intended such result it
would have clearly indicated it." The difference between a mandatory and a directory provision is often
made on grounds of necessity.

15. CONSTITUTIONAL LAW; COMELEC; JURISDICTION TO DECIDE PENDING DISQUALIFICATION CASE


NOT LOST BY HOLDING OF ELECTIONS. — With the enactment of Sections 6 and 7 of R.A. 6646 in
relation to Section 78 of B.P. 881, it is evident that the respondent Commission does not lose jurisdiction
to hear and decide a pending disqualification case under Section 78 of B.P. 881 even after the elections.

16. ID.; LEGISLATURE; HOUSE OF REPRESENTATIVES ELECTORAL TRIBUNAL (HRET); SOLE JUDGE OF
ALL CONTESTS RELATING TO ELECTIONS, RETURNS AND QUALIFICATIONS OF MEMBERS OF CONGRESS;
CANDIDATE MUST HAVE BEEN PROCLAIMED. — As to the House of Representatives Electoral Tribunal's
supposed assumption of jurisdiction over the issue of petitioner's qualifications after the May 8, 1995
elections, suffice it to say that HRET's jurisdiction as the sole judge of all contests relating to the
elections, returns and qualifications of members of Congress begins only after a candidate has become a
member of the House of Representatives. Petitioner not being a member of the House of
Representatives, it is obvious that the HRET at this point has no jurisdiction over the question.

PUNO, J ., concurring opinion:

431
1. CIVIL LAW; DOMICILE; DOMICILE OF ORIGIN AND DOMICILE OF CHOICE; ESTABLISHED BY
CANDIDATE'S CONTINUED STAY IN HER PARENT'S RESIDENCE. — There is no question that petitioner's
original domicile is in Tacloban, Leyte. Her parents were domiciled in Tacloban. Their ancestral house is
in Tacloban. They have vast real estate in the place. Petitioner went to school, and thereafter worked
there. Justice Puno considers Tacloban as her initial domicile, both her domicile of origin and her
domicile of choice. Her domicile of origin as it was the domicile of her parents when she was a minor;
and her domicile of choice, as she continued living there even after reaching the age of majority.

2. ID.; ID.; DOMICILE BY OPERATION OF LAW; ACQUIRED BY MARRIAGE AND DELIBERATE CHOICE
OF A DIFFERENT DOMICILE BY THE HUSBAND. — There is also no question that in May, 1954, petitioner
married the late President Ferdinand E. Marcos. By contracting marriage, her domicile became subject
to change by law, and the right to change it was given by Article 110 of the Civil Code. The difficult issues
start as we determine whether petitioner's marriage to former President Marcos ipso facto resulted in
the loss of her Tacloban domicile. Justice Puno respectfully submits that her marriage by itself alone did
not cause her to lose her Tacloban domicile. Article 110 of the Civil Code merely gave the husband the
right to fix the domicile of the family. In the exercise of the right, the husband may explicitly choose the
prior domicile of his wife, in which case, the wife's domicile remains unchanged. The husband can also
implicitly acquiesce to his wife's prior domicile even if it is different. It is not, therefore, the mere fact of
marriage but the deliberate choice of a different domicile by the husband that will change the domicile
of a wife from what it was prior to their marriage. The domiciliary decision made by the husband in the
exercise of the right conferred by Article 110 of the Civil Code binds the wife. Any and all acts of a wife
during her coverture contrary to the domiciliary choice of the husband cannot change in any way the
domicile legally fixed by the husband. These acts are void not only because the wife lacks the capacity to
choose her domicile but also because they are contrary to law and public policy. prLL

3. ID.; ID.; ID.; CASE AT BENCH. — In the case at bench, it is not disputed that former President
Marcos exercised his right to fix the family domicile and established it in Batac, Ilocos Norte, where he
was then the congressman. At that particular point of time and throughout their married life, petitioner
lost her domicile in Tacloban, Leyte. Since petitioner's Batac domicile has been fixed by operation of law,
it was not affected in 1959 when her husband was elected as Senator, when they lived in San Juan, Rizal
and where she registered as a voter. It was not also affected in 1965 when her husband was elected
President, when they lived in Malacañang Palace, and when she registered as a voter in San Miguel,
Manila. Nor was it affected when she served as a member of the Batasang Pambansa, Minister of
Human Settlements and Governor of Metro Manila during the incumbency of her husband as President
of the nation. Under Article 110 of the Civil Code, it was only her husband who could change the family
domicile in Batac and the evidence shows he did not effect any such change. To a large degree, this
follows the common law that "a woman on her marriage loses her own domicile and by operation of
law, acquires that of her husband, no matter where the wife actually lives or what she believes or
intends."

4. ID.; ID.; ID.; PRINCIPLE THAT AFTER THE HUSBAND'S DEATH, WIFE RETAINS LAST DOMICILE OF
HER HUSBAND, SHOULD NOW BE ABANDONED. — The more difficult task is how to interpret the effect
of the death on September 28, 1989 of former President Marcos on petitioner's Batac domicile. The

432
issue is of first impression in our jurisdiction and two (2) schools of thought contend for acceptance. One
is espoused by our distinguished colleague, Mr. Justice Davide, Jr., heavily relying on American
authorities. He echoes the theory that after the husband's death, the wife retains the last domicile of
her husband until she makes an actual change. The American case law that the wife still retains her dead
husband's domicile is based on ancient common law which we can no longer apply in the Philippine
setting today. The presumption that the wife retains the domicile of her deceased husband is an
extension of this common law concept. The concept and its extension have provided some of the most
iniquitous jurisprudence against women. The rulings relied upon by Mr. Justice Davide in CJS and AM
JUR 2d are American state court decisions handed down between the years 1917 and 1938, or before
the time when women were accorded equality of rights with men. Undeniably, the women's liberation
movement resulted in far-ranging state legislations in the United States to eliminate gender inequality.
However, it has been declared that under modern statutes changing the status of married women and
departing from the common law theory of marriage, there is no reason why a wife may not acquire a
separate domicile for every purpose known to the law. In publishing in 1969 the Restatement of the
Law, Second (Conflict of Laws 2d), the reputable American Law Institute also categorically stated that
the view of Blackstone ". . . is no longer held. As the result of statutes and court decisions, a wife now
possesses practically the same rights and powers as her unmarried sister." In light of the Family Code
which abrogated the inequality between husband and wife as started and perpetuated by the common
law, there is no reason in espousing the anomalous rule that the wife still retains the domicile of her
dead husband. Article 110 of the Civil Code which provides the statutory support for this stance has
been repealed by Article 69 of the Family Code. By its repeal, it becomes a dead-letter law, and we are
not free to resurrect it by giving it further effect in any way or manner such as by ruling that the
petitioner is still bound by the domiciliary determination of her dead husband.

5. ID.; ID.; ID.; WIFE REACQUIRED DOMICILE OF ORIGIN UPON DEATH OF HUSBAND. — Prescinding
from these premises, Justice Puno respectfully submits that the better stance is to rule that petitioner
reacquired her Tacloban domicile upon the death of her husband in 1989. This is the necessary
consequence of the view that petitioner's Batac dictated domicile did not continue after her husband's
death; otherwise, she would have no domicile and that will violate the universal rule that no person can
be without a domicile at any point of time. This stance also restores the right of petitioner to choose her
domicile before it was taken away by Article 110 of the Civil Code, a right now recognized by the Family
Code and protected by the Constitution. Likewise, Justice Puno cannot see the fairness of the common
law requiring petitioner to choose again her Tacloban domicile before she could be released from her
Batac domicile. She lost her Tacloban domicile not through her act but through the act of her deceased
husband when he fixed their domicile in Batac. Her husband is dead and he cannot rule her beyond the
grave. The law disabling her to choose her own domicile has been repealed. Considering all these,
common law should not put the burden on petitioner to prove she has abandoned her dead husband's
domicile. There is neither rhyme nor reason for this gender-based burden. Llibris

6. ID.; ID.; ID.; ID.; DELIBERATE CHOICE BY WIFE MANIFEST IN CASE AT BAR. — But even assuming
arguendo that there is need for convincing proof that petitioner chose to reacquire her Tacloban
domicile, still, the records reveal ample evidence to this effect. In her affidavit submitted to the

433
respondent COMELEC, petitioner averred among others that: "I was not permitted, however, to live and
stay in the Sto. Niño Shrine residence in Tacloban City where I wanted to stay and reside, after repairs
and renovations were completed. In August 1994, I transferred from San Jose, Tacloban City, to my
residence in Barangay Olot, Tolosa, Leyte, when PCGG permitted me to stay and live there." It is then
clear that in 1992 petitioner reestablished her domicile in the First District of Leyte. It is not disputed
that in 1992, she first lived at the house of her brother in San Jose, Tacloban City and later, in August
1994, she transferred her residence in Barangay Olot, Tolosa, Leyte. Both Tacloban City and the
municipality of Olot are within the First District of Leyte. Since petitioner reestablished her old domicile
in 1992 in the First District of Leyte, she more than complied with the constitutional requirement of
residence ". . . for a period of not less than one year immediately preceding the day of the election," i.e.,
the May 8, 1995 elections.

7. POLITICAL LAW; ELECTIONS; CERTIFICATE OF CANDIDACY; AMENDMENT TO CORRECT A BONA


FIDE MISTAKE, ALLOWED AS A MATTER OF RIGHT. — The amendment of a certificate of candidacy to
correct a bona fide mistake has been allowed by this Court as a matter of course and as a matter of
right. (Alialy v. COMELEC , 2 SCRA 957, 960 [1961]; Canceran v. COMELEC , 107 Phil. 607 [1960];
Gabaldon v. COMELEC , 99 Phil. 898 [1956])

8. CONSTITUTIONAL LAW; FREEDOM FROM HARASSMENT AND DISCRIMINATION OF BONA FIDE


CANDIDATES FOR PUBLIC OFFICE; RIGHT VIOLATED BY LEGAL AND EXTRA-LEGAL OBSTACLES AGAINST
CANDIDATE TO PREVENT HER FROM RUNNING. — Section 10, Article IX-C of the Constitution mandates
that "bona fide candidates for any public office shall be free from any form of harassment and
discrimination." A detached reading of the records of the case at bench will show that all forms of legal
and extra-legal obstacles have been thrown against petitioner to prevent her from running as the
people's representative in the First District of Leyte. In petitioner's Answer to the petition to disqualify
her, she averred that when respondent (petitioner herein) announced that she was intending to register
as a voter in Tacloban City and run for Congress in the First District of Leyte, petitioner (Montejo)
immediately opposed her intended registration by writing a letter stating that she is not a resident of
said city but of Barangay Olot, Tolosa, Leyte. After respondent (petitioner herein) had registered as a
voter in Tolosa following completion of her six-month actual residence therein, petitioner (Montejo)
filed a petition with the COMELEC to transfer the town of Tolosa from the First District to the Second
District and pursued such move up to the Supreme Court in G.R. No. 118702, his purpose being to
remove respondent (petitioner herein) as petitioner's (Montejo's) opponent in the congressional
election in the First District. He also filed a bill, along with other Leyte Congressmen, seeking to create
another legislative district, to remove the town of Tolosa out of the First District and to make it a part of
the new district, to achieve his purpose. However, such bill did not pass the Senate. Having failed on
such moves, petitioner now filed the instant petition, for the same objective, as it is obvious that he is
afraid to submit himself along with respondent (petitioner herein) for the judgment and verdict of the
electorate of the First District of Leyte in an honest, orderly, peaceful, free and clean elections on May 8,
1995. All these attempts to misuse our laws and legal processes are forms of rank harassments and
invidious discriminations against petitioner to deny her equal access to a public office. We cannot
commit any hermeneutic violence to the Constitution by torturing the meaning of equality, the end

434
result of which will allow the harassment and discrimination of petitioner who has lived a controversial
life, a past of alternating light and shadow. There is but one Constitution for all Filipinos. Petitioner
cannot be adjudged by a "different" Constitution, and the worst way to interpret the Constitution is to
inject in its interpretation, bile and bitterness.

9. POLITICAL LAW; ELECTIONS; ONE YEAR RESIDENCY REQUIREMENT; RATIONALE; CANDIDATE'S


LIFETIME CONTACTS WITH FIRST DISTRICT OF LEYTE SATISFIES INTENT. — In Gallego v. Vera, we
explained that the reason for this residence requirement is "to exclude a stranger or newcomer,
unacquainted with the conditions and needs of a community and not identified with the latter, from an
elective office to serve that community. . . ." Petitioner's lifetime contacts with the First District of Leyte
cannot be contested. Nobody can claim that she is not acquainted with its problems because she is a
stranger to the place. None can argue she cannot satisfy the intent of the Constitution.

10. ID.; ID.; ELECTION CASES; DOMINANT CONSIDERATION IN RESOLUTION THEREOF IS THE NEED
TO EFFECTUATE WILL OF THE ELECTORATE. — In resolving election cases, a dominant consideration is
the need to effectuate the will of the electorate. The election results show that petitioner received
Seventy Thousand Four Hundred Seventy-One (70,471) votes, while private respondent got only Thirty-
Six Thousand Eight Hundred Thirty-Three (36,833) votes. Petitioner is clearly the overwhelming choice of
the electorate of the First District of Leyte and this is not a sleight of statistics. We cannot frustrate this
sovereign will on highly arguable technical considerations. In case of doubt, we should lean towards a
rule that will give life to the people's political judgment.

FRANCISCO, J ., concurring opinion:

1. CIVIL LAW; DOMICILE; DEFINED. — Domicile has been defined as that place in which a person's
habitation is fixed, without any present intention of removing therefrom, and that place is properly the
domicile of a person in which he has voluntarily fixed his abode, or habitation, not for a mere special or
temporary purpose, but with a present intention of making it his permanent home (28 C.J.S. 1). It
denotes a fixed permanent residence to which when absent for business, or pleasure, or for like reasons
one intends to return, and depends on facts and circumstances, in the sense that they disclose intent.
(Ong Huan Tin v. Republic, 19 SCRA 966, 969)

2. ID.; ID.; CLASSIFICATIONS. — Domicile is classified into domicile of origin and domicile of choice.
The law attributes to every individual a domicile of origin, which is the domicile of his parents, or of the
head of his family, or of the person on whom he is legally dependent at the time of his birth. While the
domicile of origin is generally the place where one is born or reared, it may be elsewhere (28 C.J.S. 5).
Domicile of choice, on the other hand, is the place which the person has elected and chosen for himself
to displace his previous domicile; it has for its true basis or foundation the intention of the person (28
C.J.S. 6). A third classification is domicile by operation of law which attributes to a person a domicile
independent of his own intention or actual residence, ordinarily resulting from legal domestic relations,
as that of the wife arising from marriage, or the relation of a parent and a child (28 C.J.S. 7).

3. ID.; ID.; CHANGE OF DOMICILE; REQUISITES. — In order to hold that a person has abandoned his
domicile and acquired a new one called domicile of choice, the following requisites must concur,

435
namely, (a) residence or bodily presence in the new locality, (b) intention to remain there or animus
manendi, and (c) an intention to abandon the old domicile or animus non revertendi (Romualdez v. RTC ,
Br. 7, Tacloban City, 226 SCRA 408, 415).

4. POLITICAL LAW; ELECTIONS; RESIDENCE SYNONYMOUS WITH DOMICILE. — In election law,


when our Constitution speaks of residence for election purposes it means domicile (Co v. Electoral
Tribunal of the House of Representatives, 199 SCRA 692, 713; Nuval v. Guray, 52 Phil. 645, 651).

5. ID.; ID.; ID.; NOT ABANDONED OR LOST BY REGISTRATION OF VOTER IN A PLACE OTHER THAN
HIS PLACE OF ORIGIN. — In several decisions, though, the Court has laid down the rule that registration
of a voter in a place other than his place of origin is not sufficient to constitute abandonment or loss of
such residence (Faypon v. Quirino, 96 Phil. 294, 300). Respondent Commission offered no cogent reason
to depart from this rule except to surmise petitioner's intent of abandoning her domicile of origin.

6. ID.; ID.; ID.; MARITAL DOMICILE; LOST UPON DEATH OF HUSBAND; WIFE REVERTED TO HER
ORIGINAL DOMICILE; CASE AT BAR. — Tacloban, Leyte, is petitioner's domicile of origin which was
involuntarily supplanted with another, i.e., Batac, Ilocos Norte, upon her marriage in 1954 with then
Congressman Marcos. By legal fiction she followed the domicile of her husband. In my view, the reason
for the law is for the spouses to fully and effectively perform their marital duties and obligations to one
another. The question of domicile, however, is not affected by the fact that it was the legal or moral
duty of the individual to reside in a given place (28 C.J.S. 11). Thus, while the wife retains her marital
domicile so long as the marriage subsists, she automatically loses it upon the latter's termination, for the
reason behind the law then ceases. Otherwise, petitioner, after her marriage was ended by the death of
her husband, would be placed in a quite absurd and unfair situation of having been freed from all wifely
obligations yet made to hold on to one which no longer serves any meaningful purpose. It is my view
therefore that petitioner reverted to her original domicile of Tacloban, Leyte upon her husband's death
without even signifying her intention to that effect.

7. ID.; ID.; ID.; PARTY CLAIMING THAT A PERSON HAS ABANDONED OR LOST HIS RESIDENCE OF
ORIGIN MUST SHOW AND PROVE SUCH LOSS OR ABANDONMENT. — It is for the private respondent to
prove, not for petitioner to disprove, that petitioner has effectively abandoned Tacloban, Leyte for
Batac, Ilocos Norte or for some other place/s. The clear rule is that it is the party (herein private
respondent) claiming that a person has abandoned or lost his residence of origin who must show and
prove preponderantly such abandonment or loss (Faypon v. Quirino, supra at 298; 28 C.J.S. 16), because
the presumption is strongly in favor of an original or former domicile, as against an acquired one (28
C.J.S. 16). Private respondent unfortunately failed to discharge this burden as the record is devoid of
convincing proof that petitioner has acquired, whether voluntarily or involuntarily, a new domicile to
replace her domicile of origin.

8. ID.; ID.; ID.; ONE-YEAR RESIDENCE REQUIREMENT; REQUIREMENT MET IN CASE AT BENCH. —
The records, on the contrary, clearly show that petitioner has complied with the constitutional one-year
residence requirement. After her exile abroad, she returned to the Philippines in 1991 to reside in Olot,
Tolosa, Leyte, but the Presidential Commission on Good Government which sequestered her residential

436
house and other properties forbade her necessitating her transient stay in various places in Manila. In
1992, she ran for the position of president writing in her certificate of candidacy her residence as San
Juan, Metro Manila. After her loss therein, she went back to Tacloban City, acquired her residence
certificate and resided with her brother in San Jose. She resided in San Jose, Tacloban City until August
of 1994 when she was allowed by the PCGG to move and reside in her sequestered residential house in
Olot, Tolosa, Leyte. It was in the same month of August when she applied for the cancellation of her
previous registration in San Juan, Metro Manila in order to register anew as voter of Olot, Tolosa, Leyte,
which she did on January 28, 1995. From this sequence of events, I find it quite improper to use as the
reckoning period of the one-year residence requirement the date when she applied for the cancellation
of her previous registration in San Juan, Metro Manila. The fact which private respondent never
bothered to disprove is that petitioner transferred her residence after the 1992 presidential election
from San Juan, Metro Manila to San Jose, Tacloban City, and resided therein until August of 1994. She
later transferred to Olot, Tolosa, Leyte. It appearing that both Tacloban City and Tolosa, Leyte are within
the First Congressional District of Leyte, it indubitably stands that she had more than a year of residence
in the constituency she sought to be elected. Petitioner, therefore, has satisfactorily complied with the
one-year qualification required by the 1987 Constitution.

PADILLA, J ., dissenting opinion:

1. REMEDIAL LAW; SPECIAL CIVIL ACTIONS; CERTIORARI; COMELEC DID NOT COMMIT GRAVE
ABUSE OF DISCRETION IN DISQUALIFYING CANDIDATE FOR FAILURE TO COMPLY WITH THE ONE YEAR
RESIDENCE QUALIFICATION. — The one year residence period is crucial regardless of whether or not the
term "residence" is to be synonymous with "domicile." In other words, the candidate's intent and actual
presence in one district must in all situations satisfy the length of time prescribed by the fundamental
law. And this, because of a definite Constitutional purpose. He must be familiar with the environment
and problems of a district he intends to represent in Congress and the one-year residence in said district
would be the minimum period to acquire such familiarity, if not versatility. Petitioner's certificate of
candidacy filed on 8 March 1995 contains the decisive component or seed of her disqualification. It is
contained in her answer under oath of "seven months" to the query of "residence in the constituency
wherein I seek to be elected immediately preceding the election." It follows from all the above that the
Comelec committed no grave abuse of discretion in holding that petitioner is disqualified from the
position of representative for the 1st congressional district of Leyte in the elections of 8 May 1995, for
failure to meet the "not less than one-year residence in the constituency (1st district, Leyte)
immediately preceding the day of election (8 May 1995)."

2. POLITICAL LAW; ELECTIONS; DISQUALIFICATION; CANDIDATE WHO OBTAINED THE SECOND


HIGHEST NUMBER OF VOTES CAN NOT BE DECLARED WINNER OF ELECTIVE OFFICE WHERE CANDIDATE
WHO OBTAINED THE HIGHEST NUMBER OF VOTES IS DECLARED DISQUALIFIED OR NOT ELIGIBLE FOR
OFFICE. — The fact that the candidate who obtained the highest number of votes is later declared to be
disqualified or not eligible for the office to which he was elected does not necessarily entitle the
candidate who obtained the second highest number of votes to be declared the winner of the elective
office. The votes cast for a dead, disqualified, or non-eligible person may not be valid to vote the winner
into office or maintain him there. However, in the absence of a statute which clearly asserts a contrary

437
political and legislative policy on the matter, if the votes were cast in the sincere belief that the
candidate was alive, qualified, or eligible, they should not be treated as stray, void or meaningless. (Labo
vs. Comelec, G.R. 86564, August 1, 1989, 176 SCRA 1)

3. ID.; ID.; REPUBLIC ACT NO. 6646 (AN ACT INTRODUCING REFORMS IN THE ELECTORAL SYSTEM
AND FOR OTHER PURPOSES); VOTES CAST FOR A CANDIDATE DECLARED DISQUALIFIED BY FINAL
JUDGMENT SHALL NOT BE COUNTED; CANDIDATE WHO OBTAINED THE SECOND HIGHEST NUMBER OF
VOTES WHERE THE WINNING CANDIDATE IS DECLARED DISQUALIFIED DEEMED THE WINNER. — Under
Sec. 6 of RA 6646, (An Act Introducing Additional Reforms in the Electoral System and for other
purposes) (84 O.G. 905, 22 February 1988) it is provided that: . . . — Any candidate who has been
declared by final judgment to be disqualified shall not be voted for, and the votes cast for him shall not
be counted. The law is clear that in all situations, the votes cast for a disqualified candidate SHALL NOT
BE COUNTED. The law has also validated the jurisdiction of the Court or Commission on Election to
continue hearing the petition for disqualification in case a candidate is voted for and receives the
highest number of votes, if for any reason, he is not declared by final judgment before an election to be
disqualified. What happens then when after the elections are over, one is declared disqualified? Then,
votes cast for him "shall not be counted" and in legal contemplation, he no longer received the highest
number of votes. It stands to reason that Section 6 of RA 6646 does not make the second placer the
winner simply because a "winning candidate is disqualified," but that the law considers him as the
candidate who had obtained the highest number of votes as a result of the votes cast for the
disqualified candidate not being counted or considered. As this law clearly reflects the legislative policy
on the matter, then there is no reason why this Court should not re-examine and consequently abandon
the doctrine in the Jun Labo case. It has been stated that "the qualifications prescribed for elective office
cannot be erased by the electorate alone. The will of the people as expressed through the ballot cannot
cure the vice of ineligibility" most especially when it is mandated by no less than the Constitution.
ACCORDINGLY, I vote to DISMISS the petition and to order the Provincial Board of Canvassers of Leyte to
proclaim the candidate receiving the highest number of votes, from among the qualified candidates, as
the duly elected representative of the 1st district of Leyte.

REGALADO, J ., dissenting opinion:

1. CIVIL LAW; DOMICILE; DOMICILE OF ORIGIN; CONSTRUED. — The domicile of the parents at the
time of birth, or what is termed the "domicile of origin," constitutes the domicile of an infant until
abandoned, or until the acquisition of a new domicile in a different place.

2. ID.; ID.; KINDS. — Domicile is said to be of three kinds, that is, domicile by birth, domicile by
choice, and domicile by operation of law. The first is the common case of the place of birth or
domicilium originis; the second is that which is voluntarily acquired by a party or domicilium proprio
motu; the last which is consequential, as that of a wife arising from marriage, is sometimes called
domicilium necesarium.

3. ID.; ID.; DOMICILE BY OPERATION OF LAW; ACQUIRED BY MARRIAGE. — When petitioner


contracted marriage in 1954 with then Rep. Marcos, by operation of law, not only international or

438
American but of our own enactment, she acquired her husband's domicile of origin in Batac, Ilocos
Norte and correspondingly lost her own domicile of origin in Tacloban City.

4. ID.; ID.; REQUISITES FOR CHANGE OF DOMICILE. — To successfully effect a change of domicile,
one must demonstrate (a) an actual removal or an actual change of domicile, (b) a bona fide intention of
abandoning the former place of residence and establishing a new one, and (c) acts which correspond
with the purpose.

5. ID.; ID.; ONCE LOST CAN BE RECOVERED IN ACCORDANCE WITH LAW; NO AUTOMATIC
REVERSION OR REACQUISITION OF DOMICILE. — Domicile once lost in accordance with law can only be
recovered likewise in accordance with law. However, we are here being titillated with the possibility of
an automatic reversion to or reacquisition of a domicile of origin after the termination of the cause for
its loss by operation of law. The majority agrees that since petitioner lost her domicile of origin by her
marriage, the termination of the marriage also terminates that effect thereof. I am impressed by the
ingeniousness of this theory which proves that, indeed, necessity is the mother of inventions.
Regretfully, I find some difficulty in accepting either the logic or the validity of this argument.

6. ID.; ID.; VOLUNTARY ABANDONMENT OF DOMICILE DOES NOT AUTOMATICALLY RESTORE


DOMICILE OF ORIGIN. — If a party loses his domicile of origin by obtaining a new domicile of choice, he
thereby voluntarily abandons the former in favor of the latter. If, thereafter, he abandons that chosen
domicile, he does not per se recover his original domicile unless, by subsequent acts legally indicative
thereof, he evinces his intent and desire to establish the same as his new domicile, which is precisely
what petitioner belatedly and, evidently just for purposes of her candidacy, unsuccessfully tried to do.
One's subsequent abandonment of his domicile of choice cannot automatically restore his domicile of
origin, not only because there is no legal authority therefor but because it would be absurd. Pursued to
its logical consequence, that theory of ipso jure reversion would rule out the fact that said party could
already very well have obtained another domicile, either of choice or by operation of law, other than his
domicile of origin. Significantly and obviously for this reason, the Family Code, which the majority
inexplicably invokes, advisedly does not regulate this contingency since it would impinge on one's
freedom of choice.

7. ID.; ID.; ID.; CASE AT BAR. — In the instant case, petitioner not only voluntarily abandoned her
domicile of choice (unless we assume that she entered into the marital state against her will) but, on top
of that, such abandonment was further affirmed through her acquisition of a new domicile by operation
of law. In fact, this is even a case of both voluntary and legal abandonment of a domicile of origin. With
much more reason, therefore, should we reject the proposition that with the termination of her
marriage in 1989, petitioner had supposedly per se and ipso facto reacquired her domicile of origin
which she lost in 1954. Otherwise, this would be tantamount to saying that during the period of marital
coverture, she was simultaneously in possession and enjoyment of a domicile of origin which was only in
a state of suspended animation. LexLibris

8. ID.; ID.; DOMICILE BY OPERATION OF LAW; AFTER THE HUSBAND'S DEATH, THE WIFE HAS THE
RIGHT TO ELECT HER OWN DOMICILE. — The American rule is likewise to the effect that while after the

439
husband's death the wife has the right to elect her own domicile, she nevertheless retains the last
domicile of her deceased husband until she makes an actual change. In the absence of affirmative
evidence, to the contrary, the presumption is that a wife's domicile or legal residence follows that of her
husband and will continue after his death.

9. ID.; FAMILY CODE; RIGHT AND POWER TO FIX FAMILY HOME CAN NOT AFFECT DOMICILE FIXED
BY LAW. — I cannot appreciate the premises advanced in support of the majority's theory based on
Articles 68 and 69 of the Family Code. All that is of any relevance therein is that under this new code,
the right and power to fix the family domicile is now shared by the spouses. I cannot perceive how that
joint right, which in the first place was never exercised by the spouses, could affect the domicile fixed by
the law for petitioner in 1954 and, for her husband, long prior thereto. It is true that a wife now has the
coordinate power to determine the conjugal or family domicile, but that has no bearing on this case.
With the death of her husband, and each of her children having gotten married and established their
own respective domiciles, the exercise of that joint power was and is no longer called for or material in
the present factual setting of this controversy. Instead, what is of concern in petitioner's case was the
matter of her having acquired or not her own domicile of choice.

10. POLITICAL LAW; ELECTIONS; ONE YEAR RESIDENCY REQUIREMENT; NOT MET BY CANDIDATE'S
RESIDENCY FOR SEVEN (7) MONTHS IMMEDIATELY PRECEDING ELECTION; PREVIOUS RESIDENCY AT
DOMICILE OF ORIGIN NOT COUNTED WHERE THE SAME WAS LOST DUE TO MARRIAGE AND NOT
REACQUIRED AFTER HUSBAND'S DEATH. — In sum, petitioner having lost Tacloban City as her domicile
of origin since 1954 and not having automatically reacquired any domicile therein, she cannot legally
claim that her residency in the political constituency of which it is a part continued since her birth up to
the present. Respondent commission was, therefore, correct in rejecting her pretension to that effect in
her amended/corrected certificate of candidacy, and in holding her to her admission in the original
certificate that she had actually resided in that constituency for only seven months prior to the election.

DAVIDE, JR., J ., dissenting opinion:

1. REMEDIAL LAW; SPECIAL CIVIL ACTIONS; CERTIORARI; PROPER REMEDY FROM A DECISION,
ORDER OR RULING OF THE COMELEC. — Under Section 7, Subdivision A, Article IX of the Constitution,
decisions, orders, or rulings of the COMELEC may be brought to this Court only by the special civil action
for certiorari under Rule 65 of the Rules of Court (Aratuc vs. COMELEC, 88 SCRA 251 [1979]; Dario vs.
Mison, 176 SCRA 84 [1989]).

2. ID.; ID.; ID.; ID.; WRIT OF CERTIORARI; MAY BE GRANTED ONLY IN ABSENCE OR EXCESS OF
JURISDICTION OR WITH GRAVE ABUSE OF DISCRETION. — A writ of certiorari may be granted only if the
COMELEC has acted without or in excess of jurisdiction or with grave abuse of discretion (Section 1, Rule
65, Rules of Court).

3. ID.; ID.; ID.; ID.; ID.; ID.; CASE AT BENCH, NOT A CASE FOR ISSUANCE OF WRIT. — Since the
COMELEC has, undoubtedly, jurisdiction over the private respondent's petition, the only issue left is
whether it acted with grave abuse of discretion in disqualifying the petitioner. My careful and
meticulous perusal of the challenged resolution of 24 April 1995 of the COMELEC Second Division and

440
the En Banc resolution of 7 May 1995 discloses total absence of abuse of discretion, much less grave
abuse thereof. The resolution of the Second Division dispassionately and objectively discussed in minute
details the facts which established beyond cavil that herein petitioner was disqualified as a candidate on
the ground of lack of residence in the First Congressional District of Leyte. It has not misapplied,
miscomprehended, or misunderstood facts or circumstances of substance pertinent to the issue of her
residence.

4. POLITICAL LAW; ELECTIONS; DOMICILE; LOSS OR ABANDONMENT THEREOF IN CASE AT BAR. — I


respectfully submit that the petitioner herself has provided the COMELEC, either by admission or by
documentary evidence, overwhelming proof of the loss or abandonment of her domicile of origin, which
is Tacloban City and not Tolosa, Leyte. Assuming that she decided to live again in her domicile of origin,
that became her second domicile of choice, where her stay, unfortunately, was for only seven months
before the day of the election. She was then disqualified to be a candidate for the position of
Representative of the First Congressional District of Leyte. A holding to the contrary would be arbitrary.

5. ID.; ID.; ID.; DOMICILE OF CHOICE LOST BY OPERATION OF LAW BY MARRIAGE. — It may indeed
be conceded that the petitioner's domicile of choice was either Tacloban City or Tolosa, Leyte.
Nevertheless, she lost it by operation of law sometime in May 1954 upon her marriage to the then
Congressman (later, President) Ferdinand E. Marcos. A domicile by operation of law is that domicile
which the law attributes to a person, independently of his own intention or actual residence, as results
from legal domestic relations as that of the wife arising from marriage (28 C.J.S. Domicile 7, 11). Under
the governing law then, Article 110 of the Civil Code, her new domicile or her domicile of choice was the
domicile of her husband, which was Batac, Ilocos Norte. Under common law, a woman upon her
marriage loses her own domicile and, by operation of law, acquires that of her husband, no matter
where the wife actually lives or what she believes or intends. Her domicile is fixed in the sense that it is
declared to be the same as his, and subject to certain limitations, he can change her domicile by
changing his own (25 Am Jur 2d Domicile 48, 37).

6. CIVIL LAW; FAMILY CODE; FAMILY DOMICILE; FIXING THEREOF, A JOINT DECISION OF SPOUSES.
— It must, however, be pointed out that under Article 69 of the Family Code, the fixing of the family
domicile is no longer the sole prerogative of the husband, but is now a joint decision of the spouses, and
in case of disagreement the court shall decide. The said article uses the term "family domicile," and not
family residence, as "the spouses may have multiple residences, and the wife may elect to remain in one
of such residences, which may destroy the duty of the spouses to live together and its corresponding
benefits" (ALICIA V. SEMPIO-DIY, Handbook on the Family Code of the Philippines, [1988], 102).

7. ID.; DOMICILE, DOMICILE BY OPERATION OF LAW; DEATH OF HUSBAND REVIVES POWER OF


WIFE TO ACQUIRE HER OWN DOMICILE; NO AUTOMATIC RESTORATION OF WOMAN'S DOMICILE OF
ORIGIN. — The theory of automatic restoration of a woman's domicile of origin upon the death of her
husband, which the majority opinion adopts to overcome the legal effect of the petitioner's marriage on
her domicile, is unsupported by law and by jurisprudence. The settled doctrine is that after the
husband's death the wife has a right to elect her own domicile, but she retains the last domicile of her
husband until she makes an actual change (28 C.J.S. Domicile 12, 27). Or, on the death of the husband,

441
the power of the wife to acquire her own domicile is revived, but until she exercises the power her
domicile remains that of the husband at the time of his death (25 Am Jur 2d Domicile 62, 45). Note that
what is revived is not her domicile of origin but her power to acquire her own domicile.

8. ID.; ID.; LOSS OF DOMICILE; MARRIAGE, NOT A GROUND. — I find to be misplaced the reliance
by the majority opinion on Faypon vs. Quirino (96 Phil. 294 [1954]), and the subsequent cases which
established the principle that absence from original residence or domicile of origin to pursue studies,
practice one's profession, or engage in business in other states does not constitute loss of such
residence or domicile. So is the reliance on Section 117 of the Omnibus Election Code which provides
that transfer of residence to any other place by reason of one's "occupation; profession; employment in
private and public service; educational activities; work in military or naval reservations; service in the
army, navy or air force, the constabulary or national police force; or confinement or detention in
government institutions in accordance with law" is not deemed as loss of original residence. Those cases
and legal provision do not include marriage of a woman. The reason for the exclusion is, of course,
Article 110 of the Civil Code. If it were the intention of this Court or of the legislature to consider the
marriage of a woman as a circumstance which would not operate as an abandonment of domicile (of
origin or of choice), then such cases and legal provision should have expressly mentioned the same.

9. ID.; ID.; ABANDONMENT THEREOF IN CASE AT BENCH. — This Court should not accept as gospel
truth the self-serving claim of the petitioner in her affidavit that her "domicile or residence of origin is
Tacloban City," and that she "never intended to abandon this domicile or residence of origin to which
[she] always intended to return whenever absent." Such a claim of intention cannot prevail over the
effect of Article 110 of the Civil Code. Besides, the facts and circumstances or the vicissitudes of the
petitioner's life after her marriage in 1954 conclusively establish that she had indeed abandoned her
domicile of origin and had acquired a new one animo et facto (KOSSUTH KENT KENNAN, A Treatise on
Residence and Domicile, [1934], 214, 326). aisadc

10. REMEDIAL LAW; EVIDENCE; WEIGHT AND SUFFICIENCY; SELF-SERVING STATEMENT, WITHOUT
PROBATIVE VALUE. — Neither should this Court place complete trust on the petitioner's claim that she
"merely committed an honest mistake" in writing down the word "seven" in the space provided for the
residency qualification requirement in the certificate of candidacy. Such a claim is self-serving and, in the
light of the foregoing disquisitions, would be all sound and fury signifying nothing. To me, she did not
commit any mistake, honest or otherwise; what she stated was the truth.

11. ID.; ID.; BURDEN OF PROOF; ONE WHO ASSERTS A FACT HAS THE BURDEN OF PROVING IT. —
The majority opinion also disregards a basic rule in evidence that he who asserts a fact or the affirmative
of an issue has the burden of proving it (Imperial Victory Shipping Agency vs. NLRC, 200 SCRA 178
[1991]; P.T . Cerna Corp. vs. Court of Appeals, 221 SCRA 19 [1993]). Having admitted marriage to the
then Congressman Marcos, the petitioner could not deny the legal consequence thereof on the change
of her domicile to that of her husband. The majority opinion rules or at least concludes that "[b]y
operation of law (domicilium necesarium), her legal domicile at the time of her marriage automatically
became Batac, Ilocos Norte." That conclusion is consistent with Article 110 of the Civil Code. Since she is
presumed to retain her deceased husband's domicile until she exercises her revived power to acquire

442
her own domicile, the burden is upon her to prove that she has exercised her right to acquire her own
domicile. She miserably failed to discharge that burden. cdlex

ROMERO, J ., separate opinion:

POLITICAL LAW; ELECTIONS; RESIDENCE; DOMICILE BY OPERATION OF LAW; WIDOW NO LONGER


BOUND BY THE DOMICILE OF THE DEPARTED HUSBAND; WIDOW MAY CHOOSE DOMICILE; ONE YEAR
RESIDENCE REQUIREMENT, MET IN CASE AT BAR. — Bearing in mind that the term "resident" has been
held to be synonymous with "domicile" for election purposes, it is important to determine whether
petitioner's domicile was in the First District of Leyte and if so, whether she had resided there for at
least a period of one year. Undisputed is her domicile of origin, Tacloban, where her parents lived at the
time of her birth. Depending on what theory one adopts, the same may have been changed when she
married Ferdinand E. Marcos, then domiciled in Batac, by operation of law. Assuming it did, his death
certainly released her from the obligation to live with him at the residence fixed by him during his
lifetime. What may confuse the layman at this point is the fact that the term "domicile" may refer to
"domicile of origin," "domicile of choice," or "domicile by operation of law," which subject we shall not
belabor since it has been amply discussed by the ponente and in the other separate opinions. A widow,
like the petitioner and others similarly situated, can no longer be bound by the domicile of the departed
husband, if at all she was before. Neither does she automatically revert to her domicile of origin, but
exercising free will, she may opt to reestablish her domicile of origin. In returning to Tacloban and
subsequently, to Barangay Olot, Tolosa, both of which are located in the First District of Leyte, petitioner
amply demonstrated by overt acts, her election of a domicile of choice, in this case, a reversion to her
domicile of origin. Added together, the time when she set up her domicile in the two places sufficed to
meet the one-year requirement to run as Representative of the First District of Leyte.

VITUG, J., separate opinion:

1. CONSTITUTIONAL LAW; CONSTITUTIONAL PROVISIONS, GENERALLY MANDATORY IN


CHARACTER. — Constitutional provisions must be taken to be mandatory in character unless, either by
express statement or by necessary implication, a different intention is manifest (see Marcelino vs. Cruz,
121 SCRA 51).

2. ID.; COMELEC; WITH JURISDICTION OVER PRE-PROCLAMATION CONTROVERSIES. — The


Commission on Election (the "COMELEC") is constitutionally bound to enforce and administer "all laws
and regulations relative to the conduct of election . . ." (Art. IX, C, Sec. 2, Constitution) that, there being
nothing said to the contrary, should include its authority to pass upon the qualification and
disqualification prescribed by law of candidates to an elective office. Indeed, pre-proclamation
controversies are expressly placed under the COMELEC's jurisdiction to hear and resolve (Art. IX, C, Sec.
3, Constitution).

3. REMEDIAL LAW; EVIDENCE; FINDINGS OF FACT ON THE OBSERVANCE OF ONE-YEAR RESIDENCY


REQUIREMENT OF THE COMELEC, GENERALLY UPHELD ON APPEAL. — The matter before us specifically
calls for the observance of the constitutional one-year residency requirement. This issue (whether or not
there is here such compliance), to my mind, is basically a question of fact or at least inextricably linked

443
to such determination. The findings and judgment of the COMELEC, in accordance with the long
established rule and subject only to a number of exceptions under the basic heading of "grave abuse of
discretion," are not reviewable by this Court. Using the above tests, I am not convinced that we can
charge the COMELEC with having committed grave abuse of discretion in its assailed resolution.

4. CIVIL LAW; DOMICILE; PLACE OF HABITUAL RESIDENCE. — For civil law purposes, i.e., as regards
the exercise of civil rights and the fulfillment of civil obligations, the domicile of a natural person is the
place of his habitual residence (see Article 50, Civil Code).

5. POLITICAL LAW; ELECTIONS; DOMICILE SYNONYMOUS WITH RESIDENCE. — In election cases,


the Court treats domicile and residence as synonymous terms, thus: '(t)he term 'residence' as used in
the election law is synonymous with 'domicile,' which imports not only an intention to reside in a fixed
place but also personal presence in that place, coupled with conduct indicative of such intention.'
'Domicile' denotes a fixed permanent residence to which when absent for business or pleasure, or for
like reasons, one intends to return. . . . (Romualdez vs. Regional Trial Court, Branch 7, Tacloban City [226
SCRA 408, 409])

6. ID.; ID.; ID.; ELEMENTS FOR CHANGE OF DOMICILE. — Residence thus acquired, however, may
be lost by adopting another choice of domicile. In order, in turn, to acquire a new domicile by choice,
there must concur (1) residence or bodily presence in the new locality, (2) an intention to remain there,
and (3) an intention to abandon the old domicile. In other words, there must basically be animus
manendi coupled with animus non revertendi. The purpose to remain in or at the domicile of choice
must be for an indefinite period of time; the change of residence must be voluntary; and the residence
at the place chosen for the new domicile must be actual.

7. CONSTITUTIONAL LAW; COMELEC; ITS JURISDICTION ENDS WHEN JURISDICTION OF ELECTORAL


TRIBUNAL BEGINS. — The COMELEC's jurisdiction, in the case of congressional elections, ends when the
jurisdiction of the Electoral Tribunal concerned begins. It signifies that the protestee must have
theretofore been duly proclaimed and has since become a "member" of the Senate or the House of
Representatives. LexLibris

8. ID.; ID.; PROCLAMATION OF CANDIDATE, NOT A MINISTERIAL FUNCTION. — The question can be
asked on whether or not the proclamation of a candidate is just a ministerial function of the Commission
on Elections dictated solely on the number of votes cast in an election exercise. I believe, it is not. A
ministerial duty is an obligation the performance of which, being adequately defined, does not allow the
use of further judgment or discretion. The COMELEC, in its particular case, is tasked with the full
responsibility of ascertaining all the facts and conditions such as may be required by law before a
proclamation is properly done.

9. POLITICAL LAW; ELECTIONS; CANDIDATE WHO OBTAINED THE SECOND HIGHEST NUMBER OF
VOTES NOT NECESSARILY ENTITLED TO BE DECLARED WINNER OF ELECTIVE OFFICE WHERE CANDIDATE
WHO OBTAINED THE HIGHEST NUMBER OF VOTES IS LATER DECLARED DISQUALIFIED OR NOT ELIGIBLE.
— I should like to next touch base on the applicability to this case of Section 6 of Republic Act No. 6646,
in relation to Section 72 of Batas Pambansa Blg. 881. I realize that in considering the significance of the

444
law, it may be preferable to look for not so much the specific instances they ostensibly would cover as
the principle they clearly convey. Thus, I will not scoff at the argument that it should be sound to say
that votes cast in favor of the disqualified candidate, whenever ultimately declared as such, should not
be counted in his or her favor and must accordingly be considered to be stray votes. The argument,
nevertheless, is far outweighed by the rationale of the now prevailing doctrine first enunciated in the
case of Topacio vs. Paredes (23 Phil. 238 [1912]) which, although later abandoned in Ticzon vs. Comelec
(103 SCRA 687 [1981]), and Santos vs. COMELEC (137 SCRA 740 [1985]), was restored, along with the
interim case of Geronimo vs. Ramos (136 SCRA 435 [1985]), by the Labo (176 SCRA 1 (1989]), Abella (201
SCRA 253 [1991]), Labo (211 SCRA 297 [1992]) and, most recently, Benito (235 SCRA 436 [1994]) rulings.
Benito vs. Comelec was a unanimous decision penned by Justice Kapunan and concurred in by Chief
Justice Narvasa, Justices Feliciano, Padilla, Bidin, Regalado, Davide, Romero, Melo, Quiason, Puno, Vitug
and Mendoza (Justices Cruz and Bellosillo were on official leave).

MENDOZA, J., separate opinion:

1. CONSTITUTIONAL LAW; COMELEC; WITHOUT POWER TO DISQUALIFY CANDIDATE FOR LACK OF


ELIGIBILITY. — In my view the issue in this case is whether the Commission on Elections has the power
to disqualify candidates on the ground that they lack eligibility for the office to which they seek to be
elected. I think that it has none and that the qualifications of candidates may be questioned only in the
event they are elected, by filing a petition for quo warranto or an election protest in the appropriate
forum, not necessarily in the COMELEC but, as in this case, in the House of Representatives Electoral
Tribunal. That the parties in this case took part in the proceedings in the COMELEC is of no moment.
Such proceedings were unauthorized and were not rendered valid by their agreement to submit their
dispute to that body. To be sure, there are provisions denominated for "disqualification," but they are
not concerned with a declaration of the ineligibility of a candidate. These provisions are concerned with
the incapacity (due to insanity, incompetence or conviction of an offense) of a person either to be a
candidate or to continue as a candidate for public office. There is also a provision for the denial or
cancellation of certificates of candidacy, but it applies only to cases involving false representations as to
certain matters required by law to be stated in the certificates. cdll

2. POLITICAL LAW; ELECTION LAWS; ABSENCE OF PROVISION FOR PRE-PROCLAMATION CONTEST


BASED ON INELIGIBILITY. — The various election laws will be searched in vain for authorized
proceedings for determining a candidate's qualifications for an office before his election. There are none
in the Omnibus Election Code (B.P. Blg. 881), in the Electoral Reforms Law of 1987 (R.A. No. 6646), or in
the law providing for synchronized elections (R.A. No. 7166). There are, in other words, no provisions for
pre-proclamation contests but only election protests or quo warranto proceedings against winning
candidates.

3. ID.; ID.; ID.; REASONS. — Three reasons may be cited to explain the absence of an authorized
proceeding for determining before election the qualifications of a candidate. First is the fact that unless
a candidate wins and is proclaimed elected, there is no necessity for determining his eligibility for the
office. Second is the fact that the determination of a candidate's eligibility, e.g., his citizenship or, as in
this case, his domicile, may take a long time to make, extending beyond the beginning of the term of the

445
office. Third is the policy underlying the prohibition against pre-proclamation cases in elections for
President, Vice President, Senators and members of the House of Representatives. (R.A. No. 7166, § 15)
The purpose is to preserve the prerogatives of the House of Representatives Electoral Tribunal and the
other Tribunals as "sole judges" under the Constitution of the election, returns and qualifications of
members of Congress or of the President and Vice President, as the case may be. By providing in § 253
for the remedy of quo warranto for determining an elected official's qualifications after the results of
elections are proclaimed, while being conspicuously silent about a pre-proclamation remedy based on
the same ground, the Omnibus Election Code, or OEC, by its silence underscores the policy of not
authorizing any inquiry into the qualifications of candidates unless they have been elected.

4. ID.; ID.; PETITION TO DISQUALIFY CANDIDATE FOR INELIGIBILITY FALLS WITHIN THE
JURISDICTION OF ELECTORAL TRIBUNAL. — Montejo's petition before the COMELEC was not a petition
for cancellation of certificate of candidacy under § 78 of the Omnibus Election Code, but essentially a
petition to declare private respondent ineligible. It is important to note this, because, as will presently
be explained, proceedings under § 78 have for their purpose to disqualify a person from being a
candidate, whereas quo warranto proceedings have for their purpose to disqualify a person from
holding public office. Jurisdiction over quo warranto proceedings involving members of the House of
Representatives is vested in the Electoral Tribunal of that body.

5. REMEDIAL LAW; SUPREME COURT; QUO WARRANTO; QUALIFICATION OF CANDIDATE PASSED


UPON ONLY AFTER PROCLAMATION OF CANDIDATE. — In the only cases in which this Court dealt with
petitions for the cancellation of certificates of candidacy, the allegations were that the respondent
candidates had made false representations in their certificates of candidacy with regard to their
citizenship, age, or residence. But in the generality of cases in which this Court passed upon the
qualifications of respondents for office, this Court did so in the context of election protests or quo
warranto proceedings filed after the proclamation of the respondents or protestees as winners.

6. POLITICAL LAW; ELECTIONS; ABSENCE OF PROVISION FOR PRE-PROCLAMATION CONTESTS


BASED ON INELIGIBILITY; CANNOT BE SUPPLIED BY A MERE RULE OF THE COMELEC. — The lack of
provision for declaring the ineligibility of candidates, however, cannot be supplied by a mere rule. Such
an act is equivalent to the creation of a cause of action which is a substantive matter which the
COMELEC, in the exercise of its rulemaking power under Art. IX, A, § 6 of the Constitution, cannot do. It
is noteworthy that the Constitution withholds from the COMELEC even the power to decide cases
involving the right to vote, which essentially involves an inquiry into qualifications based on age,
residence and citizenship of voters. (Art. IX-C, § 2[3])

7. ID.; ID.; DISQUALIFICATION PROCEEDINGS DIFFERENTIATED FROM DECLARATION OF


INELIGIBILITY. — The assimilation in Rule 25 of the COMELEC rules of grounds for ineligibility into
grounds for disqualification is contrary to the evident intention of the law. For not only in their grounds
but also in their consequences are proceedings for "disqualification" different from those for a
declaration of "ineligibility." "Disqualification" proceedings, as already stated, are based on grounds
specified in Sections 12 and 68 of the Omnibus Election Code and in § 40 of the Local Government Code
and are for the purpose of barring an individual from becoming a candidate or from continuing as a

446
candidate for public office. In a word, their purpose is to eliminate a candidate from the race either from
the start or during its progress. "Ineligibility," on the other hand, refers to the lack of the qualifications
prescribed in the Constitution or the statutes for holding public office and the purpose of the
proceedings for declaration of ineligibility is to remove the incumbent from office.

8. ID.; ID.; POSSESSION OF QUALIFICATIONS FOR PUBLIC OFFICE DOES NOT IMPLY THAT
CANDIDATE IS NOT DISQUALIFIED. — That an individual possesses the qualifications for a public office
does not imply that he is not disqualified from becoming a candidate or continuing as a candidate for a
public office and vice versa. We have this sort of dichotomy in our Naturalization Law. (C.A. No. 473)
That an alien has the qualifications prescribed in § 2 of the law does not imply that he does not suffer
from any of disqualifications provided in § 4.

9. ID.; ID.; DISQUALIFICATION PROCEEDINGS BASED ON INELIGIBILITY; ELECTION PROTEST OR


ACTION FOR QUO WARRANTO, PROPER REMEDY. — To summarize, the declaration of ineligibility of a
candidate may only be sought in an election protest or action for quo warranto filed pursuant to § 253
of the Omnibus Election Code within 10 days after his proclamation. With respect to elective local
officials (e.g., Governor, Vice Governor, members of the Sangguniang Panlalawigan, etc.) such petition
must be filed either with the COMELEC, the Regional Trial Courts, or Municipal Trial Courts, as provided
in Art. IX-C, § 2(2) of the Constitution. In the case of the President and Vice President, the petition must
be filed with the Presidential Electoral Tribunal (Art. VII, § 4, last paragraph), and in the case of the
Senators, with the Senate Electoral Tribunal, and in the case of Congressmen, with the House of
Representatives Electoral Tribunal. (Art. VI, § 17) There is greater reason for not allowing before the
election the filing of disqualification proceedings based on alleged ineligibility in the case of candidates
for President, Vice President, Senators and members of the House of Representatives, because of the
same policy prohibiting the filing of pre-proclamation cases against such candidates.

10. REMEDIAL LAW; SPECIAL CIVIL ACTIONS; CERTIORARI; COMELEC WITHOUT JURISDICTION TO
ASSUME DISQUALIFICATION OF CANDIDATE BASED ON INELIGIBILITY. — For these reasons, I am of the
opinion that the COMELEC had no jurisdiction over SPA No. 95-009; that its proceedings in that case,
including its questioned orders, are void; and that the eligibility of petitioner Imelda Romualdez-Marcos
for the office of Representative of the First District of Leyte may only be inquired into by the HRET.
Accordingly, I vote to grant the petition and to annul the proceedings of the Commission on Elections in
SPA No. 95-009, including its questioned orders dated April 24, 1995, May 7, 1995, May 11, 1995 and
May 25, 1995, declaring petitioner Imelda Romualdez-Marcos ineligible and ordering her proclamation
as Representative of the First District of Leyte suspended. To the extent that Rule 25 of the COMELEC
Rules of Procedure authorizes proceedings for the disqualification of candidates on the ground of
ineligibility for the office, it should be considered void. LLjur

DECISION

KAPUNAN, J p:

A constitutional provision should be construed as to give it effective operation and suppress the
mischief at which it is aimed. 1 The 1987 Constitution mandates that an aspirant for election to the

447
House of Representatives be "a registered voter in the district in which he shall be elected, and a
resident thereof for a period of not less than one year immediately preceding the election." 2 The
mischief which this provision — reproduced verbatim from the 1973 Constitution — seeks to prevent is
the possibility of a "stranger or newcomer unacquainted with the conditions and needs of a community
and not identified with the latter, from an elective office to serve that community." 3

Petitioner, Imelda Romualdez-Marcos filed her Certificate of Candidacy for the position of
Representative of the First District of Leyte with the Provincial Election Supervisor on March 8, 1995,
providing the following information in item no. 8: 4

RESIDENCE IN THE CONSTITUENCY WHERE I SEEK TO BE ELECTED IMMEDIATELY PRECEDING THE


ELECTION: ______ Years and seven Months.

On March 23, 1995, private respondent Cirilo Roy Montejo, the incumbent Representative of the First
District of Leyte and a candidate for the same position, filed a "Petition for Cancellation and
Disqualification" 5 with the Commission on Elections alleging that petitioner did not meet the
constitutional requirement for residency. In his petition, private respondent contended that Mrs.
Marcos lacked the Constitution's one year residency requirement for candidates to the House of
representatives on the evidence of declarations made by her in Voter Registration Record 94-No.
3349772 6 and in her Certificate of Candidacy. He prayed that "an order be issued declaring (petitioner)
disqualified and canceling the certificate of candidacy." 7

On March 29, 1995, petitioner filed an Amended/Corrected Certificate of Candidacy, changing the entry
"seven" months to "since childhood" in item no. 8 of the amended certificate. 8 On the same day, the
Provincial Election Supervisor of Leyte informed petitioner that:

[T]his office cannot receive or accept the aforementioned Certificate of Candidacy on the ground that it
is filed out of time, the deadline for the filing of the same having already lapsed on March 20, 1995. The
Corrected/Amended Certificate of Candidacy should have been filed on or before the March 20, 1995
deadline. 9

Consequently, petitioner filed the Amended/Corrected Certificate of Candidacy with the COMELEC's
Head Office in Intramuros, Manila on March 31, 1995. Her Answer to private respondent's petition in
SPA No. 95-009 was likewise filed with the head office on the same day. In said Answer, petitioner
averred that the entry of the word "seven" in her original Certificate of Candidacy was the result of an
"honest misinterpretation" 10 which she sought to rectify by adding the words "since childhood" in her
Amended/Corrected Certificate of Candidacy and that "she has always maintained Tacloban City as her
domicile or residence." 11 Impugning respondent's motive in filing the petition seeking her
disqualification, she noted that:

When respondent (petitioner herein,) announced that she was intending to register as a voter in
Tacloban City and run for Congress in the First District of Leyte, petitioner immediately opposed her
intended registration by writing a letter stating that "she is not a resident of said city but of Barangay
Olot, Tolosa, Leyte. After respondent had registered as a voter in Tolosa following completion of her six

448
month actual residence therein, petitioner filed a petition with the COMELEC to transfer the town of
Tolosa from the First District to the Second District and pursued such a move up to the Supreme Court,
his purpose being to remove respondent as petitioner's opponent in the congressional election in the
First District. He also filed a bill, along with other Leyte Congressmen, seeking the creation of another
legislative district to remove the town of Tolosa out of the First District, to achieve his purpose.
However, such bill did not pass the Senate. Having failed on such moves, petitioner now filed the instant
petition for the same objective, as it is obvious that he is afraid to submit along with respondent for the
judgment and verdict of the electorate of the First District of Leyte in an honest, orderly, peaceful, free
and clean elections on May 8, 1995. 12

On April 24, 1995, the Second Division of the Commission on Elections (COMELEC), by a vote of 2 to 1,
13 came up with a Resolution 1) finding private respondent's Petition for Disqualification in SPA 95-009
meritorious; 2) striking off petitioner's Corrected/Amended Certificate of Candidacy of March 31, 1995;
and 3) canceling her original Certificate Candidacy. 14 Dealing with two primary issues, namely, the
validity of amending the original Certificate of Candidacy after the lapse of the deadline for filing
certificates of candidacy, and petitioner's compliance with the one year residency requirement, the
Second Division held:

"Respondent raised the affirmative defense in her Answer that the printed word "Seven" (months) was a
result of an "honest misinterpretation or honest mistake" on her part and, therefore, an amendment
should subsequently be allowed. She averred that she thought that what was asked was her "actual and
physical" presence in Tolosa and not residence of origin or domicile in the First Legislative District, to
which she could have responded "since childhood." In an accompanying affidavit, she stated that her
domicile is Tacloban City, a component of the First District, to which she always intended to return
whenever absent and which she has never abandoned. Furthermore, in her memorandum, she tried to
discredit petitioner's theory of disqualification by alleging that she has been a resident of the First
Legislative District of Leyte since childhood, although she only became a resident of the Municipality of
Tolosa for seven months. She asserts that she has always been a resident of Tacloban City, a component
of the First District, before coming to the Municipality of Tolosa.

Along this point, it is interesting to note that prior to her registration in Tolosa, respondent announced
that she would be registering in Tacloban City so that she can be a candidate for the District. However,
this intention was rebuffed when petitioner wrote the Election Officer of Tacloban not to allow
respondent since she is a resident of Tolosa and not Tacloban. She never disputed this claim and instead
implicitly acceded to it by registering in Tolosa.

This incident belies respondent's claim of 'honest misinterpretation or honest mistake.' Besides, the
Certificate of Candidacy only asks for RESIDENCE. Since on the basis of her Answer, she was quite aware
of 'residence of origin' which she interprets to be Tacloban City, it is curious why she did not cite
Tacloban City in her Certificate of Candidacy. Her explanation that she thought what was asked was her
actual and physical presence in Tolosa is not easy to believe because there is none in the question that
insinuates about Tolosa. In fact, item no. 8 in the Certificate of Candidacy speaks clearly of 'Residency in

449
the CONSTITUENCY where I seek to be elected immediately preceding the election.' Thus, the
explanation of respondent fails to be persuasive. prLL

From the foregoing, respondent's defense of an honest mistake or misinterpretation, therefore, is


devoid of merit.

To further buttress respondent's contention that an amendment may be made, she cited the case of
Alialy v. COMELEC (2 SCRA 957). The reliance of respondent on the case of Alialy is misplaced. The case
only applies to the 'inconsequential deviations which cannot affect the result of the election, or
deviations from provisions intended primarily to secure timely and orderly conduct of elections.' The
Supreme Court in that case considered the amendment only as a matter of form. But in the instant case,
the amendment cannot be considered as a matter of form or an inconsequential deviation. The change
in the number of years of residence in the place where respondent seeks to be elected is a substantial
matter which determines her qualification as a candidacy, specially those intended to suppress, accurate
material representation in the original certificate which adversely affects the filer. To admit the
amended certificate is to condone the evils brought by the shifting minds of manipulating candidate, to
the detriment of the integrity of the election.

Moreover, to allow respondent to change the seven (7) month period of her residency in order to
prolong it by claiming it was 'since childhood' is to allow an untruthfulness to be committed before this
Commission. The arithmetical accuracy of the 7 months residency the respondent indicated in her
certificate of candidacy can be gleaned from her entry in her Voter's Registration Record accomplished
on January 28, 1995 which reflects that she is a resident of Brgy. Olot, Tolosa, Leyte for 6 months at the
time of the said registration (Annex A, Petition). Said accuracy is further buttressed by her letter to the
election officer of San Juan, Metro Manila, dated August 24, 1994, requesting for the cancellation of her
registration in the Permanent List of Voters thereat so that she can be re-registered or transferred to
Brgy. Olot, Tolosa, Leyte. The dates of these three (3) different documents show the respondent's
consistent conviction that she has transferred her residence to Olot, Tolosa, Leyte from Metro Manila
only for such limited period of time, starting in the last week of August 1994 which on March 8, 1995 will
only sum up to 7 months. The Commission, therefore, cannot be persuaded to believe in the
respondent's contention that it was an error.

xxx xxx xxx

Based on these reasons the Amended/Corrected Certificate of Candidacy cannot be admitted by this
Commission.

xxx xxx xxx

Anent the second issue, and based on the foregoing discussion, it is clear that respondent has not
complied with the one year residency requirement of the Constitution.

In election cases, the term 'residence' has always been considered as synonymous with 'domicile' which
imports not only the intention to reside in a fixed place but also personal presence in that place, coupled

450
with conduct indicative of such intention. Domicile denotes a fixed permanent residence to which when
absent for business or pleasure, or for like reasons, one intends to return. (Perfecto Faypon vs. Eliseo
Quirino, 96 Phil 294; Romualdez vs. RTC-Tacloban, 226 SCRA 408). In respondent's case, when she
returned to the Philippines in 1991, the residence she chose was not Tacloban but San Juan, Metro
Manila. Thus, her animus revertendi is pointed to Metro Manila and not Tacloban.

This Division is aware that her claim that she has been a resident of the First District since childhood is
nothing more than to give her a color of qualification where she is otherwise constitutionally
disqualified. It cannot hold ground in the face of the facts admitted by the respondent in her affidavit.
Except for the time that she studied and worked for some years after graduation in Tacloban City, she
continuously lived in Manila. In 1959, after her husband was elected Senator, she lived and resided in
San Juan, Metro Manila where she was a registered voter. In 1965, she lived in San Miguel, Manila
where she was again a registered voter. In 1978, she served as member of the Batasang Pambansa as
the representative of the City of Manila and later on served as the Governor of Metro Manila. She could
not have served these positions if she had not been a resident of the City of Manila. Furthermore, when
she filed her certificate of candidacy for the office of the President in 1992, she claimed to be a resident
of San Juan, Metro Manila. As a matter of fact on August 24, 1994, respondent wrote a letter with the
election officer of San Juan, Metro Manila requesting for the cancellation of her registration in the
permanent list of voters that she may be re-registered or transferred to Barangay Olot, Tolosa, Leyte.
These facts manifest that she could not have been a resident of Tacloban City since childhood up to the
time she filed her certificate of candidacy because she became a resident of many places, including
Metro Manila. This debunks her claim that prior to her residence in Tolosa, Leyte, she was a resident of
the First Legislative District of Leyte since childhood.

In this case, respondent's conduct reveals her lack of intention to make Tacloban her domicile. She
registered as a voter in different places and on a several occasions declared that she was a resident of
Manila. Although she spent her school days in Tacloban, she is considered to have abandoned such
place when she chose to stay and reside in other different places. In the case of Romualdez vs. RTC (226
SCRA 408) the Court explained how one acquires a new domicile by choice. There must concur: (1)
residence or bodily presence in the new locality; (2) intention to remain there; and (3) intention to
abandon the old domicile. In other words there must basically be animus manendi with animus non
revertendi. When respondent chose to stay in Ilocos and later on in Manila, coupled with her intention
to stay there by registering as a voter there and expressly declaring that she is a resident of that place,
she is deemed to have abandoned Tacloban City, where she spent her childhood and school days, as her
place of domicile.

Pure intention to reside in that place is not sufficient, there must likewise be conduct indicative of such
intention. Respondent's statements to the effect that she has always intended to return to Tacloban,
without the accompanying conduct to prove that intention, is not conclusive of her choice of residence.
Respondent has not presented any evidence to show that her conduct, one year prior the election,
showed intention to reside in Tacloban. Worse, what was evident was that prior to her residence in
Tolosa, she had been a resident of Manila.

451
It is evident from these circumstances that she was not a resident of the First District of Leyte "since
childhood."

To further support the assertion that she could have not been a resident of the First District of Leyte for
more than one year, petitioner correctly pointed out that on January 28, 1995; respondent registered as
a voter at precinct No. 18-A of Olot, Tolosa, Leyte. In doing so, she placed in her Voter Registration
Record that she resided in the municipality of Tolosa for a period of six months. This may be
inconsequential as argued by the respondent since it refers only to her residence in Tolosa, Leyte. But
her failure to prove that she was a resident of the First District of Leyte prior to her residence in Tolosa
leaves nothing but a convincing proof that she had been a resident of the district for six months only."
15

In a Resolution promulgated a day before the May 8, 1995 elections, the COMELEC en banc denied
petitioner's Motion for Reconsideration 16 of the April 24, 1995 Resolution declaring her not qualified to
run for the position of Member of the House of Representatives for the First Legislative District of Leyte.
17 The Resolution tersely stated:

After deliberating on the Motion for Reconsideration, the Commission RESOLVED to DENY it, no new
substantial matters having been raised therein to warrant re-examination of the resolution granting the
petition for disqualification. 18

On May 11, 1995, the COMELEC issued a Resolution allowing petitioner's proclamation should the
results of the canvass show that she obtained the highest number of votes in the congressional elections
in the First District of Leyte. On the same day, however, the COMELEC reversed itself and issued a
second Resolution directing that the proclamation of petitioner be suspended in the event that she
obtains the highest number of votes. 19

In a Supplemental Petition dated 25 May 1995, petitioner averred that she was the overwhelming
winner of the elections for the congressional seat in the First District of Leyte held May 8, 1995 based on
the canvass completed by the Provincial Board of Canvassers on May 14, 1995. Petitioner alleged that
the canvass showed that she obtained a total of 70,471 votes compared to the 36,833 votes received by
Respondent Montejo. A copy of said Certificate of Canvass was annexed to the Supplemental Petition.
LLcd

On account of the Resolutions disqualifying petitioner from running for the congressional seat of the
First District of Leyte and the public respondent's Resolution suspending her proclamation, petitioner
comes to this court for relief.

Petitioner raises several issues in her Original and Supplemental Petitions. The principal issues may be
classified into two general areas:

I. The Issue of Petitioner's qualifications

452
Whether or not petitioner was a resident, for election purposes, of the First District of Leyte for a period
of one year at the time of the May 8, 1995 elections.

II. The Jurisdictional Issue

a) Prior to the elections

Whether or not the COMELEC properly exercised its jurisdiction in disqualifying petitioner outside the
period mandated by the Omnibus Election Code for disqualification cases under Article 78 of the said
Code.

b) After the Elections

Whether or not the House of Representatives Electoral Tribunal assumed exclusive jurisdiction over the
question of petitioner's qualifications after the May 8, 1995 elections.

I. Petitioner's qualification

A perusal of the Resolution of the COMELEC's Second Division reveals a startling confusion in the
application of settled concepts of "Domicile" and "Residence" in election law. While the COMELEC
seems to be in agreement with the general proposition that for the purposes of election law, residence
is synonymous with domicile, the Resolution reveals a tendency to substitute or mistake the concept of
domicile for actual residence, a conception not intended for the purpose of determining a candidate's
qualifications for election to the House of Representatives as required by the 1987 Constitution. As it
were, residence, for the purpose of meeting the qualification for an elective position, has a settled
meaning in our jurisdiction.

Article 50 of the Civil Code decrees that "[f]or the exercise of civil rights and the fulfillment of civil
obligations, the domicile of natural persons is their place of habitual residence." In Ong vs. Republic 20
this court took the concept of domicile to mean an individual's "permanent home", "a place to which,
whenever absent for business or for pleasure, one intends to return, and depends on facts and
circumstances in the sense that they disclose intent." 21 Based on the foregoing, domicile includes the
twin elements of "the fact of residing or physical presence in a fixed place" and animus manendi, or the
intention of returning there permanently.

Residence, in its ordinary conception, implies the factual relationship of an individual to a certain place.
It is the physical presence of a person in a given area, community or country. The essential distinction
between residence and domicile in law is that residence involves the intent to leave when the purpose
for which the resident has taken up his abode ends. One may seek a place for purposes such as pleasure,
business, or health. If a person's intent be to remain, it becomes his domicile; if his intent is to leave as
soon as his purpose is established it is residence. 22 It is thus, quite perfectly normal for an individual to
have different residences in various places. However, a person can only have a single domicile, unless,
for various reasons, he successfully abandons his domicile in favor of another domicile of choice. In
Uytengsu vs. Republic, 23 we laid this distinction quite clearly:

453
"There is a difference between domicile and residence. 'Residence' is used to indicate a place of abode,
whether permanent or temporary; 'domicile' denotes a fixed permanent residence to which, when
absent, one has the intention of returning. A man may have a residence in one place and a domicile in
another. Residence is not domicile, but domicile is residence coupled with the intention to remain for an
unlimited time. A man can have but one domicile for the same purpose at any time, but he may have
numerous places of residence. His place of residence is generally his place of domicile, but it is not by
any means necessarily so since no length of residence without intention of remaining will constitute
domicile."

For political purposes the concepts of residence and domicile are dictated by the peculiar criteria of
political laws. As these concepts have evolved in our election law, what has clearly and unequivocally
emerged is the fact that residence for election purposes is used synonymously with domicile.

In Nuval vs. Guray, 24 the Court held that "the term residence . . . is synonymous with domicile which
imports not only intention to reside in a fixed place, but also personal presence in that place, coupled
with conduct indicative of such intention." 25 Larena vs. Teves 26 reiterated the same doctrine in a case
involving the qualifications of the respondent therein to the post of Municipal President of Dumaguete,
Negros Oriental. Faypon vs. Quirino, 27 held that the absence from residence to pursue studies or
practice a profession or registration as a voter other than in the place where one is elected does not
constitute loss of residence. 28 So settled is the concept (of domicile) in our election law that in these
and other election law cases, this Court has stated that the mere absence of an individual from his
permanent residence without the intention to abandon it does not result in a loss or change of domicile.

The deliberations of the 1987 Constitution on the residence qualification for certain elective positions
have placed beyond doubt the principle that when the Constitution speaks of "residence" in election
law, it actually means only "domicile" to wit:

Mr. Nolledo: With respect to Section 5, I remember that in the 1971 Constitutional Convention, there
was an attempt to require residence in the place not less than one year immediately preceding the day
of the elections. So my question is: What is the Committee's concept of residence of a candidate for the
legislature? Is it actual residence or is it the concept of domicile or constructive residence?

Mr. Davide: Madame President, insofar as the regular members of the National Assembly are
concerned, the proposed section merely provides, among others, 'and a resident thereof,' that is, in the
district for a period of not less than one year preceding the day of the election. This was in effect lifted
from the 1973 Constitution, the interpretation given to it was domicile. 29

xxx xxx xxx

Mrs. Rosario Braid: The next question is on Section 7, page 2. I think Commissioner Nolledo has raised
the same point that "resident" has been interpreted at times as a matter of intention rather than actual
residence.

Mr. De los Reyes: Domicile

454
Ms. Rosario Braid: Yes, So, would the gentleman consider at the proper time to go back to actual
residence rather than mere intention to reside?

Mr. De los Reyes: But we might encounter some difficulty especially considering that a provision in the
Constitution in the Article on Suffrage says that Filipinos living abroad may vote as enacted by law. So,
we have to stick to the original concept that it should be by domicile and not physical residence. 30

In Co vs. Electoral Tribunal of the House of Representatives, 31 this Court concluded that the framers of
the 1987 Constitution obviously adhered to the definition given to the term residence in election law,
regarding it as having the same meaning as domicile. 32

In the light of the principles just discussed, has petitioner Imelda Romualdez Marcos satisfied the
residency requirement mandated by Article VI, Sec. 6 of the 1987 Constitution? Of what significance is
the questioned entry in petitioner's Certificate of Candidacy stating her residence in the First Legislative
District of Leyte as seven (7) months?

It is the fact of residence, not a statement in a certificate of candidacy which ought to be decisive in
determining whether or not an individual has satisfied the constitution's residency qualification
requirement. The said statement becomes material only when there is or appears to be a deliberate
attempt to mislead, misinform, or hide a fact which would otherwise render a candidate ineligible. It
would be plainly ridiculous for a candidate to deliberately and knowingly make a statement in a
certificate of candidacy which would lead to his or her disqualification.

It stands to reason therefore, that petitioner merely committed an honest mistake in jotting down the
word "seven" in the space provided for the residency qualification requirement. The circumstances
leading to her filing the questioned entry obviously resulted in the subsequent confusion which
prompted petitioner to write down the period of her actual stay in Tolosa, Leyte instead of her period of
residence in the First District, which was "since childhood" in the space provided. These circumstances
and events are amply detailed in the COMELEC's Second Division's questioned resolution, albeit with a
different interpretation. For instance, when herein petitioner announced that she would be registering
in Tacloban City to make her eligible to run in the First District, private respondent Montejo opposed the
same, claiming that petitioner was a resident of Tolosa, not Tacloban City. Petitioner then registered in
her place of actual residence in the First District, which was Tolosa, Leyte, a fact which she subsequently
noted down in her Certificate of Candidacy. A close look at said certificate would reveal the possible
source of the confusion: the entry for residence (Item No. 7) is followed immediately by the entry for
residence in the constituency where a candidate seeks election thus:

7. RESIDENCE (complete Address): Brgy. Olot, Tolosa, Leyte

POST OFFICE ADDRESS FOR ELECTION PURPOSES: Brgy. Olot, Tolosa, Leyte

8. RESIDENCE IN THE CONSTITUENCY WHERE I SEEK TO BE ELECTED IMMEDIATELY PRECEDING THE


ELECTION: ______ Years and Seven Months.

455
Having been forced by private respondent to register in her place of actual residence in Leyte instead of
petitioner's claimed domicile, it appears that petitioner had jotted down her period of stay in her actual
residence in a space which required her period of stay in her legal residence or domicile. The
juxtaposition of entries in Item 7 and Item 8 — the first requiring actual residence and the second
requiring domicile — coupled with the circumstances surrounding petitioner's registration as a voter in
Tolosa obviously led to her writing down an unintended entry for which she could be disqualified. This
honest mistake should not, however, be allowed to negate the fact of residence in the First District if
such fact were established by means more convincing than a mere entry on a piece of paper. dctai

We now proceed to the matter of petitioner's domicile.

In support of its asseveration that petitioner's domicile could not possibly be in the First District of Leyte,
the Second Division of the COMELEC, in its assailed Resolution of April 24, 1995 maintains that "except
for the time when (petitioner) studied and worked for some years after graduation in Tacloban City, she
continuously lived in Manila." The Resolution additionally cites certain facts as indicative of the fact that
petitioner's domicile ought to be any place where she lived in the last few decades except Tacloban,
Leyte. First, according to the Resolution, petitioner, in 1959, resided in San Juan, Metro Manila where
she was also registered voter. Then, in 1965, following the election of her husband to the Philippine
presidency, she lived in San Miguel, Manila where she registered as a voter. In 1978 and thereafter, she
served as a member of the Batasang Pambansa and Governor of Metro Manila. "She could not, have
served these positions if she had not been a resident of Metro Manila," the COMELEC stressed. Here is
where the confusion lies.

We have stated, many times in the past, that an individual does not lose his domicile even if he has lived
and maintained residences in different places. Residence, it bears repeating, implies a factual
relationship to a given place for various purposes. The absence from legal residence or domicile to
pursue a profession, to study or to do other things of a temporary or semi-permanent nature does not
constitute loss of residence. Thus, the assertion by the COMELEC that "she could not have been a
resident of Tacloban City since childhood up to the time she filed her certificate of candidacy because
she became a resident of many places" flies in the face of settled jurisprudence in which this Court
carefully made distinctions between (actual) residence and domicile for election law purposes. In Larena
vs. Teves, 33 supra, we stressed:

[T]his court is of the opinion and so holds that a person who has his own house wherein he lives with his
family in a municipality without having ever had the intention of abandoning it, and without having lived
either alone or with his family in another municipality, has his residence in the former municipality,
notwithstanding his having registered as an elector in the other municipality in question and having
been a candidate for various insular and provincial positions, stating every time that he is a resident of
the latter municipality.

More significantly, in Faypon vs. Quirino, 34 we explained that:

A citizen may leave the place of his birth to look for "greener pastures," as the saying goes, to improve
his lot, and that, of course includes study in other places, practice of his avocation, or engaging in

456
business. When an election is to be held, the citizen who left his birthplace to improve his lot may desire
to return to his native town to cast his ballot but for professional or business reasons, or for any other
reason, he may not absent himself from his professional or business activities; so there he registers
himself as voter as he has the qualifications to be one and is not willing to give up or lose the
opportunity to choose the officials who are to run the government especially in national elections.
Despite such registration, the animus revertendi to his home, to his domicile or residence of origin has
not forsaken him. This may be the explanation why the registration of a voter in a place other than his
residence of origin has not been deemed sufficient to constitute abandonment or loss of such residence.
It finds justification in the natural desire and longing of every person to return to his place of birth. This
strong feeling of attachment to the place of one's birth must be overcome by positive proof of
abandonment for another.

From the foregoing, it can be concluded that in its above-cited statements supporting its proposition
that petitioner was ineligible to run for the position of Representative of the First District of Leyte, the
COMELEC was obviously referring to petitioner's various places of (actual) residence, not her domicile. In
doing so, it not only ignored settled jurisprudence on residence in election law and the deliberations of
the constitutional commission but also the provisions of the Omnibus Election Code (B.P. 881). 35

What is undeniable, however, are the following set of facts which establish the fact of petitioner's
domicile, which we lift verbatim from the COMELEC's Second Division's assailed Resolution: 36

In or about 1938 when respondent was a little over 8 years old, she established her domicile in
Tacloban, Leyte (Tacloban City). She studied in the Holy Infant Academy in Tacloban from 1938 to 1949
when she graduated from high school. She pursued her college studies in St. Paul's College, now Divine
Word University in Tacloban, where she earned her degree in Education. Thereafter, she taught in the
Leyte Chinese School, still in Tacloban City. In 1952 she went to Manila to work with her cousin, the late
speaker Daniel Z. Romualdez in his office in the House of Representatives. In 1954, she married ex-
President Ferdinand E. Marcos when he was still a congressman of Ilocos Norte and registered there as a
voter. When her husband was elected Senator of the Republic in 1959, she and her husband lived
together in San Juan, Rizal where she registered as a voter. In 1965, when her husband was elected
President of the Republic of the Philippines, she lived with him in Malacañang Palace and registered as a
voter in San Miguel, Manila.

[I]n February 1986 (she claimed that) she and her family were abducted and kidnapped to Honolulu,
Hawaii. In November 1991, she came home to Manila. In 1992, respondent ran for election as President
of the Philippines and filed her Certificate of Candidacy wherein she indicated that she is a resident and
registered voter of San Juan, Metro Manila.

Applying the principles discussed to the facts found by COMELEC, what is inescapable is that petitioner
held various residences for different purposes during the past four decades. None of these purposes
unequivocally point to an intention to abandon her domicile of origin in Tacloban, Leyte. Moreover,
while petitioner was born in Manila, as a minor she naturally followed the domicile of her parents. She
grew up in Tacloban, reached her adulthood there and eventually established residence in different

457
parts of the country for various reasons. Even during her husband's presidency, at the height of the
Marcos Regime's powers, petitioner kept her close ties to her domicile of origin by establishing
residences in Tacloban, celebrating her birthdays and other important personal milestones in her home
province, instituting well-publicized projects for the benefit of her province and hometown, and
establishing a political power base where her siblings and close relatives held positions of power either
through the ballot or by appointment, always with either her influence or consent. These well-publicized
ties to her domicile of origin are part of the history and lore of the quarter century of Marcos power in
our country. Either they were entirely ignored in the COMELEC's Resolutions, or the majority of the
COMELEC did not know what the rest of the country always knew: the fact of petitioner's domicile in
Tacloban, Leyte.

Private respondent in his Comment, contends that Tacloban was not petitioner's domicile of origin
because she did not live there until she was eight years old. He avers that after leaving the place in 1952,
she "abandoned her residency (sic) therein for many years and . . . (could not) re-establish her domicile
in said place by merely expressing her intention to live there again." We do not agree.

First, a minor follows the domicile of his parents. As domicile, once acquired is retained until a new one
is gained, it follows that in spite of the fact of petitioner's being born in Manila, Tacloban, Leyte was her
domicile of origin by operation of law. This domicile was not established only when she reached the age
of eight years old, when her father brought his family back to Leyte contrary to private respondent's
averments.

Second, domicile of origin is not easily lost. To successfully effect a change of domicile, one must
demonstrate: 37

1. An actual removal or an actual change of domicile;

2. A bona fide intention of abandoning the former place of residence and establishing a new one;
and

3. Acts which correspond with the purpose.

In the absence of clear and positive proof based on these criteria, the residence of origin should be
deemed to continue. Only with evidence showing concurrence of all three requirements can the
presumption of continuity or residence be rebutted, for a change of residence requires an actual and
deliberate abandonment, and one cannot have two legal residences at the same time. 38 In the case at
bench, the evidence adduced by private respondent plainly lacks the degree of persuasiveness required
to convince this court that an abandonment of domicile of origin in favor of a domicile of choice indeed
occurred. To effect an abandonment requires the voluntary act of relinquishing petitioner's former
domicile with an intent to supplant the former domicile with one of her own choosing (domicilium
voluntarium).

In this connection, it cannot be correctly argued that petitioner lost her domicile of origin by operation
of law as a result of her marriage to the late President Ferdinand E. Marcos in 1954. For there is a clearly

458
established distinction between the Civil Code concepts of "domicile" and "residence." 39 The
presumption that the wife automatically gains the husband's domicile by operation of law upon
marriage cannot be inferred from the use of the term "residence" in Article 110 of the Civil Code
because the Civil Code is one area where the two concepts are well delineated. Dr. Arturo Tolentino,
writing on this specific area explains:

In the Civil Code, there is an obvious difference between domicile and residence. Both terms imply
relations between a person and a place; but in residence, the relation is one of fact while in domicile it is
legal or juridical, independent of the necessity of physical presence. 40

Article 110 of the Civil Code provides:

ARTICLE 110. The husband shall fix the residence of the family. But the court may exempt the wife
from living with the husband if he should live abroad unless in the service of the Republic.

A survey of jurisprudence relating to Article 110 or to the concepts of domicile or residence as they
affect the female spouse upon marriage yields nothing which would suggest that the female spouse
automatically loses her domicile of origin in favor of the husband's choice of residence upon marriage.

Article 110 is a virtual restatement of Article 58 of the Spanish Civil Code of 1889 which states: LexLib

La mujer esta obligada a seguir a su marido donde quiera que fije su residencia. Los Tribunales, sin
embargo, podran con justa causa eximirla de esta obligacion cuando el marido transende su residencia a
ultramar o' a pais extranjero.

Note the use of the phrase "donde quiera su fije de residencia" in the aforequoted article, which means
wherever (the husband) wishes to establish residence. This part of the article clearly contemplates only
actual residence because it refers to a positive act of fixing a family home or residence. Moreover, this
interpretation is further strengthened by the phrase "cuando el marido translade su residencia" in the
same provision which means, "when the husband shall transfer his residence," referring to another
positive act of relocating the family to another home or place of actual residence. The article obviously
cannot be understood to refer to domicile which is a fixed, fairly-permanent concept when it plainly
connotes the possibility of transferring from one place to another not only once, but as often as the
husband may deem fit to move his family, a circumstance more consistent with the concept of actual
residence.

The right of the husband to fix the actual residence is in harmony with the intention of the law to
strengthen and unify the family, recognizing the fact that the husband and the wife bring into the
marriage different domiciles (of origin). This difference could, for the sake of family unity, be reconciled
only by allowing the husband to fix a single place of actual residence.

Very significantly, Article 110 of the Civil Code is found under Title V under the heading: RIGHTS AND
OBLIGATIONS BETWEEN HUSBAND AND WIFE. Immediately preceding Article 110 is Article 109 which
obliges the husband and wife to live together, thus:

459
ARTICLE 109. The husband and wife are obligated to live together, observe mutual respect and fidelity
and render mutual help and support.

The duty to live together can only be fulfilled if the husband and wife are physically together. This takes
into account the situations where the couple has many residences (as in the case of petitioner). If the
husband has to stay in or transfer to any one of their residences, the wife should necessarily be with him
in order that they may "live together." Hence, it is illogical to conclude that Art. 110 refers to "domicile"
and not to "residence." Otherwise, we shall be faced with a situation where the wife is left in the
domicile while the husband, for professional or other reasons, stays in one of their (various) residences.
As Dr. Tolentino further explains:

Residence and Domicile. — Whether the word "residence" as used with reference to particular matters
is synonymous with "domicile" is a question of some difficulty, and the ultimate decision must be made
from a consideration of the purpose and intent with which the word is used. Sometimes they are used
synonymously, at other times they are distinguished from one another.

xxx xxx xxx

Residence in the civil law is a material fact, referring to the physical presence of a person in a place. A
person can have two or more residences, such as a country residence and a city residence. Residence is
acquired by living in a place; on the other hand, domicile can exist without actually living in the place.
The important thing for domicile is that, once residence has been established in one place, there be an
intention to stay there permanently, even if residence is also established in some other place. 41

In fact, even the matter of a common residence between the husband and the wife during the marriage
is not an iron-clad principle. In cases applying the Civil Code on the question of a common matrimonial
residence, our jurisprudence has recognized certain situations 42 where the spouses could not be
compelled to live with each other such that the wife is either allowed to maintain a residence different
from that of her husband or, for obviously practical reasons, revert to her original domicile (apart from
being allowed to opt for a new one). In De la Vina vs. Villareal 43 this Court held that "[a] married
woman may acquire a residence or domicile separate from that of her husband during the existence of
the marriage where the husband has given cause for divorce." 44 Note that the Court allowed the wife
either to obtain new residence or to choose a new domicile in such an event. In instances where the
wife actually opts, under the Civil Code, to live separately from her husband either by taking new
residence or reverting to her domicile of origin, the Court has held that the wife could not be compelled
to live with her husband on pain of contempt. In Arroyo vs. Vasques de Arroyo 45 the Court held that:

Upon examination of the authorities, we are convinced that it is not within the province of the courts at
this country to attempt to compel one of the spouses to cohabit with, and render conjugal rights to, the
other. Of course where the property rights of one of the pair are invaded, an action for restitution of
such rights can be maintained. But we are disinclined to sanction the doctrine that an order, enforcible
(sic) by process of contempt, may be entered to compel the restitution of the purely personal right of
consortium. At best such an order can be effective for no other purpose than to compel the spouses to
live under the same roof; and the experience of those countries where the courts of justice have

460
assumed to compel the cohabitation of married people shows that the policy of the practice is
extremely questionable. Thus in England, formerly the Ecclesiastical Court entertained suits for the
restitution of conjugal rights at the instance of either husband or wife; and if the facts were found to
warrant it, that court would make a mandatory decree, enforceable by process of contempt in case of
disobedience, requiring the delinquent party to live with the other and render conjugal rights. Yet this
practice was sometimes criticized even by the judges who felt bound to enforce such orders, and in
Weldon v. Weldon (9 P.D. 52), decided in 1883, Sir James Hannen, President in the Probate, Divorce and
Admiralty Division of the High Court of Justice, expressed his regret that the English law on the subject
was not the same as that which prevailed in Scotland, where a decree of adherence, equivalent to the
decree for the restitution of conjugal rights in England, could be obtained by the injured spouse, but
could not be enforced by imprisonment. Accordingly, in obedience to the growing sentiment against the
practice, the Matrimonial Causes Act (1884) abolished the remedy of imprisonment; though a decree for
the restitution of conjugal rights can still be procured, and in case of disobedience may serve in
appropriate cases as the basis of an order for the periodical payment of a stipend in the character of
alimony.

In the voluminous jurisprudence of the United States, only one court, so far as we can discover, has ever
attempted to make a preemptory order requiring one of the spouses to live with the other; and that was
in a case where a wife was ordered to follow and live with her husband, who had changed his domicile
to the City of New Orleans. The decision referred to (Bahn vs. Darby, 36 La. Ann., 70) was based on a
provision of the Civil Code of Louisiana similar to Article 56 of the Spanish Civil Code. It was decided
many years ago, and the doctrine evidently has not been fruitful even in the State of Louisiana. In other
states of the American Union the idea of enforcing cohabitation by process of contempt is rejected. (21
Cyc., 1148)

In a decision of January 2, 1909, the Supreme Court of Spain appears to have affirmed an order of the
Audiencia Territorial de Valladolid requiring a wife to return to the marital domicile, and in the
alternative, upon her failure to do so, to make a particular disposition of certain money and effects then
in her possession and to deliver to her husband, as administrator of the ganancial property, all income,
rents, and interest which might accrue to her from the property which she had brought to the marriage.
(113 Jur. Civ., pp. 1, 11) But it does not appear that this order for the return of the wife to the marital
domicile was sanctioned by any other penalty than the consequences that would be visited upon her in
respect to the use and control of her property; and it does not appear that her disobedience to that
order would necessarily have been followed by imprisonment for contempt.

Parenthetically when Petitioner was married to then Congressman Marcos, in 1954, petitioner was
obliged — by virtue of Article 110 of the Civil Code — to follow her husband's actual place of residence
fixed by him. The problem here is that at that time, Mr. Marcos had several places of residence, among
which were San Juan, Rizal and Batac, Ilocos Norte. There is no showing which of these places Mr.
Marcos did fix as his family's residence. But assuming that Mr. Marcos had fixed any of these places as
the conjugal residence, what petitioner gained upon marriage was actual residence. She did not lose her
domicile of origin. aisadc

461
On the other hand, the common law concept of "matrimonial domicile" appears to have been
incorporated, as a result of our jurisprudential experiences after the drafting of the Civil Code of 1950,
into the New Family Code. To underscore the difference between the intentions of the Civil Code and
the Family Code drafters, the term residence has been supplanted by the term domicile in an entirely
new provision (Art. 69) distinctly different in meaning and spirit from that found in Article 110. The
provision recognizes revolutionary changes in the concept of women's rights in the intervening years by
making the choice of domicile a product of mutual agreement between the spouses. 46

Without as much belaboring the point, the term residence may mean one thing in civil law (or under the
Civil Code) and quite another thing in political law. What stands clear is that insofar as the Civil Code is
concerned — affecting the rights and obligations of husband and wife — the term residence should only
be interpreted to mean "actual residence." The inescapable conclusion derived from this unambiguous
civil law delineation therefore, is that when petitioner married the former President in 1954, she kept
her domicile of origin and merely gained a new home, not a domicilium necesarium.

Even assuming for the sake of argument that petitioner gained a new "domicile" after her marriage and
only acquired a right to choose a new one after her husband died, petitioner's acts following her return
to the country clearly indicate that she not only impliedly but expressly chose her domicile of origin
(assuming this was lost by operation of law) as her domicile. This "choice" was unequivocally expressed
in her letters to the Chairman of the PCGG when petitioner sought the PCGG's permission to
"rehabilitate (our) ancestral house in Tacloban and Farm in Olot, Leyte . . . to make them livable for the
Marcos family to have a home in our homeland." 47 Furthermore, petitioner obtained her residence
certificate in 1992 in Tacloban, Leyte, while living in her brother's house, an act which supports the
domiciliary intention clearly manifested in her letters to the PCGG Chairman. She could not have gone
straight to her home in San Juan, as it was in a state of disrepair, having been previously looted by
vandals. Her "homes" and "residences" following her arrival in various parts of Metro Manila merely
qualified as temporary or "actual residences," not domicile. Moreover, and proceeding from our
discussion pointing out specific situations where the female spouse either reverts to her domicile of
origin or chooses a new one during the subsistence of the marriage, it would be highly illogical for us to
assume that she cannot regain her original domicile upon the death of her husband absent a positive act
of selecting a new one where situations exist within the subsistence of the marriage itself where the
wife gains a domicile different from her husband.

In the light of all the principles relating to residence and domicile enunciated by this court up to this
point, we are persuaded that the facts established by the parties weigh heavily in favor of a conclusion
supporting petitioner's claim of legal residence or domicile in the First District of Leyte.

II. The jurisdictional issue

Petitioner alleges that the jurisdiction of the COMELEC had already lapsed considering that the assailed
resolutions were rendered on April 24, 1995, fourteen (14) days before the election in violation of
Section 78 of the Omnibus Election Code. 48 Moreover, petitioner contends that it is the House of
Representatives Electoral Tribunal and not the COMELEC which has jurisdiction over the election of

462
members of the House of Representatives in accordance with Article VI, Sec. 17 of the Constitution. This
is untenable.

It is a settled doctrine that a statute requiring rendition of judgment within a specified time is generally
construed to be merely directory, 49 "so that non-compliance with them does not invalidate the
judgment on the theory that if the statute had intended such result it would have clearly indicated it."
50 The difference between a mandatory and a directory provision is often made on grounds of
necessity. Adopting the same view held by several American authorities, this court in Marcelino v. Cruz
held that: 51

The difference between a mandatory and directory provision is often determined on grounds of
expediency, the reason being that less injury results to the general public by disregarding than enforcing
the letter of the law.

In Trapp v. Mc Cormick, a case calling for the interpretation of a statute containing a limitation of thirty
(30) days within which a decree may be entered without the consent of counsel, it was held that 'the
statutory provisions which may be thus departed from with impunity, without affecting the validity of
statutory proceedings, are usually those which relate to the mode or time of doing that which is
essential to effect the aim and purpose of the Legislature or some incident of the essential act.' Thus, in
said case, the statute under examination was construed merely to be directory.

The mischief in petitioner's contending that the COMELEC should have abstained from rendering a
decision after the period stated in the Omnibus Election Code because it lacked jurisdiction, lies in the
fact that our courts and other quasi-judicial bodies would then refuse to render judgments merely on
the ground of having failed to reach a decision within a given or prescribed period. cdll

In any event, with the enactment of Sections 6 and 7 of R.A. 6646 in relation to Section 78 of B.P. 881,
52 it is evident that the respondent Commission does not lose jurisdiction to hear and decide a pending
disqualification case under Section 78 of B.P. 881 even after the elections. cdlex

As to the House of Representatives Electoral Tribunal's supposed assumption of jurisdiction over the
issue of petitioner's qualifications after the May 8, 1995 elections, suffice it to say that HRET's
jurisdiction as the sole judge of all contests relating to the elections, returns and qualifications of
members of Congress begins only after a candidate has become a member of the House of
Representatives. 53 Petitioner not being a member of the House of Representatives, it is obvious that
the HRET at this point has no jurisdiction over the question.

It would be an abdication of many of the ideals enshrined in the 1987 Constitution for us to either to
ignore or deliberately make distinctions in law solely on the basis of the personality of a petitioner in a
case. Obviously a distinction was made on such a ground here. Surely, many established principles of
law, even of election laws were flouted for the sake perpetuating power during the pre-EDSA regime.
We renege on these sacred ideals, including the meaning and spirit of EDSA by ourselves bending
established principles of law to deny an individual what he or she justly deserves in law. Moreover, in
doing so, we condemn ourselves to repeat the mistake of the past.

463
WHEREFORE, having determined that petitioner possesses the necessary residence qualifications to run
for a seat in the House of Representatives in the First District of Leyte, the COMELEC's questioned
Resolutions dated April 24, May 7, May 11, and May 25, 1995 are hereby SET ASIDE. Respondent
COMELEC is hereby directed to order the Provincial Board of Canvassers to proclaim petitioner as the
duly elected Representative of the First District of Leyte.

SO ORDERED. LLjur

Narvasa, C.J., joins J. Mendoza in his separate opinion.

Feliciano, J., is on official leave.

Padilla, Regalado and Davide, Jr., JJ., see dissenting opinion.

Romero, Vitug and Mendoza, JJ., see separate opinion.

Bellosillo and Melo, JJ., join the separate opinion of J. Puno.

Puno and Francisco, JJ., see separate concurring opinion.

Hermosisima, Jr., J., joins the dissenting opinion of J. Padilla.

Separate Opinions

PUNO, J ., concurring:

It was Aristotle who taught mankind that things that are alike should be treated alike, while things that
are unalike should be treated unalike in proportion to their unalikeness. 1 Like other candidates,
petitioner has clearly met the residence requirement provided by Section 6, Article VI of the
Constitution. 2 We cannot disqualify her and treat her unalike, for the Constitution guarantees equal
protection of the law. I proceed from the following factual and legal propositions:

First. There is no question that petitioner's original domicile is in Tacloban, Leyte. Her parents were
domiciled in Tacloban. Their ancestral house is in Tacloban. They have vast real estate in the place.
Petitioner went to school and thereafter worked there. I consider Tacloban as her initial domicile, both
her domicile of origin and her domicile of choice. Her domicile of origin as it was the domicile of her
parents when she was a minor; and her domicile of choice, as she continued living there even after
reaching the age of majority.

Second. There is also no question that in May, 1954, petitioner married the late President Ferdinand E.
Marcos. By contracting marriage, her domicile became subject to change by law, and the right to change
it was given by Article 110 of the Civil Code to her husband. Article 110 of the Civil Code provides:

"ARTICLE 110. The husband shall fix the residence of the family. But the court may exempt the wife
from living with the husband if he should live abroad unless in the service of the Republic." 3 (Emphasis
supplied)

464
In De la Viña v. Villareal and Geopano, 4 this Court explained why the domicile of the wife ought to
follow that of the husband. We held: "The reason is founded upon the theoretic identity of person and
interest between the husband and the wife, and the presumption that, from the nature of the relation,
the home of one is the home of the other. It is intended to promote, strengthen, and secure their
interests in this relation, as it ordinarily exists, where union and harmony prevail." 5 In accord with this
objective, Article 109 of the Civil Code also obligated the husband and wife "to live together." LLpr

Third. The difficult issues start as we determine whether petitioner's marriage to former President
Marcos ipso facto resulted in the loss of her Tacloban domicile. I respectfully submit that her marriage
by itself alone did not cause her to lose her Tacloban domicile. Article 110 of the Civil Code merely gave
the husband the right to fix the domicile of the family. In the exercise of the right, the husband may
explicitly choose the prior domicile of his wife, in which case, the wife's domicile remains unchanged.
The husband can also implicitly acquiesce to his wife's prior domicile even if it is different. So we held in
de la Viña, 6

". . . When married women as well as children subject to parental authority live, with the acquiescence
of their husbands or fathers, in a place distinct from where the latter live, they have their own
independent domicile. . . ."

It is not, therefore, the mere fact of marriage but the deliberate choice of a different domicile by the
husband that will change the domicile of a wife from what it was prior to their marriage. The domiciliary
decision made by the husband in the exercise of the right conferred by Article 110 of the Civil Code
binds the wife. Any and all acts of a wife during her coverture contrary to the domiciliary choice of the
husband cannot change in any way the domicile legally fixed by the husband. These acts are void not
only because the wife lacks the capacity to choose her domicile but also because they are contrary to
law and public policy.

In the case at bench, it is not disputed that former President Marcos exercised his right to fix the family
domicile and established it in Batac, Ilocos Norte, where he was then the congressman. At that
particular point of time and throughout their married life, petitioner lost her domicile in Tacloban, Leyte.
Since petitioner's Batac domicile has been fixed by operation of law, it was not affected in 1959 when
her husband was elected as Senator, when they lived in San Juan, Rizal and where she registered as a
voter. It was not also affected in 1965 when her husband was elected President, when they lived in
Malacañang Palace, and when she registered as a voter in San Miguel, Manila. Nor was it affected when
she served as a member of the Batasang Pambansa, Minister of Human Settlements and Governor of
Metro Manila during the incumbency of her husband as President of the nation. Under Article 110 of the
Civil Code, it was only her husband who could change the family domicile in Batac and the evidence
shows he did not effect any such change. To a large degree, this follows the common law that "a woman
on her marriage loses her own domicile and by operation of law, acquires that of her husband, no
matter where the wife actually lives or what she believes or intends." 7

Fourth. The more difficult task is how to interpret the effect of the death on September 28, 1989 of
former President Marcos on petitioner's Batac domicile. The issue is of first impression in our

465
jurisdiction and two (2) schools of thought contend for acceptance. One is espoused by our
distinguished colleague, Mr. Justice Davide, Jr., heavily relying on American authorities. 8 He echoes the
theory that after the husband's death, the wife retains the last domicile of her husband until she makes
an actual change.

I do not subscribe to this submission. The American case law that the wife still retains her dead
husband's domicile is based on ancient common law which we can no longer apply in the Philippine
setting today. The common law identified the domicile of a wife as that of the husband and denied to
her the power of acquiring a domicile of her own separate and apart from him. 9 Legal scholars agree
that two (2) reasons support this common law doctrine. The first reason as pinpointed by the legendary
Blackstone is derived from the view that "the very being or legal existence of the woman is suspended
during the marriage, or at least is incorporated and consolidated into that of the husband." 10 The
second reason lies in "the desirability of having the interests of each member of the family unit
governed by the same law." 11 The presumption that the wife retains the domicile of her deceased
husband is an extension of this common law concept. The concept and its extension have provided
some of the most iniquitous jurisprudence against women. It was under common law that the 1873
American case of Bradwell v. Illinois 12 was decided where women were denied the right to practice
law. It was unblushingly ruled that "the natural and proper timidity and delicacy which belongs to the
female sex evidently unfits it for many of the occupations of civil life. . . . This is the law of the Creator."
Indeed, the rulings relied upon by Mr. Justice Davide in CJS 13 and AM JUR 2d 14 are American state
court decisions handed down between the years 1917 15 and 1938, 16 or before the time when women
were accorded equality of rights with men. Undeniably, the womens' liberation movement resulted in
far-ranging state legislations in the United States to eliminate gender inequality. 17 Starting in the
decade of the seventies, the courts likewise liberalized their rulings as they started invalidating laws
infected with gender-bias. It was in 1971 when the US Supreme Court in Reed v. Reed, 18 stuck a big
blow for women equality when it declared as unconstitutional an Idaho law that required probate courts
to choose male family members over females as estate administrators. It held that mere administrative
inconvenience cannot justify a sex-based distinction. These significant changes both in law and in case
law on the status of women virtually obliterated the iniquitous common law surrendering the rights of
married women to their husbands based on the dubious theory of the parties' theoretic oneness. The
Corpus Juris Secundum editors did not miss the relevance of this revolution on women's right as they
observed: "However, it has been declared that under modern statutes changing the status of married
women and departing from the common law theory of marriage, there is no reason why a wife may not
acquire a separate domicile for every purpose known to the law." 19 In publishing in 1969 the
Restatement of the Law, Second (Conflict of Laws 2d), the reputable American Law Institute also
categorically stated that the view of Blackstone ". . . is no longer held. As the result of statutes and court
decisions, a wife now possesses practically the same rights and powers as her unmarried sister." 20

In the case at bench, we have to decide whether we should continue clinging to the anachronistic
common law that demeans women, especially married women. I submit that the Court has no choice
except to break away from this common law rule, the root of the many degradations of Filipino women.

466
Before 1988, our laws particularly the Civil Code, were full of gender discriminations against women.
Our esteemed colleague, Madam Justice Flerida Ruth Romero, cited a few of them as follows: 21

"xxx xxx xxx

"Legal Disabilities Suffered by Wives

"Not generally known is the fact that under the Civil Code, wives suffer under certain restrictions or
disabilities. For instance, the wife cannot accept gifts from others, regardless of the sex of the giver or
the value of the gift, other than from her very close relatives, without her husband's consent. She may
accept only from, say, her parents, parents-in-law, brothers, sisters and the relatives within the so-called
fourth civil degree. She may not exercise her profession or occupation or engage in business if her
husband objects on serious grounds or if his income is sufficient to support their family in accordance
with their social standing. As to what constitutes 'serious grounds' for objecting, this is within the
discretion of the husband.

"xxx xxx xxx

"Because of the present inequitable situation, the amendments to the Civil Law being proposed by the
University of the Philippines Law Center would allow absolute divorce which severes the matrimonial
ties, such that the divorced spouses are free to get married a year after the divorce is decreed by the
courts. However, in order to place the husband and wife on an equal footing insofar as the bases for
divorce are concerned, the following are specified as the grounds for absolute divorce: (1) adultery or
having a paramour committed by the respondent in any of the ways specified in the Revised Penal Code;
or (2) an attempt by the respondent against the life of the petitioner which amounts to attempted
parricide under the Revised Penal Code; (3) abandonment of the petitioner by the respondent without
just cause for a period of three consecutive years; or (4) habitual maltreatment.

"With respect to property relations, the husband is automatically the administrator of the conjugal
property owned in common by the married couple even if the wife may be the more astute or
enterprising partner. The law does not leave it to the spouses to decide who shall act as such
administrator. Consequently, the husband is authorized to engage in acts and enter into transactions
beneficial to the conjugal partnership. The wife, however, cannot similarly bind the partnership without
the husband's consent.

"And while both exercise joint parental authority over their children, it is the father whom the law
designates as the legal administrator of the property pertaining to the unemancipated child."

Taking the lead in Asia, our government exerted efforts, principally through legislations, to eliminate
inequality between men and women in our land. The watershed came on August 3, 1988 when our
Family Code took effect which, among others, terminated the unequal treatment of husband and wife as
to their rights and responsibilities. 22

The Family Code attained this elusive objective by giving new rights to married women and by abolishing
sex-based privileges of husbands. Among others, married women are now given the joint right to

467
administer the family property, whether in the absolute community system or in the system of conjugal
partnership; 23 joint parental authority over their minor children, both over their persons as well as
their properties; 24 joint responsibility for the support of the family; 25 the right to jointly manage the
household; 26 and, the right to object to their husband's exercise of profession, occupation, business or
activity. 27 Of particular relevance to the case at bench is Article 69 of the Family Code which took away
the exclusive right of the husband to fix the family domicile and gave it jointly to the husband and the
wife, thus:

"ARTICLE 69. The husband and wife shall fix the family domicile. In case of disagreement, the court
shall decide.

The court may exempt one spouse from living with the other if the latter should live abroad or there are
other valid and compelling reasons for the exemption. However, such exemption shall not apply if the
same is not compatible with the solidarity of the family." (Emphasis supplied)

Article 69 repealed Article 110 of the Civil Code. Commenting on the duty of the husband and wife to
live together, former Madam Justice Alice Sempio-Diy of the Court of Appeals specified the instances
when a wife may now refuse to live with her husband, thus: 28

"(2) The wife has the duty to live with her husband, but she may refuse to do so in certain cases like:

(a) If the place chosen by the husband as family residence is dangerous to her life;

(b) If the husband subjects her to maltreatment or abusive conduct or insults, making common life
impossible;

(c) If the husband compels her to live with his parents, but she cannot get along with her mother-
in-law and they have constant quarrels. (Del Rosario v. Del Rosario, CA, 46 OG 6122);

(d) Where the husband has continuously carried illicit relations for 10 years with different women
and treated his wife roughly and without consideration. (Dadivas v. Villanueva, 54 Phil. 92);

(e) Where the husband spent his time in gambling, giving no money to his family for food and
necessities, and at the same time insulting his wife and laying hands on her. (Panuncio v. Sula, CA, 34 OG
129);

(f) If the husband has no fixed residence and lives a vagabond life as a tramp (1 Manresa 329);

(g) If the husband is carrying on a shameful business at home (Gahn v. Darby, 38 La. Ann. 70)."

The inescapable conclusion is that our Family Code has completely emancipated the wife from the
control of the husband, thus abandoning the parties' theoretic identity of interest. No less than the late
revered Mr. Justice J.B.L. Reyes who chaired the Civil Code Revision Committee of the UP Law Center
gave this insightful view in one of his rare lectures after retirement: 29

"xxx xxx xxx

468
"The Family Code is primarily intended to reform the family law so as to emancipate the wife from the
exclusive control of the husband and to place her at parity with him insofar as the family is concerned.
The wife and the husband are now placed on equal standing by the Code. They are now joint
administrators of the family properties and exercise joint authority over the persons and properties of
their children. This means a dual authority in the family. The husband will no longer prevail over the wife
but she has to agree on all matters concerning the family." (Emphasis supplied)

In light of the Family Code which abrogated the inequality between husband and wife as started and
perpetuated by the common law, there is no reason in espousing the anomalous rule that the wife still
retains the domicile of her dead husband. Article 110 of the Civil Code which provides the statutory
support for this stance has been repealed by Article 69 of the Family Code. By its repeal, it becomes a
dead-letter law, and we are not free to resurrect it by giving it further effect in any way or manner such
as by ruling that the petitioner is still bound by the domiciliary determination of her dead husband.

Aside from reckoning with the Family Code, we have to consider our Constitution and its firm
guarantees of due process and equal protection of law. 30 It can hardly be doubted that the common
law imposition on a married woman of her dead husband's domicile even beyond his grave is patently
discriminatory to women. It is a gender-based discrimination and is not rationally related to the
objective of promoting family solidarity. It cannot survive a constitutional challenge. Indeed, compared
with our previous fundamental laws, the 1987 Constitution is more concerned with equality between
sexes as it explicitly commands that the State ". . . shall ensure fundamental equality before the law of
women and men." To be exact, Section 14, Article II provides: "The State recognizes the role of women
in nation building, and shall ensure fundamental equality before the law of women and men." We shall
be transgressing the sense and essence of this constitutional mandate if we insist on giving our women
the caveman's treatment. LLjur

Prescinding from these premises, I respectfully submit that the better stance is to rule that petitioner
reacquired her Tacloban domicile upon the death of her husband in 1989. This is the necessary
consequence of the view that petitioner's Batac dictated domicile did not continue after her husband's
death; otherwise, she would have no domicile and that will violate the universal rule that no person can
be without a domicile at any point of time. This stance also restores the right of petitioner to choose her
domicile before it was taken away by Article 110 of the Civil Code, a right now recognized by the Family
Code and protected by the Constitution. Likewise, I cannot see the fairness of the common law requiring
petitioner to choose again her Tacloban domicile before she could be released from her Batac domicile.
She lost her Tacloban domicile not through her act but through the act of her deceased husband when
he fixed their domicile in Batac. Her husband is dead and he cannot rule her beyond the grave. The law
disabling her to choose her own domicile has been repealed. Considering all these, common law should
not put the burden on petitioner to prove she has abandoned her dead husband's domicile. There is
neither rhyme nor reason for this gender-based burden.

But even assuming arguendo that there is need for convincing proof that petitioner chose to reacquire
her Tacloban domicile, still, the records reveal ample evidence to this effect. In her affidavit submitted

469
to the respondent COMELEC, petitioner averred:

"xxx xxx xxx

"36. In November, 1991, I came home to our beloved country, after several requests for my return
were denied by President Corazon C. Aquino, and after I filed suits for our Government to issue me my
passport.

37. But I came home without the mortal remains of my beloved husband, President Ferdinand E.
Marcos, which the Government unreasonably considered a threat to the national security and welfare.

38. Upon my return to the country, I wanted to immediately live and reside in Tacloban City or in
Olot, Tolosa, Leyte, even if my residences there were not livable as they had been destroyed and
cannibalized. The PCGG, however, did not permit and allow me.

39. As a consequence, I had to live at various times in the Westin Philippine Plaza in Pasay City, a
friend's apartment on Ayala Avenue, a house in South Forbes Park which my daughter rented, and
Pacific Plaza, all in Makati.

40. After the 1992 Presidential Elections, I lived and resided in the residence of my brother in San
Jose, Tacloban City, and pursued my negotiations with PCGG to recover my sequestered residences in
Tacloban City and Barangay Olot, Tolosa, Leyte.

40.1. In preparation for my observance of All Saints' Day and All Souls' Day that year, I renovated my
parents' burial grounds and entombed their bones which had been excavated, unearthed and scattered.

41. On November 29, 1993, I formally wrote PCGG Chairman Magtanggol Gunigundo for
permissions to —

'. . . rehabilitate . . . (o)ur ancestral house in Tacloban and farmhouse in Olot, Leyte . . . to make them
livable for us the Marcos family to have a home in our own motherland.'

"xxx xxx xxx

42. It was only on 06 June 1994, however, when PCGG Chairman Gunigundo, in his letter to Col.
Simeon Kempis, Jr., PCGG Region 8 Representative, allowed me to repair and renovate my Leyte
residences. I quote part of his letter:

'Dear Col. Kempis,

Upon representation by Mrs. Imelda R. Marcos to this Commission, that she intends to visit our
sequestered properties in Leyte, please allow her access thereto. She may also cause repairs and
renovation of the sequestered properties, in which event, it shall be understood that her undertaking
said repairs is not authorization for her to take over said properties, and that all expenses shall be for
her account and not reimbursable. Please extend the necessary courtesy to her.'

470
"xxx xxx xxx

43. I was not permitted, however, to live and stay in the Sto. Niño Shrine residence in Tacloban City
where I wanted to stay and reside, after repairs and renovations were completed. In August 1994, I
transferred from San Jose, Tacloban City, to my residence in Barangay Olot, Tolosa, Leyte, when PCGG
permitted me to stay and live there."

It is then clear that in 1992 petitioner reestablished her domicile in the First District of Leyte. It is not
disputed that in 1992, she first lived at the house of her brother in San Jose, Tacloban City and later, in
August 1994, she transferred her residence in Barangay Olot, Tolosa, Leyte. Both Tacloban City and the
municipality of Olot are within the First District of Leyte. Since petitioner reestablished her old domicile
in 1992 in the First District of Leyte, she more than complied with the constitutional requirement of
residence ". . . for a period of not less than one year immediately preceding the day of the election," i.e.,
the May 8, 1995 elections. LLcd

The evidence presented by the private respondent to negate the Tacloban domicile of petitioner is nil.
He presented petitioner's Voter's Registration Record filed with the Board of Election Inspectors of
Precinct 10-A of Barangay Olot, Tolosa, Leyte wherein she stated that her period of residence in said
barangay was six (6) months as of the date of her filing of said Voter's Registration Record on January
28, 1995. 31 This statement in petitioner's Voter's Registration Record is a non-prejudicial admission.
The Constitution requires at least one (1) year residence in the district in which the candidate shall be
elected. In the case at bench, the reference is the First District of Leyte. Petitioner's statement proved
that she resided in Olot six (6) months before January 28, 1995 but did not disprove that she has also
resided in Tacloban City starting 1992. As aforestated, Olot and Tacloban City are both within the First
District of Leyte, hence, her six (6) months residence in Olot should be counted not against, but in her
favor. Private respondent also presented petitioner's Certificate of Candidacy filed on March 8, 1995 32
where she placed seven (7) months after Item No. 8 which called for information regarding "residence in
the constituency where I seek to be elected immediately preceding the election." Again, this original
certificate of candidacy has no evidentiary value because on March 1, 1995 it was corrected by
petitioner. In her Amended/Corrected Certificate of Candidacy, 33 petitioner wrote "since childhood"
after Item No. 8. The amendment of a certificate of candidacy to correct a bona fide mistake has been
allowed by this Court as a matter of course and as a matter of right. As we held in Alialy v. COMELEC , 34
viz.:

"xxx xxx xxx

"The absence of the signature of the Secretary of the local chapter N.P. in the original certificate of
candidacy presented before the deadline September 11, 1959, did not render the certificate invalid. The
amendment of the certificate, although at a date after the deadline, but before the election, was
substantial compliance with the law, and the defect was cured."

It goes without saying that petitioner's erroneous Certificate of Candidacy filed on March 8, 1995 cannot
be used as evidence against her. Private respondent's petition for the disqualification of petitioner
rested alone on these two (2) brittle pieces of documentary evidence — petitioner's Voter's Registration

471
Record and her original Certificate of Candidacy. Ranged against the evidence of the petitioner showing
her ceaseless contacts with Tacloban, private respondent's two (2) pieces of evidence are too
insufficient to disqualify petitioner, more so, to deny her the right to represent the people of the First
District of Leyte who have overwhelmingly voted for her.

Fifth. Section 10, Article IX-C of the Constitution mandates that "bona fide candidates for any public
office shall be free from any form of harassment and discrimination." 35 A detached reading of the
records of the case at bench will show that all forms of legal and extra-legal obstacles have been thrown
against petitioner to prevent her from running as the people's representative in the First District of
Leyte. In petitioner's Answer to the petition to disqualify her, she averred: 36

xxx xxx xxx

"10. Petitioner's (herein private respondent Montejo) motive in filing the instant petition is devious.
When respondent (petitioner herein) announced that she was intending to register as a voter in
Tacloban City and run for Congress in the First District of Leyte, petitioner (Montejo) immediately
opposed her intended registration by writing a letter stating that 'she is not a resident of said city but of
Barangay Olot, Tolosa, Leyte.' (Annex "2" of respondent's affidavit, Annex "2"). After respondent
(petitioner herein) had registered as a voter in Tolosa following completion of her six-month actual
residence therein, petitioner (Montejo) filed a petition with the COMELEC to transfer the town of Tolosa
from the First District to the Second District and pursued such move up to the Supreme Court in G.R. No.
118702, his purpose being to remove respondent (petitioner herein) as petitioner's (Montejo's)
opponent in the congressional election in the First District. He also filed a bill, along with other Leyte
Congressmen, seeking to create another legislative district, to remove the town of Tolosa out of the First
District and to make it a part of the new district, to achieve his purpose. However, such bill did not pass
the Senate. Having failed on such moves, petitioner now filed the instant petition, for the same
objective, as it is obvious that he is afraid to submit himself along with respondent (petitioner herein)
for the judgment and verdict of the electorate of the First District of Leyte in an honest, orderly,
peaceful, free and clean elections on May 8, 1995."

These allegations which private respondent did not challenge were not lost to the perceptive eye of
Commissioner Maambong who in his Dissenting Opinion, 37 held:

"xxx xxx xxx

"Prior to the registration date — January 28, 1995 — the petitioner (herein private respondent Montejo)
wrote the Election Officer of Tacloban City not to allow respondent (petitioner herein) to register
thereat since she is a resident of Tolosa and not Tacloban City. The purpose of this move of the
petitioner (Montejo) is not lost to (sic) the Commission. In UND No. 95-001 (In the matter of the
Legislative Districts of the Provinces of Leyte, Iloilo, and South Cotabato, Out of Which the New
Provinces of Biliran, Guimaras and Saranggani Were Respectively Created), . . . Hon. Cirilo Roy G.
Montejo, Representative, First District of Leyte, wanted the Municipality of Tolosa, in the First District of
Leyte, transferred to the Second District of Leyte. The Hon. Sergio A.F. Apostol, Representative of the
Second District of Leyte, opposed the move of the petitioner (Montejo). Under Comelec Resolution No.

472
2736 (December 29, 1994), the Commission on Elections refused to make the proposed transfer.
Petitioner (Montejo) filed 'Motion for Reconsideration of Resolution No. 2736' which the Commission
denied in a Resolution promulgated on February 1, 1995. Petitioner (Montejo) filed a petition for
certiorari before the Honorable Supreme Court (Cirilo Roy G. Montejo vs. Commission on Elections, G.R.
No. 118702) questioning the resolution of the Commission. Believing that he could get a favorable ruling
from the Supreme Court, petitioner (Montejo) tried to make sure that the respondent (petitioner
herein) will register as a voter in Tolosa so that she will be forced to run as Representative not in the
First but in the Second District.

"It did not happen. On March 16, 1995, the Honorable Supreme Court unanimously promulgated a
'Decision,' penned by Associate Justice Reynato S. Puno, the dispositive portion of which reads:

'IN VIEW WHEREOF, Section 1 of Resolution No. 2736 insofar as it transferred the municipality of
Capoocan of the Second District and the municipality of Palompon of the Fourth District to the Third
District of the province of Leyte, is annulled and set aside. We also deny the Petition praying for the
transfer of the municipality of Tolosa from the First District to the Second District of the province of
Leyte. No costs.'

"Petitioner's (Montejo's) plan did not work. But the respondent (petitioner herein) was constrained to
register in the Municipality of Tolosa where her house is instead of Tacloban City, her domicile. In any
case, both Tacloban City and Tolosa are in the same First Legislative District."

All these attempts to misuse our laws and legal processes are forms of rank harassments and invidious
discriminations against petitioner to deny her equal access to a public office. We cannot commit any
hermeneutic violence to the Constitution by torturing the meaning of equality, the end result of which
will allow the harassment and discrimination of petitioner who has lived a controversial life, a past of
alternating light and shadow. There is but one Constitution for all Filipinos. Petitioner cannot be
adjudged by a "different" Constitution, and the worst way to interpret the Constitution is to inject in its
interpretation, bile and bitterness.

Sixth. In Gallego v. Vera, 38 we explained that the reason for this residence requirement is "to exclude a
stranger or newcomer, unacquainted with the conditions and needs of a community and not identified
with the latter, from an elective office to serve that community. . . ." Petitioner's lifetime contacts with
the First District of Leyte cannot be contested. Nobody can claim that she is not acquainted with its
problems because she is a stranger to the place. None can argue she cannot satisfy the intent of the
Constitution.

Seventh. In resolving election cases, a dominant consideration is the need to effectuate the will of the
electorate. The election results show that petitioner received Seventy Thousand Four Hundred Seventy-
One (70,471) votes, while private respondent got only Thirty-Six Thousand Eight Hundred Thirty-Three
(36,833) votes. Petitioner is clearly the overwhelming choice of the electorate of the First District of
Leyte and this is not a sleight of statistics. We cannot frustrate this sovereign will on highly arguable
technical considerations. In case of doubt, we should lean towards a rule that will give life to the
people's political judgment.

473
A final point. The case at bench provides the Court with the rare opportunity to rectify the inequality of
status between women and men by rejecting the iniquitous common law precedents on the domicile of
married women and by redefining domicile in accord with our own culture, law, and Constitution. To
rule that a married woman is eternally tethered to the domicile dictated by her dead husband is to
preserve the anachronistic and anomalous balance of advantage of a husband over his wife. We should
not allow the dead to govern the living even if the glories of yesteryears seduce us to shout long live the
dead! The Family Code buried this gender-based discrimination against married women and we should
not excavate what has been entombed. More importantly, the Constitution forbids it.

I vote to grant the petition.

FRANCISCO, J ., concurring:

I concur with Mr. Justice Kapunan's ponencia finding petitioner qualified for the position of
Representative of the First Congressional District of Leyte. I wish, however, to express a few comments
on the issue of petitioner's domicile.

Domicile has been defined as that place in which a person's habitation is fixed, without any present
intention of removing therefrom, and that place is properly the domicile of a person in which he has
voluntarily fixed his abode, or habitation, not for a mere special or temporary purpose, but with a
present intention of making it his permanent home (28 C.J.S. 1). It denotes a fixed permanent residence
to which when absent for business, or pleasure, or for like reasons one intends to return, and depends
on facts and circumstances, in the sense that they disclose intent. (Ong Huan Tin v. Republic, 19 SCRA
966, 969)

Domicile is classified into domicile of origin and domicile of choice. The law attributes to every individual
a domicile of origin, which is the domicile of his parents, or of the head of his family, or of the person on
whom he is legally dependent at the time of his birth. While the domicile of origin is generally the place
where one is born or reared, it may be elsewhere (28 C.J.S. 5). Domicile of choice, on the other hand, is
the place which the person has elected and chosen for himself to displace his previous domicile; it has
for its true basis or foundation the intention of the person (28 C.J.S. 6). In order to hold that a person
has abandoned his domicile and acquired a new one called domicile of choice, the following requisites
must concur, namely, (a) residence or bodily presence in the new locality, (b) intention to remain there
or animus manendi, and (c) an intention to abandon the old domicile or animus non revertendi
(Romualdez v. RTC , Br. 7, Tacloban City, 226 SCRA 408, 415). A third classification is domicile by
operation of law which attributes to a person a domicile independent of his own intention or actual
residence, ordinarily resulting from legal domestic relations, as that of the wife arising from marriage, or
the relation of a parent and a child (28 C.J.S. 7).

In election law, when our Constitution speaks of residence for election purposes it means domicile (Co v.
Electoral Tribunal of the House of Representatives, 199 SCRA 692, 713; Nuval v. Guray, 52 Phil. 645,
651). To my mind, public respondent Commission on Elections misapplied this concept of domicile which
led to petitioner's disqualification by ruling that petitioner failed to comply with the constitutionally
mandated one-year residence requirement. Apparently, public respondent Commission deemed as

474
conclusive petitioner's stay and registration as voter in many places as conduct disclosing her intent to
abandon her established domicile of origin in Tacloban, Leyte. In several decisions, though, the Court
has laid down the rule that registration of a voter in a place other than his place of origin is not sufficient
to constitute abandonment or loss of such residence (Faypon v. Quirino, 96 Phil. 294, 300). Respondent
Commission offered no cogent reason to depart from this rule except to surmise petitioner's intent of
abandoning her domicile of origin.

It has been suggested that petitioner's domicile of origin was supplanted by a new domicile due to her
marriage, a domicile by operation of law. The proposition is that upon the death of her husband in 1989
she retains her husband's domicile, i.e., Batac, Ilocos Norte, until she makes an actual change thereof. I
find this proposition quite untenable.

Tacloban, Leyte, is petitioner's domicile of origin which was involuntarily supplanted with another, i.e.,
Batac, Ilocos Norte, upon her marriage in 1954 with then Congressman Marcos. By legal fiction she
followed the domicile of her husband. In my view, the reason for the law is for the spouses to fully and
effectively perform their marital duties and obligations to one another. 1 The question of domicile,
however, is not affected by the fact that it was the legal or moral duty of the individual to reside in a
given place (28 C.J.S. 11). Thus, while the wife retains her marital domicile so long as the marriage
subsists, she automatically loses it upon the latter's termination, for the reason behind the law then
ceases. Otherwise, petitioner, after her marriage was ended by the death of her husband, would be
placed in a quite absurd and unfair situation of having been freed from all wifely obligations yet made to
hold on to one which no longer serves any meaningful purpose. CDta

It is my view therefore that petitioner reverted to her original domicile of Tacloban, Leyte upon her
husband's death without even signifying her intention to that effect. It is for the private respondent to
prove, not for petitioner to disprove, that petitioner has effectively abandoned Tacloban, Leyte for
Batac, Ilocos Norte or for some other place/s. The clear rule is that it is the party (herein private
respondent) claiming that a person has abandoned or lost his residence of origin who must show and
prove preponderantly such abandonment or loss (Faypon v. Quirino, supra at 298; 28 C.J.S. 16), because
the presumption is strongly in favor of an original or former domicile, as against an acquired one (28
C.J.S. 16). Private respondent unfortunately failed to discharge this burden as the record is devoid of
convincing proof that petitioner has acquired, whether voluntarily or involuntarily, a new domicile to
replace her domicile of origin.

The records, on the contrary, clearly show that petitioner has complied with the constitutional one-year
residence requirement. After her exile abroad, she returned to the Philippines in 1991 to reside in Olot,
Tolosa, Leyte, but the Presidential Commission on Good Government which sequestered her residential
house and other properties forbade her necessitating her transient stay in various places in Manila
(Affidavit p. 6, attached as Annex I of the Petition). In 1992, she ran for the position of president writing
in her certificate of candidacy her residence as San Juan, Metro Manila. After her loss therein, she went
back to Tacloban City, acquired her residence certificate 2 and resided with her brother in San Jose. She
resided in San Jose, Tacloban City until August of 1994 when she was allowed by the PCGG to move and
reside in her sequestered residential house in Olot, Tolosa, Leyte (Annex I, p. 6). 3 It was in the same

475
month of August when she applied for the cancellation of her previous registration in San Juan, Metro
Manila in order to register anew as voter of Olot, Tolosa, Leyte, which she did on January 28, 1995. From
this sequence of events, I find it quite improper to use as the reckoning period of the one-year residence
requirement the date when she applied for the cancellation of her previous registration in San Juan,
Metro Manila. The fact which private respondent never bothered to disprove is that petitioner
transferred her residence after the 1992 presidential election from San Juan, Metro Manila to San Jose,
Tacloban City, and resided therein until August of 1994. She later transferred to Olot, Tolosa, Leyte
(Annex I, p. 7). It appearing that both Tacloban City and Tolosa, Leyte are within the First Congressional
District of Leyte, it indubitably stands that she had more than a year of residence in the constituency she
sought to be elected. Petitioner, therefore, has satisfactorily complied with the one-year qualification
required by the 1987 Constitution.

I vote to grant the petition. dctai

PADILLA, J ., dissenting:

I regret that I cannot join the majority opinion as expressed in the well-written ponencia of Mr. Justice
Kapunan.

As in any controversy arising out of a Constitutional provision, the inquiry must begin and end with the
provision itself. The controversy should not be blurred by what, to me, are academic disquisitions. In this
particular controversy, the Constitutional provision on point states that — "no person shall be a member
of the House of Representatives unless he is a natural-born citizen of the Philippines, and on the day of
the election, is at least twenty-five (25) years of age, able to read and write, and except the party list
representatives, a registered voter in the district in which he shall be elected, and a resident thereof for
a period of not less than one year immediately preceding the day of the election." (Article VI, Section 6)

It has been argued that for purposes of our election laws, the term residence has been understood as
synonymous with domicile. This argument has been validated by no less than the Court in numerous
cases 1 where significantly the factual circumstances clearly and convincingly proved that a person does
not effectively lose his domicile of origin if the intention to reside therein is manifest with his personal
presence in the place, coupled with conduct indicative of such intention.

With this basic thesis in mind, it would not be difficult to conceive of different modalities within which
the phrase "a resident thereof (meaning, the legislative district) for a period of not less than one year"
would fit.

The first instance is where a person's residence and domicile coincide in which case a person only has to
prove that he has been domiciled in a permanent location for not less than a year before the election.

A second situation is where a person maintains a residence apart from his domicile in which case he
would have the luxury of district shopping, provided of course, he satisfies the one-year residence
period in the district as the minimum period for eligibility to the position of congressional representative
for the district.

476
In either case, one would not be constitutionally disqualified for abandoning his residence in order to
return to his domicile of origin, or better still, domicile of choice; neither would one be disqualified for
abandoning altogether his domicile in favor of his residence in the district where he desires to be a
candidate.

The most extreme circumstance would be a situation wherein a person maintains several residences in
different districts. Since his domicile of origin continues as an option as long as there is no effective
abandonment (animus non revertendi), he can practically choose the district most advantageous for
him.

All these theoretical scenarios, however, are tempered by the unambiguous limitation that "for a period
of not less than one year immediately preceding the day of the election," he must be a resident in the
district where he desires to be elected.

To my mind, the one year residence period is crucial regardless of whether or not the term "residence"
is to be synonymous with "domicile." In other words, the candidate's intent and actual presence in one
district must in all situations satisfy the length of time prescribed by the fundamental law. And this,
because of a definite Constitutional purpose. He must be familiar with the environment and problems of
a district he intends to represent in Congress and the one-year residence in said district would be the
minimum period to acquire such familiarity, if not versatility. cdll

In the case of petitioner Imelda R. Marcos, the operative facts are distinctly set out in the now assailed
decision of the Comelec 2nd Division dated 24 April 1995 (as affirmed by the Comelec en banc) —

"In or about 1938 when respondent was a little over 8 years old, she established her domicile in
Tacloban, Leyte (Tacloban City). She studied in the Holy Infant Academy in Tacloban from 1938 to 1948
when she graduated from high school. She pursued her college studies in St. Paul's College, now Divine
Word University of Tacloban, where she earned her degree in Education. Thereafter, she taught in the
Leyte Chinese High School, still in Tacloban City. In 1952 she went to Manila to work with her cousin, the
late Speaker Daniel Z. Romualdez in his office in the House of Representatives. In 1954, she married ex-
President Ferdinand Marcos when he was still a congressman of Ilocos Norte. She lived with him in
Batac, Ilocos Norte and registered there as a voter. When her husband was elected Senator of the
Republic in 1959, she and her husband lived together in San Juan, Rizal where she registered as a voter.
In 1965 when her husband was elected President of the Republic of the Philippines, she lived with him in
Malacañang Palace and registered as a voter in San Miguel, Manila.

"During the Marcos presidency, respondent served as a Member of the Batasang Pambansa, Minister of
Human Settlements and Governor of Metro Manila. She claimed that in February 1986, she and her
family were abducted and kidnapped to Honolulu, Hawaii. In November 1991, she came home to
Manila. In 1992 respondent ran for election as President of the Philippines and filed her Certificate of
Candidacy wherein she indicated that she is a resident and registered voter of San Juan, Metro Manila.
On August 24, 1994, respondent filed a letter with the election officer of San Juan, Metro Manila,
requesting for cancellation of her registration in the Permanent List of Voters in Precinct No. 157 of San
Juan, Metro Manila, in order that she may be re-registered or transferred to Brgy. Olot, Tolosa, Leyte.

477
(Annex 2-B, Answer). On August 31, 1994, respondent filed her Sworn Application for Cancellation of
Voter's Previous Registration (Annex 2-C, Answer) stating that she is a duly registered voter in 157-A,
Brgy. Maytunas, San Juan, Metro Manila that she intends to register at Brgy. Olot, Tolosa, Leyte.

"On January 28, 1995 respondent registered as a voter at Precinct No. 18-A of Olot, Tolosa, Leyte. She
filed with the Board of Election Inspectors CE Form No. 1, Voter Registration Record No. 94-3349772,
wherein she alleged that she has resided in the municipality of Tolosa for a period of 6 months (Annex A,
Petition).

"On March 8, 1995, respondent filed with the Office of the Provincial Election Supervisor, Leyte, a
Certificate of Candidacy for the position of Representative of the First District of Leyte wherein she also
alleged that she has been a resident in the constituency where she seeks to be elected for a period of 7
months. The pertinent entries therein are as follows:

7. PROFESSION OR OCCUPATION: Housewife/Teacher/Social Worker

8. RESIDENCE (complete address): Brgy. Olot, Tolosa, Leyte

Post Office Address for election purposes: Brgy. Olot, Tolosa, Leyte

9. RESIDENCE IN THE CONSTITUENCY WHEREIN I SEEK TO BE ELECTED IMMEDIATELY PRECEDING


ELECTION: _______ Years Seven Months

10. I AM NOT A PERMANENT RESIDENT OF, OR IMMIGRANT TO, A FOREIGN COUNTRY.

THAT I AM ELIGIBLE for said office; That I will support and defend the Constitution of the Republic of the
Philippines and will maintain true faith and allegiance thereto; That I will obey the laws, legal orders and
decrees promulgated by the duly-constituted authorities; That the obligation imposed by my oath is
assumed voluntarily, without mental reservation or purpose of evasion; and That the facts stated herein
are true to the best of my knowledge.

(Sgd.) Imelda Romualdez-Marcos

(Signature of Candidate)" 2

Petitioner's aforestated certificate of candidacy filed on 8 March 1995 contains the decisive component
or seed of her disqualification. It is contained in her answer under oath of "seven months" to the query
of "residence in the constituency wherein I seek to be elected immediately preceding the election."

It follows from all the above that the Comelec committed no grave abuse of discretion in holding that
petitioner is disqualified from the position of representative for the 1st congressional district of Leyte in
the elections of 8 May 1995, for failure to meet the "not less than one-year residence in the
constituency (1st district, Leyte) immediately preceding the day of election (8 May 1995)."

Having arrived at petitioner's disqualification to be a representative of the first district of Leyte, the next
important issue to resolve is whether or not the Comelec can order the Board of Canvassers to

478
determine and proclaim the winner out of the remaining qualified candidates for representative in said
district.

I am not unaware of the pronouncement made by this Court in the case of Labo vs. Comelec, G.R.
86564, August 1, 1989, 176 SCRA 1 which gave the rationale as laid down in the early 1912 case of
Topacio vs. Paredes, 23 Phil. 238 that:

". . . Sound policy dictates that public elective offices are filled by those who have received the highest
number of votes cast in the election for that office, and it is a fundamental idea in all republican forms of
government that no one can be declared elected and no measure can be declared carried unless he or it
receives a majority or plurality of the legal votes cast in the election. (20 Corpus Juris 2nd, S 243, p. 676)

The fact that the candidate who obtained the highest number of votes is later declared to be
disqualified or not eligible for the office to which he was elected does not necessarily entitle the
candidate who obtained the second highest number of votes to be declared the winner of the elective
office. The votes cast for a dead, disqualified, or non-eligible person may not be valid to vote the winner
into office or maintain him there. However, in the absence of a statute which clearly asserts a contrary
political and legislative policy on the matter, if the votes were cast in the sincere belief that the
candidate was alive, qualified, or eligible, they should not be treated as stray, void or meaningless."

Under Sec. 6 RA 6646, (An Act Introducing Additional Reforms in the Electoral System and for other
purposes) (84 O.G. 905, 22 February 1988) it is provided that:

". . . Any candidate who has been declared by final judgment to be disqualified shall not be voted for,
and the votes cast for him shall not be counted. If for any reason a candidate is not declared by final
judgment before an election to be disqualified and he is voted for and receives the winning number of
votes in such election, the Court or Commission shall continue with the trial and hearing of the action,
inquiry or protest and, upon motion of the complainant or any intervenor, may, during the pendency
thereof order the suspension of the proclamation of such candidate whenever the evidence of his guilt
is strong."

There is no need to indulge in legal hermeneutics to sense the plain and unambiguous meaning of the
provision quoted above. As the law now stands, the legislative policy does not limit its concern with the
effect of a final judgment of disqualification only before the election, but even during or after the
election. The law is clear that in all situations, the votes cast for a disqualified candidate SHALL NOT BE
COUNTED. The law has also validated the jurisdiction of the Court or Commission on Election to
continue hearing the petition for disqualification in case a candidate is voted for and receives the
highest number of votes, if for any reason, he is not declared by final judgment before an election to be
disqualified. cdasia

Since the present case is an after election scenario, the power to suspend proclamation (when evidence
of his guilt is strong) is also explicit under the law. What happens then when after the elections are over,
one is declared disqualified? Then, votes cast for him "shall not be counted" and in legal contemplation,
he no longer received the highest number of votes.

479
It stands to reason that Section 6 of RA 6646 does not make the second placer the winner simply
because a "winning candidate is disqualified," but that the law considers him as the candidate who had
obtained the highest number of votes as a result of the votes cast for the disqualified candidate not
being counted or considered,

As this law clearly reflects the legislative policy on the matter, then there is no reason why this Court
should not re-examine and consequently abandon the doctrine in the Jun Labo case. It has been stated
that "the qualifications prescribed for elective office cannot be erased by the electorate alone. The will
of the people as expressed through the ballot cannot cure the vice of ineligibility" most especially when
it is mandated by no less than the Constitution.

ACCORDINGLY, I vote to DISMISS the petition and to order the Provincial Board of Canvassers of Leyte to
proclaim the candidate receiving the highest number of votes, from among the qualified candidates, as
the duly elected representative of the 1st district of Leyte.

REGALADO, J ., dissenting:

While I agree with some of the factual bases of the majority opinion, I cannot arrive conjointly at the
same conclusion drawn therefrom. Hence, this dissent which assuredly is not formulated "on the basis
of the personality of a petitioner in a case."

I go along with the majority in their narration of antecedent facts, insofar as the same are pertinent to
this case, and which I have simplified as follows:

1. Petitioner, although born in Manila, resided during her childhood in the present Tacloban City,
she being a legitimate daughter of parents who appear to have taken up permanent residence therein.
She also went to school there and, for a time, taught in one of the schools in that city.

2. When she married then Rep. Ferdinand E. Marcos who was then domiciled in Batac, Ilocos
Norte, by operation of law she acquired a new domicile in that place in 1954.

3. In the successive years and during the events that happened thereafter, her husband having
been elected as a Senator and then as President, she lived with him and their family in San Juan, Rizal
and then in Malacañang Palace in San Miguel, Manila.

4. Over those years, she registered as a voter and actually voted in Batac, Ilocos Norte, then in San
Juan, Rizal, and also in San Miguel, Manila, all these merely in the exercise of the right of suffrage.

5. It does not appear that her husband, even after he had assumed those lofty positions
successively, ever abandoned his domicile of origin in Batac, Ilocos Norte where he maintained his
residence and invariably voted in all elections.

6. After the ouster of her husband from the presidency in 1986 and the sojourn of the Marcos
family in Honolulu, Hawaii, U.S.A., she eventually returned to the Philippines in 1991 and resided in
different places which she claimed to have been merely temporary residences.

480
7. In 1992, petitioner ran for election as President of the Philippines and in her certificate of
candidacy she indicated that she was then a registered voter and resident of San Juan, Metro Manila.

8. On August 24, 1994, she filed a letter for the cancellation of her registration in the Permanent
List of Voters in Precinct No. 157 of San Juan, Metro Manila in order that she may "be re-registered or
transferred to Brgy. Olot, Tolosa, Leyte." On August 31, 1994, she followed this up with her Sworn
Application for Cancellation of Voter's Previous Registration wherein she stated that she was a
registered voter in Precinct No. 157-A, Brgy. Maytunas, San Juan, Metro Manila and that she intended to
register in Brgy. Olot, Tolosa, Leyte.

9. On January 28, 1995, petitioner registered as a voter at Precinct No. 18-A of Olot, Tolosa, Leyte,
for which purpose she filed with the therein Board of Election Inspectors a voter's registration record
form alleging that she had resided in that municipality for six months.

10. On March 8, 1995, petitioner filed her certificate of candidacy for the position of Representative
of the First District of Leyte wherein she alleged that she had been a resident for "Seven Months" of the
constituency where she sought to be elected.

11. On March 29, 1995, she filed an "Amended/Corrected Certificate of Candidacy" wherein her
answer in the original certificate of candidacy to item "8. RESIDENCE IN THE CONSTITUENCY WHERE I
SEEK, TO BE ELECTED IMMEDIATELY PRECEDING THE ELECTION:" was changed or replaced with a new
entry reading "SINCE CHILDHOOD."

The sole issue for resolution is whether, for purposes of her candidacy, petitioner had complied with the
residency requirement of one year as mandated by no less than Section 6, Article VI of the 1987
Constitution.

I do not intend to impose upon the time of my colleagues with a dissertation on the difference between
residence and domicile. We have had enough of that and I understand that for purposes of political law
and, for that matter of international law, residence is understood to be synonymous with domicile. That
is so understood in our jurisprudence and in American Law, in contradistinction to the concept of
residence for purposes of civil, commercial and procedural laws whenever an issue thereon is relevant
or controlling.

Consequently, since in the present case the question of petitioner's residence is integrated in and
inseparable from her domicile, I am addressing the issue from the standpoint of the concept of the latter
term, specifically its permutations into the domicile of origin, domicile of choice and domicile by
operation of law, as understood in American law from which for this case we have taken our
jurisprudential bearings.

My readings inform me that the domicile of the parents at the time of birth, or what is termed the
"domicile of origin," constitutes the domicile of an infant until abandoned, or until the acquisition of a
new domicile in a different place. 1 In the instant case, we may grant that petitioner's domicile of origin,
2 at least as of 1938, was what is now Tacloban City.

481
Now, as I have observed earlier, domicile is said to be of three kinds, that is, domicile by birth, domicile
by choice, and domicile by operation of law. The first is the common case of the place of birth or
domicilium originis; the second is that which is voluntarily acquired by a party or domicilium proprio
motu; the last which is consequential, as that of a wife arising from marriage, 3 is sometimes called
domicilium necesarium. There is no debate that the domicile of origin can be lost or replaced by a
domicile of choice or a domicile by operation of law subsequently acquired by the party.

When petitioner contracted marriage in 1954 with then Rep. Marcos, by operation of law, not only
international or American but of our own enactment, 4 she acquired her husband's domicile of origin in
Batac, Ilocos Norte and correspondingly lost her own domicile of origin in Tacloban City.

Her subsequent changes of residence — to San Juan, Rizal, then to San Miguel, Manila, thereafter to
Honolulu, Hawaii, and back to now San Juan, Metro Manila — do not appear to have resulted in her
thereby acquiring new domiciles of choice. In fact, it appears that her having resided in those places was
by reason of the fortunes or misfortunes of her husband and his peregrinations in the assumption of
new official positions or the loss of them. Her residence in Honolulu and, of course, those after her
return to the Philippines were, as she claimed, against her will or only for transient purposes which
could not have invested them with the status of domiciles of choice. 5

After petitioner's return to the Philippines in 1991 and up to the present imbroglio over her requisite
residency in Tacloban City or Olot, Tolosa, Leyte, there is no showing that she ever attempted to acquire
any other domicile of choice which could have resulted in the abandonment of her legal domicile in
Batac, Ilocos Norte. On that score, we note the majority's own submission 6 that, to successfully effect a
change of domicile, one must demonstrate (a) an actual removal or an actual change of domicile, (b) a
bona fide intention of abandoning the former place of residence and establishing a new one, and (c) acts
which correspond with the purpose.

We consequently have to also note that these requirements for the acquisition of a domicile of choice
apply whether what is sought to be changed or substituted is a domicile of origin (domicilium originis) or
a domicile by operation of law (domicilium necesarium). Since petitioner had lost her domicilium originis
which had been replaced by her domicilium necesarium, it is therefore her continuing domicile in Batac,
Ilocos Norte which, if at all, can be the object of legal change under the contingencies of the case at bar.

To get out of this quandary, the majority decision echoes the dissenting opinion of Commissioner
Regalado E. Maambong in SPA 95-009 of the Commission on Elections, 7 and advances this novel
proposition:

"It may be said that petitioner lost her domicile of origin by operation of law as a result of her marriage
to the late President Ferdinand E. Marcos in 1952 (sic, 1954). By operation of law (domicilium
necesarium), her legal domicile at the time of her marriage became Batac, Ilocos Norte although there
were no indications of an intention on her part to abandon her domicile of origin. Because of her
husband's subsequent death and through the operation of the provisions of the New Family Code
already in force at the time, however, her legal domicile automatically reverted to her domicile of origin.
. . ." (Emphasis supplied).

482
Firstly, I am puzzled why although it is conceded that petitioner had acquired a domicilium necesarium
in Batac, Ilocos Norte, the majority insists on making a qualification that she did not intend to abandon
her domicile of origin. I find this bewildering since, in this situation, it is the law that declares where
petitioner's domicile is at any given time, and not her self-serving or putative intent to hold on to her
former domicile. Otherwise, contrary to their own admission that one cannot have more than one
domicile at a time, 8 the majority would be suggesting that petitioner retained Tacloban City as (for lack
of a term in law since it does not exist therein) the equivalent of what is fancied as a reserved, dormant,
potential, or residual domicile. prLL

Secondly, domicile once lost in accordance with law can only be recovered likewise in accordance with
law. However, we are here being titillated with the possibility of an automatic reversion to or
reacquisition of a domicile of origin after the termination of the cause for its loss by operation of law.
The majority agrees that since petitioner lost her domicile of origin by her marriage, the termination of
the marriage also terminates that effect thereof. I am impressed by the ingeniousness of this theory
which proves that, indeed, necessity is the mother of inventions. Regretfully, I find some difficulty in
accepting either the logic or the validity of this argument.

If a party loses his domicile of origin by obtaining a new domicile of choice, he thereby voluntarily
abandons the former in favor of the latter. If, thereafter, he abandons that chosen domicile, he does not
per se recover his original domicile unless, by subsequent acts legally indicative thereof, he evinces his
intent and desire to establish the same as his new domicile, which is precisely what petitioner belatedly
and, evidently just for purposes of her candidacy, unsuccessfully tried to do.

One's subsequent abandonment of his domicile of choice cannot automatically restore his domicile of
origin, not only because there is no legal authority therefor but because it would be absurd. Pursued to
its logical consequence, that theory of ipso jure reversion would rule out the fact that said party could
already very well have obtained another domicile, either of choice or by operation of law, other than his
domicile of origin. Significantly and obviously for this reason, the Family Code, which the majority
inexplicably invokes, advisedly does not regulate this contingency since it would impinge on one's
freedom of choice.

Now, in the instant case, petitioner not only voluntarily abandoned her domicile of choice (unless we
assume that she entered into the marital state against her will) but, on top of that, such abandonment
was further affirmed through her acquisition of a new domicile by operation of law. In fact, this is even a
case of both voluntary and legal abandonment of a domicile of origin. With much more reason,
therefore, should we reject the proposition that with the termination of her marriage in 1989, petitioner
had supposedly per se and ipso facto reacquired her domicile of origin which she lost in 1954.
Otherwise, this would be tantamount to saying that during the period of marital coverture, she was
simultaneously in possession and enjoyment of a domicile of origin which was only in a state of
suspended animation.

Thus, the American rule is likewise to the effect that while after the husband's death the wife has the
right to elect her own domicile, 9 she nevertheless retains the last domicile of her deceased husband

483
until she makes an actual change. 10 In the absence of affirmative evidence, to the contrary, the
presumption is that a wife's domicile or legal residence follows that of her husband and will continue
after his death. 11

I cannot appreciate the premises advanced in support of the majority's theory based on Articles 68 and
69 of the Family Code. All that is of any relevance therein is that under this new code, the right and
power to fix the family domicile is now shared by the spouses. I cannot perceive how that joint right,
which in the first place was never exercised by the spouses, could affect the domicile fixed by the law for
petitioner in 1954 and, for her husband, long prior thereto. It is true that a wife now has the coordinate
power to determine the conjugal or family domicile, but that has no bearing on this case. With the death
of her husband, and each of her children having gotten married and established their own respective
domiciles, the exercise of that joint power was and is no longer called for or material in the present
factual setting of this controversy. Instead, what is of concern in petitioner's case was the matter of her
having acquired or not her own domicile of choice. cdll

I agree with the majority's discourse on the virtues of the growing and expanded participation of women
in the affairs of the nation, with equal rights and recognition by Constitution and statutory conferment.
However, I have searched in vain for a specific law or judicial pronouncement which either expressly or
by necessary implication supports the majority's desired theory of automatic reacquisition of or
reversion to the domicilium originis of petitioner. Definitely, as between the settled and desirable legal
norms that should govern this issue, there is a world of difference; and, unquestionably, this should be
resolved by legislative articulation but not by the eloquence of the well-turned phrase.

In sum, petitioner having lost Tacloban City as her domicile of origin since 1954 and not having
automatically reacquired any domicile therein, she cannot legally claim that her residency in the political
constituency of which it is a part continued since her birth up to the present. Respondent commission
was, therefore, correct in rejecting her pretension to that effect in her amended/corrected certificate of
candidacy, and in holding her to her admission in the original certificate that she had actually resided in
that constituency for only seven months prior to the election. These considerations render it
unnecessary to further pass upon the procedural issues raised by petitioner.

ON THE FOREGOING PREMISES, I vote to DISMISS the petition for lack of merit. cdlex

DAVIDE, JR., J ., dissenting:

I respectfully dissent from the opinion of the majority written by Mr. Justice Santiago M. Kapunan, more
particularly on the issue of the petitioner's qualification.

Under Section 7, Subdivision A, Article IX of the Constitution, decisions, orders, or rulings of the
COMELEC may be brought to this Court only by the special civil action for certiorari under Rule 65 of the
Rules of Court (Aratuc vs. COMELEC, 88 SCRA 251 [1979]; Dario vs. Mison, 176 SCRA 84 [1989]).

Accordingly, a writ of certiorari may be granted only if the COMELEC has acted without or in excess of
jurisdiction or with grave abuse of discretion (Section 1, Rule 65, Rules of Court). Since the COMELEC

484
has, undoubtedly, jurisdiction over the private respondent's petition, the only issue left is whether it
acted with grave abuse of discretion in disqualifying the petitioner.

My careful and meticulous perusal of the challenged resolution of 24 April 1995 of the COMELEC Second
Division and the En Banc resolution of 7 May 1995 discloses total absence of abuse of discretion, much
less grave abuse thereof. The resolution of the Second Division dispassionately and objectively discussed
in minute details the facts which established beyond cavil that herein petitioner was disqualified as a
candidate on the ground of lack of residence in the First Congressional District of Leyte. It has not
misapplied, miscomprehended, or misunderstood facts or circumstances of substance pertinent to the
issue of her residence. LexLib

The majority opinion, however, overturned the COMELEC's findings of fact for lack of proof that the
petitioner has abandoned Tolosa as her domicile of origin, which is allegedly within the First
Congressional District of Leyte.

I respectfully submit that the petitioner herself has provided the COMELEC, either by admission or by
documentary evidence, overwhelming proof of the loss or abandonment of her domicile of origin, which
is Tacloban City and not Tolosa, Leyte. Assuming that she decided to live again in her domicile of origin,
that became her second domicile of choice, where her stay, unfortunately, was for only seven months
before the day of the election. She was then disqualified to be a candidate for the position of
Representative of the First Congressional District of Leyte. A holding to the contrary would be arbitrary.

It may indeed be conceded that the petitioner's domicile of choice was either Tacloban City or Tolosa,
Leyte. Nevertheless, she lost it by operation of law sometime in May 1954 upon her marriage to the
then Congressman (later, President) Ferdinand E. Marcos. A domicile by operation of law is that domicile
which the law attributes to a person, independently of his own intention or actual residence, as results
from legal domestic relations as that of the wife arising from marriage (28 C.J.S. Domicile § 7, 11). Under
the governing law then, Article 110 of the Civil Code, her new domicile or her domicile of choice was the
domicile of her husband, which was Batac, Ilocos Norte. Said Article reads as follows:

ARTICLE 110. The husband shall fix the residence of the family. But the court may exempt the wife
from living with the husband if he should live abroad unless in the service of the Republic.

Commenting thereon, civilist Arturo M. Tolentino states:

Although the duty of the spouses to live together is mutual, the husband has a predominant right
because he is empowered by law to fix the family residence. This right even predominates over some
rights recognized by law in the wife. For instance, under Article 117 the wife may engage in business or
practice a profession or occupation. But because of the power of the husband to fix the family domicile,
he may fix it at such a place as would make it impossible for the wife to continue in business or in her
profession. For justifiable reasons, however, the wife may be exempted from living in the residence
chosen by the husband. The husband cannot validly allege desertion by the wife who refuses to follow
him to a new place of residence, when it appears that they have lived for years in a suitable home

485
belonging to the wife, and that his choice of a different home is not made in good faith. (Commentaries
and Jurisprudence on the Civil Code of the Philippines, Vol. 1, 1985 ed., 339).

Under common law, a woman upon her marriage loses her own domicile and, by operation of law,
acquires that of her husband, no matter where the wife actually lives or what she believes or intends.
Her domicile is fixed in the sense that it is declared to be the same as his, and subject to certain
limitations, he can change her domicile by changing his own (25 Am Jur 2d Domicile § 48, 37).

It must, however, be pointed out that under Article 69 of the Family Code, the fixing of the family
domicile is no longer the sole prerogative of the husband, but is now a joint decision of the spouses, and
in case of disagreement the court shall decide. The said article uses the term "family domicile," and not
family residence, as "the spouses may have multiple residences, and the wife may elect to remain in one
of such residences, which may destroy the duty of the spouses to live together and its corresponding
benefits" (ALICIA V. SEMPIO-DIY, Handbook on the Family Code of the Philippines, [1988], 102).

The theory of automatic restoration of a woman's domicile of origin upon the death of her husband,
which the majority opinion adopts to overcome the legal effect of the petitioner's marriage on her
domicile, is unsupported by law and by jurisprudence. The settled doctrine is that after the husband's
death the wife has a right to elect her own domicile, but she retains the last domicile of her husband
until she makes an actual change (28 C.J.S. Domicile § 12, 27). Or, on the death of the husband, the
power of the wife to acquire her own domicile is revived, but until she exercises the power her domicile
remains that of the husband at the time of his death (25 Am Jur 2d Domicile § 62, 45). Note that what is
revived is not her domicile of origin but her power to acquire her own domicile.

Clearly, even after the death of her husband, the petitioner's domicile was that of her husband at the
time of his death — which was Batac, Ilocos Norte, since their residences in San Juan, Metro Manila, and
San Miguel, Manila, were their residences for convenience to enable her husband to effectively perform
his official duties. Their residence in San Juan was a conjugal home, and it was there to which she
returned in 1991 when she was already a widow. In her sworn certificate of candidacy for the Office of
the President in the synchronized elections of May 1992, she indicated therein that she was a resident
of San Juan, Metro Manila. She also voted in the said elections in that place.

On the basis of her evidence, it was only on 24 August 1994 when she exercised her right as a widow to
acquire her own domicile in Tolosa, Leyte, through her sworn statement requesting the Election Officer
of San Juan, Metro Manila, to cancel her registration in the permanent list of voters in Precinct 157
thereat and praying that she be "re-registered or transferred to Brgy. Olot, Tolosa, Leyte, the place of
[her] birth and permanent residence" (photocopy of Exhibit "B", attached as Annex "2" of private
respondent Montejo's Comment). Notably, she contradicted this sworn statement regarding her place
of birth when, in her Voter's Affidavit sworn to on 15 March 1992 (photocopy of Exhibit "C", attached as
Annex "3", Id.), her Voter Registration Record sworn to on 28 January 1995 (photocopy of Exhibit "E",
attached as Annex "5", Id.), and her Certificate of Candidacy sworn to on 8 March 1995 (photocopy of
Exhibit "A", attached as Annex "1", Id.), she solemnly declared that she was born in Manila.

486
The petitioner is even uncertain as to her domicile of origin. Is it Tacloban City or Tolosa, Leyte? In the
affidavit attached to her Answer to the petition for disqualification (Annex "I" of Petition), she declared
under oath that her "domicile or residence is Tacloban City." If she did intend to return to such domicile
or residence of origin why did she inform the Election Officer of San Juan that she would transfer to
Olot, Tolosa, Leyte, and indicate in her Voter's Registration Record and in her certificate of candidacy
that her residence is Olot, Tolosa, Leyte? While this uncertainty is not important insofar as residence in
the congressional district is concerned, it nevertheless proves that forty-one years had already lapsed
since she had lost or abandoned her domicile of origin by virtue of marriage and that such length of time
diminished her power of recollection or blurred her memory.

I find to be misplaced the reliance by the majority opinion on Faypon vs. Quirino (96 Phil. 294 [1954]),
and the subsequent cases which established the principle that absence from original residence or
domicile of origin to pursue studies, practice one's profession, or engage in business in other states does
not constitute loss of such residence or domicile. So is the reliance on Section 117 of the Omnibus
Election Code which provides that transfer of residence to any other place by reason of one's
"occupation; profession; employment in private and public service; educational activities; work in
military or naval reservations; service in the army, navy or air force, the constabulary or national police
force; or confinement or detention in government institutions in accordance with law" is not deemed as
loss of original residence. Those cases and legal provision do not include marriage of a woman. The
reason for the exclusion is, of course, Article 110 of the Civil Code. If it were the intention of this Court
or of the legislature to consider the marriage of a woman as a circumstance which would not operate as
an abandonment of domicile (of origin or of choice), then such cases and legal provision should have
expressly mentioned the same.

This Court should not accept as gospel truth the self-serving claim of the petitioner in her affidavit
(Annex "A" of her Answer in COMELEC SPA No. 95-009; Annex "I" of Petition) that her "domicile or
residence of origin is Tacloban City," and that she "never intended to abandon this domicile or residence
of origin to which [she] always intended to return whenever absent." Such a claim of intention cannot
prevail over the effect of Article 110 of the Civil Code. Besides, the facts and circumstances or the
vicissitudes of the petitioner's life after her marriage in 1954 conclusively establish that she had indeed
abandoned her domicile of origin and had acquired a new one animo et facto (KOSSUTH KENT KENNAN,
A Treatise on Residence and Domicile, [1934], 214, 326).

Neither should this Court place complete trust on the petitioner's claim that she "merely committed an
honest mistake" in writing down the word "seven" in the space provided for the residency qualification
requirement in the certificate of candidacy. Such a claim is self-serving and, in the light of the foregoing
disquisitions, would be all sound and fury signifying nothing. To me, she did not commit any mistake,
honest or otherwise; what she stated was the truth.

The majority opinion also disregards a basic rule in evidence that he who asserts a fact or the affirmative
of an issue has the burden of proving it (Imperial Victory Shipping Agency vs. NLRC, 200 SCRA 178
[1991]; P.T . Cerna Corp. vs. Court of Appeals, 221 SCRA 19 [1993]). Having admitted marriage to the
then Congressman Marcos, the petitioner could not deny the legal consequence thereof on the change

487
of her domicile to that of her husband. The majority opinion rules or at least concludes that "[b]y
operation of law (domicilium necesarium), her legal domicile at the time of her marriage automatically
became Batac, Ilocos Norte." That conclusion is consistent with Article 110 of the Civil Code. Since she is
presumed to retain her deceased husband's domicile until she exercises her revived power to acquire
her own domicile, the burden is upon her to prove that she has exercised her right to acquire her own
domicile. She miserably failed to discharge that burden.

I vote to deny the petition. LexLibris

ROMERO, J ., separate opinion:

Petitioner has appealed to this Court for relief after the COMELEC ruled that she was disqualified from
running for Representative of her District and that, in the event that she should, nevertheless, muster a
majority vote, her proclamation should be suspended. Not by a straightforward ruling did the COMELEC
pronounce its decision as has been its unvarying practice in the past, but by a startling succession of
"reverse somersaults." Indicative of its shifting stance vis-a-vis petitioner's certificate of candidacy were
first, the action of its Second Division disqualifying her and cancelling her original Certificate of
Candidacy by a vote of 2-1 on April 24, 1995; then the denial by the COMELEC en banc of her Motion for
Reconsideration on May 7, 1995, a day before the election; then because she persisted in running, its
decision on May 11, 1995 or three days after the election, allowing her proclamation in the event that
the results of the canvass should show that she obtained the highest number of votes (obviously noting
that petitioner had won overwhelmingly over her opponent), but almost simultaneously reversing itself
by directing that even if she wins, her proclamation should nonetheless be suspended.

Crucial to the resolution of the disqualification issue presented by the case at bench is the interpretation
to be given to the one-year residency requirement imposed by the Constitution on aspirants for a
Congressional seat. 1

Bearing in mind that the term "resident" has been held to be synonymous with "domicile" for election
purposes, it is important to determine whether petitioner's domicile was in the First District of Leyte and
if so, whether she had resided there for at least a period of one year. Undisputed is her domicile of
origin, Tacloban, where her parents lived at the time of her birth. Depending on what theory one
adopts, the same may have been changed when she married Ferdinand E. Marcos, then domiciled in
Batac, by operation of law. Assuming it did, his death certainly released her from the obligation to live
with him at the residence fixed by him during his lifetime. What may confuse the layman at this point is
the fact that the term "domicile" may refer to "domicile of origin," "domicile of choice," or "domicile by
operation of law," which subject we shall not belabor since it has been amply discussed by the ponente
and in the other separate opinions.

In any case, what assumes relevance is the divergence of legal opinion as to the effect of the husband's
death on the domicile of the widow. Some scholars opine that the widow's domicile remains unchanged;
that the deceased husband's wishes perforce still bind the wife he has left behind. Given this
interpretation, the widow cannot possibly go far enough to sever the domiciliary tie imposed by her
husband.

488
It is bad enough to interpret the law as empowering the husband unilaterally to fix the residence or
domicile of the family, as laid down in the Civil Code, 2 but to continue giving obeisance to his wishes
even after the rationale underlying the mutual duty of the spouses to live together has ceased, is to
close one's eyes to the stark realities of the present.

At the other extreme is the position that the widow automatically reverts to her domicile of origin upon
the demise of her husband. Does the law so abhor a vacuum that the widow has to be endowed
somehow with a domicile? To answer this question which is far from rhetorical, one will have to keep in
mind the basic principles of domicile. Everyone must have a domicile. Then one must have only a single
domicile for the same purpose at any given time. Once established, a domicile remains until a new one
is acquired, for no person lives who has no domiciles, as defined by the law he is subject to.

At this juncture, we are confronted with an unexplored legal terrain in this jurisdiction, rendered more
murky by the conflicting opinions of foreign legal authorities. This being the state of things, it is as
imperative as it is opportune to illumine the darkness with the beacon light of truth, as dictated by
experience and the necessity of according petitioner her right to choose her domicile in keeping with the
enlightened global trend to recognize and protect the human rights of women, no less than men.

Admittedly, the notion of placing women on par with men, insofar as civil, political and social rights are
concerned, is a relatively recent phenomenon that took seed only in the middle of this century. It is a
historical fact that for over three centuries, the Philippines had been colonized by Spain, a conservative,
Catholic country which transplanted to our shores the Old World culture, mores, attitudes and values.
Through the imposition on our government of the Spanish Civil Code in 1889, the people, both men and
women, had no choice but to accept such concepts as the husband's being the head of the family and
the wife's subordination to his authority. In such role, his was the right to make vital decisions for the
family. Many instances easily come to mind, foremost being what is related to the issue before us,
namely, that "the husband shall fix the residence of the family." 3 Because he is made responsible for
the support of the wife and the rest of the family, 4 he is also empowered to be the administrator of the
conjugal property, with a few exceptions 5 and may, therefore, dispose of the conjugal partnership
property for purposes specified under the law; 6 whereas, as a general rule, the wife cannot bind the
conjugal partnership without the husband's consent. 7 As regards the property pertaining to the
children under parental authority, the father is the legal administrator and only in his absence may the
mother assume his powers. 8 Demeaning to the wife's dignity are certain strictures on her personal
freedoms, practically relegating her to the position of minors and disabled persons. To illustrate a few:
The wife cannot, without the husband's consent, acquire any property by gratuitous title, except from
her ascendants, descendants, parents-in-law, and collateral relatives within the fourth degree. 9 With
respect to her employment, the husband wields a veto power in the case the wife exercises her
profession or occupation or engages in business, provided his income is sufficient for the family,
according to its social standing and his opposition is founded on serious and valid grounds. 10 Most
offensive, if not repulsive, to the liberal-minded is the effective prohibition upon a widow to get married
till after three hundred days following the death of her husband, unless in the meantime, she has given
birth to a child. 11 The mother who contracts a subsequent marriage loses the parental authority over
her children, unless the deceased husband, father of the latter, has expressly provided in his will that his

489
widow might marry again, and has ordered that in such case she should keep and exercise parental
authority over their children. 12 Again, an instance of a husband's overarching influence from beyond
the grave.

All these indignities and disabilities suffered by Filipino wives for hundreds of years evoked no protest
from them until the concept of human rights and equality between and among nations and individuals
found hospitable lodgment in the United Nations Charter of which the Philippines was one of the
original signatories. By then, the Spanish "conquistadores" had been overthrown by the American forces
at the turn of the century. The bedrock of the U.N. Charter was firmly anchored on this credo: "to
reaffirm faith in fundamental human rights, in the dignity and worth of the human person, in the equal
rights of men and women." (Emphasis supplied)

It took over thirty years before these egalitarian doctrines bore fruit, owing largely to the burgeoning of
the feminist movement. What may be regarded as the international bill of rights for women was
implanted in the Convention on the Elimination of All Forms of Discrimination Against Women (CEDAW)
adopted by the U.N. General Assembly which entered into force as an international treaty on September
3, 1981. In ratifying the instrument, the Philippines bound itself to implement its liberating spirit and
letter, for its Constitution, no less, declared that "The Philippines . . . adopts the generally accepted
principles of international law as part of the law of the land and adheres to the policy of peace, equality,
justice, freedom, cooperation, and amity with all nations." 13 One such principle embodied in the
CEDAW is granting to men and women "the same rights with regard to the law relating to the
movement of persons and the freedom to choose their residence and domicile." 14 (Emphasis supplied)

CEDAW's pro-women orientation which was not lost on Filipino women was reflected in the 1987
Constitution of the Philippines and later, in the Family Code, 15 both of which were speedily approved
by the first lady President of the country, Corazon C. Aquino. Notable for its emphasis on the human
rights of all individuals and its bias for equality between the sexes are the following provisions: "The
State values the dignity of every human person and guarantees full respect for human rights" 16 and
"The State recognizes the role of women in nation-building, and shall ensure the fundamental equality
before the law of women and men." 17

A major accomplishment of women in their quest for equality with men and the elimination of
discriminatory provisions of law was the deletion in the Family Code of almost all of the unreasonable
strictures on wives and the grant to them of personal rights equal to that of their husbands. Specifically,
the husband and wife are now given the right jointly to fix the family domicile; 18 concomitant to the
spouses' being jointly responsible for the support of the family is the right and duty of both spouses to
manage the household; 19 the administration and the enjoyment of the community property shall
belong to both spouses jointly; 20 the father and mother shall now jointly exercise legal guardianship
over the property of their unemancipated common child 21 and several others.

Aware of the hiatus and continuing gaps in the law, insofar as women's rights are concerned, Congress
passed a law popularly known as "Women in Development and Nation Building Act." 22 Among the
rights given to married women evidencing their capacity to act in contracts equal to that of men are:

490
(1) Women shall have the capacity to borrow and obtain loans and execute security and credit
arrangements under the same conditions as men;

(2) Women shall have equal access to all government and private sector programs granting
agricultural credit, loans and non material resources and shall enjoy equal treatment in agrarian reform
and land resettlement programs;

(3) Women shall have equal rights to act as incorporators and enter into insurance contracts; and

(4) Married women shall have rights equal to those of married men in applying for passports,
secure visas and other travel documents, without need to secure the consent of their spouses. 23

As the world draws the curtain on the Fourth World Conference of Women in Beijing, let this Court now
be the first to respond to its clarion call that "Women's Rights are Human Rights" and that "All obstacles
to women's full participation in decision-making at all levels, including the family" should be removed.
Having been herself a Member of the Philippine Delegation to the International Women's Year
Conference in Mexico in 1975, this writer is only too keenly aware of the unremitting struggle being
waged by women the world over, Filipino women not excluded, to be accepted as equals of men and to
tear down the walls of discrimination that hold them back from their proper places under the sun.
LexLib

In light of the inexorable sweep of events, local and global, legislative, executive and judicial, according
more rights to women hitherto denied them and eliminating whatever pockets of discrimination still
exist in their civil, political and social life, can it still be insisted that widows are not at liberty to choose
their domicile upon the death of their husbands but must retain the same, regardless?

I submit that a widow, like the petitioner and others similarly situated, can no longer be bound by the
domicile of the departed husband, if at all she was before. Neither does she automatically revert to her
domicile of origin, but exercising free will, she may opt to reestablish her domicile of origin. In returning
to Tacloban and subsequently, to Barangay Olot, Tolosa, both of which are located in the First District of
Leyte, petitioner amply demonstrated by overt acts, her election of a domicile of choice, in this case, a
reversion to her domicile of origin. Added together, the time when she set up her domicile in the two
places sufficed to meet the one-year requirement to run as Representative of the First District of Leyte.

In view of the foregoing expatiation, I vote to GRANT the petition. LLjur

VITUG, J ., separate opinion:

The case at bench deals with explicit Constitutional mandates.

The Constitution is not a pliable instrument. It is a bedrock in our legal system that sets up ideals and
directions and render steady our strides hence. It only looks back so as to ensure that mistakes in the
past are not repeated. A compliant transience of a constitution belittles its basic function and weakens
its goals. A constitution may well become outdated by the realities of time. When it does, it must be
changed but while it remains, we owe it respect and allegiance. Anarchy, open or subtle, has never

491
been, nor must it ever be, the answer to perceived transitory needs, let alone societal attitudes, or the
Constitution might lose its very essence.

Constitutional provisions must be taken to be mandatory in character unless, either by express


statement or by necessary implication, a different intention is manifest (see Marcelino vs. Cruz, 121
SCRA 51).

The two provisions initially brought to focus are Section 6 and Section 17 of Article VI of the
fundamental law. These provisions read:

"SECTION 6. No person shall be a Member of the House of Representatives unless he is a natural-


born citizen of the Philippines and, on the day of the election, is at least twenty-five years of age, able to
read and write, and, except the party-list representatives, a registered voter in the district in which he
shall be elected, and a resident thereof for a period of not less than one year immediately preceding the
day of the election."

"SECTION 17. The Senate and the House of Representatives shall each have an Electoral Tribunal
which shall be the sole judge of all contests relating to the election, returns, and qualifications of their
respective Members. Each Electoral Tribunal shall be composed of nine Members, three of whom shall
be Justices of the Supreme Court to be designated by the Chief Justice, and the remaining six shall be
Members of the Senate or the House of Representatives, as the case may be, who shall be chosen on
the basis of proportional representation from the political parties and the parties or organizations
registered under the party-list system represented therein. The senior Justice in the Electoral Tribunal
shall be its Chairman."

The Commission on Elections (the "COMELEC") is constitutionally bound to enforce and administer "all
laws and regulations relative to the conduct of election . . ." (Art. IX-C, Sec. 2, Constitution) that, there
being nothing said to the contrary, should include its authority to pass upon the qualification and
disqualification prescribed by law of candidates to an elective office. Indeed, pre-proclamation
controversies are expressly placed under the COMELEC's jurisdiction to hear and resolve (Art. IX-C, Sec.
3, Constitution).

The matter before us specifically calls for the observance of the constitutional one-year residency
requirement. This issue (whether or not there is here such compliance), to my mind, is basically a
question of fact or at least inextricably linked to such determination. The findings and judgment of the
COMELEC, in accordance with the long established rule and subject only to a number of exceptions
under the basic heading of "grave abuse of discretion," are not reviewable by this Court.

I do not find much need to do a complex exercise on what seems to me to be a plain matter. Generally,
the term "residence" has a broader connotation that may mean permanent (domicile), official (place
where one's official duties may require him to stay) or temporary (the place where he sojourns during a
considerable length of time). For civil law purposes, i.e., as regards the exercise of civil rights and the
fulfillment of civil obligations, the domicile of a natural person is the place of his habitual residence (see

492
Article 50, Civil Code). In election cases, the controlling rule is that heretofore announced by this Court
in Romualdez vs. Regional Trial Court, Branch 7, Tacloban City (226 SCRA 408, 409); thus:

"In election cases, the Court treats domicile and residence as synonymous terms, thus: '(t)he term
'residence' as used in the election law is synonymous with 'domicile,' which imports not only an
intention to reside in a fixed place but also personal presence in that place, coupled with conduct
indicative of such intention.' 'Domicile' denotes a fixed permanent residence to which when absent for
business or pleasure, or for like reasons, one intends to return. . . . Residence thus acquired, however,
may be lost by adopting another choice of domicile. In order, in turn, to acquire a new domicile by
choice, there must concur (1) residence or bodily presence in the new locality, (2) an intention to remain
there, and (3) an intention to abandon the old domicile. In other words, there must basically be animus
manendi coupled with animus non revertendi. The purpose to remain in or at the domicile of choice
must be for an indefinite period of time; the change of residence must be voluntary; and the residence
at the place chosen for the new domicile must be actual."

Using the above tests, I am not convinced that we can charge the COMELEC with having committed
grave abuse of discretion in its assailed resolution.

The COMELEC's jurisdiction, in the case of congressional elections, ends when the jurisdiction of the
Electoral Tribunal concerned begins. It signifies that the protestee must have theretofore been duly
proclaimed and has since become a "member" of the Senate or the House of Representatives. The
question can be asked on whether or not the proclamation of a candidate is just a ministerial function of
the Commission on Elections dictated solely on the number of votes cast in an election exercise. I
believe, it is not. A ministerial duty is an obligation the performance of which, being adequately defined,
does not allow the use of further judgment or discretion. The COMELEC, in its particular case, is tasked
with the full responsibility of ascertaining all the facts and conditions such as may be required by law
before a proclamation is properly done. LLpr

The Court, on its part, should, in my view at least, refrain from any undue encroachment on the ultimate
exercise of authority by the Electoral Tribunals on matters which, by no less than a constitutional fiat,
are explicitly within their exclusive domain. The nagging question, if it were otherwise, would be the
effect of the Court's peremptory pronouncement on the ability of the Electoral Tribunal to later come up
with its own judgment in a contest "relating to the election, returns and qualification" of its members.

Prescinding from all the foregoing, I should like to next touch base on the applicability to this case of
Section 6 of Republic Act No. 6646, in relation to Section 72 of Batas Pambansa Blg. 881, each providing
thusly:

REPUBLIC ACT NO. 6646

"xxx xxx xxx

"SECTION 6. Effect of Disqualification Case. — Any candidate who has been declared by final
judgment to be disqualified shall not be voted for, and the votes cast for him shall not be counted. If for

493
any reason a candidate is not declared by final judgment before an election to be disqualified and he is
voted for and receives the winning number of votes in such election, the Court or Commission shall
continue with the trial and hearing of the action, inquiry or protest and, upon motion of the
complainant or any intervenor, may during the pendency thereof order the suspension of the
proclamation of such candidate whenever the evidence of his guilt is strong."

BATAS PAMBANSA BLG. 881

"xxx xxx xxx

"SECTION 72. Effects of disqualification cases and priority. — The Commission and the courts shall give
priority to cases of disqualification by reason of violation of this Act to the end that a final decision shall
be rendered not later than seven days before the election in which the disqualification is sought.

"Any candidate who has been declared by final judgment to be disqualified shall not be voted for, and
the votes cast for him shall not be counted. Nevertheless, if for any reason, a candidate is not declared
by final judgment before an election to be disqualified, and he is voted for and receives the winning
number of votes in such election, his violation of the provisions of the preceding sections shall not
prevent his proclamation and assumption to office."

I realize that in considering the significance of the law, it may be preferable to look for not so much the
specific instances they ostensibly would cover as the principle they clearly convey. Thus, I will not scoff
at the argument that it should be sound to say that votes cast in favor of the disqualified candidate,
whenever ultimately declared as such, should not be counted in his or her favor and must accordingly be
considered to be stray votes. The argument, nevertheless, is far outweighed by the rationale of the now
prevailing doctrine first enunciated in the case of Topacio vs. Paredes (23 Phil. 238 [1912]) which,
although later abandoned in Ticzon vs. COMELEC (103 SCRA 687 [1981]), and Santos vs. COMELEC (137
SCRA 740 [1985]), was restored, along with the interim case of Geronimo vs. Ramos (136 SCRA 435
[1985]), by the Labo (176 SCRA 1 [1989]), Abella (201 SCRA 253 [1991]), Labo (211 SCRA 297 [1992]) and,
most recently, Benito (235 SCRA 436 [1994]) rulings. Benito vs. Comelec was a unanimous decision
penned by Justice Kapunan and concurred in by Chief Justice Narvasa, Justices Feliciano, Padilla, Bidin,
Regalado, Davide, Romero, Melo, Quiason, Puno, Vitug and Mendoza (Justices Cruz and Bellosillo were
on official leave). For easy reference, let me quote from the first Labo decision:

"Finally, there is the question of whether or not the private respondent, who filed the quo warranto
petition, can replace the petitioner as mayor. He cannot. The simple reason is that as he obtained only
the second highest number of votes in the election, he was obviously not the choice of the people of
Baguio City.

"The latest ruling of the Court on this issue is Santos v. Commission on Elections, (137 SCRA 740) decided
in 1985. In that case, the candidate who placed second was proclaimed elected after the votes for his
winning rival, who was disqualified as a turncoat and considered a non-candidate, were all disregarded
as stray. In effect, the second placer won by default. That decision was supported by eight members of
the Court then, (Cuevas, J., ponente, with Makasiar, Concepcion, Jr., Escolin, Relova, De la Fuente,

494
Alampay and Aquino, JJ., concurring.) with three dissenting (Teehankee, Acting C.J., Abad Santos and
Melencio-Herrera, JJ.) and another two reserving their vote. (Plana and Gutierrez, Jr., JJ.) One was on
official leave. (Fernando, C.J.)

"Re-examining that decision, the Court finds, and so holds, that it should be reversed in favor of the
earlier case of Geronimo v. Ramos, (136 SCRA 435) which represents the more logical and democratic
rule. That case, which reiterated the doctrine first announced in 1912 in Topacio v. Paredes, (23 Phil.
238) was supported by ten members of the Court, (Gutierrez, Jr., ponente, with Teehankee, Abad
Santos, Melencio-Herrera, Plana, Escolin, Relova, De la Fuente, Cuevas and Alampay, JJ., concurring)
without any dissent, although one reserved his vote, (Makasiar, J.) another took no part, (Aquino, J.) and
two others were on leave. (Fernando, C.J. and Concepcion, Jr., J.) There the Court held:

"'. . . it would be extremely repugnant to the basic concept of the constitutionally guaranteed right to
suffrage if a candidate who has not acquired the majority or plurality of votes is proclaimed a winner
and imposed as the representative of a constituency, the majority of which have positively declared
through their ballots that they do not choose him.

'Sound policy dictates that public elective offices are filled by those who have received the highest
number of votes cast in the election for that office, and it is a fundamental idea in all republican forms of
government that no one can be declared elected and no measure can be declared carried unless he or it
receives a majority or plurality of the legal votes cast in the election. (20 Corpus Juris 2nd, S 243, p. 676.)

'The fact that the candidate who obtained the highest number of votes is later declared to be
disqualified or not eligible for the office to which he was elected does not necessarily entitle the
candidate who obtained the second highest number of votes to be declared the winner of the elective
office. The votes cast for a dead, disqualified, or non-eligible person may not be valid to vote the winner
into office or maintain him there. However, in the absence of a statute which clearly asserts a contrary
political and legislative policy on the matter, if the votes were cast in the sincere belief that the
candidate was alive, qualified, or eligible, they should not be treated as stray, void or meaningless.' (at
pp. 20-21)"

Considering all the foregoing, I am constrained to vote for the dismissal of the petition. cdll

MENDOZA, J ., separate opinion:

In my view the issue in this case is whether the Commission on Elections has the power to disqualify
candidates on the ground that they lack eligibility for the office to which they seek to be elected. I think
that it has none and that the qualifications of candidates may be questioned only in the event they are
elected, by filing a petition for quo warranto or an election protest in the appropriate forum, not
necessarily in the COMELEC but, as in this case, in the House of Representatives Electoral Tribunal. That
the parties in this case took part in the proceedings in the COMELEC is of no moment. Such proceedings
were unauthorized and were not rendered valid by their agreement to submit their dispute to that
body.

495
The various election laws will be searched in vain for authorized proceedings for determining a
candidate's qualifications for an office before his election. There are none in the Omnibus Election Code
(B.P. Blg. 881), in the Electoral Reforms Law of 1987 (R.A. No. 6646), or in the law providing for
synchronized elections (R.A. No. 7166). There are, in other words, no provisions for pre-proclamation
contests but only election protests or quo warranto proceedings against winning candidates. cdll

To be sure, there are provisions denominated for "disqualification," but they are not concerned with a
declaration of the ineligibility of a candidate. These provisions are concerned with the incapacity (due to
insanity, incompetence or conviction of an offense) of a person either to be a candidate or to continue
as a candidate for public office. There is also a provision for the denial or cancellation of certificates of
candidacy, but it applies only to cases involving false representations as to certain matters required by
law to be stated in the certificates.

These provisions are found in the following parts of the Omnibus Election Code:

§ 12. Disqualifications. — Any person who has been declared by competent authority insane or
incompetent, or has been sentenced by final judgment for subversion, insurrection, rebellion or for any
offense for which he has been sentenced to a penalty of more than eighteen months or for a crime
involving moral turpitude, shall be disqualified to be a candidate and to hold any office, unless he has
been given plenary pardon or granted amnesty.

The disqualifications to be a candidate herein provided shall be deemed removed upon the declaration
by competent authority that said insanity or incompetence had been removed or after the expiration of
a period of five years from his service of sentence, unless within the same period he again becomes
disqualified. (Emphasis added)

§ 68. Disqualifications. — Any candidate who, in an action or protest in which he is a party is declared
by final decision of a competent court guilty of, or found by the Commission of having (a) given money
or other material consideration to influence, induce or corrupt the voters or public officials performing
electoral functions; (b) committed acts of terrorism to enhance his candidacy; (c) spent in his election
campaign an amount in excess of that allowed by this Code; (d) solicited, received or made any
contribution prohibited under Sections 89, 95, 96, 97 and 104; or (e) violated any of Sections 80, 83, 85,
86 and 261, paragraphs d, e, k, v, and cc, sub-paragraph 6, shall be disqualified from continuing as a
candidate, or if he has been elected, from holding the office. Any person who is a permanent resident of
or an immigrant to a foreign country shall not be qualified to run for any elective office under this Code,
unless said person has waived his status as permanent resident or immigrant of a foreign country in
accordance with the residence requirement provided for in the election laws. (Emphasis added)

§ 78. Petition to deny due course to or cancel a certificate of candidacy. — A verified petition seeking
to deny due course or to cancel a certificate of candidacy may be filed by any person exclusively on the
ground that any material representation contained therein as required under Section 74 hereof is false.
The petition may be filed at any time not later than twenty-five days from the time of the filing of the
certificate of candidacy and shall be decided, after due notice and hearing, not later than fifteen days
before the election. (Emphasis added)

496
the Electoral Reforms Law of 1987 (R.A. No. 6646):

§ 6. Effect of Disqualification Case. — Any candidate who has been declared by final judgment to be
disqualified shall not be voted for, and the votes cast for him shall not be counted. If for any reason a
candidate is not declared by final judgment before an election to be disqualified and he is voted for and
receives the winning number of votes in such election, the Court or Commission shall continue with the
trial and hearing of the action, inquiry or protest and, upon motion for the complainant or any
intervenor, may during the pendency thereof order the suspension of the proclamation of such
candidate whenever the evidence of his guilt is strong. (Emphasis added)

§ 7. Petition to Deny Due Course to or Cancel a Certificate of Candidacy. — The procedure


hereinabove provided shall apply to petitions to deny due course to or cancel a certificate of candidacy
as provided in Section 78 of Batas Pambansa Blg. 881.

and the Local Government Code of 1991 (R.A. No. 7160):

§ 40. Disqualifications. — The following persons are disqualified from running for any elective local
position:

(a) Those sentenced by final judgment for an offense involving moral turpitude or for an offense
punishable by one (1) year or more of imprisonment, within two (2) years after serving sentence;

(b) Those removed from office as a result of an administrative case;

(c) Those convicted by final judgment for violating the oath of allegiance to the Republic;

(d) Those with dual citizenship;

(e) Fugitive from justice in criminal or nonpolitical cases here or abroad;

(f) Permanent residents in a foreign country or those who have acquired the right to reside abroad
and continue to avail of the same right after the effectivity of this Code; and

(g) The insane or feeble-minded.

The petition filed by private respondent Cirilo Roy Montejo in the COMELEC, while entitled "For
Cancellation and Disqualification," contained no allegation that private respondent Imelda Romualdez-
Marcos made material representations in her certificate of candidacy which were false. It sought her
disqualification on the ground that "on the basis of her Voter Registration Record and Certificate of
Candidacy, [she] is disqualified from running for the position of Representative, considering that on
election day, May 8, 1995, [she] would have resided less than ten (10) months in the district where she
is seeking to be elected." For its part, the COMELEC's Second Division, in its resolution of April 24, 1995,
cancelled her certificate of candidacy and corrected certificate of candidacy on the basis of its finding
that petitioner is "not qualified to run for the position of Member of the House of Representatives for
the First Legislative District of Leyte" and not because of any finding that she had made false
representations as to material matters in her certificate of candidacy.

497
Montejo's petition before the COMELEC was therefore not a petition for cancellation of certificate of
candidacy under §78 of the Omnibus Election Code, but essentially a petition to declare private
respondent ineligible. It is important to note this, because, as will presently be explained, proceedings
under §78 have for their purpose to disqualify a person from being a candidate, whereas quo warranto
proceedings have for their purpose to disqualify a person from holding public office. Jurisdiction over
quo warranto proceedings involving members of the House of Representatives is vested in the Electoral
Tribunal of that body.

Indeed, in the only cases in which this Court dealt with petitions for the cancellation of certificates of
candidacy, the allegations were that the respondent candidates had made false representations in their
certificates of candidacy with regard to their citizenship, 1 age, 2 or residence. 3 But in the generality of
cases in which this Court passed upon the qualifications of respondents for office, this Court did so in
the context of election protests 4 or quo warranto proceedings 5 filed after the proclamation of the
respondents or protestees as winners.

Three reasons may be cited to explain the absence of an authorized proceeding for determining before
election the qualifications of a candidate. cda

First is the fact that unless a candidate wins and is proclaimed elected, there is no necessity for
determining his eligibility for the office. In contrast, whether an individual should be disqualified as a
candidate for acts constituting election offenses (e.g., vote buying, over spending, commission of
prohibited acts) is a prejudicial question which should be determined lest he wins because of the very
acts for which his disqualification is being sought. That is why it is provided that if the grounds for
disqualification are established, a candidate will not be voted for; if he has been voted for, the votes in
his favor will not be counted; and if for some reason he has been voted for and he has won, either he
will not be proclaimed or his proclamation will be set aside. 6

Second is the fact that the determination of a candidate's eligibility, e.g., his citizenship or, as in this
case, his domicile, may take a long time to make, extending beyond the beginning of the term of the
office. This is amply demonstrated in the companion case (G.R. No. 120265, Agapito A. Aquino v.
COMELEC) where the determination of Aquino's residence was still pending in the COMELEC even after
the elections of May 8, 1995. This is contrary to the summary character of proceedings relating to
certificates of candidacy. That is why the law makes the receipt of certificates of candidacy a ministerial
duty of the COMELEC and its officers. 7 The law is satisfied if candidates state in their certificates of
candidacy that they are eligible for the position which they seek to fill, leaving the determination of their
qualifications to be made after the election and only in the event they are elected. Only in cases
involving charges of false representations made in certificates of candidacy is the COMELEC given
jurisdiction.

Third is the policy underlying the prohibition against pre-proclamation cases in elections for President,
Vice President, Senators and members of the House of Representatives. (R.A. No. 7166, §15) The
purpose is to preserve the prerogatives of the House of Representatives Electoral Tribunal and the other

498
Tribunals as "sole judges" under the Constitution of the election, returns and qualifications of members
of Congress or of the President and Vice President, as the case may be.

By providing in §253 for the remedy of quo warranto for determining an elected official's qualifications
after the results of elections are proclaimed, while being conspicuously silent about a pre-proclamation
remedy based on the same ground, the Omnibus Election Code, or OEC, by its silence underscores the
policy of not authorizing any inquiry into the qualifications of candidates unless they have been elected.

Apparently realizing the lack of an authorized proceeding for declaring the ineligibility of candidates, the
COMELEC amended its rules on February 15, 1993 so as to provide in Rule 25, §1 the following:

Grounds for disqualification. — Any candidate who does not possess all the qualifications of a candidate
as provided for by the Constitution or by existing law or who commits any act declared by law to be
grounds for disqualification may be disqualified from continuing as a candidate.

The lack of provision for declaring the ineligibility of candidates, however, cannot be supplied by a mere
rule. Such an act is equivalent to the creation of a cause of action which is a substantive matter which
the COMELEC, in the exercise of its rule making power under Art. IX, A, §6 of the Constitution, cannot
do. It is noteworthy that the Constitution withholds from the COMELEC even the power to decide cases
involving the right to vote, which essentially involves an inquiry into qualifications based on age,
residence and citizenship of voters. [Art. IX, C, §2(3)]

The assimilation in Rule 25 of the COMELEC rules of grounds for ineligibility into grounds for
disqualification is contrary to the evident intention of the law. For not only in their grounds but also in
their consequences are proceedings for "disqualification" different from those for a declaration of
"ineligibility." "Disqualification" proceedings, as already stated, are based on grounds specified in §12
and §68 of the Omnibus Election Code and in §40 of the Local Government Code and are for the
purpose of barring an individual from becoming a candidate or from continuing as a candidate for public
office. In a word, their purpose is to eliminate a candidate from the race either from the start or during
its progress. "Ineligibility," on the other hand, refers to the lack of the qualifications prescribed in the
Constitution or the statutes for holding public office and the purpose of the proceedings for declaration
of ineligibility is to remove the incumbent from office. cdlex

Consequently, that an individual possesses the qualifications for a public office does not imply that he is
not disqualified from becoming a candidate or continuing as a candidate for a public office and vice
versa. We have this sort of dichotomy in our Naturalization Law. (C.A. No. 473) That an alien has the
qualifications prescribed in §2 of the law does not imply that he does not suffer from any of
disqualifications provided in §4.

Indeed, provisions for disqualifications on the ground that the candidate is guilty of prohibited election
practices or offenses, like other pre-proclamation remedies, are aimed at the detestable practice of
"grabbing the proclamation and prolonging the election protest," 8 through the use of "manufactured"
election returns or resort to other trickery for the purpose of altering the results of the election. This
rationale does not apply to cases for determining a candidate's qualifications for office before the

499
election. To the contrary, it is the candidate against whom a proceeding for disqualification is brought
who could be prejudiced because he could be prevented from assuming office even though in end he
prevails. aisadc

To summarize, the declaration of ineligibility of a candidate may only be sought in an election protest or
action for quo warranto filed pursuant to §253 of the Omnibus Election Code within 10 days after his
proclamation. With respect to elective local officials (e.g., Governor, Vice Governor, members of the
Sangguniang Panlalawigan, etc.) such petition must be filed either with the COMELEC, the Regional Trial
Courts, or Municipal Trial Courts, as provided in Art. IX, C, §2(2) of the Constitution. In the case of the
President and Vice President, the petition must be filed with the Presidential Electoral Tribunal (Art. VII,
§4, last paragraph), and in the case of the Senators, with the Senate Electoral Tribunal, and in the case of
Congressmen, with the House of Representatives Electoral Tribunal. (Art. VI, §17) There is greater
reason for not allowing before the election the filing of disqualification proceedings based on alleged
ineligibility in the case of candidates for President, Vice President, Senators and members of the House
of Representatives, because of the same policy prohibiting the filing of pre-proclamation cases against
such candidates.

For these reasons, I am of the opinion that the COMELEC had no jurisdiction over SPA No. 95-009; that
its proceedings in that case, including its questioned orders, are void; and that the eligibility of petitioner
Imelda Romualdez-Marcos for the office of Representative of the First District of Leyte may only be
inquired into by the HRET.

ACCORDINGLY, I vote to grant the petition and to annul the proceedings of the Commission on Elections
in SPA No. 95-009, including its questioned orders dated April 24, 1995, May 7, 1995, May 11, 1995 and
May 25, 1995, declaring petitioner Imelda Romualdez-Marcos ineligible and ordering her proclamation
as Representative of the First District of Leyte suspended. To the extent that Rule 25 of the COMELEC
Rules of Procedure authorizes proceedings for the disqualification of candidates on the ground of
ineligibility for the office, it should considered void.

The provincial board of canvassers should now proceed with the proclamation of petitioner.

500

S-ar putea să vă placă și